Download as pdf or txt
Download as pdf or txt
You are on page 1of 662

WILEY

PROBLEMS
MATHEMATICSFOR
IN

JEE with Summarized Concepts

VOLUME –II
WILEY
PROBLEMS
IN
MATHEMATICSFOR
VOLUME – II
JEEwith Summarized Concepts

Copyright © 2018 by Wiley India Pvt. Ltd., 4436/7, Ansari Road, Daryaganj, New Delhi-110002.

Cover Image: Carlos_bcn/iStockphoto

All rights reserved. No part of this book may be reproduced, stored in a retrieval system, or transmitted in any form
or by any means, electronic, mechanical, photocopying, recording or scanning without the written permission of
the publisher.

Limits of Liability: While the publisher and the author have used their best efforts in preparing this book, Wiley
and the author make no representation or warranties with respect to the accuracy or completeness of the
contents of this book, and specifically disclaim any implied warranties of merchantability or fitness for any
particular purpose. There are no warranties which extend beyond the descriptions contained in this paragraph.
No warranty may be created or extended by sales representatives or written sales materials.

Disclaimer: The contents of this book have been checked for accuracy. Since deviations cannot be precluded
entirely, Wiley or its author cannot guarantee full agreement. As the book is intended for educational purpose,
Wiley or its author shall not be responsible for any errors, omissions or damages arising out of the use of the
information contained in the book. This publication is designed to provide accurate and authoritative information
with regard to the subject matter covered. It is sold on the understanding that the Publisher is not engaged in
rendering professional services.

Other Wiley Editorial Offices:


John Wiley & Sons, Inc. 111 River Street, Hoboken, NJ 07030, USA
Wiley-VCH Verlag GmbH, Pappellaee 3, D-69469 Weinheim, Germany
John Wiley & Sons Australia Ltd, 42 McDougall Street, Milton, Queensland 4064, Australia
John Wiley & Sons (Asia) Pte Ltd, 1 Fusionpolis Walk #07-01 Solaris, South Tower Singapore 138628
John Wiley & Sons Canada Ltd, 22 Worcester Road, Etobicoke, Ontario, Canada, M9W 1L1

First Edition: 2018


ISBN: 978-81-265-7630-2
ISBN: 978-81-265-8690-5 (ebk)
www.wileyindia.com
Printed at:
Note to the Student
Wiley Mathematics Problem Book is specifically designed to meet the needs of engineering (JEE) aspirants and give an edge to
their preparation. The book offers complete coverage of the mathematics curriculum (of Class 12 syllabus) for JEE. It is enriched with
unique elements and features that help students recapitulate the concepts, build problem-solving skills and apply them to solve all
question-types asked in the engineering entrance examinations. The book is a valuable resource for both JEE (Main) and JEE (Advanced)
aspirants. The chapter flow of the book is aligned with JEE Main syllabus and its coverage in the classroom. However, topics specific to
JEE (Advanced) and advanced level questions are also covered both as solved examples and practice exercises.
We will now walk you through the target examinations and some key features of the book that enhance the learning experience.

  TARGET EXAMINATION  
Admission to Undergraduate Engineering Programs at IITs, NITs and other Center and State (participating) funded Technical
Institutions use the Joint Entrance Examination Main (JEE Main) score as eligibility/merit criteria. The JEE (Main) is also an eligibility test
for the Joint Entrance Examination Advanced [JEE (Advanced)], which is mandatory for the candidate if he/she is aspiring for admission
to the undergraduate program offered by the IITs. The JEE (Advanced) scores are used as an eligibility criteria for admission into IITs.
An effective exam strategy for success in these examinations can be based on the detailed analysis of previous years question papers
and planning your preparation accordingly. The Mathematics Question Paper of these examinations is a judicious mix of easy, moderate
and tough questions. The analysis of question distribution over the units of mathematics syllabus for these examinations is given below.

 EXAM ANALYSIS OF PAPERS


Mathematics question paper comes as an amalgamation of easy, moderate and tough questions. This section shows the unit-wise as
well as chapter-wise analysis of previous 9 years (2010-2018) JEE Main and JEE Advanced papers.

JEE Main
Year
Unit
2010 2011 2012 2013 2014 2015 2016 2017 2018
Algebra 14 13 13 12 12 11 12 13 12
Calculus 8 10 9 8 9 8 7 10 8
Trigonometry 2 1 1 3 2 3 3 2 3
Analytical Geometry 6 6 7 7 7 8 8 5 7

JEE Advanced
Year
Unit
2010 2011 2012 2013 2014 2015 2016 2017 2018
Algebra 16 17 12 14 12 6 12 10 8
Trigonometry 5 1 2 4 3 1 2 1 1
Analytical Geometry 13 8 9 10 7 3 9 7 9
Differential Calculus 2 7 6 2 11 5 7 8 12
Integral Calculus 8 7 10 7 5 4 5 7 4
Vector 3 3 2 3 2 1 1 3 2

Prelims_Volume II.indd 3 27-Jul-18 6:21:54 PM


MATHEMATICS JEE MAIN PAPERS ANALYSIS (2010-2018)

Prelims_Volume II.indd 4
AIEEE AIEEE AIEEE JEE Main JEE Main JEE Main JEE Main JEE Main JEE Main
Unit Chapter
2010 2011 2012 2013 2014 (Offline) 2015 (Offline) 2016 (Offline) 2017 (Offline) 2018 (Offline)
Algebra Complex Numbers and Quadratic Equations 2 2 2 3 3 2 2 1 1
Permutations and Combinations 1 1 1 1 2 1 2 1
Binomial Theorem 1 1 1 1 1 1
Sequences and Series 2 1 2 1 2 2 2 3 2
Statistics 1 1 1 1 1 1 1 1 1
Mathematical Reasoning 1 1 1 1 1 1 1
Matrices and Determinants 3 2 2 2 2 2 2 3 3
Vector Algebra 2 2 2 1 1 1 1 1 1
Probability 2 2 1 1 1 1 1 2 1
Calculus Sets, Relations and Functions 1 2 1 1 1 2 1 2 2
Limits, Continuity and Differentiability 3 3 2 2 3 3 2 2 1
Application of Derivatives 1 1 3 1 2 2 1
Integrals 1 2 2 3 2 2 1 2 2
Application of Integrals 1 1 1 1 1 1 1 1
Differential Equations 1 1 1 2 1 1
Trigonometry Trignometric Functions 2 1 1 2 2 2 2 2 3
Inverse Trignometric Functions 1 1 1
Analytical Conic Sections 2 2 3 3 3 4 5 3 5
Geometry Three-Dimensional Geometry 4 4 4 4 4 4 3 2 2

27-Jul-18 6:21:54 PM
MATHEMATICS JEE ADVANCED PAPERS ANALYSIS (2010-2018)

Prelims_Volume II.indd 5
IIT-JEE 2010 IIT-JEE 2011 IIT-JEE 2012 JEE Advanced 2013 JEE Advanced 2014 JEE Advanced 2015 JEE Advanced 2016 JEE Advanced 2017 JEE Advanced 2018
Unit Chapter
P Q R S T U P Q R S T U P Q R S T U P Q R S T U P Q R S T U P Q R S T U P Q R S T U P Q R S T U P Q R S T U
Complex Numbers 1 1 1 2 1 1 1 2 1 1 1 1 2
Quadratic Equations 1 2 1 1 1 1 1 2 1
Permutations and
1 1 1 1
Combinations
Sequence and Series 1 2 1 1 2 1 1 1 1 2 1 1
Binomial Theorem 1 1 1 1 1 1

Algebra
Logarithms 1 1 1
Matrices and
1 3 2 2 3 1 2 1 2 2 1 1 1 1 1 1 1 1
Determinants
Probability 2 1 2 1 2 1 2 2 2 2 1 2 1 2 1 1 2
Properties and
Solution of Triangles
1 1 1 1 1 1
(Heights and
Distances)
Trigonometric
2 1 1 1
Equations
Trigonometric Ratios

Trigonometry
1 1 1 1
and Identities
Inverse Trigonometric
Function (Principal 1 1 1 1 1
Values Only)
Rectangular
Coordinate System
Straight Lines and
2 1 1 1 1 1 1 2 1
Pair of Lines
Conics 1 3 2 2 2 1 1 1 1 2 1 1 2 1 2 1 2 2 2 3 1 1
Circle 2 1 2 1 2 1 1 4 1 2 1

Analytical Geometry
Three-Dimensional
1 2 1 1 1 1 2 2
Geometry
Sets and Relations 2 1 1 1 1 2
Limits 1 1 2 1 2 1 1 3 1 1
Functions 1 1 1 2 2 1 2 1 1 1 1 1
Continuity and
1 2 1 1 1 2 1 1 1 1 4 1 1 2 1
Differentiability

Differential Calculus
Differentiation 2

27-Jul-18 6:21:55 PM
IIT-JEE 2010 IIT-JEE 2011 IIT-JEE 2012 JEE Advanced 2013 JEE Advanced 2014 JEE Advanced 2015 JEE Advanced 2016 JEE Advanced 2017 JEE Advanced 2018
Unit Chapter
P Q R S T U P Q R S T U P Q R S T U P Q R S T U P Q R S T U P Q R S T U P Q R S T U P Q R S T U P Q R S T U

Prelims_Volume II.indd 6
Application of
1 1 1 1 2 1 2 2 1 1 1 3 1
Derivatives
Indefinite Integration 1
Definite Integration 1 1 2 1 1 2 1 1 1 1 1 1 1 1 1
Area Under the
3 1 1 1 1 1 1 1

Integral Calculus
Curve
Differential Equations 1 1 1 1 1 1 1 1 1
Vector Vectors 2 1 1 1 1 1 1 1 1 1 1 1 1 1 1 2 2

P: Single Correct Choice Type Q: One or More Than One Option Correct Type R: Paragraph Type
S: Matrix-Match Type     T: Reasoning Type  
U: Integer Answer Type

27-Jul-18 6:21:55 PM
FEATURES OF THE BOOK

A. Understand the Concepts 21.2 Tangent and Normal


A tangent to a point is a line which touches the curve at that point.
1. All the concepts as per the JEE curriculum
A normal to a point is the line which is perpendicular to the tan-
are explained in simple steps to develop gent at that point.
fundamental understanding of the subject. If the equation of a curve is y = f(x) and a point A(x1, y1) lies on it,
then the equation of the tangent at point A is
 dy 
y – y1 =   ( x − x1)
 dx  A
and the equation of the normal at point A is
1
y - y1 = - ( x - x1)
(dy / dx ) A

Key Point:
When the curve is given in parametric form, that is, when x =
g(t) and y = h(t), the equation of tangent at the point t = t1 is
h′(t1) 2. Important points to remember about concepts
y − h(t1) = [ x − g (t1)]
g′(t1)
highlighted as Key Points.
and the equation of normal is
g¢(t1)
y - h (t1) = - [ x - g (t1)]
h¢(t1)

B. Every Aspect of the Subject Covered


In form of formulas, figures, graphs and tables to enhance problem-solving skills.

 p p
(i) sin−1(sin θ) = θ, ∀q ∈  − , 
 2 2
(ii) cos−1(cos θ) = θ, ∀ θ ∈ [0, π]
 p p
(iii) tan−1(tan θ) = θ, ∀q ∈ − , 
 2 2
C
A B
(iv) cot−1(cos θ) = θ, ∀ θ ∈ (0, π) Table 17.1 Domain and principal ranges of all the six inverse trig-
p  onometric functions
(v) sec−1(sec θ) = θ, ∀q ∈[0 , p ] −  
2 Function Domain Principal Range
−1  p p (values of x) (values of y)
(vi) cosec (cosec θ) = θ, ∀q ∈ − ,
N r y  2 2  − {0}
L M   é p pù
y = sin−1 x [−1, 1] ê- 2 , 2 ú
ë û

y y = cos−1 x [−1, 1] [0, π]

æ p pö
y = tan−1 x (−∞, ∞) ç- , ÷
è 2 2ø
P (x1, y1)
O
Figure 21.4 ψ
ψ
x
O T M N

Figure 21.2

Features of the book.indd 3 26-Jul-18 12:06:35 PM


C. Reinforce Concepts
Illustration 21.1 Find the slope of tangent at the point that has
1. Illustrations pose a specific problem using the ordinate −3 on the curve x3 = 3y2.

concepts already presented and then work Solution: x,


we get
through the solution.
æ dy ö
3x2 = 3´ ç 2y ÷
è dx ø

dy x 2
⇒ =
dx 2 y
Now, to obtain this value, we require abscissa as well. Substituting
y = −3 in the equation of curve, we have

901

Your Turn 1
1. Find the slopes of the curve y = (x + 2)(x − 3) at the points
where it meets x−axis. Ans. −5, 5 2. Your Turn within each chapter is present to
2. Find the points on the curve y = x3 − 2x2 + x − 2 when the gra-
dient is zero.
reinforce and check the understanding of the
 1 50  students.
Ans. (1, −2) and  , − 
 3 27 
3. Find the equation of tangent and normal to the curve
x3 = y2
normal, subtangent and subnormal.
13 13 2 3
Ans. 3x − 2y − 1 = 0, 2x + 3y − 5 = 0, , , ,
3 2 3 2

Additional Solved Examples


3. Additional Solved Examples suitable for JEE 1. The number of real solutions of cos−1 x + cos−1 2x = −π is
exams are provided with in-depth solutions for (A) 0 (B) 1
(C) 2 (D)
the students to understand the logic behind
Solution:
and formula used. cos−1 x = −(π + cos−1 2x)
Range of cos−1 x ∈ [0, π]
Since cos−1 x has a range from [0, π], thus the sum of two cos−1
cannot be equal to −π a negative quantity.
Hence, the correct answer is option (A).

Features of the book.indd 4 26-Jul-18 12:06:36 PM


D. Understanding the Exam Pattern
Through Previous Years’ Solved JEE Main/AIEEE Questions and Previous Years’ Solved JEE Advanced/IIT-JEE Questions.

Previous Years' Solved JEE Advanced/


IIT-JEE Questions
1. Let (x, y) be such that Previous Years' Solved JEE Main/AIEEE
p
-1 -1 -1
sin (ax ) + cos ( y ) + cos (bxy ) = Questions
2
Match the statements in Column I with statements in Column II. æxö æ5ö p
1. If sin-1 ç ÷ + cosec -1 ç ÷ = , then a value of x is
5
è ø è4ø 2
Column I Column II
(A) 1 (B) 3
(A) If a = 1 and b = 0, then (x, y) (P) lies on the circle x2 + y2 = 1
(C) 4 (D) 5 [AIEEE 2007]
(B) If a = 1 and b = 1, then (x, y) (Q) lies on (x2 − 1)(y2 − 1) = 0
Solution: We have
(C) If a = 1 and b = x, y) (R) lies on y = x x 4 p x 4 x 3
sin-1 + sin-1 = Þ sin-1 = cos -1 Þ sin-1 = sin-1
(D) If a = 2 and b = 2, then (x, y) (S) lies on (4x − 1)(y − 1) = 0
2 2 5 5 2 5 5 5 5
Therefore, x = 3.
[IIT-JEE 2007]
Hence, the correct answer is option (B).

E. Practice to Complete Your Learning


Through Practice Exercise 1 (JEE Main) and Practice Exercise 2 (JEE Advanced). All questions types as per JEE Main and
Advanced covered.

Matrix Match Type Questions


Practice Exercise 1
24. Match the following:
1. The points on the curve y = 12x − x3 at which the gradient is List I List II
zero are
(A) Circular plate is expanded by the heat from (p) 2
(A) (0, 2), (2, 16) (B) (0, −2), (2, −16)
the radius 5 cm to 5.06 cm. Approximate
(C) (2, −16), (−2, 16) (D) (2, 16), (−2, −16) increase in the area is
2. The area of the triangle formed by the coordinate axes and a (B) If an edge of a cube increases by 1%, then the (q) 0.6 p
2
tangent to the curve xy = a at the point ( x1, y1) on it is percentage increase in the volume is
a2 x1 a2 y1 (r) 3
(A) (B) (C) 2a2 (D) 4a2 x2
y1 x1 (C) If the rate of decrease of y = − 2x + 5 is
2
3. The slope of tangent to the curve x = t 2 + 3t − 8, y = 2t 2 twice the rate of decrease
Comprehension Type x, then x is equal
of Questions
- 2t - 5 at the point (2, −1) is to (given that the rate of decrease is non−zero)
Paragraph for Questions 9−11: Let a(t) is a function of t such
(D) Rate da
of increase in the area of the equilateral 3 3
(A) 22 / 7 (B) 6 / 7 (C) − 6 (D) None of these
that = 2side
triangle of for all15the
cm, whenof each
values t and side (s) t = 0. Further
a = 0iswhen
dt
increasing at the rate of 0.1 cm/sec; is 4
y = m(t) x + c(t) is the tangent to the curve y = x2 − 2ax + a2 + a at the
(t) 4
Practice Exercise 2 point whose abscissa is 0. Then
9. If the rate of change
Integer distance of the vertex of y = x2 − 2ax
of theQuestions
Type
+ a2 + a from the origin with respect to t is k, then k =
Single/Multiple Correct Choice Type Questions x2 y2
(A) 2 29. Let(B) α be
2 the (C) 2between (D) +4 2 = 1 and the
2 angle in radians
1. For the curve represented parametically by the equations, 36 4
10. If the rate ofcircle x2 +ofy2c(t)
change = 12 at their
with points
respect to t,ofwhen t = k, is , If a = tan−1
the intersection.
x = 2 ln cot t + 1 and y = tan t + cot t
then k
(A) tangent at t = p/4 is parallel to x-axis k2.
(A) 16 − 2 22 3 (B) 8 2 + 2
(B) normal at t = p/4 is parallel to y-axis 2
 9 
(C) tangent at t = p/4 is parallel to the line y = x (C) 10 30.2 +Find
2 the minimum value 2 1+−2 x2)2 +  2 − x12 −
(D) 16of (x ,
 x2 
(D) tangent and normal intersect at the point (2, 1) where x1 ∈ (0 , 2 ) and x2 ∈ R+.

Features of the book.indd 5 26-Jul-18 12:06:39 PM


F. Check Your Performance and Problem-Solving Approach
Through Answer Key and Solution to practice exercises provided with explanation.

Answer Key
Practice Exercise 1
1. (D) 2. (C) 3. (B) 4. (C) 5. (A) 6. (A)
7. (B) 8. (C) 9. (C) 10. (A) 11. (B) 12. (C)
13. (A) 14. (D) 15. (C) 16. (C) 17. (C) 18. (A)
19. (A) 20. (D) 21. (D) 22. (B) 23. (D) 24. (B)
25. (C) 26. (C) 27. (B) 28. (A) 29. (D) 30. (C)
31. (B) 32. (D) 33. (A) 34. Solutions
(A) 35. (A) 36. (D)

Practice Exercise 1 or x = 2 x1

1. We have Therefore, the point on x-axis is (2 x1,0). Now, the tangent


dy meets y-axis where x = 0. Since
= 12 − 3 x 2 = 0 ⇒ x = ± 2
dx x12 y = 2a2 x1
Hence, the points are (2, 16) and (−2, −16).
2a2
we have y=
2. We have x1
a2 So, the point on the y-axis is
y=
x  2a 2 
Therefore,  0, 
 x1 
dy a2 The required area is
=− 2
dx x 1  2a 2 
(2 x1)  = 2a 2
Now, at ( x1 , y1). 2  x 
 1 

Features of the book.indd 6 26-Jul-18 12:06:39 PM


Contents
Note to the Student iii 18.14  Adjoint of a Square Matrix  741
18.15  Inverse of a Matrix  741
Chapter 17  Inverse Trigonometry 699 18.15.1  Theorem (Uniqueness of Inverse)  741
17.1 Introduction  699    18.15.2  Properties of Inverse of a Matrix  741
18.16 Singular and Non-Singular Matrices  742
17.2 Domain and Range of Inverse
Trigonometric Functions  699 18.17 Elementary Operations or Elementary Transformations
of a Matrix  742
17.3 Properties of Inverse Trigonometric Functions  701
18.17.1  Equivalent Matrices   742
17.4 General Values of Inverse Circular Functions  704
18.17.2  Elementary Matrix   742
Additional Solved Examples  714 18.18 Inverse of a Matrix by Elementary Operations (Elementary
Previous Years’ Solved JEE Main/AIEEE Questions  76 Operations on Matrix Equation)  743
18.18.1  Using Row Operation  743
Previous Years’ Solved JEE Advanced/IIT-JEE Questions  718 18.18.2  Using Column Operation  743
Practice Exercise 1  721 18.19  Rank of a Matrix  744
Practice Exercise 2  724 18.20  Echelon Form of a Matrix  744
18.21  Homogeneous Linear Equations  744
Single/Multiple Correct Choice Type Questions  724
18.21.1 Solution of Homogeneous System of Linear
Matrix Match Type Questions  724
Equations 745
Integer Type Question   725
18.22 System of Linear Non-Homogeneous Equations  745
Answer Key  725 18.22.1 Matrix Method of Solving Non-Homogeneous
Solution 725 System of Linear Equations  746
18.23  Minor of Any Element of a Matrix   746
Solved JEE 2017 Questions  734 18.24 Cofactor of Any Element of a Matrix  747
18.25  Determinant of Any Matrix   747
Chapter 18  Matrices and 18.26  Properties of Determinants  747
Determinants 735 18.27  Sum of Determinants   750
18.1  Definition of a Matrix  735 18.28  Multiplication of Determinants  750
18.2  Order of a Matrix  735 18.29  Differentiation of Determinants  752
18.3  Types of a Matrix  735 18.30  Special Determinants   752
18.4  Equality of Matrices  736   18.30.1 Symmetric Determinant  752
18.5 Addition and Subtraction of Matrices  736   18.30.2 Skew-Symmetric Determinant  753
   18.5.1  Properties of Matrix Addition  736   18.30.3 Circulant Determinants  753
18.6 Multiplication of a Matrix by a Scalar  736 18.31 Solution of System of Linear Equations  753
   18.6.1 Properties of Multiplication of a Matrix by a Scalar  736 18.31.1 Solution of System of Two Linear Equations in Two
Unknowns 753
18.7  Multiplication of Two Matrices  737
18.31.2  Solution of System of Three Linear Equations in
   18.7.1  Properties of Matrix Multiplication   738
Three Unknowns  753
18.8  Operations Regarding Matrices  738 18.31.3 Solution of System of Three Equations in Two
18.8.1  Transpose of a Matrix  738 Unknowns   754
18.8.2  Conjugate of a Matrix   739   18.31.4 Cramer’s Rule  754
18.8.3  Transpose of the Conjugate of a Matrix  739 18.31.5 System of Homogeneous Linear Equations  755
18.8.4  Trace of a Matrix   739
Additional Solved Examples  756
  18.9 Types of a Matrix on the Basis of Operations  739
18.10  Definition of a Determinant  740 Previous Years' Solved JEE Main/AIEEE Questions  761
18.11  Evaluation of Determinants  740 Previous Years' Solved JEE Advanced/IIT-JEE Questions  768
18.11.1  Determinants of the First Order  740
Practice Exercise 1  774
18.11.2  Determinants of the Second Order  740
18.11.3  Determinants of the Third Order   740 Practice Exercise 2  780
18.12 Minors  740 Single/Multiple Correct Choice Type Questions  780
18.13 Cofactors  741 Comprehension Type Questions  781

Prelims_Volume II.indd 7 27-Jul-18 6:21:55 PM


xii Contents

Matrix Match Type Questions  782 20.5 Derivative of Second Order y″ or y2  877
Integer Type Questions  782 20.6 Differentiation of a Function with Respect to Another
Answer Key  782 Function 878

Solutions 783 Additional Solved Examples  879

Solved JEE 2017 Questions  794 Previous Years' Solved JEE Main/AIEEE Questions  881
Previous Years' Solved JEE Advanced/IIT-JEE Questions  882
Chapter 19  Limit, Continuity and Practice Exercise 1  882
Differentiability 799
Practice Exercise 2  886
19.1  Limit of a Function  799
Single/Multiple Correct Choice Type Questions  886
 19.2 Definition  799
Comprehension Type Questions   886
19.2.1  Informal Definition of Limit  799 Matrix Match Type Questions  887
19.2.2  Formal Definition of Limit  799 Integer Type Questions  887
19.2.3  Right Hand Limit  800
19.2.4  Left Hand Limit  800 Answer Key  888
19.3  Algebra of Limits  800 Solutions 888
19.4  Evaluation of Limits  800 Solved JEE 2017 Questions  895
19.4.1 Simplification  800
19.5  Use of Standard Limits  801
Chapter 21  Applications
19.6  Some More Standard Forms  802
of Derivatives 897
19.7  Use of Expansion  803
21.1 Geometrical Interpretation of Derivative  897
 19.8 L’Hospital’s Rule  804
21.2  Tangent and Normal  897
19.9 Sandwich Theorem (Squeeze Play Theorem)  804
21.2.1 Length of  Tangent, Normal, Subtangent and
19.10 Continuity  805 Subnormal 898
19.10.2  Geometrical Meaning of Continuity  805 21.3  Angles Between Two Curves  899
19.10.3  Continuity in an Open Interval  806
21.4  dy/dx as Rate Measures  900
19.10.4  Continuity in a Closed Interval   806
19.10.5  Properties of Continuous Functions   807 21.5  Errors and Approximations  900
19.10.6  Intermediate Value Theorem   807 21.6  Monotonicity of Function  901
19.10.7  Types of Discontinuities   807 21.6.1  Increasing Behaviour of Function  901
19.11 Differentiability  808 21.6.2  Decreasing Behaviour of Function  902
19.11.1 Differentiability in an Interval   809  21.6.3 Non-Decreasing Behaviour 902
19.11.2  Properties of Differentiability  809  21.6.4 Non-Increasing Behaviour 902
21.7 Maxima and Minima of Functions of a Single Variable  903
Additional Solved Examples  810
  21.7.1 Concept of Local Maximum and Local Minimum  904
Previous Years’ Solved JEE Main/AIEEE Questions  814 21.8  Mean Value Theorems  906
Previous Years’ Solved JEE Advanced/IIT-JEE Questions  819  21.8.1 Rolle’s Theorem 906
  21.8.2  Lagrange’s Mean Value Theorem  907
Practice Exercise 1  830
21.9  Geometrical Problems  907
Practice Exercise 2  842
Additional Solved Examples   909
Single/Multiple Correct Choice Type Questions  842
Comprehension Type Questions   843 Previous Years' Solved JEE Main/AIEEE Questions  911
Matrix Match Type Questions  844 Previous Years' Solved JEE Advanced/IIT-JEE Questions  918
Integer Type Questions  844
Practice Exercise 1  927
Answer Key  845
Practice Exercise 2  933
Solutions 845
Single/Multiple Correct Choice Type Questions  933
Solved JEE 2017 Questions  868
Comprehension Type Questions  934
Matrix Match Type Questions  935
Chapter 20  Differentiation 873 Integer Type Questions  936
20.1 Introduction  873 Answer Key  936
20.2 Differentiation from First Principle  873
Solutions 937
20.3 Derivatives of Some of the Frequently Used Functions  874
20.4  Rules to Find Out Derivatives  874 Solved JEE 2017 Questions  964

Prelims_Volume II.indd 8 27-Jul-18 6:21:55 PM


Contents xiii

Chapter 22  Indefinite Integration 969 Chapter 24  Area Under the Curves 1071
22.1 Primitive or Anti-Derivative of a Function  969 24.1  Curve Tracing  1071
22.2 Indefinite Integral and Indefinite Integration  969 24.2  Steps to Draw Curve   1071
22.2.1  Fundamental Properties of Integration  969 24.3 Area of Bounded Region  1072
  22.2.2  Fundamental Formulas on Integration  969 24.4  Area Enclosed Between Two Curves  1073
22.3  Methods of Integration  972
Additional Solved Examples   1076
22.3.1  Integration by Substitution  972
22.3.2  Integration by Parts  976 Previous Years’ Solved JEE Main/AIEEE Questions  1079
22.4  Integration by Partial Fractions  979 Previous Years’ Solved JEE Advanced/IIT-JEE Questions  1083
Additional Solved Examples  993 Practice Exercise 1  1089
Previous Years' Solved JEE Main/AIEEE Questions  998 Practice Exercise 2  1092
Previous Years' Solved JEE Advanced/IIT-JEE Questions   1002 Single/Multiple Correct Choice Type Questions  1092
Practice exercise 1  1003 Comprehension Type Questions  1092
Matrix Match Type Questions  1093
Practice Exercise 2  1008
Integer Type Question  1094
Single/Multiple Correct Choice Type Questions  1008
Answer Key  1094
Comprehension Type Questions  1009
Matrix Match Type Questions  1009 Solutions 1094
Answer Key  1010 Solved JEE 2017 Questions  1109
Solutions 1010
Chapter 25  Differential Equations 1111
Solved JEE 2017 Questions  1019
25.1 Introduction  1111
25.2  Basic Definition  1111
Chapter 23  Definite Integration 1021
25.3  Order of a Differential Equation  1111
23.1 Definition  1021
25.4  Degree of a Differential Equation  1111
23.2 Geometrical Meaning of Definite Integration  1021
25.5 Formation of a Differential Equation  1111
23.3 Definite Integration as the Limit of Sum   1022   25.5.1 Steps for Formation of Differential Equations  1112
23.4  Properties of Definite Integration   1022 25.6  Solution of a Differential Equation  1113
23.5 Properties Based on Periodic Function  1028  25.6.1 General Solution 1113
23.6  Properties Based on Inequality  1030  25.6.2 Particular Solution 1113
23.7  Newton–Leibnitz Rule   1030 25.7 Differential Equations of First-Order and First-Degree  1114
23.8  Summation of Series by Integration  1031 25.7.1 Geometrical Interpretation of the Differential
23.8.1 Method to Express the Infinite Series as Definite Equations of First-Order and First-Degree  1114
Integral   1031 25.8 Solution of First-Order and First-Degree Differential
  23.9 Reduction Formulae for Definite Integration  1032 Equations 1114
23.10  Wallis Formulae  1033   25.9 Variable Separable Type Differential Equation  1114
25.10 Equation Reducible to Variable Separable Type
Additional Solved Examples  1034
Differential Equation  1114
Previous Years’ Solved JEE Main/AIEEE Questions  1036 25.11 Homogeneous Type Differential Equation  1115
Previous Years’ Solved JEE Advanced/IIT-JEE Questions  1040 25.11.1 Steps for Solving Homogeneous
Differential Equation  1115
Practice Exercise 1  1048
25.12 Non-Homogeneous Type Differential Equation  1116
Practice Exercise 2  1052 25.13  Exact Differential Equation  1119
Single/Multiple Correct Choice Type Questions   1052   25.13.1 Integrating Factor  1119
Comprehension Type Questions  1052    25.13.2  Some Useful Results  1119
Matrix Match Type Questions  1053 25.14  Linear Differential Equation  1120
Integer Type Questions   1054 25.14.1  Linear Differential Equation of First Order  1120
25.14.2 Equation Reducible to Linear Differential
Answer Key  1054
Equation (Bernoulli’s Differential Equation)  1121
Solutions 1055 25.15 Solution of Differential Equation of the First Order but of
Solved JEE 2017 Questions  1067 Higher Degree  1122

Prelims_Volume II.indd 9 27-Jul-18 6:21:55 PM


xiv Contents

25.16 Applications of Differential Equation  1124 26.14.3 Vector Normal to the Plane of Two Given Vectors   1176
25.16.1  Problem Based on Rate of Change  1124 26.14.4  Area of Parallelogram and Triangle  1176
25.16.2 Problem Based on Geometry: Some Results on 26.14.5  Moment of a Force  1177
Tangents and Normal  1125 26.14.6  Moment of a Couple   1177
Additional Solved Examples  1128 26.15  Scalar Triple Product  1178
26.15.1 Geometrical Interpretation of Scalar Triple
Previous Years’ Solved JEE Main/AIEEE Questions  1139 Product 1178
Previous Years’ Solved JEE Advanced/IIT-JEE Questions  1143 26.15.2  Properties of Scalar Triple Product  1178
  26.15.3 Tetrahedron   1178
Practice Exercise 1  1143 26.15.4  Properties of a Tetrahedron  1179
Practice Exercise 2  1147 26.15.5  Volume of a Tetrahedron   1179
26.15.6  Reciprocal System of Vectors   1179
Single/Multiple Correct Choice Type Questions   1147
26.16  Vector Triple Product  1180
Comprehension Type Questions  1147
   26.16.1  Properties of Vector Triple Product  1180
Answer Key  1148 26.17 Scalar or Vector Product of Four Vectors  1181
Solutions 1149   26.17.1 Scalar Product  1181
  26.17.2 Vector Product  1181
Solved JEE 2017 Questions  1160
26.18  Method to Prove Collinearity  1181
26.19  Vector Equation  1182
Chapter 26  Vector Algebra 1163
26.1 Introduction  1163 Additional Solved Examples  1183
  26.1.1  Scalar and Vector Quantities  1163 Previous Years’ Solved JEE Main/AIEEE Questions  1185
26.2  Representation of a Vector  1163
Previous Years’ Solved JEE Advanced/IIT-JEE Questions  1190
26.3  Types of Vectors  1163
26.4 Rectangular Resolution of Vectors (Orthogonal System of Practice Exercise 1  1197
Vectors): Resolution of a Vector in Two Dimensions  1164 Practice Exercise 2  1200
26.5 Resolution of a Vector in Three Dimensions  1164
Single/Multiple Correct Choice Type Questions  1200
26.6  Properties of Vectors  1165
Comprehension Type Questions   1201
26.7  Fundamental Theorems of Vectors  1168 Integer Type Questions   1201
26.7.1 Fundamental Theorems of Vectors in Two
Dimensions 1168 Answer Key  1201
26.7.2 Fundamental Theorems of Vectors in Three Solutions 1202
Dimensions 1168
Solved JEE 2017 Questions  1211
26.8  Linear Combinations of Vectors  1168
26.8.1  Collinear and Non-Collinear Vectors   1168 Chapter 27  Three-Dimensional
26.8.2  Relation Between Two Parallel Vectors  1168
26.8.3  Test of Collinearity of Three Points   1168
Geometry 1213
26.8.4  Test of Coplanarity of Three Vectors   1168 27.1 Rectangular Coordinate System in Space  1213
26.8.5  Test of Coplanarity of Four Points   1168 27.1.1  Coordinates of a Point in Space  1213
26.9 Linearly Dependent and Independent Vectors  1168    27.1.2  Signs of Coordinates of a Point  1213
26.9.1  Linearly Independent Vectors   1168 27.2 Other Methods of Defining the Position of Any Point
   26.9.2  Linearly Dependent Vectors   1168 P in Space  1213
26.10 Position Vector of a Dividing Point (Section 27.2.1 Cylindrical Coordinates  1213
Formulae) 1169 27.2.2  Spherical Polar Coordinates  1213
26.11 Bisector of the Angle Between Two Vectors  1170 27.3  Shifting the Origin  1214
26.12  Product of Two Vectors  1171  27.4 Distance Formula  1214
26.13 Scalar or Dot Product of Two Vectors  1171    27.4.1  Distance of a Point from Coordinate Axes  1214
26.13.1 Geometrical Interpretation of Scalar Product  1171  27.5 Section Formula  1214
26.13.2  Properties of Scalar Product  1172 27.5.1 Internal Division  1214
26.13.3 Components of a Vector Along and Perpendicular to    27.5.4  Coordinates of the General Point  1214
Another Vector  1173 27.6   Triangle and Tetrahedron  1215
   26.13.4  Work Done by a Force   1173 27.6.1  Coordinates of the Centroid  1215
26.14 Vector or Cross-Product of Two Vectors  1174 27.6.2  Area of a Triangle  1215
26.14.1 Geometrical Interpretation of the Vector 27.6.4  Condition of Collinearity  1215
Product 1175 27.7  Direction Cosines of a Line  1215
26.14.2  Properties of Vector Product  1175 27.7.1  Relation Between the Direction Cosines  1216

Prelims_Volume II.indd 10 27-Jul-18 6:21:55 PM


Contents xv

 27.8 Direction Ratios  1216 27.21  Line and Plane  1228


27.8.1 Direction Cosine and Direction Ratio of a 27.21.1 Conversion of Unsymmetrical Form of Line to
Line joining Two Given Points  1217 Symmetrical Form  1228
27.9  Projection of a Line  1218 27.21.2  Angle Between Line and Plane  1229
27.9.1 Perpendicular Distance of a Point 27.21.3  Intersection of Line and Plane   1230
from a Line  1218 27.21.4  Coplanarity of Two Lines   1230
27.10 Equation of a Straight Line in Space  1219 27.21.5  Image of a Line in Plane  1230
27.10.1 Vector Equation of a Line Passing Through 27.22 Sphere  1231
a Given Point and Parallel to a 27.22.1  Equation of Sphere in Different Forms  1231
Given Vector  1219 Additional Solved Examples  1232
27.10.2 Cartesian Equation of a Line Passing
Through a Given Point and Given Previous Years’ Solved JEE Main/AIEEE Questions  1235
Direction Ratios  1219 Previous Years’ Solved JEE Advanced/IIT-JEE Questions  1241
27.10.3 Vector Equation of a Line Passing Through
Practice Exercise 1  1247
Two Given Points  1219
27.10.4 Cartesian Equation of a Line Passing Practice Exercise 2  1249
Through Two Given Points  1219
Single/Multiple Correct Choice Type Questions  1249
27.11  Angle Between Two Lines  1220 Comprehension Type Questions   1250
  27.11.1 Cartesian Form  1220 Integer Type Questions   1250
    27.11.2  Vector Form  1221
Answer Key  1251
27.12  Intersections of Two Lines  1221
27.13 Shortest Distance Between Two Non-intersecting Solutions 1251
Lines 1222 Solved JEE 2017 Questions  1260
27.13.1 Vector Form  1222
27.13.2 Cartesian Form  1223
Chapter 28  Probability 1265
27.14  Point and Line  1223
27.14.1 Foot of Perpendicular from a Given Point to 28.1 Introduction  1265
the Given Line  1223 28.2 Concept of Probability in Set Theoretic Language  1265
27.14.2 Reflection or Image of a Point in a Straight  28.2.1 Random Experiment 1265
Line 1224 28.2.2  Sample Space and Sample Points  1265
27.15  The Plane  1225  28.2.3 Trial 1265
27.16 Equation of Plane in Different Forms  1225  28.2.4 Event 1265
28.2.5  Algebra of Events  1266
27.16.1  General Equation of Plane  1225
28.2.6  Equally Likely Events  1266
27.16.2  Equation of Coordinate Planes   1225
28.2.7  Mutually Exclusive Events   1266
27.16.3  Equation of a Plane in Vector Form  1225
 28.2.8 Exhaustive Events 1266
27.16.4  Equation of Plane in Various Forms  1225
27.16.5 Equation of Plane Parallel to Coordinate 28.3 Definition of Probability with Discrete
Plane or Perpendicular to Coordinates Sample Space  1267
Axis 1225 28.4  Axiomatic Definition  1267
27.16.6 Equation of Plane Perpendicular to Coordinate 28.5  Basic Theories  1267
Plane or Parallel to Coordinates Axis  1226 28.6  Conditional Probability  1268
27.16.7 Equation of Plane Passing Through a Point and 28.7  Independent Events  1268
Having Given Direction Ratio  1226
28.8  Total Probability  1270
27.16.8 Equation of Plane Passing Through Three
Non-Collinear Points  1226 28.9 Bayes’ Theorem or Inverse Probability  1271
27.17  Point and Plane  1226 28.10 Random Variable and Probability Distribution  1272
27.17.1  Position of Two Points w.r.t the Plane  1226 28.10.1 Probability Distribution of Random Variable   1272
  27.17.2 Perpendicular Distance  1227 28.11  Binomial Distribution  1273
   27.17.3  Image of a Point About Plane Mirror  1227 28.11.1 Recurrence Formula for Binomial Distribution  1274
27.18  Angle Between Two Planes  1227 28.11.2 Mean and Variance of Binomial Distribution   1274
  27.18.1 Cartesian Form  1227 28.12  Poisson Distribution  1275
27.18.2  Vector Form   1227 28.13 Probability of Events in Experiments with
27.19  Angle Bisectors of Two Planes  1228 Countable Infinite Sample Space  1275
  27.19.1 Cartesian Form  1228 28.14  Important Information  1277
  27.19.2 Vector Form  1228
Additional Solved Examples  1278
27.20  Family of Plane  1228

Prelims_Volume II.indd 11 27-Jul-18 6:21:55 PM


xvi Contents

Previous Years’ Solved JEE Main/AIEEE Questions  1281 Answer Key  1299
Previous Years’ Solved JEE Advanced/IIT-JEE Questions  1285 Solutions 1299
Practice Exercise 1  1292 Solved JEE 2017 Questions  1310
Practice Exercise 2  1296
Single/Multiple Correct Choice Type Questions  1296
Appendix: Chapterwise Solved
Comprehension Type Questions   1297 JEE 2018 Questions A-1
Matrix Match Type Questions  1297
Integer Type Questions   1298

Prelims_Volume II.indd 12 27-Jul-18 6:21:55 PM


17 Inverse Trigonometry

17.1 Introduction p
Similarly, even if cot æç - ö÷ = - 3 but cot -1 ( - 3 ) ≠ -
p
è 6 ø 6
The inverse of a function f : A → B exists if f is one-one onto, that is,
a bijection and is given by f (x) = y ⇒ f  -1(y) = x. because principal range of cot -1x is (0,p).
Consider the sine function with domain R and range [-1, 1]. So, cot-1 ( - 3 ) = 5p only.
Clearly this function is not a bijection and so it is not invertible. If 6
we restrict the domain of it in such a way that it becomes one–one, Note:
then it would become invertible. If we consider sine as a function See Fig. 17.1. Here, sin-1 x, cosec-1 x, tan-1 x, belong to I and IV
1. 
é p pù quadrants.
with domain ê - , ú and co-domain [-1, 1], then it is a bijection p /2
ë 2 2û
and therefore, invertible. The inverse of sine function is defined as
é p pù I
sin-1 x = q Û sinq - x , where ê - , ú and x Î [ -1,1]
ë 2 2û
Hence, sin-1 x is an angle and it denotes the smallest numerical IV
angle, whose sine is x.
-p/2
17.2 Domain and Range of Inverse Figure 17.1
Trigonometric Functions 2. See Fig. 17.2. Here, cos-1 x, sec-1 x, cot-1 x, belong to I and II
p quadrant.
We know that tan = 3
3
p
This is written in inverse trigonometry as = tan-1 3 . II I
3
0
But, tan 4p is also equal to 3
3
Does it mean, 4p = tan-1 3 ?
3
The answer is no, tan-1 3 is taken as the numerically least ­angle Figure 17.2
whose tangent is 3 . This is done to associate a single value to 3.  I quadrant is common to all the inverse functions.
tan-1 3 to safeguard the definition of a function. 4.  III quadrant is not used in inverse functions.
So, the equations tan x = y and x = tan-1 y are not identical p
5. IV quadrant is used in the clockwise direction, that is, - ≤ y ≤ 0.
because the former associates many values of x to a single value 2
of y, while the latter associates a single x to a particular value of y. The principal range of inverse trigonometric functions is the most
In the same way, the remaining five inverse trigonometric func- important thing in this lesson. All formula and problems are linked
tions are also defined. To assign a unique angle to a particular in some way or the other to that only.
value of trigonometric ratio, we introduce a term called ‘principal
range’. The principal ranges of all the inverse trigonometric func- 1. See Fig. 17.3. If sin y = x, then y = Y (1, p /2 )
tions have been fixed. For example, principal range of sin-1 x is sin-1 x, under certain condition.
 p p -1 ≤ siny ≤ 1, but sin y = x.
 - 2 , 2  , that is, we have to search for an angle in this interval only. y = sin-1x
  Hence,
X
1 p 5p 1 13p 1 -1 ≤ x ≤ 1 O
sin-1 = only, although sin = , sin = , etc. Again,
2 6 6 2 6 2
p
sin y = -1 Þ y = -
æ ö 2 (-1,-p /2)
ç -1 1 1 ÷ p
ç note that sin ¹ ÷ and    sin y = 1⇒ y = Figure 17.3
çç 2 sin 1 ÷ 2
÷
è 2ø Keeping in mind numerically smallest angles or real numbers.

Mathematical Problem Book for JEE.indb 699 07-06-2018 12:26:38


700 Mathematics Problem Book for JEE

Hence, So,
p p Domain: x ∈ R
- £y£
2 2 Range: y ∈ (0, p )

These restrictions on the values of x and y provide us with the 5. See Fig. 17.7. If sec y = x, then
p
domain and range for the function, y = sin-1 x. y = sec-1 x
So, p y = p /2
where | x | ≥ 1 and 0 £ y £ p , y ¹
Domain: x ∈ [-1,1] 2 X
Here,
é p pù
Range: y Î ê - , ú Domain: x ∈ R - (-1,1) (1,0)
ë 2 2û y = sec−1x
ìp ü
2. See Fig. 17.4. If cos y = x, then y = cos-1 x, Range: y Î [0 , p ] - í ý
î2 þ Figure 17.7
under certain conditions.
6. See Fig. 17.8. If cosec y = x, then y
-1 ≤ cos y ≤ 1 ⇒ -1 ≤ x ≤ 1 y = cosec-1 x
p p (1,p /2)
cos y = -1 ⇒ y = p where | x | ≥ 1 and - £ y £ , y ¹ 0
cos y = 1 ⇒ y = 0 2 2
x
Y Here,
(–1, p /2) Domain: x ∈ R - (-1,1)
(−1,−p) O y = cosec−1x
é p pù
y = cos–1x Range: y Î ê - , ú - {0}
ë 2 2û Figure 17.8
X
O (1, 0)
We list below (Table 17.1) the domain and principal ranges of all
the six inverse trigonometric functions.
Table 17.1 Domain and principal ranges of all the six inverse trig-
Figure 17.4 onometric functions
Hence, 0  ≤ y ≤ p {as cos x is a decreasing function in [0, π]}.
Function Domain Principal Range
T hese restrictions on the values of x and y provide us the (values of x) (values of y)
domain and range for the function, y = cos-1 x.
So, é p pù
y = sin-1 x [-1, 1] ê- 2 , 2 ú
Domain: x ∈ [-1,1] ë û
Range: y ∈ [0, p ] y = cos-1 x [-1, 1] [0, π]
3. See Fig. 17.5. If tan y = x, then y
y = tan-1 x, under certain æ p pö
conditions. y = p /2
y = tan-1 x (-∞, ∞) ç- , ÷
è 2 2ø
tan y ∈ R ⇒ x ∈ R,
p p X é p pù
-∞ < tan y < ∞ ⇒ - < y < O y = cosec-1 x (-∞, -1] ∪ [1, ∞) ê - 2 , 2 ú - {0}
2 2 ë û
Thus,
y = -p /2 y = tan–1x p
Domain: x ∈ R; y = sec-1 x (-∞, -1] ∪ [1, ∞) [0,p ] - ìí 2 üý
æ p pö Figure 17.5 î þ
Range: y Î ç - , ÷
è 2 2ø y = cot-1 x (-∞, ∞) (0, π)

4. See Fig. 17.6. If cot y = x, then


y = cot-1 x, under certain Illustration 17.1   Evaluate the following:
y =p
conditions. æ 3ö
(A) tan-1 (-1) (B) cot-1 (-1) (C) sin-1 çç - ÷÷
cot y ∈ R ⇒ x ∈ R; è 2 ø
Solution:
-∞ < cot y < ∞ ⇒ 0 < y < p (0, p/2)
These conditions on x and y make æ -p ö
x (A) tan ç ÷ = -1
the function, cot y = x one-one y = cot x
−1 è 4 ø
and onto, so that the inverse Hence,
function exists, that is, y = cot -1 x is Figure 17.6 p ì -p ü
tan-1 ( -1) = - ísince Î range of tan-1 x ý
meaningful. 4 î 4 þ

Mathematical Problem Book for JEE.indb 700 07-06-2018 12:26:49


Chapter 17 | Inverse Trigonometry 701

æ 3p ö  (v) sec-1(- x) = p - sec-1 x, ∀ x ∈ (-∞, -1] ∪ [1, ∞)


(B) cot ç ÷ = -1 (vi) cosec-1(- x) = -cosec-1 x, ∀ x ∈ (-∞, -1] ∪ [1, ∞)
è 4 ø
Hence, Proof (i): sin-1 (-x) = -sin-1 x, provided that -1 ≤ x ≤ 1
3p ì 3p ü As (-x) ∈ [-1, 1]
cot -1( -1) = ísince Î range of cot -1 x ý
4 î 4 þ ⇒ x∈ [-1, 1]
é p pù
Let sin-1(-x) = θ. Then q Î ê - , ú .
æ -p ö - 3 ë 2 2û
(C) sin ç ÷=     (-x) = sinθ
è 3 ø 2 ⇒ x = -sinθ
Hence, ⇒ x = sin(-θ)
æ - 3 ö -p ì -p ü ⇒ (-θ) = sin-1x
sin-1 çç ÷÷ = ísince Î range of sin-1 x ý ⇒ θ = -sin-1x
è 2 ø 3 î 3 þ
⇒ sin-1(-x) = -sin-1x, ∀ x ∈ [-1, 1]
Proof (ii): cos-1 (-x) = p - cos-1 x, provided that -1 ≤ x ≤ 1
Illustration 17.2   Simplify As (-x) ∈ [-1, 1]
-1 
- 2 -1  -1 -1 -1  -1 
⇒ x∈ [-1, 1]
sin   + cos  2  - tan ( - 3 ) + cot   Let cos-1 (-x) = θ. Then q ∈ [0, p].
 2  3
       -x = cosθ
Solution: The value is ⇒ x = cos (p - θ)
-p 2p æ -p ö æ 2p ö ⇒ cos-1x = p - θ
+ -ç ÷+ç ÷ ⇒ θ = p - cos-1x
4 3 è 3 ø è 3 ø
⇒ cos-1(-x) = p - cos-1x, ∀ x ∈ [-1, 1]
p 2p p 2p 17p Similarly, we can do the remaining ones from (iii) to (vi).
=- + + + =
4 3 3 3 12 Property 4
p
 (i) sin-1 x + cos -1 x = , ∀ x ∈ [-1, 1]
17.3 Properties of Inverse Trigonometric 2
Functions p
 (ii) tan-1 x + cot -1 x = , ∀ x ∈ R
2
Property 1
-1 -1 p
 p p (iii) sec x + cosec x = , ∀ x ∈ (-∞, -1] ∪ [1, ∞)
 (i) sin-1(sin q) = q, ∀q ∈  - ,  2
 2 2
 (ii) cos-1(cos q) = q, ∀ q ∈ [0, p] Proof (i): sin-1x + cos-1x = p/2, ∀ x ∈ [-1, 1]
Let sin-1x = θ. Then ∀ x ∈ [-1, 1].
 p p
(iii) tan-1(tan q) = q, ∀q ∈ - ,  where, θ ∈ [-p/2, p/2]
 2 2
-1
(iv) cot (cos q) = q, ∀ q ∈ (0, p)   - p /2 ≤ θ ≤ p /2
p  ⇒ - p /2 ≤ -θ ≤ p /2
 (v) sec-1(sec q) = q, ∀q ∈[0 , p ] -   ⇒ 0 ≤ p /2 -θ ≤ p 
2
⇒ (p/2 - θ) ∈ [0, p]
(vi) cosec-1(cosec q) = q, ∀q ∈  - p , p  - {0} Since,
 2 2
 
Property 2 sin-1x = θ
 (i) sin(sin-1 x) = x, ∀ x ∈ [-1, 1] ⇒ x = sinθ
⇒ x = cos (p/2 - θ)
 (ii) cos(cos-1 x) = x, ∀ x ∈ [-1, 1] ⇒ cos-1x = p/2 - θ 
(iii) tan(tan-1 x) = x, ∀ x ∈ R ⇒ θ + cos-1x = p/2
(iv) cot(cot-1 x) = x, ∀ x ∈ R Hence, sin-1x + cos-1x = p/2.
 (v) sec(sec-1 x) = x, ∀ x ∈ (-∞, -1] ∪ [1, ∞) Proof (ii): tan-1x + cot-1x = π/2, ∀ x ∈ R
(vi) cosec(cosec-1 x) = x, ∀ x ∈ (-∞, -1] ∪ [1, ∞) Let tan-1x = θ. Then x ∈ R.
Property 3 æ p pö
where, q Î ç - , ÷
è 2 2ø
 (i) sin-1(- x) = -sin-1 x, ∀ x ∈ [-1, 1]
p p
 (ii) cos-1(- x) = p - cos-1 x, ∀ x ∈ [-1, 1]   -<q <
2 2
(iii) tan-1(- x) = -tan-1 x, ∀ x ∈ R p p
⇒ - < -q <
(iv) cot-1(- x) = p - cot-1 x, ∀ x ∈ R 2 2

Mathematical Problem Book for JEE.indb 701 07-06-2018 12:27:00


702 Mathematics Problem Book for JEE

p æ 1ö
⇒ 0< -q < p ⇒ ç ÷ ∈ [-1, 1] - {0}
2 èxø
⇒ (p/2 - θ ) ∈ (0, p)
æ 1ö
⇒ q = cos -1 ç ÷
Since, èxø
tan-1x = θ æ 1ö
⇒ x = tanθ Hence, sec -1x = cos-1 çç ÷÷÷
è xø
⇒ x = cot (p/2 - θ)
Proof (iii): Let cot-1x = q
⇒ cot-1x = p/2 - θ where, -∞ < x < ∞ and 0 < θ < p
⇒ θ + cot-1x = p/2 Now, consider two cases,
Hence, tan-1x + cot-1x = p/2.
Case I: x>0
Proof (iii): sec-1x + cosec-1x = p/2, ∀ x ∈ (-∞, -1] ∪ [1, ∞) cot-1 x = θ ⇒ θ ∈ (0, p/2)
Let sec-1x = θ. Then x ∈ (-∞, -1] ∪ [1, ∞). 1 æ 1ö
⇒x = cotθ ⇒ = tanθ ⇒ θ = tan-1 ç ÷
ìp ü x èxø
where, q Î [0 , p ] - í ý
î2 þ Hence, tan-1(1/x) = cot-1x, for all x > 0.
ìp ü Case II: x<0
⇒ q Î [0 , p ] - í ý
î2 þ ⇒ θ ∈ (p/2, p)
p é p pù p ⇒ p/2 < θ < p
⇒ - q Î ê- , ú , - q ¹ 0
2 ë 2 2û 2 ⇒ -p/2 < (θ - p) < 0
Since, ⇒ (θ - p) ∈ (-p/2, 0)
sec-1x = θ Hence, cot-1x = θ
⇒ x = sec θ ⇒ cotθ = x
1
⇒ x = cosec (p/2 - θ) ⇒ = tanθ
x
⇒ cosec-1x = p/2 - θ
1
⇒ θ + cosec-1x = p/2 ⇒ = - tan(p - θ)
x
Hence, sec-1x + cosec-1 x = p/2.
1
⇒ = tan (θ - p)
Property 5 x
æ 1ö æ 1ö
  (i) sin-1 ç ÷ = cosec -1x , " x ∈ (-∞, -1] ∪ [1, ∞) ⇒ (θ - p) = tan-1 ç ÷
èxø èxø
 1 æ 1ö
 (ii) cos -1   = sec -1 x , ∀ x ∈ (-∞, -1] ∪ [1, ∞) ⇒ tan-1 ç ÷ = - p + θ
 x èxø
 1   cot x , ∀ x > 0
-1
(iii) tan-1   =  æ 1ö
 x   -p + cot -1 x , ∀ x < 0 So, tan-1 ç ÷ = - p + cot-1x, when x < 0.
èxø
Proof (i): Let cosec-1x = θ. Then x = cosec θ. Hence,
é p pù 1  cot -1 x , ∀ x > 0
where, q Î ê - , ú - {0} tan-1 = 
ë 2 2û x  -p + cot -1 x , ∀ x < 0

sin θ = æ 1 ö for x ∈ (-∞, -1] ∪ [1, ∞) Note:


ç ÷
èxø
Conversion of inverse trigonometric ratio in their domain
1
⇒ æç ö÷ ∈ [-1, 1] - {0}
èxø x 1- x 2
 (i) sin-1 x = cos -1 1- x 2 = tan-1 = cot -1
1- x 2 x
1
⇒ q = sin-1 æç ö÷
èxø æ 1 ö -1 æ 1 ö
= sec -1 ç ÷÷ = cosec ç ÷
æ 1ö ç 2 èxø
Hence, cosec -1x = sin-1 ç ÷ è 1- x ø
èxø
æ 1- x 2 ö 1
Proof (ii): Let sec-1x = θ. Then x = sec θ.  (ii) cos -1 x = sin-1 1- x 2 = tan-1 ç ÷ = sec -1
ç x ÷ x
ìp ü è ø
where, q Î [0 , p ] - í ý
î2 þ æ 1 ö æ x ö
æ 1ö = cosec -1 ç -1
÷÷ = cot çç ÷÷
cos θ = ç ÷ for x ∈ (-∞, -1] ∪ [1, ∞) ç 2 2
è 1- x ø è 1- x ø
èxø

Mathematical Problem Book for JEE.indb 702 07-06-2018 12:27:14


Chapter 17 | Inverse Trigonometry 703

-1 -1
æ x ö -1
æ 1 ö -1 æ 1 ö æ x2 x3 ö
(iii) tan x = sin çç ÷÷ = cos çç ÷÷ = cot ç ÷ Illustration 17.6   If sin-1 çç x - + - ××× ÷÷
è 1+ x
2
ø è 1+ x
2
ø èxø è 2 4 ø
-1 æ 2 ö p
4 6
æ 1+ x 2 ö x x
+ cos çç x - + - ××× ÷÷ = for 0 < | x | < 2 , then find the
= sec -1 1+ x 2 = cosec -1 ç ÷ 2 4
ç x ÷ è ø 2
è ø
value of x.
Illustration 17.3   Evaluate the following: Solution: We know that
æ 5p ö é æ 2p öù
(A)  sec-1[sec(-30°)]  (B)  sin-1 ç sin ÷   (C)  sin-1 êsin ç ÷ú p
è 3 ø ë è 3 øû sin-1 y + cos -1 y = , | y | £1
2
Solution:
Hence, according to question,
(A) sec-1[sec(-30°)] = sec-1(sec 30°) = 30°
x2 x3 x4 x6
æ 5p ö -1 æ 3ö p x- + - ××× = x 2 - + - ×××
(B)   sin-1 ç sin ÷ = sin çç - ÷÷ = - 2 4 2 4
è 3ø è 2 ø 3
p öù -1 é æ p öù p x x2 x x2
-1 é æ ⇒ = , (\ 0 < | x | < 2 ) ⇒ =
(C)   sin êsin ç p - ÷ ú = sin êsin çè 3 ÷ø ú = 3 x x 2
2 + x 2 + x2
ë è 3 øû ë û 1+ 1+
2 2
Illustration 17.4   If q = sin-1 x + cos-1 x - tan-1 x, x ≥ 0, then the ⇒ 2x + x3 = 2x2 + x3 ⇒ x = x2
smallest interval in which q lies is
Hence,
p 3p p
(A)  £ q £ (B)  0 £ q £ x - x2 = 0 ⇒ x(1 - x) = 0 ⇒ x = 0 and x = 1, but x ≠ 0.
2 4 4
So, x = 1.
p p
(C)  - p £ q £ 0 (D)  £ q £
4 4 2 2p
Illustration 17.7   If sin-1 x + sin-1 y = , then cos-1 x + cos-1 y =
3
Solution: ______.
p
q = sin-1 x + cos -1 x - tan-1 x = - tan-1 x 2p p p
2 (A)  (B)  (C)  (D)  π
We know 3 3 6
 p p Solution:
tan-1 x = A where x¨R and A ¨  - , 
 2 2 2p
sin-1 x + sin-1 y =
Hence, 3
p p -1 p p p p p 2p
≤ - tan x≤ ⇒ ≤q ≤ ⇒ - cos -1 x + - cos -1 y =
4 2 2 4 2 2 2 3
p
Illustration 17.5   Find the value of x which satisfies the equation ⇒ cos -1 x + cos -1 y =
3
é æ 1 öù
tan(cos -1 x ) = sin êcot -1 ç ÷ ú .
ë è 2 øû
Your Turn 1
æ 1ö 1
Solution: Put cot -1 ç ÷ = q . Then cot q = .
è2ø 2 -1 -1 æ 1 ö p
1. If sin x + cot ç ÷ = , then x is
Hence, è2ø 2
2 1
sinq =
5 (A) 0 (B) 
5
Put cos-1 x = f, then x = cosf. 2 3
(C)  (D)   Ans. (B)
Also 5 2
2
tanf = sinq =
5 2. The value of sin(cos-1 x) is
Therefore (A)  (1 + x2)3/2 (B)  (1 + x2)-3/2
5 (C)  (1 - x2)1/2 (D)  (1 + x2)-1/2
x = cosf =
3 Ans. (C)

Mathematical Problem Book for JEE.indb 703 07-06-2018 12:27:37


704 Mathematics Problem Book for JEE

p ì -1 æ x - y ö
3. The number of real solutions of tan-1 x ( x + 1) + sin-1 x 2 + x + 1 =
2 ïtan ç ÷ , if xy > -1
p ï è 1+ xy ø
( x + 1) + sin-1 2
x + x + 1 = is ï æ x-y ö
2 (ii) tan-1 x - tan-1 y = ïíp + tan-1 ç ÷ , if x > 0 , y < 0 and xy < -1
(A)  Zero (B)  One ï è 1+ xy ø
ï æ ö
(C)  Two (D)  Infinite Ans. (C) ï-p + tan-1 ç x - y ÷ , if x < 0 , y > 0 and xy < -1
ïî è 1+ xy ø
1 -2 6
4. Evaluate cos 2cos-1 x + sin-1 x at x = .  Ans.
5 5 Note:
æ 5p ö -1 æ 5p ö é x + y + z - xyz ù
5. Find the value of cos -1 ç cos ÷ + sin ç sin ÷ . Ans. 0  (i) tan-1 x + tan-1 y + tan-1 z = tan-1 ê ú
è 3 ø è 3 ø ë1- xy - yz - zx û
-1 -1 11p é S - S + S - ××× ù
6. The equation 2 cos x + sin x = has (ii) tan-1 x1 + tan-1 x 2 + ××× + tan-1 x n = tan-1 ê 1 3 5 ú
6 ë1- S2 + S4 - S6 + ×× × û
(A) No solution (B) Only one solution where Sk denotes the sum of the products of x1, x2, …, xn taken k
(C) Two solutions (D) Three solutions Ans. (A) at a time.

Proof (i) Let tan-1 x = A and tan-1 y = B where


17.4 General Values of Inverse ææ pp pp öö
xx,, yy Î
ÎRR and
and AA,,BBÎ
Îçç-- ,, ÷÷ . Then
Circular Functions èè 22 22 øø
We know that if α is the smallest angle whose sine is x, then all tan A + tan B x + y
the angles whose sine is x can be written as np + (-1)n a, where tan( A + B ) = =
1- tan A tan B 1- xy
n = 0, 1, 2, …. Therefore, the general value of sin-1 x can be taken
as np + (-1)n a .
Case (a): When x, y > 0 and xy < 1, then
Thus, we have
-1
x 1=)nnap, -+1( ≤ n
-1x) ≤a1,;-1 ≤ x ≤ 1; x+y
a , -x1 ≤= xnsin
-1
sin-1 x = np + ( -1)nsin p≤ 1+; ( - tan( A + B ) = >0
1- xy
if
if sina = x , then if sina = x , then sin a = x , then
p pp p Therefore, tan(A + B) lies in 1st or 3rd quadrant.
p p
- ≤a ≤ - ≤ a ≤- ≤ a ≤
2 2 2 2 2 2 æ pö æ pö
x > 0 Þ A Î ç 0, ÷ , y > 0 Þ B Î ç 0, ÷
Similarly, general values of other inverse circular functions are è 2ø è 2ø
given as follows: Þ A + B Î (0, p )

cos-1 x = 2np ± a , -1 ≤ x ≤ 1; If cos a = x, 0 ≤ a ≤ p As (A+B) lies in 1st or 3rd quadrant. So,
p p æ pö
tan-1 x = np − a , x ∈ R ; If tan a = x, - <a < A + B Î ç 0, ÷
2 2 è 2ø
cot-1 x = np − a , x ∈ R ; If cot a = x, 0 ≤ a ≤ p  x+y
⇒ tan-1[tan( A + B )] = tan-1 
sec-1 x = 2np ± a , x ≥ 1 or x ≤ -1;  1- xy 
p  x+y
If sec a = x , 0 £ a £ p and ¹ A + B = tan-1 
2  1- xy 
cosec-1 x = np + (-1)n a , x ≥ 1 or x ≤ -1;  x+y
p p tan-1 x + tan-1 y = tan-1 
If coseca = x , - £ a £ and x ¹ 0  1- xy 
2 2
Case (b): When x, y < 0 and xy < 1, then
Property 6
x+y
ì -1 æ x + y ö tan( A + B ) = <0
ïtan ç ÷ , if x > 0 , y > 0 and xy < 1 1- xy
ï è 1- xy ø
ï æ x+y ö Therefore, tan(A + B) lies in 2nd or in 4th quadrant.
ï
 (i) tan-1 x + tan-1 y = íp + tan-1 ç ÷ , if x > 0 , y > 0 and xy > 1
ï è 1- xy ø  p   p 
ï x < 0 ⇒ A ∈  - , 0  , y < 0 ⇒ B ∈  - , 0
ï-p + tan-1 æç x + y ö÷ , if x < 0 , y < 0 and xy > 1  2   2 
ïî è 1- xy ø ⇒ A + B ∈( -p , 0 )

Mathematical Problem Book for JEE.indb 704 07-06-2018 12:27:55


Chapter 17 | Inverse Trigonometry 705

As (A+B) lies in 2nd or 4th quadrant. So, As (A+B) lies in 1st or 3rd quadrant. So,
æ pö
æ p ö A + B Î ç -p , - ÷
A + B Îç - , 0 ÷ è 2ø
è 2 ø
 x+y
 x+y ⇒ tan-1[tan( A + B + p )] = tan-1 
⇒ tan-1[tan( A + B )] = tan-1   1- xy 
 1- xy 
 x+y
 x+y A + B + p = tan-1 
A + B = tan-1   1- xy 
 1- xy 
 x+y
 x+y tan-1 x + tan-1 y = -p + tan-1 
tan-1 x + tan-1 y = tan-1   1- xy 
 1- xy 
Proof (ii): L et tan-1 x = A and tan-1 y = B where
Case (c): When x > 0, y < 0
æ p pö
x , y Î R and A, B Î ç - , ÷ . Then
æ pö æ p ö è 2 2ø
x > 0 Þ A Î ç 0, ÷ , y < 0 Þ B Î ç - , 0 ÷
è 2ø è 2 ø tan A - tan B x - y
tan( A - B ) = =
æ p pö 1+ tan A tan B 1+ xy
Þ A + B Îç - , ÷
è 2 2ø Case (a): When xy > -1, then
-1 x+y-1  x-y
⇒ tan [tan( A + B )] = tan  tan( A - B ) =
 1- xy  1+ xy
 x+y æ pö æ pö
A + B = tan-1  if x > 0 , A Î ç 0 , ÷ , y > 0 Þ B Î ç 0 , ÷
 1- xy  è 2ø è 2ø
 x+y æ p pö
tan-1 x + tan-1 y = tan-1  Þ A - B Îç - , ÷
 1- xy  è 2 2ø

Same for x < 0, y > 0. So,


 x-y
tan-1[tan( A - B )] = tan-1 
Case (d): When x, y > 0 and xy > 1, then  1+ xy 
 x-y
x+y A - B = tan-1 
tan( A + B ) =
1- xy
<0  1+ xy 
 x-y
Therefore, tan(A + B) lies in 2nd or in 4th quadrant. tan-1 x - tan-1 y = tan-1 
 1+ xy 
 p  p Same for all values of x and y with xy > -1.
x > 0 ⇒ A ∈ 0 ,  , y > 0 ⇒ B ∈ 0 , 
 2  2
Case (b): When x > 0, y < 0 and xy < -1, then
⇒ A + B ∈(0 , p ) x-y
tan( A - B ) = <0
1+ xy
As (A+B) lies in 2nd or 4th quadrant. So,
Therefore, tan(A - B) lies in 2nd or in 4th quadrant.
æp ö
A + B Îç ,p ÷ æ pö æ p ö
è2 ø x > 0 Þ A Î ç 0, ÷ , y < 0 Þ B Î ç - , 0 ÷
è 2ø è 2 ø
 x+y
⇒ tan-1 [tan( A + B - p )] = tan-1  Þ A - B Î (0, p )
 1- xy 
 x+y As (A−B) lies in 2nd or 4th quadrant. So,
A + B - p = tan-1 
 1- xy  æp ö
A - B Îç ,p ÷
è2 ø
 x+y
tan-1 x + tan-1 y = p + tan-1 
 1- xy   x-y
⇒ tan-1[tan( A - B - p )] = tan-1 
 1+ xy 
Case (e): When x, y < 0 and xy > 1, then
 x-y
A - B - p = tan-1 
x+y  1+ xy 
tan( A + B ) = <0
1- xy  x-y
tan-1 x - tan-1 y = p + tan-1 
Therefore, tan(A + B) lies in 1st or in 3rd quadrant.  1+ xy 

æ p ö æ p ö Case (c): When x < 0, y > 0 and xy < -1, then


x < 0 Þ AÎç - , 0 ÷ , y < 0 Þ B Îç - , 0 ÷
è 2 ø è 2 ø x-y
tan( A - B ) = >0
Þ A + B Î ( -p , 0 ) 1+ xy

Mathematical Problem Book for JEE.indb 705 07-06-2018 12:28:17


706 Mathematics Problem Book for JEE

Therefore, tan(A - B) lies in 1st or in 3rd quadrant. Hence,


 p   p æ tan 2q ö
x < 0 ⇒ A ∈  - , 0 , y > 0 ⇒ B ∈  0 ,  tan-1 ç
-1 -1
÷ + tan ( cot q ) + tan ( cot q ) = 0
3
 2   2 è 2 ø
⇒ A - B ∈( -p , 0 )
Illustration 17.9   Find the number of positive integral
As (A−B) lies in 1st or 3rd quadrant. So, y 3
æ pö solutions of the equation tan-1 x + cos -1 = sin-1 or
A - B Î ç -p , - ÷ 1+ y 2 10
è 2ø tan-1 x + cot-1 y = tan-1 3.
 x-y Solution:
⇒ tan-1[tan( A - B + p )] = tan-1 
 1+ xy  1 1
tan-1 x + tan-1 = tan-1 3 or tan-1  = tan-1 3 - tan-1 x
 x-y
-1 y y
A - B + p = tan 
 1+ xy  1 3- x 1+ 3 x
or tan-1 = tan-1 ⇒ y=
 x-y y 1+ 3 x 3- x
tan-1 x - tan-1 y = -p + tan-1 
 1+ xy  As x, y are positive integers, x = 1, 2 and corresponding y = 2, 7.

Hence, solutions are (x, y) = (1, 2), (2, 7).
Illustration 17.8   Prove that
é æ3ö æ 3 öù
 p p Illustration 17.10   Find the value of tan êsin-1 ç ÷ + cos -1 ç ÷ ú.
è5ø è 13 ø û
-1  1  0 , if 4 < q < 2 ë
-1 -1
tan  tan 2q  + tan (cotq ) + tan (cot q ) = 
3 Solution:
2  p , iff 0 < q < p
 é æ3ö æ 3 öù æ -1 3 2ö
4 tan êsin-1 ç ÷ + cos -1 ç ÷ ú = tan ç tan + tan-1 ÷
ë è 5 ø è 13 ø û è 4 3ø
Solution:
p æ 3 2 ö
Case (a): If 0 < q <  , then cot q > 1, cot3 q > 1. ç -1 + ÷  -1  17 12   17
4 = tan ç tan 4 3 ÷ = tan  tan  ×   =
çç 3 2   12 6   6
Hence, 1- × ÷÷
è 4 3ø
ìï cot q + cot 3q üï
tan-1 (cot q ) + tan-1 (cot3 q ) = p + tan-1 í ý 1 1
4
ïî 1- cot q ïþ Illustration 17.11   Find the value of tan-1 + tan-1 .
2 3
Solution:
Taking cot q common from numerator and using,
1 1
+
(1+cot2 q ) = cosec2 q tan -1 1
+ tan -1 1
= tan 2 3 = tan-11 = p
-1
2 3 1 1 4
1- ×
- 1 ì cot q × cosec q × sin q ü
2 4

= p + tan í- 2 3
ý
î cos q - sin q
4 4
þ Property 7
-1 ì - sinq cosq ü
ìsin-1( x 1- y 2 + y 1- x 2 ),
= p + tan í 2 ý ï
î cos q - sin2 q þ ï if x , y Î [ -1,1] and x 2 + y 2 £ 1
ï
-1 ì 1 ü ï or iff xy < 0 and x 2 + y 2 > 1
= p + tan í- tan 2q ý ïï
î 2 þ
(1) sin-1 x + sin-1 y = íp - sin-1( x 1- y 2 + y 1- x 2 ),
-1 æ 1 ö ï
= p - tan ç tan 2q ÷ ï if x , y Î (0 ,1] and x 2 + y 2 > 1
è2 ø ï -1 2 2
p ï-p - sin ( x 1- y + y 1- x ),
since 2q < and tan 2q > 0 ï
2 ïî if x , y Î [ -1, 0 ) and x 2 + y 2 > 1
Hence,
ìsin-1( x 1- y 2 - y 1- x 2 ),
æ1 ö ï
tan-1 ç tan 2q ÷ + tan-1 (cot q ) + tan-1 (cot3 q ) = p
è2 ø ï if x , y Î [ -1,1] and x 2 + y 2 £ 1
ï
p p
ï or iff xy > 0 and x 2 + y 2 > 1
Case (b): If < q < , then 0 < cot q < 1, 0 < cot3 q < 1 ïï
4 2 (2) sin-1 x - sin-1 y = íp - sin-1( x 1- y 2 - y 1- x 2 ),
Therefore, ï
æ 1 ö ï if x Î (0 ,1], y Î [-1, 0 ) and x 2 + y 2 > 1
tan-1 (cot q ) + tan-1 (cot3 q ) = tan-1 ç - tan 2q ÷ ï -1 2 2
è 2 ø ï-p - sin ( x 1- y - y 1- x ),
ï
æ1 ö ïî if x Î [ -1, 0 ), y Î (0 ,1] and x 2 + y 2 > 1
= -tan-1 ç tan2q ÷ {since 2q > p and tan2q < 0}
è2 ø

Mathematical Problem Book for JEE.indb 706 07-06-2018 12:28:37


Chapter 17 | Inverse Trigonometry 707

Proof (1): Let sin-1 x = A and sin-1 y = B where


sin-1 x + sin-1 y = sin-1 ( x 1- y 2 + y 1- x 2 )
é p pù
x, y ∈ [-1,1] and A, B Î ê - , ú
ë 2 2û Proof (2): Replace y by -y in Proof (1).
A + B ∈ [-p, p] Illustration 17.12   Find the value of x which satisfies the equa-
sin (A + B) = sin A cos B + sin B cos A
1 2
2 2
tion sin-1 + sin-1 = sin-1 x .
sin( A + B ) = x 1- y + y 1- x 3 3

Case (a):  If x, y ∈ [-1, 0) and x2 + y2 > 1, then Solution:


1 2 é1 4 2 1ù é 5+4 2ù
é pö sin-1 + sin-1 = sin-1 ê 1- + 1- ú = sin-1 ê ú
A + B Î ê -p , - ÷ 3 3 êë 3 9 3 9 ûú êë 9 úû
ë 2ø
and sin(A + B) < 0 and cos (A + B) < 0 5+4 2
Therefore, x = .
9
p p
A+B < - Þ A< - -B
2 2 Illustration 17.13   Find the value of C which satisfies the equa-
Þ cos A < - sin B Þ 1- x < - y Þ x + y > 1
2 2 2
3 æ 12 ö
tion sin-1 + cos -1 ç ÷ = sin-1 C .
-1 -1 2 2 5 è 13 ø
sin [sin( A + B )] = sin [ x 1- y + y 1- x ]
Solution: Given,
sin-1{sin[ -p - ( A + B )]} = sin-1 [ x 1- y 2 + y 1- x 2 ]
3 12
sin-1 C = sin-1 + cos-1
-p - ( A + B ) = sin-1( x 1- y 2 + y 1- x 2 ) 5 13
Hence,
( A + B ) = -p - sin-1( x 1- y 2 + y 1- x 2 ) 3 5 ìï 3 25 5 9 ïü
sin-1 C = sin-1 + sin-1 = sin-1 í 1- + 1- ý
5 13 îï 5 169 13 25 þï
sin-1 x + sin-1 y = -p - sin-1 ( x 1- y 2 + y 1- x 2 )
æ 56 ö 56
Case (b):  If x, y ∈ (0,1] and x 2 + y 2 > 1, then = sin-1 ç ÷ Þ C =
è 65 ø 65
æp ù
A + B Îç ,p ú Property 8
è2 û
and sin(A + B) > 0 and cos (A + B) < 0 ìcos -1 ( xy - 1- x 2 × 1- y 2 ),
ï
p p ï if x , y Î [ -1,1] and x + y ³ 0
A+B > Þ A> -B (1) -1 -1
cos x + cos y = í
2 2
ï2p - cos -1 ( xy - 1- x 2 × 1- y 2 ),
Þ cos A < sin B Þ 1- x 2 < y Þ x 2 + y 2 > 1 ï
î if x , y Î [ -1,1] and x + y £ 0
sin-1 [sin( A + B )] = sin-1 [ x 1- y 2 + y 1- x 2 ] ìcos-1 ( xy + 1- x 2 × 1- y 2 ),
ï
ï if x , y Î [ -1,1] and x £ y
sin-1 {sin[p - ( A + B )]} = sin-1 [ x 1- y 2 + y 1- x 2 ] (2) -1 -1
cos x - cos y = í

ï- cos-1( xy + 1- x 2 × 1- y 2 ),
p - ( A + B ) = sin-1 ( x 1- y 2 + y 1- x 2 ) ï
î if x Î (0 ,1], y Î [ -1, 0 ) and x ³ y
( A + B ) = p - sin-1 ( x 1- y 2 + y 1- x 2 ) Proof (1): Let cos-1 x = A and cos-1 y = B where

x, y ∈ [-1,1] and A, B ∈ [0, p]
sin-1 x + sin-1 y = p - sin-1( x 1- y 2 + y 1- x 2 )
⇒ A + B ∈ [0, 2p]
Case (c):  If x, y ∈ [-1,1] and x 2 + y 2 ≤ 1, then cos (A + B) = cos A cos B – sin B sin A
é p pù cos( A + B ) = xy - 1- x 2 1- y 2
A + B Î ê- , ú
ë 2 2û
Case (a): If x, y ∈ [-1,1] and x + y ≥ 0, then
and sin(A + B) > 0 and cos (A + B) > 0
A + B ∈ [0, p]
p p
A+B £ Þ A£ -B A+B≤p⇒A≤p-B
2 2
⇒ cos A ≥ -cos B ⇒ cos A + cos B ≥ 0 ⇒ x + y ≥ 0
Þ cos A ³ sin B Þ 1- x 2 ³ y Þ x 2 + y 2 £ 1
cos -1[cos( A + B )] = cos -1 ( xy - 1- x 2 1- y 2 )
sin-1 (sin( A + B )) = sin-1 ( x 1- y 2 + y 1- x 2 )
A + B = cos -1( xy - 1- x 2 1- y 2 )
( A + B ) = sin-1 ( x 1- y 2 + y 1- x 2 ) cos -1 x + cos -1 y = cos -1 ( xy - 1- x 2 1- y 2 )

Mathematical Problem Book for JEE.indb 707 07-06-2018 12:29:12


708 Mathematics Problem Book for JEE

Case (b): If x, y ∈ [-1,1] and x + y ≤ 0, then x y


Illustration 17.16   If cos -1 + cos -1 = q , then
A + B ∈ [p, 2p] 2 3
A+B≥p⇒A≥p-B 9x2 - 12xy cos q + 4y2 is equal to
⇒ cos A ≤ -cos B ⇒ cos A + cos B ≤ 0 ⇒ x + y ≤ 0 (A)  36 sin2 q (B)  36 cos2 q
(C)  36 tan2 q (D)  None of these
cos -1[cos( A + B )] = cos -1 ( xy - 1- x 2 1- y 2 )

Solution:
cos -1{cos[2p - (A + B)]} = cos -1( xy - 1- x 2 1- y 2 ) x y

cos -1 + cos -1 = q
2p - ( A + B ) = cos -1 ( xy - 1- x 2 1- y 2 ) 2 3

x y æ x2 ö æ y2 ö
cos -1 x + cos -1 y = 2p - cos -1( xy - 1- x 2 1- y 2 ) Þ × - çç 1- ÷÷ çç 1- ÷÷ = cosq
2 3 4 ø 9 ø
è è
Proof (2): Replace y by -y in Proof (1).
⇒ (xy - 6 cosq )2 = (4 - x2)(9 - y2)
Illustration 17.14   If cos-1x + cos-1y +cos-1z = p, then prove that ⇒ 9x2 - 12xy cosq + 4y2 = 36(1 - cos2q) = 36sin2q
x2 + y2 + z2 + 2xyz = 1.

Solution: Given,
Your Turn 2
cos-1x + cos-1y + cos-1z = p ìx 1 ü
1. If f ( x ) = cos -1 x + cos -1 í + 3 - 3 x 2 ý , then
⇒ cos-1x + cos-1y = p - cos-1z = cos-1(-z) î 2 2 þ
⇒ cos[cos-1x + cos-1y] = cos[cos-1(-z)] æ2ö p æ2ö 2 p
(A)  f ç ÷ = (B)  f ç ÷ = 2 cos -1 -
Let cos-1x = A and cos-1y = B. Then è3ø 3 è3ø 3 3
cos(A + B) = cosA cosB - sinA sinB 1 p 1 1 p
(C)  f æç ö÷ = (D)  f æç ö÷ = 2 cos -1 - Ans. (A), (D)
⇒ cos(A + B) = xy - 1- x 2
1- y 2 è3ø 3 è3ø 3 3

æxö æyö x 2 2 xy y2
⇒ (A + B) = cos-1 [ xy - 1- x 2 1- y 2 ] 2. If cos -1 ç ÷ + cos -1 ç ÷ = a , then - cos a + 2 =
èaø èbø a 2
ab b
⇒ cos-1 ( xy - 1- x 2 1- y 2 ) = cos-1(-z) (A) sin2 a (B) cos2 a
2
⇒ xy - 1- x 1- y = -z 2
(C) tan2 a (D) cot2 a  Ans. (A)
⇒ (xy + z)2 = (1 - x2) (1 - y2) ⇒ x2y2 + z2 + 2xyz = 1 - x2 - y2 + x2y2 3. All possible values of p and q for which
⇒ x2 + y2 + z2 + 2xyz = 1 3p
cos -1 p + cos -1 1- p + cos -1 1- q = holds is
Hence, proved. 4
4 1 4 1 1 1
Illustration 17.15 If a = sin-1 + sin-1 and b = cos -1 + cos -1 , (A)  p = -1, q = (B)  q > 1, p =
5 3 5 3 2 2
then 1
(C)  0 £ p £ 1, q = (D)  None of these Ans. (C)
(A)  a < b (B)  a = b 2
(C)  a > b (D)  None of these -1 -1 p
4. The number of solutions of sin x + sin 2 x = is
Solution: 3
é4 1 1 16 ù (A)  0 (B) 1 (C)  2 (D) Infinite Ans. (B)
a = sin-1 ê 1- + 1- ú
êë 5 9 3 25 ûú æ 3ö æ 5ö
5. Obtain the value of cos -1 ç - ÷ + sin-1 ç - ÷ in terms of cos-1
è 5ø è 13 ø
é8 2 3 ù æ 8 2 +3ö æ 16 ö
= sin-1 ê + ú = sin-1 çç ÷÷ function. Ans. cos -1 ç - ÷
êë 15 15 ûú è 15 ø è 65 ø
-1 12 -1 4 -1 63
6.   sin + cos + tan =
8 2 +3 p 13 5 16
Since therefore
< 1, \ a<
15 2
(A) 0 (B)  p (C)  π (D)  2p Ans. (C)
8 2 +3 p 2 3
< 1, \a <
15 2 7. If a, b, c be positive real numbers and the value of
a(a + b + c ) b(a + b + c )
æp 4 p 1ö p q = tan-1 + tan-1 , then tanq is
b = ç - sin-1 + - sin-1 ÷ = (p - a ) > bc ca
è2 5 2 3ø 2
⇒a<b (A) 0 (B) 1 (C)  a + b + c (D)  None of these Ans. (A)

Mathematical Problem Book for JEE.indb 708 07-06-2018 12:29:39


Chapter 17 | Inverse Trigonometry 709

Property 9  p p
Proof (2): Let sin-1x = A where x ∈[ -1,1] and A ∈  - ,  . Then
ì -1 2 1  2 2
ï-p - sin (2 x 1- x ), if - 1 £ x < -
ï 2 x = sin A
ï 1 1 sin 3A = 3 sin A - 4 sin3 A = 3x - 4x3
(1) 2 sin-1 x = ísin-1 (2 x 1- x 2 ), if - £x£
ï 2 2 é 3p 3p ù
ï 1 where 3 AÎê - , ú
-1
ïp - sin (2 x 1- x ), if
2
< x £1 ë 2 2 û
î 2
ì 1 é 3p p ö
-1 3 Case (a): If 3 AÎ ê - , - ÷ , then
ï-p - sin (3 x - 4 x ), if - 1 £ x < - 2 ë 2 2ø
ï
ï 1 1 é 3p p ö é 1ö
(2) 3 sin-1 x = ísin-1(3 x - 4 x 3 ), if - £ x £ A Î ê- , - ÷ Þ x Î ê -1, - ÷
ï 2 2
ë 6 6ø ë 2 ø
ï -1 3 1
ïp - sin (3 x - 4 x ), if 2 < x £ 1 sin-1 (sin 3A) = sin-1 (3x - 4x3)
î
⇒ sin-1 [sin(-p - 3A)] = sin-1 (3x - 4x3)
é p pù ⇒ -p - 3A = sin-1 (3x - 4x3)
Proof (1): Let sin-1x = A where x Î [ -1,1] and A Î ê - , ú . Then
ë 2 2û ⇒ 3A = -p - sin-1 (3x - 4x3)
x = sin A ⇒ 3sin-1 x = -p - sin-1 (3x - 4x3)
sin 2 A = 2 sin A cos A = 2 x 1- x 2 æ p 3p ù
Case (b): If 3 AÎ ç , ú , then
where 2A ∈ [-p,p] è2 2 û
 p  p p  1  æp p ù æ1 ù
Case (a): If 2 A ∈  -p , -  , then, A ∈  - , -  ⇒ x ∈  -1, -  A Î ç , ú Þ x Î ç ,1ú
 2   2 4   2 è 6 2û è2 û
 p  p p  1  sin-1 (sin 3A) = sin-1 (3x - 4x3)
2 A ∈  -p , -  , then, A ∈  - , -  ⇒ x ∈  -1, - 
 2   2 4   2 ⇒ sin-1 [sin(p - 3A)] = sin-1 (3x - 4x3)
⇒ p - 3A = sin-1 (3x - 4x3)
sin-1 (sin 2 A) = sin-1 (2 x 1- x 2 ) ⇒ 3A = p - sin-1 (3x - 4x3)
⇒ 3sin-1 x = p - sin-1 (3x - 4x3)
⇒ sin-1 [(sin( -p - 2 A)] = sin-1 (2 x 1- x 2 )
é p pù
⇒ -p - 2 A = sin-1 (2 x 1- x 2 ) Case (c): If 3 AÎ ê - , ú , then
ë 2 2û
Þ 2 A = -p - sin-1 (2 x 1- x 2 )
é p pù é 1 1ù
A Î ê- , ú Þ x Î ê- , ú
-1 -1
Þ 2 sin x = -p - sin (2 x 1- x ) 2 ë 6 6û ë 2 2û

sin-1 (sin 3A) = sin-1 (3x - 4x3)
æp ù
Case (b): If 2 A Î ç , p ú , then ⇒ 3A = sin-1 (3x - 4x3)
è2 û
⇒ 3sin-1 x = sin-1 (3x - 4x3)
æp p ù æ 1 ù
AÎç , ú Þ x Îç ,1ú Property 10
è 4 2û è 2 û
ìï2p - cos-1(2 x 2 - 1), if - 1 £ x < 0
sin-1 (sin 2 A) = sin-1 (2 x 1- x 2 ) (1) 2 cos -1 x = í
-1 2
îïcos (2 x - 1), if 0 £ x £ 1
Þ sin-1 [sin(p - 2 A)] = sin-1 (2 x 1- x 2 )
 -1 3 1
Þ p - 2 A = sin-1 (2 x 1- x 2 ) 2p + cos ( 4 x - 3 x ), if - 1 ≤ x < - 2

 1 1
Þ 2 A = p - sin-1 (2 x 1- x 2 ) (2) 3 cos -1 x = 2p - cos -1( 4 x 3 - 3 x ), if - ≤ x ≤
 2 2
Þ 2 sin-1 x = p - sin-1 (2 x 1- x 2 )  -1 3 1
cos ( 4 x - 3 x ), if 2 < x ≤ 1

é p pù
Case (c): If 2 A Î ê - , ú , then Proof (1): Let cos-1 x = A where x ∈ [-1, 1] and A ∈ [0, p]. Then
ë 2 2û
é p pù é 1 1 ù x = cos A
A Î ê- , ú Þ x Î ê- , ú cos 2A = 2 cos2 A - 1 = 2x2 - 1
ë 4 4û ë 2 2û
where 2A ∈ [0, 2p]
sin-1 (sin 2 A) = sin-1 (2 x 1- x 2 )
Case (a): If 2 AÎ[0 , p ], then
⇒ 2 A = sin-1 (2 x 1- x 2 )
é pù
A Î ê0 , ú Þ x Î [0 ,1]
⇒ 2 sin-1 x = sin-1 (2 x 1- x 2 ) ë 2û

Mathematical Problem Book for JEE.indb 709 07-06-2018 12:30:06


710 Mathematics Problem Book for JEE

cos-1 (cos 2A) = cos-1 (2x2 - 1) æ p pö


⇒ 2A = cos-1 (2x2 - 1) Proof (1): Let tan-1 x = A where x Î R and A Î ç - , ÷ . Then
⇒ 2cos-1 x = cos-1 (2x2 - 1) è 2 2ø
x = tan A
Case (b): If 2 AÎ(p , 2p ], then 2 tan A 2x
tan 2 A = =
æp ù 1- tan A 1- x 2
2

A Î ç , p ú Þ x Î [ -1, 0 )
è2 û where 2A ∈ (-p,p)
cos-1 [cos(2p - 2A)] = cos-1 (2x2 - 1) æ pö
Case (a): If 2 AÎ ç -p , - ÷ , then
⇒ 2p - 2A = cos-1 (2x2 - 1) è 2ø
⇒ 2A = 2p - cos-1 (2x2 - 1)
⇒ 2cos-1 x = 2p - cos-1 (2x2 - 1) æ p pö
A Î ç - , - ÷ Þ x Î ( -¥ , -1)
è 2 4ø
Proof (2): Let cos-1 x = A where x ∈ [-1, 1] and A ∈ [0, p]. Then
x = cos A æ 2x ö
cos 3A = 4 cos3 A - 3 cos A = 4x3 - 3x tan-1 (tan 2 A) = tan-1 ç ÷
è 1- x 2 ø
where 3A ∈ [0, 3p]
æ 2x ö
Þ tan-1 [tan(p + 2 A)] = tan-1 ç ÷
Case (a): If 3 AÎ[0 , p ), then è 1- x 2 ø
é pö æ1 ù æ 2x ö
A Î ê0 , ÷ Þ x Î ç ,1ú Þ p + 2 A = tan-1 ç
ë 3ø è2 û ÷
è 1- x 2 ø
cos-1 (cos 3A) = cos-1 (4x3 - 3x)
æ 2x ö
⇒ 3A = cos-1 (4x3 - 3x) Þ 2 A = -p + tan-1 ç ÷
⇒ 3cos-1 x = cos-1 (4x3 - 3x) è 1- x 2 ø

Case (b): If 3 AÎ[p , 2p ], then æ 2x ö


Þ 2 tan-1 x = -p + tan-1 ç ÷
é p 2p ù é 1 1ù è 1- x 2 ø
A Î ê , ú Þ x Î ê- , ú
ë3 3 û ë 2 2û é p pù
Case (b): If 2 AÎ ê - , ú , then
cos-1 [cos(2p - 3A)] = cos-1 (4x3 - 3x) ë 2 2û
⇒ 2p - 3A = cos-1 (4x3 - 3x)
⇒ 3A = 2p - cos-1 (4x3 - 3x) é p pù
⇒ 3cos-1 x = 2p - cos-1 (4x3 - 3x) A Î ê - , - ú Þ x Î [ -1,1]
ë 4 4û
Case (c): If 3 AÎ(2p , 3p ), then
æ 2x ö
tan-1 (tan 2 A) = tan-1 ç ÷
æ 2p ù é 1ö è 1- x 2 ø
AÎç , p ú Þ x Î ê -1, - ÷
è 3 û ë 2ø
 2x 
⇒ 2 A = tan-1 
cos-1 [cos(3A - 2p)] = cos-1 (4x3 - 3x)  1- x 2 
⇒ 3A - 2p = cos-1 (4x3 - 3x)
⇒ 3A = 2p + cos-1 (4x3 - 3x)  2x 
⇒ 2 tan-1 x = tan-1 
 1- x 2 
⇒ 3cos-1 x = 2p + cos-1 (4x3 - 3x)
Property 11
æp ö
ì -1 æ 2 x ö
Case (c): If 2 AÎ ç , p ÷ , then
ï-p + tan ç 2÷
, if x < -1 è2 ø
ï è 1- x ø
ï æ 2x ö æp p ö
(1) 2 tan-1 x = ítan-1 ç 2÷
, if - 1 £ x £ 1 A Î ç , ÷ Þ x Î (1, ¥ )
ï è 1- x ø è4 2ø
ï æ 2x ö
ïp + tan-1 ç , if x > 1 æ 2x ö
2÷ tan-1 (tan 2 A) = tan-1 ç ÷
îï è 1- x ø è 1- x 2 ø
ì -1 æ 3 x - x ö
3
1 æ 2x ö
ï-p + tan çç 2 ÷÷ , if x < - Þ tan-1 [tan( -p + 2 A)] = tan-1 ç ÷
ï è 1- 3 x ø 3 è 1- x 2 ø
ï
ï æ 3x - x3 ö 1 1 æ 2x ö
(2) 3 tan-1 x = ítan-1 çç 2 ÷
÷ , if - £x£ Þ -p + 2 A = tan-1 ç ÷
ï è 1- 3 x ø 3 3 è 1- x 2 ø
ï
ïp + tan-1 æç 3 x - x ö÷ , if x > 1
3
æ 2x ö
ï ç 1- 3 x ÷2 Þ 2 A = p + tan-1 ç ÷
î è ø 3 è 1- x 2 ø

Mathematical Problem Book for JEE.indb 710 07-06-2018 12:30:33


Chapter 17 | Inverse Trigonometry 711

æ 2x ö æ 3x - x 3 ö
Þ 2 tan-1 x = p + tan-1 ç ÷ Þ -p + 3 A = tan-1 ç 2 ÷
è 1- x 2 ø è 1- 3 x ø
æ p pö æ 3x - x3 ö
Proof (2): Let tan-1 x = A where x ∈ R and A ∈ ç - , ÷ . Then Þ 3 A = p + tan-1 ç
è 2 2ø 2 ÷
x = tan A è 1- 3 x ø
3 tan A - tan3 A 3 x - x 3 æ 3x - x3 ö
tan 3 A = = 3 tan-1 x = p + tan-1 ç 2 ÷
1- 3 tan2 A 1- 3 x 2 è 1- 3 x ø
æ 3p 3p ö Illustration 17.17  Let a, b and γ are three angles given by
3 AÎ ç - , ÷
è 2 2 ø

æ 3p p ö
a = 2 tan-1 ( )
2 - 1 , b = 3 sin-1
1
2
æ 1ö æ 1ö
+ sin-1 ç - ÷ and g = cos -1 ç ÷ .
è 2ø è3ø
Case (a): If 3 AÎ ç - , - ÷ , then
è 2 2ø Then

æ p pö æ 1 ö (A)  a > b (B)  b > γ


A Î ç - , - ÷ Þ x Î ç -¥ , -
÷ (C)  a > γ (D)  None of these
è 2 6ø è 3ø
 3x - x3  Solution:
tan-1 ( tan 3 A) = tan-1  
 1- 3 x 2  a = 2 tan-1 ( )
2 - 1 = 2 tan-1 tan
p
8
 3x - x3  p p 1
⇒ tan-1  tan (p + 3 A) = tan-1   = 2× = = cos -1
 1- 3 x 2  8 4 2
æ 3x - x 3 ö p p 7p
Þ p + 3 A = tan-1 ç b = 3× - =
2 ÷ 4 6 12
è 1- 3 x ø
Therefore, b > a. Also,
æ 3x - x 3 ö 1 1
Þ 3 A = -p + tan-1 ç 2 ÷ <
è 1- 3 x ø 3 2
æ 3x - x 3 ö 1 1 p
Þ 3 tan-1 x = -p + tan-1 ç 2 ÷
Þ cos -1 > cos -1 =
è 1- 3 x ø 3 2 4
So, γ > a.
é p pù
Case (b): If 3 AÎ ê - , ú , then  1
ë 2 2û Again cos -1   belongs to the first quadrant and b is in the
 3
é p pù é 1 1 ù second quadrant.
A Î ê- , ú Þ x Î ê- , ú
ë 6 6û ë 3 3û Hence, b > γ .

 3x - x3  Illustration 17.18   The value of


tan-1 ( tan 3 A) = tan-1  
 1- 3 x 2  é æ 1 öù
sin ê2 tan-1 ç ÷ ú + cos éë tan-1 2 2 ùû =
 3x - x  3
ë è 3 øû
⇒ 3 A = tan-1 
 1- 3 x 2  16 14
(A)  (B) 
 3x - x  3 15 15
⇒ 3 tan-1 x = tan-1 
 1- 3 x 2  12 11
(C)  (D) 
15 15
æ p 3p ö
Case (c): If 3 AÎ ç , ÷ , then Solution:
è2 2 ø
é æ 1 öù
æp p ö æ 1 ö sin ê2 tan-1 ç ÷ ú + cos[tan-1(2 2 )]
AÎç , ÷ Þ x Îç ,¥÷ ë è 3 øû
è 6 2 ø è 3 ø
é 2 ù
-1  3 x
- x3  ê -1 3 ú
tan ( tan 3 A) = tan 
-1
= sin ê tan + cos[tan-1(2 2 )]

 1- 3 x 2  ê 1 úú
1-
ë 9û
 3x - x3 
⇒ tan-1  tan ( -p + 3 A) = tan-1   é 3ù
= sin ê tan-1 ú + cos[tan-1 2 2 ]
 1- 3 x 2  ë 4û

Mathematical Problem Book for JEE.indb 711 07-06-2018 12:31:01


712 Mathematics Problem Book for JEE

é 3ù é 1 ù 3 1 14 2x
= sin êsin-1 ú + cos êcos -1 ú = + = sin2 A =
ë 5û ë 3 û 5 3 15 1+ x 2
æ 2x ö
Property 12 Þ sin-1[sin(p - 2 A)] = sin-1 ç ÷
è 1+ x 2 ø
 -1 
2x 
 -p - sin  1+ x 2  , if x < -1 æ 2x ö
Þ p - 2 A = sin-1 ç
 ÷
è 1+ x 2 ø
  2x 
2 tan x = sin-1 
-1
, if - 1 ≤ x ≤ 1
  1+ x 2  æ 2x ö
Þ 2 A = p - sin-1 ç ÷
 è 1+ x 2 ø
 2x 
p - sin-1  , if x > 1
  1+ x 2  æ 2x ö
Þ 2 tan-1 A = p - sin-1 ç ÷
è 1+ x 2 ø
æ p pö
Proof: Let tan-1 x = A where x Î R and A Î ç - , ÷ . Then
è 2 2ø Illustration 17.19   The solution set of the equation
x = tan A sin-1 x = 2tan-1 x is
2 tan A 2x (A)  {1, 2} (B) 
{-1, 2}
sin 2 A = =
1+ tan A 1+ x 2
2
(C) {-1,1,0} (D)  {1, 1/2, 0}
where 2A ∈ (-p,p) Solution:
æ pö 2x
Case (a): If 2 AÎ ç -p , - ÷ , then sin-1 x = 2tan-1 x ⇒ sin-1 x = sin-1
è 2ø 1+ x 2
æ p pö 2x
⇒ = x ⇒ x3 - x = 0 ⇒ x(x + 1) (x - 1) = 0 ⇒ x = {-1,1,0}
A Î ç - , - ÷ Þ x Î ( -¥ , -1) 1+ x 2
è 2 4ø
2x
Illustration 17.20   If 2 tan-1 x + sin-1 is independent of x,
2x 1+ x 2
sin2 A =
1+ x 2 then
 2x  (A)  x ∈ [1, + ∞) (B)  x ∈ [-1,1]
⇒ sin-1[sin( -p - 2 A)] = sin-1 
 1+ x 2  (C)  x ∈ (-∞, -1] (D) None of these
æ 2x ö Solution: Let x = tanq. Then
Þ -p - 2 A = sin-1 ç 2 ÷
è 1+ x ø 2x 2 tanq
sin-1 = sin-1 = sin-1(sin 2q )
æ 2x ö 1+ x 2
1+ tan2 q
Þ 2 A = -p - sin-1 ç ÷
è 1+ x 2 ø 2x
Hence, 2 tan-1 x + sin-1 = 2q + sin-1(sin 2q ).
1+ x 2
æ 2x ö
Þ 2 tan-1 A = -p - sin-1 ç ÷ p pp p
è 1+ x 2 ø If - £ 2q -£ £, then
2q £ , then
2 22 2
é p pù --11 2 x -1 2 x
Case (b): If 2 AÎ ê - , ú , then -1
2 tan x + sin x + sin
2 tan = 2q + 2q = 4 tan-1x
ë 2 2û 1+ x 2 1+ x 2
é p pù which is not independent of x
A Î ê - , ú Þ x Î [ -1,1]
ë 4 4û p p
If - ≤ p - 2q ≤ , then
2x 2 2
sin2 A =
1+ x 2 2x
2 tan-1 x + sin-1
 2x  1+ x 2
⇒ sin-1[sin(2 A)] = sin-1 
 1+ x 2  = 2q + sin-1 [sin(p - 2q )] = 2q + p - 2q = p
 2x  which is independent of x
⇒ 2 A = sin-1 
 1+ x 2  é p pù é p 3p ù
Hence, q Ï ê - , ú but q Î ê - , ú and from the principal
 2x  ë 4 4û ë 4 4 û
⇒ 2 tan-1 A = sin-1 
 1+ x 2  value of tan-1 x.
é p pù
æp ö q Î ê- , ú
Case (c): If 2 AÎ ç , p ÷ , then ë 2 2û
è2 ø
Hence,
æp p ö p p
A Î ç , ÷ Þ x Î (1, ¥ ) q Î éê , ùú
è4 2ø ë4 2û

Mathematical Problem Book for JEE.indb 712 07-06-2018 12:31:31


Chapter 17 | Inverse Trigonometry 713

2x Illustration 17.21   Write in the simplest form:


Þ tan-1 x + sin-1 =p
1+ x 2
æ cos x ö p 3p
p tan-1 ç ÷ where - £ x £
Also at q = , è 1+ sin x ø 2 2
4
2x p p p Solution:
2 tan-1 x + sin-1= 2 × + sin-1 1 = + = p
1+ x 2 4 2 2 é æp ö ù
ê sin ç - x ÷ ú
ép p ù æ cos x ö è2 ø ú
Hence, the given function = p = constant if q Î ê , ú , that is, tan-1 ç ÷ = tan ê
-1
ë4 2û è 1+ sin x ø ê1+ cos æ p - x ö ú
êë ç ÷ú
x ∈ [1, + ∞). è2 øû
Property 13 é æp x ö æp x öù
ê 2 sin ç 4 - 2 ÷ cos ç 4 - 2 ÷ ú
 -1 
1- x 2  = tan ê
-1 è ø è øú
 - cos  1+ x 2  , if x ≤ 0 ê 2æp xö ú
 2 cos ç - ÷
2 tan-1 x =  êë è4 2ø úû
 - cos -1  1- x  , if x ≥ 0
2

 
 1+ x 2 
é æ p x öù p x
= tan-1 ê tan ç - ÷ ú = -
p p  ë è 4 2 øû 4 2
Proof: Let tan-1 x = A where x ∈ R and A ∈ ,  . Then
 2 2
x = tan A Illustration 17.22   Find the angle
1- tan2 A 1- x 2
cos 2 A = = æ 3p ö
1+ tan2 A 1+ x 2 (A) tan-1 ç tan ÷ (B) sin-1 sin 5 (where 5 is in radians).
è 4 ø
where 2A ∈ (-p,p)
Solution:
Case (a): If 2 A ∈( -p , 0 ), then
 3p 
 p  (A) Let tan-1  tan  = q
A ∈ - , 0 ⇒ x ∈( -∞ , 0 )  4 
 2 
 p
1- x 2 tan-1 tan  p -  = q
cos2 A =  4
1+ x 2
 p
tan-1  - tan  = q
æ 1- x 2 ö  4
Þ cos -1 éëcos ( -2 A ) ùû = cos -1 ç ÷
è 1+ x 2 ø p é æ p p öù
⇒ - tan-1 tan = q êAs tan-1 tanq = q , if q Î ç - , ÷ ú
æ 1- x ö 2 4 ë è 2 2 øû
Þ -2 A = cos -1 ç ÷
è 1+ x 2 ø p
⇒ - =q
æ 1- x 2 ö 4
Þ -2 A = - cos -1 ç ÷
è 1+ x 2 ø (B)  We know

æ 1- x 2 ö é p pù
Þ 2 tan-1 x = - cos -1 ç ÷ sin-1 sin q = q, q ∈ ê - , ú » [ -1.57,1.57] (1)
è 1+ x 2 ø ë 2 2û
Hence, sin-1 sin 5 ≠ 5 as 5 ∉ [-1.57, 1.57].
Case (b): If 2 A ∈[0 , p ], then
Therefore,
 p sin 5 = sin (p + 5 - p )
A ∈ 0 ,  ⇒ x ∈[0 , ∞ )
 2 = - sin (5 - p)
1- x 2 Since (5 - p) ∉ [-1.57, 1.57], so we again add and subtract p.
cos2 A =
1+ x 2
⇒ sin 5 = - sin (p + 5 - 2p )
æ 1- x 2 ö
Þ cos -1[cos(2 A)] = cos -1 ç ÷ = + sin (5 - 2p) [Since (5 - 2p) ∈ [- 1.57, 1.57]]
è 1+ x 2 ø
Hence,
æ 1- x 2 ö
Þ 2 A = cos -1 ç ÷ sin-1 sin 5 = sin-1 sin (5 - 2p) = 5 - 2p
è 1+ x 2 ø
Note: To solve this type of problem, the procedure is to add

æ 1- x 2 ö and subtract p till it belongs to the principal value range of
Þ 2 tan x = cos ç
-1 -1
÷
è 1+ x 2 ø respective inverse trigonometric function.

Mathematical Problem Book for JEE.indb 713 07-06-2018 12:31:55


714 Mathematics Problem Book for JEE

Solution: We know,
Your Turn 3
p
cos -1 x = - sin-1 x
1. If 2 tan-1(cos x) = tan-1(cosec2 x), then x = 2
p Given,
(A)  (B)  π
2 p
p p cos -1x - cos -1 y =
(C)  (D)   Ans. (D) 3
6 3
p p p
1 1 1 ⇒ - sin-1 x - + sin-1 y =
2. 4 tan-1 - tan-1 + tan-1 = 2 2 3
5 70 99
p p p
(A)  (B)  sin-1 y - sin-1 x =
2 3 3
2p
p -1 -1
sin y + sin x = (1)
(C)  (D)  None of these Ans. (C) 3
4
1 1 2sin-1 y = p
3. 4 tan-1 - tan-1 is equal to
5 239 p
p sin-1 y = (2)
2
(A)  π (B)  2
⇒y=1
p p
(C)  (D)  Ans. (D) Put Eq. (2) in Eq. (1).
3 4
a3 æ1 a ö b3 æ1 bö p 2p
4. cosec2 ç tan-1 ÷ + sec2 ç tan-1 ÷ is equal to + sin-1 x =
2 è2 bø 2 è2 aø 2 3

(A) (a - b)(a2 + b2) (B) (a + b)(a2 - b2) p


sin-1 x =
(C) (a + b) (a2 + b2) (D)  None of these Ans. (D) 6
2a 1- b 2
2x 1
5. If sin-1 - cos -1 = tan-1 , then x = Þx=
1+ a 2 1+ b 2 1- x 2 2
(A)  a (B)  b 1 
a+b a-b Thus, solution set is  , 1 .
(C)  (D)  Ans. (D) 2
1- ab 1+ ab
Hence, the correct answer is option (B).
1- x 2
6. The formula cos -1 = 2 tan-1 x holds only for
1+ x 2 3. Which of the following is the solution set of the equation
(A)  x ∈ R (B) |x| ≤ 1 sin-1 x = cos-1 x + sin-1(3x - 2)?
(C)  x ∈ (-1,1] (D) 
x ∈ [0,+∞] Ans. (D)
(A)  { }
1
2
,1
1 
(B)   , 1
2 
Additional Solved Examples
1. The number of real solutions of x+cos-1 cos-1
2x = -p is
1 
(C)   , 1
3 
(D)  { }
1
3
,1

(A)  0 (B)  1 Solution: We know


(C)  2 (D)  Infinitely many
p
Solution: sin-1 x + cos -1 x =
2
cos-1 x = -(p + cos-1 2x)
p p
Range of cos-1 x ∈ [0, p] Þ = 2 cos -1 x + - cos -1(3 x - 2)
2 2
Since cos-1 x has a range from [0, p], thus the sum of two cos-1 cos-1(3x - 2) = 2 cos-1x
cannot be equal to -p a negative quantity. cos-1(3x - 2) = cos-1(2x2 - 1)
Hence, the correct answer is option (A).
3x - 2 = 2x2 - 1
2p
2. Number of pairs (x, y) satisfying sin-1 x + sin-1 y = and 2x2 - 3x + 1 = 0
3
p (2x - 1) (x - 1) = 0
is
cos -1 x - cos -1 y =
3 1
⇒x=
(A) 0 (B)  1 2
(C) 2 (D)  None of these ⇒x=1

Mathematical Problem Book for JEE.indb 714 07-06-2018 12:32:15


Chapter 17 | Inverse Trigonometry 715

Thus, only at
1
2 { }
, 1 above expression has a solution 2t 2 - p t -
3p 2
8
=0

Hence, the correct answer is option (A). ⇒ 16t2 - 8pt - 3p2 = 0


æ 1ö æ 1ö æ 1ö p ⇒ (4t + p)(4t - 3p) = 0
4. If cot -1 ç ÷ + cot -1 ç ÷ + cot -1 ç ÷ = , then
èaø èbø ècø 2 -p 3p
Þt = ,t =
(A)  a + b + c = abc (B)  ab + bc + ca = 1 4 4
(C)  ab + bc + ca = abc (D)  None of these So,
-p 3p
Solution: We know, tan-1 x = , tan-1 x =
4 4
1
tan-1 a = cot -1 (a > 0 ) x = -1, x = -1
a
Thus, x = -1.
p Hence, the correct answer is option (A).
Þ tan-1 a + tan-1 b + tan-1 c =
2
p
æ a + b + c - abc ö p 7. If sin-1 6 x + sin-1 6 3 x = - , then x is equal to
-1
tan ç ÷= 2
è 1- ab - bc - ca ø 2
1
⇒ 1 - ab - bc - ca = 0 (A)  -1 (B) 
12
ab + bc + ca = 1
Hence, the correct answer is option (B). 1
(C)  - (D)  None of these
12
5. 3sin-1 x = sin-1(3x - 4x3) holds good for all
(A)  |x| ≤ 1 (B) 0 ≤ x ≤ 1 Solution: See Fig. 17.9. Given,
(C) |x| ≤ 1/2 (D)  None of these p
sin-1 6 x + sin-1 6 3 x = -
Solution: We know, 2
é -p p ù -p p
sin-1( z ) Î ê , ú £ sin-1 x £
ë 2 2û 2 2
So, for Thus sum of two angles
3sin-1( x ) = sin-1(3x - 4x3) a, b can only be negative
-p p
£ sin-1 ( 3 x - 4 x 3 ) £ when both the angles
2 2 are negative but angles
-p p are negative only when
Þ £ 3 sin-1 x £ x is negative or equal
2 2
negative.
-p p Putting x = -1, we have
£ sin-1 x £
6 6
sin-1( -6) + sin-1( -6 3 ).
é 1 1ù 1
Þ x Î ê- , ú ⇒| x | ≤ Since sin-1 x has x∈[-1, 0] Figure 17.9
ë 2 2 û 2
so x = -1 cannot be a
Hence, the correct answer is option (C). solution.
5p 2 1
6. If (tan-1 x )2 + (cos -1 x )2 = , then x equals Now, putting x = - ,
8 12
(A)  -1 (B) 1 æ
éæ -1 ö ù æ 1 öö
(C) 0 (D)  None of these sin-1 êç ÷ ú + sin-1 ç 6 3 ç - ÷ ÷
12
ëè ø û ç è 12 ø ÷ø
è
Solution: We know,
p  1  3
tan-1 x + cot -1 x = = sin-1  -  + sin-1  - 
2  2  2 
So,
p p p
æp ö 5p
2 2 =- - =-
(tan x ) + ç - tan-1 x ÷ =
-1 2 6 3 2
è2 ø 8
So,
p2 5p 2 1
-1
2(tan x ) + - p tan-1 x =
2
x=-
4 8 12
Let tan-1 x = t. Then Hence, the correct answer is option (C).

Mathematical Problem Book for JEE.indb 715 07-06-2018 12:32:35


716 Mathematics Problem Book for JEE

8. sin-1 x > cos-1x hold for p


æ 1 ö and tan-1(1) =
(A)  All values of x (B)  x Î ç 0 , ÷   4
è 2ø So,
 1  p p 3p
(C)  x ∈ ,1 (D)  x = 0.75 + =
 2  4 2 4

Solution: See Fig. 17.10. Hence, the correct answer is option (C).
y
p Previous Years' Solved JEE Main/AIEEE
cos x
−1
Questions
p /2
sin−1x æxö æ5ö p
1. If sin-1 ç ÷ + cosec -1 ç ÷ = , then a value of x is
è5ø è4ø 2
x
(1/ 2,0) (1, 0) (A) 1 (B) 3
(C) 4 (D) 5 [AIEEE 2007]
−p /2
Solution: We have
x 4 p x 4 x 3
sin-1 + sin-1 = Þ sin-1 = cos -1 Þ sin-1 = sin-1
Figure 17.10 5 5 2 5 5 5 5
Therefore, x = 3.
-p p
0 £ cos -1 x £ p ; £ sin-1 x £ Hence, the correct answer is option (B).
2 2
æ 5 2ö
 1  2. The value of cot ç cosec -1 + tan-1 ÷ is
Clearly, sin-1 x > cos -1 x ∀x ∈ ,1 . è 3 3ø
 2 
6 3
Hence, the correct answer is option (C). (A) (B)
  17   17
æ -1 ö æ 1ö
9. The value of tan-1 + cos -1 ç ÷ + sin-1 ç - ÷ is 4 5
è 2ø è 2ø (C) (D)  [AIEEE 2008]
  17   17
p 5p
(A) (B) 
æ 5 2ö
  4 12 Solution: Let us consider that, E = cot ç cosec -1 + tan-1 ÷ .
è 3 3ø
3p 11p
(C)  (D) 
4 12 Therefore,
é æ 3 2 öù
Solution: é -1 æ 3 ö -1 æ 2 ö ù
ê -1 ç 4 + 3 ÷ ú
E = cot ê tan ç ÷ + tan ç ÷ ú = cot ê tan ç ÷ú
-p p -p
< tan-1 x < ;
p
£ sin-1 x £ ; 0 £ cos -1 x £ p ë è4ø è 3 øû ê çç 1- 3 × 2 ÷÷ ú
2 2 2 2 êë è 4 3 ø úû
p æ 17 ö 6
tan-1(1) = (1) = cot ç tan-1 ÷ =
4 è 6 ø 17
æ 1 ö 2p Hence, the correct answer is option (A).
cos -1 ç - ÷ = (2)
è 2ø 3
3. Statement I: The equation (sin-1x)3 +(cos-1x)3 - ap 3 = 0 has a
æ 1 ö -p
-1 1
sin ç - ÷ = (3) solution for all a ³ .
è 2ø 6 32
Thus, Eqs. (1) + (2) + (3) gives p
Statement II: For any x ∈ R, sin-1 x + cos -1 x = and
2
 1  1 p 2p p
tan-1 (1) + cos -1  -  + sin-1  -  = + - æ
2
p ö 9p 2
 2  2 4 3 6 0 £ ç sin-1 x - ÷ £ .
è 4ø 16
3p + 8p - 2p 9p 3p
= = = (A)  Both statements I and II are true.
12 12 4
(B)  Both statements I and II are false.
Alternative Solution: Since (C)  Statement I is true and statement II is false.
p (D)  Statement I is false and statement II is true.
sin-1 x + cos -1 x =
2 [JEE MAIN 2014 (ONLINE SET-3)]

Mathematical Problem Book for JEE.indb 716 07-06-2018 12:32:55


Chapter 17 | Inverse Trigonometry 717

Solution: é æ -p ö ù -p
Therefore, tan-1 ê tan ç ÷ú = .
(sin-1 x)3 + (cos-1 x)3 - ap 3 =0 ë è 4 øû 4
Þ (sin x + cos x )[(sin x ) - sin x cos x + (cos -1 x )2 ] = ap 3
-1 -1 -1 2 -1 -1
Hence, the correct answer is option (C).
p
Þ [(sin-1 x + cos -1 x )2 - 3 sin-1 x cos -1 x ] = a p 3p 2 æ 2x ö 1
2 5. Let tan-1 y = tan-1 x + tan-1 ç 2 ÷
, where x < . Then a
è 1 - x ø 3
2
æp ö æp ö value of y is
Þ ç ÷ - 3 sin-1 x ç - sin-1 x ÷ = 2ap 2
è2ø è2 ø 3x + x 3 3x - x 3
(A)  (B) 
p 2
1- 3 x 2 1+ 3 x 2
- 2ap 2
4 p
Þ = (sin-1 x ) - (sin-1 x )2 3x + x 3 3x - x 3
3 2 (C)  (D) 
1+ 3 x 2 1- 3 x 2
p 2 - 8ap 2 p
Þ = (sin-1 x ) - (sin-1x)2
12 2 [JEE MAIN 2015 (OFFLINE)]

p -1
2
æ p ö æ p ö p - 8ap
2 2 2
Solution: Since,
Þ (sin-1 x )2 - sin x + ç ÷ = ç ÷ -
2 è4ø è4ø 12
æ x+y ö
æ
2
p ö p 2 p 2 8 ap 2 32ap 2 - p 2
2
tan-1 x + tan-1 y = tan-1 ç ÷ for xy < 1
Þ ç sin-1 x - ÷ = - + = è 1- xy ø
è 4 ø 16 12 12 3 48
Now,
Now æ 2x ö 2x
2
æ 1- x 2 - 1 ö 2
x ×ç 2 ÷
= = -2 ç 2 ÷
= -2 +
è 1- x ø 1- x è 1- x ø
2
p p -p p p p p 1- x 2
- £ sin-1 x £ Þ - £ sin-1 x - £ -
2 2 2 4 4 2 4 Further
1 1 1 2
-3p p p x < Þ 0 £ x 2 < Þ - < - x 2 £ 0 Þ < 1- x 2 £ 1
or £ sin-1 x - £ 3 3 3 3
4 4 4
1 3 2 2
æ p ö 9p 2
2
Þ 1£ < Þ2£ < 3 Þ 0 £ -2 + <1
Þ 0 £ ç sin-1 x - ÷ £ 1- x 2 2 1- x 2 1- x 2
è 4ø 16
æ 2x ö
Therefore, Þ x ×ç 2 ÷
Î (0 ,1)
è 1- x ø
32ap 2 - p 2 9p 2 32a - 1 9 Hence,
0£ £ Þ0£ £
48 16 48 16 æ 2x ö
1 28 æ 2x ö çx+ ÷
⇒ 0 ≤ 32a - 1 ≤ 27 ⇒ 1 £ 32a £ 28 Þ é 1 7ù -1 -1
tan ( x ) + tan ç -1
= tan ç 1 - x2 ÷
£a£ Þ aÎ ê , ú 2 ÷ 2
32 32 ë 32 8 û è 1- x ø ç 1- 2 x ÷
ç ÷
1 28 é 1 7 ù è 1- x 2 ø
1 £ 32a £ 28 Þ £a£ Þ aÎ ê , ú
32 32 ë 32 8 û
æ 3x - x3 ö
= tan-1 ç 2 ÷
= tan-1( y ) (given)
Therefore, Statement I is false and II is true. è 1- 3 x ø
Hence, the correct answer is option (D).
æ 3x - x 3 ö
æ 43p ö Þ y =ç 2 ÷
4. The principal value of tan-1 ç cot ÷ is è 1- 3 x ø
è 4 ø
Hence, the correct answer is option (D).
3p 3p
(A)  - (B) 
4 4 æ 2x ö
6. If f ( x ) = 2 tan-1 x + sin-1 ç 2 ÷
, x > 1, then f(5) is equal to
p p è 1+ x ø
(C)  - (D) 
4 4 p
(A)  (B)  π
2
[JEE MAIN 2014 (ONLINE SET-4)] æ 65 ö
Solution: (C)  4 tan−1(5) (D)  tan-1 ç ÷
è 156 ø
43p æ 44p p ö æ pö æ pö [JEE MAIN 2015 (ONLINE SET-1)]
cot = cot ç - ÷ = cot ç 11p - ÷ = cot ç p - ÷
4 è 4 4ø è 4ø è 4ø
Solution:
3p æp p ö p æ -p ö æ 2x ö
= cot = cot ç + ÷ = - tan = tan ç ÷ f ( x ) = 2 tan-1 x + sin-1 ç , x > 1, f(5) = ?
4 2 ÷
è2 4ø 4 è 4 ø è 1+ x ø

Mathematical Problem Book for JEE.indb 717 07-06-2018 12:33:23


718 Mathematics Problem Book for JEE

We know that The slope of normal is −2 and the equation of normal is


ì -1 æ 2 x ö p æ pö
ï sin ç 1+ x 2 ÷ ; 1£ x £ 1 y- = -2 ç x - ÷
ï è ø 3 è 6ø
ï æ 2x ö p
2 tan-1 x = í-p - sin-1 ç 2 ÷
; x < -1 = -2 x +
ï è 1+ x ø 3
ï æ 2x ö Therefore,
ï p - sin-1 ç 2 ÷
; x >1
ïî è 1+ x ø 2p
y = -2 x +
⇒ f(x) = 2 tan−1x + (π - 2 tan−1x) ⇒ f(x) = π ∀ x > 1 ⇒ f(5) = π 3
Hence, the correct answer is option (B). 2p
2x + y =
3
æ 1+ sin x ö æ pö
7. Consider f ( x ) = tan-1 çç ÷÷ , x Î ç 0 , ÷ . A normal to x y
è 1- sin x ø è 2ø Þ + =1
p / 3 2p / 3
p æ 2p ö
y = f(x) at x = also passes through the point So, the normal passes through the point ç 0 ,
6 ÷.
è 3 ø
æp ö Hence, the correct answer is option (C).
(A)  ç , 0 ÷ (B)  (0, 0)
è4 ø

æ 2p ö
(C)  ç 0 , ÷
æp ö
(D)  ç , 0 ÷
Previous Years' Solved JEE Advanced/
è 3 ø è6 ø IIT-JEE Questions
[JEE MAIN 2016 (OFFLINE)]
1. Let (x, y) be such that
Solution: We have p
sin-1(ax ) + cos -1( y ) + cos -1(bxy ) =
æ 1+ sin x ö æ pö 2
f ( x ) = tan-1 çç ÷÷ , x Î ç 0 , ÷
è 1- sin x ø è 2ø Match the statements in Column I with statements in Column II.

 sin2 ( x / 2) + cos2 ( x / 2) + 2 sin( x / 2)cos( x / 2)  Column I Column II


f ( x ) = tan-1  
 sin ( x / 2) + cos ( x / 2) + 2 sin( x / 2)cos( x / 2) 
2 2
(A) If a = 1 and b = 0, then (x, y) (P) lies on the circle x2 + y2 = 1

æ [cos( x / 2) + sin( x / 2)]2 ö (B) If a = 1 and b = 1, then (x, y) (Q) lies on (x2 - 1)(y2 - 1) = 0
f ( x ) = tan-1 ç ÷
ç [cos( x / 2) - sin( x / 2)]2 ÷ (C) If a = 1 and b = 2, then (x, y) (R) lies on y = x
è ø
æ | cos( x / 2) + sin( x / 2)| ö (D) If a = 2 and b = 2, then (x, y) (S) lies on (4x2 - 1)(y2 - 1) = 0
æ pö
f ( x ) = tan-1 ç ÷ , x Î ç 0,, ÷
è | cos( x / 2 ) - sin( x / 2 )| ø è 2ø [IIT-JEE 2007]
é cos( x / 2) + sin( x / 2) ù Solution: If a = 1 and b = 0, then
f ( x ) = tan-1 ê ú
ë cos( x / 2) - sin( x / 2) û p p
sin-1 x + cos -1 y + =
é1+ tan( x / 2) ù 2 2
f ( x ) = tan-1 ê ú
ë 1- tan( x / 2) û Þ sin-1 x = - cos -1 y
é æ p x öù Þ sin-1 x = - sin-1 1- y 2
f ( x ) = tan-1 ê tan ç + ÷ ú
ë è 4 2 øû Þ - x = 1- y 2

p x
f (x) = + Þ x2 + y2 =1
4 2
(A) → (P)
æ p ö p x 3p + p p
fç ÷= + = = If a = 1 and b = 1, then
è 6 ø 4 12 12 3
æp x ö p
The point is ç , ÷ , Therefore, sin-1 x + cos -1 y + cos -1 xy =
2
è6 3ø
cos -1 x - cos -1 y = cos -1 xy
1
f ’(x) =
2 Þ xy + 1- y 2 1- x 2 = xy

p  1 Þ ( x 2 - 1)( y 2 - 1) = 0
⇒ f’   =
 6 2 (B) → (Q)

Mathematical Problem Book for JEE.indb 718 07-06-2018 12:33:52


é 23 æ n
öù é 23 ù
cot ê å cot -1 ç 1+ å 2k ÷ ú = cot ê å cot -1(1+ n(n + 1))ú
ëChapter
n=1 è17 |k =Inverse
1 ë n=1
ø û Trigonometry û
719
23
= cot å (tan-1(n + 1) - tan-1 n)
n=1
If a = 1 and b = 2, then = cot(tan-1 24 - tan-1 1)
sin-1 x + cos-1 y + cos-1(2xy) = -cos-1 x - cos-1 y = cos-1(2xy) é æ 23 ö ù
= cot ê tan-1 ç ÷ ú
Þ xy + 1- x 2 1- y 2 = 2 xy ë è 25 ø û
25
Þ 1- x 2 1- y 2 = xy =
23
⇒ 1 - x2 - y2 + x2 y2 = x2 y2
Hence, the correct answer is option (B).
⇒ x2 + y2 = 1
(C)→(P) 4. Match List I to List II.
If a = 2 and b = 2, we get List I List II
-1 p -1 -1
sin 2 x + cos y + cos (2 xy ) = 3.
2 P. Let y(x) = cos (3cos−1 x), x ∈ [−1, 1], x ± 1. 1
2
cos-12x - cos-1 y = cos-1(2xy)
1 ì 2 d 2 y( x ) dy ( x ) ü
2 xy + 1- 4 x 2 1- y 2 = 2 xy Then í( x - 1) 2
+x ý equals
y( x ) î dx dx þ
⇒ (4 x2 - 1)(y2 - 1) = 0
Q. Let A1, AT , …, An (n > 2) be the vertices of a regular 2. 2
(D)→(S) polygon of n sides with its centre at the origin.
Hence, the correct matches are (A)→(P); (B)→(Q); (C)→(P); 
Let ak be the position vector of the point Ak,
(D)→(S).

2. If 0 < x < 1, then 1+ x 2 {[ x cos(cot -1 x ) + sin(cot -1 x )]2 - 1}1/2 is


k = 1, 2,  , n. If å n-1
k =1 (ak ´ ak -1 ) = å n-1
(ak × ak +1 ) ,
k =1

equal to then the minimum value of n is


x R. If the normal from the point P(h, 1) on the ellipse 3. 8
(A)  (B)  x
1+ x 2 x2 x2
+ = 1 is perpendicular to the line x + y = 8,
6 3
(C)  x 1+ x 1+ x 2
2
(D)  then the value of h is
Solution: We have S. Number of positive solutions satisfying the equation 4. 9
æ 1 ö -1 æ 1 ö -1 æ 2 ö
1+ x 2 {[ x cos(cot -1 x ) + sin(cot -1 x )]2 - 1}1/2 tan-1 ç ÷ + tan ç ÷ = tan ç 2 ÷ is
è 2 x + 1 ø è 4 x + 1 ø èx ø
éæ 1 1 ö
2
ù
1+ x êç x coscos
2 -1
+ sinsin-1
÷ - 1ú
êëè 1+ x 2 1+ x 2 ø úû P Q R S
1
(A) 4 3 2 1
é 2
ù2 (B) 2 4 3 1
2 æ x2 1 ö
1+ x çê + ÷ - 1ú (C) 4 3 1 2
êëè 1+ x 2 1+ x 2 ø úû (D) 2 4 1 3

1
[JEE ADVANCED 2014]
1+ x 2 [( x 2 + 1)2 - 1]2
Solution: For (P) in List I:
= x 1+ x 2
y(x) = cos(3cos-1 x)
Hence, the correct answer is option (C).
Hence,
é 23 æ n
öù dy æ -1 ö 3 sin(cos -1 x )
3. The value of cot ê å cot -1 ç 1+ å 2k ÷ ú is = -{sin(3 cos -1 x )}3 ç ÷=
ë n=1 è k =1 ø û dx è 1- x ø
2
1- x 2
23 25 2
(A)  (B)   dy 
{ ( )}
2

25 23 =  1- x 2  = sin 3 cos -1 x
 dx 
23 24
(C)  (D)   [JEE ADVANCED 2013] æ dy ö
2

24 23 Þ (1- x 2 ) ç ÷ = 9 sin2 (3 cos -1 x )


è dx ø
Solution: We have = 9[1- cos2 (3 cos -1 x )]
é 23 æ n
öù é 23 ù = 9(1- y 2 )
cot ê å cot -1 ç 1+ å 2k ÷ ú = cot ê å cot -1(1+ n(n + 1))ú
ë n=1 è k =1 ø û ë n=1 û Now differentiating
23 2
= cot å (tan-1(n + 1) - tan-1 n) dy d 2 y æ dy ö ì dy ü
(1- x 2 )2 + ç ÷ ( -2 x ) = 9 í-2 y ý

n=1 dx dx 2 è dx ø î dx þ
= cot(tan-1 24 - tan-1 1)
é æ 23 ö ù
= cot ê tan-1 ç ÷ ú
ë è 25 ø û
25
=
Mathematical Problem Book for JEE.indb 719
23 07-06-2018 12:34:17
720 Mathematics Problem Book for JEE

d2 y dy 6x 3y
Þ (1- x 2 ) -x = -9 y - = 3 (1)
dx 2 dx cosq sinq
d2 y dy Since,
Þ ( x 2 - 1) +x = 9y
dx 2 dx
ax by
Therefore, - = a2 - b 2
cosq sinq
(P)→(4)
For (Q) in List I: Therefore, slope of this normal is
n-1 æ 6 ö æ- 3ö 6 sinq
       = çç - ÷÷ çç ÷÷ = = 2 tanq
∑ (a
k =1
k × ak +1 ) = a1 × a2 + a2 × a3 +  + an-1 × an
è cosq ø è sinq ø 3 cosq

Since (h, 1) lies on this normal,


Therefore,

6h 3l
- = 3 (2)
cosq sinq
Now this normal is ^ to x + y = 8, Hence, its slope is
an a3 -1
=1
-1
a2
a1
(See Fig. 17.13.) Therefore,
1
2 tan q = 1 ⇒ tan q =
2 2
n So,
Figure 17.11
2 1
cosq = , sinq =
2p 3 3
= (a + a +  + a )sin
2 2 2

n
So, Eq. (2) becomes
2p
= (n - 1)a2 sin
n  (1)
6h 3  3
6 ×3
 - =3⇒ h=6
(See Fig. 17.11.) Since all a1 are equal 2 1  2 
3 3
n-1
  2p
Also ∑ (a
k =1
k ⋅ ak +1 ) = ( a + a  + a )cos
2 2 2

n 6h 3  3
6 ×3
Figure 17.13
- =3⇒ h=6
2p 2 1  2 
= (n - 1)a2 cos (2) 3
n 3

From Eqs. (1) and (2), Therefore,
2p 2p h=2
sin = cos Hence,
n n
2p 2p p (R) → (2)
⇒ tan = 1⇒ = ⇒n=8
n n 4 For (S) in List I:
Therefore,
(Q) → (3) æ 1 ö -1 æ 1 ö -1 æ 2 ö
tan-1 ç ÷ + tan ç ÷ = tan ç 2 ÷
For (R) in List I: è 2 x + 1ø è 4 x + 1ø èx ø
(See Fig. 17.12.) Equation of normal at ( 6 cosq , 3 sinq ) is ì 1 1 ü
ïï 2 x + 1 + 4 x + 1 ïï
-1 -1 æ 2 ö
y Þ tan í
1 ý = tan ç 2 ÷
ï 1- ï èx ø
P(h,1) ïî (2 x + 1)( 4 x + 1) ïþ
ì 4 x + 1+ 2 x + 1 ü
x ïï ïï æ 2 ö
Þ tan í 8 2x + 6 x + 1 ý = tan-1 ç 2 ÷
2
-1

ï 8 x + 6 x + 1 - 1 ï èx ø
ïî 8 x 2 + 6 x + 1 ïþ
æ 6x + 2 ö -1 2
Þ tan-1 ç 2 ÷ = tan 2
Figure 17.12 è 8x + 6x ø x

Mathematical Problem Book for JEE.indb 720 07-06-2018 12:34:36


Chapter 17 | Inverse Trigonometry 721

Therefore, 3. sin-1 sin(16) is equal to


6x3 + 2x2 = 16x2 + 12x (A) 5π - 16 (B) 16 - 5π
⇒ 2x (3x2 + x - 8x - 6) = 0 (C) 6π - 16 (D)  None of these
⇒ x (3x2 - 7x - 6) = 0 4. If q = cot-1 7 + cot-1 8 + cot-1 18, then cot θ is equal to
(A)  2 (B) 3
⇒ x (3x2 - 9x + 2x - 6) = 0
(C)  4 (D)  None of these
⇒ x [3x (x - 3) + 2(x - 3)] = 0
é æ 1 öù
⇒ x (x - 3) (3x + 2) = 0 5.   cos ê2 tan-1 ç ÷ ú equals
ë è 7 øû
Therefore,
2 (A)  sin(4 cot-13) (B) 
sin(3 cot-14)
x = 0, 3, - -1
(C)  cos(3 cot 4) (D) cos(4 cot-13)
3
Hence, number of +ve solutions = 1 6. The value of tan-1{2sin[sec-1(2)]} is
Therefore, p p
(A)  (B) 
(S) → (1) 6 4
p p
Hence, the correct answer is option (A). (C)  (D) 
3 2
æ6ö æ4ö
5. If a = 3 sin-1 ç ÷ and b = 3 cos -1 ç ÷ , where the inverse
è 11 ø è9ø 7. The value of sin [sin-1( 5 / 4 ) + tan-1( 5 / 11)] is
trigonometric functions take only the principal values, then 5 4
the correct option(s) is(are) (A)  (B) 
4 11 35
(A) cos β > 0 (B)  sin β < 0
55
(C)  cos (α + β) > 0 (D) cos α < 0 (C)  (D)  None of these
8
[JEE ADVANCED 2015]
-1 -1 -1
Solution: 8. The x satisfying sin x + sin (1- x ) = cos x are
(A)  1, 0 (B) 1, -1
æ6ö æ 6 ö p
a = 3 sin-1 ç ÷ > 3 sin-1 ç ÷ = 1
è 11 ø è 12 ø 2 (C) 0, (D)  None of these
2
p 9. The equation sin-1 x = 2 sin-1 a holds true for
Þ <a <p (1)
2  (A)  -1 ≤ a ≤ 1 (B)  a ≥ 0
-1 æ 4 ö æ 1ö 1 1
b = 3 cos ç ÷ > 3 cos -1 ç ÷ = p (C)  - £a£ (D)  All real values of a
è9ø è2ø 2 2
Also
ïì 1 é æ 9p ö æ 9p ö ù ïü
 4  3p 10. The principal value of cos -1 í êcos ç 10 ÷ - sin ç ÷ ú ý is
p < 3 cos -1   < ïî 2 ë è ø è 10 ø û ïþ
 9 2
3p 3p 7p
Þp < b < (2) (A)  (B) 
2 20 20
From Eqs. (1) and (2) 7p
(C)  (D 
None of these
cos α < 0, cos β < 0, sin β < 0, 10
Also, 11. The number of positive integral solutions of the equation
3p 5p æ y ö æ 3 ö
< (a + b ) < tan-1 x + cos -1 ç ÷ = sin-1 ç ÷ is
2 2 ç 1+ y 2 ÷ è 10 ø
è ø
⇒ cos (α + β) > 0
(A) 1 (B)  2
Hence, the correct answer is option (C). (C)  0 (D)  None of these
1 1
Practice Exercise 1 12. If A = tan-1 , B = tan-1 , then
7 3
1. The value of tan(tan-11 + tan-1 2 + tan-1 3) = (A)  cos 2A = sin 2A (B)  cos 2A = sin 2B
(A) 0 (B)  tan 1 (C)  cos 2A = cos 2B (D)  cos 2A = sin 4B
1 1 -1
13. The equation 2 cos x = sin (2 x 1- x ) is valid for all values
1 2
(C)   tan (D) tan
3 6 of x satisfying
2. cos-1(cos x) = p + x, then x belongs to (A)  -1 ≤ x ≤ 1 (B) 0 ≤ x ≤ 1
(A)  (0, π) (B) (π, 2π) 1 1
(C)  0 ≤ x ≤ (D)  ≤ x ≤1
(C) [0, π] (D)  None of these 2 2

Mathematical Problem Book for JEE.indb 721 07-06-2018 12:34:54


722 Mathematics Problem Book for JEE

14. The value of the expression é -1 æ 4 ö ù


æ 2ö æ 5ö æ 10 ö 23. Value of cos ê2 cos ç ÷ ú equals
tan-1 çç -1
÷÷ + sin çç
-1
÷÷ - cos çç ÷÷ is ë è 5 øû
è 2 ø è 5 ø è 10 ø 6 7
(A)  (B) 
æ 1+ 2 ö æ 2 + 1ö 25 25
(A)  cot -1 çç ÷÷ cot -1 çç
(B)  ÷÷ 4 8
è 1- 2 ø è 2 - 1ø (C)  (D) 
25 25
æ 1+ 2 ö æ 1- 2 ö
(C)  -p + cot -1 çç ÷÷ (D)  p - cot -1 çç ÷÷ é -1 æ 15p ö ù
è 1- 2 ø è 1+ 2 ø 24. The value of cos ê tan ç tan ÷ is
ë è 4 ø úû
é æ -17p ö ù 1 1
15.   cos -1 êcos ç ÷ ú is equal to
ë è 5 øû (A)  -
(B) 
2 2
17p 3p (C)  1 (D)  None of these
(A)  - (B) 
5 5
25. Number of solutions to the equation
2p 2 tan-1(cos x) = tan-1(2cosec x) in the interval (-2π, 2π) is
(C)  (D)  None of these
5 (A)  4 (B)  3
16. If 0 ≤ x ≤ 1 and q = sin-1 x + cos-1 x - tan-1 x, then (C)  7 (D)  No solution
p p 26. Number of solutions to the equation
(A)  q £ (B)  q³
2 4 æ 3ö
sin-1 x - cos -1 x = cos -1 çç ÷÷ is
p p p è 2 ø
(C)  q = (D) £q £
4   4 2 (A)  One (B)  Two
æ3ö (C)  Four (D)  None of these
17. If  tan-1 x = 2 cot -1 7 + cos -1 ç ÷ , then x =
è5ø 27. The number of real solutions of the equation
117 117 p
(A)  (B)  tan-1 x ( x + 1) + sin-1 x 2 + x + 1 = is
44 125 2
(A)  Zero (B) One
7 117
(C)  (D)  (C)  Two (D) Infinite
24 84
2n 2n

18. If tan -1 1+ x 2 - 1- x 2
= b , then x =
28. If å sin
i =1
-1
x i = np , then åx
i =1
i is
1+ x + 1- x
2 2
(A)  n (B) 2n
1- tan b n(n +1)
(A) tan β (B)  ± (C)  (D)  None of these
1+ tan b 2

(C)  ± sin2 b (D) 


± cos2 b 29. The value of sin-1{sin[2 cot + -1 ( 2 - 1)]} is
x -1 x +1 p p 3p
19. If  tan-1 + tan-1 = , then x = (A)  - (B) 
x -2 x +2 4 4 4
1 7p
(A) 3 (B)  (C)  (D)  None of these
3 4
1
(C)  ± (D)  None of these -1
2 ìï é p 1 æ a öù ép 1 æ a ö ù üï
30. The value of ítan ê + sin-1 ç ÷ ú + tan ê - sin-1 ç ÷ ú ý ,
20. Let f(x) = sec-1 x + tan-1 x. Then f(x) is real for îï ë 4 2 è b øû ë4 2 è b ø û þï
(A)  x ∈ [-1, 1] (B)  x ∈ R where 0 < a < b is
(C)  x ∈ (-∞, 0) (D) (-∞, -1] ∪ [1, ∞) b a
(A) (B) 
21. If cos-1 x - sin-1 x = 0, then x is equal to
  2a 2a
1
(A)  ± (B)  1 b 2 - a2 b 2 - a2
2 (C)  (D) 
2b 2a
1
(C)  (D) 
2 31. If 4 cos-1 x + sin-1 x = p, then x equals to
2
1 1
10 10 (A)  (B) 
22. If å cos-1 x i = 0, then
i =1
å x i is
i=1
2 2

(A) 0 (B) 10 3
(C) 1 (D) 
(C) 5 (D)  None of these 2

Mathematical Problem Book for JEE.indb 722 07-06-2018 12:35:26


Chapter 17 | Inverse Trigonometry 723

æ sin1- 1 ö ìï é æ ïì é æ 3p ö ù ïü ö ù üï
32.  tan-1 ç ÷ equals 42. Value of sin-1 ísin êcos -1 ç cos ísin-1 êsin ç
ç ÷ ú ý ÷ ú ý equals
è cos 1 ø îï êë è ïî ë è 4 ø û ïþ ÷ø úû þï
p
(A) 0 (B)  1- 3p 5p
2 (A)  (B) 
4 4
p 1 p
(C)  - 1 (D)  - p
2 2 4 (C)  (D)  None of these
4
2x
33. If x ≥ 1, then 2 tan-1 x + sin-1 is equal to 43. tan-1(tan4) + cot-1(cot4) equals
1+ x 2
(A) 8 (B)  π - 8
(A) 4tan-1x (B)  π (C) 8 - 2π (D) 0
(C) 0 (D)  None of these
34. The inequality log2 x < sin-1(sin5) hold if p
44. If tan-1( x ) + tan-1 y + tan-1 z = and
2
(A)  x ∈ (0, 25-2p) (B)  x ∈ (25-2p, ∞)
p - 5
(x - y)2 + (y - z)2 + (z - x)2 = 0, then x 2 + y 2 + z 2 equals
(C)  x ∈ (22 , ∞) (D)  None of these
(A) 0 (B) 4
35. If a £ tan-1 x + cot -1 x + sin-1 x £ b "x Î (0 ,1), then (C)  1 (D)  None of these
p 45. The greatest of tan1, tan-11, sin1, sin-11 is
(A)  a = 0 , b = (B)  α = 0, β = π
2 (A)  tan1 (B)  tan-11
p (C) sin1 (D)  sin-1 1
(C)  a = -1, b = p (D) a = ,b =p
2 ænö p
46. If cot -1 ç ÷ > , n being a natural number, then maximum
9p 1 1 1 èp ø 6
36. If 2 sin-1 x + 3 sin-1 y + 4 sin-1 z =
then + + equals
2 2x 3y 4z value of n is
11 13 (A) 1 (B)  5
(A)  (B)  (C)  9 (D)  None of these
12 12
47. Two angles of a triangle are cot -12 and cot -13. Then the third
15 17
(C)  (D)  angle is
12 12
p 3p
(A)  (B) 
é 4 4
æ4ö æ 2 öù
37.  tan-1 êcos -1 ç ÷ + sin-1 ç ÷ ú is
ë è 5 ø è 3 øû p p
(C)  (D) 
17 17 6 3
(A)  (B) 
6 16 2p ö 2p ö
æ -1 æ
6 48. The value of cos -1 ç cos ÷ - sin ç sin ÷ is equal to
(C)  (D)  None of these è 3 ø è 3 ø
17
(A) 0 (B)  4p
38. If sin-1 x + sin-1 y + sin-1 z = p , then 3

x 1- x 2 + y 1- y 2 + z 1- z 2 equals 4p p
(C)  - 3 (D) 
3
(A)  xyz (B) 1
(C)  0 (D) 2xyz
49. If in ∆  ABC, ∠A = sin-1(x), ∠B = sin-1(y) and ∠C = sin-1(z), then
1- sin x + 1+ sin x x 1- y 2 1- z 2 + y 1- x 2 1- z 2 + z 1- x 2 1- y 2 is equal to
39. The value of cot - 1 if 0° < x < 90° is
1- sin x - 1+ sin x
(A)  xyz (B)  x + y + z
x
(A)  x (B)  1 1 1
2 (C)  + + (D)  None of these
x y z
x x
(C)  p - -
(D) 
2 2 æ x 3 ö -1 æ 2 x - k ö
50. If A = tan-1 çç ÷÷ and B = tan ç ÷ , then the value of
40. The value of + tan2(sec-1 2)
is cot2(cosec-1 3) è 2 k - x ø è k 3 ø
(A)  13 (B) 15 A - B (independent of x) is
(C) 12 (D) 11 p p
(A)  (B) 
41. The principal value of sin-1(sin10) is 2 3
(A)  10 (B) 10 - 3π p p
(C)  (D) 
(C) 3π - 10 (D)  None of these 6 8

Mathematical Problem Book for JEE.indb 723 07-06-2018 12:35:54


724 Mathematics Problem Book for JEE

Practice Exercise 2 (C)  -π - 2sin-1(x) for -1 £ x £ -


1
2
Single/Multiple Correct Choice Type Questions 1
(D) 2cos-1(x) for ë xÅ 1
1. If y = cot-12 + cot-18 + cot-118 + … + ∞, then tan y is 2
p p
(A)  (B) 
2 3 Matrix Match Type Questions
p
(C)  (D)  None of these 8. Match the following:
4
æ x ö æ y ö Column I Column II
2. If x, y > 0, then the range of sin-1 ç 2 ÷
+ sin-1 ç 2 ÷
is
è 1+ x ø è 1+ y ø p
 2 cos x - 3 sin x  (i)
(A) If y = cos -1   , then
(A) [0, π] (B) (-π, π]  13 14

é 2p ù æ 2p ù dy
ç 0, ú
(C)  ê0 , ú (D)  is
ë 3 û è 3 û dx

3. For 0 < θ < 2θ ; sin-1sinθ > cos-1sinθ is true when -1 é æ 46p ö ù p
(B) cos êsin ç (ii)
÷ ú is 2
ë è 7 øû
æp ö æ 3p ö
çp ,
(A)  ç , p ÷ (B)  ÷
è4 ø è 4 ø
(C) If |x| ≥ 1, a > 0 and sum of series æ1 ö
æ p 3p ö æ 3p ö x  sec -1 | x | + cosec -1 | x | 
n (iii) ç ,¥ ÷
(C)  ç , ÷ (D)  , 2p ÷ è2 ø
è4 4 ø
ç
è 4 ø ∑ 
n=1  ap


4. For which value of x, sin[cot-1(x + 1)] = cos(tan-1x) is is finite, then value of a is
1 (iv) 1
(A)  (B) 0  1
2 cosec -1 x + cos -1  
(D) Let f(x) =  x  Then
1 .
(C) 1 (D)  - cosec x
2
the greatest value is
5. T he number of integral values of k for which the equation
sin-1x + tan-1x = sin-1sinx + 2k - 1 has a real solution is 9. Match the following:
(A)  1 (B) 3 Column I Column II
(C)  2 (D) 4
æ 1ö æ 1ö æ 1ö (i) p
(A) tan-1 ç ÷ + tan-1 ç ÷ + tan-1 ç ÷ +  + ¥
è3ø è7ø è 13 ø 2
1
6. Let f(x) = and its anti-derivative
4 - 3 cos2 x + 5 sin2 x p
-1 æ 12 ö -1 æ 4 ö -1 æ 63 ö
1 (B) sin ç 13 ÷ + cos ç 5 ÷ + tan ç 16 ÷ (ii)
F(x) = tan-1[ g( x )] + c. Then è ø è ø è ø 4
3
æp ö -1 æ 4 ö -1 æ 1 ö
(A)  g(x) is equal to 3 tanx (B)  g ç 4 ÷ is equal to 3 (C) sin ç ÷ + 2 tan ç ÷ (iii) p
è ø è5ø è3ø
æp ö æp ö p
(C)  g’ ç 3 ÷ is equal to 6 (D)  g’ ç 3 ÷ is equal to 12 -1 -1
æ 41 ö
(iv)
è ø è ø (D) cot 9 + cosec çç ÷÷ 3
è 4 ø
7.  sin-1(2 x 1- x 2 ) for -1 ≤ x ≤ 1 is equal to
10. Match the following:
1
(A)  π - 2sin-1(x) for ≤ x ≤1 Column I Column II
2
é 7 - 5( x 2 + 3) ù p
1 1 (A) The maximum value of sec -1 ê ú is (i) 6
(B) 2sin-1(x) for - £x£ ë 2( x + 2) û
2

2 2

Mathematical Problem Book for JEE.indb 724 07-06-2018 12:36:17


Chapter 17 | Inverse Trigonometry 725

Column I Column II Column I Column II


(B) The minimum value of 1 1
é 5ù é 1ù (ii) (D) If y = tan-1 æç ö÷ + tan-1 ( b ) , (0 < b < 1) and
cosec -1 ê3 x 2 + ú + sec -1 ê3 x 2 + ú (where [.] 3 è2ø
ë 4û ë 4û (iv) 2
p
denotes the greatest integer function) is 0 < y ≤ 4 , then the maximum value of b will be

(C) The number of solution of equation 2p


(iii) Integer Type Question
p 3
sin-1(| x 2 - 1|) + cos -1(| 2 x 2 - 5 |) = is
2 11. The value of 2cot (cot-13 + cot-17 + cot-113 + cot-121) is
_________.

Answer Key
Practice Exercise 1
 1. (A)  2. (D)  3. (A)  4. (B)  5. (A)
 6. (C)  7. (C)  8. (C)  9. (C) 10. (D)
11. (B) 12. (D) 13. (D) 14. (C) 15. (B)
16. (D) 17. (A) 18. (C) 19. (C) 20. (D)
21. (C) 22. (B) 23. (B) 24. (A) 25. (A)
26. (A) 27. (C) 28. (B) 29. (D) 30. (C)
31. (D) 32. (D) 33. (B) 34. (A) 35. (B)
36. (B) 37. (A) 38. (D) 39. (C) 40. (D)
41. (C) 42. (C) 43. (C) 44. (C) 45. (D)
46. (B) 47. (B) 48. (D) 49. (A) 50. (C)

Practice Exercise 2

1. (C) 2. (D) 3. (C) 4. (D)  5. (D)


6.  (A), (B), (D) 7.  (A), (B), (C) 8. (A) →(iv); (A) →(ii);
9.  10. (A) →(iii);
(B) →(i); (C)→(iii); (B) →(iii); (C)→(i); (B) →(i); (C)→(iv);
(D) →(ii) (D) →(ii) (D) →(ii)
11. 3

Solutions
Practice Exercise 1 = tan[tan-1(1) + tan-1( -1)]

ép p ù
= tan ê - ú = tan (0) = 0
æ x+y ö ë4 4û
1. Hint: tan-1 x + tan-1 y = tan-1 ç ÷
è 1- xy ø 2. Since 0 ≤ cos-1(x) ≤ p and given
é æ 2 + 3 öù cos-1(cos x) = p + x
tan ê tan-1(1) + tan-1 ç ÷ú
ë è 1- 6 ø û is true only when x belongs to some negative angle but no
option is such.
é æ 5 öù 11p
= tan ê tan-1(1) + tan-1 ç ÷ ú 3. Since 5p < 16 < ,
ë è -5 ø û 2

Mathematical Problem Book for JEE.indb 725 07-06-2018 12:36:24


726 Mathematics Problem Book for JEE

p p
Þ 0 < 16 - 5p < 8. Hint: cos -1 x = - sin-1 x
2 2
p p
Þ- < 5p - 16 < 0 sin-1 x + sin-1(1- x ) = - sin-1 x
2 2
Now, p
2 sin-1 x + sin-1(1- x ) =
sin-1(sin16) = sin-1[sin(5p - 16)] 2
       = 5π - 16 Clearly x = 0 satisfies equation
4. Given p p
0+ =
q = cot-1 7 + cot-18 + cot-118 2 2

55 1
= cot -1 + cot -1 18 x= also satisfies the equation
15 2
æ 11 ö p p p
ç 3 ´ 18 - 1 ÷ 2´ + =
= cot ç ÷
-1
6 6 2
çç 11 + 8 ÷÷ 1ö
è 3 ø æ
Þ ç x = 0, ÷
-1 è 2ø
= cot  (3)
⇒ cot θ = 3 9. Hint: 2 sin-1 x = sin-1(2 x 1- x 2 )
5. We have
é æ 1 öù æ 7 ö sin-1 x = sin-1(2a 1- a2 ) when
cos ê2 tan-1 ç ÷ ú = cos ç tan-1 ÷ -1 ≤ x ≤ 1
ë è 7 ø û è 24 ø
æ 24 ö 24 -1 £ 2a 1- a2 £ 1
= cos ç cos -1 ÷ =
è 25 ø 25 0 £ 4 a2 (1- a2 ) £ 1
æ 1ö 1
sin( 4 cot -1 3) = sin ç 4 tan-1 ÷ 0 £ a2 (1- a2 ) £
è 3ø 4

æ 3ö æ 24 ö 1
= sin ç 2 tan-1 ÷ = sin ç tan-1 ÷ a2 (1- a2 ) £
è 4ø è 7 ø 4

æ 24 ö 24 1
= sin ç sin-1 ÷ = Þ a2 £
25 ø 25 2
è
1 1
æ 1ö Þ- £a£+
Hence, cos ç 2 tan-1 ÷ = sin( 4 cot -1 3). 2 2
è 7ø
10. Hint: cos-1[cos(-x)] = p - x
6. See Fig. 17.14.
æ pö ïì 1 é æ p ö æ p ö ù ïü
tan-1 ç 2 sin ÷ cos -1 í êcos ç p - 10 ÷ - sin ç p - 10 ÷ ú ý
è 3ø 2 îï 2 ë è ø è ø û þï
3
æ
-1 3ö -1 é 1 æ p p öù
tan çç 2 ´ ÷ = tan ( 3 ) = cos -1 ê ç - cos - sin ÷ ú
è 2 ÷ø ë 2è 10 10 ø û
/3
p é æ 1 p 1 p öù
= 1 = cos -1 ê( -1) ç cos + sin ÷ ú
3 ë è 2 10 2 10 ø û
Figure 17.14
é æ p p p p öù
= cos -1 ê( -1) ç cos cos + sin × sin ÷ ú
ë è 4 10 4 10 ø û
7. Hint: ( 5 )2 + ( 11)2 = 4 2
é æ p p öù
See Fig. 17.15. = cos -1 ê( -1)cos ç - ÷ ú
ë è 4 10 ø û
sin [α + α] = sin 2α
= 2 sin α cos α æ 3p ö 3p 17p
= p - cos -1 ç cos ÷ = p - =
è 20 ø 20 20
5 11 55
= 2⋅ ⋅ = Figure 17.15 Hence, none of the given alternatives are correct.
4 4 8

Mathematical Problem Book for JEE.indb 726 07-06-2018 12:36:45


Chapter 17 | Inverse Trigonometry 727

x+y æ 1+ 2 ö
11. Hint: tan-1 x + tan-1 y = tan-1 = -p + cos -1 çç ÷÷
1- xy
è 1- 2 ø
æ y ö æ 3 ö
tan x + cos ç
-1 -1
÷ = sin-1 ç ÷ 15. Hint: cos (2π - q) = cos q
ç 1+ y 2 ÷ è 10 ø
è ø
é æ 17p ö ù
cos -1 êcos ç - ÷ú
Þ tan-1 x + tan-1 y = tan-1 3 ë è 5 øû
x+y
Þ tan-1 = tan-1 3   3p    3p  3p
1- xy = cos -1 cos  4p -   = cos -1  cos  =
  5    5  5

x+y
Þ =3
1- xy p
16. Hint: sin-1 x + cos -1 x =
⇒ x + y = 3 - 3xy 2
⇒ x + y + 3xy = 3 q = sin-1 x + cos-1x - tan-1x
Hence, only integral solution possible is (3, 0) and (0, 3). p
= - tan-1 x
2
1- tan2 q 2 tanq
12. Hint: cos 2q = , sin 2q = Given,
1+ tan2 q 1+ tan2 q
p p p p
0 ≤ x ≤ 1 Þ 0 £ tan-1 x £ Þ ³ - tan-1 x ³
-11 1 1 1 4 2 2 4
A = tan and B = tan-1 Þ tan A = and tan B =
 7 3 7 3 p p p p
Þ 0 £ tan-1 x £ Þ ³ - tan-1 x ³
1 4 2 2 4
1-             
1- tan2 A 49 48 24
cos 2 A = = = =
1+ tan2 A 1+ 1 50 25 a+b
17. Hint: tan-1 a + tan-1 b = tan-1
49 1- ab
1 2 1 4
2´ tan-1 x = 2 tan-1 + tan-1
2 tan B 3 6 3
sin 2B = = = 3 = = 7 3
1+ tan2 B 1+ 1 10 10 5
9 9 æ 49 - 1 ö -1 4
tan-1 x = cot -1 ç ÷ + tan
3 4 24 è 14 ø 3
sin 4 B = 2 sin 2B × cos 2B = 2 × × =
5 5 25 æ 48 ö 4
= cot -1 ç ÷ + tan-1
Hence, cos2A = sin4B. è 14 ø 3
7 4
13. Let x = cos A , x Î [ -1,1] and A Î [0 , p ] . Then 7 -1 4 -1 24
+
= tan-1
+ tan = tan 3
24 3 7 4
2 cos -1 ( cos A ) = sin-1(2 cos A 1- cos2 A ) 1- ×
24 3
⇒ 2 cos (cos A) = sin (2 cos A . sin A) = sin-1(sin2 A)
-1 -1
7 4 39
+
117
Þ 2 A = sin-1(sin 2 A) = tan 24 3 = tan 24 = tan-1
-1 -1
7 11 44
1-
Now, left hand and right hand will be equal for 18 18

é pù é pù é 1 ù 117
2 A Î [0 , 2p ] Þ 2 A Î ê0 , ú Þ A Î ê0 , ú Þ x Î ê ,1ú Þ tan-1 x = tan-1
ë 2û ë 4û ë 2 û 44

14. We have a c a+b c +d


18. Hint: If = then =
æ 2ö æ 5ö æ 10 ö b d a-b c -d
tan-1 çç -1
÷÷ + sin çç
-1
÷÷ - cos çç ÷÷
è 2 ø è 5 ø è 10 ø 1+ x 2 - 1- x 2 1+ x 2 - 1- x 2
tan-1 = b Þ tan b =
æ 1 ö -1 æ 1 ö 1+ x 2 + 1- x 2 1+ x 2 + 1- x 2
= tan-1 ç -1
÷ + tan ç ÷ - tan (3)
è 2ø è2ø 1+ tan b 1+ x 2
Þ =
æ 2+ 2 ö 1- tan b 1- x 2
= tan-1 çç -1
÷÷ - tan (3)
è 2 2 - 1 ø
Squaring both sides, we get
æ 1- 2 ö 1+ tan2 b + 2 tan b 1+ x 2
= tan-1 çç ÷÷ =
è 1+ 2 ø 1+ tan2 b - 2 tan b 1- x 2

Mathematical Problem Book for JEE.indb 727 07-06-2018 12:37:14


728 Mathematics Problem Book for JEE

Using componendo and dividendo, we get é æ p öù é æ p öù p


= cos ê - tan ç tan ÷ ú = cos ê tan-1 ç tan ÷ ú = cos
  x 2 = sin 2 b ë è 4 øû ë è 4 øû 4
Þ x = ± sin2 b 1
=
2
a+b
19. Hint: tan-1 a + tan-1 b = tan-1
1- ab 2x
25. Hint: 2 tan-1 x = tan-1
x -1 x +1 1- x 2
+
p 2tan-1(cos x) = tan-1(2cosec x)
tan-1 x - 2 x + 2 =
x -1 x +1 4 2 cos x
1- × Þ tan-1 = tan-1 2 cosec x
x -2 x +2 1- cos2 x
Hence, ⇒ tan-1 2cot x · cosec x = tan-1 2cosec x
p x2 + x - 2 + x2 - x - 2
tan = cot x · cosec x – cosec x = 0 ⇒ cosec x (cot x - 1) = 0
4 x2 - 4 - x2 +1 ⇒ cosec x = 0 which is not possible, and
2x2 - 4 cot x = 1 which has 4 solutions.
1=
-3
26. Hint: sin-1 x = cos -1 1- x 2
2x2 1
= 3
2 2 sin-1 x - cos -1 x = cos -1
2
1
x =±
2 3 ìï 3 1 üï
Þ sin-1 x = cos -1 x + cos -1 = cos -1 í x × - 1- x 2 ý
2 îï 2 2 þï
20. Hint: Value of sec x and cosec x does not lie between +1 and -1.
If φ (x) = sec-1 x, then we know that x∈ (-∞, -1] ∪ [1, ∞). ìï 3 x 1 üï
Þ cos -1 1- x 2 = cos -1 í - 1- x 2 ý
Also g(x) = tan-1 x, then x ∈ R. ïî 2 2 ïþ
Hence, for holding f(x) = sec-1 x + tan-1 x, we have
x ∈ (-∞, -1] ∪ [1, ∞) 3x 1
Þ 1- x 2 = - 1- x 2
p p 1 2 2
21. Hint: sin = cos =
4 4 2 3 3x
Þ 1- x 2 =
cos-1 x - sin-1 x = 0 2 2
p Squaring both sides, we have
⇒ cos -1 x - + cos -1 x = 0
2 9 3x 2 3
(1- x 2 ) = ⇒ x =±
p 4 4 2
Þ 2 cos -1 x =
2
Hence, 3
-1 p 1 Only x = satisfies the equation.
cos x = Þ x = 2
4 2
1- x 2
22. Since, cos-1 x2 has to be zero separately, only the 27. Hint: sin-1 x = cot -1
10
x
å cos
i =1
-1
xi = 0
tan-1 x (x + 1) + sin-1 x 2 + x + 1 =
p
2
That is,
p
cos-1 x1 = 0, cos-1 x2 = 0 … cos-1 x10 = 0 Þ sin-1 x 2 + x + 1 = - tan-1 x (x + 1) = cot -1 x (x + 1)
2
⇒ x1 = 1  x2 = 1 … x10 = 1
1
Adding all the terms, we get Þ sin-1 x 2 + x + 1 = sin-1
10
x + x +1
2

åx i = 10
1
i =1
Þ x2 + x +1 = Þ x 2 + x + 1= 1
23. We have x2 + x +1
é ⇒ x(x + 1) = 0, x = 0, x = -1
æ 4ö æ 16 ö ù 7
cos ç 2 cos -1 ÷ = cos êcos -1 ç 2 ´ - 1÷ ú = Both the values satisfies the equation, so there are two
è 5ø ë è 25 ø û 25
solutions.
24. Hint: tan (2mπ - θ) = -tan θ, cot(-θ) = cos θ
28. Given
ìï é æ p ö ù üï ìï é æ p ö ù üï
cos ítan-1 ê tan ç 4p - ÷ ú ý = cos ítan-1 ê - tan ç ÷ ú ý sin-1 x1 + sin-1 x2 + … + sin-1 x2n = np
îï ë è 4 ø û þï îï ë è 4 ø û þï
which is possible only if

Mathematical Problem Book for JEE.indb 728 07-06-2018 12:37:39


éæ 1 1ö ù
2
é 1 1ù
ê ç cos - sin ÷ ú ê cos 2 - sin 2 ú
Chapter -17 2
1 ê è| Inverse 2ø ú
Trigonometry -1 729
= tan = - tan ê ú
êæ 2 1 2 1öú ê cos 1 + sin 1 ú
ê ç cos - sin ÷ ú ë 2 2û
ëè 2 2 øû
p é 1ù
sin-1 x1 = sin-1 x 2 =  = sin-1 x 2 n =
2 ê 1- tan 2 ú
-1
= - tan ê ú
⇒ x1 = x2 = … = x2n = 1 ê1+ tan 1 ú
ë 2û
Therefore, x1 + x2 + … + x2n = 2n.
é p 1 ù
29. We have ê tan 4 - tan 2 ú
-1 -1 é æ p 1 öù
= - tan ê ú = - tan ê tan ç - ÷ ú
sin-1{sin[2 cot -1( 2 - 1)]} = sin-1{sin[cot-1(-1)]} p
ê1+ tan × tan ú1 ë è 4 2 øû
ë 4 2û
æ 3p ö
= sin-1 ç sin ÷ æp 1ö 1 p
è 4 ø = -ç - ÷ = -
è 4 2ø 2 4
é æ
-1 p öù p
= sin êsin ç p - ÷ ú = 2x
ë è 4 øû 4 33. Hint: 2 tan-1 x = p - sin-1 , when x ≥ 1
1+ x 2
-1
é æp ö æp öù 1 æaö With the given condition
30. We have ê tan ç + a ÷ + tan ç - a ÷ ú , where a = sin-1 ç ÷ 2x
ë è 4 ø è 4 ø û 2 èbø 2 tan-1 x = p - sin-1
1+ x 2
-1
æ 1+ tana 1- tana ö Hence,
=ç + ÷
è 1- tana 1+ tana ø 2x 2x 2x
-1 2 tan-1 x + sin-1 = p - sin-1 + sin-1 =p
é 2(1+ tan2 a ) ù 1+ x 2 1+ x 2 1+ x 2
=ê ú
ë 1- tan a û
2
34. Hint: sin-1(sin5) = sin-1[sin(-2p + 5)]
-1
æ 2 ö 2 cos 2a log2 x < sin-1(sin5)
=ç ÷ =
è cos 2a ø 2 ⇒ log2 x < sin-1[sin(-2p + 5)]
⇒ log2 x < -2p + 5 = 5 - 2p
1 a
Now, a = sin-1 ⇒ x < 25-2p
2 b
Also, x ≠ 0 is positive.
a
Þ = sin2a Therefore, required value of x belongs to x ∈(0 , 25-2p ).
b
a2 b 2 - a2 æ p öü
35. Hint: tan-1 x Î ç 0 , ÷ ï
Þ cos2a = 1- 2
= è 4øï
b b
æ p p öï
b 2 - a2 cot -1 x Î ç , ÷ ý "x Î (0 ,1)
Therefore, given expression is . è 2 4 øï
2b
æ pöï
sin-1 x Î ç 0 , ÷ ï
p è 2 ø ïþ
31. Hint: sin-1 x + cos -1 x =
2
Let tan-1 x + cot -1 x + sin-1 x = z .
p
4 cos -1 x + - cos -1 x = p p p
2 At x = 0 , z = but x ≠ 0, thus a ¹ .
2 2
p p At x = 1,
Þ 3 cos -1 x = Þ cos -1 x =
2 6 z=π⇒β=π
p 3 From option, it is clear that z ≥ 0
Þ x = cos =
6 2 α = 0, β = π

A Aö
2 -p p
æ 36. Hint: £ sin-1 p £
32. Hint: 1- sin A = ç sin - cos ÷ 2 2
è 2 2 ø
æp ö
æ sin1- 1 ö -1 æ 1- sin1 ö -1 æ 1- sin1 ö 2(sin-1 x ) + 3(sin-1 y ) + 4(sin-1 z ) = 9 × ç ÷
tan-1 ç ÷ = tan ç - ÷ = - tan ç ÷ è2ø
è cos1 ø è cos1 ø è cos1 ø
p
⇒ sin-1 x = sin-1 y = sin-1 z = -
éæ 1 1ö ù
2
é 1 1ù 2
ê ç cos - sin ÷ ú ê cos 2 - sin 2 ú
-1 ê è 2 2ø ú -1 ⇒x=y=z=1
= tan = - tan ê ú
êæ 2 1 2 1öú ê cos 1 + sin 1 ú 1 1 1 1 1 1 13
ê ç cos - sin ÷ ú ë 2 2û Þ + + = + + =
ëè 2 2 øû 2 x 3 y 4 y 2 3 4 12

é 1ù
-1
ê 1- tan 2 ú
= - tan ê ú
ê 1+ tan 1 ú
ë 2û

Mathematical Problem Book for JEE.indb 729 07-06-2018 12:38:05


730 Mathematics Problem Book for JEE

a+b p
37. Hint: tan-1 a + tan-1 b = tan-1 Þ- < 3p - 10 < 0
1- ab 2
Given equation must be as follows: Now,
ì æ4ö æ 2 öü sin-1(sin10 ) = sin-1[sin(3p - 10 )]
tan-1 ícot -1 ç ÷ + sin-1 ç ÷ý
î è3ø è 3 øþ = 3p - 10

  3  2  42. We have
= tan tan-1   + tan-1   
  4   3  ìï é
æ ì é æ 3p ö ù ü ö ù üï
sin-1 ísin êcos -1 ç cos ísin-1 êsin ç ÷ ú ý ÷÷ ú ý
  3 2  ç ë è 4 ø û þ ø úû þï
îï êë è î
 -1  4 + 3    -1  17   17
= tan tan    = tan tan  6   = 6 ì é p öù ü
3 2   æ
  1- ×   = sin-1 ísin êcos -1 ç cos ÷ ú ý
  4 3   î ë è 4 øû þ

38. Given æ pö p
= sin-1 ç sin ÷ =
è 4ø 4
sin-1 x + sin-1 y + sin-1 z = p
43. Since
Let sin-1 x = a ,sin-1 y = b ,sin-1 z = y . Then 3p
p <4<
a+b+g=p 2
Now, p
0 < 4 -p <
2
x 1- x 2 + y 1- y 2 + z 1- z 2
Now,
1
= (sin 2a + sin 2 b + sin 2g ) tan-1(tan 4 ) + cot -1(cot 4 )
2
= tan-1[tan(p + 4 - p )] + cot -1[cot(p + 4 - p )]
1
= ( 4 sina sin b sin g )
2 = tan-1[tan( 4 - p )] + cot -1[cot( 4 - p )]
= 2 sin a sin b sin g =4-π+4-π
= 8 - 2π
= 2 xyz
44. Given
39. We have
p
tan-1 x + tan-1 y + tan-1 z =
1- sin x + 1+ sin x 2
cot -1
1- sin x - 1+ sin x Let tan-1 x = a , tan-1 y = b , tan-1 z = y . Then

æ x x x x ö x = tan α, y = tan β, z = tan g


ç cos 2 - sin 2 + cos 2 + sin 2 ÷ Hence,
cot ç ÷
ç cos x - sin x - cos x - sin x ÷ p
è 2 2 2 2 ø a + b +g =
2
æ -1 xö æp ö
= cot ç - cot ÷ Þ tan(a + b ) = tan ç - g ÷
è 2ø è 2 ø

x tana + tan b 1 x+y 1
=p - Þ = Þ =
2 1- tana tan b tan g 1- xy z

40. We have ⇒ xy + yz = zx = 1
Now,
tan2 ( sex -12) = cot 2 (cosec -1 3)
( x - y )2 + ( y - z )2 + ( z - x )2 = 0
é æ 3 öù é -1 æ 2 2 ö ù
= tan2 ê tan-1 çç Þ x 2 + y 2 + z 2 = xy + yz + zx = 1
÷÷ ú = cot êcot çç 1 ÷÷ ú
2

êë è 1 ø ûú êë è ø úû
45.  tan1, tan-1 1,sin1,sin-1 1
= ( 3 ) + (2 2 ) = 3 + 8 = 11
2 2
p p
tan 57°, ,sin 57°,
41. Since 4 2
7p 3.14 3 3.14
3p < 10 < 1.539 , , ,
2 4 2 2

Mathematical Problem Book for JEE.indb 730 07-06-2018 12:38:30


Chapter 17 | Inverse Trigonometry 731

1.539, 0.78, 0.866, 1.57 Practice Exercise 2


So, sin-1 1 is the greatest value.
1. y = cot-12 + cot-18 + cot-118 + … + ∞
46. Given
nth term = cot-1 2n2
ænö p ⇒ tn = cot-1(2n - 1) - cot-1(2n + 1)
cot -1 ç ÷ >
èp ø 6
æ -1 -1 -1 é (2n - 1)(2n + 1) + 1ù ö
æ nö p ç∵cot (2n - 1) - cot (2n + 1) = cot ê ÷
Þ cot ç cot -1 ÷ < cot ç ë 2n + 1- 2n + 1 úû ÷
è pø 6 ç ÷
è = cot -1 2n2 ø
n
Þ < 3 t1 = cot-11 - cot-13
p
t2 = cot-13 - cot-15
Þ n < 3 × p = 5.5 (nearly) t3 = cot-15 - cot-17
So, the maximum value of n is 5. : : :
⇒ y = cot-11 - cot-1(2n + 1)
47. We have
As n → ∞, cot-1(2n + 1) → 0, so
-1 æ 2 ´ 3 - 1 ö p p
cot-1 2 + cot-1 3 = cot ç ÷= y=
è 2+3 ø 4 4

p 3p 1+ x 2
Hence, third angle = p - = . 2. ³ 2 ( ∵ x > 0)
4 4 x
48. We have
x 1 -1 æ x ö æ pù
⇒ £ ⇒ sin ç Î 0, ú
2 ÷ ç
æ 2p ö -1 æ 2p ö 1+ x 2 2 è 1+ x ø è 6 û
cos -1 ç cos ÷ - sin ç ÷
è 3 ø è 3 ø
æ y ö æ pù
2p p p Similarly, sin-1 ç Î 0, ú .
2 ÷ ç
= - = è 1+ y ø è 2 û
3 3 3
æ 2p ö
49. Given So, range of the given expression is ç 0 , ÷.
è 3 ø
x = sin A, y = sin B, z = sin C
p
Now, 3. sin-1sinθ > - sin-1sinθ
2
sin A cos B cos C + sin B cos A cos C + sin C cos A cos B
p
⇒ sin-1sinθ >
-sin A sin B sin C = sin (A + B + C) = sin np = 0 4
Therefore, given expression = sin A sin B sin C = xyz. Therefore,
50. We have 1 p 3p
sinθ > Þ <q <
2 4 4
tan A - tan B
tan( A - B ) =
1+ tan A tan B é æ 1 öù
4. sin[cot-1(x + 1)] = sin êsin-1 ç ÷ú
x 3 2x - k êë è x + 2 x + 2 ø úû
2
-
2k - x k 3
= 1
x 3 2x - k ⇒ sin[cot-1(x + 1)] =
1+ ´
2k - x k 3 x + 2x + 2
2

3 xk - 4 kx + 2k 2 + 2 x 2 - xk é æ 1 öù 1
= cos(tan-1 x ) = cos êcos -1 ç ÷ú =
2k 2 3 - xk 3 + 2 x 2 3 - kx 3 êë è 1+ x
2
ø úû 1+ x 2

2 x + 2k - 2kx
2 2
1 1 1
= = ⇒ =
3 (2 x 2 + 2k 2 - 2kx ) 3 x 2 + 2 x + 2 1+ x 2

p 1
Hence, A - B = . ⇒x=-
6 2

Mathematical Problem Book for JEE.indb 731 07-06-2018 12:38:49


732 Mathematics Problem Book for JEE

5. sin-1x + tan-1x - sin-1 sin x = 2k - 1 1 (n + 1) - n


Tn = tan-1 = tan-1
é 3p 3p ù 1 + n + n2 1+ (n + 1)n
The range of sin-1x + tan-1x - sin-1sinx is ê - - 1, + 1ú .
ë 4 4 û = tan-1(n + 1) - tan-1n
Then Putting n = 1, 2, 3, …, and adding, we get
3p 3p 3p 3p Sn = tan-1(n + 1) - tan-11
- - 1 £ 2k - 1 £ +1 ⇒ - £k £ +1 p p p p
4 4 8 8 S∞ = tan-1(∞) - = - =
Hence, the integral values of k are -1, 0, 1, 2. 4 2 4 4
(B) Since
1 1
6. F(x) = ò 4 - 3 cos2 x + 5 sin2 x dx = ò 9 - 8 cos2 x dx  12   12  4
sin-1   = tan-1   and cos -1 = tan-1
3
 13   5 5 4
sec2 x sec2 x 1
=ò dx = ò dx = tan-1(3 tan x ) + c
9 sec x - 8
2
1+ 9 tan2 x 3 Therefore,

æp ö æp ö 12 3 63
⇒ g(x) = 3 tanx, g ç ÷ = 3, g’ ç ÷ = 12 LHS = tan-1 + tan-1 + tan-1
4
è ø è3ø 5 4 16
12 3
-1
7. Let y = sin (2 x 1- x )
2
Since × > 1 , we have
5 4
Put x = sinθ 12 3
+
12
-1 -1 3 -1 5 4 = p - tan-1 63
é p pù tan + tan = p + tan
θ ∈ ê- , ú 5 4 12 3 16
ë 2 2û 1- ×
5 4
⇒ y = sin-1(sin2θ) So,
 12 3 63
2 sin-1 x ,  1 1  tan-1 + tan-1 + tan-1 = p
x ∈ - ,  5 4 16
  2 2
 1 2/3 3
 -1  1  (C) 2 tan-1 = tan-1 = tan-1
⇒ y = p - 2 sin x x ∈ ,1
3 1 4
  2  1-
 9
1
 -p - 2 sin-1 x -1 ≤ x ≤ -
 2 and
4 4
sin-1 = tan-1
5 3
æ 2 cos x - 3 sin x ö Therefore,
8. (A)  y = cos -1 ç ÷
è 13 ø 4 1 4 3
sin-1 + 2 tan-1 = tan-1 + tan-1
3 5 3 3 4
y = cos-1 cos (α + x) ∀ α = tan-1
2 4 4 p
= tan-1 + cot -1 =
dy 3 3 2
Therefore, =1.
dx (D) cosec-1x = cot -1 x 2 - 1
-1 æp ö p Hence,
(B) cos cos ç ÷ =
è 14 ø 14
41 41  5
n n
1 cosec-1 = cot -1 - 1 = cot -1  
¥
æ p ö ¥
æ 1 ö 4 16  4
(C) å ç ÷ = å ç ÷ = finite ⇒ a > 2
n=1 è 2ap ø n=1 è 2a ø Therefore,
æ 1ö æ 1ö 41 5
sin-1 ç ÷ + cos -1 ç ÷ cot -1 9 + cos ec -1= cot -1 9 + cot -1
(D) f(x) = èxø è x ø = p sin x 4 4
cosecx 2 1 4
+
-1 1 -1 4 -1 9 5 = tan-1 1 = p
= tan + tan = tan
 1  1  1 9 5 1 4
9. (A) tan-1   + tan-1   + tan-1   +  + ∞ 1- ⋅ 4
 3  7  13  9 5

Mathematical Problem Book for JEE.indb 732 07-06-2018 12:39:14


Chapter 17 | Inverse Trigonometry 733

æ 7 - 5( x 2 + 3) ö -1 é 1 5ù 1
10. (A)  sec -1 ç ÷ = sec ê 2 - ú (D) y = tan-1 + tan-1 b , (0 < b < 1)
è 2( x 2
+ 2 ) ø ë ( x + 2 ) 2 û 2

Since  1/ 2 + b   1 
⇒ y = tan-1  , ∵ b < 1
1 1  1- b / 2  2
£
x +2 2
2
-1  1+ 2b  p
0 < tan  ≤
 2 - b  4
1 5
- ≤ -2
x +2 2
2
 1+ 2b 
⇒0<  ≤1
2p  2 - b 
Therefore, the maximum value is sec-1(-2) = .
3 ⇒ 0 < (1 + 2b) ≤ (2 - b), (1 + 2b > 0)
p 1
(B) Minimum value = cosec-12 + sec-11 = , when ⇒ 3b ≤ 1 ⇒ b ≤
6 3
 2 1 1
⇒ bmax = 
3 x + 4  = 1 3

11.    1  cot -1  11 -1  1-1-1 11  -1  1
++tan + tan -1 1  1-1 1   -1  1  
 1+-1tan
p cot  tan-1 cot +tan
tan-1 tan tan
  + tan +tan-+1 tan
    + tan  
(C) -1 2 -1
sin x - 1 + cos 2 x - 5 =
2
  3  
37   3  7  13   7  13 21  13  21   21 

2
-1-1 x + tan -1 x + y -1-1  x + y - x+y
⇒ |x2 - 1| = |2x2 - 5| Using, tan-Using
1 , tan
x + tan Using tan--11xy+=tan
y = ,tan tan, we
 y= tan 1 ,we get , we get
1 - xy 1-get xy  1- xy 
 
⇒ x2 = 2
  1  ncot -1   1  1  31-1 1   -13 1   3
-1 1 
cot  tan-1 cot +tatan  ta+n-tan =   =
=  + tan
⇒x=± 2 (Two solutions)
  2 28    22  8   2 8   2

Mathematical Problem Book for JEE.indb 733 07-06-2018 12:39:26


734 Mathematics Problem Book for JEE

Solved JEE 2017 Questions


JEE Main 2017 tan a + tan b
because tan(a + b ) =
1- tan a tan b
 1+ x 2 + 1 - x 2  1
1. The value of tan-1   , x < , x ≠ 0 , is equal p q
 2 2 2 ⇒ +
to  1+ x - 1 - x  4 2
p 1 p 1 Substitutingq = cos -1 x 2 , we get
(A) - cos -1( x 2 ) (B) + cos -1( x 2 )
4 2 4 2
p 1
p p + cos -1( x 2 )
(C) - cos -1( x 2 ) (D) + cos -1( x 2 ) 4 2
4 4
(ONLINE) Hence, the correct answer is option (B).

Solution: It is given that 2. A value of x satisfying the equation sin[cot−1(1 + x)] =


 1+ x 2 + 1 - x 2  cos[tan−1x] is
tan-1   1
 1+ x 2 - 1 - x 2  (A) (B) 0
  2
Substituting x2 = cos q, we get 1
(C)
–1 (D) -
2
q = cos−1(x2)
(ONLINE)
2q 2q
Using the identities 1+ cosq = 2 cos and 1- cosq = 2 sin , Solution: We have the following two cases:
we get 2 2
1 1
  From Fig. (a): sin(cot -1(1+ x )) = =
2q q  q q 2 2
 2 cos + 2 sin2   2 cos 2 + 2 sin 2 
1+ 1+ x + 2 x x + 2x + 2
tan-1  2 2  = tan-1  1
 q q  q q -1
From Fig. (b): cos(tan x ) =
2 2  2 cos - 2 sin 
 2 cos 2 - 2 sin 2   2 2 1+ x 2
 
 q q
+ sin 
cos
-1 2 2
= tan 
q q
 cos - sin  1 + (1 + x)2 1 + x2
 2 2
1 x
q
Dividing the equation by cos , we get
2
 sin(q / 2)  l b
 1+ 
tan  cos(q / 2) 
-1 1+x 1
 1- sin(q / 2 )  (a) (b)
 cos(q / 2) 
Therefore, the value of x satisfying the given equation is found as
sin(q / 2) q follows:
Now, substituting = tan , we get
cos(q / 2) 2
sin[cot -1(1+ x )] = cos[tan-1( x )]
1+ tan(q / 2) 
tan-1   = x2 + 2x + 2 = 1 + x2
1- tan(q / 2) 
p That is,
We know that tan = 1; therefore, 1
4 2 x = -1⇒ x = -
2
 p q 
tan-1  tan  +  
  4 2 Hence, the correct answer is option (A).

Mathematical Problem Book for JEE.indb 734 07-06-2018 12:39:45


18 Matrices and
Determinants

Dev Nagri College (DN)—31 players: 2 (TT), 15 (hockey),


MATRICES 3 (badminton), 5 (tennis) and 6 (basketball).
Put this information in a matrix form.
18.1  Definition of a Matrix
Solution: The above information can be put in a tabular form as
A rectangular array of symbols (which could be real or complex
follows.
numbers) along rows and columns is called a matrix.
Thus, a system of m × n symbols arranged in a rectangular for- Colleges Number of players
mation along m rows and n columns and bounded by the brackets TT Hockey Badminton Tennis Basketball
[.] is called an m by n matrix (which is written as m × n matrix).
MC (35) 5 11 5 6 8
Thus,
NAS (22) 3 13 2 4 0
 a11 a12 ............... a1n 
  DN (31) 2 15 3 5 6
a21 a22 ............... a2n 
 
A              The number 4 represents the number of players the NAS College
 
            has sent for playing tennis. The number 15 represents the number
 
am1 am2 ............. amn  of players the DN College has sent for playing hockey. Similarly,
the number 8 represents the number of players that the Meerut
is a matrix of order m × n. In a compact form, the above matrix is College has sent for playing basketball. The above can be put in a
represented by rectangular array form as follows:
A = [aij], 1 ≤ i ≤ m, 1 ≤ j ≤ n or simply [aij]m×n 5 11 5 6 8 
 
3 13 2 4 0 
The numbers a11, a12, … of this rectangular array are called the ele-  
2 15 3 5 6 
ments of the matrix. The element aij belongs to the ith row and the
jth column and is called the (i, j)th element of a matrix. This is a 3 × 5 matrix, where 3 represents the number of colleges
(number of rows) participating and 5 represents the number of
18.2  Order of a Matrix games (number of columns) being played in the meet.

If any matrix A contains ‘m’ rows and ‘n’ columns then m × n is


termed as order of matrix. 18.3  Types of a Matrix
Order is generally written as suffix of the array. The elements that appear in the rectangular array are known as
Now any matrix of order m × n will have the notation [aij]m × n. entries. Depending upon these entries, matrices are of the follow-
That is, ing types:
A = [aij ]m´n or (aij )m´n or aij 1. Row matrix: A single row matrix is called a row matrix or a row
m´n

It is obvious that 1 ≤ i ≤ m and 1 ≤ j ≤ n. vector.


For example, the matrix [a11 a12 … a1n] is a 1 × n row matrix.
Illustration 18.1   In the inter-sports meet of local colleges, the 2. Column matrix: A single column matrix is called a column
games to be played are TT, hockey, badminton, tennis and basket- matrix or a column vector.
ball. Three colleges of Meerut sent the following number of play-  a11 
 
ers. a 
 21 
Meerut College (MC)—35 players: 5 (TT), 11 (hockey), For example, the matrix   is an m × 1 column matrix.
 ... 
5 (badminton), 6 (tennis) and 8 (basketball).  ... 
 
Nanak Chand College (NAS)—22 players: 3 (TT), 13 (hockey), a 
 m 1 
2 (badminton), 4 (tennis) and none for basketball.

Mathematical Problem Book for JEE.indb 735 07-06-2018 12:39:50


736 Mathematics Problem Book for JEE

3. Square matrix: If m = n, i.e. if the number of rows and columns 18.4  Equality of Matrices
of a matrix are equal, say n, then it is called a square matrix of
order n. Two matrices A and B are said to be equal, written as A = B, if
4. Null (or zero) matrix: If all the elements of a matrix are equal to 1. they both are of the same order, i.e. have the same number of
zero, then it is called a null matrix and is denoted by Om×n or O. rows and columns and
5. Diagonal matrix: A square matrix in which all its non-diagonal 2. the elements in the corresponding places of the two matrices
elements are zero is called a diagonal matrix. Thus, in a diago- are the same.
nal matrix aij = 0 if i ≠ j.
The diagonal matrices of orders 2 and 3 are as follows: 18.5 Addition and Subtraction
k 0 0
of Matrices
k1 0   1 
  ,  0 k2 0 Let A = [aij] and B = [bij] be two matrices of the same order m × n.
 0 k2   
 0 0 k3  Then, their sum (or difference) A + B (or A - B) is defined as another
matrix of the same order, say C = [cij] such that any element of C is
The elements aij of a square matrix for which i = j are called the the sum (or difference) of the corresponding elements of A and B.
diagonal elements of a matrix and the diagonal along which all Therefore,
these elements lie is called the principal diagonal or the lead- C = A ± B = [aij ± bij]
ing diagonal or the diagonal of the matrix. 1 2 4 
6. Scalar matrix: A square matrix in which all the diagonal Illustration 18.2  Find A + B and A – B where A    and
0 5 3 
elements are equal and all other elements are equal to zero is 7 3 2 
B 
called a scalar matrix. 5 1 9 
That is, in a scalar matrix aij = k, for i = j and aij = 0 for i ≠ j. Thus,
k 0 0  Solution: Here, both A and B are 2 × 3 matrices. Therefore,
 
0 k 0  is a scalar matrix. 1 7 2  3 4  2  8 5 6 
  AB    
0 0 k  0  5 5  1 3  9  5 6 12 
7. Unit matrix or identity matrix: A square matrix in which all and
its diagonal elements are equal to 1 and all other elements are 1 7 2  3 4  2  6 1 2 
equal to zero is called a unit matrix or an identity matrix, denot- AB    
0  5 5  1 3  9  5 4 6 
ed by U or I.
For example, unit (or identity) matrices of orders 2 and 3 are
1 0 0  18.5.1  Properties of Matrix Addition
1 0   
  and 0 1 0  , respectively. 1.  A + B = B + A
0 1  
0 0 1 2.  A + (B + C) = (A + B) + C
3.  A + O = O + A = A; here O {null matrix} will be additive identity.
8. Negative of a matrix: Let A = [aij]m×n be a matrix. Then, the
4. If A is a given matrix, then the matrix -A is the additive inverse
negative of the matrix A is defined as the matrix [-aij]m×n and
of A for A + (-A) = null matrix O.
is denoted by -A.
5. If A, B and C are three matrices of the same order, then
9. A square matrix in which all elements below leading diagonal A + B = A + C ⇒ B = C   (left cancellation law)
or all elements above leading diagonal are zero is called a tri-
and
angular matrix.
(i) Upper triangular matrix: A square matrix A = [aij] is called an B + A = C + A ⇒ B = C   (right cancellation law)
upper triangular matrix if aij = 0, for all i > j. Thus, in an upper
triangular matrix all elements below diagonals are zero. 18.6 Multiplication of a Matrix
a b c 
  by a Scalar
For example, A  0 p q  is an upper triangular matrix.
  Let A = [aij]m × n be a matrix and k a scalar. Then, the matrix obtained
0 0 r  by multiplying each element of matrix A by k is called the scalar
  (ii) Lower triangular matrix: A square matrix A = [aij] is called multiple of A and is denoted by kA.
a lower triangular matrix if aij = 0 for all i < j. Thus, in a low-
er triangular matrix, all elements above diagonal are zero. 18.6.1 Properties of Multiplication of a Matrix
1 0 0  by a Scalar
 
For example, B  2 3 0  is a lower triangular matrix. 1. If k1 and k2 are scalars and A be a matrix, then (k1 + k2)A = k1A
 
4 5 6  + k2A.

Mathematical Problem Book for JEE.indb 736 07-06-2018 12:39:57


Chapter 18 | Matrices and Determinants 737

2. If k1 and k2 are scalars and A be a matrix, then k1(k2 A) = (k1k2)A.


R1  R1C1 R1C2 
3. If A and B are two matrices of the same order and k is a scalar,    
AB  R2  [C1 C2 ]12  R2C1 R2C2 
then k(A + B) = kA + kB.    
That is, the scalar multiplication of matrices distributes over the R3  R3C1 R3C2 
31 32
addition of matrices. 3.  Few important things for the multiplication
4. If A is any matrix and k be a scalar, then (-k)A = -(kA) = k(-A). (a)  Condition for product AB to exist or to be defined: If A
and B are two matrices then their product is defined or in
18.7  Multiplication of Two Matrices other words A is conformable to B for multiplication if the
number of columns of A is the same as the number of rows
Let A = [aij] be an m × p matrix and B = [bij] be a p × n matrix. in B. That is, if A is a matrix of order m × p and B is a matrix of
These matrices A and B are such that the number of columns of order p × n, the matrix AB will be of order m × n.
A are the same as the number of rows of B, each being equal to (b) Pre-multiplication and post-multiplication: When we
p. Then, the product AB (in the order it is written) will be a ma- say multiply A by B then it could mean both AB or BA where
trix C = [cij] of the type m × n. A and B are any numbers. But when A and B are matrices
Here cij will be the element of C occurring in ith row and the jth then as seen above AB and BA do not necessarily mean the
column, and it will be row by column product of ith row of A having same thing. If AB is defined for matrix multiplication, BA
p columns with the jth column of B having p rows, the elements of may not be defined. To avoid this, when we say product AB
which are it would mean the matrix A post-multiplied by B and when
ai1 ai 2 … aip and b1j we say product BA it would mean matrix A pre-multiplied
b2 j by B. In AB, A is called the pre-factor and B the post-factor.
 (c) In the case when both A and B are square matrices of the
bpj same order then also both AB and BA are defined and the
product matrix is also a matrix of the same order but still
Therefore, AB ≠ BA.
p (d)  Again we know that for two scalars a and b when ab = 0 it
cij = ai1 b1j + ai2 b2j + … + aipbpj = 
k 1
aik bkj
means that either a or b (or both) is zero. But for two matri-
ces A and B, AB = O, i.e. a null matrix, does not necessarily
The summation is to be performed with respect to repeated suffix k. imply that either A or B = O as shown above because nei-
This gives us the particular i-jth element of C which is of order ther A nor B is null matrix whereas AB is a null matrix.
m × n. For obtaining an element of C occurring in the second
0 0  0 0 
row and the third column, we shall put i = 2 and j = 3. Therefore, A , B   
1 0  0 1
p
c23   a2k bk 3  a21b13  a22b23    a2 pbp3 0 0 
k 1 AB   
0 0 
Since there are m rows in A, i can take values from 1 to m. Similarly, But, A ¹ 0 and B ¹ 0
there are n columns in B, j can take values from 1 to n. Thus, we
shall get all the mn elements of C. Again  1 1 1  1 2 3 
   
p Illustration 18.3   If A  3 2 1 , B  2 4 6  , then
cij   aik bkj (18.1)
     
k 1
2 1 0  1 2 3 
compute AB and BA.
This gives us the i-j th element of AB which is of order m × n having
Solution: Here, A is 3 × 3 and B is 3 × 3. Hence, both AB and BA are
m rows and n columns.
defined and each will be 3 × 3 matrix. Let
1. Elements of the jth column of AB: For obtaining elements of
the jth column, j will remain fixed for the jth column whereas i C11 C12 C13 
 
will change from 1 to m as there are m rows in AB. AB  C  C21 C22 C23 
 
Hence, giving i the values 1, 2, 3, …, m and keeping j fixed in C31 C32 C33 
Eq. (18.1) we shall get all the elements of the jth column of AB.
Therefore, the jth column of AB is where Cij means the product of the element at i th row of A with the
element at j th column of B.
p p p
For example, C23 = product of the second row of A with the third

k 1
a1k bkj ,  a2k bkj , … ,  amk bkj
k 1 k 1 column of B. That is,
3 
2. An easy way to remember: If we denote the ordered set of  
rows of A by R1, R2, R3 each having two elements and ordered [  3 2  1] 6   33  2 6  13  0
 
set of columns of B by C1, C2, each having two elements, then 3 

Mathematical Problem Book for JEE.indb 737 07-06-2018 12:40:07


738 Mathematics Problem Book for JEE

Similarly, we can find other elements of C. 2 3 4


We can also say that by the product of the first row of A with the  
5 1 2
three columns of B, we shall get the three elements of the first row A 
6 4 2
of C. That is,  
7 4 6  43
R1C1, R1C2, R1C3
and similarly take the second row of A and multiply with all the and
columns of B and we will get the three elements of the second row 0 0 
 
of C, i.e. R2C1, R2C2, R2C3 and elements of the third row of C will be O  0 0 
 
R3C1, R3C2, R3C3. Therefore, 0 0  32

 11 12  11 12  1 4  12 13  1 6  13 
  Then AO = O.
AB = 31 22  11 32  2 4  12 33  2 6  13 

  If A is an m×n matrix then ImA = A = AIn where Im and In are
5. 
 21 12  0 1 22  1 4  0 2 23  1 6  0 3
identity matrices of order m and n, respectively. If A is a square
0 0 0  matrix of order n and I is the identity matrix of order n, then AI =
 
 0 0 0   O (i.e. null matrix) A = IA. Thus, I is the multiplicative identity.
 
0 0 0  6. For a square matrix A, positive integral powers of A, i.e. An, can
be obtained by multiplying A by itself n times, i.e.
Similarly, BA can also be computed. A2 = A × A
Illustration 18.4 If A and B are matrices such that both AB and A3 = A × A × A = A2 × A
and so on.
A + B are defined, prove that both A and B are square matrices of
the same order. 7. Matrix polynomial: If  f(x) = a xm + a xm-1 + a xm-2 +  …
0 1 2
+ am is a polynomial in x and A is a square matrix of order n,
Solution: We know that two matrices A and B are conformable for then
addition if they are of the same order. Thus, if A be m × n then B f(A) = a0Am + a1Am-1 + a2Am-2 + … + amIn
should also be m × n as A + B is defined.
  is called matrix polynomial. For example, if f(x) = 3x2 - 2x + 5
Again since AB is also defined therefore the number of columns in
and A is a square matrix of third order then matrix polynomial is
A (i.e., n) should be equal to the number of rows in B (i.e., m). Hence,
f(A) = 3A2 - 2A + 5I3.
n = m and in that case both A and B will be the square matrices of
order equal to m = n.
18.8  Operations Regarding Matrices
Illustration 18.5 If A is any m × n matrix and both AB and BA are
defined prove that B should be an n × m matrix. 18.8.1  Transpose of a Matrix
Solution: Since A is m × n and AB is defined, therefore B should If A is a given matrix of the type m × n then the matrix obtained by
be n × p because the number of columns of A should be equal to changing the rows of A into columns and columns of A into rows
number of rows of B. is called transpose of matrix A and is denoted by A′ or AT. As there
Again B is now n × p and A is m × n. are m rows in A, therefore there will be m columns in A′ and similarly
Since BA is also defined, therefore p would be equal to m by the as there are n columns in A, there will be n rows in A′.
same argument as above. Thus if A = [aij]m×n then
Therefore, B is n × m matrix. A′ = AT = [aji]n×m
For example,
18.7.1  Properties of Matrix Multiplication
2 1 
1. 
Multiplication of matrices is distributive with respect to 2 3 7   
If A    , then A  3 0  .
addition of matrices. That is, 1 0 2   
7 2 
A(B + C) = AB + AC
2. 
Matrix multiplication is associative if conformability is assured. Properties of Transpose
That is, 1.  (A′)′ = A
A(BC) = (AB)C 2.  (KA)′ = KA′, with K being a scalar
3. 
The multiplication of matrices is not always commutative. That 3.  (A ± B)′ = A′ ± B′
is, AB is not always equal to BA (AB ≠ BA). 4.  (AB)′ = B′ A′
4. 
Multiplication of a matrix A by a null matrix conformable with 5.  (ABC)′ = C′B′A′
A will give null matrix. Consider

Mathematical Problem Book for JEE.indb 738 07-06-2018 12:40:11


Chapter 18 | Matrices and Determinants 739

18.8.2  Conjugate of a Matrix 4.  For any square matrix A,


Let A = [aij] be a given matrix. Then the matrix obtained by (a)  A + A′ is symmetric.
replacing all the elements by their conjugate complex is called the
(b)  A – A′ is skew-symmetric.
conjugate of matrix A. It is represented by A , i.e. A = [aij ] .
(c)  AA′ and A′A are symmetric matrices.
Properties of Conjugates 5.  If A is a symmetric matrix, then all positive integral powers
1.   ( A) = A of A are symmetric.
2.   ( A + B ) = A + B 6.  If A is a skew-symmetric matrix, then all positive even inte-
3.   (a A) = a A , with a  being any number gral powers of A are symmetric and all positive odd integral
powers of A are skew-symmetric.
4.   ( AB ) = A B , with A and B being conformable for
multiplication. 3.  Hermitian matrix: A square matrix A = [aij] is said to be
Hermitian if the (i, j)th element of A is equal to the conjugate
complex of the (j, i)th element of A, i.e. aij = a ji for all i and j.
18.8.3  Transpose of the Conjugate of a Matrix
4.  Skew-Hermitian matrix: A square matrix A = [aij] is said to
Transpose of the conjugate of a matrix is equal to the conjugate be skew-Hermitian if the (i, j)th element of A is equal to the
of the transpose of a matrix A, i.e. ( A )¢ = ( A¢) and is written as Aθ. negative of the conjugate complex of the (j, i)th element of A, i.e.
aij = - a ji for all i and j.
Properties of Transpose Conjugate
1.  (Aq )q = A
Hermitian and Skew-Hermitian Matrices
2.  (A + B)q = Aq+ Bq A square matrix A = [aij] is said to be Hermitian matrix if aij = a ji
3.  (kA)q= k Aq, k being any number ∀ i, j, i.e. A = Aq.
4.  (AB)q = BqAq 1.  If A is a Hermitian matrix then aii = aii ⇒ aii is real ∀ i. Thus
every diagonal element of a Hermitian matrix must be real.
2.  A Hermitian matrix over the set of real numbers is actually a
18.8.4  Trace of a Matrix real symmetric matrix. A square matrix A = [aij] is said to be
Let A be a square matrix of order n. The sum of the elements skew-Hermitian if aij = - a ji , ∀ i, j, i.e. Aq = -A.
of A lying along the principal diagonal is called the trace of A. 3.  If A is a skew-Hermitian matrix then aii = - aii ⇒ aii + aii = 0,
We shall write the trace of A as tr A. Thus, if A = [aij]n×n, then i.e. aii must be purely imaginary or zero.
n
tr A   ai  a11  a22    ann . 4.  A skew-Hermitian matrix over the set of real numbers is
i 1
actually a real skew-symmetric matrix.

Trace of a Matrix
5.  Orthogonal matrix: A square matrix A is said to be orthogonal
n if A′A = I = AA′.
tr(A) =  aii = a11 + a22 +  + ann
i 1 6.  Unitary matrix: A square matrix A is said to be unitary if AqA
= I = AAq.
7.  Idempotent matrix: A square matrix A such that A2 = A is called
18.9 Types of a Matrix on the an idempotent matrix.
Basis of Operations 8.  Nilpotent matrix: A square matrix A will be called a nilpotent
matrix if Ak = O (null matrix) where k is a positive integer. If how-
1.  Symmetric matrix: A square matrix A = [aij] is said to be ever k is the least positive integer for which Ak = O then k is the
symmetric if its (i, j)th element is the same as its (j, i)th element, index of the nilpotent matrix A.
i.e. aij = aji for all i, j.
9.  Involutory matrix: A square matrix A such that A2 = I is called
2.  Skew-symmetric matrix: A square matrix A = [aij] is said to be the involutory matrix.
skew-symmetric if the (i, j)th element of A is the negative of the
(j, i)th element of A, i.e. if aij = -aji for all i, j.
Your Turn 1
Properties of Symmetric and Skew-Symmetric Matrices
1.  If A is a symmetric matrix, then A′ = A. 1 1 1 1 3 
   
2.  If A is a skew-symmetric matrix, then A′ = –A. If A  2 0 3 and B  0 2  then AB + BA = O.
 1.   
   
3.  Diagonal elements of a skew-symmetric matrix are zero. 3 1 2  1 4 
(True/False)Ans.  False

Mathematical Problem Book for JEE.indb 739 07-06-2018 12:40:22


740 Mathematics Problem Book for JEE

x  3 2 y  x  0 7  18.11  Evaluation of Determinants


If 
 2.    then x = -3, y = -2 and z = 4.
 z  1 4 a  6  3 2a 
(True/False) Ans. True 18.11.1  Determinants of the First Order
1 2 1  If A = [a11], then |A| = a11
 
 3. If A  0 1 1 , then A3 - 3A2 - A + 9I equals  .
  18.11.2  Determinants of the Second Order
 3 1 1  Ans. Zero
a1 b1
The notation consisting of 22 numbers termed as
1 2 2  a2 b2
 
 4. If 3 A  2 1 2  and A × A′ = I, then find x + y.
 elements, arranged in two rows and two columns, is called a
  determinant of second order. The elements a1 and b2 are said to
 x 2 y  Ans.  -3
lie along the principal diagonal; the elements a2 and b1 are said to lie
 5. 
For the three matrices A, B and C, along the secondary diagonal.
The value of the determinant is obtained by forming the
0 1 0  i  1 0  product of the elements along the principal diagonal and
A ,B ,C   ,
1 0  i 0  0 1 subtracting from it the product of the elements along the
secondary diagonal. Thus,
verify the following relations:
(a)  A2 = B2 = C2 = I a1 b1
= a1 b2 - a2 b1 (18.2)
(b) AB = -BA; AC = -CA; BC = -CB a2 b2

  6. Use matrix multiplication to divide Rs. 30000 in two parts such 18.11.3  Determinants of the Third Order
that the total annual interest at 9% on the first part and 11% a1 b1 c1
on the second part amounts Rs. 3060.
Ans.   First part → 12000 The notation a2 b2 c2 consisting of 32 elements, arranged
Second part → 18000 a3 b3 c3
in three rows and three columns, is called a determinant of third
0 a2 ab ac 
b 
c   order. Its value is
   
If A   c
 7.  a  and B  ab b2 bc  , show that
0 a1b2c3 + a2b3c1 + a3b1c2 - a1b3c2 - a2b1c3 - a3b2c1
   
 b  a 0  ac bc c 2 
This may be written as
AB = BA = O3×3.
a1(b2c3 - b3c2) - b1(a2c3 - a3c2) + c1(a2b3 - a3b2)
3 2 3
 
Express the matrix A  4 5 3 as the sum of a symmetric
 8.  b2 c2 a c2 a2 b2
  or a1 - b1 2 + c1
b3 c3 a3 c3 a3 b3
2 4 5
and a skew-symmetric matrix. We can therefore write
 3 3 5 / 2  0 1 1/ 2 
    a1 b1 c1

Ans.  A  3 5 7/2    1 0 1/ 2  b c2 a c2 a b2
    a2 b2 c2 = a1 2 - b1 2 + c1 2 (18.3)
5 / 2 7 / 2 5  1/ 2 1/ 2 0  b3 c3 a3 c3 a3 b3
a3 b3 c3
 9. 
Let A and B be symmetric matrices of the same order. Then,
show that Note that each term of a second-order determinant is the product
of two quantities and each term of a third-order determinant is the
(a)  AB - BA is a skew-symmetric matrix.
product of three quantities.
(b)  AB + BA is a symmetric matrix.

3 4  2 1 2 
18.12 Minors
10.  If A   ,B  , show that (AB)T = BTAT.
The minor of a given element of a determinant is the determinant
1 1  1 3 4 
of the elements which remain after deleting the row and the
column in which the given element occurs.
DETERMINANTS The minor of a1 in Eq. (18.2) is b2 and b2 may be considered a
determinant of first order. Similarly, the minor of a2 is b1.
18.10   Definition of a Determinant For example, the minor of a1 in Eq. (18.3) is
b2 c2
and the
b3 c3
Every square matrix A can be associated to a number or an a1 c1
expression which is known as the determinant of A and is denoted minor of b2 in Eq. (18.3) is a c .
3 3
by |A| or det A.

Mathematical Problem Book for JEE.indb 740 07-06-2018 12:40:37


Chapter 18 | Matrices and Determinants 741

18.13  Cofactors The necessary and sufficient condition for a square matrix A to
possess the inverse is that |A| ≠ 0.
In Eq. (18.3), the elements a1, b1, c1 are multiplied by 1
If A be an invertible matrix, then the inverse of A is adj A. It
-1 | A|
b2 c2 a2 c2 a2 b2 is usual to denote the inverse of A by A .
, - ,
b3 c3 a3 c3 a3 b3
18.15.1  Theorem (Uniqueness of Inverse)
These expressions are called the cofactors of the elements a1, b1, c1.
Theorem: Inverse of a square matrix if it exists is unique.
Generally, the cofactor of an element is its minor with its sign or
Proof: Let A = [aij ]n´n be a square matrix. Let inverse of A exist.
opposite sign prefixed in accordance with the following rule.
To prove: Inverse of A is unique.
For any determinant if aij is the element at the intersection of the
If possible, let B and C be two inverses of A. Then
i th row and j th column, then the cofactor of aij has positive sign or
negative sign before minor of aij according to i + j is even or odd. The AB = BA = In and AC = CA = In
determinant may be expanded along any chosen row or column. Now
The cofactors of the elements a1, b1, c1, a2, b2, c2, a3, b3, c3 will B = B ln = B( AC ) [since AC = In ]
be denoted by A1, B1, C1, A2, B2, C2, A3, B3, C3, respectively.  = (BA) C = InC = C
For example, element b3 in Eq. (18.3) lies at the intersection of
Hence B = C . This implies that the inverse of A is unique.
the third row and the second column. Since 3 + 2 = 5 is an odd
number, we have 18.15.2  Properties of Inverse of a Matrix
a c
B3 = - 1 1 1. (AB)-1 = B-1A-1 2. (A′)-1 = (A-1)′
a2 c2 3. (A-1)q  = (Aq )-1
a1 c1 0 1 2 
The cofactor B2 of the element b2 is + because element  
a3 c3 Illustration 18.7   Find the inverse of the matrix A  1 2 3 .
 
b2 lies at the intersection of the second row and the second col- 3 1 1
umn, and 2 + 2 = 4 is an even number. Solution: We find the determinant of A,
Let the determinant in Eq. (18.3) be denoted by D. When the 0 1 2
cofactors are used, the expansion of the determinant takes the A= 1 2 3
compact form: 3 1 1
D = a1A1 + b1B1 + c1C1 = a2A2 + b2B2 + c2C2 = a3A3 + b3B3 + c3C3 Expanding along R1 we get
D = a1A1 + a2A2 + a3A3 = b1B1 + b2B2 + b3B3 = c1C1 + c2C2 + c3C3 |A| = 0(2 - 3) - 1(1 - 9) + 2(1 - 6) = 8 - 10 = -2
and Since |A| ≠ 0, therefore A-1 exists.
a2A1 + b2B1 + c2C1 = 0 = a2A3 + b2B3 + c2C3, etc.
Now the cofactors of the elements of the first row of |A| are
2 3 1 3 1 2
18.14  Adjoint of a Square Matrix 1 1
,- ,
3 1 3 1
, that is, are -1, 8, -5, respectively.

Let A = [aij]n×n be any n × n matrix. The transpose B′ of the matrix The cofactors of the elements of the second row of |A| are
B = [Cij]n×n, where Cij denotes the cofactor of the element aij in the 1 2 0 2 0 1
determinant |A|, is called the adjoint of the matrix A and is denoted - , ,- , that is, are 1, -6, 3, respectively.
1 1 3 1 3 1
by the symbol adj A.
The cofactors of the elements of the third row of |A| are
a b 
Illustration 18.6  If A =   , then find adj A. 1 2 0 2 0 1
g d  ,- , , that is, are -1, 2, -1, respectively.
2 3 1 3 1 2
Solution: In |A|, the cofactor of a is d and the cofactor of b is -g.
Also the cofactor of g is -b and the cofactor of d is a. Therefore, the Therefore, adj A = the transpose of the matrix B where
matrix B formed of the cofactor of the elements of |A| is 1 8 5
 
d -g  B   1 6 3 
B=  
- b a  1 2 1
So,
d -b  1 1 1
Now, adj A = the transpose of the matrix B =  .  
 -g a  adj A  8 6 2 

 
18.15  Inverse of a Matrix Now
5 3 1

Let A be any n-rowed square matrix. Then, a matrix B, if it exists, 1


A-1 = adj A
such that AB = BA = In, is called inverse of A. | A|

Mathematical Problem Book for JEE.indb 741 07-06-2018 12:40:53


742 Mathematics Problem Book for JEE

Here |A| = –2. Therefore 1 2 6 


 
1 1 1  1/ 2 1/ 2 1/ 2  B  2 3 8 
1     
A1    8 6 2    4 3 1  2 0 10 
2   
5 3 1  5 / 2 3 / 2 1/ 2 
  3. The elementary operation of the addition to the elements
of the i th row, the corresponding elements of the j th row
18.16 Singular and Non-Singular Matrices multiplied by a non-zero number k is denoted by Ri ® Ri + kR j .
A square matrix A is said to be non-singular or singular according Similarly, the elementary operation of the addition to the
elements of the i th column, the corresponding elements of
to |A| ≠ 0 or |A| = 0.
the j th column multiplied by a non-zero number k is denoted
by C i ® C i + kC j .
18.17 Elementary Operations or
Example: Let
Elementary Transformations 1 2 3  1 4 3 
   
of a Matrix A  2 3 4  , B  2 7 4 
   
2 0 5  2 4 5 
Any of the following operations is called an elementary
transformation (operation).
 1.  The interchange of any two rows (or columns). On applying the elementary operation C2 ® C2 + 2C1, matrix
 2. The multiplication of the elements of any row (or column) by A becomes matrix B.
a non-zero number.
18.17.1  Equivalent Matrices
 3. The addition to the elements of any row (or column), the
corresponding elements of any other row (or column) Two matrices A and B are said to be equivalent if one can be
multiplied by a non-zero number. obtained from other by applying a finite number of elementary
Any elementary transformation is called a row transformation operations on the other matrix. If A and B are equivalent matrices,
or column transformation considering as it applies to rows or we write A ~ B .
columns. Example: Let
Clearly, there will be a total of six elementary operations
1 2 3  2 3 8 
(transformations) on a matrix, three of them are due to rows    
and are called row operations whereas three of them are due A  2 3 4  and B  1 2 6 
   
to columns and are called column operations. 
2 0 5  2 0 10 
 1. The elementary operations of interchange of the i th row and
Now,
the j th row is denoted by Ri « R j and the interchange of the
i th column and the j th column is denoted by C i « C j . 1 2 3  é2 3 4 ù
 
Example: Let A  2 3 4  ~ ê 1 2 3 ú [applying R1 « R2 ]
1 2 3    ê ú
  2 0 5  êë2 0 5 úû
A  2 3 4 
  2 3 8 
2 0 5   
~ 1 2 6  = B  [applying C3 ® 2C3 ]
Applying R1 « R3 , i.e. interchanging the first row and the  
2 0 10 
third row, matrix A becomes the matrix
2 0 5  Here, A ~ B as B has been obtained from A by applying two ele-
 
B  2 3 4  mentary operations.
 
1 2 3 
18.17.2  Elementary Matrix
 2. 
The elementary operation of the multiplication of the
elements of the i th row by a non-zero number k is denoted by A matrix obtained from unit matrix by a single elementary
Ri ® kRi . operation is called an elementary matrix.
Similarly, the multiplication of the elements of the i th column Example: Let
by a non-zero number k is denoted by C i ® kC i . 1 0 
Example: Let I 
0 1
1 2 3 
  Then,
A  2 3 4 
  2 0 
2 0 5  A  [R1  2R1]
0 1
On multiplying the elements of the third column of matrix A
by 2, i.e. on applying C3 ® 2C3 , we get the new matrix is an elementary matrix.

Mathematical Problem Book for JEE.indb 742 07-06-2018 12:41:13


Chapter 18 | Matrices and Determinants 743

18.18 Inverse of a Matrix by Elementary 1 0 1 2


  
1 0

Operations (Elementary ⇒ 0 1 2    1
  
0 0 A

( R3 ® R3 + 5R2 )
0 0 2   5 3 1
Operations on Matrix Equation)
1 0 1  2 1 0
Let A, B and X be three square matrices of the same order such that      1 
⇒ 0 1 2    1 0 0 A R3  R3 
     2 
X = AB (18.4) 0 0 1  5 2 3 2 1 2 
The matrix Eq. (18.4) will also be valid if we apply a row operation 1 0 0  1 2  1 2 1 2
   
on matrix X [occurring on the left-hand side of Eq. (18.4)] and the ⇒ 0 1 2    1 0 0 A (R1 ® R1 + R3 )
   
same row operation on matrix A (the first factor of product AB on 0 0 1 5 2  3 2 1 2 
the matrix on the right-hand side). 1 0 0  1 2  1 2 1 2
Thus, on the application of a sequence of row operations on    
⇒ 0 1 0    4 3 1  A (R2 ® R2 - 2R3 )
the matrix equation X = AB (these row operations are applied on X    
0 0 1 5 2  3 2 1 2 
and on the first matrix A of product AB simultaneously), the matrix
equation is still valid (we assume this fact without proof ). Hence,
1 2  1 2 1 2 
Similarly, a sequence of elementary column operations on the  
1
matrix equation X = AB can be applied simultaneously on X and A   4 3 1 
 
on the second matrix B of product AB and the equation will be 5 2 3 2 1 2 
still valid.
We also show the result using column operation. We have
In view of the above-mentioned fact, it is clear that we can find
the inverse of a matrix A, if it exists, by using either a sequence of    A = AI
elementary row operations or a sequence of elementary column 0 1 2  1 0 0
   
operations but not both simultaneously. ⇒ 1 2 3  A 0 1 0 
   
3 1 1 0 0 1
18.18.1  Using Row Operation
1 0 2  01 10 02 0 1 0 
Apply a series of row operations on A = IA till we get I = BA.      
⇒ 2 1 3  A 12
  01 03  A(C11 
 0 C20)  (C1  C2 )
Now by definition of inverse of a matrix, B = A-1 .   
1 3 1 01 03 1 0 0 1
18.18.2  Using Column Operation 1 0 01  0 0  1 0  1 0 
       
Apply a series of column operations on A = AI till we get I = BA. ⇒ 2 1 21 1 A 11 0 A 12  0 (C23 C(3C3 2C1C) 3  2C1)
       
By definition of inverse, B is inverse of A. 1 3 11 3 01 0 01  0 1 
1 0 01 00 0 1 10 1 1 
Illustration 18.8   Obtain the inverse of the matrix using eleme­      
⇒ 2 1 02 A11 0 0 A21 0 (C32  C3 (CC32) C3  C2 )
0 1 2       
  1 3 21 30 2 0 10 0 1 
ntary operations, A  1 2 3 .
  1 0 0  1 0 10  1 2 0 1 1 2 
3 1 1       1  1 
⇒ 2 1 0  2A 1 0  A1  1 0C 3  1  C 3  C 3  C 3 
Solution: We will use row operation first. We have       2  2 
1 3 1  1 03 01  1 2 0 0 1 2 
A = IA  1 0 10  0 02 121 2 1 1 2 
0 1 2  1 0 0       
    ⇒  0 1 00  1 A 01  A0 11 0 (C11  C(1C1  2CC 2 )1
 2C2 )
       
⇒  
1 2 3  0 1
 
0 A
 5 351  3 10 0 01 2 0 1 2 

3 1 1 0 0 1
1 0 10  0 012 11 21 2 1 1 2 
1 2 3 0 1       
   0 ⇒ 0 1 00  1 A 04 A 0 41 0 (1C1  C(1C1 5C3C)1  5C3 )
       
⇒ 0 1 2  = 1 0 0  A ( R1 « R2 ) 0 3 01  3 15 2 05 21 2 0 1 2 
 
3 1 1 0 0 1
 1 10 00 0  121 2  121 21 21 2 
1 2 3  0        
1 0 ⇒ 0 01 10 0AA44 3 3 1 1  (C2(C
    2 
C2 C23C33 C) 3 )
       
⇒ 0 1 2   1 0 0 A ( R3 ® R3 - 3R1) 0 00 01 1  5 25 2 323 21 21 2 
   
0 5 8  0 3 1
Hence,
1 0 1 2 1 0  1 2  1 2 1 2 
     
⇒ 0 1 2    1 0 0  A ( R1 ® R1 - 2R2 ) A1   4 3 1 
     
0 5 8   0 3 1 5 2  3 2 1 2 

Mathematical Problem Book for JEE.indb 743 07-06-2018 12:41:38


744 Mathematics Problem Book for JEE

18.19  Rank of a Matrix 18.21  Homogeneous Linear Equations


A number r is said to be the rank of a matrix A if it possesses the The equations
following two properties: a11x1  a12 x 2    a1n x n  0 

1. There is at least one square submatrix of A of order r whose a21x1  a22 x 2    a2n x n  0 

 (18.5)
determinant is not equal to zero. .............................................. 
2. If the matrix A contains any square submatrix of order r + 1, 
.............................................. 
then the determinant of every square submatrix of A of order 
am1x1  am2 x 2    amn x n  0  
r + 1 should be zero.
In short, the rank of a matrix is the order of any highest order represent a system of m homogeneous equations in n unknowns
non-vanishing minor of the matrix. x1, x2, …, xn. Let
 a11 a12 ... a1n   x1  0 
18.20  Echelon Form of a Matrix 
a21 a22 ... a2n 
  
x 2 
 
0 
     
A matrix A is said to be in Echelon form if either A is null matrix or it a a32 ... a3n  x 3  0 
A   31  , X    ,O   
satisfies the following conditions:  ... ... ... ...   ...  ... 
     
1. Every non-zero row in A precedes every zero row.  ... ... .... ...   ...  ... 
     
2. The number of zeros before the first non-zero element in a row am1 am2 ... amn  mn x n  0  m1
n1
is less than the number of such zeros in the next row.
Also rank of a matrix in Echelon form is equal to the number of where A, X, O are m × n, n × 1, m × 1 matrices, respectively. Then,
non-zero rows of the matrix. obviously we can write the system of equations (18.3) in the form
of a single matrix equation
0 8 12 4 
  AX = O (18.6)
For example, A  0 0 2 5 is in its Echelon form with two The matrix A is called the coefficient matrix of the system of equa-
 
0 0 0 0  tions.
non-zero rows. Therefore rank of A = 2. 1. If | A | = 0 , the system has infinitely many solutions.
To reduce the given matrix A = [aij]m×n in Echelon form, use 2. If A ñ 0, the system has zero solution or trivial solutions.
elementary transformations to make These conclusions can also be written on the basis of the rank
method as follows:
a21, a31, …, am1 = 0
Suppose we have m equations in n unknowns. Then the
Then coefficient matrix A will be of the type m × n. Let r be the rank of
a32, a42, …, am2 = 0 the matrix A. Obviously, r cannot be greater than n (the number of
columns of the matrix A). Therefore, we have either r = n or r < n.
and so on. 1. Case I:     If r = n, the equation AX = O will have n - n, i.e. no
For example, let linearly independent solutions. In this case, the zero
 1 1 1 3  solution will be the only solution. We know that zero
 
A  3 1 2 2  vector forms a linearly dependent set.
  2.  Case II:  If r < n, we shall have n - r linearly independent
2 4 7 7 
solutions. Any linear combination of these n - r
solutions will also be a solution of AX = O. Thus, in this
Operating R2 → R2 - 3R1, R3→R3 - 2R1, we get
case the equation AX = O will have an infinite number
 1 1 1 3  of solutions.
  3.  Case III: Suppose m < n, i.e. the number of solutions is less
A  0 2 5 7  than the number of unknowns. Since r ≤ m, therefore
 
0 2 5 13 r is definitely less than n. Hence, in this case the
given system of equations must possess a non-zero
Operating R3 → R3+ R2
solution. The number of solutions of the equation AX
 1 1 1 3 = O will be infinite.
 
A  0 2 5 7  Illustration 18.9  Does the following system of equations
 
0 0 0 20 
 possess a common non-zero solution?
x + 2y + 3z = 0
which is an Echelon form with 3 non-zero rows. Therefore, rank of A 3x + 4y + 4z = 0
= 3. 7x + 10y + 12z = 0

Mathematical Problem Book for JEE.indb 744 07-06-2018 12:41:45


Chapter 18 | Matrices and Determinants 745

Solution: Determinant of coefficient matrix is |A| = -2 which is system of equations has one or more solutions, the equations are
non-zero. said to be consistent, otherwise they are said to be inconsistent.
Therefore, x = y = z = 0 is the only solution. If B ≠ 0, the system (18.7) is said to be non-homogenous.
Alternate method (Using Rank): The given system of equations adj A
-1
can be written in the form of the single matrix equation as 1.  If |A| ≠ 0 → X = A-1B , where A =
| A|
1 2 3  x  0  The given system has unique solution.
    
AX  3 4 4  y   0   O 2.  If |A| = 0, since AX = B , we have
    

7 10 12  
z  
0  (adj A) AX = (adj A)B ⇒ | A | X = (adj A)B
We shall start reducing the coefficient matrix A to triangular form ⇒ (adj A)B = 0 [since | A | = 0]
by applying only E-row transformations on it. Applying R2 → R2 -
3R1, R3 → R3 - 7R1, the given system of equations is equivalent to which is true for infinite values of X.

1 2 3  x  T herefore, for infinitely many solutions to the system we


   should have
0 2 5  y   O
   (adj A) B = 0
0 4 9  z 
Clearly for no solution we should have
Here, we find that the determinant of the matrix on the left-
hand side of this equation is not equal to zero. Therefore, the (adj A)B ñ 0
rank of this matrix is 3. So, there is no need of further applying T hese conclusions can also be written on the basis of the rank
E-row transformation on the coefficient matrix. The rank of the
method as follows: The matrix
coefficient matrix A is 3, i.e. equal to the number of unknowns.
Therefore, the given system of equations does not possess any  a11 a12 ... a1n b1 
linearly independent solution. The zero solution, i.e. x = y = z = 0 is  
a a22 ... a2n b2 
the only solution of the given system of equations. [ A B ]   21 
 ... ... ... ... ... 
 
18.21.1 Solution of Homogeneous System of am1 am2 ... amn bm 
Linear Equations
is called the augmented matrix of the given system of equations.
Let AX = O be a homogeneous system of n linear equations with n Suppose the coefficient matrix A is of the type m × n, i.e. we
unknowns. Now if A is non-singular then the system of equations have m equations in n unknowns. Write the augmented matrix
will have a unique solution, i.e. trivial solution, and if A is a singular, [A B] and reduce it to an Echelon form by applying only E-row
then the system of equations will have infinitely many solutions. transformations and comparing the ranks of the augmented
matrix [A B] and the coefficient matrix A. Then, the following
18.22 System of Linear different cases arise:
Non-Homogeneous Equations Case I:  Rank A < Rank [A B]
In this case, the equations AX = B are inconsistent, i.e. they have
Let the equations no solution.

a11x1  a12 x 2    a1n x n  b1 Case II:  Rank A = Rank [A B] = r (say).
 In this case, the equations AX = B are consistent, i.e. they possess
a21x1  a22 x 2    a2n x n  b2 
 a solution. If r < m, then in the process of reducing the matrix [A
..............................................  (18.7)
 B] to the Echelon form, (m - r) equations will then be replaced by
..............................................  an equivalent system of r equations. From these r equations, we

am1x1  am2 x 2    amn x n  bm  shall be able to express the values of some r unknowns in terms
of the remaining n - r unknowns which can be given any arbitrary
be a system of m non-homogeneous equations in n unknowns x1, chosen values.
x2, …, xn. If we write If r = n, then n - r = 0, so that no variable is to be assigned arbitrary
values and therefore in this case there will be a unique solution.
 a11 a12 ... a1n   x1   b1 
      If r < n, then n - r variables can be assigned arbitrary values.
a a22 ... a2n   x  b  So, in this case there will be an infinite number of solutions. Only
A   21  , X   2 ,B   2 
 ... ... ... ...   ...   ...  n - r + 1 solutions will be linearly independent and the rest of the
      solutions will be linear combinations of them.
am1 am2 ... amn  mn x n  bm 
n1 m1
If m < r, then r ≤ m < n. Thus, in this case n - r > 0. Therefore, when
where A, X, B are m × n, n × 1, m × 1 matrices, respectively, the the number of equations is less than the number of unknowns,
above equations can be written in the form of a single matrix the equations will always have an infinite number of solutions
equation AX = B. provided they are consistent.
Any set of values of x1, x2, …, xn which simultaneously satisfy all For a non-singular matrix A:
these equations is called a solution of the system (18.7). When the AX = B ⇒ X = A-1B

Mathematical Problem Book for JEE.indb 745 07-06-2018 12:41:57


746 Mathematics Problem Book for JEE

By comparing entries on both the sides, we have a unique solution


for a given system of equations. Your Turn 2
Illustration 18.10   Show that the equations 2x + 6y + 11 = 0,  2 3 2 
 
6x + 20y - 6z + 3 = 0 and 6y - 18z + 1 = 0 are not consistent.   1.  Evaluate the determinant  1 2 3  .
 
2 1 3  Ans. -37
Solution:
2 6 0 1 4 5 
 
∆ = | A | = 6 20 -6 = 0   2. Compute the adjoint of the matrix A  3 2 6  and verify
 
0 6 -18 0 1 0 
  that A (adj A) = (adj A)A = A I.
-11 6 0
 1 3 2 
∆1 = -3 20 -6 ≠ 0  
  3. Find the inverse of matrix A  3 0 1 using elementary
-1 6 -18  
 2 1 0 
2 -11 6 transformation.
∆ 2 = 6 -3 20 ≠ 0  1 2 3
 
0 -1 6 Ans.  2 4 7 
 
2 6 -11 3 5 9 
∆ 3 = 6 20 -3 ≠ 0 1 2 3 
 
0 6 -1   4. Reduce the matrix A  4 5 6  in Echelon form and hence
 
3 4 5 
So, the system is inconsistent.
Alternate method: The given system of equations is equivalent to find its rank.
Ans. 2
the single matrix equation:
  5.  Solve the following system of equations using matrix method:
2 6 0  x  11 x + 2y + z = 7
    
AX  6 20 6  y    3   B   x + 3z = 11
    
0 6 18  z   1   2x - 3y = 1
Ans.  x = 2, y = 1, z = 3
We shall reduce the coefficient matrix A to triangular form by E-row   6.  Solve the following system of homogeneous equations:
operations on it and apply the same operations on the right-hand (i) 2x + 3y - z = 0
side, i.e. on the matrix B.      x - y - 2z = 0
Performing R2 → R2 - 3R1, we have        3x + y+ 2z = 0
(ii)  x + y - 6z = 0
2 6 3  x  11     x - y + 2z = 0
    
0 2 6  y    30    -3x + y + 2z = 0  Ans.  Only trivial solution
    
0 6 18  z   1 

Performing R3 → R3 - 3R2, we have 18.23  Minor of Any Element of a Matrix


2 6 0  x  11 a11 a12 a13
     Consider the determinant D = a21 a22 a23
0 2 6  y    30 
     a31 a32 a33
0 0 0  z  91
If we leave the row and the column passing through the element
The last equation of this system is 0x + 0y + 0z = -91. This shows
aij, then the second-order determinant thus obtained is called the
that the given system is not consistent.
minor of the element aij and we shall denote it by Mij. In this way,
we can get nine minors corresponding to the nine elements of D.
18.22.1 Matrix Method of Solving For example,
Non-Homogeneous System of Linear a12 a13
Equations Minor of element a21 = = M21
a32 a33
1. If A is a non-singular matrix, then the system of equations given a11 a13
by AX = B has a unique solution given by X = A-1B. Minor of element a32 = = M32
2. If A is a singular matrix and (adj A)D = 0, then the system of equa- a21 a23
tions given by AX = D is consistent with infinitely many solutions. a22 a23
3. If A is a singular matrix and (adj A)D ≠ 0, then the system of Minor of element a11 = = M11 and so on
a32 a33
equation given by AX = D is inconsistent.

Mathematical Problem Book for JEE.indb 746 07-06-2018 12:42:11


Chapter 18 | Matrices and Determinants 747

18.24 Cofactor of Any Element of a Matrix 2.  The numbers ai, bi, ci (i =1, 2, 3) are called the elements of
the determinant.
The minor Mij multiplied by (-1)i+j is called cofactor of the element
aij. We shall denote the cofactor of an element by the Cij. With this 3. The determinant obtained by deleting the i th row and j th
notation, cofactor of aij = Cij = (-1)i+jMij. column is called the minor of element at the i th row and
the j th column. The cofactor of this element is (-1)i+j (minor).
Note that
18.25  Determinant of Any Matrix
a1 b1 c1
If matrix A = [aij] is a square matrix of order ‘n’, then D = a2 b2 c2 = a1A1 + b1B1 + c1C1
n  n 
Determinant of A =  a1k C1k    a2k C2k      a3 b3 c3
k 1  k 1  where A1, B1 and C1 are the cofactors of a1, b1 and c1,
n  n 
=  ak1C k1    ak 2C k 2      respectively.
k 1  k 1 
where Cik represents cofactor of the element of the i th row and 18.26  Properties of Determinants
the kth column of matrix A.
 1. If two rows (or columns) in a determinant are interchanged,
For 3 × 3 order matrix A;
the sign of the determinant changes. For example, by
det A (or |A|) = a11C11 + a12C12 + a13C13 a1 b1
= a21C21 + a22C22 + a23C23 interchanging the two rows of the determinant , we
a2 b2
= a31C31 + a32C32 + a33C33 a b
2 2
= a11C11 + a21C21 + a31C31 get the determinant a b .
1 1
= a12C12 + a22C22 + a32C32
But we have
= a13C13 + a23C23 + a33C33
a2 b2 a1 b1
Thus determinant of a matrix can be obtained by adding the prod- =-
a1 b1 a2 b2
ucts of elements of any row or column by their cofactors.
Note:  If elements of a row (or column) are multiplied by the  2. If the numbers in one row are added m times the numbers in
cofactors of any other row (or column), then the sum of these another row, the value of the determinant remains unaltered.
products is zero. For example, For example,
a11c21 + a12c22 + a13c23 = 0 a1 + ma2 b1 + mb2 a b
= 1 1
2 3 4 a2 b2 a2 b2
Illustration 18.11   Evaluate the determinant D = 5 - 2 1 . This rule can be extended to more number of rows for higher
1 2 3 order determinants.
Solution: We can do it in two ways.
(a)  Expanding along the second row, we have   3. 
If rows and columns are interchanged, the value of the
determinant remains unaltered. For example,
3 4 2 4 2 3
D = -5 -2 -1 a1 b1 a a
2 3 1 3 1 2 = 1 2
a2 b2 b1 b2
= - 5 (9 - 8) - 2 (6 - 4) - 1 (4 - 3)
= - 5 - 4 - 1 = - 10 Another way of saying this is that it makes no difference if
(b)  Expanding along the third column, we have we reflect the numbers of the determinant in the line of the
principal diagonal. This means that any statement that can
5 -2 2 3 2 3
D=4 -1 +3 truly be made about rows in particular results (1) and (2) can
1 2 1 2 5 -2 equally well be made about columns.

= 4 (10 + 2) - 1 (4 - 3) + 3 (- 4 - 15)  4. If all the numbers in any row are zeros, the value of the
= 48 - 1 - 57 = - 10 determinant is zero. For example,

Hence, determinant is -10. a1 b1 c1


0 0 0 = 0
Basic Concepts a3 b3 c3
1. A
 determinant of order 3 consisting of three rows and three  5. If two rows are identical, the value of the determinant is zero.
columns is written as For example,
a1 b1 c1
a2 b2 c2 = a1(b2c3 - c2b3 ) - b1(a2c3 - c2a3 ) + c1(a2b3 - b2a3 ) a1 b1 c1
a3 b3 c3 a2 b2 c2 = 0
a1 b1 c1

Mathematical Problem Book for JEE.indb 747 07-06-2018 12:42:26


748 Mathematics Problem Book for JEE

 6. If the elements of a row are multiplied by any number m, the Therefore,
determinant is multiplied by m. For example, D = (a - b) (b - c) (c - a)
ma1 mb1 mc1 a1 b1 c1 Alternative method:
a2 b2 c2 = m a2 b2 c2 1 a a2
a3 b3 c3 a3 b3 c3 D = 1 b b2
  
 7. Row-column operations: The value of determinant remains 1 c c2
unchanged when any row (or column) is multiplied by a
number or any expression and then added or subtracted from Subtracting the second row from the first and then the third row
any other row (or column). That is, from the second, we have

a1 a2 a3 a1 + ma2 - na3 a2 a3 0 a - b a2 - b 2 0 1 a+b


b1 b2 b3 = b1 + mb2 - nb3 b2 b3 D= 0 b-c b2 - c 2 = (a - b) (b - c) 0 1 b + c
c1 c2 c3 c1 + mc2 - nc3 c2 c3 1 c c2 1 c c2
The above operation is written using C1 ® C1 + mC2 - nC3 Now expanding along the first column, we have
which means C1 is replaced by C1 + mC2 - nC3 .
D = (a - b) (b - c) [(b + c) - (a + b)] = (a - b) (b - c) (c - a)
  8. Determinant of a triangular matrix is the product of its diago-
nal elements. For example, Illustration 18.13   Show that
a1 b1 c1 a1 0 0 a1 0 0 b+c c b
0 b2 c2 = a2 b2 0 = 0 b2 0 = a1b2c3 D= c c+a a = 4abc
0 0 c3 a3 b3 c3 0 0 c3 b a b+a
  9. If a determinant D becomes zero on putting x = a, then we say Solution:
that (x - a) is a factor of D. For example, if
2(b + c ) 2(c + a) 2(a + b )
x 1 3
D= c c+a a by R1: R1 + R2 + R3
∆ = x2 2 9 b a a+b
3
x 3 27
then Now take 2 as a common factor and then apply R2: R2 - R1 and
D = 0 if x = 3 R3: R3 - R1
b+c c +a a+b
Hence, (x - 3) is a factor of D.
D=2 -b 0 -b
1 a a2 -c -c 0
Illustration 18.12   Show that 1 b b2 = (a - b) (b - c) (c - a).
Now apply C2: C2 - C1
2
1 c c
b+c a-b a+b
Solution: Let
D=2 -b b -b
1 a a2 -c 0 0
D = 1 b b2
Now expand through R3 to get
1 c c2
D = 2[(-c) {-ab + b2 - ab - b2}] = 4abc
If b is put equal to a, two rows are exactly alike. Therefore, D = 0
when b = a. Hence, (a - b) is a factor of D [this follows from the Illustration 18.14   Show that
factor theorem which states that for f(x), if f(a) = 0, then (x - a)
is a factor of f(x)]. 1 1 1
Similarly, (b - c) and (c - a) are factors. D= a b c 0
Again, D is of third degree in a, b and c. 2 2
a  bc b  ca c  ab 2

We already know the three linear factors are (a - b), (b - c) and


(c - a). If there is another factor, it must be a mere number. Thus Solution: Applying C1: C1 - C2 and C2 : C2 - C3 we get
1 a a2
0 0 1
1 b b2 = N (a - b) (b - c) (c - a), where N is a number
a-b b-c c
1 c c2
a2 - b2 + c ( a - b ) b2 - c 2 + a(b - c ) c 2 - ab
By equating coefficients of bc2 on both sides, we get N = 1.

Mathematical Problem Book for JEE.indb 748 07-06-2018 12:42:45


Chapter 18 | Matrices and Determinants 749

0 0 1 a b+c c
= (a - b) (b - c) 1 1 c =2 b c +a a (C2 → C2 - C3)
a + b + c b + c + a c - ab 2 c a+b b

= (a - b) (b - c) [(b + c + a) - (a + b + c)] = 0 a b c
(Expanding along R1) =2 b c a
Note: If a determinant can be so transformed that two elements c a b
in a row or column are made zero, then the determinant can be
expanded in terms of that row or column. 1 a a2
1 a bc
1 a a2 1 bc b + c Illustration 18.17   Show that 1 b ca = 1 b b2 .
Illustration 18.15   Show that 1 b b2 = 1 ca c + a . 1 c ab 1 c c2
1 c c 2 1 ab a + b
Solution: Let D stand for the determinant on the left. Then
Solution: We have
a a2 abc a a2 1
1 bc b + c 1 bc a + b + c - a
1 abc
1 ca c + a = 1 ca a + b + c - b D = b b2 abc = b b2 1
abc abc
1 ab a + b 1 ab a + b + c - c c c2 abc c c2 1

1 bc a + b + c 1 bc a a 1 a2 1 a a2
= 1 ca a + b + c - 1 ca b = - b 1 b2 = 1 b b2
1 ab a + b + c 1 ab c c 1 c2 1 c c2
1 bc 1 bc 1 a
= (a + b + c) 1 ca 1 + ca 1 b
Additional Properties of Determinants:
1 ab 1 ab 1 c
 1. The determinant remains unaltered if its rows are changed
bc 1 a into columns and the columns into rows.
= ca 1 b , since the first determinant vanishes  2. If all the elements of a row (or column) are zero, then the
ab 1 c determinant is zero.
abc a a2 1 a a2  3. If the elements of a row (column) are proportional (or
1 2 abc 2 identical) to the elements of any other row (column), then
= abc b b = 1 b b
abc abc the determinant is zero.
2
abc c c 1 c c2
 4. The interchange of any two rows (columns) of the
determinant changes its sign.
Illustration 18.16   Without expanding the determinants, prove
 5. If all the elements of a row (column) of a determinant are
a+b b+c c +a a b c multiplied by a non-zero constant, then the determinant
that b+c c +a a+b = 2 b c a . gets multiplied by the same constant.
c +a a+b b+c c a b  6. A determinant remains unaltered under a column (Ci)
operation of the form Ci + a Cj + b Ck (j, k ≠ i) or a row (Ri)
Solution: The determinant on the left is equal to
operation of the form Ri + a Rj + b Rk (j, k ≠ i).
a+b b+c c +a  7. If each element in any row (column) is the sum of r terms,
b+c c +a a+b (C1 → C1 + C2 + C3) then the determinant can be expressed as the sum of r
c +a a+b b+c determinants.
  8. If the determinant D = f(x) and f(a) = 0, then (x - a) is a
2 (a + b + c ) b + c c + a
factor of the determinant. In other words, if two rows (or
 = 2 (a + b + c ) c + a a + b
two columns) become proportional (identical) for x = a,
2 (a + b + c ) a + b b + c then (x - a) is a factor of determinant. In general, if r rows
a+b+c b+c c +a become identical for x = a, then (x - a)r-1 is a factor of the
=2 a+b+c c +a a+b (C1 → C1 - C2) determinant.
a+b+c a+b b+c 9. If in a determinant (of order 3 or more) the elements in all
the rows (columns) are in AP with same or different common
a b+c c +a difference, the value of the determinant is zero.
=2 b c +a a+b (C3 → C3 - C1) 10. The determinant value of an odd-order skew-symmetric
c a+b b+c determinant is always zero.

Mathematical Problem Book for JEE.indb 749 07-06-2018 12:42:59


750 Mathematics Problem Book for JEE

18.27  Sum of Determinants Now,


n 2n  1
a1 b1 c1 d1 b1 c1  2 k  1  1  2  22    2 n  1 
k 1 2 1
 2n  1
Let D1 = a2 b2 c2 and D = d 2 b2 c2 be two
2 (sum of n terms of a GP)
a3 b3 c3 d3 b3 c3 n 2(3n  1)
third-order determinants in which the corresponding second and  2 (3k  1)  2 { 1  3  32    3n  1}
k 1

3 1
 3n  1
third columns are identical. Then
and
a1 + d1 b1 c1 n
D1 + D 2 = a2 + d2 b2 c2  3( 4 k  1) 
k 1
4n  1
a3 + d3 b3 c3 Hence,
This fact is evident if we expand all the three determinants in terms 2n  1 x 2n  1
n
of column 1 and compare the results. 
k 1
Ak  3n  1 y 3n  1 = 0 (since C1 = C3)
p1 q1 r1 4n  1 z 4n  1
Similarly, if ∆ 3 = a2 b2 c2 , then
n
a3 b3 c3 2r + 1 Cr 1
2 n
Illustration 18.19   If f (r ) = n + 2n + 1 2 n +1 ,
a1  p1 b1  q1 c1  r1
2 2 2 2
cos (n ) cos n cos (n + 1)
∆1  ∆ 3  a2 b2 c2
n
a3 b3 c3 0 ≤ r ≤ n, then prove that  f (r ) 
r 0
0.
Here, we note that the corresponding second and third rows are
Solution: Since R2 and R3 are constants (independent of the vari-
identical.
able r), we have
Similarly, the determinant
n n n
d1 + e1 + f1 d2 + e2 + f2 d3 + e3 + f3
 (2r  1) r 0 n
Cr 1
b1 b2 b3 n
r0 r0

c1 c2 c3  f (r ) 
r 0
n2  2n  1 2n n 1
2 2 2
can be decomposed into the sum of three determinants cos (n ) cos n cos2 (n  1)
d1 d2 d3 e1 e2 e3 f1 f2 f3
b1 b2 b3 + b1 b2 b3 + b1 b2 b3 Now,
n
c1 c2 c3 c1 c2 c3 c1 c2 c3  (2r  1)  1  3   (2n  1)  (n  1)2
r 0
It may be observed that the determinant
n
a1 + b1 c1 + d1 e1 + f1 
r 0
n
C r  nC 0  nC1    nC n  2n
a2 + b2 c 2 + d2 e2 + f2
n
a3 + b3 c 3 + d3 e3 + f3
 1
r 0
1 + 1 +  (n + 1) times = n + 1
can be expressed as sum of 2 × 2 × 2 = 8 determinants.
Hence
2k - 1 x 2n - 1
n (n  1)2 2n n 1
Illustration 18.18   If Ak = 2 (3k - 1) y 3n - 1 , prove that å Akn = 0.
2n - 1 3 ( 4 k - 1) z 4n - 1
k =1 
r0
f (r )  (n  1) 2
2 n
n 1  0 (since R1  R2 )
2 2 2 2
n cos (n ) cos n cos (n  1)
3n - 1 , prove that å Ak = 0.
k =1
4n - 1
18.28  Multiplication of Determinants
Solution: Observe that all the determinants A1, A2 ,… , An have Two determinants of the same order, i.e. each consisting of the
identical second and third columns. Hence, same number of rows and equal number of columns, can be
n multiplied to give a determinant of the same order. Thus, if A is
 2k  1 x 2n  1 a 2 × 2 determinant and B is another 2 × 2 determinant, A × B =
k 1 C is also 2 × 2 determinant. The multiplication is performed by a
n n method of working the row of A on the columns of B.

k 1
Ak   2 (3k  1)
k 1
y 3n  1 The method is as follows: If
n a1 a2 a a2
A= ;B= 1

k 1
3 ( 4 k  1) z 4n  1 b1 b2 b1 b 2

Mathematical Problem Book for JEE.indb 750 07-06-2018 12:43:23


Chapter 18 | Matrices and Determinants 751

a1a1 + a2 b1 a1a 2 + a2 b2 Minor of x is yz - x 2 of y is y 2 - zx , of z is xy - z 2 .


then AB =
b1a1 + b2 b1 b1a 2 + b2 b2 The left-hand side determinant in the problem is therefore
To cite a numerical example for a 3 × 3 determinant, we have X -Y Z
1 3 4 2 1 2 -Y Z -X
2 -1 6 ´ 0 1 3 Z -X Y
3 0 2 1 2 4 where the capital letters denote the minor of the corresponding
1´ 2 + 3 ´ 0 + 4 ´ 1 1´ 1 + 3 ´ 1 + 4 ´ 2 1´ 2 + 3 ´ 3 + 4 ´ 4 small letters. Therefore,
= 2 ´ 2 + ( -1) ´ 0 + 6 ´ 1 2 ´ 1 + ( -1) ´ 1 + 6 ´ 2 2 ´ 2 + ( -1) ´ 3 + 6 ´ 4 X -Y Z x y z x y z
3 ´ 2 + 0 ´ 0 + 2 ´1 3 ´ 1+ 0 ´ 1+ 2 ´ 2 3´2 + 0´3 + 2´ 4 LHS = -Y Z - X = y z x y z x
6 12 27 Z -X Y z x y z x y
= 10 13 25 2
x +y +z 2 2
xy + yz + zx xz + xy + yz
8 7 14 2 2 2
= xy + yz + zx x +y +z xy + yz + zx
(The first row is obtained by working the first row elements 1, 3, 4, xy + yz + zx xy + yz + zx x2 + y2 + z2
       
respectively, on 2, 0, 1 the first column; then on 1, 1, 2 the second
column; then on 2, 3, 4 the third column. Likewise for the second r2 u2 u2
and the third rows.)
      = u2 r2 u2 in the notation of the problem
Verification: u2 u2 r2
1 3 4 1 3 4
 (R2  2R1)
A  2 1 6  0 7 2 Illustration 18.21   For all values of A, B, C and P, Q, R, show that
3 0 2 0 9 10  (R5  3R1)
 70  18  52 cos ( A - P ) cos ( A - Q ) cos ( A + R )
2 1 2 0 3 6 cos (B - P ) cos (B - Q ) cos (B - R ) = 0
B  0 1 3  0 1 3  (R1  2R3 ) cos (C - P ) cos (C - Q ) cos (C - R )
1 2 4 1 2 4 Solution: The given determinant is the product of
 9 6  3
cos A sin A 0 cos P cos Q cos R
6 12 27 2 4 9 2 4 9 D1 = cos B sin B 0 and D 2 = sin P sin Q sin R
C  10 13 25  10 13 25  3 0 2 2  R2  (R1  R3 ) cos C sin C 0 0 0 0
8 7 14 8 7 14 8 7 14
and ∆1 = ∆2 = 0 and hence ∆1. ∆2 = 0.
2 4 5
Alternately
= 3 0 2 0 = 6(14 - 40 ) = - 156
cos ( A - P ) cos ( A - Q ) cos ( A - R )
8 7 7
cos (B - P ) cos (B - Q ) cos (B - R )
Therefore, AB = - 156 = C cos (C - P ) cos (C - Q ) cos (C - R )
Multiplication can also be performed row by row, column by
row or column by column. cos A cos ( A - Q ) cos ( A - R )
= cos P cos B cos (B - Q ) cos (B - R )
Illustration 18.20   Show that
cos C cos (C - Q ) cos (C - R )
yz - x 2 zx - y 2 xy - z 2 r2 u2 u2 sin A cos ( A - Q ) cos ( A - R )
zx - y 2 xy - z 2 yz - x 2 = u2 r2 u2 + sin P sin B cos (B - Q ) cos (B - R )
xy - z 2 yz - x 2 zx - y 2 u2 u2 r2 . sin C cos (C - Q ) cos (C - R )

= (cos P ) A1 + (sin P ) B1

where r 2 = x 2 + y 2 + z 2 and u2 = yz + zx + xy .
where
Solution: Consider the determinant cos A sin A sin Q sin A sin R
A1 = cos B sin B sin Q sin B sin R
x y z
cos C sin C sin Q sin C sin R
∆= y z x
z x y (using C2  C2  (cos Q ) C1, C3  C3  (cos R ) C1)

Mathematical Problem Book for JEE.indb 751 07-06-2018 12:43:43


752 Mathematics Problem Book for JEE

So cos A sin A sin A Illustration 18.22  If a is a repeated root of a quadratic equation
A1 = sin Q sin R cos B sin C sin B = 0 f(x) = 0 and A(x), B(x) and C(x) are polynomials of degrees 3, 4 and 5,
cos C sin C sin C respectively, then show that
 (second and third columns are identical) A( x ) B( x ) C ( x )
Similarly, it may be proved that B1 = 0. A(a ) B(a ) C (a )
A¢(a ) B ¢(a ) C ¢(a )
Product of Two Determinants
is divisible by f(x), where the prime symbol denotes the derivatives.
a1 b1 c1 a 1 b1 g 1
a2 b2 c2 a 2 b2 g 2 Solution: Let
a3 b3 c3 a 3 b3 g 3 A( x ) B( x ) C ( x )

g(x) = A(a ) B(a ) C (a )
a1a 1 + b1b1 + c1g 1 a1a 2 + b1b 2 + c1g 2 a1a 3 + b1b 3 + c1g 3
A¢(a ) B ¢(a ) C ¢(a )
= a2a 1 + b2 b1 + c2g 1 a2a 2 + b2 b 2 + c2g 2 a2a 3 + b2 b 3 + c2g 3
a3a 1 + b3 b1 + c3g 1 a3a 2 + b3 b 2 + c3g 2 a3a 3 + b3 b 3 + c3g 3 Then
A¢( x ) B ¢( x ) C ¢( x )
Here, we have multiplied rows by rows. We can also multiply
g′(x) = A(a ) B(a ) C (a )
rows by columns or columns by rows, or columns by columns.
Note: If D = |aij| is a determinant of order n, then the value of A¢(a ) B ¢(a ) C ¢(a )
the determinant |Aij|, where Aij is the cofactor of aij, is Dn-1. This
Now
is known as power cofactor formula.
A(a ) B(a ) C (a )
g(a) = A(a ) B(a ) C (a )
18.29  Differentiation of Determinants
A¢(a ) B ¢(a ) C ¢(a )
Following is the differentiation of a determinant whose elements
are functions of a variable x. Let Since two rows are identical, we have g(a ) = 0
f ( x ) g( x ) A′(a ) B ′(a ) C ′(a )
F( x ) =
h( x ) u( x ) g′(a ) = A(a ) B(a ) C (a )
A′(a ) B ′(a ) C ′(a )
Then
F(x) = f(x) × u(x) - g(x) × h(x) Since two rows are identical, we have g′(a ) = 0.
and Since g(a) = 0 and also g′(a) = 0, a is a repeated root of g(x) = 0
d Therefore,
F′(x) = F(x)
dx g(x) = (x - a)2h(x)(18.8)
= {f(x) × u′(x) + u(x) × f ′(x)} - {g(x) × h′(x) + h(x) g′(x)} Since a is a repeated root of f(x) = 0, we have
f ¢( x ) g¢( x ) f ( x ) g( x ) f(x) = N(x - a)2(18.9)
= +
h( x ) u( x ) h¢( x ) u¢( x ) where N is some number. From Eqs. (18.8) and (18.9), we find that
g(x), i.e. the given determinant is divisible by f(x).
Thus, F′(x) is the sum of two determinants of which the first
one is obtained by differentiating the elements of the first row
alone and retaining the second row without any change and Differentiation of a Determinant
the second one is obtained by differentiating the elements of a (x) ab1( x ) b1( x )
the second row. Let ∆ ( x ) Let1 ∆ ( x )  . Then . Then
a2 ( x ) ab2 ( x ) b2 ( x )
Similarly, if
a1 ’( x ) ab ’( x ) a1b( 1x’() x ) ab1( x ) b1( x )
f1( x ) g1( x ) h1( x ) ∆ ’( x )  ∆ ’( x )  1  
a2 ( x ) ab2 ( x ) a2b’(2 (xx) ) ab2 ’( x ) b2 ’( x )
F(x) = f2 ( x ) g2 ( x ) h2 ( x )
f3 ( x ) g3 ( x ) h3 ( x ) where the prime symbol denotes the derivative with respect to x.
then
f1¢( x ) g1¢( x ) h1¢( x ) f1( x ) g1( x ) h1( x ) 18.30  Special Determinants
f
F′(x) = 2 ( x ) g2 ( x ) h2 ( x ) + f2¢( x ) g2¢ ( x ) h2¢ ( x )
f3 ( x ) g3 ( x ) h3 ( x ) f3 ( x ) g3 ( x ) h3 ( x ) 18.30.1  Symmetric Determinant
f1( x ) g1( x ) h1( x ) If the elements of a determinant are such that aij = aji (where aij is
+ f2 ( x ) g2 ( x ) h2 ( x ) the element of i th row and j th column), then the determinant is said
f3¢( x ) g3¢ ( x ) h3¢ ( x ) to be a symmetric determinant. The elements situated at equal

Mathematical Problem Book for JEE.indb 752 07-06-2018 12:43:57


Chapter 18 | Matrices and Determinants 753

distances from the diagonal are equal both in magnitude and sign. Solving the system we get
For example, c1 b1
a h g c1 b2 - c2b1 c2 b2
x = = ;
h b f = abc + 2fgh - af 2 - bg2 - ch2 a1b2 - a2b1 a1 b1
g f c a2 b2
a1 c1
18.30.2  Skew-Symmetric Determinant a c - a2c1 a c
y = 12 = 2 2
If aij = -aji (where aij is the element of i th row and j th column), then a1b2 - a2b1 a1 b1
the determinant is said to be a skew-symmetric determinant, a2 b2
which means that all the diagonal elements are zero and the
Note: The given equations are consistent and independent if and
elements situated at equal distances from the diagonal are equal
a b
in magnitude but opposite in sign. The value of a skew-symmetric only if 1 1 ≠ 0.
a2 b2
determinant of odd order is zero. For example,
 0 3 5  Illustration 18.24   Solve the system 4x + y = 13, 3x - 2y = 7 using
 
A   3 0 4 determinants.
  Solution: The solution requires the values of three determinants.
5 4 0 
The denominator D is formed by writing the coefficients of x and
A 0
y in order
4 1
18.30.3  Circulant Determinants D= = - 8 - 3 = - 11
3 -2
In these determinants, the elements of the rows (or columns) are in D1, the numerator of x, is formed by replacing the coefficients of x
cyclic arrangement. For example,
by the constant terms
a b c 13 1
D1 = = - 26 - 7 = - 33
b c a = -(a3 + b3 + c 3 - 3abc ) 7 -2
c a b D2, the numerator of y, is formed by replacing the coefficients of y
1 by the constant terms
= - (a + b + c ) × {(a - b )2 + (b - c )2 + (c - a)2 }
2 4 13
D2 = = 28 - 39 = - 11
3 7
a b c
Then
Illustration 18.23   Evaluate the determinant D = b c a D1 - 33
x= = = 3
c a b D - 11
and show that it is negative for all positive values of a, b and c. ∆ 2 - 11
and y= = =1
Solution: Expanding along the first row, we have ∆ - 11
c a b a b c 18.31.2  Solution of System of Three Linear
D=a -b +c Equations in Three Unknowns
a b c b c a
Consider the system of three linear equations in three unknowns:
D = a(bc - a2) - b(b2 - ca) + c(ab - c2) = 3abc - a3 - b3 - c3
a1x + b1y + c1z = d1
  = - (a3 + b3 + c3 - 3abc) = - (a + b + c) {a2 + b2 + c2 - ab - bc - ca}
a2x + b2y + c2z = d2
(a + b + c )
 = - {(a - b )2 + (b - c )2 + (c - a)2 } a3x + b3y + c3z = d3
2
Consider
is negative if a, b and c are positive.
a1 b1 c1 d1 b1 c1
D = a2 b2 c2 , D1 = d2 b2 c2
18.31 Solution of System of Linear a3 b3 c3 d3 b3 c3
Equations a1 d1 c1 a1 b1 d1
D 2 = a2 d2 c2 , D2 = a2 b2 d2
18.31.1 Solution of System of Two Linear a3 d3 c3 a3 b3 d3
Equations in Two Unknowns
 1. If ∆ ≠ 0, system has unique solution given by
Consider the system of two linear equations in two unknowns:
D1 D D
a1x + b1y = c1 x= ,y= 2,z= 3
a2x + b2y = c2 D D D

Mathematical Problem Book for JEE.indb 753 07-06-2018 12:44:12


754 Mathematics Problem Book for JEE

That is, system is consistent with independent solution. Let Dj be the determinant obtained from D after replacing the j th
  2. If ∆ = 0 and ∆1 = ∆2 = ∆3 = 0 then system has infinite many solu- b1
tions. That is, system is consistent with dependent solution. .
 3. If ∆ = 0 and any of ∆1, ∆2, ∆3 are non-zero then the system has
column by . .
no solution. That is, system is inconsistent.
.
18.31.3 Solution of System of Three Equations bn
in Two Unknowns Then, if D ≠ 0, we have
The following system of equations D1 D D
a1x + b1y + c1 = 0 ; a2 x + b2 y + c2 = 0 ; a3 x + b3 y + c3 = 0 x1 = , x2 = 2 , … , xn = n
D D D
is consistent if
When D = 0, we have the following cases:
a1 b1 c1 Case 1:  If D = 0 and the other determinants D1 = D2 = … = Dn
a2 b2 c2 = 0 = 0, then system of equation has infinitely many solutions if all
a3 b3 c3 cofactors of D1, D2, …, Dn and D are zero. If any one cofactor of D1,
D2, D3,…, Dn is non-zero then system has no solution.
Illustration 18.25  Find those values of c for which the Example: x + 3 y + 2 z = 1; 2 x + 6 y + 4 z = 5; 3 x + 9 y + 6 z = 9
equations 2 x + 3 y = 3; (c + 2) x + (c + 4 ) y = (c + 6 ) and (c + 2)2 x + (c +Here,
4 )2 y D=x(c=+D6y)2= Dz = D = 0 yet system has no solution whereas
2 2 2
(c + 2) x + (c + 4 ) y = (c + 6 ) are consistent. Also solve the equations for x + 3 y + 2 z = 1 ; 2 x + 6 y + 4 z = 2; 3 x + 9 y + 6 z = 3
those values of c. has infinitely many solutions.
Solution: The condition for consistency is Case 2: If D = 0 but any one of the D1, D2, …, Dn is not equal to zero
then the system has no solution, hence is inconsistent.
2 3 3
 c 2 c 4 c 6  0 Cramer’s Rule
If
(c  2)2 (c  4 )2 (c  6 )2
a1 b1 c1
-1 3 0 ∆ = a2 b2 c2 ≠ 0
⇒ -2 c+4 2 = 0 (C1 → C1 - C2) a3 b3 c3
-2(2c + 6 ) (c + 4 )2 2(2c + 10 ) then solution of linear equations a1x + b1y + c1z = d1, a2 x + b2 y
2
 ( 1){(c  4 )(2c  10 )  (c  4 ) }  3{ 2(2c  10 )  2(2c  6 )}  0 + c2 z = d2 and a3 x + b3 y + c3 z = d3 is given by {where (d1, d2 , d3 )
≠ (0 , 0 , 0 )}
 c 2  8c  16  2c 2  18c  40  12c  60  12c  36  0
∆x ∆y ∆
⇒ - c2 -10 c = 0 ⇒ c = 0 or c = -10 x= , y= , z= z
∆ ∆ ∆
For c = 0, the three equations are
where
2x + 2y = 3; 2x + 4y = 6; 4x + 16y = 36
d1 b1 c1 a1 d1 c1
and the solution is x = -3; y = 3. For c = -10, the equations are
∆ x = d2 b2 c2 , ∆ y = a2 d2 c2 ,
2x + 3y = 3
-8 x - 6 y = -4 Þ 4 x + 3 y = 2 d3 b3 c3 a3 d3 c3
64 x - 36 y = 16 Þ 16 x + 9 y = 4 a1 b1 d1
1 4 ∆ z = a2 b2 d2
and the corresponding solution is x   ; y  .
2 3 a3 b3 d3
18.31.4  Cramer’s Rule 1. If any of Dx, Dy, Dz ∈ R and D ≠ 0, the system of equation
Consider the system of n linear equations in n unknowns given by will have unique solution and is said to be consistent
a11x1 + a12 x 2 +  + a1n x n = b1 independent.
2. If Dx = Dy = Dz = 0 and D is also zero, then the system of
a21x1 + a22 x 2 +  + a2n x n = b2 equation will have infinitely many solutions and is said to
……………………………… be consistent dependent.
……………………………… 3. If Dx, Dy, Dz are non-zero and D is zero, then the system of
a x + a x +  + ann x n = bn equations will have no solution and is said to be inconsistent.
n1 1 n2 2
Let
a11 a12 … a1n Illustration 18.26   Solve the following system using determi-
a21 a22 … a2n nants:
D= x + 4y + 4z = 7
: :
3x + 2y + 2z = 6
an1 an2 … ann
9x + 6y + 2z = 14

Mathematical Problem Book for JEE.indb 754 07-06-2018 12:44:38


Chapter 18 | Matrices and Determinants 755

Solution: The solution requires the values of four determinants: Solution: The condition for the existence of non-trivial solution
The denominator (trivial solution is x = y = z = 0) is
1 4 4 l sin a cos a
D = 3 2 2 = 40 1 cos a sin a =0
9 6 2 -1 sin a - cos a

D1, the numerator of x is 0 sin a ( l  1) cos a (1  l )


7 4 4  0 cos a  sin a sin a  cos a = 0
D1 = 6 2 2 = 40 1 sin a  cos a
14 6 2  ( l 1) sin a (sin a  cos a )  (1 l ) cos a (cos a  sin a )  0
 l (sin2 a  cos2 a )  sin2 a  cos2 a  2 sin a cos a  0
D2, the numerator of y is
 p
 l  sin 2a  cos 2a = 2 sin  2a + 
1 7 4  4
D2 = 3 6 2 = 20 l
 1  1⇒- 2 ≤ l ≤ 2
9 14 2 2
For l = 1,
D3, the numerator of z is
 p 1 p
sin  2a +  = = sin
1 4 7  4 2 4
D3 = 3 2 6 = 40 p p
since, 2a + =
9 6 14 4 4
Then General solution:
p p
D1 40 2a + = np + ( -1)n
x = = =1 4 4
D 40 p p
D 20 1 2a = np + ( -1)n -
y = 2 = = 4 4
D 40 2
If n is even, 2a = np
D 40 p
z = 3 = =1 If n is odd, 2a = np - .
D 40 2

18.31.5 System of Homogeneous Linear Your Turn 3


Equations  1. Evaluate the determinant
Existence of non-trivial solution: If the three equations (homo- 265 240 219
geneous) 240 225 198
a1x + b1y + c1z = 0 219 198 181
   Ans.  Zero
a2 x + b2 y + c2 z = 0
and    a3x + b3y + c3z = 0 x 3 6 2 x 7 4 5 x
 2. 
If 3 6 x = x 7 2 = 5 x 3 = 0 then what is the
are considered then there always exists a solution, i.e. x = y = z = 0.
6 x 3 7 2 x x 4 5
This is called trivial solution.
If the three equations are to have a solution other than value of x? Ans.  x = -9
x = 0 = y = z, such a solution is known as non-trivial solution. The my + nz mq + nr mb + nc
condition required for the existence of such a solution is
If D = kz - mx
 3.  kr - mp kc - ma and D is the product of
a1 b1 c1 nx + ky np + kq na + kb
a2 b2 c2 = 0 x y z
a3 b3 c3 two determinants one of which is p q r , then find the
a b c
other one. Also show that D = 0.
Illustration 18.27  Let l and a be real. Find the set of all values
0 m n
of l for which the system of linear equations
Ans.  - m 0 k
lx + (sin a ) y + (cos a ) z = 0
n k 0
x + (cos a ) y + (sin a ) z = 0
If the equations x = ay + z, y = az + x and z = ax + y are
 4. 
- x + (sin a ) y - (cos a ) z = 0
consistent having non-trivial solution, then prove that
has a non-trivial solution. For l = 1, find all the values of a. a3 + 3a = 0.

Mathematical Problem Book for JEE.indb 755 07-06-2018 12:44:58


756 Mathematics Problem Book for JEE

and therefore cik = 0.


x2 - x x3 x 4 -1
Thus, cik = 0 whenever i > k.
If f ( x ) = 2 x - 1 3 x 2
 5.  4 x 3 , then find the coefficient of x
Hence, the matrix AB is also a triangular matrix.
2 6x 12 x 2 1 2 2 
 
in f(x). Ans.  6  3. If A = 2 1 2  , show that A2 - 4A - 5I = 0, where I and 0 are
 
2 2 1
Additional Solved Examples the unit matrix and the null matrix of order 3, respectively. Use
this result to find A-1.
0 2b g  Solution: Given
 -g  is
 1. Determine the values of a, b, g when a b 1 2 2 
 
orthogonal. a -b g  A = 2 1 2 
 
2 2 1
Solution: Let Therefore,
0 2b g  0 a a  1 2 2 
  
1 2 2
A = a b -g  ⇒ A′ = 2 b b - b  A = A⋅A = 2 1 2  × 2 1
2  
   2
a -b g   g -g g   
2 2 1 2 2 1
Given A is orthogonal. Then AA′ = I. Hence 1 4  4 2  2  4 2  4  2  9 8 8 
 
0 2b g  0 a a  é 1 0 0ù = 2  2  4 4  1 4 4  2  2  = 8 9 8 
 
a b -g 2 b b - b  = ê0 1 0 ú 2  4  2 4  2  2 4  4  1 8 8 9 
   ê ú
a -b g   g -g g  ê0 0 1ú So,
ë û
9 8 8  1 0 0
 4b 2 +g 2 1 2 2  
2b 2 - g 2 -2 b 2 + g 2  é 1 0 0ù    
 2 2
 ê0 1 0 ú
A2 - 4A - 5I = 8 9 8  - 4 2
  1 2  - 5 0 1 0

⇒  2b - g a 2 + b 2 +g 2 a 2 - b 2 -g 2  =   0 0 1
  ê ú 8 8 9  2 2 1
2
 -2 b + g
2
a 2 - b 2 - g 2 a 2 + b 2 + g 2  êë0 0 1úû
9 8 8  4 8 8 5 0 0
     
Equating the corresponding elements, we have = 8 9 8  - 4 8 4 8  - 5 0 5 0
     
4b 2 + g 2 = 1 (1) 8 8 9  8 8 4  0 0 5 
2b 2 - g 2 = 0  (2) 9  4  5 8  8  0 8  8  0  0 0 0 
a 2 + b 2 + g 2 = 1 (3)    
= 8  8  0 9  4  5 8  8  0  = 0 0 0 
   
From Eqs. (1) and (2), we get 8  8  0 8  8  0 9  4  5 0 0 0 
1 Therefore,
6b 2 = 1 ⇒ b 2 =
6 A2 - 4A - 5I = 0 ⇒ 5I = A2 - 4A
So, By multiplying by A-1, we get
1
g =2 5A-1 = A - 4I
3
From Eq. (3), 1 2 2  4 0 0 
1 1 1    
= 2 1 2  - 0 4 0 
a2 = 1 - b2 - g 2 = 1 - - =    
6 3 2 2 2 1 0 0 4 
Hence,
1 1 1 1 4 2  0 2  0  3 2 2
a=± , b=± and g = ±    
2 6 3 = 2  0 1 4 2  0  =  2 3 2 
   
2  0 2  0 1 4   2 2 3
 2. Show that the product of two triangular matrices is itself
triangular. Therefore,
Solution: Let A = [aij]n×n and B = [bjk]n×n be two triangular matrices.
3 2 2  3 / 5 2/5 2/5 
Then aij = 0 when i > j. Also 1    
A-1 =   = 2/5 3 / 5 2 / 5 
bjk = 0 when j > k 5  2 3 2   
 2 2 3  2 / 5 2 / 5 3 / 5
n
Let AB = [cik]n×n. Then, cik = 
j 1
aij b jk .  4.  Find a square matrix A of order 2 × 2 such that A2 = I2.
Suppose that i > k: a b 
Solution: Let A =   be the required matrix. Then, A2 = I. So
(1) If j < i, then aij = 0 and therefore cik = 0. c d 
(2) If i < j, then j > k because i > k. In this case, bjk = 0.

Mathematical Problem Book for JEE.indb 756 07-06-2018 12:45:19


Chapter 18 | Matrices and Determinants 757

a b  a b  1 0  1
    =   ⇒S= (A - Aq ) = Q
c d  c d  0 1 2i

a2  bc ab  bd  Hence, expression (1) for A is unique.


1 0 
  =    6. Discuss for all values of l, the system of equations x + y + 4z
2 0 1
ac  cd cb  d  = 6, x + 2y -2z = 6, xl + y + z = 6 with regards to existence and
Comparing respective entries we get nature of solutions.
a2 + bc = 1  (1) Solution: The matrix form of the given system is
b + bd = 0 (2)
ac + cd = 0 (3)  1 1 4   x  6 
cb + d2 = 1 (4)  1 2 -2  y  = 6 
     
These must hold simultaneously.  l 1 1   z  6 
If a + d = 0, the above four equations hold simultaneously if
d = -a and a2 + bc = 1. The given system of equations will have a unique solution iff the
a b  coefficient matrix is non-singular. Using R2 → R2 - R1, R3 → R3 - lR1,
Hence, one possible square root of I is A = g -a  where a, b, we get
 
g are the three numbers related by the condition a 2 + bg = 1. 1 1 4  x  6 
0 1 - 6   y  =  0  (1)
If a + d ≠ 0, then the above four equations hold simultaneously if 
b = 0, c = 0, a = 1, d = 1 or if b = 0, c = 0, a = -1, d = -1. 0 1- l 1- 4 l   z  6 - 6 l 
1 0  1 0 
Hence,  ,   , i.e. ±I are the values of A. Therefore, the coefficient matrix will be non-singular iff
0 1  0 1
7
  5. Show that every square matrix A can be uniquely expressed as 1 - 4l + 6 - 6l ≠ 0 ⇒ l ≠
10
P + iQ, where P and Q are Hermitian matrices. 7
Thus, the given system will have a unique solution if l ≠ .
Solution: Let 10
1 1 q 7
P = (A + Aq ) and Q = ( A - A ) In case l = , Eq. (1) becomes
2 2i 10
Then 1 1 4  x   6 
A = P + iQ  (1)      
0 1 6  y    0 
Now      
0 3 / 10 18 / 10  z  18 / 10 
q
1 q  1
Pq =  ( A + A ) = ( A + Aq )q 3
 2  2 Using R3 → R3 - R gives
10 2
1 1 1
= {Aq + (Aq )q } = (Aq + A) = (A + Aq ) = P 1 1 4  x   6 
2 2 2      
0 1 6  y    0 
Therefore, P is a Hermitian matrix.      
Also 0 0 0  z  18 / 10 
q
1 q   1 1 This shows that the equations are not consistent in this case.
Qq =  ( A - A ) =   ( A - Aq )q = - {Aq - (Aq )q }
 2i  2i
  2i
 7. Let a, b, c be positive real numbers with abc = 1.
1 q 1
= - (A - A) = (A - Aq ) = Q a b c 
2i 2i  
Let A = b c a  . If ATA = I where AT is the transpose of A and
Therefore, Q is also a Hermitian matrix.  
c a b 
Thus, A can be expressed in the form (1).
I is the identity matrix, then determine the value of a3 + b3 + c3.
For A to be unique, let A = R + iS, where R and S are both Hermitian
matrices. We have Solution: We have
Aq = (R + iS)q = + Rq (iS)q
= + i Rq Sq =
- Rq iSq det (ATA) = det(I) = 1
= R - iS (since R and S are both Hermitian matrices) This implies that

Therefore, [det(A)]2 = 1 or det(A) = ± 1


A + Aq = (R + iS) + (R - iS) = 2R Now,
1 det(A) = 3abc - (a3 + b3 + c3)
⇒ R = (A + Aq ) = P 1
2 2 2 2
= - (a + b + c )[(a - b ) + (b - c ) + (c - a) ] < 0
Also, 2
A - Aq = (R + iS) - (R - iS) = 2iS as a, b, c are positive.

Mathematical Problem Book for JEE.indb 757 07-06-2018 12:45:41


758 Mathematics Problem Book for JEE

Hence,
a 1 a2
det(A) = -1
⇒ 3abc - (a3 + b3 + c3) = -1   |B3| = 0 d 0 = a2df
⇒ a3 + b3 + c3 = 4 f g 0

 8. If M is a 3 × 3 matrix, where MTM = I and det(M) = 1, then prove If adf ≠ 0, then |B2| = |B3| ≠ 0. Hence, no solution exists.
that det(M - I) = 0.
10.  Show that if A and B are symmetric and commute, then
Solution:
(a)  A-1B (b) AB-1
(M - I)T = MT - I = MT - MTM = MT (I - M)
⇒ |(M - I)T| = |M - I| = |MT| |I - M| = |I - M| (c) A-1B-1are symmetric.
⇒ |M - I| = 0 Solution:
Alternate method (a)  Since A and B commute: AB = BA
det(M - I) = det(M - I) det(MT) = det(MMT - MT) Pre- and post-multiplying both sides by A-1, we get
= det(I - MT) = - det(MT - I) = - det(M - I)T = -det(M - I)
A-1(AB)A-1 = A-1(BA)A-1
⇒ det(M - I) = 0
⇒ (A-1A)(BA-1) = A-1B(AA-1) (by associativity)
a2  ⇒ I(BA-1) = (A-1B)I
a 0 1 a 1 1 f 
        ⇒ BA-1 = A-1B
 9. A = 1 c b  , B  0 d c  , U  g  , V   0  . If there is a Now,
     
1 d b  f g h  h  0 
  (A-1B)′ = (BA-1′ = (A-1)′B′ (by reversal law)
vector matrix X, such that AX = U has infinitely many solutions, = A-1B [as B′ = B (symmetric) and (A-1)′ = (A′)-1 = A-1]
then prove that BX = V cannot have a unique solution. If afd ≠ Hence, A-1B is symmetric.
0 then prove that BX = V has no solution. (b)  Pre-and post-multiplying by B-1, we get
Solution: AX = U has infinite solutions. This implies |A| = 0 which B-1(AB)B-1 = B-1(BA)B-1
gives ⇒ (B-1A)BB-1 = B-1B(AB-1)
a 0 1 ⇒ B-1A = AB-1
1 c b = 0 ⇒ ab = 1 or c = d Now,
1 d b (AB-1)′= (B-1A)′ = A′B-1)′
= AB-1
[as A = A′ (symmetric) and (B-1)′ = (B′)-1 = B-1]
and -1
Hence, AB is symmetric.
a 0 f
(c) Since A and B are symmetric, we have
|A1| = 1 c g = 0 ⇒ g = h
AB = BA
1 d h
⇒ (BA) -1 = (AB) -1
a f 1 ⇒ A-1B-1 = B-1A-1
|A2| = 1 g b = 0 ⇒ g = h ⇒ (A-1B-1)′ = (B-1A-1)′ = (A-1)′ ⋅ (B-1)′ = A-1B-1
1 h b [as (A-1)′ = A-1 and (B-1)′ = B-1]

f 0 1 Hence, A-1B-1 is symmetric.

|A3| = g c b =0 11. Let a > 0, d > 0. Find the value of the determinant


h d b 1 1 1
⇒ g = h, c = d ⇒ c = d and g = h a a(a + d ) (a + d )(a + 2d )
Now BX = V. 1 1 1
a 1 1 (a + d ) (a + d )(a + 2d ) (a + 2d )(a + 3d )
1 1 1
|B| = 0 d c = 0 (since C2 and C3 are equal)
( a + 2d ) (a + 2d )(a + 3d ) (a + 3d )(a + 4 d )
f g h
Solution:
This means BX = V has no unique solution. 1 1 1
2 a a(a + d ) (a + d )(a + 2d )
a 1 1
1 1 1
  |B1| = 0 d c = 0        
(since c = d, g = h) D=
(a + d ) (a + d )(a + 2d ) (a + 2d )(a + 3d )
0 g h
1 1 1
( a + 2d ) (a + 2d )(a + 3d ) (a + 3d )(a + 4 d )
a a2 1
  |B2| = 0 0 c = a2cf = a2df (since c = d ) Take common 1
f 0 h from R1
a(a + d )(a + 2d )

Mathematical Problem Book for JEE.indb 758 07-06-2018 12:45:54


Chapter 18 | Matrices and Determinants 759

1 Now, by using Cramer’s rule


(a + d )(a + 2d )(a + 3d ) from R2 a  b  c  0  x : y : z  1: 1: 1
1 or a  bw  cw 2  0  x : y : z  1: w : w 2
from R3
(a + 2d )(a + 3d )(a + 4 d ) or a  bw 2  cw  0  x : y : z  1: w 2 : w
(a + d )(a + 2d ) a + 2d a x3 +1 x2 x
1
∆= (a + 2d )(a + 3d ) a + 3d a + d 3 2
a(a + d )2 (a + 2d )3 (a + 3d )2 (a + 4 d ) 14. If y + 1 y y = 0 and x, y, z are all different, then prove
(a + 3d )(a + 4 d ) a + 4 d a + 2d 3 2
z +1 z z
Applying R2 → R2 - R1, R3 → R3 - R2 in D′ where
that xyz = -1.
1 Solution:
D= D′,
a(a + d )2 (a + 2d )3 (a + 3d )2 (a + 4 d )
x3 x2 x 1 x2 x x2 x 1 1 x2 x
we get 3 2 2 2 2
LHS =  y y y +1 y y = xyz y y 1+1 y y
D′ = 4d4
3 2 2 2 2
z z z 1 z z z z 1 1 z z
4d 4
⇒D=
a(a + d ) (a + 2d )3 (a + 3d )2 (a + 4 d )
2
x2 x 1
2
12. Given that a = cosq + i sinq, b = cos 2q - i sin 2q, c = cos 3q = ( xyz +1) y y 1
a b c z2 z 1
+ i sin 3q and if b c a = 0 , show that q = 2np, n ∈ Z.
Operate R3 - R1 and R2 - R1 on the determinants. We get
c a b
Solution: x2 x 1
2 2
a b c LHS = ( xyz + 1) y - x y-x 0
1 2 2
∆ = b c a = - (a + b + c ) ((a - b )2 + (b - c )2 + (c - a)2 ) = 0 z -x z-x 0
2
c a b
= (xyz + 1)(x + y)(x - z)(z - x) = 0, given x ≠ y ≠ z
⇒ a + b + c = 0 or a = b = c ⇒ xyz + 1 = 0 ⇒ xyz = -1
If a + b + c = 0, we have 15. If a2 + b2 + c2 = 1, prove that
cosq + cos2q + cos 3q = 0,
sinq - sin 2q + sin 3q = 0
a2 + (b2 + c 2 )cos f ab(1- cos f ) ac (1- cos f )
This gives ba(1- cos f ) b2 + (c 2 + a2 )cos f bc (1- cos f )
cos 2q (2cosq + 1) = 0
ca(1- cos f ) cb(1- cos f ) c 2 + (a2 + b2 )cos f
and sin 2q (1 - 2 cosq ) = 0 (1)
which is not possible as cos 2q = 0 gives sin 2q ≠ 0, cosq ≠ 1/2 and is independent of a, b and c.
cosq  = -1/2 gives sin 2q ≠ 0, cosq ≠ 1/2.
Therefore, Eq. (1) does not hold simultaneously, and so Solution: Let

a+b+c≠0 a2 + (b2 + c 2 )cos f ab(1- cos f ) ac (1- cos f )


Therefore, 2 2 2
a = b = c or eiq = e-2iq = e3iq ∆= ba(1- cos f ) b + (c + a )cos f bc (1- cos f )
which is satisfied only by eiq = 1, i.e. cosq = 1, sinq = 0. So ca(1- cos f ) cb(1- cos f ) c + (a2 + b2 )cos f
2

q = 2np, n∈ Z.
Multiplying C1, C2, C3 by a, b, c, respectively, and taking a, b, c
13. If x, y, z are not all zero and if ax + by + cz = 0, bx + cy + az = 0, common from R1, R2, R3, respectively, we get
cx + ay + bz = 0, prove that x:y:z = 1:1:1 or 1:w :w 2 or 1:w 2:w,
a2 + (b2 + c 2 )cos f b2 (1- cos f ) c 2 (1- cos f )
where w  is the complex cube roots of unity. abc
∆= a2 (1- cos f ) b2 + (c 2 + a2 )cos f c 2 (1- cos f )
Solution: For non-trivial solution, abc
a2 (1- cos f ) b2 (1- cos f ) c 2 + (a2 + b2 )cos f
a b c
Applying C1 → C1 + C2 + C3 we have
b c a =0
c a b a2 + b 2 + c 2 b2 (1- cos f ) c 2 (1- cos f )
D = a2 + b 2 + c 2 b2 + (c 2 + a2 )cos f c 2 (1- cos f )
⇒ (a + b + c) + + - ab - bc - ca) = 0 
(a2 b2 c2 (1)
a2 + b 2 + c 2 b2 (1- cos f ) c 2 + (a2 + b2 )cos f
⇒ (a + b + c) (a + w b + w 2c)(a + w 2b + w c) = 0 (2)

Mathematical Problem Book for JEE.indb 759 07-06-2018 12:46:12


760 Mathematics Problem Book for JEE

Taking a2 + b2 + c2 common from C1, we get n 1 n 1 n 1


C r 1 Cr C r 1
2 2
1 b (1- cos f ) c (1- cos f ) n(n  1)(n  2) n n n
 C r 1 Cr C r 1
D = (a2 + b2 + c2) 1 b2 + (c 2 + a2 )cos f c 2 (1- cos f ) r ( r  1)( r  2) n 1 n 1 n 1
C r 1 Cr C r 1
2 2 2 2
1 b (1- cos f ) c + (a + b )cos f n 2
C3
Applying R2 → R2 - R1 and R3 → R3 - R1, we get  r 2
(n  1, r  1)
C3
1 b2 (1- cos f ) c 2 (1- cos f )   
D=1 18. If a, b, c are coplanar vectors, then show that
2 2 2 
0 (b + c + a )cos f 0  
a b c
0 0 (c + a + b2 )cos f
2 2
      
a  a a b a  c  0
(since a2 + b2 + c2 = 1)      
b  a b b b  c
= +
( a2 + b2 c2)2 cos2f
{by property since all elements are zero
  
below leading diagonal} Solution: Since a , b , c are coplanar there must exist k1, k2, k3 not
  
= 12 cos2f = cos2f, which is independent of a, b and c. all zero (say k1 ≠ 0) such that k1a + k2b + k3c = 0 . Now by operating
16. If the system of equations x = cy + bz, y = az + cx and z = bx + ay C1→ k1C1 + k2C2 + k3C3 , we have
    
has a non-zero solution and at least one of a, b, c is a proper k1a + k2b + k3c b c
fraction, prove that a2 + b2 + c2 < 3 and abc > -1. 1          
LHS = k1a × a + k2a × b + k3a × c a × b a × c (k ≠ 0)
Solution: We are given that system of equations has non-trivial k1           1
k1b × a + k2b × b + k3b × c b × b b × c
solution. So
1 - c -b     
k1a + k2b + k3c b c
-c 1 -a = 0        
= 1 a × [k1a + k2b + k3c ] a × b a × c
-b - a 1 k1        
b × [k1a + k2b + k3c ] b × b b × c
⇒ 1( 1 - a2) + c(-c - ab) - b(ac + b) = 0   
0 b c
⇒ 1 - 2abc - a2 - b2 - c2 = 0 or a2 + b2 + c2 + 2abc = 1 1      
= 0 a ×b a ×c = 0
⇒ a2 + b2c2 + 2abc = 1 - b2 - c2 + b2c2 = (1 - b2)(1 - c2) k1     
0 b ×b b ×c
⇒ (a + bc)2 = (1 - b2)(1 - c2)
Similarly, 19. Let a, b, c be real numbers with a2 + b2 + c2 = 1. Show that the
equation
(b + ac)2 = (1 - a2)(1 - b2) and (c + ab)2 = (1 - a2)(1 - b2)
ax - by - c bx + ay cx + a
Hence, (1 - a2), (1 - b2) and (1 - c2) all have same sign. Since at least bx + ay -ax + by - c cy + b =0
one of them is proper fraction, it implies all of them are positive. So
cx + a cy + b -ax - by + c
1 - a2 > 0, 1 - b2 > 0, 1 - c2 > 0
represents a straight line.
⇒ a2 + b2 + c2 < 3 ⇒ 1 - 2abc < 3 ⇒ abc > -1
Solution: Given
17. If numbers n, r are two different positive integers such that
ax  by  c bx  ay cx  a
n ≥ r + 2 and it is given that
n n n
bx  ay  ax  by  c cy  b 0
Cr C r +1 Cr +2
cx  a cy  b  ax  by  c
D(n, r) = n +1
Cr n +1
C r +1 n +1
Cr +2
Using C1 → aC1 + bC2 + cC3 gives
n+2 n+2 n+2
Cr C r +1 Cr +2
( a2  b 2  c 2 ) x bx  ay cx  a
then show that
2 2 2
n+2
C3 (a  b  c ) y  ax  by  c cy  b 0
D(n, r) = r +2 D(n - 1, r - 1)
C3 ( a2  b 2  c 2 ) cy  b a
ax  by  c
Solution: We know that
m x bx + ay cx + a
mC = m-1C
k
k k-1 Þ y -ax + by - c cy + b =0
Now
1 cy + b -ax - by + c
n n -1 n n -1 n n -1
C r -1 Cr C r +1
r r +1 r +2 Applying C2→ C2 - bC1 and C3 → C3 - cC1, we get
D(n, r) = n + 1 nC n +1 n
Cr
n+2 n
C r +1
r -1 x ay a
r r +1 r +2
n + 2 n +1 n + 2 n +1 n + 2 n +1 D = y -ax - c b =0
C r -1 Cr C r +1
r r +1 r +2 1 cy -ax - by

Mathematical Problem Book for JEE.indb 760 07-06-2018 12:46:32


Chapter 18 | Matrices and Determinants 761

Applying R3 → R3 + xR1 + yR2, we get 1 0 0


x ay a 1 x 0 = xy
D= y -ax - c b = 0 1 0 y

x2 + y2 +1 0 0 Hence, D is divisible by both x and y.


Hence, the correct answer is option (B).
⇒ (x2 + y2 +1)(aby + a2x + ac) = 0
⇒ ax + by + c = 0 5 5a a 
 
as (x2 + y2 +1) ≠ 0 being sum of three positive numbers.  2. Let A  0 a 5a  . If A2 = 25, then a equals
 
20. If f(x) is a polynomial of degree < 3, then prove that 0 0 5 

f (a) (A)  52 (B)   1 (C) 1/5 (D)   5


1 a [AIEEE 2007]
( x  a) 1 a a2 Solution: We have
f (b ) f (x)
1 b  1 b b2  5 5a a  5 5a a 
( x  b) ( x  a)( x  b )( x  c )   
f (c ) 1 c c2 A2  0 a 5a  0 a 5a 
1 c   
x c 0 0 5  0 0 5 

25 25a  5a
2
5a  25a 2  5a 
Solution:
 
 0 a2 5a 2  25a 
f (x) A B C  
= + +   (1)  0 0 25 
( x - a)( x - b )( x - c ) ( x - a) ( x - b ) ( x - c )
2
f (a) f (b ) f (c )  A  25(25a 2 )  62525a 22
A=- , B=- and C = -
(a - b )(c - a) (a - b )(b - c ) (b - c )(c - a)  25  62525a 22
1
Therefore, a 
5
( c - b )f ( a) ( c - a)f (b ) (b - a)f ( c )
- + Hence, the correct answer is option (C).
f (x) ( x - a) ( x - b) (x - c)
=  3. 
Let A be a 2 × 2 matrix with real entries. Let I be the 2 × 2
( x - a)( x - b )( x - c ) (a - b )(b - c )(c - a)
identity matrix. Denote by tr (A), the sum of diagonal entries
f (a) of A. Assume that A2 = I.
1 a
( x  a ) 1 a a2 Statement-1: If A ≠ I and A ≠ - I, then det A = - 1.
f (b ) Statement-2: If A ≠ I and A ≠ - I, then tr (A) ≠ 0.
 1 b  1 b b2
( x  b)
1 c c2 (A)  Statement-1 is false, Statement-2 is true.
f (c )
1 c (B) Statement-1 is true, Statement-2 is true; Statement-2 is a
(x  c)
correct explanation for Statement-1.
(C) Statement-1 is true, Statement-2 is true; Statement-2 is
not a correct explanation for Statement-1.
Previous Years' Solved JEE Main/AIEEE (D)  Statement-1 is true, Statement-2 is false.
Questions [AIEEE 2008]
Solution: Let us consider that
1 1 1 a b 
 1. If D = 1 1+ x 1 for x ≠ 0, y ≠ 0 then D is A 

c d 
1 1 1+ y Therefore,
(A)  divisible by neither x nor y  (B)  divisible by both x and y é a2 + bc ab + bd ù é 1 0 ù
A2 = ê ú=ê ú
(C)  divisible by x but not y   (D)  divisible by y but not x êëac + dc bc + d 2 úû ë0 1û
[AIEEE 2007]
2 2
Solution: We have ⇒ a + bc = 1 = bc + d ;
1 1 1 and (a  d )c  0  (a  d )b
D = 1 1+ x 1 As A ≠ I, A ≠ -I, a = -d, we have
1 1 1+ y
1- bc b
det A = = -1+ bc - bc = -1
Therefore, C2 ® C2 - C1 and C3 ® C3 - C1 imply that c - 1- bc

Mathematical Problem Book for JEE.indb 761 07-06-2018 12:46:49


a a 1 a  1 a 1 b 1 c  1
LHS  b b  1 b  1  ( 1)n a  1 b  1 c  1
c c  1 c 1 a b c

a a 1 a  1 a 1 a  1 a
762 Mathematics Problem Book for JEE n
 b b  1 b  1  ( 1) b  1 b  1 b
c c  1 c 1 c  1 c 1 c
Therefore, Statement-1 is true. However, tr(A) = 0 and therefore a a 1 a  1 a 1 a a  1
Statement-2 is false.
 b b  1 b  1  ( 1)n 1 b  1 b b  1
Hence, the correct answer is option (D).
c c  1 c 1 c  1 c c 1
 4. 
Let a, b, c be any real numbers. Suppose that there are real
a a 1 a  1 a a 1 a  1
numbers x, y, z not all zero such that x = cy + bz, y = az + cx and
z = bx + ay. Then a2 + b2 + c2 + 2abc is equal to  b b  1 b  1  ( 1)n 2 b b  1 b  1 (C1  C2 )
c c  1 c 1 c c  1 c 1
(A)   2 (B)   -1 (C)   0
(D)   1
[AIEEE 2008] Now,
Solution: The system of equations x - cy - bz = 0, cx - y + az = 0 a a +1 a -1
and bx + ay - z = 0 have non-trivial solution if LHS = -b b + 1 b - 1 1+ ( -1)n + 2  = 0
c c -1 c +1
1 -c -b
c -1 a = 0 ⇒ 4b(a + c )(1+ ( -1)n + 2 ) = 0
b a -1 ⇒ 4b(a + c ) ≠ 0 ⇒ 1+ ( -1)n + 2 = 0

⇒ 1 (1- a2 ) + c ( -c - ab ) - b(ca + b ) = 0 which is true only if n + 2 is odd, that is, n is odd integer.
2 2 2
⇒ a + b + c + 2abc = 1 Hence, the correct answer is option (C).
 7. Let A be a 2 × 2 matrix
Hence, the correct answer is option (D). Statement-1: adj(adj A)= A
 5. 
Let A be a square matrix all of whose entries are integers. Statement-2:  adj A = A
Then which one of the following is true? (A) Statement-1 is true, Statement-2 is true; Statement-2 is a
(A)  If det(A) = ± 1, then A-1 exists but all its entries are not correct explanation for Statement-1.
necessarily integers. (B) Statement-1 is true, Statement-2 is true; Statement-2 is
(B) If det(A) ≠ ± 1, then A-1 exists and all its entries are non- not a correct explanation for Statement-1.
integers. (C) Statement-1 is true, Statement-2 is false.
(C) If det(A) = ± 1, then A-1 exists and all its entries are integers. (D) Statement-1 is false, Statement-2 is true.
(D)  If det(A) = ± 1, then A-1 need not exist. [AIEEE 2009]
[AIEEE 2008] Solution: We have
Solution: It is given that each entry of A is integer. Therefore, n -1 2 -1
adj A = A = A = A
the cofactor of every entry is an integer and so each entry in the n -2 0
adjoint of matrix A is an integer. So Now, adj (adj A) = A A= A A= A
Hence, the correct answer is option (B).
1
det A = ±1 and A-1 = (adj A)  8. 
The number of 3 × 3 non-singular matrices, with four entries
det( A) as 1 and all other entries as 0, is
(A) 5 (B) 6
This implies that all entries in A-1 are integers. (C)  at least 7 (D)  less than 4
Hence, the correct answer is option (C). [AIEEE 2010]
Solution: Let us consider the following matrix:
Let a, b, c be such that b(a + c) ≠ 0. If
 6.  1 X X
 
a a +1 a -1 a +1 b +1 c -1 X 1 X
 
-b b + 1 b - 1 + a - 1 b -1 c + 1 = 0 , then the X X 1 
c c - 1 c + 1 ( -1)n + 2 a ( -1)n +1b (-
-1)n c which are six non-singular matrices because six blanks (i.e. X) can
value of ‘n’ is be filled by five zeros and one 1. In the same manner, we have the
(A) zero (B)  any even integer matrix
(C)  any odd integer (D)  any integer X X 1 
[AIEEE 2009]  
X 1 X 
Solution: We have  
 1 X X 
a a 1 a  1 a 1 b 1 c  1
n
LHS  b b  1 b  1  ( 1) a  1 b  1 c  1 which are six non-singular matrices. Therefore, in the required
c c  1 c 1 a b c case, there are more than 7.
Hence, the correct answer is option (C).
a a 1 a  1 a 1 a  1 a
Let A be a 2 × 2 matrix with non-zero entries and let A2 = I,
 9. 
 b b  1 b  1  ( 1)n b  1 b  1 b where I is 2 × 2 identity matrix. Define tr (A) = sum of diagonal
c c  1 c 1 c  1 c 1 c elements of A and A = determinant of matrix A.

a a 1 a  1 a 1 a a  1
 b b  1 b  1  ( 1)n 1 b  1 b b  1
c c  1 c 1 c  1 c c 1
Mathematical Problem Book for JEE.indb 762 07-06-2018 12:47:01
Chapter 18 | Matrices and Determinants 763

Statement-1:  tr(A) = 0 1 0 0 
 
Statement-2:  A  1 Let A  2 1 0  . If u1 and u2 are column matrices such
12. 
 
(A)  Statement-1 is true, Statement-2 is true; Statement-2 is 3 2 1
not a correct explanation for Statement-1. 1 0 
(B) Statement-1 is true, Statement-2 is false.    
that Au1  0  and Au2  1 , then u1 + u2 is equal to
(C) Statement-1 is false, Statement-2 is true.    
0  0 
(D) Statement-1 is true, Statement-2 is true; Statement-2 is a
correct explanation for Statement-1. 1 1
   
[AIEEE 2010] (B)   1 
(A)   1   
   
Solution: Let  0  
1
 a b 1
A =   , a, b , c , d ≠ 0 1 
 c d    
(C)  1  (D)  1
Therefore,    
a2  bc ab   bd   0  1
a 2b  aa bb a b2 a22 bc
A2   A   A   A   ab  bd  
[AIEEE 2012]
c d c
c dd  c d  
 
   d 2 
2bc
      ac  cd ac  cd
 bc  d   Solution: We have
2
 a2 
bca 1, bc  1d, 2bc d 2  1aband
 1 and  acbd cd
 bdab  ac
 0 cd  0 1 0 0 
 
A  2 1 0 
Therefore, c ≠ 0 and b ≠ 0 Þ a + d = 0. Trace A = a + d = 0. Thus,  

3 2 1
A = ad - bc = -a2 - bc = -1.
Hence, the correct answer is option (B). a  d 
   
10.  The number of values of k for which the linear equations Let us consider that u1  b  ; u2  e  . Therefore
   
4x + ky+ 2z = 0 c  f 
kx+ 4y + z = 0 1 1 
   
2x + 2y + z = 0 Au1  0   u1  2 
possess a non-zero solution is    
0   1 
(A)  2 (B) 1
(C) zero (D) 3 0  0 
   
[AIEEE 2011] Au2  1  u2   1 
   
Solution: 0  2 
4 k 2
Therefore,
k 4 1  0  k 2  6k  8  0  k  4 and 2 1 
2 2 1  
u1  u2  1
 
Hence, the correct answer is option (A). 1
11. 
Let A and B be two symmetric matrices of order 3. Hence, the correct answer is option (D).
Statement-1: A(BA) and (AB)A are symmetric matrices.
Statement-2: AB is symmetric matrix if matrix multiplication Let P and Q be 3 × 3 matrices with P ≠ Q. If P3 = Q3 and
13. 
of A and B is commutative. P2Q = Q2P, then determinant of (P2 + Q2) is equal to
(A)  Statement-1 is true, Statement-2 is true; Statement-2 is (A)  -2  
(B) 1 (C) 0 (D)  -1
not a correct explanation for Statement-1. [AIEEE 2012]
(B)  Statement-1 is true, Statement-2 is false. Solution: We have P 3 = Q 3. Therefore,
(C)  Statement-1 is false, Statement-2 is true. P 3  P 2Q  Q 3  Q 2 P  P 2 ( P  Q )  Q 2 ( Q  P )
(D) Statement-1 is true, Statement-2 is true; Statement-2 is a
correct explanation for Statement-1.  P 2 (P  Q )  Q2 (P  Q )  O
[AIEEE 2011]  ( P 2  Q 2 )( P  Q )  O
Solution: We have, AT = A and B T = B. Therefore,  | P 2  Q 2 | 0
T T T T T
( A(BA)) = (BA) A = ( A B ) A = ( AB ) A = A(BA) Hence, the correct answer is option (C).
T T T T T
and (( AB ) A) = A ( AB ) = A(B A ) = A(BA) = ( AB ) A
and    14. If a, b ≠ 0, and f(n) = a n + b n and
Therefore, Statement-1 is correct. 3 1+ f (1) 1+ f (2)
Also ( AB )T = B T AT = BA = AB (since AB is commutative) 1+ f (1) 1+ f (2) 1+ f (3) = K (1- a )2 (1- b )2 (a - b )2 ,
Therefore, Statement-2 is also correct, but it is not a correct expla- 1 + f ( 2 ) 1 + f ( 3) 1 + f ( 4 )
nation of Statement-1.
Hence, the correct answer is option (A). then K is equal to

Mathematical Problem Book for JEE.indb 763 07-06-2018 12:47:20


764 Mathematics Problem Book for JEE

1 Then A-1 is
(A)  1 (B)  -1 (C)  ab (D)  3 1 2  3 2 1
ab    
[JEE MAIN 2014 (OFFLINE)] (A)  3 0 2 (B)  3 2 0 
 
 
1 0 1 1 1 0 
Solution:
1+ 1+ 1 1+ a + b 1+ a 2 + b 2 1 1 1 1 1 1 0 1 3 1 2 3
2 2 3 3    
1+ a + b 1+ a + b 1+ a + b =1 a b 1 a a2 (C)  0 2 3 (D)  0 1 1
   
1+ a 2 + b 2 1+ a 3 + b 3 1+ a 4 + b 4 1 a2 b2 1 b b2 1 1 1 0 2 3
= {(1- a) (1- b ) (a - b )2} [JEE MAIN 2014 (ONLINE SET 2)]
On comparison with the given equation, we get K = 1. Solution:
Hence, the correct answer is option (A). 1 2 3 0 0 1
   
A 0 2 3  1 0 0 
15. If A is a 3 × 3 non-singular matrix such that AA′ = A′A and    
0 1 1 0 1 0 
B = A-1A′, then BB′ equals
(A)  B-1 (B) (B-1) (C)  I + B (D)  I Applying C1 ↔ C3 on both matrices we get
[JEE MAIN 2014 (OFFLINE)]
Solution: 3 2 1 1 0 0 
   
BB′ = A-1A′(A-1A′)′ = A-1A′A(A-1)′ A 3 2 0   0 0 1
   
 = A-1AA′(A-1)′ = IA′(A-1)′ = I(A-1A)′ = I⋅I′ = I2 = I 1 1 0  0 1 0 

Hence, the correct answer is option (D). Applying C2 ↔ C3 on both matrices we get
16. 
If a, b, c are non-zero real numbers and if the system of 3 1 2  1 0 0  3 1 2 
equations      
A 3 0 2   0 1 0  ⇒ A1  3 0 2 
(a - 1) x = y + z      
(b - 1) y = z + x 1 0 1 0 0 1 1 0 1
(c - 1) z = x + y
Hence, the correct answer is option (A).
has a non-trivial solution, then ab + bc + ca equals
(A)  a + b + c   (B)  abc   (C) 1 (D)  -1 Let for i = 1, 2, 3, pi(x) be a polynomial of degree 2 in x,
18. 
[JEE MAIN 2014 (ONLINE SET 1)] pi ′ ( x ) and pi ′′ ( x ) be the first-and second-order derivatives of
Solution: For the non-trivial solution p ( x ) p ( x ) p ( x ) 
 1 1 1 
1- a 1 1  
pi(x), respectively. Let A( x )  p2 ( x ) p2( x ) p2( x )
1 1- b 1 =0  
p3 ( x ) p3( x ) p3( x )
1 1 1- c  
Þ (1- a){(1- b )(1- c ) - 1} - 11
( - c - 1) + 11
( - 1+ b ) and B(x) = [A(x)]T A(x). Then, the determinant of B(x)
Þ (1- a){1- c - b + bc - 1} + c + b = 0 (A)  is a polynomial of degree 6 in x.
Þ - c - b + bc + ac + ab - abc + c + b = 0 (B)  is a polynomial of degree 3 in x.
⇒ ab + bc + ca = abc (C)  is a polynomial of degree 2 in x.
Hence, the correct answer is option (B). (D)  does not depend on x.
[JEE MAIN 2014 (ONLINE SET 2)]
If B is a 3 × 3 matrix such that B2 = 0, then det[(I + B)50 - 50B]
17.  Solution:
is equal to 2
B( x ) = A( x ) A( x ) = A( x )
(A)  1 (B) 2 (C) 3 (D) 50
[JEE MAIN 2014 (ONLINE SET 1)] Now highest power in the determinant of A(x) can be 3, as pi ¢ ( x )
Solution: is of degree 1 and pi ¢¢ ( x ) is constant. Hence, B( x ) must have
det [(I + B) 50 - 50 B]⇒[(I + B) 50 = I + 50 B] maximum degree 6.
[using induction process (I + B)n = I + nB) (assuming B2 = 0)] Hence, the correct answer is option (A).
Therefore,
det[(I + B)50 - 50B]= det[I + 50B - 50B] = 1 a2 b2 c2 a2 b2 c2
Hence, the correct answer is option (A). If (a + l )2 (b + l )2
19.  (c + l )2 = k l a b c , l ≠ 0, then
18.  Let A be a 3 × 3 matrix such that (a - l )2 (b - l )2 (c - l )2 1 1 1

1 2 3 0 0 1 k is equal to
   
A 0 2 3  1 0 0  (A)  4labc (B)  -4labc (C) 4l 2 (D)  -4l 2
   
0 1 1 0 1 0  [JEE MAIN 2014 (ONLINE SET 3)]

Mathematical Problem Book for JEE.indb 764 07-06-2018 12:47:36


Chapter 18 | Matrices and Determinants 765

Solution:
a2 b2 c2 r 2r - 1 3r - 2
a2 b2 c2 n -1
n
(a + l )2 (b + l )2 (c + l )2 = 4 al 4 bl 4cl 22. 
If 2
n -1 a , then the value of ∑ ∆r
r =1
(a - l )2 (b - l )2 (c - l )2 l 2 - 2al l 2 - 2bl l 2 - 2c l 1 1
n(n - 1) (n - 1)2 (n - 1)(3n + 4 )
(R2 → R2 - R3 and R3 → R3 - R1) 2 2
(A)  depends only on a
a2 b2 c2
(B)  depends only on n
= 4l a b c (C)  depends both on a and n
2
l - 2al 2
l - 2bl 2
l - 2c l (D)  is independent of both a and n
ìa 2
b2
c2
a 2
b2 c2 ü [JEE MAIN 2014 (ONLINE SET 4)]
ïï ïï
= 4l í a b c + a b c ý Solution:
ï 2 -2al -2bl -2c l ïï
ïî l l2 l2 þ n -1

 a2 
∑ ∆ r = ∆1 + ∆2 + + ∆ n -1
 b2 c2  r =1
 
 4l 3  a b c  0
  1 1 1
 1 1 1 
(since, two rows are proportional) n
= n -1 a
2 2 2 2 2 2 2
a
b c a b c
n æ n - 1ö
 4l 3 a
b 4 lc3 (asince,b k l c 4(l 3
 kk l 4l42l)3  k  4 l 2 )
since, (n - 1) (n - 1)2 ç ÷ (3n + 4 )
2 è 2 ø
1
1 1 1 1 1

Hence, the correct answer is option (C). 2 3 4
y  n
1 2 x    6  + n -1 a
 and B  x  be such that AB    , then 2
20.  If A  
3 1 2    8  n æ n - 1ö
1  (n - 1) (n - 1)2 ç ÷ (3n + 4 )
2 è 2 ø
(A)  y = 2x (B) y = - 2x (C)  y = x (D)  y = - x
[JEE MAIN 2014 (ONLINE SET 3)]
Solution: We have n  1 2(n  1)  1 3(n  1)  2
6  n
AB      n 1 a
8  2
Therefore, n  n  1
(n  1) (n  1)2   (3
3n  4 )
y  2  2 
1 2 x    6  y  2 x  x  6 
  x        
3 1 2 23   8  3 y  x  2  8 
1 31 1 2   n  1 1 3  2( n  1)  1 1 4  3(n  1)  2
Thus, n
 n 1 a
y + 3 x = 6 and 3 y - x = 6 2
⇒y+3x=3y-x n  n  1
⇒2y=4x ( n  1) ( n  1)2   (3n  4 )
⇒y=2x
2  2 
n æ n - 1ö
Hence, the correct answer is option (A). (n - 1) (n - 1)2 ç ÷ (3n - 4 )
2 è 2 ø
Let A and B be any two 3 × 3 matrices. If A is symmetric and B
21.  n
is skew-symmetric, then the matrix AB - BA is = n -1 a
2
(A)  skew-symmetric
n(n - 1) 1
(B) symmetric (n - 1)2 (n - 1)(3n + 4)
(C)  neither symmetric nor skew-symmetric 2 2
(D) I or - I, where I is an identity matrix  n  1
0 0 
  ( 3n  4  3n  4 )
[JEE MAIN 2014 (ONLINE SET 4)]  2 
Solution: We have n
 n 1 a (R1  R1  R2 )
(AB - BA)′ = (AB)′ - (BA)′ = B′A′ - A′B′ = (-B)(A) - A(-B) = AB - BA 2
Therefore, AB - BA is symmetric. n(n  1) 1
(n  1)2 (n  1)(3n  4 )
Hence, the correct answer is option (B). 2 2

Mathematical Problem Book for JEE.indb 765 07-06-2018 12:47:52


766 Mathematics Problem Book for JEE

-8(n - 1) ì n n(n - 1) ü 25. 


The least value of the product xyz for which the determinant
2
= í (n - 1) - (n - 1) ý=0
2 î 2 2 þ x 1 1
1 y 1 is non-negative is
Hence, the correct answer is option (D).
1 1 z
 1 2 2
 
23. If A  2 1 2  is a matrix satisfying the equation AAT = 9I, (A)  -2 2 (B)  -16 2
 
a 2 b  (C) −8 (D) −1
where I is a 3 × 3 identity matrix, then the ordered pair (a, b) [JEE MAIN 2015 (ONLINE SET 1)]
is equal to Solution: To find the least value of xyz where
(A)  (−1, 1) (B)  (2, 1) x 1 1
(C)  (−2, −1) (D)  (2, −1) 1 y 1 ≥0
[JEE MAIN 2015 (OFFLINE)]
Solution: We have 1 1 z
1 2 2  ⇒ x(yz - 1) - 1 (z - 1) + (1 - y) = 0
 
A  2 1 2  ⇒ xyz - x - y - z + 2 ≥ 0
  ⇒ xyz ≥ x + y + z - 2  (1)
a 2 b 
For x, y, z,
A AT  9I
x+y+z 3
1 2 2  1 2 a  9 0 0  ³ xyz ⇒ x + y + z ≥ 3 3 xyz (2)
     3
 2 1 2  2 1 2   0 9 0  From Eqs. (1) and (2)
    
a 2 b  2 2 b  0 0 9 
xyz  3 3 xyz  2
 9 0 (a  4  2b )  4 0 0  ⇒ t3 ≥ 3t - 2 where t = 3 xyz
  

 0 9 (2a  2  2b )  0 9 0  ⇒ t3 - 3t + 2 ≥ 0 ⇒ (t - 1) (t2 + t - 2) ≥ 0
  ⇒ (t - 1) (t + 2) (t - 1) ≥ 0
 2 
(a  4  2b ) (2a  2  2b ) (a  4  b )  0 0 9 
2
⇒ (t - 1)2 (t + 2) ≥ 0 ⇒ t + 2 ≥ 0 ⇒ t ≥ −2
⇒ a + 2b = −4; a2 + b2 + 4 = 9; 2a - 2b = −2; ⇒ 3 xyz ³ -2 ⇒ xyz ≥ −8
⇒ a = −2, b = −1 ⇒ (a, b) ≡ (−2, −1) Hence, the correct answer is option (C).
Hence, the correct answer is option (C).
24. The set of all values of l for which the system of linear 0 1
equations If A  
26.   , then which one of the following statements is
1 0 
2x1 - 2x2 + x3 = lx1
not correct?
2x1 - 3x2 + 2x3 = lx2
(A)  A4 - I = A2 + I (B)  A3 - I = A(A - I)
−x1 + 2x2 = lx3 (C)  A2 + I = A(A2 - I) (D)  A3 + I = A(A3 - I)
has a non-trivial solution [JEE MAIN 2015 (ONLINE SET 1)]
(A) is a singleton. Solution:
(B) contains two elements.
(C) contains more than two elements. 0 1 0 1 0 1 1 0 
A  A2     ⇒ A2     I
(D) is an empty set. 1 0  ⇒ 
1 0 
1 0 
  0 1
[JEE MAIN 2015 (OFFLINE)] Statement (A): A4 - I = (−I)2 - I = I - I = 0 and A2 + I = 0. So, option
Solution: The system of linear equation (A) is correct.
2x1 - 2x2 + x3 = lx1 Statement (B): A3 - I = A⋅A2 - I = A(−I) - I = − A - I and A(A - I) =
A2 - AI = −I - A. So, option (B) is correct.
2x1 - 3x2 + 2x3 = lx2
Statement (C): A2 + I = 0 and A(A2 - I) = A(−I - I) = −2AI = −2A. So,
−x1 + 2x2 = lx3
option (C) is incorrect.
has a non-trivial solution if
Statement (D): A3 + I = A⋅A2 + I = A(−I) + I= −A + I and A(A3 - I) =
2-l -2 1 A(−A - I) = −A2 - A= −(−I) - A = I - A. So, option (D)
2 -3 - l 2 = 0 is correct.
-1 2 -l Hence, the correct answer is option (C).
⇒ (2 - l ) [(3l + l2) - 4] + 2(−2l + 2) + 1(4 - 3 - l ) = 0 27.  If A is a 3 × 3 matrix such that |5 adj A| = 5, then |A| is equal to
3 2 1 1
⇒ l + l - 5l + 3 = 0 ⇒ l = 1, 1, 3 (A)  ± (B)  ± 5 (C)  ± 1 (D)  ±
5 25
Therefore, l ∈{1, 3}.
Hence, the correct answer is option (B). [JEE MAIN 2015 (ONLINE SET 2)]

Mathematical Problem Book for JEE.indb 766 07-06-2018 12:48:11


Chapter 18 | Matrices and Determinants 767

Solution: Order of A is 3; |5 adj A| = 5 Solution: We have the matrix


|5 (adj A)| = 5 ⇒ (5)3 |adj A| = 5⇒ (5)2 |A|3−1 =1 5a b 
A 
2 1 1  3 2 
⇒ A = ⇒ A =±
25 5 A adj A = AAT(1)
Hence, the correct answer is option (A). Now
2 b  5a 3
adj A    and AT   
x2 +1 x +1 x -2 3 5a  b 2 
2
If 2 x + 3 x - 1
28.  3x 3 x - 3 = ax - 12, then ‘a’ is equal to Substituting these values into Eq. (1), we get
2
x + 2x + 3 2x -1 2x -1 5a b   2 b  5a b  5a 3
    
(A) 12 (B) 24 (C) −12 (D) −24  3 2  3 5a   3 2  b 2 
[JEE MAIN 2015 (ONLINE SET 2)]
10a  3b 5ab  5ab  25a2  b2 15a  2b 
Solution:   
 0 3b  10a   15a  2b 13 
x2 + x x +1 x -2
2
∆ = 2x + 3x - 1 3x 3 x - 3 = ax - 12 (1) 10a  3b 0  25a2  b2 15a  2b 
  

 0 10a  3b  
 15a  2b 13 
x2 + 2x + 3 2x -1 2x -1

10a + 3b = 25a2 + b2(2)
Operating R2→R2 - (R1 + R3) gives
10a + 3b = 13 (3)
x2 + x x +1 x -2 15a
∆= -4 0 0 and 15a −2b = 0 Þ b = (4)
2
x2 + 2x + 3 2x -1 2x -1 Substituting the value of b into Eq. (3), we get
x +1 x - 2 x +1 x - 2
⇒ ∆ = -( -4 ) = 4(2 x - 1) æ 15a ö
2x -1 2x -1 1 1 10a + 3 ç ÷ = 13
è 2 ø
= 4(2x - 1) (x + 1 - x + 2) = 4(2x - 1)(3) = 24x - 12 (2)
20a + 45a
Þ = 13
Now from Eqs. (1) and (2) 2
∆ = ax - 12 = 24x - 12 Þ 65a = 13 × 2
⇒ a = 24 2
Hence, the correct answer is option (B). Þ a=
5
29.  The system of linear equations
Substituting the value of a in Eq. (4), we get
x ly - z = 0
lx - y - z = 0 15 2
b= ´ =3
x + y - lz = 0 2 5
has a non-trivial solution for Therefore
æ2ö
5a + b = 5 ç ÷ + 3 = 5
(A)  exactly three values of l. è5ø
(B)  infinitely many values of l.
Hence, the correct answer is option (C).
(C)  exactly one value of l.
(D)  exactly two values of l. 31. The number of distinct real roots of the equation
[JEE MAIN 2016 (OFFLINE)] cos x sin x sin x
Solution: For non-trivial solution, we have é p pù
sin x cos x sin x = 0 in the interval ê - , ú is
ë 4 4û
1 l -1 sin x sin x cos x
l -1 -1 = 0 (A)   1 (B)   4 (C)   2 (D)   3
1 1 -l [JEE MAIN 2016 (ONLINE SET 1)]
Solution:
⇒ 1(l + 1) - l(−l2 + 1) - 1(l + 1) = 0
⇒ l(l2 - 1) = 0 cos x sin x sin x
⇒ l = −1, 0, 1 sin x cos x sin x = 0
Hence, the correct answer is option (A). sin x sin x cos x
5a b  cos x - sin x 0 sin x
30. If A =   and A adj A = AAT, then 5a + b is equal to
 3 2  Þ 0 cos x - sin x sin x = 0
(A) 13   (B)  −1 (C) 5 (D) 4 sin x - cos x sin x - cos x cos x
[JEE MAIN 2016 (OFFLINE)] (C1 ® C1 - C3 , C2 ® C2 - C3 )

Mathematical Problem Book for JEE.indb 767 07-06-2018 12:48:32


768 Mathematics Problem Book for JEE

1 0 sin x Solution: We have


2
Þ (cos x - sin x ) 0 1 sin x = 0 é -4 -1ù
A=ê ú
-1 -1 cos x ë 3 1û
Therefore,
1 0 sin x
é -4 -1ù é -4 -1ù é13 3 ù
2
Þ (cos x - sin x ) 0 1 sin x =0 A2 = ê úê ú=ê ú
ë 3 1 û ë 3 1 û ë -9 -2 û
-1 0 cos x + sin x
(R3 ® R3 + R2 ) é 20 5 ù
|A2016 - 2A2015 - A2014| = |A|2014 × |A2 - 2A - I| = ê ú
ë -15 -5û
⇒ (cos x - sin x )2 (cos x + 2 sin x ) = 0
Therefore,
Therefore,
20 5
2sin x + cos x = 0 or sin x = cos x A2 - 2 A - I = = -100 + 75 = -25
-15 -5
1
For 2sin x + cos x = 0, tan x = − ; therefore, one solution in 4 -1 2014
2 A= = -4 + 3 = -1Þ A =1
 p p 3 -1
x ∈ - , 
 4 4 |A2016 - 2A2015 - A2014| = (−25) × 1 = −25
 p p
For sin x = cos x, one solution in x ∈  - ,  Hence, the correct answer is option (D).
 4 4
Therefore, the total number of solutions is 2.
  Previous Years' Solved JEE Advanced/
Hence, the correct answer is option (C).
IIT-JEE Questions
é 3 1 ù
ê ú
32. If P = ê 2 2 ú , A = é 1 1ù and Q = PAPT, then PTQ2015P is 1.  Consider the system of equations
ê 1 ê0 1ú
3ú ë û - 2y + 3z = -1
ê- ú
ë 2 2 û -x + y - 2z = k
x - 3y + 4z = 1
é0 2015ù é2015 0 ù Statement-1:  The system of equations has no solution for
(A)  ê (B)  ê
ë0 0 úû ë 1 2015 ú
û k≠3
1 3 -1
é 1 2015ù 2015 1 ù
(C)  ê (D)  éê Statement-2:  The determinant -1 -2 k ≠ 0 for k ≠ 3
ë0 1 úû ë 0 2015 ú
û 1 4 1
[JEE MAIN 2016 (ONLINE SET 1)] (A) Statement-1 is true, Statement-2 is true; Statement-2 is a
Solution: We have correct explanation for Statement-1.
Q = PAPT (B) Statement-1 is true, Statement-2 is true; Statement-2 is
Q2 = PAPTPAPT not a correct explanation for Statement-1.
 3 / 2 1/ 2   3 / 2 (C) Statement-1 is true, Statement-2 is false.
1/ 2  1 0 
PP T   
 
 I (D)  Statement-1 is false, Statement-2 is true.
 1/ 2 3 / 2   1/ 2 3 / 2  0 1 [IIT-JEE 2008]
Solution: We have
1 1 1 1 1 2  x - 2 y + 3 z = -1
A2  AA     
0 1 0 1 0 1 - x + y - 2z = k
1 2  0 1 1 3 x - 3y + 4z = 1
A3      and so on
0 1 1 1 0 1 1 2 3
Now   1 1 2  0
Q2 = PA2PT 1 3 4
Q2015 = PA2015PT 1 2 3
é 1 2015ù  x  k 1 2  ( 3  k )
PTQ2015P = PTPA2015PTP = A2015 =ê
ë0 1 úû 1 3 4
Hence, the correct answer is option (C). 1 1 3
 y  1 k 2  (k  3)
é -4 -1ù
33. If A = ê ú , then the determinant of the matrix 1 1 4
ë 3 1û
(A2016 - 2A2015 - A2014) is 1 2 1
(A) −175   (B) 2014 (C) 2016 (D) −25  z  1 1 k  (k  3)
1 3 1
[JEE MAIN 2016 (ONLINE SET 2)]

Mathematical Problem Book for JEE.indb 768 07-06-2018 12:48:52


Chapter 18 | Matrices and Determinants 769

Statement-1:  If k ≠ 3, then ∆ = 0 and ∆ x , ∆ y , ∆ z ≠ 0. Solution: The six matrices A for which A = 0 are
Hence, the system of equation has no solution.
Therefore Statement-1 is true. 0 0 1
 
Statement-2 is true and is a correct explanation of Statement-1. 0 0 1  inconsistent
 
Hence, the correct answer is option (A). 1 1 1
Paragraph for Questions 2 to 4: Let be the set of all 3 × 3 0 1 0 
symmetric matrices all of whose entries are either 0 or 1. Five of  
1 1 1  incconsistent
these entries are 1 and 4 of them are 0.  
0 1 0 
[IIT-JEE 2009]
 2.  The number of matrices in is 1 1 1
 
(A) 12 (B) 6 (C) 9 (D) 3 1 0 0   infinite solutions
 
Solution: If two zeros are the entries in the diagonal, then 1 0 0 
3
C2 × 3C1. 1 1 0 
 
If all the entries in the principle diagonal is 1, then 3C1. 1 1 0   inconsistent
 
So total matrices = 12. 0 0 1
Hence, the correct answer is option (A).
1 0 1
 3. The number of matrices A in for which the system of linear  
x  1 0 1 0   inconsistent
     
equations A y   0  has a unique solution is 1 0 1
    1 0 0 
z  0   
0 1 1  inffinitesolutions
(A)  less than 4     (B)  at least 4 but less than 7  
0 1 1
(C)  at least 7 but less than 10  (D)  at least 10
Hence, the correct answer is option (B).
Solution: Let
0 a b   5. The number of 3 × 3 matrices A whose entries are either 0
 
x  1
A = a 0 c     
b c 1 or 1 and for which the system A y   0  has exactly two
   
z  0 
For unique solution, A ≠ 0 . distinct solutions is
Either b = 0 or c = 0 Þ A ¹ 0 ⇒ 2 matrices (A) 0 (B) 29 - 1 (C) 168 (D) 2
[IIT-JEE 2010]
0 a b 
  Solution: Let
a 1 c  either a = 0 or c = 0 Þ A ¹ 0 Þ 2 matrices
  a1 b1 c1 
b c 0   
A  a2 b2 c2 
 
1 a b  a3 b3 c3 
 
a 0 c  either a = 0 or b = 0 Þ A ¹ 0 Þ 2 matrices
  where ai, bi, ci for i = 1, 2, 3 have values 0 or 1. Then the given sys-
b c 0  tem is equivalent to
1 a b  a1x + b1y + c1z = 0
  a2x + b2y + c2z = 0
a 1 c  a3x + b3y + c3z = 0
 
b c 1
which represents three distinct planes.
If a = b = 0 Þ A = 0 However, three planes cannot intersect at two distinct points.
If a = c = 0 Þ A = 0 Therefore, the number of such 3 × 3 matrices will be zero.
Hence, the correct answer is option (A).
If b = c = 0 Þ A = 0
Paragraph for Questions 6 to 8: Let p be an odd prime number
So, there will be only 6 matrices. and Tp be the following set of 2 × 2 matrices:
Hence, the correct answer is option (B).  a b  
Tp   A    : a, b , c  {0 ,1,… , p  1} 
 4.  The number of matrices A in for which the system of linear  c d  
x  1 [IIT-JEE 2010]
   
equations A y   0  is inconsistent is  6. The number of A in Tp such that A is either symmetric or
    skew-symmetric or both, and det(A) divisible by p is
z  0 
(A) (p - 1)2 (B) 2(p - 1)
(A) 0 (B)  more than 2     (C) 2 (D) 1 (C) (p - 1)2 + 1 (D) 2p - 1

Mathematical Problem Book for JEE.indb 769 07-06-2018 12:49:10


770 Mathematics Problem Book for JEE

Solution: If A is symmetric then b = c. So Solution:


2 2
A = a - b = (a + b )(a - b ) A  [2k  1]3 , B  0 (Since B is a skew-symmetric matrix of order 3)

which is divisible by p if (a + b) is divisible by p or (a - b) is divisible n -1


⇒ det( adj A) + det(adj B ) = A
by p.
Now (a + b) is divisible by p if (a, b) can take values (1, p - 1), = [(2k + 1)3 ]2 = 106 ⇒ 2k + 1 = 10 ⇒ 2k = 9
(2, p - 2), (3, p - 3), …, (p - 1, 1). [k ] = 4
Therefore, (p - 1) ways.
Also (a - b) is divisible by p only when a - b = 0, that is, a = b. Then Hence, the correct answer is (4).
(a, b) can take values (0, 0), (1, 1), (2, 2), …, (p - 1, p - 1). 10. Let M and N be two 3 × 3 non-singular skew-symmetric
Therefore, p ways. ­matrices such that MN = NM. If PT denotes the transpose of P,
If A is skew-symmetric then a = 0 and b = -c or b + c = 0, which gives then M 2N 2 ( MT N )-1( MN -1)T is equal to
A = 0 when b2 = 0 ⇒ b = 0, c = 0. But this possibility is already (A)  M2 (B)  -N2 (C)  -M2 (D)  MN
included when A is symmetric and (a, b) = (0,0). [IIT-JEE 2011]
Again if A is both symmetric and skew-symmetric, then clearly A is Solution: We have MN = NM . Then
null matrix which case is already included. Hence total number of
ways = p + p - 1 = 2p - 1. M 2N 2 ( MT N )-1( MN -1)T = M 2N 2N -1( MT )-1(N -1)T × MT
Hence, the correct answer is option (D).  M 22N ( MTT )11(N 11)TT MTT   M 22  N( M )11(N TT )11MTT
 7. The number of A in Tp such that the trace of A is not divisible  M 22NM 11N 11MTT   M  NMM 11N 11M   MNN 11M   M 22
by p but det (A) is divisible by p is
[Note: The trace of a matrix is the sum of its diagonal entries.] Note: A skew-symmetric matrix of order 3 cannot be non-singular,
(A) (p − 1) (p2 − p + 1) (B)  p3 − (p − 1)2 hence the question is incorrect.
(C) (p − 1)2 (D) (p − 1) (p2 − 2) Hence, the correct answer is option (C).

Solution: As tr(A) is not divisible by p ⇒ a ≠ 0. 11. Let w ¹ 1 be a cube root of unity and S be the set of all non-­
det(A) is divisible by p ⇒ a2 - bc is divisible by p. é 1 a bù
The number of ways of selection of a, b and c is ê ú
singular matrices of the form ê w 1 c ú , where each of a,
( p - 1)[( p - 1) ´ 1] = ( p - 1)2 ê 2 ú
ëw w 1û
Hence, the correct answer is option (C). b, and c is either w or w  2. Then, the number of distinct m
­ atrices
 8.  The number of A in Tp such that det (A) is not divisible by p is in the set S is
(A) 2p2 (B)  p3 − 5p (C)  p3 − 3p (D)  p3 − p2 (A) 2 (B) 6 (C) 4 (D) 8
Solution: The total number of A = p ´ p ´ p = p . 3 [IIT-JEE 2011]
The number of A such that det(A) is divisible by p equals Solution: For being non-singular
( p  1)2  number of A in which a = 0 é 1 a bù
ê ú
= ( p - 1)2 + p + p - 1 êw 1 cú ¹ 0
ê 2 ú
= p2 ëw w 1û
Þ acw 2 - (a + c )w + 1 ¹ 0 Þ (aw - 1)(cw - 1) ¹ 0
The required number is p3 - p2 .
Hence, the correct answer is option (D). Þ a ¹ w 2 and c ¹ w 2 Þ a = w and c = w and b = w or w 2
 9. Let k be a positive real number and let Hence, the number of possible triplets of (a, b, c) is 2, that is,
(w, w 2, w ) and (w, w, w ).
  Hence, the correct answer is option (A).
2k  1 2 k 2 k 
 
A 2 k 1 2k  12. Let M be a 3 × 3 matrix satisfying
 
2 k 2k 1  é0 ù é -1ù é1ù é1ù é1ù é 0 ù
 0 M êê 1úú = êê 2 úú , M êê -1úú = êê 1 úú and M êê1úú = êê 0 úú
 2k  1 k 
  ëê0 ûú êë 3 úû êë 0 úû êë -1úû êë1úû êë12 úû
and   B  1 2k 0 2 k .
 
  k 2 k 0  Then, the sum of the diagonal entries of M is _____.
[IIT-JEE 2011]
If det(adj A) + det(adj B) = 106, then [k] is equal to  . Solution: Let
[Note: adj M denotes the adjoint of a square matrix M and [k] a b c 
denotes the largest integer less than or equal to k].  
M  d e f 
[IIT-JEE 2010]  
g h i 

Mathematical Problem Book for JEE.indb 770 07-06-2018 12:49:27


Chapter 18 | Matrices and Determinants 771

0  1 1 4 4 
     
M 1   2   b  1, e  2, h  3
    15. If the adjoint of a 3 × 3 matrix P is 2 1 7  , then the pos-
0   3  1 1 3 
1  1  sible value(s) of the determinant of P is (are)
    (A)  -2 (B)  -1 (C) 1 (D) 2
M 1   1   a  0 , d  3, g  2
    [IIT-JEE 2012]
 0  1
Solution:
1  0  2 n -1
   adj P = P as ( adj ( P ) = P )
M 1   0   g  h  i  12  i  7
   adjP = 1(3 - 7) - 4(6 - 7) + 4(2 - 1) = 4
1 12 

Hence, P  2 or  2
Therefore, the sum of diagonal elements = 9.
Hence, the correct answers are options (A) and (D).
Hence, the correct answer is (9).
16. 
For 3 × 3 matrices M and N, which of the following statement(s)
13. Let P = [aij] be a 3 × 3 matrix and let Q = [bij], where is (are) NOT correct?
(A)  NTMN is symmetric or skew-symmetric, according to M is
bij = 2i + jaij for 1 ≤ i, j ≤ 3. If the determinant of P is 2, then the
symmetric or skew-symmetric
determinant of the matrix Q is
(B) MN - NM is skew-symmetric for all symmetric matrices M
(A) 210 (B) 211 (C) 212 (D) 213 and N
[IIT-JEE 2012] (C)  MN is symmetric for all symmetric matrices M and N
Solution: (D) (adj M)·(adj N) = adj(MN) for all invertible matrices M
and N
22 a11 23 a12 24 a13 [JEE ADVANCED 2013]
Q  23 a21 24 a22 25 a23 Solution: We have
24 a31 25 a32 26 a33 (NTMN)T = -NTMT(NT)T = NTMTN
(A) If M is skew symmetric, then (NTMN)T = -NTMN. Therefore,
a11 a12 a13
2 3 4
it is concluded that it is skew-symmetric.
 Q  2 2 2 2a21 2a22 2a23 If M is symmetric, then (MTMN)T = NTMN. Therefore, it
22 a31 22 a32 22 a33 is concluded that it is symmetric. Hence, option (A) is
correct.
a11 a12 a13
9 2 (B)  We have
 Q  2 22 a21 a22 a23
( MN - NM )T = ( MN )T - (NM )T
a31 a32 a33
= N T MT - MT N T
 Q  212 P
= -( MT MT - N T MT )
 Q  213
= -( MN - NM )
Hence, the correct answer is option (D). Therefore, it is concluded that it is skew-symmetric and
14. If P is a 3 × 3 matrix such that PT = 2P + I, where PT is the trans- hence option (B) is correct.
pose of P and I is the 3 × 3 identity matrix, then there exists a (C) (MN)T = NTMT. Symmetricity and skew-symmetricity depend
x  0  on the nature of M and N; therefore, option (C) is incorrect.
   
column matrix X  y   0  such that (D) adj(MN) = adj(N) adj M; therefore, option (D) is incorrect.
   
z  0  Hence, the correct answers are options (C) and (D).
0 
  Let w be a complex cube root of unity with w ≠ 1 and P = [pij]
17. 
(A)  PX  0    (B)  PX = X (C)  PX = 2X (D)  PX = −X
  be an n × n matrix with pij = w i+j. Then, P2 ≠ 0, when n = ?
0 
[IIT-JEE 2012] (A) 57 (B) 55 (C) 58 (D) 56
[JEE ADVANCED 2013]
Solution: Given
P T = 2P + I Solution: We have P  [ Pij ]nn , P 2 ≠ 0. Now
ˆ ˆ
Þ P = 2P T + I = 2(2P + I ) + I Pij = w i + j
ÞP+I =0 w 2 1 w w 2 1 ... 
Þ PX + X = 0  
 1 w w2 1 w ... 
 PX   X P= 
2
w w 1 w w2 ... 
Hence, the correct answer is option (D).  ... ...
 ... ... ... ...  n×n

Mathematical Problem Book for JEE.indb 771 07-06-2018 12:49:40


772 Mathematics Problem Book for JEE

w 2 1 w w 2 1 Solution: Given MN = NM, therefore a2 − b2 = (a + b) (a − b) of alge-


…
  bra of numbers is applicable
2  1 w w2 1 w … Therefore,
P = 
2
1 w w2 M2 = N4
w w …
… … … … … ⇒ M2 − N4 = 0 (null matrix)
 …
⇒ (M + N2) (M − N2) = 0
w 2 1 w w 2 1 … Now since M ≠ N2 (given), therefore, the possibilities are
 
 1 w w 2 1 w … (M + N2) = 0 and M − N2 ≠ 0 (1)
 2  or (M + N2) ≠ 0 and M - N2 ≠ 0 (2)
w w 1 w w 2 …
 … … … … … … Now we know, if A and B are non-null square matrix and AB = 0
  then A and B both are singular, i.e. |A| = 0 and |B| = 0 and AB = 0
Therefore, Note: For example, let A be non-singular. Then
B = I(B) = A−1 AB = 0
(w 4 + 1+ w 2 ) + (w 4 + 1+ w 2 ) +  = 0 So AB = 0 assumed. Therefore, B is singular, which is a contradiction.
This is possible only when n is a multiple of 3. Therefore, n can be So, A has to be singular. Similarly, then B also has to be singular.
55, 58, 56 (P2 ≠ 0). Therefore, from Eqs. (1) and (2), we conclude the only possibility is
|M + N2| = 0
Hence, the correct answers are options (B), (C) and (D). Now checking options:
(A)  |M2 + MN2| = |M||M + N2| = 0
18. 
Let M be a 2 × 2 symmetric matrix with integer entries. Then
M is invertible if Therefore, (A) is correct.
(A)  The first column of M is the transpose of the second row (B) (M2 +MN2) U = 0
of M Since M2 + MN2 is singular, therefore, U has infinitely many
(B) The second row of M is the transpose of the first column possible values (non-trivial solutions). So (B) is true.
of M (C)  False, since |M2 + MN2| = 0
(C) M is a diagonal matrix with non-zero entries in the main (D) False. Since |M2 + MN2| = 0, therefore, U is not a necessarily a
diagonal zero matrix.
(D) The product of entries in the main diagonal of M is not the Hence, the correct answers are options (A) and (B).
square of an integer 20. Let X and Y be two arbitrary, 3 × 3, non-zero, skew-symmetric
[JEE ADVANCED 2014] matrices and Z be an arbitrary 3 × 3, non-zero, symmetric matrix.
Solution: Let Then which of the following matrices is (are) skew-symmetric?
a b 
M  a, b , d  I (A)  Y3Z4 - Z4Y3 (B)  X44 + Y44
b d 
(C)  X4Z3 - Z3X4 (D)  X23 + Y23
a  b  a a [JEE ADVANCED 2015]
(A)        a  b  d  |M|  0 Solution:
b  d  a a
X, Y → skew-symmetric matrices of order 3 × 3
   ⇒ Not invertible. Therefore, (A) is false. Z → symmetric matrix of order 3 × 3
and X, Y, Z ≠ 0
[ d ] = [a b ] Þ a = b = d
(B) b
Checking all the options:
Þ |M| = 0 ⇒Not invertible. Therefore, (B) is false. Option (A)  (Y3Z4 - Z4Y3)T = (Y3Z4)T - (Z4Y3)T
éa o ù = (Z4)T (Y3)T - (Y3)T(Z4)T
(C) If M is a diagonal matrix, then M = ê ú Þ |M| = ad ¹ 0
ëo d û = (ZT)4(YT)3 - (YT)3(ZT)4 = −Z4Y3 + Y3Z4
⇒ M invertible. Therefore, (C) is correct . Option (B)  (X44 + Y44)T = (XT)44 + (YT)44
= X44 + Y44 ⇒ (symmetric)
(D) Given ad ≠ b2 . Now |M| = ad − b2 ≠ 0 for M to be invertible.
Therefore, (D) is true. Option (C)   (X4Z3 - Z3X4)T = (ZT)3 (XT)4 - (XT)4(ZT)3
Hence, the correct answers are options (C) and (D). = Z3X4 - X4Z3 ⇒ (skew-symmetric)
Option (D)  (X + Y23)T = (XT)23 + (YT)23
23
Let M and N be two 3 × 3 matrices such MN = NM. Further, if
19. 
= (−X)23 + (−Y)23 = −(X23 + Y23) ⇒ (skew-symmetric)
M ≠ N2 and M2 = N4, then
(A)  Determinant of (M2 + MN2) is 0. Hence, the correct answers are options (C) and (D).
(B) There is a 3 × 3 non-zero matrix U such that (M2 + MN2)U 21. Which of the following values of a satisfy the equation
is the zero matrix.
(C)  Determinant of (M2 + MN2) ≠ 1. (1+ a )2 (1+ 2a )2 (1+ 3a )2
(D) For a 3 × 3 matrix U, if (M2 + MN2)U equals the zero matrix (2 + a )2 (2 + 2a )2 (2 + 3a )2 = -648a
then U is the zero matrix.
(3 + a )2 (3 + 2a )2 (3 + 3a )2
[JEE ADVANCED 2014]

Mathematical Problem Book for JEE.indb 772 07-06-2018 12:49:47


Chapter 18 | Matrices and Determinants 773

(A)   −4 (B) 9    (C) −9 (D) 4 ⇒ 6a + 10 = k(1)


[JEE ADVANCED 2015] ⇒ |P| = 2k
Solution: So,

(1+ a )2 (1+ 2a )2 (1+ 3a )2 (adj P ) (adj P )


Q = kI =
P 2
(2 + a )2 (2 + 2a )2 (2 + 3a )2 = -648a
Now,
(3 + a )2 (3 + 2a )2 (3 + 3a 2
1 -(3a + 4 ) -k
q23 = C32 = = ⇒ 12a + 16 = k(2)
2 2 8
R2 → R2 - R1 and R3 → R3 - R2 give
Solving Eqs. (1) and (2), we get
−4 = −k ⇒ k = 4
(1+ a )2 (1+ 2a )2 (1+ 3a )2
Using the value of k in Eq. (1), we get
(3 + 2a ) (3 + 4a ) (3 + 6a ) = -648a
6a + 10 = 4 ⇒ a = −1
(5 + 2a ) (5 + 4a ) (5 + 6a ) That is, a = −1 and k = 4.

Hence, option (A) is incorrect.
R3 → R3 - R2 gives The values of a = −1 and k = 4 satisfy the equation given in
option (B).
(1+ a )2 (1+ 2a )2 (1+ 3a )2
Hence, option (B) is correct.
3 + 2a 3 + 4a 3 + 6a = -648a Now,
2 2 2 k2
det Q = =8
C2 → C2 - C1 and C3 → C3 - C2 give 2
Therefore,
(1+ a )2 a (2 + 3a ) a (2 + 5a )
det(P adj Q) = (det P)det(adj Q) = (2 × 4)(detQ)2
(3 + 2a ) 2a 2a = -648a = 8 × 82 = 23 × 26 = 29
(2 ) 0 0 Hence, the correct answers are options (B) and (C).

⇒ 2(2a 2 )[(2 + 3a ) - (2 + 5a )] = -648a 23. The total number of distinct x ∈R for which
⇒ −8a 3 = −648a x x2 1+ x 2
⇒ a 2 = 81 or a = 0 2x 4 x2 1+ 8 x 3 = 10 is _________.
⇒ a  = 0 or ± 9
3x 9 x 2 1+ 27 x 3
Hence, the correct answers are options (B) and (C).
[JEE ADVANCED 2016]
3 1 2 
  Solution: It is given that
22. Let P  2 0 a  , where a ∈R. Suppose Q = [qij] is a
 
3 5 0  x x2 1+ x 3
matrix such that PQ = kI, where k ∈R, k ≠ 0 and I is the 2 x (2 x )2 1+ (2 x )3 = 10
k k2 3 x (3 x )2 1+ (3 x )3
identity matrix of order 3. If q23 = - and det(Q) = , then
8 2
(A)  a = 0, k = 8 (B) 4a - k + 8 = 0 x x2 1 x x2 x3
(C) det(P adj (Q)) = 29 (D) det(Q adj (P)) = 213 ⇒ 2x 4x 2
1 + 2x 4x 2
8 x 3 = 10
[JEE ADVANCED 2016] 3x 9x2 1 3x 9x2 27 x 3
Solution: It is given that
3 1 2  1 1 1 1 1 1
 
P  2 0 a  and Q = [qij] ⇒x 2 2 3 2
1+x 2 2 6 2
23 = 10
 
3 5 0  3 3 2
1 3 3 2
33
Now,
3 -2 1 1 1 1 1 1
C32 = - = −(3a + 4)
2 a ⇒x 2 2 3 2 6
1 + 6x 2 2 2
23 = 10
Here, |P| = 12a + 20, 3 32 1 3 32 33
PQ = kI ⇒ Q = kIP−1
Now, |P| |Q| = k3. Therefore, 1 1 1
2 Þ x 3 (1+ 6 x 3 ) 1 2 4 = 10
k
(12a + 20 ) = k3 1 3 9
2

Mathematical Problem Book for JEE.indb 773 07-06-2018 12:50:04


774 Mathematics Problem Book for JEE

Using C2→C2 - C1 and C3 → C3 - C1 we get Solution: It is given that


P50 - Q = I
1 0 0
⇒ Q = P50 - I = [qij]3×3
x 3 (1+ 6 x 3 ) 1 1 3 = 10 The given matrix is
1 2 8  1 0 0
 
⇒ x3(1 + 6x3) 1(8 - 6) = 10  
P   a 1 0
⇒ x3(1 + 6x3) = 5  2 
⇒ 6(x3)2 + x3 - 5 = 6 a a 1

⇒ 6(x3)2 + 6x3 - 5x3 - 5 = 0 Therefore,
⇒ 6x3(x3 + 1) - 5(x3 + 1) = 0  1 0 0  1 0 0  1
⇒ (6x3 - 5) (x2 - x + 1) (x + 1) = 0     0 
0
2     
Therefore, x = −1 and (5/6)1/3. P   a 1 0   a 1 0    2a 1 0
 2  2   
Hence, there are two district values for x. Thus, 2 is the answer. a a 1 a a 1 3a2 2a 1
Hence, the correct answer is (2).
 1 0 0   1 0 0   1 0 0 
-1+ 3i 
24. Let z = , where i = -1, and r, s Î{1, 2, 3}. Let 3     
2 P   2a 1 0  a 1 0    3a 1 0
 2    
(  z )r z 2 s  3a 2a 1 a2 a 1 6a2 3a 1
P    and I be the identity matrix of order 2. Then,


2s
 z z r   1 0 0 

the total number of ordered pairs (r, s) for which P2 = −I is  
P 4   4a 1 0
_____.  2 
10a 4 a 1
[JEE ADVANCED 2016]
Solution: It is given that  1 0 0
 
-1 3 2p 2p P 50  50a 1 0 
z= +i = cos + i sin = e i 2p / 3 = w  
2 2 3 3  T50 50a 1

(w  is cube root of unity), where r, s Î {1, 2, 3}. Therefore, using difference method, we get
(  z ) r 2s 
z  1 0  S50  a2  3a2  6a2  10a2    T50
It is also given that P   and I   .
2s r  0 1 S50  a2  3a2  6a2    T49  T50
 z z 
Since P2 = -I, we have 0  a2  2a2  3a2    T50
é( - z )r z 2 s ù é( z )r z 2s ù é1 0ù  T50  a2  2a2  3a2    50a2
P2 = ê úê ú = -ê ú
êë z 2 s zr úû êë z 2 s r
z úû ë 0 1û a2 (50 )(51) 2
T50  a2 (1  2  3    50 )   a (25)(51)
2
é ( - z )2r + z 4 s ( - z )r × z 2 s + z r z 2 s ù é -1 0 ù
=ê ú=ê ú Therefore,
êë( - z )r z 2 s + z r × z 2 s z 4 s + ( z )2r úû ë 0 -1û  0 0 0
   
50
That is, we have I  I  50a 0 0   [qij ]33
 
(−z)2r + z4s = −1 and z4s + z2r = −1  T50 50a 0 
and ((−z)r + zr)z2s = 0 and z2r + z4s = −1
⇒((−w)r + (w)r)·w 2s = 0 Hence, we get the following values:
Now, w 2s ≠ 0; therefore, q31 = T50 = 5(51)a2
(−w )r + (w )r = 0 q32 = 50a
where r is the odd number and hence r = 1, 3. q21 = 50a
When r = 1 Therefore,
(−w)2 + w 4s = −1 ⇒ w 4s = −1 - w 2 = + w
Now, s can be 1 (since s ≠ 3). q31 + q32 (25)(51)(a2 ) (25)(51)(16 )
= + 1 = 1+ = 103
That is, (r, s) = (1, 1), that is, the total number of ordered pair (r, s) is q21 50a (50 )( 4 )
one (single) for which P2 = -I. Hence, the correct answer option is (B).
Hence, the correct answer is (1).
 1 0 0
  Practice Exercise 1
25. Let P   4 1 0  and I be the identity matrix of order 3. If Q
 
16 4 1 1 0 0 
q +q  
= [qij] is a matrix such that P50 - Q = I, then 31 32 equals   1. If A = 0 1 0  , then A2 is equal to
q21  
a b 1
(A) 52 (B) 103 (C) 201 (D) 205
[JEE ADVANCED 2016] (A)  Unit matrix (B) Null matrix (C) A (D) -A

Mathematical Problem Book for JEE.indb 774 07-06-2018 12:50:24


Chapter 18 | Matrices and Determinants 775

a 0 0  12. If A and B are symmetric matrices of the same order, then


 
  2. If A = 0 a 0  , then |A| |adj A| is equal to (A)   AB is a symmetric matrix
  (B)   A - B is skew-symmetric matrix
0 0 a 
(C)   AB + BA is a symmetric matrix
(A) a25 (B) a27    (C) a81 (D) None of these (D)   AB - BA is a symmetric matrix
  3. If A is a 3 × 3 skew-symmetric matrix, then |A| is given by 13. If A is any square matrix, then
(A) 0 (B) -1   (C) 1 (D) None of these (A)   A + A′ is skew-symmetric (B)   A - A′ is symmetric
(C)   AA′ is symmetric (D)   None of these
  4. If A and B are two square matrices of the same order and m is
a positive integer, then 14. If A is a square matrix such that A3 = I then A-1 is equal to
(A)   I (B)   A    (C)   A2  (D)   None of these
(A + B)m = mC Am + mC Am - 1B + mC Am - 2B2 + … + mCm - 1ABm - 1
0 1 2
+ mC Bm if 15. If A is any square matrix then which of the following is not
m
symmetric?
(A)  AB = BA (B)  AB + BA = 0 (A)   A + A′ (B)   A - A′ (C)   AA′  (D)   A′A
(C)  Am = 0, Bm = 0 (D)  None of these 16. Let A be a skew-symmetric matrix of order n then
  5. If A = (aij)3 × 3 is a skew-symmetric matrix, then (A)   |A| = 0 if n is even (B)   |A| = 0 if n is odd
(A)  aii = 0,∀ i (B)  A - A′ is null matrix (C)   |A| = 0 for all n ∈ N (D)   None of these
(C)  |A| ≠ 0 (D)  None of these 17.  Each diagonal element of skew-symmetric matrix is
(A)   Zero (B)   Positive
3  (C)   Non-real (D)   Negative
7 1 2    4 
  6.    4   2   is equal to
9 2 1   2   4 2
5  18. If A    , then the value of (A - 2I)(A - 3I) is
1 1
  43
(A)   43  (B)       (A)   Unit matrix (B)   Non-singular matrix
44  45 (C)   Null matrix (D)   None of these

  19. Matrix A has m rows and n + 5 columns, matrix B has m rows


(C)   45  (D)   None of these and 11 - n columns. If both AB and BA exist, then
44 
(A)   AB and BA are square matrices
10 0  (B)   AB and BA are of order 8 × 8 and 3 × 13, respectively
  7. For any matrix A of order 2 × 2, if A(adj A) =   , then

 0 10  (C)   AB = BA
|A| is equal to
(D)   None of these
(A)   20 (B)   100 (C)   10  (D)   0
1 3 3 
  8.  I f a matrix A is symmetric as well as skew-symmetric, then A  
20. If A  1 4 3  , then A-1 is equal to
is a  
1 3 4 
(A)   Diagonal matrix (B)   Null matrix
(C)   Unit matrix (D)   None of these  7 3 3  7 3 3
   
(A)    0 1 0  (B)   1 1 0 
  9. If A and B be two square matrices such that AB = O, then    
1 0 5  1 0 1 
(A)   det A = 0 or det B = 0 (B)   det B = 0
7 3 3
(C)   B = A-1 (D)   det A = 0  
(C)   0 1 0  (D)   None of these
0 0 0 
  1 0 1 
10. If A = 1 0 0  , then
 
0 1 0  é i -i ù é 1 -1ù , then A8 equals
21. If A = ê ú and B = ê
- i i ú
(A)   A2 = A (B)   A2 =0 ë û ë -1 1 û
(A)   128B (B)   132B (C)   116B (D)   8B
(C)   A2 = I (D)   A3 =O
22. If a, b , g are three real numbers and
0 0 0 
  é 1 cos(a - b ) cos(a - g ) ù
11. If A = 0 0 0  , then A is ê
  A = êcos( b - a ) 1 cos( b - g )úú then
0 1 0 
êë cos(g - a ) cos(g - b ) 1 úû
(A)   An invertible matrix (B)   An idempotent matrix (A)   A is skew-symmetric (B)   A is invertible
(C)   A nilpotent matrix (D)   None of these (C)   A is non-singular (D)   |A| = 0

Mathematical Problem Book for JEE.indb 775 07-06-2018 12:50:33


776 Mathematics Problem Book for JEE

a1 b1 c1 1 3
33. Let w = - + i . Then, the value of the determinant
23. If a2 b2 c2 ≠ 0, then the number of solutions of the 2 2
a3 b3 c3 1 1 1
system of equations a1x + b1y + c1z = 0, a2x + b2y + c2z = 0 and
a3x + b3y + c3z = 0 is 1 -1- w 2 w 2 is
1 w2 w4
(A)   Infinite number of solutions
(B)   Only one unique solution (A)   3w (B)   3w (w- 1) (C)   3w 2  (D)   3w (1 - w)
(C)   More than one solution
(D)   None of these
24. Let A and B be two square matrices of the same dimension sin 2 x e x sin x + x cos x sin x + x 2 cos x
and let [A, B] = AB - BA. Then for three 2 × 2 matrices A, B, C, 34. If D(x) = cos x + sin x ex + x 1+ x 2 , then
[[A, B], C] + [[B, C], A] + [[C, A], B] is equal to x 2x x
(A)   1 (B)   0 e cos x e e
(C)   ABC - CBA (D)   None of these  
25. If the matrices A, B, (A + B) are non-singular, then (A)   D′(0) = 0 (B)   ∆   = 0
2 
[ A( A + B )-1B ]-1 is equal to  
(C)   ∆   = 0 (D)   All the above
(A)   A + B (B)   A-1 + B-1 4 
-1
(C)   (A + B) (D)   None of these
f ( x + a ) f ( x + 2a ) f ( x + 3a )
26. If A and B matrices commute then 35. Let g(x) = f (a ) f (2a ) f (3a ) , where a is a
(A)   A-1 and B also commute
f ′(a ) f ′(2a ) f ′(3a )
(B)   B-1 and A also commute
(C)   A-1 and B-1 also commute g( x )
(D)   All the above constant. Then lim is equal to
x ®0 x
27. If A, B and C are three matrices conformable for multiplication, (A)   0 (B)   1   (C)   -1 (D)   None of these
then (ACB) -1 is equal to
2 3
(A)   A-1B-1C-1 (B)   B-1C-1A-1 4x - 4 ( x - 2) x
(C)   C-1B-1A-1 (D) Cannot be determined 36. If D(x) = 8 x - 4 2 (x -2 2) 2
( x + 1)3 , then the coefficient
1 0 0  12 x - 4 3 ( x - 2 3 )2 ( x - 1)3
 
28. If A = 0 1 0  , then A2 is equal to of x in D(x) is
 
a b 1 (A)   64(5 - 2 - 3 )    (B) 64(5 + 2 - 3 )
(A)   Unit matrix    (B)   Null matrix (C)   A (D)   -A (C)   64(5 + 2 + 3 )    (D)   None of these
29.  Trace of a skew-symmetric matrix is always equal to
a2 + x 2 ab - cx ac + bx x c -b
(A)   ∑aij (B)   ∑aii­    (C)   Zero  (D)   None of these 2 2
37. If D1 = ab + cx b + x bc + ax and D 2 = -c x a ,
30. The matrix of the transformation reflection in the line
ac - bx bc - ax c2 + x2 b -a x
x + y = 0 is
1 0    1 0   0 1 then
(A)     (B)   0 1  (C)       (D)    
(A)   D1 = D2 (B)   D1 = D22
0 1 1 0   0 1 1 0 
(C)   D1 = 2D2 (D)   None of these

xn x n+2 x n+3 38. If a, b and c are even natural numbers, then


 1 1 1
31. If yn y n+2 y n+3 = (y - z)(z - x) (x - y)     , then a -1 a a +1
x y z D = b -1 b b +1 is equal to
zn z n+2 z n+3
c -1 c c +1
(A)   n = 2 (B)   n = -2 (C)   n = -1 (D)   n = 1
(A)   a + b + c (B)   a2 + b2 + c2
( 2r ) x N(N + 1)
(C)   abc (D)  None of these
32. If Dr = (6r 2 - 1) y N 2 ( 2N + 3) , where N ∈ natural
( 4 r 3 - 2Nr ) z N 3 (N + 1) cos x 1 0 p /2

N
39. If f(x) = 1 2 cos x 1 , then ò f ( x ) dx equals    
0
numbers, then ∑ ∆r is equal to 0 1 2 cos x
r =1

(A)   N (B)   N2   (C)   Zero (D)   None of these (A)   1/4 (B)   1/3 (C)   1/2 (D)   1

Mathematical Problem Book for JEE.indb 776 07-06-2018 12:50:54


Chapter 18 | Matrices and Determinants 777

B B A
2
x cos x ex
p /2
2 (C)  B A B (D)   None of these
40. If f(x) = sin x x sec x , then the value of ò f ( x ) dx
A B B
tan x 1 2 -p / 2

50. If A, B, C are angles of a triangle ABC, then the value of the


is equal to A B C
(A)   0    (B)   1    
(C)   2   (D)   None of these sin sin sin
2 2 2
determinant B A
x2 + x + 3 1 4 sin( A + B + C ) sin cos
4 3
41. If the expression 2 x + x + 2 x + 1 2 3 is equal to 2 2
(A + B + C) C
x2 + x 1 1 cos tan( A + B + C ) sin
2 2
ax4 + bx3 + cx2 + dx + e, then the value of e is equal to is less than or equal to
(A)   1/2 (B)   1/4    (C)   1/8 (D)   None of these
(A)   zero   (B)   1   (C)   2   (D)   None of these
x 2 5
a b aa + b
51.  The sum of two non-integral roots of 3 x 3 = 0 is
42. The determinant D = b c ba + c is equal to
5 4 x
aa + b ba + c 0
zero if (A)   5 (B)   -5   (C)   -18 (D)   None of these
(A)   a, b, c are in AP   (B)  a, b, c are in GP
1 1 1
(C)   a, b, c are in HP   (D)  None of these
52.  The value of D = (2 x + 2- x )2 (3 x + 3- x )2 (5 x + 5- x )2 is
43. If a, b, g are the roots of the equation x3 + px + q = 0 (where p
(2 x - 2- x )2 (3 x - 3- x )2 (5 x - 5- x )2
a b g
≠ 0, q ≠ 0), then the value of the determinant b g a is (A)   0 (B)   30x     (C)   30-x (D)  None of these
g a b 53. There are three points (a, x), (b, y) and (c, z) such that the
(A)   p  (B)   q  (C)   p2 - 2q  (D)   None of these straight lines joining any two of them are not equally
inclined to the coordinate axes where a, b, c, x, y, z ∈ R. If
44. The number of values of k for which the system of equations x +a y +b z +c
(k + 1)x + 8y = 4k, kx + (k + 3)y = 3k - 1 has infinitely many y
y + b z + c x + a = 0 and a + c = -b, then x, - , z are in
solutions is 2
(A)   0    (B)   1     (C)   2   (D) infinite z +c x +a y +b
(A)   AP (B)   GP    (C)   HP (D)   None of these
a2 (1+ x ) ab ac
45. The determinant D = ab b2 (1+ x ) bc is n -2

2
å1 n(n - 1) n2
divisible by ac bc c (1+ x ) k =0
n
(A)   1 + x   (B)   (1 + x)2  (C)   x2   (D)   None of these 54. If å1 (n + 1)(n - 1) n( n + 1) = 72, then n is equal to
k =1
46. If the system of equations ax + y + z = 0, x + by + z = 0 and n -1
x + y + cz = 0 (a, b, c ≠ 1) has a non-trivial solution, then the
1 1 1
å1 n2 + 1 n2
k =1
value of + + is
1- a 1- b 1- c
(A)   6     
(B)   9    
(C)   8    (D)   None of these
(A)   -1    (B)   0    (C)   1   (D)   None of these
47. If the system of equations x + ay = 0, az + y = 0 and ax + z = 0 1+ a2 + a 4 1+ ab + a2b2 1+ ac + a2c 2
has infinite solutions, then the value of a is 55. D = 1+ ab + a2b2 1+ b 2 + b 4 1+ bc + b2c 2 equals
(A)   -1    (B)   1    
(C)   0   (D)   No real values 2 2
1+ ac + a c 1+ bc + b c 2 2 2
1+ c + c 4

48. If A + B + C = p, then the value of determinant


(A)   (a + b + c)6 (B)   (a - b)2(b - c)2(c - a)2
sin2 A cot A 1 (C)   4(a - b)(b - c)(c - a) (D)   None of these
sin2 B cot B 1 is equal to
2
56. If in a triangle ABC,
sin C cot C 1
1 1 1
(A)   0    (B)   1    (C)   -1   
(D)   None of these
cot( A / 2) cot(B / 2) cot(C / 2) =0
49. For A = a2 + b2 + c 2 , B = ab + bc + ca, (a3 + b3 + c 3 - 3abc )2 is tan(B / 2) + tan(C / 2) tan(C / 2) + tan( A / 2) tan( A / 2) + tan(B / 2)
equal to
B A B A B B then the triangle must be
(A)  B B A (B)   B B A (A)   Equilateral (B)   Obtuse angle
A B B B A B (C)   Isosceles (D)   None of these

Mathematical Problem Book for JEE.indb 777 07-06-2018 12:51:10


778 Mathematics Problem Book for JEE

sin x cos x cos x éx + a b c ù


57. The number of distinct real roots of cos x sin x cos x = 0 68. The values of x for which the matrix êê a x +b c úú is
cos x cos x sin x êë a b x + c úû
p p non-singular are
in the interval - £ x £ is
4 4 (A)   R - {0} (B)   R - {- (a + b + c)}
(A)   0 (B)   2   (C)   1 (D)   3 (C)   R - {0, - (a + b + c)} (D)   None of these

ln x x 0 é2 3 ù -1
ln x 69. If A = ê ú , then A is equal to
58. If f(x) = , then 1/ x 1 x is ë5 -2 û
x 1 1
-1/ x 2 0 2 (A)   A′ (B)   2A   (C)   A  (D)   A
2 19
(A)   x3f ′(x) (B)   x 2f ¢¢¢( x ) 70.  The inverse of a skew-symmetric matrix is
(C)   x 3f ¢¢¢( x ) (D)   None of these (A)  A symmetric matrix if it exists
59. If the system of equations lx + (b - a)y + (c - a)z = 0, (a - b)x (B)  A skew-symmetric matrix if it exists
+ ly + (c - b)z = 0 and (a - c)x + (b - c)y + lz = 0 has a (C)  Transpose of the original matrix
non-trivial solution, then the value of λ is (D)  May not exist
(A)   l = 0 (B)   l = 1    (C)   l = -1 (D)   None of these é1 2 3 ù
60. If A′ is the transpose of a square matrix A, then 71. If A = êê1 3 5 úú , then adj (adj A) is
(A)   |A| ≠ |A′| (B)   |A| = |A′| ëê1 5 12 úû
(C)   |A| + |A′| = 0 (D)  |A| = |A′| only when A is
é3 3 3 ù é1 2 3 ù
symmetric
(A)  ê6 9 15 ú (B)   ê1 3 5 ú
é cosq sinq ù ê ú ê ú
1 0ù
61. If I = éê , J = é 0 1ù and B = ê êë9 15 36 úû êë1 5 12 úû
cosq úû
ú , then B
ë 0 1û
ê ú ë - sinq
ë -1 0 û é3 6 9 ù
equals
(C)  ê3 9 15 ú (D)   None of these
(A)   I cos θ + J sin θ    (B)   I sin θ + J cos θ ê ú
ëê3 15 36 úû
(C)   I cos θ - J sin θ    (D)   - I cos θ + J sinθ
72. Let A be a square matrix of order 3 such that transpose of
62. If In is the identity matrix of order n, then (In) -1 inverse of A is A itself. Then |adj (adj A)| is equal to
(A)   does not exist      (B)   is equal to In (A)   9 (B)   27    (C)   4    (D)   1
(C)   equals O       (D)   nIn éa aù
63. If for a matrix A, A2 + I = O where I is the identity matrix, then 73. If A = êê b b úú , then AAT is
A equals êë g c úû
(A)  
 1 0  i 0    (C)    1 2   (D)   -1 0  (A)   A non-singular matrix (B)   A singular matrix
0 1 (B)     -1 1  0 -1
  0 i      (C)   An identity matrix (D)   None of these
64. The number of non-zero diagonal matrices of order 4 satisfying 74. If A and B are two non-singular square matrices of the same
A2 = A is order, the adjoint of AB is equal to
(A)   2 (B)   4 (C)   16 (D)   15 (A)   (adj A) (adj B) (B)   (adj B) (adj A)
65. If A and B are symmetric matrices of order n (A ≠ B), then (C)   adj(BA) (D)   adj A + adj B
(A)   A + B is skew-symmetric (B)   A + B is symmetric 75. If Ak = 0 (null matrix) for some positive integral value of k,
(C)   A + B is a diagonal matrix (D)   A + B is a zero matrix then I + A + A2 + … + Ak-1 is equal to
(A)   Null matrix (B)   (I + A)k
c -b ù é a2 ab ac ù
é0 ê ú -1
(C)   (I - A) (D)   None of these
66. If A = êê -c a úú and B = êab b bc ú , then AB is equal
2
0
ê 2ú
76.  The matrix X for which  1 - 4  X =  -16 -6  is
êë b -a 0 úû êë ac bc c úû 3 -2   7 2 
  
to
é 1 2ù
(A)   A3 (B)   B2     (C)   O (D)   I ê- ú
(A)   é -2 4 ù (B)   ê 5 5 ú  
-1 2 ê -3 1ú ê- 3 1 ú
é1/ 25 0 ù ìï é 5 0 ù üï ë û
67. If ê ú = íê ú ý , then the value of x is êë 10 5 úû
ë x 1/ 25û ï ë -a 5 û ï
î þ é 6 2ù é -16 -6 ù
(C)   ê ú (D)   ê
(A)   a/125 (B)   2a/25  (C)   2a/125 (D)  None of these ê11 2 ú ë 7 2 úû
êë 2 úû

Mathematical Problem Book for JEE.indb 778 07-06-2018 12:51:30


Chapter 18 | Matrices and Determinants 779

éa 0 ù é1 0ù 85. The system of equations 102x + 95y + 88z = 81, 3x + 10y + 17z


77. If A = ê ú and B = ê5 1ú , then the value of a for which = 24, 57x + 50y + 43z = 36 has
ë 1 1 û ë û
(A)   Many solutions (B)   No solution
A2 = B is
(A)   1    (B)   -1    (C)   4    (D)   No real values (C)   A unique solution (D)   None of these.

é1 0 0 ù x2 ( x - 1)2 ( x - 2)2
78. If A = êê0 1 0 úú , then the trace of the matrix A is equal to 86. If ax3 + bx2 + cx + d = ( x - 1)2 ( x - 2)2 ( x - 3)2 , then
êëa b -1úû
( x - 2)2 ( x - 3)2 ( x - 4 )2
(A)   1    (B)   -1    (C)   0    (D)   a + b
79. Let A and B are the non-singular square matrices, then which (A)   a = 1, b = 2, c = 3, d = -8
of the following is always correct? (B)   a = -1, b = 2, c = 3, d = -8
(A)   (AB)θ = Aθ Bθ (B)   (AB)′ = B′A′ (C)   a = 0, b = 0, c = 0, d = 8
(C)   A(adj B) = B (adj A) (D)  |adj A| = |A|n-2 (D)   a = 0, b = 0, c = 0, d = -8

80. If lr2 + mr2 + nr2 = 1 , where r = 1, 2, 3 and l1l2 + m1m2 + n1n2 = 0 1 sin A sin2 A
2
l1 m1 n1 87.  If in a DABC, 1 sin B sin2 B = 0, then the triangle is
…etc, then, D = l2 m2 n2 is equal to 1 sin C sin2 C
l3 m3 n3
(A)   Equilateral or isosceles
(A)   -1   (B)   1    (C)  ±1   (D)   3
(B)   Equilateral or right angled
81. If a is a root of x4 = 1 with negative principal argument, (C)   Right angled or isosceles
then the principal argument of D(a) where
(D)   None of these
1 1 1 88. If a, b and c are sides of a DABC and
D(a) = a n a n +1 a n + 3 is
a2 b2 c2
1 1 2 2
0 (a + 1) (b + 1) (c + 1)2 = 0, then
a n +1 a n
(a - 1)2 (b - 1)2 (c - 1)2
5p 3p p p
(A)     (B)  -    (C)     (D)  -
4 4 4 4
(A)   ABC is an equilateral triangle
x a a (B)   ABC is a right angled triangle
82. If a, b > 0 and D(x) = b x a , then (C)   ABC is an Isosceles triangle
b b x (D)   None of these
(A)   D(x) is increasing in ( - ab , ab ) 10
C4 10
C5 11
Cm
(B)   D(x) is decreasing in ( ab , ∞ ) 11 11 12
89.  The value of C6 C7 Cm+2 is zero, when m is
(C)   D(x) has a local minimum at x = ab 12 12 13
(D)   None of these C8 C9 Cm+ 4

cos( x + a ) cos( x + b ) cos( x + g ) (A)   6 (B)   4   (C)   5 (D)   None of these


83. If f (x) = sin( x + a ) sin( x + b ) sin( x + g ) and f (2) = 5, 2 1 [sin2 q ]
sin( b - a ) sin(g - a ) sin(a - b )
90. If D = [sin2 q ] cosq i (here [⋅] is greatest integer
20
i 1 sinq
then å f (r ) is equal to
r =1
(A)   5   (B)   10    (C)   100   
(D)   None of these function and i = -1 ), then arg(D) ∈

(A)   {- tan-1 2, - p + tan-1 2}


æp ö
84. 
The value of f ç ÷ where
è6ø æ -3p -p ö -1 -1
(B)   ç , ÷ ∪ {- tan 2, -p + tan 2}
2
cos q cosq sinq - sinq è 4 2 ø
2
f(q) = cosq sinq sin q cosq is é -3p -p ù
(C)   ê ,
sinq - cosq 0 ë 4 2 úû

(A)   0    (B)   1   (C)   2 (D)   3 (D)   None of these

Mathematical Problem Book for JEE.indb 779 07-06-2018 12:51:49


780 Mathematics Problem Book for JEE

98. If a, b and c are positive integers and x = cy + bz, y = az + cx,


ex sin 2 x tan x 2
z = bx + ay, where x, y and z are not all zero, then number of
91. If D(x) = ln(1+ x ) cos x sin x = A + Bx + Cx2 + …, then B ordered triplet (a, b, c) satisfying above is
cos x 2 e x - 1 sin x 2 (A)   0 (B) 1
is equal to (C)   Finitely many (D)   Infinitely many
(A)   0 (B)   1   (C)   2 (D)   None of these
Practice Exercise 2
92. Let x1y1z1, x2y2z2 and x3y3z3 be three 3-digit even numbers

x1 y1 z1 Single/Multiple Correct Choice Type Questions


and D = x 2 y2 z2 . Then, D is 1. If A is matrix of size n × n such that A2 + A + 2I = 0, then
x3 y3 z3 (A)   A is non-singular (B)   A is symmetric
(A)   Divisible by 2 but not necessarily by 4 1
(C)   |A| ≠ 0 (D)   A-1 = - (A + I)
(B)   Divisible by 4 but not necessarily by 8 2
(C)   Divisible by 8
(D)   None of these
93. If a, b , g are the roots of x3 + ax2 + b = 0, then the determinant 1
2r -1 sin rq
a b g r (r + 1) n
 2. If Dr = x y z then å D r is
D = b g a equals r =1
g a b æ n + 1ö n
n
sin ç ÷q sin q
n
2 -1 è 2 ø 2
(A)   -a3 (B)   a3 - 3b (C)   a2 + 3b (D)   a3 n +1 q
sin
94. Let a, b, c be cube roots of unity and 2
a2 + b 2 c2 c2 equal to
2 2 2 2
(A)   0 (B)   Independent of n
D= a b +c a , then (C)   Independent of q (D)   Independent of x, y and z
2 2
b b c + a2
2
x+a b c
(A)   Re(D)   = 0 (B)   Im(D) = 0  3.  The value of x satisfying a x +b c = 0 is
(C)   Re(D) + Im(D) = 0 (D)   Re(D)Im(D) = 4
a b x+c
95. Given q2 - pr < 0, p > 0, then the value of (A)   0 (B)   a + b + c
p q px + qy (C)   -(a + b + c) (D)   None of these
D= q r qx + ry is 1 2 2 
px + qy qx + ry 0  
 4. Let A = 2 1 2  . Then
(A)   0 (B)   Positive (C)   Negative (D)   q2 + pr
2 2 1
96. If p, q, r are in AP, then the determinant 1
(A)  A - 4A - 5I3 = 0
2 (B)  A-1 = (A - 4I3)
5
(C)  A3 is not invertible (D)  A2 is invertible
a2 + 2n +1 + 2 p b2 + 2n + 2 + 3q c 2 + p
 5. 
If A and B are invertible square matrices of the same order,
2n + p 2n +1 + q 2q is equal to then which of the following is correct?
a2 + 2 n + p b2 + 2n +1 + 2q c2 - r (A)  adj(AB) = (adjB) (adjA)
(B) (adjA)′ = (adjA′)
(A)   1 (B)   0 (C)  |adjA| = |A|n - 1, where n is the order of matrix A
(D) adj(adjB) = |B|n - 2 B, where n is the order of matrix B
(C)   a2b2c2 - 2n (D)   (a2 + b2 + c2) - 2nq
3 3 4 
 
-1 a a2  6. If A = 2 3 4  . Then
 
97.  For the determinant D = sin x cos y sin x (a ∈ R) 0 1 1
cos y sin x cos y (A)  adj(adjA) = A (B) |adj(adjA)| = 1
(A)  If D is non-zero, it is independent of a (C) |adjA| = 1 (D)  None of these
(B)  D is always independent of a  7.  System of equation
(C) If D is independent of a, then sin x + sin y can be equal x + 3y + 2z = 6
3 x + ly + 2z = 7
to
2 x + 3y + 2z = m
(D)   None of these has

Mathematical Problem Book for JEE.indb 780 07-06-2018 12:52:00


Chapter 18 | Matrices and Determinants 781

(A)  Unique solution if l = 2, m ≠ 6 For example, to find right inverse of matrix


(B)  Infinitely many solution if l = 4, m = 6 1 1
 
(C)  No solution if l = 5, m = 7 A = 1 1 
(D)  No solution if l = 3, m = 5  
2 3 
 8.  Which of the following statement is always true? we take
(A)  Adjoint of a symmetric matrix is a symmetric matrix
x y z 
(B)  Adjoint of a unit matrix is unit matrix R =  
(C)  A (adj A) = (adj A) A u v w 
(D)  Adjoint of a diagonal matrix is diagonal matrix and solve AR = I3 that is,
1 1 1  1 1 1 0 0 
    x y z   
 9. If A =  1 1 1  , then
 1 1     0 1 0 
  u v w   
 1 1 1  2 3  0 0 1

(A)  A3 = 9A (B)  A3 = 27A ⇒ x - u = 1 y - v = 0 z-w=0
(C)  A + A = A2 (D)  A-1 does not exist x + u = 0 y + v = 1 z+w=0
2x + 3u = 0 2y + 3v = 0 2z + 3w = 1
Comprehension Type Questions As this system of equations is inconsistent, we say there is no right
1 0 0  inverse for matrix A.
 
Paragraph for Questions 10-12: Let A = 2 1 0  . If U1, U2 and 16. Which of the following matrices is not left inverse of matrix
  1 1
3 2 1  
U3 are columns matrices satisfying 1 1  ?
 
1  2  2  2 3 
     
AU1  0  , AU2  3  , AU3  3  1 1 
       0  2 7 3 
0  0  1   
(A)   2 2 
 (B)   1 1 
and U is 3 × 3 matrix whose columns are U1, U2­ and U3 then  1 1  0
 2 2 0   2 2 

10.  The value of |U| is
 1 1 
(A)   3 (B)   -3 (C)   3/2 (D)   2  0 0 3 1
(C)   2 2 
 (D)   1 1


11.  The sum of the elements of U-1 is  1 1
 2 2 0  
 2 2
0

(A)   -1 (B)   0 (C)   1 (D)   3 
1 1 2 
3  17.  The number of right inverses for the matrix   is
  2 1 1
12.  The value of [3 2 0] U 2  is
  (A)  0 (B) 1 (C) 2 (D) Infinite
0 
18. 
For which of the following matrices number of left inverses is
(A)   5 (B)   5/2 (C)   4 (D)   3/2
greater than the number of right inverses?
Paragraph for Questions 13-15: Consider the determinant  1 2 4 3 2 1
(A)    (B)   
a1 a2 a3 3 2 1 3 2 1
D = b1 b2 b3
1 4  3 3 
d1 d2 d3    
(C)  2 3 (D)  1 1
Mij = Minor of the element of i th row and j th column    
5 4  4 4 
Cij = Cofactor of the element of i th row and j th column
13.  Value of b1 × C31 + b2 × C32 + b3 × C33 is Paragraph for Questions 19-21: Let A = [aij]3 be a square matrix
(A)  0 (B)  D (C) 2D (D)  D2 of order 3 whose elements are distinct integers from 1, 2, …, 9. The
14. If all the elements of the determinant are multiplied by 2, matrix is formed so that the sum of numbers in every row, column
then the value of new determinant is and diagonal is a multiple of 9.
(A)  0 (B) 8D (C) 2D  (D) 29 × D
19. The number of possible combinations of three distinct num-
15.  a3 M13 - b3 × M23 + d3 × M33 is equal to bers from 1 to 9 that have a sum of 9 or 18 is
(A)  0 (B) 4D (C) 2D  (D)  D (A)  10 (B)  7 (C)  8 (D)  9
Paragraph for Questions 16-18: Let A be an m × n matrix. If there 20.  The element a22 must be a multiple of
exists a matrix L of type n × m such that LA = In, then L is called left (A)  2 (B)  3 (C)  4 (D)  9
inverse of A. Similarly, if there exists a matrix R of type n × m such 21.  The maximum value of trace of the matrix A is
that AR = Im, then R is called right inverse of A. (A)  18 (B)  19 (C)  12  (D)  none

Mathematical Problem Book for JEE.indb 781 07-06-2018 12:52:14


782 Mathematics Problem Book for JEE

Matrix Match Type Questions Integer Type Questions


22.  Match the following: é 1 -1 1 ù
Column I Column II 24. If A = êê 0 2 -3 úú and B = (adj A) and C = 5A, then find the
(A) 
A is a real skew-symmetric matrix (p)  BA - AB
ëê 2 1 0 úû
such that A2 + I = 0. Then | adjB |
A is a matrix such that A2 = A. If (I + A)n
(B)  (q) A is of even value of .
|C |
= I + l A, then l equals (n ∈ N) order
If for a matrix A, A2 = A, and B = I - A,
(C)  (r)  A 25. If A = é1 2 ù and f (x) = (1 + x) (1 - x)-1 and f (A)  = -l A, then
then AB + BA + I - (I - A)2 equals ê1 1ú
ë û
(D) 
A is a matrix with complex entries (s) 2n - 1
and A* stands for transpose of find the value of l.
complex conjugate of A. If A* = A éa b c ù
and B* = B, then (AB - BA)* equals 26. If A = êb c a ú , abc = 1, A′A = I, then find maximum value
ê ú
(t) nC1 + nC2 ... êë c a b úû
+ nCn
of a3 + b3 + c3.
23.  Match the following:
é 1 0ù
Column I Column II 27. If A = ê ú and A = 8 A + KI2, then find the value of |k|.
2
ë -1 7 û
Let |A| = |aij|3 × 3 ≠ 0. Each element aij is
(A)  (p) 0
multiplied by ki - j. Let |B| the resulting 28. 
Find the coefficient of x in the determinant
determinant, where k1|A| + k2|B| = 0. (1+ x )a1b1 (1+ x )a1b2 (1+ x )a1b3
Then k1 + k2 =
(1+ x )a2 b1
(1+ x ) a2 b2
(1+ x )a2 b3 , where ai , bj ∈ N
(B) 
The maximum value of a third-order (q) 4
(1+ x )a3b1 (1+ x )a3b2 (1+ x )a3b3
determinant each of its entries are ±1
equals cos( x + a ) cos( x + b ) cos( x + g )
1 cos a cos b (r) 1 29. If f(x) = sin( x + a ) sin( x + b ) sin( x + g ) and f(2) = 6,
(C) cos a 1 cos g = sin( b - g ) sin(g - a ) sin(a - b )
cos b cos g 1 25
1
0 cos a cos b
then find
5 å f (r ) .
r =1
cos a 0 cos g x 1 1
cos b cos g 0 30. Let f(x) = sin 2px 2 x 2
1 . If f(x) be an odd function and its
if cos2a + cos2b + cos2g = x 3
3x 4
1
x2 + x x +1 x -2 (s) 2 odd value is equal to g(x), then find the value of λ. Also f(1)
g(1) = -4λ
(D)  2 x 2 + 3 x - 1 3x 3 x - 3 = Ax + B,
2
x + 2x + 3 2x -1 2x -1 f ( x + 1) f ( x + 8 ) f ( x + 1)
where A and B are determinants of order 3. 31. If f(x) satisfies the equation 1 2 -5 =0
Then A + 2B = 2 3 l
1 2
(t)  for all real x and if f is periodic with period 7, then find the
2 4 value of |l|.

Answer Key
Practice Exercise 1
 1. (A)   2. (D)   3. (A)   4. (A)   5. (A)   6. (A)
  7. (C)  8. (B)   9. (A) 10. (D) 11. (C) 12. (C)
13. (C) 14. (C) 15. (B) 16. (B) 17. (A) 18. (C)

Mathematical Problem Book for JEE.indb 782 07-06-2018 12:52:24


Chapter 18 | Matrices and Determinants 783

19. (A) 20. (B) 21. (A) 22. (D) 23. (B) 24. (B)


25. (D) 26. (C) 27. (B) 28. (A) 29. (C) 30. (D)
31. (C) 32. (C) 33. (B) 34. (D) 35. (A) 36. (D)
37. (B) 38. (D) 39. (B) 40. (A) 41. (A) 42. (B)
43. (D) 44. (B) 45. (C) 46. (C) 47. (A) 48. (A)
49. (A) 50. (C) 51. (B) 52. (A) 53. (A) 54. (C)
55. (B) 56. (C) 57. (C) 58. (A) 59. (A) 60. (B)
61. (A) 62. (B) 63. (B) 64. (D) 65. (B) 66. (C)
67. (C) 68. (C) 69. (D) 70. (B) 71. (A) 72. (D)
73. (B) 74. (B) 75. (C) 76. (C) 77. (D) 78. (A)
79. (B) 80. (B) 81. (B) 82. (C) 83. (C) 84. (B)
85. (A) 86. (D) 87. (A) 88. (C) 89. (C) 90. (D)
91. (A) 92. (A) 93. (D) 94. (B) 95. (C) 96. (B)
97.  (D) 98. (A)

Practice Exercise 2
 1.  (A), (C), (D)   2.  (A), (B), (C), (D)   3.  (A), (D)   4.  (A), (B), (D)   5.  (A), (B), (C), (D)   6.  (A), (B), (C)
  7.  (B), (C), (D)   8.  (A), (B), (C), (D)  9.  (A), (D) 10. (A) 11. (B) 12. (A)
13. (A) 14. (B) 15. (D) 16. (C) 17. (D) 18. (C)
19. (A) 20. (B) 21. (A) 22. (A) → (q), (B) → (s, t), (C) → (r), (D) → (p)
23. (A) → (p, t), (B) → (q), (C) → (r), (D) → (p, t) 24. 1 25. 1 26.  2, 4
27. 7 28. 0 29. 150 30.  -1 31. 4

Solutions
Practice Exercise 1 é3ù
é7 1 2 ù ê ú é 4 ù é21+ 4 + 10 ù é 8 ù é 35 ù é 8 ù é 43 ù
  6. ê ú ê 4ú + 2 ê ú = ê ú + ê ú= ê ú + ê ú=ê ú
é 1 0 0 ù é 1 0 0 ù é 1 0 0ù ë9 2 1û ê 5 ú ë 2 û ë 27 + 8 + 5 û ë 4 û ë 40 û ë 4 û ë 44 û
  1.  A = êê0 1 0 úú êê0 1 0 úú = êê0 1 0 úú = I (Unit matrix)
2 ë û
êëa b -1úû êëa b -1úû êë0 0 1úû é10 0 ù é 1 0ù
  7.  A (adj A) = ê ú = 10 ê0 1ú =10I. Also A(adj A) = |A|I.
ë 0 10 û ë û
  2.  A adjA = A A 2 = A 3 = (a3 )3 = a9
Hence, |A| = 10.
  3. A = 0 as the determinant of any skew-symmetric matrix of   8. Let A be symmetric as well as skew-symmetric matrix. Then
odd order is zero. A¢ = A and A¢ = - A
  4.  If we consider m = 2 ⇒ A = -A or 2A = 0 or A = 0
(A + B)2 = (A + B) (A + B) = A2 + AB + BA + B2   9.  AB = 0 ⇒ AB = 0 ⇒ A B = 0 ⇒ either A = 0 or B = 0
= A2 + 2AB + B2 (if AB = BA)
Similarly é0 0 0 ù
ê ú
(A + B)m = mC m-1B + mC Am-2B2 + … 10.  A = ê 1 0 0 ú
0A + C1A +
m m mC m
2 mB
êë0 1 0 úû
holds if AB = BA.
é0 0 0 ù é0 0 0 ù é0 0 0 ù
 5.  We know that [aij ]¢ = -[aij ] for skew-symmetric matrix. Then
A = êê 1 0 0 úú êê 1 0 0 úú = ê0 0 0 ú
2
ê ú
[aji] + [aij] = 0 êë0 1 0 úû êë0 1 0 úû êë 1 0 0 úû
⇒aji + aij = 0 é0 0 0 ù é0 0 0 ù é0 0 0 ù
For i = j, we get A3 = ê 1 0 0 ú ê0 0 0 ú = ê0 0 0 ú
aii + aii = 0 ⇒ aii = 0 ê úê ú ê ú
êë0 1 0 úû êë 1 0 0 úû êë0 0 0 úû
This means diagonal elements of every skew-symmetric
matrix are zero. Hence, A3 = 0.

Mathematical Problem Book for JEE.indb 783 07-06-2018 12:52:37


784 Mathematics Problem Book for JEE

é0 0 0 ù é 7 -1 -1ù é 7 -3 -3ù
11.  A = êê0 0 0 úú ê ú
C A = ê -3 1 0 ú ⇒ adj A = ê -1 1 0 ú
ê ú
êë0 1 0 úû êë -3 0 1 úû êë -1 0 1 úû
Since A2 = 0, hence, A is a nilpotent matrix with index 2.
Therefore,
12. A and B are symmetric matrices of same order, i.e. A = A′, B = B′. é 7 -3 -3ù
1
Then A-1 = adj A = êê -1 1 0 úú
(AB + BA)T = (AB)T + (BA)T A
êë -1 0 1 úû
  = BT × AT + AT × BT = BA + AB = AB + BA
éi -i ù é 1 -1ù
So, AB + BA is a symmetric matrix. 21.  A = ê ú and B = ê ú
ë -i iû ë -1 1 û
13.  A is a square matrix
(AA′)′ = (A′)′⋅A′ = AA′ éi -i ù é i -i ù é -2 2 ù é 1 -1ù
A2 = ê = = -2 ê ú = -2B
i úû êë -i i úû êë 2 -2 úû
Hence, AA′ is a symmetric matrix.
ë -i ë -1 1 û
14.  A is a square matrix and A3 = I ⇒ A3 = I ⇒ A2 × A = I. So A2 is
the inverse of A  1 -1  1 -1  2 -2  1 -1
A4 = 4 B 2 = 4     = 4  = 8  = 8B
Hence, A-1 = A2.  -1 1   -1 1   -2 2   -1 1 
15.  A is a square matrix é 2 -2 ù é 1 -1ù
(A - A′)′ = A′ - (A′)′ = A′ - A A8 = 64 B 2 = 64 ê ú = 128 ê ú = 128B
So, A - A′ is not a symmetric matrix. ë -2 2 û ë -1 1 û

16. Because determinant of every skew-symmetric matrix of odd 22.  Because cos(-θ) = cos θ ⇒ A is a symmetric matrix.
Alternative solution:
order is zero, therefore, A = 0 if n is odd.
Given determinant can also be written as the product of two
17.  Each diagonal element of skew-symmetric matrix is zero. determinants as follows:
é 4 2ù 1 cos(a - b ) cos(a - g )
18.  A = ê ú
ë -1 1û cos( b - a ) 1 cos( b - g )
cos(g - a ) cos(g - b ) 1
é 4 2 ù + é -2 0 ù é 2 2 ù
A - 2I = ê

ú ê ú=ê ú cos a sina 0 cos a sina 0
ë -1 1û ë 0 -2 û ë -1 -1û
= cos b sin b 0 ´ cos b sin b 0  (row by row)
é 4 2 ù + é -3 0 ù é 1 2 ù
A - 3I = ê ú ê ú=ê ú cos g sin g 0 cos g sing 0
ë -1 1û ë 0 -3û ë -1 -2 û
=0×0=0
Hence,
1 a c
é 2 2 ù é 1 2 ù é0 0 ù
(A - 2I) (A - 3I) = ê ú=ê ú =ê ú Also A = a 1 b where a = cos (a - b ), b = cos (b - g ),
ë -1 -1û ë -1 -2 û ë0 0 û b c 1
19.  O(A) = m × (n + 5) and O(B) = m × (11 - n) c = cos (g - a)
AB exists ⇒ n + 5 = m ⇒ m - n = 5  (1) = 1 - a - b - c + 2abc
2 2 2
BA exists ⇒ 11 - n = m ⇒ m + n = 11  (2) = 1 - [cos2 (a - b ) + cos2 (b - g ) + cos2 (g - a) - 2 cos (a - b)
Solving Eqs. (1) and (2), we have cos (b  - g ) cos (g - a)]
m = 8 and n = 3
= 1 - [1+ cos2 (a - b) - sin2 (b - g ) + cos (g - a){cos (g - a)
Therefore, O(A) = 8 × 8 and O(B) = 8 × 8 - 2 cos (a - b ) cos (b - g )}]
Therefore, AB and BA both are square matrices of order
= 1 - [1 + cos (a - g ) cos (a - 2β + g ) + cos (g - a){cos (g - a)
8 × 8.
- cos (a - g ) cos (a - 2b  + g )}]
é1 3 3 ù = 1 - [1 + cos (a - g ) cos (a - 2b  + g ) - cos (g - a ) cos (a - 2b + g )]
20.  A = êê1 4 3 úú =0
êë1 3 4 úû
23. If A ≠ 0 , then homogeneous system of linear equations
1 3 3 AX = 0 has only trivial solution, i.e. X = 0.
A= 1 4 3
24. [[A, B], C] + [[B,C], A] + [[C,A], B]
1 3 4
= [AB - BA, C] + [BC - CB, A] + [CA - AC, B]
  = 1(16 - 9) - 3(4 - 3) + 3(3 - 4)
  =7-3-3=1 = (AB - BA)C - C(AB - BA) + (BC - CB)A - A(BC - CB) + (CA - AC) B

Mathematical Problem Book for JEE.indb 784 07-06-2018 12:52:53


Chapter 18 | Matrices and Determinants 785

- B(CA - AC) 34.  We can write D(x) as product of two determinants as follows:
= ABC - BAC - CAB + CBA + BCA - CBA - ABC sin x cos x 1 cos x sin x 0
+ ACB + CAB - ACB - BCA + BAC = 0 D (x) = 1 1 1 ´ ex x 0 =0
ex 0 0 1 x2 0
25. [A(A + B)-1B]-1 = B-1 (A + B) A-1 which cannot be simplified
further, in general. which is independent of (x)⇒D′ (x) = 0 ∀ x.
26.  A and B matrices commute so AB = BA. g( x ) é0 ù
A-1 × B-1 = (BA)-1 = (AB)-1 = B-1 × A-1 35.  lim
x
ê 0 form as g(0 ) = 0 ú
x ®0 ë û
So, A-1 and B-1 also commute. g¢( x )
lim = g¢(0 ) = 0
27. A, B, C are three conformable matrices for multiplication 1
x ®0

(ACB)-1 = B-1×C-1×A-1. Hence, (A) is the correct answer.


1 0 0  36.  Note that D(x) is a polynomial of degree at most 6 in x.
  If D (x) = a0 + a1x + a2x2 + … + a6x6, then
28.  A = 0 1 0 
a b -1 D′(x) = a1 + 2a2x + … + 6a6x5 ⇒ a1 = D′(0).
Now
 1 0 0   1 0 0   1 0 0
A = 0 1 0  0 1 0  = 0 1 0 
2 4 ( x - 2)2 x3
a b -1 a b -1 0 0 1 D′(x) = 8 ( x - 2 2 )2 ( x + 1)3
2
A is a unit matrix of third order. 12 ( x - 2 3 )2 ( x - 1)3
29. Trace of a skew-symmetric matrix is always equal to zero. Now 4x - 4 2( x - 2) x3
A = - A′ ⇒ ∑ aii = 0 + 8x - 4 2 2( x - 2 2 ) ( x + 1)3

30. After reflection in line x + y = 0, y becomes x. Therefore, we 12 x - 4 3 2( x - 2 3 ) ( x - 1)3


need a matrix which when multiplied by
4x - 4 ( x - 2)2 3x2
é x1 y1 ù é - y1 - x1 ù
ê x y ú changes it to ê - y - x ú + 8x - 4 2 (x -2 2) 2
3( x + 1)2
ë 2 2û ë 2 2û

We observe that 12 x - 4 3 ( x - 2 3 )2 3( x - 1)2



é x1 y1 ù é 0 -1ù é - y1 - x1 ù -4 4 0
êx =
ë 2 y 2 úû êë -1 0 úû êë - y 2 - x 2 úû ⇒ D′(0) = 0 + 0 + -4 2 8 3
é 0 -1ù -4 3 12 3
Hence, matrix of transformation is ê ú.
ë -1 0 û ⇒ D′(0) = 48(1 + 2 - 3 )
31. The degree of any term of the determinant is
n + n + 2 + n + 3 = 2 ⇒ n = -1 x c -b
Hence, (C) is the correct answer. 37.  We have 2 D = - c x a
N b -a x
æ N(N + 1) ö
32.   å 2r = 2 çè 2 ø
÷ = N (N+1) Cofactors of 1st row are: x2 + a2, ab + cx, ac - bx
r =1
Cofactors of 2nd row are: ab - cx, x2 + b2, ac + bx
N
æ (N + 1)(2N + 1) ö Cofactors of 3rd row are: ac + bx, bc - ax, x2 + c2
å (6r 2 - 1) = 6N çè 6
÷-N
ø
Therefore, determinant of cofactors of ∆2 is
r =1
x 2 + a2 ab + cx ac - bx
= N(2N2 + 2N + N + 1) - N = 2N3 + 3N2 = N2 (2N + 3)
2 2
ab - cx x +b ax + bc
N
å (4r 3
- 2Nr ) = N3 (N + 1) ac + bx bc - ax x2 + c2
r =1

Hence, (C) is the correct answer. x 2 + a2 ab - cx ac + bx


= ab + cx x +b2 2
bc - ax
33. Since 1+ w + w 2 = 0 , the given determinant is
1 1 1 3 0 0 ac - bx ax + bc x2 + c2
1 w w2 = 1 w w 2 (R1→ R1 + R2 + R3) = ∆22
2
(∵ adj A = A where A is of order)
1 w2 w 1 w2 w Thus, ∆1 = ∆22.
= 3(w 2 - w 4 ) = 3(w 2 - w ) = 3w (w - 1) By applying C1 → C1 + C2 + C3, we get D = 0.
38. 
Hence, (B) is the correct answer. Hence, (D) is the correct answer.

Mathematical Problem Book for JEE.indb 785 07-06-2018 12:53:16


786 Mathematics Problem Book for JEE

p /2
1
39.  Direct expansion gives f(x) = cos 3x. Hence, ò cos 3 xdx = - .
3
48.  R1 → R1 -R2, R2 → R2 - R3
0
Hence, (B) is the correct answer. sin( A + B ) sin( A - B ) cot A - cot B 0
 /2 D = sin(B + C ) sin(B - C ) cot B - cot C 0
40.  f(-x) = -f(x). It is an odd function. Hence,  f ( x )  0.
sin2 C
cot C 1
 / 2
Hence, (A) is the correct answer. sin(B - A)
sin C sin( A - B ) 0
3 1 4 sin A × sin B
=
  sin(C - B )
41. Putting x = 0, we get e = 1 2 3 . sin A sin( B - C ) 0
sin B × sin C
0 1 1
sin2 C cot C 1
3 1 1
C3 → C3 - C1 gives 2 2 2 = 0 Expanding along the third column, we have
0 1 1 sin( A - B ) × sin(C - B ) sin(B - C ) × sin(B - A)
D= -
sin B sin B
42. By applying R3 → R3 - a R1 - R2, we get
sin( A - B )
( b2 - ac ) ( aa2 + 2ba + c) = 0 [ - sin(B - C ) + sin(B - C )] = 0
sin B
Hence, (B) is the correct answer.
49. We know that
43.  Use concept of polynomial roots. Sum of the roots = 0. a c b a c b
By applying R1 → R1 + R2 + R3 and using a + b + g, we get D = 0. 3 3 3 2
(a + b + c - 3abc ) = b a c b a c
44. Here D = 0 for k = 3, 1; Dx = 0 for k = 2, 1, Dy = 0 for k = 1. c b a c b a
Hence, k = 1. A B B B A B
Alternate method: = B A B = B B A
For infinitely many solutions the two equations become
B B A A B B
identical, so    
k +1 8 4k
= = ⇒ k =1 æ A+B +C ö
k k + 3 3k - 1 50. sin(A+B+C) = cos ç ÷ = 0. Hence
è 2 ø
45. D(0) = 0 and D′(0) = 0 ⇒ x is a repeated root of D ⇒ D is A B C 1
D = sin sin sin ≤
divisible by x2. 2 2 2 8
46. If given homogeneous system has non-trivial solution then x -5 2 5
C1 - C2, C2 - C3
51.  D = 0 x 3
a 1 1 a -1 0 1 5- x 4 x
1 b 1 = 0 Þ 1- b b - 1 1 = 0 1 2 5
1 1 c 0 1- c c ⇒ (x - 5) 0 x 3 = 0 (C1 → C1 - C3)
Expanding along R1 we get -1 4 x

(a - 1)[(b - 1)c - (1 - c)] + (1 - b)(1 - c) = 0 0 6 5+ x


⇒ (x - 5) 0 x 3 = 0 (R1 → R1 + R3)
⇒ (1 - a)(1 - b)c + (1- a)(1 - c) + (1 - b)(1 - c) = 0
-1 4 x
Dividing by (1 - a)(1 - b)(1 - c), we get
⇒ (x - 5) [18 - x(5 + x)] = 0
c 1 1 Therefore, the non-integral roots are the roots of
+ + =0
1- c 1- b 1- a x2 + 5x -18 = 0
 1- c - 1 1 1 52. Using R2 → R2 - R3 we get
⇒ - + + =0
 1- c  1- b 1- a 1 1 1
1 1 1 D = 2 ⋅ 2 x ⋅ 2 ⋅ 2 - x 2 ⋅ 3 x ⋅ 2 ⋅ 3- x 2 ⋅ 5 x ⋅ 2 ⋅ 5- x =4
⇒ + + =1
1- a 1- b 1- c x
(2 - 2 ) -x 2 x
(3 - 3 ) -x 2 x
(5 - 5 ) -x 2

1 a 0 1 1 1
=0
47.  0 1 a = 0 Þ 1+ a(a2 ) = 0 Þ a3 = -1 Þ a = -1 ⇒ 1 1 1
a 0 1 (2 x - 2- x )2 (3 x - 3- x )2 (5 x - 5- x )2
Hence, (A) is the correct answer. Hence, (A) is the correct answer.

Mathematical Problem Book for JEE.indb 786 07-06-2018 12:53:36


Chapter 18 | Matrices and Determinants 787

53. From the given conditions, Applying C1 → C1 + C2 + C3, we get


y-x z-y z-x sin x + 2 cos x cos x cos x
≠ ±1, ≠ ± 1, ≠±1
b-a c -b c -a D = sin x + 2 cos x sin x cos x
⇒x+a≠y+b≠z+c sin x + 2 cos x cos x sin x
The determinant is a symmetric one. The determinant will be 1 cos x cos x
equal to zero if x + a + y + b + z + c = 0
= (sin x + 2 cos x) 1 sin x cos x
But a + b + c = 0 (given). So
1 cos x sin x
x+y+z=0
 y y Applying R2 → R2 - R1 and R3 → R3 - R1, we get
⇒ x + z = 2   ⇒ x, - , z are in AP
 2 2 1 cos x cos x
Hence, (A) is the correct answer. D = (sin x + 2 cos x) 0 sin x - cos x 0
n - 1 n2 - n n2 0 0 sin x - cos x
54. The determinant = n n2 - 1 n2 + n D = (sin x + 2 cos x)(sin x - cos x)2
n - 1 n2 + 1 n2 Thus, D = 0. This gives
(sin x + 2 cos x)(sin x - cos x)2 = 0
n - 1 n2 - n n2 ⇒ tan x = - 2 and tan x = 1
p p
= n n2 - 1 n2 + n (R3 → R3 - R1) As - ≤ x ≤ , we get - 1 ≤ tan x ≤ 1. Since, tan x = 1 we have
4 4
0 n +1 0 p
x=
4
= - (n + 1)[n(n2 - 1) - n3] = n(n + 1) = 72 = 8 × 9
Hence, (C) is the correct answer.
2
1 a a 1 1 1 æ 1ö
x 2 ç - ÷ - (1- ln x )2 x
55. D = 1 b b2 1- ln x xø -1- 2 + 2 ln x
a b c 58.  f ′ (x) = , f ¢¢( x ) = è =
x2 x4 x3
1 c c 2 a2 b2 c2
⇒ x3f  " (x) = 2ln x - 3
2
1 1 1 2 1
D = ln x(2) - x    = 2ln x - 3 ⇒ D = x3 f "(x)
= a b c = [(a - b )(b - c )(c - a)]2 x x
a2 b2 c2 Hence, (A) is the correct answer.

Hence, (B) is the correct answer. 59.  As the given system of equations has a non-trivial solution,
1 1 1 l b-a c -a
cot A /2 cot B /2 cot C /2 =0
a-b l c -b = 0
56. 
tan B /2 + tan C /2 tan C /2 + tan A/2 tan A/2 + tan B /2 a-c b-c l
When λ = 0 then determinants become skew-symmetric
Operating C1 - C2 and C2 - C3 determinants of odd order, which is equal to zero. Thus, λ = 0.
0 0 1 60. A = A′. Then |A| = |A′| because the expansion of a determinant
tan B /2 - tan A/2 tan C /2 - tan B /2 row-wise is same as the expansion of a determinant column-
cot C /2 =0
tan A/2 tan B /2 tan B /2 tan C /2 wise.
tan B /2 - tan A/2 tan C /2 - tan B /2 tan A/2 + tan B /2
 1 0  0 1
61.  I =   , J =  -1 0 
⇒ (tan B/2 - tan A/2 )(tan C/2 - tan B/2) ×  0 1  
0 0 1  cosq sinq   1 0   0 1
B=   =   cosq +   sinq
1 1
cot C /2 =0  - sinq cosq  0 1  -1 0 
tan A/2 tan B /2 tan B /2 tan C /2    = I cos q + J sin q
1 1 tan A/2 + tan B /2
62.  In is an identity matrix.
Expanding along R1 we get
(tan B/2 - tan A/2 )(tan C/2 - tan B/2) (tan C/2 - tan A/2) = 0  -1 0 
63. A2 + I = 0 ⇒ A2 =   of (I is of second order) ⇒
⇒ A = B or B = C or C = A  0 -1
⇒ D must be an isosceles triangle  i 0
 
0 i  .
sin x cos x cos x  
57. Let D = cos x sin x cos x 64. Each diagonal element is either 0 or 1. So number of matrices
cos x cos x sin x = 24 - 1.

Mathematical Problem Book for JEE.indb 787 07-06-2018 12:53:56


788 Mathematics Problem Book for JEE

65.  A and B are symmetric. A = A′, B = B′. So 3 6 9 


(A + B)′ = A′ + B′ = A + B  
adj(adj A) = |A|A 3 9 15  
 (as |A| = 3)
 2  3 15 36 
0 b  a ab ac  0 0 0 
c
    
66.  AB   c a  ab b2 bc   0 0 0   0
0 72.  Given that ( A-1)T = A Þ ( AT )-1 = A Þ AAT = I Þ A = ±1
    
 b a 0  ac bc c 2  0 0 0  22 4
  Now, |adj(adj A)| = A = A =1
-1 1
67. Using A = adj A , we get a 2  a2 ab  ab ag  ac 
A a a  
  a b r 
 b    ab  ab b 2  b2 bg  bc 
 5 0  1 1 5 0  73.  b  a b c   
   
 a 5 

25 a 5 
g c   ag  ac bg  bc g 2  c 2 

  5 0  -1
2 a a 0 a b g
1 5 0 5 0
⇒    = a 5 a 5 = AA = b T
b 0 a b c =0
- a 5 625
      
g c 0 0 0 0

1  25 0 
=   Therefore, AAT is a singular matrix.
625 10a 25
74.  A and B are non-singular, so AB is non-singular. Hence
 1 25 0 
             =   AB adj(AB) = |AB| I (1)
2 a 125 1 25
AB(adj B adj A) = A(B adj B) adj A
Now
= A(|B| I) adj A
1 25 0   1 25 0  2a
   x  = |B| (A(adj A))
 x 1 25 2a 125 1 25 125 = |B| |A| I(2)
adj(AB) = (adj B) (adj A)
x +a b c
75. Let B = I + A + A2 + … + Ak-1 so that
68.  a x +b c =0
B(I - A) = (I + A + A2 + … + Ak-1) (I - A)
a b x +c
= I - A + A - A2 + … - Ak-1 + Ak-1 + Ak-1 - Ak
1 b c = I - Ak = I - 0 = I ⇒ B = (1 - A)-1
⇒ (x + a + b + c) 1 x + b c 1 4  16 6 
76. Let A    and B    . Then the matrix
1 b x +c 3 2   7 2 
1 b c equation is AX = B.
⇒ (x + a + b + c) 0 x 0 =0 1 -4
Since |A| = = -2 + 12 ≠ 0; A is an invertible matrix.
0 0 x 3 -2
Let Cij be the cofactors of elements aij in A = [aij].
⇒ x2 (x + a + b + c) = 0
Then
Hence, x = 0 or x = -(a + b + c). C11 = (-1)1+1 (-2) = -2
2 3  C12 = (-1)1+2 3 = -3
69.  A     A  4  15  19
5 2  C21 = (-1)2+1 (-4) = 4
C22 = (-1)2+2 1 = 1
 -2 -5  -2 -5
CA =   ⇒ adj A = C A′ =  -3 2  2 4 
 -3 2    Therefore, adj A =   , so
3 1
1 2 3 1 2 3  1
A1      A 1 1 2 4 
19 5 2  19 5 2  19 A-1 =adj A =  
| A| 10 3 1
70. If A is a skew-symmetric matrix of odd order, then |A| = 0. Now, AX = B ⇒ A-1(AX) = A-1B ⇒ X = A-1B
So, inverse does not exist.  6 2
1 2 4  16 6   
Let A be of even order. Then ⇒ X     11 

10 3 1 
 7 2 
  2
AA -1 = A -1A = I n ⇒ (AA -1) T = (A -1A) T = I n 2 
⇒ (A-1)T AT = AT(A-1)T = In ⇒ (A-1)T (-A) = (-A)(A-1)T = In a 0  a 0  a 2 0
77.  A2 =    =  
So, (A-1)T = -A-1 (inverse of a matrix is unique). 1 1  1 1 a + 1 1
71. adj(adj A) = |A|n - 2A, where n is the order of matrix. Since n = Clearly, no real value of a.
3 here 78.  Trace (A) = sum of diagonal elements = 1

Mathematical Problem Book for JEE.indb 788 07-06-2018 12:54:16


Chapter 18 | Matrices and Determinants 789

79. If A and B are the non-singular matrices, then 86. Apply C1 → C1 - C2; C2 → C2 - C3, we get
( AB )¢ = B¢A¢ is always correct.
(2 x - 1) (2 x - 3) ( x - 2)2
80.  Multiply determinant row to row and solve.
Hence, (B) is the correct answer. (2 x - 3) (2 x - 5) ( x - 3)2
81. Clearly, a = - i, where i2 = - 1. So (2 x - 5) (2 x - 7) ( x - 4 )2

R1 → R1 - R2 and R2 → R2 - R3 gives
1 1 1 1 1 1
1
D(a) = an × 1 a a 3 = 1 -i i 2 2 ( 2 x - 5)
an
1 i 1 0 2 2 (2 x - 7 )
1 0
a (2 x - 5) (2 x - 7) ( x - 4 )2

3p
= 1(- i) + 1 (i2) + (1 +i2) = - 1 - i, arg is - R1 → R1 - R2 gives
4
x a a 0 0 2
82.  Given that D(x) = b x a . We have 2 2 (2 x - 7 ) = - 8
b b x (2 x - 5) (2 x - 7) ( x - 4 )2
1 a a x 0 a x a 0 Therefore, the value of determinant is independent of x.
D′(x) = 0 x a + b 1 a + b x 0 a = b = c = 0 and d = - 8.
0 b x b 0 x b b 1 87. Since a = 2R sin A, b = 2R sin B, c = 2R sin C
  ⇒ D′(x) = 3(x2 - ab)
1 sin A sin2 A 1 a a2
Now sign scheme for D′(x) 1
1 sin B sin2 B = 3
1 b b2 = 0
+ – + 8 R
1 sin C sin2 C 1 c c2

– ab ab ⇒ (a - b)(b - c)(c - a) = 0
⇒ a = b or b = c or c = a
Local max Local min Therefore, at least two of a, b, c are equal. So, the triangle is
Since D(x) is increasing in ( -¥ , - ab ) È ( ab , ¥ ) , D (x) is isosceles or equilateral.
Hence, (A) is the correct answer.
decreasing in ( - ab , ab )
88. When a = b or b = c or c = a, the determinant reduces to zero.
D(x) has a local minimum at x = ab
It is not necessary that a = b = c for determinant to be zero.
D(x) has a local maximum at x = - ab Therefore, triangle is isosceles.
83. Clearly, f ′(x) = 0 ⇒ f(x) = constant. But f(2) = 5. Therefore, Hence, (C) is the correct answer.
f(x) = 5. Now 89.  C2 → C2 + C1
20 20
å f (r ) = å 5 = 5 × 20 = 100 10
C4 11
C5 11
Cm
r =1 r =1
11 12 12
Hence, (C) is the correct answer. D= C6 C7 Cm+2
12 13 13
84.  C1 → C1 - sin q C3 and C2 → C2 + sin q C3 C8 C9 Cm+ 4
For m = 5, C2 ≡ C3.
1 0 - sinq
Hence, (C) is the correct answer.
f(q ) = 0 1 cosq
0 ,sin q ≠ 1
2
sinq - cosq 0 90.  [sin2 q ] = 
2
Again R3 - sin q R1 + cos q R2, we get 1,sin q = 1
If sin q ≠ 1 ⇒ D = 2 sin q cos q - 2i - 1
2
1 0 - sinq
æp ö Re(D) = 2sin q cos q - 1
f(q ) = 0 1 cosq = 1 ⇒ f ç ÷ = 1
è6ø - 2 ≤ Re(D) ≤ 0
0 0 1 3p p
- ≤ arg D ≤ -
85.  Here determinant of coefficient matrix 4 2
102 95 88 If sin2 q = 1, sin q = ±1, cos q = 0
∆ = 3 10 17 arg(D) = arg(1 - 2i) or arg(- 1 - 2i)
57 50 43
ex 2 cos 2 x 2 x sec2 x 2
= 0 (using C1 → C1 + C3 - 2C2 ) 91. D′(x) = ln(1+ x ) cos x sin x
Similarly,
D1 = D2 = D3 = 0 cos x 2 ex -1 sin x 2
Hence, system has infinite many solutions.

Mathematical Problem Book for JEE.indb 789 07-06-2018 12:54:35


790 Mathematics Problem Book for JEE

⇒ (-ve) (+ve) < 0


ex sin 2 x tan x 2
ex sin 2 x tan x 2
1 96 Applying R1 → R1 - R3 and 2q = p + r we get
+ - sin x cos x + ln(1+ x ) cos x sin x
(1+ x )
2 x 2 -2 x sin x 2 ex 2 x cos x 2 2 n +1 - 2 n + p 2 n + 2 - 2 n +1 + q p+r
cos x e -1 sin x n n +1
2 +p 2 +q p+r
= B + 2Cx + … 2
a +2 + p n 2
b +2 n +1
+ 2q c2 - r
Put x = 0. So
2n + p 2 n +1 + q p+r
1 2 0 1 0 0 1 0 0 n n +1
= 2 +p 2 +q p+r =0
B = 0 1 0 + 1 0 -1 + 0 1 0 = 0 2 n 2 n +1
a +2 + p b +2 + 2q c 2 - r
1 0 0 1 0 0 0 1 0

97.  D = 1(sin2 x - cos2 y) + a2(sin2 x - cos2 y)


100 x1 + 10 y1 + z1 y1 z1 2 A y1 z1
92. D = 100 x 2 + 10 y 2 + z2 y2 z 2 = 2B y 2 z2 ,    = (1 + a2) (sin2 x - cos2 y)
100 x 3 + 10 y 3 + z3 y3 z3 2C y 3 z3 98. The given equations are x - cy - bz = 0, cx - y + az = 0 and
bx + ay - z = 0.
where A, B, C ∈ I It has non-trivial solutions so
A y1 z1 1 -c -b
=2
 B y2 z 2 ∈I c -1 a = 0
C y3 z3 b a -1
⇒ 1(1 - a2) + c(- c - ab) - b(ac + b) = 0
which is divisible by 2 but not necessarily by 4 or 8. ⇒ 1 - 2abc = a2 + b2 + c2 > 0
1
93. Because a , b, g are roots of x3 + ax2 + b = 0, therefore, ⇒ abc <
2
    a + b + g = -a, ab + bg + ga = 0, abg = -b
1
a b g So, 0 < abc < . Clearly, no triplet (a, b, c) of positive integers
2
b g a = -(a + b + g )[a 2 + b 2 + g 2 - ab - bg - g a ] can satisfy it.
g a b
= -( - a)[( - a)2 - 3 × 0]
Practice Exercise 2
1.  A(A + I) = - 2I. Now
= a × a2 = a3
|A(A + I)| = |-2I|
0 -2b2 -2a2 ⇒ |A| |A + I| = (- 2)n ≠ 0
94.  D = a2
2
b2 + c 2
2 2
a2
2
(R1 → R1 - R2 - R3 )
2
1
⇒ |A| ≠ 0, A - ( A + I ) = I { }
b b c +a 1
⇒ A-1 = - (A + I)
0 b 2
a2 2
    = - 2 a2 b2 + c 2 a2 1
2 2 2 2
 2r 1  r(r  1)  sinrq
b b c +a
n x y z
0 b2 a2 2.  
r 1
∆r   n  1
sin   q sin
nq
      = - 2 a2 c2 0 (R1 → R2 - R1 and R3 → R3 - R1) n  2  2
2n  1
b2 0 c2 n 1 q
sin
2
    = 4a2b2c2 = 4 [as a, b, c are the cube roots of unity]
Therefore, Im (D) = 0  n + 1 nq
sin  q sin
n  2  2
95. Applying R3 → R3 - xR1 - yR2 we get 2n - 1
n +1 sin q / 2
p q px + qy = x y z =0
D= q r qx + ry  n + 1 nq
n
sin 
  q sin
0 0 -( px 2 + 2qxy + ry 2 ) 2n - 1
2 2
n +1 sinq / 2
⇒ (q2 - pr)(px2 + 2qxy + ry2) < 0   
As q2 - pr < 0 ⇒ Discriminant of quad < 0

Mathematical Problem Book for JEE.indb 790 07-06-2018 12:54:53


Chapter 18 | Matrices and Determinants 791

3.  Operating C1 → C1 + C2 + C3 we get adj(adj A) = | A |3 - 2 A = A and | adj (adj A) | = | A | = 1


x +a+b+c b c Also, | adj A | = | A |3 - 1 = | A |2 = 12 = 1
x +a+b+c x +b c =0  7. x + 3y + 2z = 6 (1)
x +a+b+c b x+c x + ly + 2z = 7 (2)

x + 3y + 2z = m(3)
1 b c
(A) If l = 2, then D = 0. Therefore, unique solution is not
⇒ (x + a + b + c) 1 x + b c =0 possible
1 b x +c (B)  If l = 4, m = 6
Now R2 → R2 - R1 and R3 → R3 - R1, we get x + 3y = 6 - 2z
x2(x + a + b + c) = 0 x + 4y = 7 - 2z
⇒ Either x = 0 or x = -(a + b + c) Therefore, y = 1 and x = 3 - 2z.
Substituting in Eq. (3), we get 3 - 2z + 3 + 2z = 6 is satisfied.
1 2 2  1 2 2  Therefore, infinite solutions.
  
4.  A2 - 4A - 5I3 = 2 1 2  2 1 2  (C)  l = 5, m = 7
  
Consider Eqs. (2) and (3).
2 2 1 2 2 1
x + 5y = 7 - 2z
1 2 2  1 0 0 x + 3y = 7 - 2z
   
-4 2 1 2   5 0 1 0 Therefore, y = 0, x = 7 - 2z are the solutions.
    Substituting in Eq. (1) we have 7 - 2z + 2z = 6 is not
2 2 1 0 0 1
satisfied.
9 8 8  4 8 8  5 0 0  Therefore, no solution.
     
= 8 9 8   8 4 8  +  0 5 0  (D) If l = 3, m = 5, then Eqs. (1) and (2) have no solution. There-
      fore, no solution.
8 8 9  8 8 4   0 0 5
 8. By the properties of adjoint of a matrix, adjoint of a symmetric
0 0 0  matrix is again a symmetric matrix.
 
= 0 0 0  = 0 Similarly,
 
0 0 0  A (adj A) = (adj A) A = |A|I
Therefore, Also adjoints of unit and diagonal matrices are also unit and
A2 - 4A - 5I3 = 0 diagonal matrices.
⇒ A-1 A2 - 4A-1 A - 5A-1 I3 = 0 Hence, all options are correct.
⇒ (A-1 A) A - 4I3 - 5A-1 =0 3 3 3 
⇒ IA - 4I3 - 5A-1 = 0  
 9.  A2 =  3 3 3  = 3A
1  
⇒ A-1 = (A - 4I3)  3 3 3 
5
Also, A3 = A2A = 3A × A = 3A2 = 3 × (3A) = 9A and |A| = 0
Therefore, A-1 does not exist.
9 8 8 
  x 
|A | = 8 9 8  = 9 (81 - 64) - 8(72 - 64) + 8(64 - 72)
2
 
  10.  Let U1 be y  so that
8 8 9   
z 
= 9 × 17 - 8 × 8 + 8 × (- 8) = 133 - 128 = 5 ≠ 0
1 0 0  x  1 x   1 
Therefore, A2 is invertible.          
2 1 0  y  = 0  ⇒ y  = 2 
A3 = A × A2 = A × (4A - 5I3) = 4A2 - 5A          
3 2 1 z  0  z   1 
36 32 32   5 10 10  31 22 22   
      2  2 
= 32 36 32  + 10 5 10  = 22 31 22     
     
Similarly, U2 = 1 , U3 = 1 .
32 32 36  10 10 5  22 22 31    
4  3
Therefore, |A3| ≠ 0 and so |A3| is invertible.
 5. 
Here, (A), (B), (C), (D)  are the properties of adjoint. Hence, (A), 1 2 2 
 
(B), (C) and (D) are the correct answers. Hence, U = 2 1 1 and |U| = 3
 
 6. We have  1 4 3
3 -3 4
1 2 0 
|A| = 2 -3 4 = 3 (- 3 + 4) - 2(- 3 + 4) + 0 = 1  
11.  adj U = 7 5 3
0 -1 1  

 9 6 3 

Mathematical Problem Book for JEE.indb 791 07-06-2018 12:55:06


792 Mathematics Problem Book for JEE

adjU Now, a - 3b = 1, 2a + 2b = 0 and 4a + b = 0, which is not pos-


Hence, U-1 = and the sum of the elements of U-1 = 0 sible.
3
For (C)
12. The value of  
a b c  1 4 
é3ù é 1 2 2 ù é3ù   = 2 3
d e f   
[ 3 2 0 ] êê2 úú = [3
U 2 0] êê -2 -1 -1úú êê2 úú 5 4 
⇒ a + 2b + 5c = 1
ëê0 úû ëê 1 -4 -3úû ëê0 úû
4a - 3b + 4c = 0
é3ù d + 2e + 5f = 0
   = [ -1 4 4 ] êê2 úú = - 3 + 8 = 5 4d - 3e + 4f = 1
êë0 úû Therefore, there are infinite number of left inverses
Right inverse:
13. b1 × C31 + b2 × C32 + b3 × C33 1 4  1 0 0 
  a b c   
a2 a3 2 3   = 0 1 0 
a a a a   d e f   
= b1 - b2 1 3 + b3 1 2 =0 5 4  0 0 1
b2 b3 b1 b3 b1 b2
14.  The value of new determinant = 23D = 8D ⇒ a + 4d = 1, 2a - 3d = 0 and 5a + 4d = 0
which is not possible.
15. a3 M13 - b3 ⋅ M23 + d3 ⋅ M33 = a3 C13 + b3 ⋅ C23 + d3 ⋅ C33 = D
Therefore, there is no right inverse.
by definition.
16.  As second row of all the options is same, we look at the ele- a11 a12 a13 
ments of the first row.  
In a matrix a21 a22
19.  a23  , a22 must be a multiple of 3 (3, 6
a b c   
Let left inverse be   . Then a31 a32 a33 
d e f 
or 9) because from the above possible combinations only 3, 6
1 1 and 9 are repeated four times in a row or column or diagonal.
a b c     
  1 1  = 1 0  20.  Clearly the maximum value of the sum of the diagonal
d e f    0 1
2 3    elements is 18 which are called the trace or the matrix A.
Therefore, 21.  Possible combinations are (1, 2, 6), (1, 3, 5), (1, 8, 9), (2, 3, 4),
a + b + 2c = 1 (2, 7, 9), (3, 6, 9), (3, 7, 8), (4, 5, 9), (4, 6, 8), (5, 6, 7)
- a + b + 3c = 0 Hence, total 10 combinations are possible.
1- 5c 1+ c
⇒b= and a = 22. (A) →(q), (B) → (s, t), (C) → (r), (D) → (p)
2 2
Thus, matrices in options (A), (B) and (D) are the inverses and (A)  A2 = -I, therefore A is of even order
matrix in option (C) is not the left inverse. (B)  (I + A)n = C0 In + C1 I A + C2 I A2 + … + Cn I An
= Co I + C1 A + C2 A + … + Cn A = I + (2n - 1) A
17.  Let right inverse be
a b  Therefore, l = 2n - 1
  (C)  A2 = A and B = I - A
c d 
  AB + BA + I - (I + A2 - 2A) = AB + BA - A + 2A
e f  = AB + BA + A
Now, = A (I - A) + (I - A) A + A = A - A + A - A + A = A
a - c + 2e = 1
(D)  A* = A, B* = B
b - d + 2f = 0
(AB - BA)* = B* A* - A*B* = BA - AB
2a - c + e = 0
2b - d + f = 1 23. (A) → (p, t), (B) → (q), (C) → (r), (D) → (p, t)
Therefore, infinite solutions. a11 a12 a13
(A)  |A| = a21 a22 a23
18. 
By observation, there cannot be any left inverse for (B) and
(D), so we will check for (A) and (C) only. a31 a32 a33
a b 
  a11 k -1a12 k -2a13 k 2a11 k a12 a13
For (A) let left inverse be c d  . Then  |B| = k a21 -1 1 2
a22 k a23 = 3 k a21 ka22 a23 = |A|
e f  k
2
k a31 k a32 a33 k 2a31 ka32 a33
a b  1 0 0 
   1 2 4  
 c d    = 0 1 0  k1 |A| + k2 |B| = 0
  3 2 1  
e f  0 0 1 k1 + k2 = 0

Mathematical Problem Book for JEE.indb 792 07-06-2018 12:55:26


Chapter 18 | Matrices and Determinants 793

1 -1 1 Therefore,
 0 -1
(B)  1 1 -1 = 4 2 2    =  -1 -2 = -A
f (A) = (I + A) (I - A)-1 =   1
1 1 1  1 2  - 0   -1 -1
 2 
1 cos a cos b 0 cos a cos b Now, comparing the above equation with f (A) = -lA,
(C)  cos a 1 cos g = cos a 0 cos g we have l = 1
cos b cos g 1 cos b cos g 0 26.  A′A = I. Therefore
|A′A| = | I | ⇒ |A| = ±1
⇒ sin2 g - cosa (cosa - cosb cosg ) + cosb (cosa cosg - cosb )
= -cosa (-cosb cos g ) + cos b (cos a cos g ) a b c
⇒ b c a = ±1
⇒ sin2g - cos2a + 2 cosa cosb cosg - cos2b
= 2cosa cosb cosg c a b
⇒ 3abc - a3 - b3 - c3 = ±1
⇒ sin2g = cos2a + cos2b ⇒ cos2a + cos2b + cos2 g = 1
⇒ a3 + b3 + c3 = 2 and 4
x2 + x x +1 x -2 27.  Here, |A - lI| = 0
(D)  2 x 2 + 3 x - 1 3x 3x - 3 1- l 0
=0
2
x + 2x + 3 2x -1 2x -1 -1 7 - l
⇒ (1 - l) (7 - l) = 0
R2 → R2 - (R1 + R3) gives ⇒ l2 - 8l + 7 = 0
⇒ A2 - 8A + 7I2 = 0
x2 + x x +1
x -2 ⇒ A2 = 8A - 7I2
x +1 x - 2
-4 0 0 =4 ⇒ k = - 7 ⇒ |k| = 7
2 x -1 2x -1
x2 + 2x + 3 2x -1 2x -1 28.  Let

x + 1 -3 (1+ x )a1b1 (1+ x )a1b2 (1+ x )a1b3


= 4 2 x - 1 0 = (24x - 12)
(1+ x )a2 b1 (1+ x )a2 b2 (1+ x )a2 b3 = l0 + l1x + l2x2 + l3x3 + …
Therefore, A = 24, B = -12 and A + 2B = 0. (1+ x )a3b1 (1+ x )a3b2 (1+ x )a3b3
| adjB | | adj(adjA)| | A |( 3 -1)
2
| A |3 For l1, differentiate w.r.t. x and put x = 0. So l1 = 0.
24.  = = =
|C | | 5A | 3
5 | A| 125 29. Clearly f ′(x) = 0. Therefore
Now, |A| = 5. Therefore, f(x) = c = 6
| adjB | Therefore,
|C | = 1 25 25
å f (r ) = rå= 16 = 150
r =1
 1 0  1 2 2 2
25. I + A =   +  =   30.  f(-x) = -f(x) = g(x). Therefore,
0 1 1 1  1 2
f(x) × g(x) = -(f(x))2 or f(1) g(1) = -(f(1))2
 1 0 1 2  0 -2 1 1 1
2
and I - A =   - 1 1 =  -1 0 
 0 1     =- 0 2 1 = -4

Now, |I - A| = 0 - 2 = - 2. So 1 3 1
⇒ lf(1) g(1) = 4 ⇒ l (-4) = 4 ⇒ l = -1
0 2 
adj (I - A) =  
 1 0 31.  On solving, we get
(2l + 15) f(x + 1) - (l + 10) f(x + 8) - f(x + 1) = 0
 0 -1
(I - A)-1 =  1  ⇒ (2l + 14) f(x + 1) = (l + 10) f(x + 8)
- 0 Since, f is periodic with period 7, therefore
 2 
f(x + 1) = f(x + 8)
As ⇒ 2l + 14 = l + 10
f (x) = (1 + x) (1 - x)-1 ⇒|l|=4

Mathematical Problem Book for JEE.indb 793 07-06-2018 12:55:45


794 Mathematics Problem Book for JEE

Solved JEE 2017 Questions


JEE Main 2017 Using a = 1 in the given system of equations, we get
x+y+z=1
 2 -3
1. If A =   , then adj(3A + 12A) is equal to
2 x+y+z=1
 -4 1  x + by + z = 0
 51 63  51 84 
(A)
84 72  (B)   We see that there is only one value of b; therefore, S is singleton
  63 72  set.
 72 -63
é 72 -84 ù Hence, the correct answer is option (C).
(C) (D) ê
 -84 51  ú
 ë -63 51 û
 3. Let k be an integer such that the triangle with vertices (k, -3k),
(OFFLINE) (5, k) and (-k, 2) has area 28 sq. units. Then the orthocentre of
this triangle is at the point
Solution: The given matrix is
 3  3
 2 -3 (A)
 1,  (B)  1, - 
A=    4  4
 -4 1 
Therefore,  1  1
(C)
 2,  (D)  2, - 
2 16 -9   2  2
A = 
 -12 13  (OFFLINE)
That is,
 48 -27 Solution: We can write the given vertices of the triangle in the fol-
3 A2 =  
 -36 39  lowing form:
Also, k -3k 1
 24 -36  1
12 A =   5 k 1 = 28
 -48 12  2
-k 2 1
Hence, That is,
 72 -63      5k2 + 13k - 46 = 0
3 A2 + 12 A =  
 -84 51  or 5k2 + 13k + 66 = 0
Therefore,
 51 68  From the above, we confirm that no real solution exists. Therefore,
adj(3 A2 + 12 A) =  
84 72  -23
k= or k = 2
Hence, the correct answer is option (A). 5
Since it is given that k is an integer, we consider only k = 2.
2. If S is the set of distinct values of b for which the following Therefore, the vertices are obtained as (2, -6), (5, 2) and (-2, 2) as
system of linear equations depicted in the following figure.
x+y+z=1 A(2, −6)

x + ay + z = 1
ax + by + z = 0
H
E
has no solution, then S is (2, b )
(A) an infinite set.
(B) a finite set containing two or more elements.
(C) a singleton.
B(5, 2) C(−2, 2)
(D) an empty set. D
(OFFLINE) Thus, solving the equations of two altitudes, the orthocentre of the
Solution: For Δ = 0 (and at the one of the solutions of Δ1, Δ2,  1
triangle is obtained as  2,  .
Δ3 ≠ 0):  2
1 1 1 Hence, the correct answer is option (C).
D= 1 a 1 =0  0 cos x - sin x 
a b 1  
4. If S =  x ∈[0 , 2p ] : sin x 0 cos x = 0 , then
 cos x sin x 0 
1(a - b) - 1(1 - a) + 1(b - a2) = 0  
2a - b - 1 + b - a2 = 0 p 
a2 - 2a + 1 = 0 ⇒ a = 1
∑ tan  3 + x  is equal to
x ∈S

Mathematical Problem Book for JEE.indb 794 07-06-2018 12:55:59


Chapter 18 | Matrices and Determinants 795

-2 + 3
(A) (B) 4 + 2 3 Solution: The system of equations can be written in the matrix
form as
-4 - 2 3
(C) (D) -2 - 3 - l   x  0 
2 4
(ONLINE) 4
 l 2   y  = 0 
Solution: Solving the determinant  l 2 2   z  0 
0 cos x - sin x The system has infinite solutions; thus, we get
sin x 0 cos x = 0
cos x sin x 0 2 4 -l
we get 4 l 2 =0
0[0 - sin x cos x] - cos x[0 - cos2x] - sin x[sin2x - 0] = 0 l 2 2
⇒ cos3x - sin3x = 0
⇒ 0 = 2(2l - 4) - 4(8 - 2l) - l(8 - l2)
Using a3 - b3 = (a - b)(a2 + b2 + ab). We get ⇒ 4l - 8 - 32 + 8l - 8l + l3 = 0
cos3x - sin3x = (cos x - sin x)(cos2x + sin2x + sin x cos x) = 0 ⇒ l3 + 4l - 40 = 0
Using cos2x + sin2x = 1. Now, We can solve this by graphical method
(cos x - sin x)(1 + sin x cos x) = 0
y = x3 + 4x - 40
⇒ cos x - sin x = 0 ⇒ cos x = sin x
y
⇒ tan x = 1
p
⇒x= (1)
4
x
p  tan a + tan b
Now, evaluating ∑ tan  3 + x  using, tan(a + b ) =
1-tan a tan b
,
we get x ∈S −40

p  p p 
∑ tan  + x  = tan  + 
 3   3 4
[from Eq. (1)] For x = 0, y = -40: If we take y = -40, then we have
x =p / 4
-40 = x3 + 4x - 40
 p p  tan(p / 3) + tan(p / 4 ) 3 + 1 1+ 3
⇒ x3 + 4x = 0
⇒ tan  +  = = =
 3 4  1- tan(p / 3) tan(p / 4 ) 1- 3 × 1 1- 3
⇒ x(x2 + 4) = 0
Multiplying and dividing by 1+ 3 , we get ⇒ x = 0, x2 + 4 = 0
⇒ x = ± 2i
p  1+ 3 1+ 3
∑ tan  3 + x  = 1- 3 × 1+ 3 The given equation of line intersects x only at one point; therefore,
the real value of l is only one.
Using a2 - b2 = (a + b)(a - b) Hence, the correct answer is option (B).

p  (1+ 3 )2 1+ 3 + 2 3 4 + 2 3 6. Let A be any 3 × 3 invertible matrix. Then, which one of the


∑ tan  3 + x  = (1- 3)
=
-2
=
-2 following is not always true?
4 2 3 adj(adj(A)) = A ⋅ (adj(A))-1
(A)
= + = -2 - 3
( -2) ( -2) 2
adj( adj(A)) = A ⋅ (adj(A))-1
(B)
Hence, the correct answer is option (D). adj(A) = A ⋅ A-1
(C)

5. The number of real values of l, for which the system of linear adj(adj(A)) = A ⋅ A
(D)
equations (ONLINE)

2x + 4y - l z = 0 Solution: From the properties of invertible matrices, option (1) is


not true.
4x + l y + 2z = 0
Hence, the correct answer is option (A).
l x + 2y + 2z = 0
has infinitely many solutions, is 7. For two 3 × 3 matrices A and B, let A + B = 2B′ and 3A + 2B =
(A)
0 (B) 1 I3, where B′ is the transpose of B and I3 is 3 × 3 identity matrix.
Then
(C)
2 (D) 3
(ONLINE) 10A + 5B = 3I3
(A) (B) 5A + 10B = 2I3

Mathematical Problem Book for JEE.indb 795 07-06-2018 12:56:14


796 Mathematics Problem Book for JEE

3A + 6B = 2I3
(C) (D) B + 2A = I3 é1 0 0 ù
(ONLINE)
• Option (C): The determinant êê0 -1 0 úú is possible: 1(1) -
Solution: It is given that 0(0) + 0(0) = +1. êë0 0 -1úû

A + B = 2B′ (1) é -1 0 0 ù
Taking transpose on both sides, we get • Option (D): The determinant êê 0 -1 0 úú is not possible:
-1(1) - 0(0) + 0(0) = -1. êë 0 0 -1úû
    A′ + B′ = 2B (2)
3A + 2B = I3 (3) Hence, the correct answers are options (B) and (D).

That is, 2. For a real number a, if the system


3 A′ + 2B′ = I3 (4)
 1 a a 2 x  1 
Substituting Eq. (2) in Eq. (4), we get     
 a 1 a   y  =  -1
3(2B -B′) + 2B′ = I3  2 
a a 1   z   1 
That is,
6B -B′ = I3
of linear equations, has infinitely many solutions, then 1 + a
A+ B + a 2 = ______.
Writing as B′ = , we get
2 Solution: It is given that
A+B
6B - = I3
2  1 a a 2 x  1 
Therefore,     
 a 1 a   y  =  -1
 2 
12B - A - B = 2I3
a a 1   z   1 
11B - A = 2I3
Therefore,
11B - A = 6A + 4B [from Eq. (3)]
 1 a a 2
Therefore,  
7B = 7 A Þ A = B a 1 a =0
 2 
a a 1
I3 = 3A + 2A = 5A = 5B
( - a 2 ) - a (a - a 3 ) + a 2 (a 2 - a 2 ) = 0
⇒ 11
10A + 5B = 10A + 5A = 15A = 3I3
   ⇒ (1- a 2 ) - a 2 (1- a 2 ) = 0 ⇒ (1- a 2 )(1- a 2 ) = 0
Hence, the correct answer is option (A).
⇒ (1- a 2 )2 = 0
⇒ a 2 = 1⇒ a = ±1
JEE Advanced 2017
For a = 1, the given system of linear equations has no solution.
1. Which of the following is(are) NOT the square of a 3 × 3 matrix
That is,
with real entries?
é +1 +1 +1ù é x ù é 1 ù
 1 0 0 é1 0 0 ù ê +1 +1 +1ú ê y ú = ê -1ú
0 1 0 
(A) (B) êê0 1 0 úú ê úê ú ê ú
  êë +1 +1 +1úû êë z úû êë 1 úû
0 0 1 êë0 0 -1úû
x+y+z=1
1 0 0  é -1 0 0 ù x + y + z = -1
0 -1 0 
(C) (D) êê 0 -1 0 úú x+y+z=1
 
0 0 -1 êë 0 0 -1úû Since two planes are parallel, a = 1 is rejected and for a = -1, the
given system of linear equations has coincident planes.
Solution: For a matrix to be a square of matrix with real entries, its
determinant should be positive.  1 -1 1   x   1   x - y + z = 1
 -1 1 -1  y  =  -1 ⇒  - x + y - z = -1⇒ x - y + z = 1
 1 0 0      
• Option (A): The determinant 0 1 0  is possible: 1(1) -  1 -1 1   z   1   x - y + 1 = 1
0(0) + 0(0) = +1. 0 0 1
Therefore, a = -1 holds well. Therefore,
é1 0 0 ù 1 + a + a 2 = 1 + (-1) + (-1)2
• Option (B): The determinant êê0 1 0 úú is not possible: =1-1+1=1
1(-1) - 0(0) + 0(0) = -1. êë0 0 -1úû Hence, the correct answer is (1).

Mathematical Problem Book for JEE.indb 796 07-06-2018 12:56:31


Chapter 18 | Matrices and Determinants 797

3. How many 3 × 3 matrices M with entries from {0, 1, 2} are a12 + b12 + c12 a1a2 + b1b2 + c2 a1a3 + b1b3 + c1c3 
there, for which the sum of the diagonal entries of MTM is 5?  
(A) 126 (B) 198 = a2a1 + b2b1 + c2c1 a22 + b22 + c22
 a2a3 + b2b3 + c2c3 
(C) 162 (D) 135  
a3a1 + b3b1 + c3c1 a3a2 + b3b2 + c3c2 a32 + b32 + c32 
Solution: Let us consider a 3 × 3 matrix
Þ (a12 + b12 + c12 ) + (a22 + b22 + c22 ) + (a32 + b32 + c32 ) = 5
a1 a2 a3 
  There are two possible cases:
M = b1 b2 b3 
c1 c2 c3  (i) 02 + 02 + 02 + 02 + 12 + 12 + 12 + 12 + 12 = 5
This has 9C5 combinations possible.
Therefore,
(ii) 12 + 22 + 02 + 02 + 02 +02 +02 +02 +02 = 5
a1 b1 c1 
  This has 9C7 × 2C1 possible combinations.
MT = a2 b2 c3 
Therefore,
a3 b3 c3 
9 9! 9! 2!
C5 + 9C7 × 2C1 = +
T
It is given that sum of diagonal of M M is 5. Therefore, 5 !× 4 ! 7 ! 2 ! 1!1!
9 ´ 8 ´ 7 ´ 6 ´ 5! 9 ´ 8 ´ 7!
a1 b1 c1  a1 a2 a3  Þ + ´ 2 Þ 126 + 72 = 198
   5 ! ´ 4 ´ 3 ´ 2 ´ 1 7 !´ 2 ´ 1
MT M = a2 b2 c2  b1 b2 b3 
Thus, the total number of 3 × 3 matrices is 198.
a3 b3 c3  c1 c2 c3 
Hence, the correct answer is option (B).

Mathematical Problem Book for JEE.indb 797 07-06-2018 12:56:38


Mathematical Problem Book for JEE.indb 798 07-06-2018 12:56:38
19 Limit, Continuity and
Differentiability
19.1  Limit of a Function 19.2.2  Formal Definition of Limit
The concept of limit is used to discuss the behaviour of a function Let f(x) be defined on an open interval about x0, except possible
close to a certain point. For example, at x0 itself, we say that f(x) approaches the limit L as x approaches
x2 -1 x0 and write lim f ( x ) = L , if for every number ∈ > 0, there exists a
f (x) = x ® x0
x -1 corresponding number δ > 0 such that for all x
Clearly the function is not defined at x = 1, but for values close to
0 < | x - x0 | < δ ⇒ | f(x) -L | < ∈
x = 1 the function can be written as
f(x) = x + 1
Illustration 19.1   Show that lim (5 x - 3) = 2.
As x approaches 1 (written as x → 1), f(x) approaches the value 2 x ®1
[f(x) → 2]. We write this as Solution: See Fig. 19.1. Set x0 = 1, f(x) = 5x - 3, and L = 2 in the
lim f ( x ) = 2 definition of limit. For any given ∈ > 0, we have to find a suitable
x ®1
δ > 0 so that if x ≠ 1 and x is within distance δ of x0 = 1, that is, if
It must be noted that it is not necessary for the function to be
0 < | x - 1| < δ,
undefined at the point where limit is calculated. In the above
example, lim f ( x ) is the same as the value of function at x  = 2, then f(x) is within the distance ∈ of L = 2, that is
x ®2 |f(x) - 2| < ∈
that is, 3.
Sometimes, functions approach different values as x-approaches We find δ by working backwards from the ∈ inequality
x0 from left and right. By left we mean x < x0 and right means x > |(5x - 3) - 2| = |5x - 5| < ∈
x0. This is written as x→ x 0- and x→ x 0+ , respectively. For example, 5|x - 1| < ∈
f (x) = [x] (greatest integer function) |x - 1| < (∈/5)
For any integer n, Thus, we can take δ = ∈/5.
lim f ( x ) = n -1 (19.1)
x ® n-

and lim+ f ( x ) = n (19.2)


x ®n
In such cases we say that lim f ( x ) does not exist. The limit in
x ®n
Eq. (19.1) is said to be the left hand limit (L.H.L.) at x = n and that in
Eq. (19.2) is called the right hand limit (R.H.L.) at x = n.

19.2 Definition
19.2.1  Informal Definition of Limit
Let f(x) be defined on an open interval about x0, except possibly at
x0 itself. If f(x) gets arbitrarily close to L for all x sufficiently close to
x0, we say that function approaches the limit L as x approaches x0,
and we write
lim f ( x ) = L
x ® x0

This definition is “informal” because phrases like arbitrarily close Figure 19.1
and sufficiently close are imprecise and their meaning depends on
the context. If 0 < |x - 1| < δ = ∈/5, then
The definition is clear enough and enables us to recognize and |(5x - 3) - 2| = |5x - 5| = 5|x - 1| < 5(∈/5) = ∈
evaluate limits of specific functions. This proves that limx→1(5x - 3) = 2.

Mathematical Problem Book for JEE.indb 799 07-06-2018 12:56:49


800 Mathematics Problem Book for JEE

The value of δ = ∈/5 is not the only value that will make 0 < |x - 1| Left and Right Limits: Let y = f(x) be a given function, and x = a
< δ imply |5x - 5| < ∈. Any smaller positive δ will do as well. The defi- is the point under consideration.
nition does not ask for a “best” positive δ, just one, that will work. Left tendency of f(x) at x = a is called its left limit and right ten-
For limit L to exist as x approaches x0, a function f must be dency is called its right limit.
defined on both sides of x0, and its values f(x) must approach L as x f (a + 0 ) = lim f (a + h) and f (a - 0 ) = lim f (a - h) where ‘h’ is a
approaches x0 from either side. Because of this, ordinary limits are h®0 h®0
sometimes called two-side limits. small positive number.
Thus, for the existence of the limit of f(x) at x = a, f(a- 0) = f(a + 0)
19.2.3  Right Hand Limit
19.3  Algebra of Limits
If lim + f ( x ) = L for every number ∈ > 0, there exists a correspond-
x ® x0
lim f(x)
Let lim[ x )l±
c1f ( = 1 and
c2 g( lim)] =g(x)
xlim[ = l f. (Then
1f ( xc)12±
clim[ g(±x )]
cx2)] = lim[
lim[ c2 g(cx1f)]( x=)]c±1l1lim[
± c2cl22g( x )] = c1l1 ± c2l2
ing number δ > 0 such that for all x.    x ®a x ®a x ®a x ®a x ®a x ®a
x0 < x < x0 + δ ⇒ |f(x) - L| < ∈ 1. lim[c1f ( x ) ± c2 g( x )] = lim[c1f ( x )] ± lim[c2 g( x )] = c1l1 ± c2l2 ,
x ®a x ®a x ®a
Then we call it right hand limit. For example: where c1 and c2 are given constants.
x
lim = +1 2.  lim f ( x ) × g( x ) = lim f ( x ) × lim g( x ) = l1 × l2
x ®0+ |x| x ®a x ®a x ®a

19.2.4  Left Hand Limit f ( x ) xlim f (x) l


3.  lim = ®a = 1, l2 ¹ 0
x ® a g( x ) lim g( x ) l2
If lim - f ( x ) = L for every number ∈ > 0, there exists a correspond­ x ®a
x ® x0
4. lim f [ g( x )] = f [ lim g( x )] = f (l2 ), if and only if f(x) is continuous
ing number δ > 0 such that for all x. x ®a x ®a
at x = l2.
x0 - δ < x < x0 ⇒ |f(x) - L| < ∈
In particular, lim ln [g(x)] = ln l2 if l2 > 0.
Then we call it left hand limit. For example: x ®a
x All these theorems must be used with utmost care. For example,
lim = -1 we have assumed that l1 and l2 are finite. If these are not finite,
x ®0- | x |
the given theorems will not be applicable.
x
Through graph of , we can easily visualize the things written sin x
| x| For example: lim = 1, and if we try to apply the theorems,
above. x ®0 x
sin x 1
we get lim = lim sin x × lim , which does not exist.
x ®0 x x ®0 x ®0 x
right hand limit Which of course is an absurd result, we are getting this absurd
result because in this case the given limit cannot be written
y=1 1
as the product of two limits as lim does not exist. Similarly,
x ®0 x
é sin x ù , where [.] denotes the greatest integer function ≠
lim
x ®0 êë x û
ú
y = −1 left hand
é sin x ù
ê xlim . Here [x] is not continuous at x = 1.
limit
ë ® 0 x úû
Figure 19.2 1
lim
log[1+ f ( x )]
g( x ) g( x )
5.  lim [1+ f ( x )] =e
x ®a

Now, from the discussions we have just gone through we can x ®a


easily say that limit of a function will exist iff LHL and RHL both
are finite, unique and equal (Fig. 19.2). 19.4  Evaluation of Limits
For example: lim [ x ] will not exist as LHL = lim- [ x ] = 0;
x ®1 x ®1 Following are indeterminate forms:
RHL =  lim+ [ x ] = 1 0
x ®1 1.      2.  ∞     3.   0 × ∞    4.  ∞ - ∞
This can be seen graphically, in Fig. 19.3. 0 ∞
y 5.   0°    6.  ∞°    7.   1∞
We shall divide the ways of evaluation of limits in five categories:
RHL
1 19.4.1 Simplification
x In this method, we can use:
LHL 1 1.  Direct substitution
2. Rationalisation
3.  Factorization
4. Use of formulas like binomial expansion, trigonometric
Figure 19.3 formulas, etc.

Mathematical Problem Book for JEE.indb 800 07-06-2018 12:57:08


Chapter 19 | Limit, Continuity and Differentiability 801

1. Direct substitution: We can directly substitute the number at 3. Factorization method: We factorize numerator and denom-
which limit is to be find. For example inator of the rational function, so that common factors in
numerator and denominator cancel out. By doing this, we in
• lim ( x 2 + 3 x - 2) can be find out by this method.
x ®1 turn are eliminating the factors which are making the function
in one of the indeterminate form. For example:
  lim ( x 2 + 3 x - 2) = 2
x ®1
x -4 ( x 1/ 4 - 2) ( x 1/ 4 + 2)
lim = lim
•   xlim
®-2
| x |  = 2 x ®16 x 1/ 4 - 2 x ®16 ( x 1/ 4 - 2)
But before using this method, we have to see that LHL should
1/ 4
= lim ( x + 2) = 4
x ®16
remain equal to RHL.
For example: In lim sec -1 x , if we directly substitute, we will 4. Use of formulas: We can use formulas which we have stud-
x ®1 ied in other different topics to make functions simplified.
get lim sec -1 x as 0. But LHL of lim sec -1 x will not exist. So, For example:
x ®1 x ®1
answer should be, limit does not exist. x x
2 sin cos
2. Rationalisation method:  Rationalisation is followed when sin x 2 2
lim = lim
we have powers in fractions on expressions in numerator and x ®0 x x ®0 x
sin sin
denominator or in both. After rationalization, the terms are 2 2
factorised, which on cancellation give the result. x
= lim 2 cos = 2
x ®0 2
Illustration 19.2   Find lim ( x - x 2 + x ).
x ®¥ 19.5  Use of Standard Limits
Solution: This is apparently of the form ∞ minus ∞ and can be
∞ These standard forms are used in case f(x) → 0 when x → a.
converted to form by multiplying numerator and the denomi-
∞ sin f ( x )
nator by the conjugate. Therefore, 1.   lim =1
x ®a f (x)

( x - x 2 + x )( x + x 2 + x ) 2.   lim cos f ( x ) = 1
lim ( x - x 2 + x ) = lim x ®a
x ®¥ x ®¥ 2
x+ x +x tan f ( x )
3.   lim =1
2
x -( x + x ) 2 æ -x ö x ®a f (x)
= lim = lim ç ÷
x ®¥
x + x 2 + x x ®¥ çè x + x 2 + x ÷ø sin-1[f ( x )]
4.   lim =1
æ ö -1 -1 x ®a f (x)
-1
= lim ç ÷= =
x ®¥ ç 1 ÷ 1+ 1 2 tan-1[f ( x )]
ç 1+ 1+ ÷ 5.   lim =1
è x ø x ®a f (x)
a
æ x 2 + 8 - 10 - x 2 ö sin
Illustration 19.3   Find lim ç ÷.
Illustration 19.4   Evaluate lim
n
.
x ®1ç 2 2 ÷ b
è x +3 - 5- x ø n ®¥
tan
3-3 0 n +1
Solution: This is of the form = if we put x = 1.
2-2 0 1 a
Solution: As n ®¥ , ® 0 and also tends to zero.
0 n n
To eliminate the factor, multiply and divide by the conjugate of
0 a
sin
numerator and the conjugate of the denominator. Therefore, a
sin should be written as n so that it looks like lim sinq .
n a q ®0 q
( x 2 + 8 + 10 - x 2 ) n
Limit = lim ( x 2 + 8 - 10 - x 2 )
x ®1
( x 2 + 8 + 10 - x 2 ) a öæ b ö
æ
ç sin n ÷ ç n + 1 ÷ a(n + 1)
( x2 + 3 + 5- x2 ) The given limit = lim ç ÷ç ÷×
´ n ®¥
çç a ÷÷ ç tan b ÷ n × b
( x 2 + 3 + 5 - x 2 )( x 2 + 3 - 5 - x 2 ) è n ø çè n + 1 ÷ø

x2 + 3 + 5- x2 æ sin a ö æ b ö
( x 2 + 8 ) - (10 - x 2 )
= lim ´ ç n ÷ ç n +1 ÷ a æ 1 ö
x ®1
x 2 + 8 + 10 - x 2 ( x 2 + 3) - ( 5 - x 2 ) = lim ç ÷ç ÷ × ç 1+ ÷
n ®¥
çç a ÷÷ ç tan b ÷ b è n ø
æ x2 + 3 + 5- x2 ö è n ø çè n + 1 ÷ø
= lim ç ÷ ´ 1= 2 + 2 = 2 a a
x ®1ç 2 2 ÷ 3+3 3 = 1 ´ 1 ´ x1 =
è x + 8 + 10 - x ø b b

Mathematical Problem Book for JEE.indb 801 07-06-2018 12:57:36


802 Mathematics Problem Book for JEE

sin x (1- cos x ) 1 ( x + 4)6


Illustration 19.5   Show that lim = . é x ù x
x ®0 x3 2 ê æ 6 ö6 ú
Solution: lim ç 1+ ÷
x ®¥ êè x ø ú
êë úû e6
x2 = =
sin x × 2 sin x +4
e1
sin x (1- cos x ) 2 éæ 1 ö x ù x
lim = lim
x ®0 x3 x ®0 x3 lim êç 1+ ÷ ú
x ®¥ êè
ë x ø úû
x
sin3 3
x +4 4
x 2 æ 1ö 1 6 ( x + 4) 24
= lim 4 cos × 3 = lim 4(1) ç ÷ = (since = 6 + , which tends to 6 and =1+ which
x ®0 2 x x ®0 è2ø 2 x x x x
tends to 1.)
3 x - sin-1 x
Illustration 19.6   If f ( x ) = , then lim f ( x ) is equal = e5
4 x - tan-1 x x ®0
to _____. e tan x - e x
Illustration 19.9   Evaluate lim .
Solution: x ® 0 tan x - x

sin-1 x Solution:
-1 3- e tan x - e x e x ´ e(tan x - x ) - e x
3 x - sin x x 3 -1 2
lim = lim = = lim = lim
x ® 0 4 x - tan-1 x x ®0 tan-1 x 4 - 1 3 x ® 0 tan x - x x ®0 tan x - x
4-
x
e x (e tan x - x - 1)
= lim
x ®0 tan x - x
19.6  Some More Standard Forms
= e0 × 1 [as x → 0, tan x - x → 0]
These standard forms are used in case f(x) → 0 when x → a.
=1×1=1
1

6. lim [1+ f ( x )] f ( x ) = e
x ®a Reasons for the non-existence of the limit: lim f(x) will not
x ®a
b f(x)
-1 exist due to any of these three reasons:
7. lim = loge b (b > 0) 1. f(x) is not defined in the neighbourhood of x = a.
x ®a f (x)
2. f(x) does not have a unique tendency.
log[1+ f ( x )] 3. Left and right tendencies of f(x) are not the same.
8. lim =1
x ®a f (x)
Some standard limits of indeterminate forms
x n - an sin ax
9. lim = nan -1  1.  lim = a ∀a ∈R, where x in radian
x ®a x -a x ®0 x
x ×2 x - x tan ax
Illustration 19.7   Evaluate lim .  2.  lim = a ∀a ∈R, where x in radian
x ® 0 1- cos x x ®0 x
0
Solution: This is form, so the given limit becomes sin-1 ax
0  3.  lim =a∀a∈R
x ®0 x
2
æ ö
æ 2 x -1 ö 1 ç x ÷ tan-1 ax
x 2 (2 x -1)  4.  lim = a ∀a ∈R
lim = lim çç ÷÷ ç ÷ x ®0 x
x ®0 æ
ç 2 sin
2 x ö
÷x
x ®0
è x ø 2 çç sin x ÷÷ x
è 2ø è 2 ø 1/ x a æ aö
2  5.  lim (1+ ax ) = e = lim ç 1+ ÷ ∀a ∈R
1æ 1 ö x ®0 x ®¥ è xø
= loge 2 ´ ç ÷ = 2 loge 2 = loge 4
2 è 1/ 2 ø loga (1+ x )
 6.  lim = loga e (a > 0, a ≠ 1)
x ®0 x
x +4
æ x +6 ö ax -1
Illustration 19.8   Evaluate lim ç ÷ .  7.  lim = loge a , a > 0
x ®¥ è x + 1
ø x ®0 x
Solution: The problem depends upon reducing the given expres- log x
x
 8.  lim = 0 (m > 0)
æ 1ö x ®¥ xm
sion to the form lim ç 1+ ÷ which is equal to e.
x ®¥ è x ø x m - am m m -n
x +4  9.  lim = a , where m, n are rational numers
æ 6 ö x ®a x n - an n
ç 1+ x ÷
The given limit = lim ç ÷ (1+ x )m - 1
10.  lim =m
x ®¥
çç 1+ 1 ÷÷ x ®0 m
è x ø

Mathematical Problem Book for JEE.indb 802 07-06-2018 12:58:07


Chapter 19 | Limit, Continuity and Differentiability 803

19.7  Use of Expansion Some more expansions

x3 x5 x7
x x x2 x3  7.  sin x = x - + - +
 1.  e = 1+ + + + 3! 5! 7!
1! 2 ! 3 !
x2 x4 x6
x x log a x 2 (log a)2  8.  cos x = 1- + - +
 2.  a = 1+ + +  (a > 0) 2! 4! 6!
1! 2!
x3 2 5
x2 x3 x4  9.  tan x = x + + x +
 3.  log (1+ x ) = x - + - +  (-1 < x < 1) 3 15
2 3 4
æ x 11x 2 ö -1 12 3 12 32 5
 4.  (1+ x )1/ x = e çç 1- + +  ÷÷ 10.  sin x = x + x + x +
2 24 3! 5!
è ø
x3 x5
-x x x2 x3 11.  tan-1 x = x - + +
 5.  e = 1- + - + 3 5
1! 2 ! 3 !
p 12 1232 5
x2 x3 x4 12.  cos -1 x = - x - x3 - x -
 6.  log (1- x ) = - x - - - + 2 3! 5!
2 3 4
log(5 + x ) - log(5 - x ) Illustration 19.12   Find the value of a, b and c such that
Illustration 19.10   Evaluate lim .
x ®0 x
axe x - b log(1+ x ) + cxe - x
Solution: lim =2
x ®0 x 2 sin x
é æ x öù é æ x öù
log ê5 ç 1+ ÷ ú - log ê5 ç 1- ÷ ú
ë è 5 øû ë è 5 øû Solution: Using the expansion, we have
lim
x ®0 x æ x2 ö æ x2 x3 ö æ x x2 x3 ö
lim ax çç 1+ x + +  ÷÷ - b çç x - + -  ÷÷ + cx çç 1- + - +  ÷÷
æ xö æ xö x ®0
è 2! ø è 2 3 ø è 1! 2 ! 3 ! ø =2
log 5 + log ç 1+ ÷ - log 5 - log ç 1- ÷
è 5ø è 5ø 2æ x3 x5 ö
= lim x çç x - + -  ÷÷
x ®0 x è 3! 5! ø
æ xö æ xö b
log ç 1+ ÷ log ç 1- ÷ æ ö æa b cö
è 5ø è 5ø = 1+ 1= 2 x (a - b + c ) + x 2 ç a + - c ÷ + x 3 ç - + ÷ + 
= lim - è 2 ø è2 3 2ø
x ®0 æxö æ xö 5 5 5 Þ lim =2
5ç ÷ -5 ç - ÷ x ®0
2 æ x 3
x 5 ö
è5ø è 5ø x çç x - + -  ÷÷
è 3! 5! ø
ae x - b Now, above limit would exist if least power in numerator is greater
Illustration 19.11   Evaluate lim = 2. Find a and b.
x ®0 x than or equal to least power in denominator.
Solution: That is, coefficient of x and x2 must be zero and coefficient of x3
should be 2. That is,
æ x x2 ö
a çç 1+ + +  + ¥ ÷÷ - b b a b c
1! 2 ! a - b + c = 0, a + -c =0 , - + =2
lim è ø =2 2 2 3 2
x ®0 x
On solving, we get a = 3, b = 12, c = 9.
ax 2
(a - b ) + xa + + + ¥ tan-1 x - sin-1 x
2! Illustration 19.13   Evaluate lim .
lim =2 x ®0 sin3 x
x ®0 x
Solution:
Since, limit is finite, (a-b) = 0 ⇒b = a æ x3 x5 ö æ 12 x 3 12 32 5 ö
Therefore, çç x - + +  ÷÷ - çç x + + x +  ÷÷
3 5 3 ! 5 !
lim è ø è ø
ax 2 x ®0
æ 3 5 ö
3
xa + + + ¥ x x
2! çç x - + -  ÷÷
lim =2 è 3! 5! ø
x ®0 x
æ æ 1 12 ö 3 æ 1 12 32 ö 5 ö
ax ç - çç + ÷÷ x + çç - ÷÷ x +  ÷÷
lim a + + + ¥ = 2 ç 3 3!
x ®0 2! è è ø è 5 5! ø ø
= lim 3
x ®0
æ x2 x4 ö
⇒ a = 2 x 3 çç 1- + -  ÷÷
Hence, b = 2. è 3! 5! ø

Mathematical Problem Book for JEE.indb 803 07-06-2018 12:58:32


804 Mathematics Problem Book for JEE

Solution:
æ 1 12 ö æ 1 12 32 ö 2
- çç + ÷÷ + çç - ÷ x + px - qx æ0 ö
è 3 3 ! ø è 5 5 ! ÷ø -1 lim ç form ÷
= lim 3
= x ®0 r x - sx è0 ø
x ®0
æ x 2
x 4 ö 2
çç 1- +  ÷÷ Applying L’Hospital’s rule, we have
è 3! 5! ø
p x log p - q x log q log p - log q log( p / q )
lim = =
x ®0 r x log r - s x log s log r - log s log (r / s )
19.8  L’Hospital’s Rule
L′Hospital’s rule is applicable only in
0
and

indeterminate
19.9 Sandwich Theorem (Squeeze
0 ∞ Play Theorem)
0 ∞
forms. For other forms, first we have to convert them into or   Sandwich theorem helps in calculating the limits, when limits can-
0 ∞
form, then we can use it. It states that not be calculated using the above discussed methods.
Sandwich theorem: See Fig. 19.4. If f(x), g(x) and h(x) are any three
f (x) f ¢( x ) f ¢¢( x ) …
lim = lim = lim = functions such that,
x ® a g( x ) x ® a g¢( x ) x ® a g¢¢( x )
f(x) ≤ g(x) ≤ h(x) ∀ x ∈ neighbourhood of x = a
0 ∞ and lim f ( x ) = lim[h( x )] = l (say)
till we are getting or   form. x ®a x ®a
0 ∞ Then lim g( x ) = l
x ®a
0
Proof: For form: y-axis
0

f (x) f ( x ) - f (a) h(x)


lim = lim  [as f(a) = 0 and g(a) = 0]
x ®a g( x ) x ®a g( x ) - g(a)
g(x)
f ( x ) - f (a)
= lim x - a = lim f ¢( x ) and so on
f(x)
x ® a g( x ) - g( a ) x ® a g¢( x ) x
x -a x=a

For form:

1 1 Figure 19.4
First put f ( x ) = and g( x ) = .
F( x ) G( x ) Sandwich theorem: Let f, g and h be three functions such that in
0 the neighbourhood of a, f(x) ≤g(x) ≤h(x) and lim f ( x ) = l = lim h( x ).
Then proceed in same way as form’s proof. x ®a x ®a
0 Then lim g( x ) = l .
x ®a
L’Hospital’s Rule for calculating limits: Let f and g be
log x
differentiable functions in the neighbourhood of a which satisfy Illustration 19.16   Evaluate lim .
x ®¥ [x]
1. g′(x) ≠ 0 for any x in the neighbourhood
Solution: We can have
f (x) f ¢( x )
2.   lim f ( x ) = 0 = lim g( x ). Then lim = lim log x log x
x ®a x ®a x ® a g( x ) x ® a g¢( x ) f (x) = and h( x ) = as x - 1 < [x] ≤ x.
x x -1
0
This is L’Hospital’s Rule for form. However, if lim f ( x ) ® ¥ and log x 1/ x
0 x ®a lim = lim  = 0 (using L′Hospital’s rule)
x ®¥ x x ®¥ 1
lim g( x ) ® ¥ , then again we can apply the L’Hospital’s Rule.
x ®a log x 1/ x
lim = lim =0 (using L′Hospital’s rule)
x ®¥ x - 1 x ®¥ 1

Illustration 19.14  Evaluate A =  lim+ (sin x ) x . Therefore,


x ®0
log x
Solution: lim =0
x ®¥ [x]
log(sin x ) cos x /sin x
lim x log(sin x ) lim lim
A = e x ®0 + = e x ®0 + 1/ x = e x ®0 + -1/ x 2
Your Turn 1
cos x
lim - x 2 lim - 2 x cos 2 x
= e x →0 + sin x = e x →0 + = e0 = 1 1. Does lim { x } exists. ({ } represents fractional part function).
x ®0
x x
Ans. No
p -q | x - 1|
Illustration 19.15  Evaluate lim . 2. Find the value of lim .  Ans.  1
x ®0 r x - sx x ®0 x + 1

Mathematical Problem Book for JEE.indb 804 07-06-2018 12:59:02


Chapter 19 | Limit, Continuity and Differentiability 805

x -1 If f(x) is not continuous at x = a, we say that f(x) is discontinuous


 3. Evaluate lim . Ans.  1/3 at x = a.
x ®1 x3 -1
sec -1(2 - x )
 4. Find the value of lim+ . Ans.  Limit does not exist 19.10.2  Geometrical Meaning of Continuity
x ®1 x2
æ1 ö Function f(x) will be continuous at x = c if there is no break in the
 5. Find the value of lim ç sin x ÷ . Ans.  0 graph of function f(x) at the point [c, f(c)].
x ®¥ è x ø
In an interval, function is said to be continuous if there is no
3 2
 6. Find the value of lim ( x - 3 x + 2 x - 1) . Ans.  59 break in graph of function in the entire interval.
x ®5
a For example:
|x|
 7. Find the value of lim . (α > 0) Ans.  0 • f(x) = sin x is continuous in its entire domain (Fig. 19.5).
ex x ®0

 8. Write any five indeterminate forms. y


0 ∞
Ans.  , , ∞-∞, 1∞ , 0 × ∞
0 ∞
lim f ( x ) x
[f ( x )]
  9. lim is always equal to x ®a . (True/False)
x ® a [ g( x )] lim g( x )
x ®a
Ans.  False
x
10.  Find the value of lim . Ans.  Limit does not exist Figure 19.5
x ®1 [ x ]
p
• f(x) = tan x is discontinuous at x = (2n + 1) where n ∈I.
Some theorems on limits 2
(See Fig. 19.6.)
1. Let f(x) and g(x) are two functions such that lim f ( x ) and y
x ®a

lim g( x ) exist finitely. Then lim (f ± g )( x ), lim f ( x ) g( x ) exist


x ®a x ®a x ®a

f (x) 3π
and if lim g( x ) ¹ 0 , lim exists. However, the converse 2
x
x ®a x ® a g( x ) −π −π π π
0
2 2
of any of the above is not necessarily true.
2.   lim f [ g( x )] = f [ lim g( x )] provided lim g( x ) exists and lies in
x ®a x ®a x ®a

the domain of f(x). Figure 19.6


Methods for Calculating the Limits of the Form lim [f(x)]g(x) f(x) will be discontinuous at x = a, in any of the following cases:
x ®a
(i)  lim- f ( x ) and lim+ f ( x ) exist but are not equal.
when lim g(x) = ∞ x ®a x ®a
x ®a
For example, y = [x] at x ∈ I. (See Fig. 19.7.)
Here, we discuss two different cases:
lim g( x ) ( f ( x ) -1)
1. When lim f(x) = 1. In this case, lim [f(x)]g(x) = e x ®a . y
x ®a x ®a
2
2. When lim f(x) ≠ 1 but f(x) is positive in the neighbourhood
x ®a
1
of x = a. In this case we write,
lim g( x )×ln[ f ( x )] x
[f(x)]g(x) = eg(x) ln f(x) ⇒ lim [f(x)]g(x) =  e x ®a −2 −1 1 2 3
x ®a

−1

19.10 Continuity −2

Figure 19.7
19.10.1  Continuity of a Function
A function f(x) is said to be continuous at x = a if (ii)  lim- f ( x ) and lim+ f ( x ) exist but not equal to f(a).
x ®a x ®a
lim- f ( x ) = f (a) = lim+ f ( x ) ì x2 - 4
x ®a x ®a
ï x ¹2
For example, f(x) =  í x - 2 at x = 2. (See Fig. 19.8.)
That is, L.H.L  =  R.H.L.  =  Value of the function at ‘a’, that is, ï5
î x =2
lim f ( x ) = f (a).
x ®a

Mathematical Problem Book for JEE.indb 805 07-06-2018 12:59:32


806 Mathematics Problem Book for JEE

( 2 cos x - 1) ( 2 cos x + 1) (cos x + sin x )


5 = lim × × sin x
x ®p / 4 ( 2 cos x + 1)(cos x - sin x ) (cos x + sin x )
æ 2 cos2 x - 1 ö (cos x + sin x ) sin x
= lim çç ÷
x ®p / 4 cos2 x - sin2 x ÷
è ø ( 2 cos x + 1)
x=2 sin x (cos x + sin x )
= lim
x →p / 4 2 cos x + 1
1 æ 1 1 ö
ç + ÷
2è 2 2ø 1
= =
1 2
Figure 19.8 2× +1
2
(iii)  f(a) is not defined.

For example, y =
1
at x = 0. (See Fig. 19.9.) 19.10.3  Continuity in an Open Interval
x
A function f(x) is said to be continuous in an open interval (a, b) if it
is continuous at each point of (a, b).

19.10.4  Continuity in a Closed Interval


See Fig. 19.11. A function f(x) is said to be continuous in a closed
interval [a, b] if it is
•  continuous at each point in (a, b),
y
two sides
continuity continuity continuity
from right from left
Figure 19.9 y = f(x)
(iv)  At least one of the limits does not exist.
x
a c
æ 1ö b
For example, y =  sin ç ÷ at x = 0. (See Fig. 19.10.)
èxø
y Figure 19.11 Continuity at points a, b and c

•  f(x) is continuous from right at x = a, that is,


lim f ( x ) = f (a)
x ®a+

2
x •  f(x) is continuous from left at x = b, that is,
−π − π1 1 1 1 2
3π 2π π π lim f ( x ) = f (b )
x ®b -

Continuity of a Function: A function f(x) is said to be


Figure 19.10 continuous at x = a if lim- f ( x ) = lim+ f ( x ) = f(a), that is,
x ®a x ®a

2 cos x - 1 L.H.L. = R.H.L. = Value of the function at a, that is, lim f ( x ) = f (a).


Illustration 19.17   f ( x ) = æ p ö except
for all x in x ®a
cot x - 1 ç0, ÷
è 2ø f(x) will be discontinuous at x = a in any of the following cases:
p æ p ö p
at x = . Define f ç ÷ so that f(x) may be continuous at x = . 1.   lim- f ( x ) and lim+ f ( x ) exist but are not equal.
4 è4ø 4 x ®a x ®a
p æp ö
Solution: f(x) will be continuous at x = ifif lim f ( x ) = f ç ÷ 2.   lim- f ( x ) and lim+ f ( x ) exist and are equal but not equal
4 x ®p / 4 è4ø x ®a x ®a
p æp ö to f(a).
x = if lim f ( x ) = f ç ÷
4 x ®p / 4 è4ø 3. f(a) is not defined.
æp ö 2 cos x - 1 4. At least one of the limits does not exist.
⇒ f ç ÷ = lim
è4ø x ®p / 4 cot x - 1 If f and g are continuous at x = a, then f + g, kf and fg are continuous
at x = a. Moreover, if g(a) ≠ 0, then f/g is also continuous at x = a.
( 2 cos x - 1) sinx Further if g is continuous at a and f is continuous at g(a), then the
= lim
x ®p / 4 cos x - sin x composition of f( g(a)) is continuous at x = a.

Mathematical Problem Book for JEE.indb 806 07-06-2018 12:59:50


Chapter 19 | Limit, Continuity and Differentiability 807

log( x + 2) - x 2n sin x x∈I ∩ D (That is, integer lying in the set of domain)
Illustration 19.18   If f ( x ) = lim , then
n ®¥ x 2n + 1 ⇒ x∈I- {-1}
examine continuity of f(x).
Solution: 19.10.5  Properties of Continuous Functions
If |x| < 1, then Let f (x) and g(x) are continuous functions at x = a. Then
2n
log( x + 2) - x sin x log( x + 2) 1. c f (x) is continuous at x = a where c is any constant
f ( x ) = lim 2n
 =  = log ( x + 2) 2. f (x) ± g(x) is continuous at x = a
n ®¥ x +1 1
3. f (x)⋅g(x) is continuous at x = a
If |x| > 1, then
4. f (x)/g(x) is continuous at x = a, provided g(a) ≠ 0
log( x + 2) - x 2n sin x
f ( x ) = lim 19.10.6  Intermediate Value Theorem
n ®¥ x 2n + 1
log( x + 2) If c is any real number between f(a) and f(b), then there exists at
- sin x least one solution of the equation f(x) = c in the open interval (a, b),
= lim x 2n = - sin x
n ®¥ 1 if y = f(x) is continuous in the interval. (See Fig. 19.12.)
1+ 2 n
x
If | x | = 1, then f (b)
log( x + 2) - x 2n sin x log( x + 2) - sin x
f ( x ) = lim  = 
n ®¥ 2n
x +1 2 c
ì - sin x , x < -1
ï log( x + 2), f (a)
ïï -1< x < 1
Thus, f ( x ) = í - sin x , x >1 a b
ï
ï log( x + 2) - sin x
ïî , x = ±1
2 Figure 19.12
Obviously f(x) is discontinuous at x = ± 1.
19.10.7  Types of Discontinuities
æ p ö
Illustration 19.19   Let f ( x ) = [ x ]sin ç ÷ where [.] denotes Basically there are two types of discontinuity:
è [ x + 1] ø
greatest integer function. Then find domain of f and the points of 1. Removable discontinuity: If lim f ( x ) exists but is not equal
x ®a
discontinuity of f in the domain. to f (a), then f (x) has a removable discontinuity at x = a and it
æ p ö can be removed by redefining f (x) for x = a.
Solution: Let f ( x ) = [ x ]sin ç ÷ . Then domain of f(x) is x∈ R
è [ x + 1] ø
Properties of a continuous function: A function f(x), continu-
excluding the points where [x  +  1]  =  0 (As denominator cannot
be zero). ous on the interval [a, b], possesses the following properties:
0≤x+1<1 (i) f(x) is bounded on [a, b], that is, there exist m and M such that
⇒ -1 ≤ x < 0 m ≤ f(x) ≤ M for all x ∈ [a, b], m and M are respectively mini-
That is, for all x∈ [-1, 0), denominator is zero. So, domain is mum and maximum values of f(x).
x ∈ R - [-1 , 0) (ii) Let A be any number such that min f(x) ≤ A ≤ max f(x). Then
there exists a point x0∈ [a, b] such that f(x0) = A. It is called
Points of Discontinuity intermediate value theorem.

Check the continuity at x = a (where a∈I )


Illustration 19.20   Redefine the function f(x) = [x] + [-x] in such a
æ p ö way that it could become continuous for x∈ (0, 2).
LHL = lim [a - h]sin ç ÷
h®0 è [a + 1- h] ø
Solution: Here lim f ( x )  = -1 but f(1) = 0.
x ®1
æp ö
⇒ LHL = (a - 1)sin ç ÷ (19.3) Hence, f(x) has a removable discontinuity at x = 1.
èaø
To remove this we define f(x) as
æ p ö
RHL = lim [a + h]sin ç ÷ f(x) = [x] + [-x], x∈ (0, 1) ∪ (1, 2)  
h®0 è [a + 1+ h] ø
f(x) = -1, x = 1
æ p ö
⇒ RHL = a sin ç ÷ (19.4) Now, f(x) is continuous for x∈ (0, 2).
è a + 1ø
2. Non-removable discontinuity: If lim f ( x ) does not exist, then
From Eqs. (19.3) and (19.4), we get LHL ≠ RHL. So, f(x) is discontinu- x ®a
ous at x = a (That is, at integral values of x). we cannot remove this discontinuity. So this becomes a non-
So, points of discontinuity are removable discontinuity or essential discontinuity.

Mathematical Problem Book for JEE.indb 807 07-06-2018 13:00:07


808 Mathematics Problem Book for JEE

provided the limit exists. That is,


Illustration 19.21   Prove that f(x) = [x] has essential discontinuity
f ( x + h) - f ( x ) f ( x - h) - f ( x )
at any x∈ I. lim  =  lim-
h®0+ h h→ 0 -h
Solution: Proof is obvious as lim f ( x ) does not exist for any a∈I.
x ®a The function is said to be differentiable at x = a if Right hand deriv-
Hence, f(x) = [x] has essential discontinuity at any x∈I. ative (RHD) at x = a denoted by
f ( a + h) - f ( a )
Your Turn 2 f  ′(a + 0) =  lim
h®0 -h
1
 1. Is f ( x ) = continuous in (0, 2)? Ans.  Yes exists and Left hand derivative (LHD) at x = a denoted by
x
f ( a - h) - f ( a )
  2. For continuity lim+ f ( x ) or lim- f ( x ) should be f  ′(a - 0) =  lim
x ®a x ®a h®0 -h
also exists.
equal to f(a). (True/False) Ans.   False
In both these cases, we have assumed h > 0.
ì x sin (1/ x ) x ¹ 0 A function whose graph is otherwise smooth will fail to have a
  3. f ( x ) = í is continuous in its domain. derivative where the graph has
î 0 x =0
1. A corner, where the one-sided derivatives differ (Fig. 19.13).
(True/False) Ans.  True
1 1 P
 4. Is f ( x ) =+ continuous in (-∞, 0]? Ans.  No
x |x|
  5. A continuous function can have some points
where limit does not exists. (True/False) Ans.  False
ìax + 1 x ³ 1 Q−
  6. I f f ( x ) = í is continuous, then ‘a’ should be Q+
î x + 2 x <1
_____.
Ans.  2
 7. y = |x| is a continuous function. (True/False) Ans.   True Figure 19.13
[x] 2. A cusp, where the slope of PQ approaches ∞ from one side and
 8. Number of points of discontinuities of f ( x ) = in [1, 3]
1+ | x | -∞ from the other (Fig. 19.14).
are _____.
Ans.   2 points
 9. f(x) = {x} + [x] is a continuous function. (True/False) P
Ans.  True
x2
10. If f(x) = 2x and g(x) =  + 1 , then which of the following can
2
be a discontinuous function. Q− Q+
(A)  f(x) + g(x), (B)  f(x) - g(x),
g( x )
(C)  f(x) g(x) (D)   Ans.  (D)
f (x)
Figure 19.14
19.11 Differentiability 3. A vertical tangent, where the slope of PQ approaches ∞ from
Let y = f(x) be a given function. If at some point, abscissa is x1 and at both sides or approaches -∞ from both sides (here, -∞)
other point abscissa is x2, then it is quite natural that ordinate can (see Fig. 19.15).
be represented by y1 and y2, respectively at those points.
∆y = y2 - y1, ∆y represents change in ‘y’.
∆x = x2 - x1, ∆x represents change in ‘x’.
then     ∆y = f(x1 + ∆x) - f(x1)
clearly increment can be positive, negative or may even be zero.
Differential coefficient of y = f(x), with respect to x is defined as the
Dy P
limiting value of   as ∆x tends to zero.
D x
dy
It is usually denoted by or f ′(x) symbolically.
dx
The derivative of the function with respect to x is the function f  ′(x)
whose value at x is
f ( x + h) - f ( x )
f ¢( x ) = lim
h®0 h Figure 19.15

Mathematical Problem Book for JEE.indb 808 07-06-2018 13:00:23


Chapter 19 | Limit, Continuity and Differentiability 809

4. A discontinuity (Fig. 19.16). 19.11.2  Properties of Differentiability


(a) (b)
Let f(x) and g(x) are differentiable functions at x = a. Then
1. cf (x) is differentiable at x = a where c is any constant
2. f(x) ± g(x) is differentiable at x = a
p p 3. f(x)⋅g(x) is differentiable at x = a
4. f(x)/g(x) is differentiable at x = a, provided g(a) ≠ 0

Figure 19.16 Differentiability


A function is continuous at every point where it has a derivative. Let y = f(x) be a continuous function at a point x = a. It is said
Proof: Given that f ′(c) exists, we must show that lim f ( x ) = f (c ), or, f ( x ) - f (a)
to be differentiable at x = a if lim exists and is finite.
x ®c x ®a x -a
equivalently, that lim f (c + h) = f (c ). If h ≠ 0, then If this limit exists we call it the derivative of y = f(x) at a
h®0

f (c + h) = f (c ) + [( f (c + h) - f (c )] Right Hand Derivative:


f ( a + h) - f ( a )
f ( c + h) - f ( c ) RHD at f(x) at x = a is, Rf ′(a) = lim , h > 0.
= f (c ) + ×h h®0 h
h
Left Hand Derivative:
Now take limits as h→ 0.
f ( a - h) - f ( a )
LHD at f(x) at x = a is, lim , h > 0.
f ( c + h) - f ( c ) h®0 -h
lim f (c + h) = lim f (c ) + lim × lim h
h®0 h®0 h®0 h h®0
Clearly, f(x) is differentiable at x = a if and only if Rf ′(a) = Lf ′(a).
= f(c) + f ′(c)⋅0 = f(c) + 0 = f(c) Notes: If a function f(x) is differentiable at x = a, then it is also
Similar arguments with one-sided limits show that if f has a deriv- continuous at x = a. But if a function is continuous at a point, it
ative from one side (right or left) at x = c, then f is continuous from is not necessarily differentiable at that point. Let us consider the
that side at x = c. function f(x) = |x|.
f ( x ) - f (a)
19.11.1 Differentiability in an Interval Geometrically, we interpret f ¢(a) = lim as the slope of
x ®a x -a
1. Differentiability in an open interval (a, b): The function of the graph at the point [a, f(a)]. The line through [a, f(a)] which
y = f(x) is said to be differentiable in (a, b) if it is differentiable has this slope is called the tangent line at [a, f(a)]. Thus, if there is
no tangent line at a certain point, the function is not differentia-
at each point x∈ (a, b).
ble at that point. In other words, a function is not differentiable
2. In a closed interval [a, b]: The function y = f(x) is said to be
at a point where the curve suddenly changes direction (corner
differentiable in [a, b] if f ′(a + 0), f ′ (b - 0) exist and point) or the tangent is vertical at some point.
f  ′(x) exist for all x∈ (a, b).

ì | x - 1| ([ x ] - x ), x ¹1
Illustration 19.22   If f ( x ) = í
î 0 , x =1 Your Turn 3
.
Test the differentiability at x  =  1, where [ ] denotes the greatest
1. 
Which of the following functions, whose graphs are given, will
integer function.
have derivatives at all points:
Solution: Check the differentiability at x = 1 (A)  (B) 
f (1+ h) - f (1)
R [ f ′(1)] = lim (x > 1)
h→ 0 h
| 1+ h - 1| {[1+ h] - (1+ h)} - 0
= lim
h®0 h (C)  (D) 
h(1- 1- h) h( -h)
= lim = lim =0
h®0 h h®0 h
f (1- h) - f (1)
L[f ¢(1)] = lim (x < 1) Ans.  (A)
h®0 -h
2.  Is |x + 1| is differentiable at x = -1? Ans.  No
f (1- h) - 0
= lim 1 æ 1ö 1
h®0 -h 3.  Derivative of y = sin ç ÷ at x = 0 is . (True/False)
x èxø 2
| 1- h - 1| {[1- h] - (1- h)} - 0 Ans.  False
= lim
h®0 -h ìï x 2 + 3 x + b x ³ 1
4. If f ( x ) = í is continuous and differentiable,
h(0 - 1+ h) ïî 2ax + 3 x <1
= lim =1
h®0 -h then find out value of a and b? Ans.  a = 5/2 and b = 4
Lf ′(1) ≠ Rf ′(1) 5.  If f(x) = 3x + x sinx, then f ′(1) is .
Hence, f(x) is not differentiable at x = 1. Ans.  3 + cos1 + sin1

Mathematical Problem Book for JEE.indb 809 07-06-2018 13:00:43


810 Mathematics Problem Book for JEE

f (x) Since
If f(x) = x2 and g(x) = x, then derivative of
 6.  at x = 0 is 1. L.H. limit ≠ R.H. limit
g( x )
(True/False) Therefore, limit does not exist.
Ans.  False Hence, the correct answer is option (D).
f (x)
 7. If lim = 2 and f (1) = 0, then f ′(1) =  . nk sin2 n !
x ®1 x - 1  4. If lim = 0 for
Ans.   2 n ®¥ n +1
 8. Find values of ‘x’ for which derivative of f(x) = {x} exists. (A)  All k (B)  0 ≤ k < 1
(where {.} denotes fractional part of x) Ans.  R - I (C)  k = 1 (D) For k > 1
f ¢( x + h) - f ¢( x ) Solution:
 9. f ′′(x) =  lim . (True/False) Ans.  True
h®0 h nk sin2 n ! nk
lim = lim × (sin2 n !)
10.  Which of the following functions is/are differentiable n ®¥ n +1 n ®¥ n + 1

(A)  y = log |x| (B)  y = |x|
lim sin2 n ! does not exist but if
1 n ®¥
(C)  y = |x|2 + 2x + 3 (D)   Ans.  (C) nk nk sin2 n !
1 - | x |2 lim = 0 , then lim =0
n ®¥ n + 1 n ®¥ n +1
Additional Solved Examples But
nk
lim =0 ⇔0≤k<1
æp ö æp ö n ®¥ n + 1
 1. If lim n cos ç ÷ sin ç ÷ = k, then k is
n ®¥ è 4n ø è 4n ø Hence, the correct answer is option (B).
p p
(A)  (B)  (C)  π (D)  None of these ïì (cos x )
1/sin x
for x ¹0
4 3 5. If f ( x ) = í
îï k for x =0
Solution:
p The value of k, so that f is continuous at x = 0 is
sin (A) 0 (B) 1 (C) 1/2 (D)  None of these
n p n 2 n×p =p
k = lim sin = lim
n ®¥ 2 2n n ®¥ 2 p / 2n 2n 4 Solution: Given, f(0) = k. Now
Hence, the correct answer is option (A). lim f ( x ) = lim (cos x )1/sin x [1∞ form]
x ®0 x ®0
 2. If lim (1+ a sin x )cosec x = 3, then a is 1
x ®0 (cos x -1)
= lim e sin x
(A)  ln 2 (B)  ln 3 (C)  ln 4 (D)  log 2 x ®0
2

Solution: = lim (e )( -2 sin x / 2 ) (( 2 sin x / 2 ) cos x / 2 )


x ®0
x
3 = lim (1+ a sin x )cosec x [1∞ form] = lim e
- tan
2 = e0 = 1
x →0
x ®0
= lim e cosec x . a sin x = e a For f(x) to be continuous at x = 0, k should be equal to 1.
x ®0
Hence, the correct answer is option (B).
Hence,
ea = 3 ⇒a = loge3 = ln 3 6. If f(x) = [ 2sinx ] , where [x] denotes the greatest integer
Hence, the correct answer is option (B). function, then
x sin{ x } (A)  f(x) is continuous at x = 0
 3. lim , where {x} denotes the fractional part of x, is (B)  Maximum value of f(x) is 1 in interval [-2π, 2π]
x ®1 x -1
equal to np p
(C)  f(x) is discontinuous at x = + , n ∈ I
(A)  -1 (B) 0 (C) 1 (D)  Does not exist 2 4
(D)  f(x) is differentiable at x = nπ, n ∈ I
Solution:
lim { x } = lim ( x - [ x ]) = 1- 0 = 1 Solution:
x ®1- 0 x ®1- 0
f ( x ) = [ 2 sin x ]
lim { x } = lim ( x - [ x ]) = 1- 1 = 0
x ®1+ 0 x ®1+ 0 Hence,
Therefore, p p p p
f ( x ) = -2, - £ x < - = -1, - £ x < 0 = 0 , 0 £ x <
2 4 4 4
x sin{ x } x
lim = lim sin { x } = -∞⋅sin (1) = -∞ p p p p
x ®1- 0 x -1 x ®1- 0 x - 1 f ( x ) = -2, - £ x < -f(x) = -1, - £ x < 0 = 0 , 0 £ x <
2 4 4 4
x sin{ x } x sin{ x } { x } x sin{ x } x - 1
lim = lim × = lim p p p p
x ®1+ 0 x -1 x ®1+ 0 {x} x - 1 x ®1+ 0 { x } fx( x-)1= -2, - £ x < - = -1, - £ x f(x)
< 0 = 0, 0 £ x <
2 4 4 4
= 1´ 1´ 1 = 1

Mathematical Problem Book for JEE.indb 810 07-06-2018 13:01:11


Chapter 19 | Limit, Continuity and Differentiability 811

p p p 3p 3p From Eqs. (1) and (2), for f(x) to be not differentiable but
f(x) = 1 , £ x £ = 1, £ x £ =0, < x £ pcontinuous at x = 0, possible values of p are given by 0 < p ≤ 1.
4 2 2 4 4
Hence, the correct answer is option (C).
p p p 3p 3p
= 1 , £ x £f(x) = 1 , £ x £ =0, < x £p
4 2 2 4 4 e - (1+ x )1/ x
9.  lim =
p p p 3p 3p x ®0 tan x
= 1 , £ x £ = 1 , £ x £ f(x) = 0 , < x £p
4 2 2 4 4 (A)  e (B)  e/2 (C)  e2 (D)  None of these
Clearly f(x) is discontinuous at Solution:
p p 3p e - (1+ x )1/ x
x = - , 0 , , , etc. lim
4 4 4 x ®0 x
(tan x ) ×
General value corresponding to x
p p 3p np p æ x 11 2 ö
x =- , , is + e - e ç 1- + x -÷
4 4 4 2 4 è 2 24 ø [using expansion]
= lim
p x ®0 x
Maximum value of f(x) in [-2π, 2π] is 1 at x = .
2
e x æ 11 ö
f(x) is discontinuous and non-differentiable at x  =  0, therefore ç 1- x +  ÷ e
2 è 12 ø=
choice (D) is not correct. = lim
x ®0 x 2
Hence, the correct answer is option (B).
(27 - 2 x )1/ 3 - 3 Hence, the correct answer is option (B).
7. The value of f(0), so that the function f(x) = 1/ 5
9 - 3 (243 + 5 x ) æ 1 q 1 q 1 q ö
(x ≠ 0) is continuous is 10.  lim ç tanq + tan + 2 tan 2 +  + n tan n ÷ =
n ®¥ è 2 2 2 2 2 2 ø
(A) 2/3 (B) 6 (C) 2 (D) 4
Solution: 1 1
(A)  (B)  - 2 cot 2θ
1/ 3
(27 - 2 x ) - (27) 1/ 3 q q
lim f ( x ) = lim
x ®0 x ® 0 3[3 - (243 + 5 x )1/ 5 ] (C)  2 cot 2θ (D)  None of these
(27 - 2 x )1/ 3 - (27)1/ 3 Solution:
( -2 x ) tan θ = cot θ − 2 cot 2θ
= lim (27 - 2 x ) - 27
x ®0 é (243 + 5 x )1/ 5 - (243))1/ 5 ù Therefore,
-3 ê ×5x ú 1 q 1 q
ë 243 + 5 x - 243 û tan = cot - cot q
2 2 2 2
1
(27)-2 / 3 1 q 1 q 1 q
2 3 2 5 1 tan n = n cot n - n -1 cot n -1
= ⋅ = ⋅ ⋅ ⋅ 81 = 2 2n 2 2 2 2 2
15 1 (243)-4 / 5 15 3 9
5 Hence,
1 q
Hence, the correct answer is option (C). Sn = cot n - 2 cot 2q
2n 2
ì p 1
ï x sin , x ¹0 So, required limit is 
8.  Let  f ( x ) = í x
ïî 0 , x =0 æ ö
ç 1 ÷
Then f(x) is continuous but not differentiable at x = 0 if lim Sn = lim ç n - 2 cot 2q ÷ = 1 - 2 cot 2q
n ®¥ n ®¥ ç 2 tanq / 2n q ÷ q
(A)  p < 0 (B)  p = 0 (C) 0 < p ≤ 1 (D)  p ≥ 1 ç n
× n ÷
è q /2 2 ø
Solution:
f(0) = 0 Hence, the correct answer is option (B).
For f(x) to be continuous at x = 0 | x 3 - 6 x 2 + 11x - 6 |
1 p 11. Let f ( x ) = . Find the set of points ‘a’, where
lim f ( x ) = 0 Þ lim x sin = 0 x 3 - 6 x 2 + 11x - 6
x ®0 x ®0 x
lim f ( x ) does not exists.
This is possible only when p > 0. (1) x ®a

p 1
h sin - 0 Solution: We write,
f ( h) - f ( 0 ) h
f ′(0) = lim = lim ì-1, x <1
h® 0 h h® 0 h ï
| x - 1| | x - 2 | | x - 3 | ï1, 1< x < 2
1 f (x) = × × =í
= lim h p -1 sin x -1 x - 2 x - 3 ï-1, 2< x <3
h®0 h
ïî1, x >3
f ′(0) will exist only when p > 1.
Hence, f(x) will not be differentiable if p ≤ 1. (2) Therefore, the limits exists at all points except at x = 1, 2, 3.

Mathematical Problem Book for JEE.indb 811 07-06-2018 13:01:35


812 Mathematics Problem Book for JEE

As lim- f ( x ) = -1 , Hence, the correct answer is the function which is neither


x ®1 differentiable nor continuous at x = 0.
lim f ( x ) = 1
x ®1+ 14. 
Find a polynomial of least degree, such that
So, lim f ( x ) does not exist. 1/ x
x ®1 æ x2 + f ( x) ö
lim çç 1+ ÷ = e2
Similarly for x = 2 and x = 3.
x ®0
è x 2 ÷ø
Therefore, required set is {1, 2, 3}.
Solution: Now,
Hence, the correct answer is {1, 2, 3}. 1/ x
æ x2 + f ( x) ö
lim çç 1+ ÷÷
(1+ x )1/ x + e ( x - 1) x ®0
è x2 ø
12.  Evaluate lim .
x ®0 x
1/ x
Solution: æ x2 + f ( x) ö
Þ lim çç 1+ ÷ = L (say)
1
ln(1+ x )
x ®0
è x 2 ÷ø
(1+ x )1/ x + e ( x - 1) ex -e
lim = lim +e
x ®0 x x ®0 x x2 + f ( x)
exists only when lim = 0 (i.e., it converts to 1∞ form).
x x2 x x2
x ®0 x2
1- + -  - + -
e 2 3 -e e 2 3 -1 So, the least degree in f(x) is of degree 2. That is,
= lim + e = e lim +e
x →0 x x →0 x f(x) = a2x2 + a3x3 + . . .
x x2
 − 2 + 3 −L  1 2x  Now,
e − + −L
  2 3  1/ x
= e lim + e (using L′H rule) æ x2 + f ( x) ö
x →0 1 L = lim çç 1+ ÷÷ = e2
x ®0
è x2 ø
e e
=- +e = æ x2 +f ( x ) ö 1
2 2 lim ç 2 ÷ lim
x2 +f ( x )
x
=e è x ø = e 2 = e x ®0 = e2
x ®0
x3
Hence, the correct answer is (e/2).
Hence,
e1/ x - 1
Check the function f(x) = lim
13.  for continuity and x2 + f ( x) x 2 + a2 x 2 + a3 x 3 + 
x ® 0 e1/ x + 1 lim = 2 Þ lim =2
3
x ®0 x x ®0 x3
differentiability at x = 0.
So, a2 = -1, a3 = 2 and a4, a5 are any arbitrary constants. Since, we
e1/ x - 1
Solution: Let f ( x ) = . Then, want polynomial of least degree. Hence,
e1/ x + 1
f(x) = -x2 + 2x3
LHL = lim- f ( x ) = lim f (0 - h)
x ®0 h®0 Hence, the correct answer is f(x) = −x2 + 2x3.
-1/ h 1/ h
e -1 (1/ e - 1) 0 - 1
= lim = lim = = -1 (2 x - 1)3 .
h®0 e -1/ h
+1 h ® 0 (1/ e1/ h + 1) 0 + 1 15.  Evaluate lim
x ®0 æ x2 ö
1 sin 2 x In çç 1+ ÷÷
[as h → 0 ⇒ → ∞ ⇒ e1/h → ∞ ⇒ 1/e1/h → 0] (1) è 2 ø
h
Solution:
RHL = lim+ f ( x ) = lim f (0 + h)
x ®0 h®0 3
æ 2 x - 1ö
e1/ h - 1 (1- 1/ e1/ h ) çç ÷÷
= lim = lim (2 x - 1)3 1 è x ø
1/ h × =

h®0 e + 1 h®0 (1+ 1/ e1/ h ) sin 2 x æ x 2 ö sin 2 x 1 æ x2 ö
ln çç 1 + ÷÷ × 2 ln çç 1 + ÷
[Dividing numerator and denominator by e1/h] è 2 ø x x è 2 ÷ø
1- 0
= = 1 [using Eq. (1)] 3 3
1+ 0 æ 2 x - 1ö é æ 2 x - 1öù 3
lim çç ÷÷ = ê lim çç ÷÷ ú = (ln 2)
Clearly, lim f ( x ) ¹ lim+ f ( x ) x ®0
è x ø êë x ® 0
è x ø úû
x ®0- x ®0
é 1 ù
sin 2 x 1 æ x2 ö êæ x2 ö x ú
2

Hence, lim f ( x ) does not exist. lim = 2 and lim 2 ln çç 1 + ÷÷ = lim ln êçç 1 + ÷÷ ú
x ®0 x ®0 x x ®0 x
è 2 ø x ® 0 êè 2 ø ú
As lim f ( x ) does not exist, function is neither continuous nor é 1 ù ë û
sin 2 x 1 æ x2 ö êæ x2 ö x ú
2
x ®0
lim = 2 and lim 2 ln çç 1 + ÷ = lim ln ç 1 + ÷
differentiable at x = 0. x ®0 x x ®0 x
è 2 ÷ø x ® 0 êêçè 2 ÷ø úú
ë û

Mathematical Problem Book for JEE.indb 812 07-06-2018 13:02:02


Chapter 19 | Limit, Continuity and Differentiability 813

é 1 ù px
sin 2 x 1 æ x2 ö êæ x2 ö x ú
2 which sin = 1 , that is, x is an odd integer.
lim = 2 and lim 2 ln çç 1 + ÷÷ = lim ln êçç 1 + ÷÷ ú 2
x ®0 x x ®0 x
è 2 ø x ® 0 êè 2 ø ú
ë û So, x = (2n + 1) where n∈ I.
1 Check continuity at x = (2n + 1)
æ x2 ö 1
= ln lim çç 1 + ÷ x2 = lne e1/ 2 = LHL =  lim f ( x ) = 0 (1)
x ®0
è 2 ÷ø 2 x ®2 n +1

and f(2n + 1) = 1 (2)


(ln 2)3
Therefore, required limit =  = (ln 2)3 from Eqs. (1) and (2), we get
1
2´ LHL ≠ f(2n + 1)
2
Therefore, f(x) is discontinuous at x = 2n + 1 (That is, at odd inte-
Hence, the correct answer is (ln 2)3. gers).
ì - æç 1 + 1 ö÷ Hence, f(x) is discontinuous at x = (2n + 1).
ï | x| x ø
16.  Let f(x) = í xe è , x ¹ 0 . Then test whether Hence, in the given interval, f(x) is discontinuous at x = 1.
ï 0, x =0
î 18.  Show that the function f(x) is continuous at x = 0 but its
(A)  f(x) is continuous at x = 0 derivative does not exists at x = 0 if
(B)  f(x) is differentiable at x = 0 ìï x sin(log x 2 ); x ¹0
f(x) =  í
Solution: ïî 0; x =0
(A)  f(0+) = lim xe -2 / x (since |x| = x)
x ®0+ Solution: Test for continuity
x LHL = lim f (0 - h) = lim( -h)sin log( -h)2
= lim =0 2/ x
h®0 h®0

e x ®0 2
= - lim h sin log h
h®0
f(0-) = lim x e 0 = lim x = 0
x ® 0- x ®0 As h → 0, log h2 → -∞
f(0) = 0 Hence, sin log h2 oscillates between -1 and +1. So,
Hence, f(x) is continuous at x = 0. LHL =  - lim (h) ´ lim (sin log h2 )
h®0 h®0
(B)  Regarding differentiability
= -0 × (number between -1 and +1) = 0
f ( x ) - f (0) RHL = lim f (0 + h)
f ¢(0 + ) = lim h®0
x ®0 x
xe -2 / x 1 = lim h sin log h2 = lim h × lim sin log h2
= lim = lim 2 / x = 0 h®0 h®0 h®0
x ®0 x x ®0 e
= 0 × (oscillating between -1 and + 1) = 0
f ( x ) - f (0) f(0) = 0 (given)
f ¢(0 - ) = lim
x ®0 x ⇒ LHL = RHL = f(0)
x -0 Hence, f(x) is continuous at x = 0.
= lim =1
x ®0 x Test for differentiability
The two values are different. f ( 0 - h) - f ( 0 )
Lf ¢(0 ) = lim
Hence, f(x) is not differentiable at x = 0. h®0 -h
17. 
Discuss the continuity of f(x) in [0, 2] where -h sin log( -h)2 - 0
2n = lim = lim sin(log h2 )
æ pxö h®0 -h h®0
f ( x ) = lim ç sin ÷
n ®¥ è 2 ø As the expression oscillates between -1 and +1, the limit does not
Solution: Since exists. Therefore, Left hand derivative is not defined.
ì 0; | x |< 1 Hence, the function is not differentiable at x = 0.
lim x 2n = í
n ®¥ î 1; | x |= 1 1/ x
ì 5e + 2
x ¹0
19.  If f(x) =  ïí 3 - e1/ x
Therefore, ,
0 ; px ï ,
sin <1 î0 x =0
2n  2
 p x 
f ( x ) = lim  sin  = Examine (i) f(x) (ii) x2f(x) for continuity and differentiability
n→∞  2  px at x = 0.
 1; sin =1
2 Solution:
Thus, f(x) is continuous for all x, except for those values of x for (i) For f(x)

Mathematical Problem Book for JEE.indb 813 07-06-2018 13:02:25


814 Mathematics Problem Book for JEE

5e1/ h + 2 æ 1 ö
RHL =  lim f ( x ) = lim f (0 + h) = lim ( h - 1)sin ç ÷ - sin 1
x ®0 + h®0 h ® 0 3 - e1/ h f ( h) - f ( 0 ) è h - 1ø
f ¢(0 ) = lim Þ f ¢(0 ) = lim
h®0 h h®0 h
5 + 2e -1/ h 5 + 0
= lim = = -5 This limit also does not exist, which implies that f is also not differ-
h ® 0 3e -1/ h - 1 0 -1
entiable at x = 0.
Value of function = f(0) = 0 Hence, the correct answer is option (A).
RHL ≠ Value of function
3. Let f (x) = x | x | and g(x) = sin x.
Hence, f(x) is not continuous and hence not differentiable
Statement-1:  gof is differentiable at x = 0 and its derivative is
also.
continuous at that point.
(ii) Let F(x) = x2 f(x). Then Statement-2:  gof is twice differentiable at x = 0.
(A) Statement-1 is true, Statement-2 is true; Statement-2 is a
F ( 0 - h) - F ( 0 ) h2f ( -h) - 0 correct explanation for Statement-1
L[F ¢(0 )] = lim = lim
h®0 -h h®0 -h (B) Statement-1 is true, Statement-2 is true; Statement-2 is not
= lim hf ( -h) = 0 a correct explanation for Statement-1
h®0
(C)  Statement-1 is true, Statement-2 is false
F ( 0 + h) - F ( 0 ) h2f ( h) - 0 (D)  Statement-1 is false, Statement-2 is true [AIEEE 2009]
RF ¢(0 ) = lim = lim = lim hf ( h) = 0
h®0 h h®0 h h®0 Solution: We have,
Hence, LF ′ (0) = RF′(0). f (x) = x | x | and g(x) = sin x

ïì- sin x ,
2
Hence, F(x) is differentiable at x = 0, so it is always continuous x <0
at x = 0. gof ( x ) = sin( x | x |) = í
2
îïsin x , x ³0

Previous Years’ Solved JEE Main/AIEEE Therefore,


ìï-2 x cos x 2 , x <0
Questions ( gof )¢( x ) = í
2
ïî 2 x cos x , x ³0
1.  The function f : R - {0} → R given by
ïì-2 cos x + 4 x sin x ,
2 2 2
x <0
1 2 ( gof )¢¢( x ) = í
f ( x) = - 2x 2 2 2
îï 2 cos x - 4 x sin x , x ³0
x e -1
Now LHD at 0,
can be made continuous at x = 0 by defining f(0) as
(A) 2 (B)  -1 (C) 0 (D) 1 gof (0 - h) - gof (0 ) - sin h2 - 0 æ sin h2 ö
lim = = lim çç 2 ÷÷ ´ h = 0
[AIEEE 2007] h® 0 -h -h h® 0
è h ø
Solution: We have And RHD at 0,
1 2 e 2 x - 1- 2 x æ sin h2 ö
lim - 2 x Þ lim gof (0 + h) - gof (0 ) sin h2 - 0
x ®0 x e - 1 x ®0 x (e 2 x - 1) lim = = lim çç 2 ÷÷ ´ h = 0
h® 0 h h h® 0
è h ø
2e 2 x - 2 4e2 x Clearly, L(gof )′ (0) = 0 = R(gof )′ (0)
Þ lim 2x 2 x
Þ lim 2 x =1
x ® 0 (e - 1) + 2 xe x ®0 4e + 4 xe 2 x Therefore, gof is differentiable at x  =  0 and also its derivative is
Hence, the correct answer is option (D). continuous at x = 0.
Now,
ì æ 1 ö
( x - 1)sin ç ÷ , if x ¹ 1 . Then which one of the ( gof )′′ ( x ) = -2[ x ( - sin x 2 )2 x + cos x 2 ], x < 0
Let f ( x ) = ïí
2.  è x - 1ø
ï0 , = 2[ x ( - sin x 2 )2 x + cos x 2 ], x > 0
î if x = 1
Now, gof ′′ (0)
following is true?
(A)  f is neither differentiable at x = 0 nor at x = 1 - cos(h2 )2( - h) - 0
LHD = lim = -2(1) = -2
(B)  f is differentiable at x = 0 and at x = 1 h→ 0 -h
(C)  f is differentiable at x = 0 but not at x = 1 cos(h2 )2(h) - 0
RHD = lim =2
(D)  f is differentiable at x = 1 but not at x = 0 [AIEEE 2008] h→ 0 h
Solution: We have L(gof )′′ (0) = -2 and R(gof )′′ (0) = 2. Hence L(gof )′′ (0) ≠ R(gof )′′ (0)
æ 1 ö That is, gof(x) is not twice differentiable at x = 0.
(1+ h - 1)sin ç ÷ Hence, the correct answer is option (C).
f (1+ h) - f (1) è 1+ h - 1 ø
f ¢(1) = lim Þ f ¢(1) = lim 1
h®0 h h®0 h 4.  Let f : R → R be a continuous function defined by f ( x ) = .
e + 2e - x
x
h æ 1ö æ 1ö 1
= lim sin ç ÷ Þ f ¢(1) = lim sin ç ÷ Statement-1: f (c ) = , for some c ∈ R.
h®0 h èhø h®0 èhø 3
As the limit does not exist, therefore, f is not differentiable at x = 1. 1
Statement-2: 0 < f ( x ) £ , for all x ∈ R.
Similarly, 2 2

Mathematical Problem Book for JEE.indb 814 07-06-2018 13:02:45


Chapter 19 | Limit, Continuity and Differentiability 815

(A) Statement-1 is true, Statement-2 is true; Statement-2 is not


the correct explanation for Statement-1 x + x2 - x x + x2 - x x + x2 + x
RHL = lim 3/2
= lim 3 / 2
´
(B)  Statement-1 is true, Statement-2 is false x ®0 x x ®0 x x + x2 + x
(C)  Statement-1 is false, Statement-2 is true x + x2 - x x 1
(D) Statement-1 is true, Statement-2 is true; Statement-2 is the = lim = =
x ®0
x 3/2 2
( x+ x + x) 2
( x+ x + x) 2
correct explanation for Statement-1
[AIEEE 2010] Therefore,
Solution:
1 1 3
q= Þ p+2 = Þ p = -
1 ex (e 2 x + 2)e x - 2e 2 x × e x 2 2 2
f (x) = x -x
= 2x Þ f ¢( x ) =
e + 2e e +2 (e2 x + 2 ) 2
Hence, the correct answer is option (B).
f ¢( x ) = 0 Þ e 2 x + 2 = 2e 2 x Þ e 2 x = 2 Þ e x = 2
æ 2 x - 1ö
2 2 1 1
If f : R → R is a function defined by f ( x ) = [ x ]cos ç
7.  ÷p ,
Maximum f ( x ) = = = . That is, 0 < f ( x ) £ "x Î R è 2 ø
2+2 4 2 2 2 2 where [x] denotes the greatest integer function, then f is
1 1 1 (A)  Continuous for every real x
Since 0 < < , therefore for some c Î R , we have , f ( c ) = .
3 2 2 3 (B)  Discontinuous only at x = 0
(C)  Discontinuous only at non-zero integral values of x
Therefore, we conclude that Statement-1 is true, Statement-2 is
(D)  Continuous only at x = 0
true; Statement-2 is the correct explanation for Statement-1
Hence, the correct answer is option (D). [AIEEE 2012]
Solution:
æ 1- cos[2( x - 2)] ö
5.  lim çç ÷÷  2 x - 1  1
x ®2 x -2 f ( x ) = [ x ]cos  p = [ x ]cos  x -  p = [ x ]sinp x
è ø  2   2
(A) Equals 2 (B) Equals - 2 f(x) = [x] sin p x, that is, f is continuous for every real x.
1
(C) Equals (D)  Does not exist Hence, the correct answer is option (A).
2
[AIEEE 2011]
(1- cos 2 x )(3 + cos x )
Solution: We have 8.  lim is equal to
2
x ®0 x tan 4 x
2 sin ( x - 2) 2 | sin( x - 2)|
lim = lim 1 1
x ®2 x -2 x ®2 x -2 (A)  (B) 1 (C) 2 (D)  -
2 4
If x → 2-, then |sin(x - 2)| = -sin(x - 2);
[JEE MAIN 2013, 2015(OFFLINE)]
If x → 2+, then ⇒ |sin(x - 2)| = sin(x - 2)
Solution: We have
Therefore, RHL ≠ LHL
Thus, it is concluded that the limit does not exist. (1- cos 2 x )(3 + cos x )
lim
Hence, the correct answer is option (D). x →0 x tan 4 x
2 sin2 x (3 + cos x ) x 2(3 + 1)
6.  The value of p and q for which the function = lim = =2
x →0 x 2 tan 4 x × 4 x 1× 4
ì sin( p + 1) x + sin x 4x
ï , x <0
ï x Hence, the correct answer is option (C).
ï
f (x) = í q, x = 0 is continuous for all x in R is
ï sin(p cos2 x )
ï x + x2 - x 9.  lim is equal to
, x >0 x ®0 x2
ïî x 3/2
p
5 1 3 1 (A)  -π (B)  π (C)  (D) 1
(A)  p = , q = (B)  p = - , q = 2
2 2 2 2
[JEE MAIN 2014 (OFFLINE)]
1 3 1 3
(C)  p = , q = (D)  p = , q = - Solution:
2 2 2 2
sin(p cos2 x ) sin[p (1- sin2 x )] sin(p - p sin2 x )
[AIEEE 2011] lim 2
= lim 2
= lim
Solution: x ®0 x x ®0 x x ®0 x2

sin( p + 1) x + sin x sin(p sin2 x ) sin(p sin2 x ) p sin2 x


LHL = lim = lim ( p + 1)cos( p + 1) x + cos x = lim = lim × =1×π=π
x ®0 x x ®0 x →0 x2 x → 0 p sin2 x x2
= p + 1+ 1 = p + 2 Hence, the correct answer is option (B).

Mathematical Problem Book for JEE.indb 815 07-06-2018 13:03:08


816 Mathematics Problem Book for JEE

æ9ö 2 æ 1- cos 3 x ö LHD = -2


If f(x) is continuous and f ç ÷ = , then lim f ç
10.  ÷ is
2
è ø 9 x ® 0 è x2 ø RHD = 2
equal to Therefore, h ′(x) is continuous at x = 0 but is not differentiable at x = 0.
(A) 9/2 (B) 2/9 (C) 0 (D) 8/9 Hence, the correct answer is option (C).
[JEE MAIN 2014 (ONLINE SET-1)] 13.  Let f, g : R → R be two functions defined by
Solution:   1
 x sin   , x ≠0
ì 2 3x ü f (x) =   x and g( x ) = xf ( x )
æ 1- cos 3 x ö ï 2 sin 2 ï  0 , x =0
lim f ç ÷ = xlim fí ý 
x ®0 è x2 ø ®0 ï x
2
ï
î þ Statement I: f is a continuous function at x = 0.
Statement II: g is a differentiable function at x = 0.
é 2 sin2 (3 x / 2) ù ìï é sin(3 x / 2) ù 9 üï
2
ú = lim f í ê (A)  Both statements I and II are false.
⇒ xlim fê 2 ú ´ ý
® 0 ê (9 x / 4 ) ´ ( 4 / 9) ú x ® 0
ë û ïî ë (3xx / 2) û 2 ïþ (B)  Both statements I and II are true.
(C)  Statement I is true, statement II is false.
9 2
⇒ f æç ö÷ = (D)  Statement I is false, statement II is true.
è2ø 9 [JEE MAIN 2014 (ONLINE SET-3)]
Hence, the correct answer is option (B). Solution:
tan( x - 2)[ x 2 + ( k - 2 ) x - 2k ] 1 1
11.  If lim = 5, then k is equal to f ( x ) = x sin , x ≠ 0 ; g( x ) = x 2 sin , x ≠ 0
x ®2x - 4x + 4 2 x x
(A) 0 (B) 1 (C) 2 (D) 3 f ( x ) = 0, x = 0; g( x ) = 0 , x=0
[JEE MAIN 2014 (ONLINE SET-2)]
lim f ( x ) = f (0 )
Solution: x ®0

tan( x - 2) x 2 + kx - 2 x - 2k 1 1
lim × =5 (0 + h)2 sin - g(0 ) h 2 sin
x ®2 ( x - 2) g ’( 0 ) = lim ( 0 + h) = lim h =0
( x - 2)
h® 0 0+h h® 0 h
tan( x - 2) x ( x - 2) + k ( x - 2) h® 0
Þ lim lim =5
x ®2 x - 2 x ®2 ( x - 2) Hence, the correct answer is option (B).

tan( x - 2) ( x - 2) ( x + k ) 14.  If the function


Þ lim lim =5 ⇒2+k=5⇒ k=3
x ®2 x - 2 x ®2 ( x - 2) ì 2 + cos x - 1 ü
ï ,x ¹pï
Hence, the correct answer is option (D). f ( x ) = í (p - x )2
ý
ïk , x = p ï
12.  Let f(x) = x|x|, g(x) = sin x and h(x)=(gof)(x). Then î þ
(A)  h(x) is not differentiable at x = 0. is continuous at x = π, then k equals
(B) h(x) is differentiable at x = 0, but h’(x) is not continuous 1 1
at x = 0. (A) 0 (B)  (C) 2 (D) 
2 4
(C) h’(x) is continuous at x = 0 but it is not differentiable at
[JEE MAIN 2014 (ONLINE SET-4)]
x = 0.
Solution:
(D)  h’(x) is differentiable at x = 0.
[JEE MAIN 2014 (ONLINE SET-2)] 2 + cos x - 1 2 + cos(p + h) - 1 2 - cos h -1
lim = lim = lim
Solution: x →p (p - x )2 h → 0 (p - p - h)2 h → 0 h2
f (x) = x2, x>0
= 0, x=0 h
2 sin2
= -x2, x<0 (2 - cos h - 1) 1- cos h 2
= lim = lim = lim
g (x) = sin x h ® 0 h2 ( 2 - cos h + 1) h ® 0 h2 ( 2 - cos h + 1) h ® 0 h2 ( 2 - cos h + 1)
h (x) = g (f(x)) = sin x2, x>0
= 0, x=0 h æ hö
2 sin2 ç sin 2 ÷
2 1
= -sin x2, x<0 = lim 2 = lim 2 ç ÷ ´ lim
Now h® 0 h h® 0
çç h ÷÷ h ® 0 4( 2 - cos h + 1)
´ 4( 2 - cos h + 1)
h′ (x) = 2 x cos x2, x>0 4 è 2 ø
= 0, x = 0 (since, LHD = RHD at 0) 1 1
= 2´ =
= -2 x cos x2, x<0 4( 2 ) 4
h″(x) = -4 x2 sin x2 + 2cos x2, x>0
= 0, x=0 1
Therefore, k =
= 4 x2 sin x2 - 2cos x2, x<0 4
At x = 0, Hence, the correct answer is option (D).

Mathematical Problem Book for JEE.indb 816 07-06-2018 13:03:27


Chapter 19 | Limit, Continuity and Differentiability 817

Let f : R → R be a function such that |f(x)| ≤ x2, for all x ∈ R. Then
15.  é x4 ù x4
at x = 0 is Þ f ( x) = 2 ê - x3 + x2 ú = - 2x3 + 2x2
(A)  Continuous but not differentiable êë 4 úû 2
(B)  Continuous as well as differentiable ⇒ f(2) = 8 - 16 + 8 = 0
(C)  Neither continuous nor differentiable Hence, the correct answer is option (B).
(D)  Differentiable but not continuous 2

[JEE MAIN 2014 (ONLINE SET-4)] e x - cos x


17.  lim is equal to
Solution:
x ®0 sin2 x
3 5
|f (x)| ≤ x2 (A) 3 (B)  (C)  (D) 2
When x = 0 2 4
|f (0)|≤ 0 ⇒|f (0)| cannot be negative. [JEE MAIN 2015 (ONLINE SET-1)]
Now, Solution:
f ( 0 + h) - f ( 0 ) f ( h)
f ¢(0 ) = lim = lim 2
e x - cos x æ 0 ö 2 xe x + sin x æ 0
2

ö
h®0 h h®0 h lim ç form ÷ = lim ç form

2
x ®0 sin x è 0 ø x ®0 sin 2 x è0 ø
Now,
f ( h)
2 2

f (h) £ h2 = h Þ
2
£h 2 x (2 xe x ) + 2e x + cos x 3
= lim = (Using L’Hospital’s rule)
h x ®0 2 cos 2 x 2
f ( h) Hence, the correct answer is option (B).
Þ lim - h £ lim £ lim h
h® 0 h® 0 h h® 0
18.  For x ∈ R f(x) = |log 2 - sinx| and g(x) = f [f (x)], then
f ( h)
Therefore, by Squeeze principle lim = 0 , that is, f ¢(0 ) = 0. (A)  g is differentiable at x = 0 and g ′(0) = -sin(log 2).
h®0 h
(B)  g is not differentiable at x = 0.
Thus, f (x) is differentiable at 0 and hence continuous. (C)  g ′(0) = cos(log 2).
Hence, the correct answer is option (B). (D)  g ′(0) = -cos(log 2).
[JEE MAIN 2016 (OFFLINE)]
16. 
Let f (x) be a polynomial of degree four having extreme values
Solution: We have
é f (x)ù f(x) = |log 2 - sinx|
at x = 1 and x = 2. If lim ê1+ 2 ú = 3, then f(2) is equal to
x ®0 ë x û g(x) = f(f(x))
(A)  -4 (B) 0 (C) 4 (D)  -8 Therefore,
g(x) = |log 2 - sin f(x)|
[JEE MAIN 2015 (OFFLINE)]
f(x) = log 2 - sinx (log 2 - sin x) > 0 in neighbourhood of x = 0
Solution: That is,
f ¢( x ) = a( x - 1)( x - 2)( x - k ) g(x) = |log 2 - sin(log 2 - sin x)|
Þ f ¢( x ) = a[ x 3 - (3 + k ) x 2 + (2 + 3k ) x - 2k ] = log 2 - sin(log 2 - sin x) [g(x) is constant function at
x = 0]
é x 4 (3 + k ) x 3 (2 + 3k ) x 2 ù Therefore,
Þ f (x) = a ê - + - 2kx ú + C
êë 4 3 2 úû g  ′(x) = -cos(log 2 - sin x)(-cos x)
   = cos x · cos(log 2 - sin x)
f (x) é x 2 (3 + k ) x (2 + 3k ) 2k ù C Substituting x = 0, we get
Þ = aê - + - ú+ 2
x 2
êë 4 3 2 x úû x g  ′(0) = cos(log 2)
Hence, the correct answer is option (C).
æ f (x) ö ïì é (2 + 3k ) x 2 - 4 kx + 2C ù üï 1/ n
lim ç 1+ 2 ÷ = lim í1+ a ê ú ý = 3 (given) æ (n + 1)(n + 2)3n ö
x ®0 è x ø x ®0 ïî êë 2x2 úû ïþ 19.  lim ç ÷ is equal to
n ®¥ è n2n ø
ïì [a(2 + 3k ) + 2] x - 4 kax + 2Ca ïü
2
18 27 9
Þ lim í ý=3 (A)  3 log 3 - 2 (B)  (C)  2 (D)  2
x ®0 ï
î 2x2 þï e4 e e
⇒ Ca = 0  (1) [JEE MAIN 2016 (OFFLINE)]
Solution: We have
ì [a(2 + 3k ) + 2](2 x ) - 4 ka ü 1/ n
and lim í ý=3 æ (n + 1)(n + 2)(n + 2n) ö
x ®0 î 4x þ y = lim ç ÷

n ®¥ è n2n ø
⇒ ka = 0
1 é æ n + 1ö æ n+2ö æ n + 2n ö ù
ì [(2 + 3k ) + 2](2) ü Þ ln y = lim êln ç n ÷ + ln ç n ÷ +  + ln ç n ÷ ú
and lim í ý = 3 ⇒ a(2 + 3k) + 2 = 6 n ®¥ n ë è ø è ø è øû
x ®0 î 4 þ
⇒ 2a + 3ak = 4 ⇒ a = 2 (As ka = 0) 1é æ 1ö æ 2ö æ 2n ö ù
= lim êln ç 1+ n ÷ + ln ç 1+ n ÷ +  + ln ç 1+ n ÷ ú
⇒ C = 0 and k = 0 n ®¥ n ë è ø è ø è øû

Mathematical Problem Book for JEE.indb 817 07-06-2018 13:03:50


818 Mathematics Problem Book for JEE

1 2n æ r ö p +2 p -2 -p - 2
= lim
n ®¥ n
å ln çè1+ n ÷ø (A) 
2
(B) 
2
(C) 
2
(D)  -1 - cos-1(2)
r =1
2 [JEE MAIN 2016 (ONLINE SET-1)]
2
= ò ln(1+ x )dx = ( x + 1)ln( x + 1) - ( x + 1) 0 Solution: We have
0 f(1+) = f(1) = f(1-) and f ′(1-) = f ′(1+)
= 3(ln 3) - 3 + 1 = 3(ln 3) - 2 = ln 27 - ln e2 1
-1 = - Þ 1- (1+ b )2 = 1 ⇒ (1 + b) = 0 ⇒ b = -1
2
æ 27 ö 27 1- (1+ b )
ln y = ln ç 2 ÷ Þ y = 2
èe ø e p
-1 = a + cos -1(0 ) = a +
Hence, the correct answer is option (C). 2
20. 
If f(x) is a differentiable function in the interval (0, ∞) such that p æp +2ö
a = -1- = -ç ÷
t 2 f ( x ) - x 2f (t ) 2 è 2 ø
f(1) = 1 and lim = 1, for each x > 0, then f(3/2) is
t®x t-x Therefore,
equal to
23 13 25 31 a æp +2ö p
(A)  (B)  (C)  (D)  =ç ÷ = 1+
18 6 9 18 b è 2 ø 2
[JEE MAIN 2016 (ONLINE SET-1)] Hence, the correct answer is option (A).
Solution: We have æ a 4 ö
2x
22.  If lim ç 1+ - 2 ÷ = e 3 , then a is equal to
t 2 f ( x ) - x 2 f (t ) x ®¥ è x x ø
lim =1
t ® x t-x 3 1 2
(A) 2 (B)  (C)  (D) 
Applying L’Hospital’s rule, we get 2 2 3
[JEE MAIN 2016 (ONLINE SET-1)]
2t f ( x ) - x 2f ¢(t )
lim =1 Solution: We have
t ® x 1 2x
æa 4 ö
lim ç - 2 ÷
2 x f ( x ) - x 2f ¢( x ) = 1 e èx x ø
x ®¥
= e3

x 2f ¢( x ) - 2 x f ( x ) + 1 = 0 æ 8ö
Þ lim ç 2a - ÷ = 3
Therefore, x ®¥ è xø
2 1 3
f ¢( x ) - f ( x ) = - 2 ⇒ 2a = 3 ⇒ a =
x x 2
Integrating factor is Hence, the correct answer is option (B).
2
- ò dx 1 23.  Let a, b Î , (a ≠ 0). If the function f defined as
e x = e -2 ln x =
x2
ì 2x2
æ 1 ö 1 x -4 +1 æ1 1 ö ï , 0 £ x <1
ç f ( x )× 2 ÷ = - ò 4 + C = - +C =ç ´ 3 ÷+C ï a
è x ø x -4 + 1 è3 x ø ï
f (x) = í a, 1£ x < 2
ï 2
Therefore, ï 2b - 4b , 2 £ x <¥
ïî x 3
1
f (x) = + Cx 2
3x is continuous in the interval [0, ∞), then an ordered pair (a, b) is
1 2 (A)  ( - 2 ,1- 3 ) (B)  ( 2 , -1+ 3 )
f (1) = 1Þ 1 = + C Þ C =
3 3 (C)  ( 2 ,1- 3 ) (D)  ( - 2 ,1+ 3 )
1æ 1 ö [JEE MAIN 2016 (ONLINE SET-2)]
f ( x) = ç + 2x2 ÷
3è x ø Solution: We have
æ 3 ö 1 é 2 æ 9 ö ù 31 ì 2x2
Þ f ç ÷ = ê + ç 2 ´ ÷ú = ï , 0 £ x <1
è 2 ø 3 ë 3 è 4 ø û 18 ï a
ï
Hence, the correct answer is option (D). f (x) = í a, 1£ x < 2
ï 2
21.  If the function ï 2b - 4b , 2 £ x <¥
ïì- x , x <1 ïî x 3
f (x) = í -1
îïa + cos ( x + b ), 1 £ x £ 2 Now,
is differentiable at x = 1, then a/b is equal to f(1-) = f(1) = f(1+)

Mathematical Problem Book for JEE.indb 818 07-06-2018 13:04:23


Chapter 19 | Limit, Continuity and Differentiability 819

2(1- h)2 2 1 2x2 -1 4 x 2 -1 2 1


f (1) = lim = (A)  +   (B)  +   (C)  + 2    (D) 
h®0 a a 3x 3 3 x 3 x x x
f(1) = f(1+) = a [IIT-JEE 2007]
That is, Solution: We have
2
= a Þ a2 = 2 Þ a = - 2 , 2 t 2 f ( x ) - x 2 f (t )
a lim =1
- + t®x t-x
f( 2 ) = f( 2) = f( 2 )
2t f ( x ) - x 2f ′(t )
+ 2b2 - 4b 2b(b - 2) b(b - 2) ⇒ lim =1
f ( 2 ) = f ( 2 ) = lim = = t→x 1- 0
h ® 0 ( 2 + h)3 2 2 2
- Þ 2 x f ( x ) - x 2f ¢( x ) = 1
f( 2 ) = a
x 2f ¢( x ) - 2 x f ( x ) -1
Therefore, Þ = 4
b(b - 2) x4 x
= a Þ b2 - 2b = 2a
2 æ f ( x ) ö -1
Þ d ç 2 ÷ = 4 (1)
• When a = 2, we get è x ø x
b2 - 2b = 2 On integrating both sides of this equation, we get
b2 - 2b - 2 = 0
f (x) 1
2± 4+8 = +c
b= x2 3x3
2
Now, since
b = 1± 3 2
f (1) = 1Þ c =
• When a = - 2 , we get 3
b2 - 2b = - 2
we get
b2 - 2b +2 = 0
(b - 1)2 + 1 = 0  (Not possible) 1 2 2
f (x) = + x
3x 3
Therefore, the possible ordered pair is ( 2 ,1+ 3 ), ( 2 ,1- 3 ).
Hence, the correct answer is option (A).
Hence, the correct answer is option (C).
2. In the following [x] denotes the greatest integer less than or
(1- cos 2 x )2
24.  lim is equal to x.
x ® 0 2 x tan x - x tan 2 x

1 1 Column I Column II
(A) 2 (B)  - (C)  -2 (D) 
2 2 (A) x|x| (P) continuous in (-1, 1)
[JEE MAIN 2016 (ONLINE SET-2)] (B) (Q) differentiable in (-1, 1)
|x|
Solution: We have
2
æ 1- cos 2 x ö (C) x + [x] (R) strictly increasing in (-1, 1)
( 2 x )4 ç ÷
lim è 4 x2 ø (D) |x - 1| + |x + 1| (S) not differentiable at least at
x ®0 æ tan 2 x ö one point in (-1, 1)
2 x ç tan x - ÷
è 2 ø
[IIT-JEE 2007]
æ 4 1ö
ç 16 ´ x ´ ÷ Solution:
è 4ø
Þ lim (A) →(P), (Q), (R)
x ®0 æ tan x ö
2 x ç tan x - ÷ f(x) = x| x |
è 1- tan2 x ø
2 x 3 (1- tan2 x )
Þ lim = -2
x ® 0 (tan x - tan3 x - tan x )

Hence, the correct answer is option (C).

Previous Years’ Solved JEE Advanced/


IIT-JEE Questions
1. Let f(x) be differentiable on the interval (0, ∞) such that f(1) = 1, Figure 19.17
and (B) →(P), (S)
t 2 f ( x ) - x 2 f (t )
lim = 1 for each x > 0. Then f(x) is f (x) = x
t→x t-x

Mathematical Problem Book for JEE.indb 819 07-06-2018 13:04:50


820 Mathematics Problem Book for JEE

Statement-1:
f ′(t) = -sin t
f ′(p + t) = sin t
Also, since
f ′(t) · f ′(p + t) = -sin2 t
which is negative, the equation f ′(t) = 0 has at least one solution
in [t, t + π].
Statement-2:
Figure 19.18 Since f(x) = 2 + cos x is a periodic function with period 2π, we get
(C)→(R), (S) f (2p + t) = f (t)
f (x) = x +[x] Hence, the correct answer is option (B).
 x - 1, -1< x < 0 4. Let f(x) be a non-constant twice differentiable function defined
=
 x, 0 ≤ x <1 æ 1ö
on (-∞, ∞) such that f ( x ) = f (1- x ) and f ¢ ç ÷ = 0 . Then
è4ø
(A)  f ′′(x) vanishes at least twice on [0, 1]
æ 1ö
(B)  f ¢ ç ÷ = 0
è2ø
1/ 2
 1
−1 1 (C)  ∫ f  x +  sin x dx = 0
 2
-1/ 2
1/ 2 1
−2 sin p t sin p t
(D)  ò f (t )e dt = ò f (1- t )e dt
0 1/ 2

[IIT-JEE 2008]
Solution: We have
Figure 19.19 f(x) = f(1 - x)(1)
(D)→(P), (Q) 1
Put x = + x . Then
f(x) = | x - 1| + | x + 1| 2
æ1 ö æ1 ö
f ç + x÷=f ç -x÷
è2 ø è2 ø
1  æ1 ö
So, f  + x  is an even function and sin x × f ç + x ÷ is an odd
2  è2 ø
function. Therefore,
1
2
−1 1  1
∫ f  x + 2  sin x dx = 0
-1
2
Figure 19.20 Differentiate equation (1), we get
Hence, the correct matches are (A)ã(P, Q, R); (B)ã(P, S); f ′(x) = -f ′(1 - x) (2)
(C)ã(R, S); (D)ã(P, Q). 1
Put x = , we get
3. Let f(x) = 2 + cos x for all real x. 2
Statement-1: For each real t, there exists a point c in [t, t + π] æ 1ö æ 1ö
f ¢ ç ÷ = -f ¢ ç ÷
such that f ′(c) = 0. Because è2ø è2ø
Statement-2: f(t) = f(t + 2π) for each real t. æ 1ö
(A) Statement-1 is True, Statement-2 is True; Statement-2 is a Þ f ¢ç ÷ = 0
è2ø
correct explanation for Statement-1
(B) Statement-1 is True, Statement-2 is True, Statement-2 is 1
Now, put x = in Eq. (2)
NOT a correct explanation for Statement-1 4
(C)  Statement-1 is True, Statement-2 is False æ 1ö æ3ö
(D)  Statement-1 is False, Statement-2 is True f ¢ ç ÷ = -f ¢ ç ÷ = 0
4
è ø è4ø
[IIT-JEE 2007]
Solution: We have f(x) = 2 + cos x for all values of x. æ 1ö æ 1ö æ3ö
Þ f ¢ç ÷ = f ¢ç ÷ = f ¢ç ÷
f ′(x) = -sin x è4ø è2ø è4ø

Mathematical Problem Book for JEE.indb 820 07-06-2018 13:05:03


Chapter 19 | Limit, Continuity and Differentiability 821

æ 1 1ö Solution:
Using Rolle’s theorem, f ″(x) = 0 has at least one solution in ç , ÷
è4 2ø x2
 1 3 a - a2 - x 2 -
and also in  ,  . 4 = lim 1 1
 2 4 L = lim 4
-
2
x ®0 x x ®0
x (a + a - x ) 4 x
2 2 2
So, f ′′(x) = 0 vanishes at least twice on [0, 1]. Now,
( 4 - a ) - a2 - x 2
1 = lim
2
sin p t
x ®0
4 x 2 ( a + a2 - x 2 )
ò f (t )e dt
0 1
Numerator → 0 if a = 2, then L = .
Put t = 1 - x. Then 64
1
Hence, the correct answers are options (A) and (C).
2
sin p (1- x )
ò f (1- x ) × e × ( -dx ) 7. Let f be a real-valued function defined on the interval (0, ∞) by
1 x
1 f ( x ) = ln x + ò 1+ sin t dt . Then which of the following state-
= ò f (1- x )e sinp x × dx 0
1 ment(s) is (are) true?
2
(A)  f ″(x) exists for all x ∈ (0, ∞)
Hence, the correct answers are options (A), (B), (C)  and (D). (B) f ′(x) exists for all x ∈ (0, ∞) and f ′ is continuous on (0, ∞), but
not differentiable on (0, ∞)
5. Let f and g be real valued functions defined on interval (-1, 1)
(C)  There exists α > 1 such that |f ′(x)| < |f (x)| for all x ∈ (a , ∞)
such that g″(x) is continuous, g(0) ≠ 0, g′(0) = 0, g″(0) ≠ 0, and
f(x) = g(x) sin x. (D)  There exists β > 0 such that |f(x)| + |f ′(x)| ≤ b for all x ∈ (0, ∞)
[IIT-JEE 2010]
Statement-1: lim[ g( x )cot x - g(0 )cosec x ] = f ¢¢(0 )
x ®0 Solution:
Statement-2: f ′(0) = g(0) 1
f ¢( x ) = + 1+ sin x
(A) Statement-1 is True, Statement-2 is True; Statement-2 is a x
correct explanation for Statement-1 p
f ′(x) is not differentiable at sin x = -1 or x = 2np - , n ÎN .
(B) Statement-1 is True, Statement-2 is True; Statement-2 is 2
not a correct explanation for Statement-1 In x ∈ (1, ∞)  f (x) > 0, f ′(x) > 0
(C)  Statement-1 is True, Statement-2 is False Consider,
(D)  Statement-1 is False, Statement-2 is True x
1
[IIT-JEE 2008] f (x ) f = ln x + ∫ 1+ sin t dt - - 1+ sin x
Solution: We have 0
x
f(x) = g(x) sin x x  1
=  ∫ 1+ sin t dt - 1+ sin x  + ln x -
f ¢( x ) = g( x )cos x + g¢( x ) × sin x 0  x
Þ f ¢(0 ) = g(0 ) x
f ¢¢( x ) = 2g¢( x )cos x - g( x ) sin x + sin x × g¢¢( x ) Consider, g( x ) = ò 1+ sin t dt - 1+ sin x
Þ f ¢¢(0 ) = 2g¢(0 ) = 0 0

Now, It can be proved that g( x ) ³ 2 2 - 10 " x Î (0 , ¥ )


lim[ g( x )cot x - g(0 )cosec x ] 1
x ®0 Now there exists some a > 1 such that - ln x £ 2 2 - 10 for all
x
g( x ) × cos x - g(0 ) æ 0 ö 1
= lim ç form ÷ x ∈ (a, ∞) as - ln x is strictly decreasing function.
x ®0 sin x è0 ø x
g¢( x )cos x - g( x ) sin x 1
= lim So, g( x ) ³ - ln x .
x ®0 cos x x
= g¢(0 ) = f ¢¢(0 )
Hence, the correct answers are options (B) and (C).
Hence, the correct answer is option (B).
8. Let f : R → R be a function such that f (x + y) = f (x) + f (y), ∀x, y
x2
2 2
∈ R. If f(x) is differentiable at x = 0, then
a- a - x -
6. Let L = lim 4 , a > 0. If L is finite, then (A)  f(x) is differentiable only in a finite interval containing zero
x ®0 x4 (B)  f(x) is continuous ∀x ∈ R
1 1 (C)  f ′(x) is constant ∀x ∈ R
(A)  a = 2 (B)  a = 1 (C)  L = (D)  L =
64 32 (D)  f(x) is differentiable except at finitely many points
[IIT-JEE 2009] [IIT-JEE 2011]

Mathematical Problem Book for JEE.indb 821 07-06-2018 13:05:25


822 Mathematics Problem Book for JEE

Solution: Since f (0) = 0 and b-x


11. Let f : (0 ,1) ®  be defined by f ( x ) = , where b is a
f ( x + h) - f ( x ) 1- bx
f ¢( x ) = lim
h®0 h constant such that 0 < b < 1. Then
f ( h) (A)  f is not invertible on (0, 1)
= lim = f ¢(0 ) = k (say ) 1
h®0 h (B)  f ≠ f -1 on (0, 1) and f ¢(b ) =
f ¢(0 )
Þ f ( x ) = kx + cÞÞf (fx( x) =) =kxkx+(cAsÞf (f0( )x=) =0 ).kx (As f (0 ) = 0 ).
1
(C)  f = f -1 on (0, 1) and f ¢(b ) =
Therefore, f(x) is continuous for all x ∈ R and f ′(x) = k, that is, con- f ¢(0 )
stant for all x ∈ R. (D)  f -1 is differentiable on (0, 1)
Hence, the correct answers are options (B) and (C). [IIT-JEE 2011]
Solution:
9.  If lim[1+ x ln(1+ b2 )]1/ x = 2b sin2 q , b > 0 and q ∈ (-p,p], then b-x
x ®0 f (x) =
the value of θ is 1- bx
p p p p b-x b-y
(A)  ± (B)  ± (C)  ± (D)  ± Let y = . Then x =
4 3 6 2 1- bx 1- by
([IIT-JEE 2011] b-x b-y
Solution: y= . Then x =
1- bx 1- by
2
eln(1+ b ) = 2b sin2 q b-y
0 < x < 1Þ 0 < <1
1+ b 2 1- by
⇒ sin2 q =
2b b-y 1
> 0 Þ y < b or y >
2 1+ b 2 1- by b
⇒ sin q = 1 as ≥1
2b b-y 1
- 1 > 0 Þ -1 < y <
q = ±p / 2 1- by b

Hence, the correct answer is option (D). ⇒ -1 < y < b
ì p p Hence, the correct answer is option (A).
ï- x - 2 , x£-
2
ï æ x2 + x +1 ö
ï - cos x - p < x £ 0 , 12.  If lim çç - ax - b ÷÷ = 4 , then
10. If f ( x ) = í then x ®¥
è x + 1 ø
ï 2
ï x -1 0 < x £1 (A)  a = 1, b = 4 (B)  a = 1, b = -4
ï ln x x >1 (C)  a = 2, b = -3 (D)  a = 2, b = 3
î
[IIT-JEE 2012]
(A)  f(x) is continuous at x = -π/2 Solution: Given
(B)  f(x) is not differentiable at x = 0
(C)  f(x) is differentiable at x = 1 æ x2 + x +1 ö
lim çç - ax - b ÷÷ = 4
(D)  f(x) is differentiable at x = -3/2 x ®¥
è x + 1 ø
[IIT-JEE 2011]
Solution: x 2 + x + 1- ax 2 - ax - bx - b
⇒ lim =4
x →∞ ( x + 1)
lim - f ( x ) = 0 = f ( -p / 2) (1- a) x 2 + (1- a - b ) x + (1-
- b)
p
x→ - ⇒ lim =4
2 x →∞ ( x + 1)
 p ⇒ 1- a = 0 and 1- a - b = 4 ⇒ b = -4 , a = 1
lim + f ( x ) = cos  -  = 0
x→ -
p  2
2 Hence, the correct answer is option (B).

ì -1, x £ -p / 2  2 p
ïsin x , -p / 2 < x £ 0  x cos x , x≠0
ï 
f ¢( x ) = í
ï 1, 0 < x £1 13.  Let f ( x ) =  x ∈ . Then f  is
ïî 1/ x , x >1  0, x=0


Clearly, f (x) is not differentiable at x = 0 as f ′(0-) = 0 and f ′(0+) = 1.
f (x) is differentiable at x = 1 as f ′(1-) = f ′(1+) = 1. (A)  Differentiable both at x = 0 and at x = 2
Hence, the correct answers are options (A), (B), (C) and (D). (B)  Differentiable at x = 0 but not differentiable at x = 2

Mathematical Problem Book for JEE.indb 822 07-06-2018 13:05:46


Chapter 19 | Limit, Continuity and Differentiability 823

(C)  Not differentiable at x = 0 but differentiable at x = 2 (A)  an - 1 - bn - 1 = 0 (B)  an - bn = 1


(D)  Differentiable neither at x = 0 nor at x = 2 (C)  an - bn + 1 = 1 (D)  an - 1 - bn = -1
[IIT-JEE 2012] [IIT-JEE 2012]
Solution: Solution: At x = 2n
f ( 0 + h) - f ( 0 ) L.H.L = lim[bn + cos p (2n - h)] = bn + 1
f ¢(0 ) = lim h®0
h®0 h R.H.L = lim[an + sinp (2n + h)] = an
p h®0
h2 cos - 0
h f(2n) = an
= lim
h®0 h For continuity bn + 1 = an
æp ö
= lim h cos ç ÷ = 0 At x = 2n + 1
h®0 èhø
L.H.L = lim [an + sinp (2n + 1- h)] = an
h®0
so, f(x) is differentiable at x = 0
R.H.L = lim {bn +1 + cos[p (2n + 1- h)]} = bn +1 - 1
f (2 + h) - f (2 ) h®0
f ¢(2+ ) = lim
h®0 h f(2n + 1) = an
p For continuity
(2 + h)2 cos -0
2+h an = bn + 1 - 1
= lim
h®0 h an - 1 - bn = -1
2æ p ö Hence, the correct answers are options (B) and (D).
(2 + h) cos ç ÷
= lim è2+hø
15. a Î (the set of all real numbers), a ≠ -1,
h®0 h
(1a + 2a +  + na ) 1
+ (2 + h)2 æ p p ö lim =
f ¢(2 ) = lim sin ç - ÷ n ®¥ ( n + 1)a -1[( na + 1) + ( na + 2 ) +  + ( na + n)] 60
h®0 h è 2 2 + hø
then a = ?
(2 + h)2 é p ×h ù
= lim sin ê ú -15 -17
h®0 h ë 2(2 + h) û (A) 5 (B) 7 (C)  (D) 
2 2
(2 + h)2 sin(p h 2(2 + h)) ph
= lim ´ =p [JEE ADVANCED 2013]
h®0 h (p h 2(2 + h)) 2(2 + h) Solution: We have
n
-
Again, f ¢(2 ) = lim
f (2 - h) - f (2 ) å (r )a 1
r =1
h®0 -h lim =
n ®¥ é n ù 60
æ p ö (n + 1)a -1 ê å (na + r )ú
2
(2 - h) cos ç ÷ ë r =1 û
è 2-hø
= lim That is,
h®0 -h a
a n
æ p ö r r
2
-(2 - h) cos ç ÷ na ∑  
 n 1 1
∑  n  1
= lim è 2-hø lim = ⇒ lim r =1
=
r  60 n→∞  1  a -1 n
h®0 -h n→∞   r  60
n(n + 1)a -1∑  a +   1+  ∑  a + n 
2 ép p ù  n n
(2 - h) sin ê - r =1
ë 2 2 - h úû 1
= lim
∫x
a
h®0 h dx
1
(2 - h)2 é -p h ù = 0
=
= lim × sin ê ú
1
60
h®0 h ë 2(2 - h) û ∫ (a + x ) dx
(2 - h)2 ph p 0
= - lim × sin ´ = -p 1
h®0 ph 2(2 - h) 2(2 - h) x a+1
0 1
2(2 - h) =
 1 60
Hence, the correct answer is option (B). x2 
(a + 1)  ax +  
 2 
14.  For every integer n, let an and bn be real numbers. Let function  0

f :  ®  be given by 2 1
=
ì a + sinp x , for x Î [2n, 2n + 1] (2a + 1)(a + 1) 60
f (x) = í n , for all integers n. If f is
îbn + cos p x , for x Î(2n - 1, 2n) 2a2 + 3a + 1 = 120
continuous, then which of the following hold(s) for all n? 2a2 + 3a - 119 = 0

Mathematical Problem Book for JEE.indb 823 07-06-2018 13:06:02


824 Mathematics Problem Book for JEE

Therefore, Therefore,
-3 ± 9 + 8(119 ) -3 ± 961 -3 ± 31 h(c) = 0
a= = =
4 4 4 ⇒ f(c) - g(c) = 0
17 ⇒ f(c) = g(c)
Thus, a = 7, - .
2 Hence, the correct answers are options (A) and (D).
Hence, the correct answers are options (B) and (D).
17. Let f : [a, b] → [1, ∞) be a continuous function and let g :  ® 
16. For every pair of continuous function f, g : [0 ,1] ®  such that be defined as
max {f(x) : x∈[0,1]} = max {g(x) : x∈ [0,1]}, the correct state-
ment(s) is (are): ì
ï0 if x < a,
(A) [f(c)]2 + 3f(c)  = [g(c)]2 + 3g(c)  for some c ∈ [0, 1] ï x
(B)  [f(c)]2 + f(c)  = [g(c)]2 + 3g(c)  for some c ∈ [0, 1] g( x ) = í ò f (t )dt if a £ x £ b , then
a
ï b
(C) [f(c)]2 + 3f(c)  = [g(c)]2 + g(c)  for some c ∈ [0, 1] ï f (t )dt
î òa
if x > b
(D)  [f(c)]2 = [g(c)]2 for some c∈ [0, 1]
[JEE ADVANCED 2014] (A)  g(x) is continuous but not differentiable at a
Solution: Suppose f(x) is maximum at c1 and g(x) is maximum at c2. (B)  g(x) is differentiable on 
When f(x) is maximum g(x) may or may not be maximum (Fig. 19.21). (C)  g(x) is continuous but not differentiable at b
(D) g(x) is continuous and differentiable at either a or b but
not both
f (x) [JEE ADVANCED 2014]
Solution: Checking continuity of g (Fig. 19.24).
g(x)

c1 c2 f

Figure 19.21
Therefore, in the function h(x) = f(x) - g(x), we get
h(c1) = f(c1) - g(c1) ≥ 0 and h(c2) = f(c2) - g(c2) ≤ 0
Hence, h(x) = 0 for some c ∈ [0, 1]. (See Figs. 19.22 and 19.23.) 1

f (x) b
a
g(x)
Figure 19.24

lim g(a - h) = 0
h→ 0
a+h a
lim g(a + h) = ∫ lim f (t )dt = ∫ f (t )dt
h→ 0 a h→ 0 a

a
g(a) = ò f (t )dt = 0
c1 a
c2
Hence, g is continuous at a.
Figure 19.22 Similarly, g is continuous at b. As

b
g (b - h) = g(b ) = g (b + h) = ò f (t )dt
h®0 h®0 a

f (x) Now
g(x) ì 0, x <a
ï
g¢( x ) = íf ( x ), a < x < b
ï 0, x >b
î
Since f(x) ≥ 1 in [a, b] given, so as we cross a and b according to
c1 c2
g(x) function, there are sharp edges encountered due to abrupt
change in the slopes from 0 to k and then from k to 0, where k ≥ 1
Figure 19.23 (Fig. 19.25).

Mathematical Problem Book for JEE.indb 824 07-06-2018 13:06:11


Chapter 19 | Limit, Continuity and Differentiability 825

Solution: Since,
x2
F ( x ) = ò f ( t ) dt
0

Slope k > 1 Hence,


d 2 d
g(x) f ¢( x ) = f ( x 2 ) (x )- f( 0) 0
dx dx
Slope 0
(By Newton-Leibnitz rule)

a b = 2x f ( x2 )
(1)
Now according to question
F ′( x ) = f ′( x ) ⇒ 2 x f ( x 2 ) = f ′( x )
Þ 2 xf ( x ) = f ¢(Þ xf ( x )Î=(0f ¢,(2x))) ( As x Î (0 , 2))
x )2( As

Sharp edge
Hence,
Figure 19.25 f ¢( x ) f ¢( x )
= 2x Þ ò dx = 2 ò xdx
g′(a+) = f (a + h) ≥ 1, etc. f (x) f (x)
Hence, the correct answers are options (A) and (C). 2 x2
Þ loge f ( x ) = +c
18. The largest value of the non-negative integer a for which 2
1- x 2 2
+c
ì -ax + sin( x - 1) + a ü1- x 1 Þ f (x) = ex = e x × ec
lim í ý = is _____.
x ®1î x + sin( x - 1) - 1 þ 4
By initial condition f(0) = 1
[JEE ADVANCED 2014]
Hence,
Solution: 2
1 = e o × e c ⇒ ec = 1
1- x 2
t )2
ì -ax + sin( x - 1) + a ü1- x 1 Þ f ( x ) = e x \ f ( t ) = e( = et
lim í ý =
x ®1 î x + sin( x - 1) - 1 þ 4 Therefore,
2
(1 + x ) t )2
 sin( x - 1) - a( x - 1)  1 Þ f ( x ) = e x \ f ( t ) = e( = et
⇒ lim   =
x →1 sin( x - 1) + ( x - 1)  4 Now
x2
F ( x ) = ∫ e t dt = [e t ]0x = e x - 1
2 2

1+ x
ì sin( x - 1) ü 0
ï x -1 - a ï 1 2
Þ lim í ý = ⇒ F (2 ) = e 2 - 1 = e 4 - 1
x ®1 sin( x - 1) 4
ï +1ï
î x -1 þ Hence, the correct answer is option (B).
1+ 1
ì1 - a ü 1 192 x 3 æ 1ö
Þí ý = 20. Let f ¢( x ) = for all x Î with f ç ÷ = 0. If
î 1+ 1 þ 4 2 + sin4 p x è2ø
2 1
æ 1- a ö 1
Þç
è 2 ø 4
÷ = m£ ò f ( x )dx £ M , then the possible values of m and M are
1/ 2
Hence, 1 1
1- a 1 (A)  m = 13, M = 24 (B)  m = ,M=
=± 4 2
2 2 (C)  m = -11, M = 0 (D)  m = 1, M = 12
⇒ 1 - a = ±1 ⇒ a = 0, 2
[JEE ADVANCED 2015]
Therefore, largest value = 2.
Solution:
Hence, the correct answer is (2).
192 x 3
19. Let f : [0, 2] →  be a function which is continuous on [0, 2] f ¢( x ) = "x Î  ,
2 + sin4p x
and is differentiable on (0, 2) with f (0) = 1.
x2 As
Let F(x) = ò f ( t ) dt for x∈[0, 2]. If F ′(x) = f ′(x) for all 1
0 ≤ x ≤1
x ∈ (0, 2), then F(2) equals 2

(A)  e2 - 1 (B)  e4 - 1 (C)  e - 1 (D)  e4 Therefore,

[JEE ADVANCED 2014] 8 ≤ f ′( x ) ≤ 96

Mathematical Problem Book for JEE.indb 825 07-06-2018 13:06:38


826 Mathematics Problem Book for JEE

x x x
ìï-3ax 2 - 2, x <1
⇒ ∫ 8 ≤ ∫ f ′( x ) ≤ ∫ 96 (B)  f ( x ) = í
2
12 12 12
îï bx + a , x >1
 1
⇒ 8 x - 4 ≤ f ( x ) - f   ≤ 96 x - 48 Therefore, f(x) is differentiable "x Î . So,
 2
1 1 1 f(1-) = f(1+)
⇒ ∫ 8 x - 4 ≤ ∫ f ( x ) ≤ ∫ 96 x - 48 ⇒ -3a - 2 = b + a2
12 12 12 ⇒ a2 + 3a + 2 = -b
1
⇒ (a + 2)(a + 1) = -b(1)
⇒ 1 ≤ ∫ f ( x ) ≤ 12
12 Also,
ì-6ax ; x <1
f ¢( x ) = í
Hence, the correct answer is option (D). î b; x >1
21. Match the Column I to Column II ⇒ f ′(1-) = f ′(1+)
⇒ -6a = b(2)
Column I Column II Hence, from Eqs. (1) and (2),
a2 + 3a + 2 = 6a
(A) In  3, if the magnitude of the projection (P) 1 ⇒ a = 1 or a = 2
vector of the vector a i + b j on 3i + j ⇒ (B) → (P), (Q)

is 3 and if a = 2 + 3b , then possible (C) (3 - 3ω + 2ω2)4x + 3 + (2 + 3ω - 3ω2)4x + 3 + (-3 + 2ω


value(s) of |α| is (are) + 3ω2)4x+3 = 0
⇒ [1 - 3ω + 2(1 + ω2)]4x + 3 + [2(1 + ω) + ω - 3ω2]4x + 3
(B) Let a and b be real numbers such that the (Q) 2
+ [-3 + ω2 + 2(ω + ω2)]4x + 3 = 0
function
⇒ [1 - 3ω - 2ω]4x+3 + [-5ω2 + ω]4x+3 + (ω2)4x + 3 (1 - 5ω)4x + 3 = 0
ïì-3ax - 2,
2
x <1 ⇒ (1 - 5ω)4x + 3 (1 + ωx + ω2x) = 0
f (x) = í
2
îï bx + a , x ³1 ω(1 - 5ω)4x + 3 ≠ 0 ⇒ 1 + ωx + ω2x = 0
is differentiable for all x Î . Then possible ⇒ x = 3k + 1 or x = 3k + 2; k ∈ z
value(s) of a is (are) ⇒ x ∈ {1, 2, 4, 5}
⇒ (C) → (P), (Q), (S), (T)
(C) Let ω ≠ 1 be a complex cube root of unity. (R) 3
If (3 - 3ω + 2ω2)4x+3 + (2 + 3ω - 3ω2)4x+3 2ab
(D) HM of ‘a’ and ‘b’ = = 4 , where a, b > 0.
+ (-3 + 2ω + 3ω2)4x+3 = 0. Then possible a+b
value(s) of x is (are) Now, a, 5, q, b are in AP, where q > 0. Therefore
a + b = 5 + q(3)
(D) Let the harmonic mean of two positive real (S) 4
numbers a and b be 4. If q is a positive real 5+b
Also a + q = 10 and q = (4)
number such that a, 5, q, b is an arithmetic 2
⇒ b = 2q - 5 (5)
progression, then the value(s) of |q - a| is (are)
Therefore, from Eqs. (3), (4) and (5),
(T) 5
5
a= or a = 6
[JEE ADVANCED 2015] 2
15
Solution: ⇒ q= or 4 ⇒ |q - a| = 5 or 2
2
 
(A)  Let a = a i + b j and b = 3 i + j . ⇒ (D) → (Q), (T)

  | a × b | Hence, the correct matches are (A) ã (P, Q); (B) ã (P, Q);
Therefore, magnitude of projection of a on b =  (C) ã (P, Q, S, T); (D) ã (Q, T)
|b |
22. Let m and n be two positive integers greater than 1.
| 3a + b |
= = 3 Þ æ e cos(a ) - e ö
n
3a + b = ±2 3 æeö m
3 +1 If lim ç ÷ = - ç ÷ , then the value of is _____.
a ®0 ç a m ÷ 2
è ø n
è ø
⇒ 3 (2 + 3 b ) + b = ±2 3 [JEE ADVANCED 2015]
⇒ β = 0 or b = - 3 ⇒ α = 2 or α = -1 Solution: m, n Î and m, n > 1
 e cos(a ) - e   e(e cos a -1 - 1) 
n n

⇒ |α| = 2 or 1 L = lim   = lim  


m
⇒ (A)  → (P), (Q)  a
a →0   a → 0  am 

Mathematical Problem Book for JEE.indb 826 07-06-2018 13:06:57


Chapter 19 | Limit, Continuity and Differentiability 827

 e(e cos a -1 - 1) (cos a n - 1) 


n
Paragraph for Questions 24 and 25: Let F :  ®  be a thrice
= lim  ⋅  dif­ferentiable function. Suppose that F(1) = 0, F(3) = -4 and F ′(x) < 0
a → 0  (cos a n - 1) am 
 for all x ∈ (1/2, 3). Let f(x) = xF(x) for all x ∈ R.
 cos a n - 1 [JEE ADVANCED 2015]
= e. lim  
a →0  a rn  24. The correct statement(s) is (are)
(A)  f ′(1) < 0
é - sina n × na n -1 ù (B)  f (2) < 0
= e × lim ê ú
ë m ×a
a ®0 ê m -1
úû (C)  f ′(x) ≠ 0 for any x ∈ (1, 3)
(D)  f ′(x) = 0 for some x ∈ (1, 3)
æ -n ö éæ sina n ö a 2n -1 ù
= e × ç ÷ × lim êçç ÷ ú Solution: F :  → , thrice differentiable,
è m ø a ®0 êëè a n ÷ø a m -1 úû F(1) = 0, F(3) = 4,
-n -e F ′(x) < 0 ∀∈ (1/2, 3),
=⋅ e lim (a 2n - m ) = (given) f(x) = xF(x) "x Î 
m a → 0 2
Since
m
⇒ alim a 2n - m = æ1 ö
®0 2n F ′(x) < 0 "x Î ç , 3 ÷
⇒ m = 2n è2 ø

⇒ m =2 1 
So, F(x) is a decreasing function on  , 3 . Therefore,
n 2
Hence, the correct answer is (2). F(1) > F(x) > F(3) ∀ 1 < x < 3
23. Let f, g: [-1, 2] →  be continuous functions which are twice ⇒ 0 > F(x) > -4 ∀ x ∈ (1, 3)
differentiable on the interval (-1, 2). Let the values of f and g at Hence, f ′( x ) =  x ) + F
′(
xF ( x ) < 0 ∀ x ∈(1, 3).
the points -1, 0 and 2 be as given in the following table: - ve - ve

e x = -1 x = 0 x = 2 Also


f(x) 3 6 0 f ′(1) = F ′(1) + F(1) = F ′(1) + 0
g(x) 0 1 -1 æ1 ö
⇒ f ′(1) = F  ′(1) < 0 as F ′(x) < 0 "x Î ç , 3 ÷
In each of the intervals (-1, 0) and (0, 2) the function (f - 3g)′′ è2 ø
never vanishes. Then the correct statement(s) is (are) Further f (2) = 2
F (2) < 0.
(A)  f ′(x) - 3g ′(x) = 0 has exactly three solutions in (-1, 0) ∪ (0, 2) - ve
(B)  f ′(x) - 3g ′(x) = 0 has exactly one solution in (-1, 0)
Hence, the correct answers are options (A), (B) and (C).
(C)  f ′(x) - 3g ′(x) = 0 has exactly one solution in (0, 2)
(D) f ′(x) - 3g ′(x) = 0 has exactly two solutions in (-1, 0) and 3 3

òx ò x F ¢¢( x )dx = 40, then the correct


2 3
exactly two solutions in (0, 2) 25. If F ¢( x )dx = -12 and
[JEE ADVANCED 2015] 1 1

Solution: f, g: [-1, 2] → , expression(s) is (are)


f(x) is twice differentiable on (-1, 2) 3

f(-1) = 3, g(-1) = 0, (A)  9f ¢(3) + f ¢(1) - 32 = 0 (B)  ò f ( x )dx = 12


f(0) = 6, g(0) = 1 1

f(2) = 0, g(2) = -1 3

(f - 3g)′′ ≠ 0 on (-1, 0) and (0, 2) (C)  9f ¢(3) - f ¢(1) + 32 = 0 (D)  ò f ( x )dx = -12
1
Number of solutions of f ′(x) - 3g ′(x) = 0 in (-1, 0) ∪ (0, 2) = ?
Let h(x) = f(x) - 3g(x). Then Solution:
h(-1) = f(-1) - 3g(-1) = 3, 3 3

òx F ¢( x )dx = -12 and ò x 3F ¢¢( x )dx = 40 ;


2
h(0) = f(0) - 3g(0) = 6 - 3(1) = 3
1 1
Therefore, by Rolle’s theorem, h ′(x) that is, f ′(x) - 3g ′(x) = 0 has at
3
least one root in (-1, 0).
9f ¢(3) + f ¢(1) ± 32 = ?, ò f ( x )dx = P
Also h(2) = f(2) - 3g(2) = 0 - 3(-1) = 3
1
Hence, again by Rolle’s theorem, f ′(x) - 3g ′(x) = 0 has at least one
root in (0, 2). Here,
That is, f ′(x) - 3g ′(x) = 0 has at least 2 roots in (-1, 2). 9f ¢(3) ± f ¢(1) = 9(3F ¢(3) + F (3) ± (F ¢(1) + F (1))
Since (f - 3g)′′ ≠ 0 for (-1, 0) and (0, 2) = 9[3F ¢(3) - 4 ] ± F ¢(1) = 27F ¢(3) ± F ¢(1) - 36 (1)
So, f(x) has no point of inflexion in (-1, 0) and (0, 2). Therefore,
Now
(f ′ - 3g ′)(x) ≠ 0 in (-1, 0) and (0, 2), that is, (f ′ - 3g ′)(x) ≠ 0 exactly 3 3
3
once in (-1, 0) and exactly once in (0, 2).
òx F ¢( x )dx = x 2F ( x ) - ò 2 xF ( x )dx
2
1
Hence, the correct answers are options (B) and (C). 1 1

Mathematical Problem Book for JEE.indb 827 07-06-2018 13:07:21


828 Mathematics Problem Book for JEE

3 Now,
= [9F (3) - F (1)] - 2 ò f ( x )dx  1 1 
lim x f   = lim+ x  + cx  = lim+ (1+ (cx 2 )) = 1
1 x →0+  x  x →0  x  x →0
3
Hence, option (B) is incorrect.
Þ -12 = -36 - 2 ò f ( x )dx
1
Now,
æ c ö
3
lim x 2 f ¢( x ) = lim+ x 2 ç 1- 2 ÷ = lim+ ( x 2 - c ) = -c ¹ 0
Þ ò f ( x )dx = -12 x ®0+ x ®0 è x ø x ®0
1 (2) Hence, option (C) is incorrect.
Also, We cannot say anything about f ( x ) £ 2 "x Î (0 , 2) because we do
3 3
3 not know the value of c.
ò x F ¢¢( x )dx = x F ¢( x ) 1 - ò 3 x
3 3 2
F ¢( x )dx
Hence, option (D) is incorrect.
1 1
Hence, the correct answer is option (A).
Þ 40 = 27F ¢(3) - F ¢(1) - 3( -12)
Þ 27F ¢(3) - F ¢(1) = 4 (3) x 2 sin( b x )
27. Let a , b ∈ be such that lim = 1. Then 6(α +  β)
x → 0 a x - sin x
Hence, from Eqs. (1) and (3), we get
equals _______.
27F ¢(3) - F ¢(1) - 36 = 4 - 36 = -32 [JEE ADVANCED 2016]
⇒ 9f ′(3) - f ′(1) + 32 = 0 x 2 sin b x
Solution: It is given that a , b ∈ such that lim = 1.
x ® 0 a x - sin x
Hence, the correct answers are options (C) and (D).
Therefore,
26. Let f : (0 , ¥ ) ®  be a differentiable function such that x 2 sin b x
lim =1
f (x) x ® 0 a x - sin x
f ¢( x ) = 2 - for all x ∈ (0,∞) and f (1) ≠ 1. Then
x æ sin b x ö
x2 ç ÷b x
æ 1ö æ 1ö bx ø
(A)  lim+ f ¢ ç ÷ = 1 (B)  lim+ xf ç ÷ = 2 Þ lim è =1
x ®0 èxø x ®0 èxø x ®0 a x - sin x
(C)  lim+ x 2f ¢( x ) = 0 (D)  f ( x ) £ 2 for all x Î (0,2) x3
x ®0 ⇒ b lim =1
[JEE ADVANCED 2016] x →0  x3 x5 
ax -x - + - 
Solution: It is given that  3! 5! 
f (x)
f : ( 0 , ∞ ) →  , f ′( x ) = 2 - x3
x ⇒ b lim =1
x →0 x3 x5
x (a - 1) + - +
Now, the linear differential equation is 3! 5!
f (x)
f ¢( x ) + =2 For finite limit α = 1,
x
1
The integrating factor is 3! × β = 1 Þ b =
1 6
ò x dx
e = eln x = x
Then, æ 1ö
Therefore, 6(a + b ) = 6 ç 1+ ÷ = 6 + 1 = 7
è 6ø
ò d( xf ( x )) = ò 2 x dx + c Hence, the correct answer is (7).
2 æ c ö
xf ( x ) = x + c Þ f ( x ) = ç x + ÷ "x Î (0 , ¥ ) 28. Let a, b Î and f :  →  be defined by
è x ø
f(x) = a cos(|x3 - x|) + b|x| sin(|x3 + x|). Then f is
Now, (A)  Differentiable at x = 0 if a = 0 and b = 1.
f (1) ≠ 1⇒ 1 ≠ 1+ c ⇒ c ≠ 0 (B)  Differentiable at x = 1 if a = 1 and b = 0.
and (C)  NOT differentiable at x = 0 if a = 1 and b = 0.
c æ c ö (D)  NOT differentiable at x = 1 if a = 1 and b = 1.
f ¢( x ) = 1- Þ lim+ f ¢( x ) = lim+ ç 1- 2 ÷ = 1
x2 x ®0 x ®0 è x ø [JEE ADVANCED 2016]
Solution: The given function is
æ 1ö
Þ lim+ f ¢ ç ÷ = lim+ (1- cx 2 ) = 1 f(x) = a cos(|x3 - x|) + b|x| sin(|x3 + x|)
x ®0 è x ø x ®0 which is an even function.
Hence, option (A) is correct. f(x) = a cos(x3 - x) + bx sin(x3 + x)

Mathematical Problem Book for JEE.indb 828 07-06-2018 13:07:52


Chapter 19 | Limit, Continuity and Differentiability 829

For a function to be differentiable at x = 0, the function must be Therefore,


continuous. é æ öù
rx
f(0) = a cos(0 - 0) + b(0)sin(0) = a ê n 1 ç 1+ ÷ú
Therefore, ln f ( x ) = x lim ê å ln ç n ÷ú
n ®¥ ê 2 2
r =1 n ç r x ÷ú
f (0 + ) = lim [a cos(h3 - h) + bh sin(h3 + h)] = a ê ç 1+ 2 ÷ú
h→ 0 ë è n øû
1
æ 1+ xy ö
f (0 - ) = lim [a(cos( -h3 + h) + b( -h)sin( -h3 - h)] ln f ( x ) = x ò ln ç dy
h®0 2 2÷
0 è 1+ x y ø

= lim [a cos(h3 - h) + bh sin(h3 + h)] = a dt


h→ 0 Substituting xy = t, we get ⇒ dy = . Therefore,
x
which is continuous at x = 0; hence, f(x) is differentiable for all x x
x æ 1+ t ö æ 1+ t ö
x ò0 è 1+ t 2 ø
values of a and b. Therefore, ln f ( x ) = ln ç ÷ dt = ò ln ç 2÷
dt
f(1) = acos(1 - 1) + b 1sin(1 + 1) = a + b sin 2 0 è 1+ t ø

Applying Newton-Leibniz rule, we get


f (1+ ) = lim a cos (1+ h)3 - (1+ h) + b(1+ h)sin (1+ h)3 + (1+ h) = f (1)
h→ 0 f ¢( x ) æ 1+ x ö æ 1+ x ö
= ln ç ÷ Þ f ¢( x ) = f ( x )ln ç 2÷
- 3 3
f (1 ) = lim a cos(1- h) - (1- h) + b(1- h)sin(1- h) + (1- h) = f (1) f (x) è 1+ x 2 ø è 1+ x ø
h®0
It is obvious that f ( x ) > 0 "x > 0.
Thus, f(x) is continuous and we can also see that f is differentiable
at x = 0 and x = 1. f ¢(2) æ3ö
For x = 2: = ln ç ÷ < 0 Þ f ¢(2) £ 0
f (2 ) è5ø
Hence, the correct answers are options (A) and (B).
Hence, option (C) is correct.
x /n
æ nö æ nö ö That is, f ′(x) ≥ 0 ∀ x ∈ [0,1], so f(x) is an increasing function.
ç æ
n n ( x + n ) ç x + ÷ ç x + ÷ ÷
ç è 2ø è nø ÷ That is,
29. Let f ( x ) = lim ç 2 ö÷
, for all
æ 2ö æ
ç n !( x 2 + n2 ) ç x 2 + n ÷ç x 2 + n ÷ ÷
n ®¥
 1  2
ç f  ≤f 
ç
è è 4 ÷ø çè n2 ÷ø ÷ø  3  3
x > 0. Then Hence, option (B) is correct.
æ 1ö æ 1ö æ 2 ö Hence, the correct answers are options (B) and (C).
(A)  f ç ÷ ³ f (1)
(B)  f ç ÷ £ f ç ÷
è2ø è3ø è3ø
é 1 ù é 1 ù
f ¢(3) f ¢(2) 30. Let f : ê - , 2ú ®  and g : ê - , 2ú ®  be function
(C)  f ¢(2) £ 0 (D)  ³ ë 2 û ë 2 û
f ( 3) f ( 2 ) defined by f(x) = [x - 3] and g(x) = |x|f(x) + |4x - 7|f(x), [⋅]
2

[JEE ADVANCED 2016] denotes the greatest integer less than or equal to y for y Î.
Then
Solution: The given function is
é 1 ù
x /n (A)  f is discontinuous exactly at three points in ê - , 2 ú .
é nn ( x + n)( x + n/ 2)( x + n/n) ù ë 2 û
f ( x ) = lim ê 2 2 2 2 2 2 2 ú "x > 0
n ®¥ ê n !( x + n )( x + n / 4 )( x + n /n ) ú
ë û é 1 ù
(B)  f is discontinuous exactly at four points in ê - , 2 ú .
x /n ë 2 û
 x 2x nx 
 nnnn  1+   1+    1+  2 
æ 1 ö
  n  n  n (n !)  (C)  g is NOT differentiable exactly at four points in ç - , 2 ÷ .
⇒ f ( x ) = lim  × è 2 ø
n→∞  x 2   4 x 2   n2 x 2  (n2 )n 
 n ! n ! 1+ 
  n2   1+ n2    1+ n2  
æ 1 ö
(D)  g is NOT differentiable exactly at five points in ç - , 2 ÷ .
    
è 2 ø
x n
  [JEE ADVANCED 2016]
x 2x nx
  1+   1+    1+   Solution: It is given that
  n  n  n 
f ( x ) = lim  1 
n→∞  x 2 
4 x 2  n2 x 2   f :  , 2 → 
 1+ 1+ 2    1+ 2   2 
  n2   n   n  
é 1 ù
x /n g : ê - , 2ú ® 
é æ rx öù ë 2 û
ê n ç 1+ ÷ú
= lim Õ ç
ê n ÷ú Therefore,
n ®¥ ê
r =1 ç r2x2 ÷ú
ê ç 1+ 2 ÷ú f(x) = [x2 - 3], g(x) = [|x| + |4x - 7|]f(x)
ë è n øû
f(x) = [x2] - 3

Mathematical Problem Book for JEE.indb 829 07-06-2018 13:08:19


830 Mathematics Problem Book for JEE

  1  sin x
 -3 x ∈  - 2 , 0  8.   lim =
  1  x ®¥ x

 -3, x ∈  - ,1

  2  (A)  1 (B) 0 (C) 
Does not exist (D)  None of these
 -3 x ∈ [ 0 , 1)
  2 sin2 3 x
  -2, x ∈[1, 2 ) 9.   lim =
x ∈[1, 2 ) 
x ®0 x2
 -2
2   (A)  6 (B) 9 (C)  18 (D) 3
f (x) = [x ] - 3 =  =  -1, x ∈[ 2 , 3 )
 -1 x ∈[ 2 , 3 )  sina - cos a
  10.   lim =
  0, x ∈[ 3 , 2) a ®p / 4 p
a-
0 x ∈[ 3 , 2)  4
 
  1, x=2 (A)  2 (B)  1/ 2 (C) 
1 (D)  None of these
 
1 x =2 11.   lim (sec q - tanq ) =
 x ®p / 2

g( x ) = (| x | + | 4 x - 7 |) ⋅ f ( x ) (A)  0 (B) 1/2 (C) 


2 (D) ∞
Let h(x) = |x| + |4x - 7|. Then tan 2 x - x
12.   lim =
g(x) = h(x)·f(x) x ®0 3 x - sin x
where h(x) is continuous for all x and it has sharp edge at 0 and 7/4; (A)  0 (B) 1 (C) 
1/2 (D) 1/3
f(x) is discontinuous at 2x -p
x = 1, 2 , 3 , 2 13.   lim =
x ®p / 2 cos x
Hence, option (B) is correct. -2 (D)  None of these
(A)  2 (B) 1 (C) 
Thus, g(x) is non-differentiable at
(a + h)2 sin(a + h) - a2 sin a
14.   lim =
x = 0 ,1, 2 , 3 , 2 h®0 h
Hence, option (D) is correct. ( A)  a cos a + a2 sin a (B)  a sin a + a2 cos a
Hence, the correct answers are options (B) and (D). (C) 2a sin a + a2 cos a (D) 2a cos a + a2 sin a

Practice Exercise 1 (1+ x )1/ 2 - (1- x )1/ 2


15.   lim =
x ®0 x
ì 1 1 (D)  -1
(A)  0 (B) 1/2 (C) 
ï x sin , x ¹ 0
1. If f ( x ) = í x , then lim f ( x ) =
ïî
x ®0 a + 2x - 3x
0, x = 0 16.   lim =
x ®a 3a + x - 2 x
(A) 1 (B) 0 (C)  -1 (D)  None of these
1 2 2 2
(2 x - 3)( x - 1) (A)  (B)  (C)  (D) 
2.  lim = 3 3 3 3 3
x ®1 2 x 2 + x - 3

(A)  -1/10 (B) 1/10 (C)  -1/8 (D)  None of these x .2 x - x


17.   lim =
x ® 0 1- cos x
f (x) - 3
3. If f(9) = 9, f ′(9) = 4, then lim =
x ®9 x -3 (A)  0 (B)  log 4 (C) 
log 2 (D)  None of these
(A) 2 (B) 4 (C)  -2 (D)  -4 tan x - sin x
18.   lim =
|x| x ®0 x3
4.   lim =
x ®0 x 1 1 2
(A)  (B)  - (C)  (D)  None of these
(A) 1 (B)  -1 (C) 0 (D)  Does not exist 2 2 3
x +h - x 19.   lim x x =
5.   lim =
h®0 h x ®0

1 1 (A)  0 (B) 1 (C) 


e (D)  None of these
(A)  (B)  2 x (D) 
(C)  x
2 x x (2 x + 1)40 ( 4 x - 1)5
20.   lim =
x ®¥ (2 x + 3)45
2x -1
6.   lim = (A)  16 (B) 24 (C) 
32 (D) 8
x ® 0 (1+ x )1/ 2 - 1

(A)  log 2 (B)  log 4 (C) 


log 2 (D)  None of these é x ù
21.   lim ê -1 ú =
x ® 0 ë tan 2 x û
x -1
7.  lim =
x ®1 2 x 2
- 7x + 5 1
(A)  0 (B)  (C) 
1 (D) ∞
-1/3 (D)  None of these
(A)  1/3 (B) 1/11 (C)  2

Mathematical Problem Book for JEE.indb 830 07-06-2018 13:08:48


Chapter 19 | Limit, Continuity and Differentiability 831

2 33. Given that f ′(2) = 6 and f ′(1) = 4, then


e x - cos x
22.   lim = f (2h + 2 + h2 ) - f (2)
x ®0 x2 lim
h ® 0 f ( h - h2 + 1) - f (1)
3 1
(A)  (B)  - (C) 
1 (D)  None of these (A)  Does not exist (B)  Is equal to -3/2
2 2
(C)  Is equal to 3/2 (D)  Is equal to 3
log(a + x ) - log a log x - 1 e x - e- x
23.   lim + k lim = 1, then 34.   lim is
x ®0 x x ®e x - e x ® 0 sin x

(A)  0 (B) 1 (C) 


2 (D) Non-existent
æ 1ö
(A)  k = e ç 1- ÷ (B)  k = e(1 + a)
è aø é 3 sin x - 3 cos x ù
35.   lim ê ú=
(C)  k = e(2 - a) (D)  The equality is not possible x ®p / 6 ê
ë 6x -p úû

1 (A)  - 3 (D)  -1/ 3


3 (B)  1/ 3 (C) 
(1- cos 2 x )
24.   lim 2 = cos(sin x ) - 1
x ®0 x 36.   lim =
x ®0 x2
(A)  1 (B)  -1 (C) 
0 (D)  None of these (A)  1 (B)  -1 (C) 
1/2 (D)  -1/2
1
sin x + log(1- x )
25.   lim is equal to 37.   lim (3n + 4 n ) n =
x ®0 x2 n ®¥

1 1 (A)  3 (B) 4 (C) 


∞ (D)  e
- (D)  None of these
(A)  0 (B)  (C)  2x
2 2 æ a b ö
38. If lim ç 1+ + 2 ÷ = e 2 , then the values of a and b are
x ®¥ è x x ø
x + sin x
26.   lim = ( A)  a = 1, b = 2 (B)  a = 1, b ∈ R
x ®¥ x - cos x
(C)  a ∈ R, b = 2 (D)  a ∈ R, b ∈ R
-1 (D)  None of these
(A)  0 (B) 1 (C) 
p
1/ x
-q
é æp öù 2 =
39. limp
27.   xlim ê tan ç 4 + x ÷ ú = q ® cot q
®0 ë è øû 2

(A)  1 (B)  -1 (C)  e2 (D)  e (A)  0 (B)  -1 (C) 


1 (D) ∞
2 3 x -1
1/ x 4 ö
æ 1+ 5 x 2 ö 40. lim æç 1- ÷ =
28.   lim çç ÷ = x ®¥ è x - 1ø
x ® 0 1+ 3 x 2 ÷
è ø
(A)  e12 (B)  e-12 (C) 
e4 (D)  e3
e-2 (D)  e-1
(A)  e2 (B)  e (C) 
é e x - e sin x ù
4 x2 + 5x + 8 41. lim ê ú is equal to
29. The value of lim is x ® 0 ê x - sin x ú
ë û
x ®-¥ 4x + 5
(A)  -1 (B) 0 (C) 
1 (D)  None of these
(A)  –1/2 (B) 0 (C) 
1/2 (D) 1
x2 + 3x + 2
x tan 2 x - 2 x tan x 42. The value of lim is equal to
30.   lim is x ®-1 x 2 + 4 x + 3
x ®0 (1- cos 2 x )2
(A)  0 (B) 1 (C) 
2 (D) 1/2
1 1
(A)  2 (B)  -2 (C)  (D)  - 2
2 2 43. The value of lim log(1+ x ) is equal to
x x ®0
x
æ x -3 ö 1
31. For x Î R , lim ç ÷ is equal to (A)  e (B)  e2 (C)  (D) 2
x ®¥ è x + 2 ø 2
x +1
(A)  e (B)  e-1 (C) 
e-5 (D)  e5 æ 3x - 4 ö 3
44. The value of lim ç ÷ is equal to
x ®¥ è 3 x + 2 ø
[(a - n) nx - tan x ]sin nx
32. If lim = 0 , where n is a non-zero real
x ®0 x2 (A)  e-1/3 (B)  e-2/3 (C) 
e-1 (D)  e-2
number, then a is equal to ( x + 1)(3 x + 4 )
45. The value of lim is equal to
n +1 1
x ®¥ x 2 ( x - 8)
(A)  0 (B)  n (D)  n +
(C) 
n n (A)  2 (B) 3 (C) 
1 (D) 0

Mathematical Problem Book for JEE.indb 831 07-06-2018 13:09:16


832 Mathematics Problem Book for JEE

n
ì sin[ x ] Sn +1 - Sn
ï , when [ x ] ¹ 0 57.  If Sn = å ak and lim an = a, then lim is equal to
46. If f ( x ) = í [x] where [x] is the greatest integer k =1
n ®¥ n ®¥ n
ï
î 0 , when [ x ] = 0 åk
k =1
function, then xlim f (x) =
®0 2a (D)  2a
(A)  0 (B)  a (C) 
(A)  -1 (B) 1 (C) 
0 (D)  None of these
4 + 3an
1- (10 )n -a 58. If a1 = 1 and an +1 = , n ³ 1 and if lim an = a, then the
47. If lim = , then the value of a is 3 + 2an n→∞
n ®¥ 1+ (10 )n +1 10 value of a is
(A)  0 (B)  -1 (C) 
1 (D) 2 (A)  2 (B)  - 2 (C) 
2 (D)  None of these
3
log[1+ x ]
48. The value of lim = æxö æxö æxö æ x ö
x ®0 sin3 x 59. The value of lim cos ç ÷ cos ç ÷ cos ç ÷ cos ç n ÷ is
n ®¥ è2ø è4ø è8ø è2 ø
(A)  0 (B) 1 (C) 
3 (D)  None of these
sin x x
4q (tanq - sinq ) (A)  1 (B)  (C)  (D)  None of these
49. lim is x sin x
q → 0 (1 - cos 2q )2
1 1 1 1
60. lim + + +  + n equals
(A)  1/ 2 (B) 1/2 (C) 
1 (D) 2 n ®¥ 2 22 23 2

27 x - 9 x - 3 x + 1 (A)  2 (B)  -1 (C) 


1 (D) 3
50. The value of lim is
x ® 0 5 - 4 + cos x
ì1 2 3 nü
2
61. lim í 2 + 2 + 2 +  + 2 ý is
5 (log 3) (B)  8 5 log 3 n ®¥ î n n n n þ
(A) 
2
(C)  16 5 log 3 (D)  8 5 (log 3) 1 (D)  ∞
(A)  1/2 (B) 0 (C) 

xn 1- n2
51. The value of lim where x < -1 is 62. The value of lim will be
n n ®¥ å n
n ®¥ x +1

(A)  1/2 (B)  -1/2 (C) 


1 (D)  None of these (A)  -2 (B)  -1 (C) 
2 (D) 1

1 1 1 1 1
52. The value of lim + + + + + is 1 - 2 + 3 - 4 + 5 - 6 +  - 2n
n ®¥ 1.3 3.5 5.7 7.9 (2n - 1)(2n + 1) 63. If x n = , then lim x n is equal to
n2 + 1 + 4 n2 - 1 n ®¥
equal to
(A)  1/2 (B) 1/3 (C) 
1/4 (D)  None of these 1 2 2
(A)  (B)  - (C)  (D) 1
53. The value of the constant a and b such that 3 3 3
æ x2 +1 ö ( x + 1)10 + ( x + 2)10 +  + ( x + 100 )10
lim çç - a x - b ÷÷ = 0 are respectively 64. lim is equal to
x ®¥ x + 1
è ø x ®¥ x 10 + 1010
(1, -1) (D)  (0, 1)
(A)  (1, 1) (B) (-1, 1) (C)  (A)  0 (B) 1 (C) 
10 (D) 100
54. Let f : R ® R be a differentiable function having 1+ 2 + 3 +  + n
f(x)
65. The value of lim is equal to
ò
3
4t dt n→∞ n2 + 100
æ 1 ö 6
f (2) = 6 , f ’(2) = ç ÷ . Then lim equals 1
è 48 ø x ®2 x -2 (A)  ∞ (B)  (C) 
2 (D) 0
2
(A)  12 (B) 18 (C) 
24 (D) 36
x
ò cos t
2
é 1 2 n ù 66. The value of lim 0 dt is
55.   lim ê + + + is equal to
n ®¥ ë 1- n2 1- n2 1- n2 úû x ®0 x

1 -1 (D)  None of these


(A)  0 (B) 1 (C) 
1
(A)  0 (B)  - (C)  (D)  None of these
2 2 67. If f ( x ) = | x - 2 | , then
é 1 4 9 n ù 2
56.   lim ê 3 + 3 + 3 +  + 3 ú = (A)  lim f ( x ) ¹ 0 (B)  lim f ( x ) ¹ 0
n ®¥ ê n + 1 n + 1 n + 1 x ®2 -
ë n + 1úû x ®2 +

(C)  lim f ( x ) ¹ lim f ( x ) (D)  f(x) is continuous at x = 2


(A)  1 (B) 2/3 (C) 
1/3 (D) 0 x ®2 + x ®2 -

Mathematical Problem Book for JEE.indb 832 07-06-2018 13:09:51


Chapter 19 | Limit, Continuity and Differentiability 833

ì k cos x p ìïe1/ x , when x ¹ 0


ïï p - 2 x , when x ¹ 2 75. If f ( x ) = í , then
68. If the function f ( x ) = í be continuous at ïî0 , when x = 0
ï3, p
when x =
ïî 2 (A)  lim f ( x ) = e (B)  lim f ( x ) = 0
x ®0 + x ®0 +
p (C)  f(x) is discontinuous at x = 0 (D)  None of these
x = , then k =
2
ì x2 - 4 x + 3
(A)  3 (B) 6 (C) 
12 (D)  None of these ï , for x ¹ 1
76. If f ( x ) = í x 2 - 1 , then
ï 2 , for x = 1
log(1+ ax ) - log(1- bx ) î
69. The function f ( x ) = is not defined at
x (A)  lim f ( x ) = 2 (B)  lim f ( x ) = 3
x = 0. The value which should be assigned to f at x = 0 so that x ®1+ x ®1-
it is continuous at x = 0 is (C)  f(x) is discontinuous at x = 1 (D)  None of these
(A)  a - b (B)  a + b (C) loga + logb (D) loga - logb x +1
77. T he points at which the function f ( x ) = 2 is discon-
tinuous are x + x - 12
ì x 3 + x 2 - 16 x + 20
ï , if x ¹ 2 (A)  -3, 4 (B) 3, -4 (C) 
-1, -3, 4 (D)  -1, 3, 4
70. Let f ( x ) = í ( x - 2)2 . If f(x) be continuous for
ïk , if x = 2
î  sin x
+ cos x , when x ≠ 0
all x, then k = 78. If f ( x ) =  x , then
 2, when x = 0
(A)  7 (B)  -7 (C) 
±7 (D)  None of these
(A)  lim f ( x ) ¹ 2 (B)  lim f ( x ) = 0
ìï x 2 + k , when x ³ 0 x ®0 + x ®0 -
71. Let f ( x ) = í 2 . If the function f(x) be (C)  f(x) is continuous at x = 0 (D)  None of these
ïî- x - k , when x < 0
continuous at x = 0, then k = ì 2 1
ï x sin , when x ¹ 0
79. If f ( x ) = í x , then
2 (D)  -2
(A)  0 (B) 1 (C)  ïî 0 , when x = 0
72. I n order that the function f(x) = (x + 1)1/x is continuous at x = 0,
(A)  f(0 + 0) = 1 (B)  f(0 - 0) = 1
f(0) must be defined as
(C)  f is continuous at x = 0 (D)  None of these
(A)  f(0) = 0 (B)  f(0) = e
80. The value of k so that the function
(C)  f(0) = 1/e (D)  f(0) = 1
ìïk (2 x - x 2 ), when x < 0
ìx, when 0 < x < 1/ 2 f (x) = í is continuous at x = 0 is
ï ïî cos x , when x ³ 0
73. If f ( x ) = í1, when x = 1/ 2 , then
ï1- x , when1/2 < x < 1 (A)  1 (B) 2 (C) 
4 (D)  None of these
î
ì x
(A)  ï , when x ¹ 0
lim f ( x ) = 2 81. If f ( x ) = í e1/ x + 1 , then
x ®1/ 2 +
ï 0 , when x = 0
(B)  lim f ( x ) = 2 î
x ®1/ 2 -
(A)  lim f ( x ) = 1 (B)  lim f ( x ) = 1
1 x ®0 + x ®0 -
(C)  f(x) is continuous at x =
2 (C)  f(x) is continuous at x = 0 (D)  None of these
1 ìï(1+ 2 x )1/ x , for x ¹ 0
(D)  f(x) is discontinuous at x = 82. If f ( x ) = í , then
2
îï e 2 , for x = 0
ì( x 2 / a) - a, when x < a (A)  lim f ( x ) = e (B)  lim f ( x ) = e 2
ï x ®0 + x ®0 -
74. If f ( x ) = í 0, when x = a, then
ï 2
(C)  f(x) is discontinuous at x = 0 (D)  None of these
î a - ( x / a ), when x >a
ïì2 , for x ¹ 0
1/ x

(A)  lim f ( x ) = a 83. If f ( x ) = í , then


x ®a îï 3, for x = 0
(B)  f(x) is continuous at x = a lim f ( x ) = 0 f (x) = ¥
(A)  (B)  xlim
(C)  f(x) is discontinuous at x = a x ®0 + ®0 -

(D)  None of these (C)  f(x) is continuous at x = 0 (D)  None of these

Mathematical Problem Book for JEE.indb 833 07-06-2018 13:10:24


834 Mathematics Problem Book for JEE

ì1 2 ìïsin-1 | x |, when x ¹ 0
ï sin x , x ¹ 0 91. If f ( x ) = í , then
84. If f ( x ) = í x , then 0 , when x = 0
ïî
ïî 0, x = 0
(A)  lim f ( x ) ¹ 0 (B)  lim f ( x ) ¹ 0
x ®0 + x ®0 -
(A)  lim f ( x ) ¹ 0 (B)  lim f ( x ) ¹ 0
x ®0 + x ®0 -
(C)  f(x) is continuous at x = 0 (D)  None of these
(C)  f(x) is continuous at x = 0 (D)  None of these
ì sin 2 x
ï , when x ¹ 0
ì x - 1, x < 0 92. If f ( x ) = í 5 x is continuous at x = 0, then the
ï ïî k , when x = 0
ï 1
85. If f ( x ) = í , x = 0 , then
value of k will be
ï 4
ïî x 2 , x > 0 2 2
(A)  1 (B)  - (D)  None of these
(C) 
lim f ( x ) = 1 5 5
(A)  (B)  lim f ( x ) = 1
x ®0 + x ®0 -
ïì1+ x , when 0 £ x £ 1
2

(C)  f(x) is discontinuous at x = 0 (D)  None of these 93. If f ( x ) = í , then


îï1- x , when x > 1
 hich of the following statements is true for graph f(x) =
86. W
(A)  lim+ f ( x ) ¹ 0 (B)  lim- f ( x ) ¹ 2
log x ? x ®1 x ®1

(A)  Graph shows that function is continuous (C)  f(x) is discontinuous at x = 1 (D)  None of these
(B)  Graph shows that function is discontinuous
ì x2 -1
(C)  Graph finds for negative and positive values of x ï , when x ¹ -1
94. If f ( x ) = í x + 1 , then
(D)  Graph is symmetric along x-axis ï - 2, when x = -1
î
ì x2 -1 (A)  lim f ( x ) = -2 (B)  lim f ( x ) = -2
ï , when x ¹ 1 x ®( -1)- x ®( -1)+
87. If function f ( x ) = í x - 1 is continuous at x = 1,
ï (C)  f(x) is continuous at x = -1 (D)  All the above are correct
î k , when x = 1
then the value of k will be ì5
ï 2 - x , when x < 2
(A)  -1 (B) 2 (C)  -3 (D)  -2 ï
95. If f ( x ) = í1, when x = 2 , then
x
88. At which points the function f ( x ) = , where [·] is greatest ï 3
[x] ï x - , when x > 2
integer function, is discontinuous î 2
(A)  f(x) is continuous at x = 2
(A)  Only positive integers
(B)  f(x) is discontinuous at x = 2
(B)  All positive and negative integers and (0, 1)
(C)  lim f ( x ) = 1
(C)  All rational numbers x ®2

(D)  None of these (D)  None of these

ì sin2 ax 96. If f(x) = | x - b|, then function


ï , when x ¹ 0
89. For the function f ( x ) = í x 2 which one is a (A)  Is continuous at x = 1
ï 1, when x = 0 (B)  Is continuous at x = b
î
true statement (C)  Is discontinuous at x = b
(D)  None of these
(A)  f(x) is continuous at x = 0
(B)  f(x) is discontinuous at x = 0, when a ≠ ±1 ì| x - a |
ï , when x ¹ a
97. If f ( x ) = í x - a , then
(C)  f(x) is continuous at x = a ïî 1, when x = a
(D)  None of these
(A)  f(x) is continuous at x = a
ì - x2 , when x £ 0 (B)  f(x) is discontinuous at x = a
ï
ï 5x - 4, when 0 < x £ 1 (C)  lim f ( x ) = 1
90. If f ( x ) = í , then x ®a
2
ï4 x - 3 x , when 1 < x < 2
(D)  None of these
ï 3x + 4, when x ³ 2
î
ïì x , when x ¹ 1
2

(A)  f(x) is continuous at x = 0 98. If f ( x ) = í , then


ïî 2, when x = 1
(B)  f(x) is continuous at x = 2
(C)  f(x) is discontinuous at x = 1 (A)  lim f ( x ) = 2
x ®1
(D)  None of these (B)  f(x) is continuous at x = 1

Mathematical Problem Book for JEE.indb 834 07-06-2018 13:10:51


Chapter 19 | Limit, Continuity and Differentiability 835

(C)  f(x) is discontinuous at x = 1 ì x 4 - 16


(D)  None of these ï , when x ¹ 2
105. If f ( x ) = í x - 2 , then
ì1+ x , when x £ 2 ï 16 , when x = 2
99. If f ( x ) = í , then î
î5 - x , when x > 2 (A)  f(x) is continuous at x = 2
( A)  f(x) is continuous at x = 2 (B)  f(x) is discountinous at x = 2
(B)  f(x) is discontinuous at x = 2
(C)  f(x) is continuous at x = 3 (C)  lim f ( x ) = 16
x ®2
(D)  None of these
(D)  None of these
 3p
 1, when 0 < x ≤
4 ïì x , when x £ 1
2
100. If f ( x ) =  , then 106. If f ( x ) = í , then
2
2 sin x , when 3p
< x <p îï x + 5, when x > 1
 9 4
( A)  f(x) is continuous at x = 0 (A)  f(x) is continuous at x = 1
(B)  f(x) is continuous at x = p (B)  f(x) is discontinuous at x = 1
3p (C)  lim f ( x ) = 1
(C)  f(x) is continuous at x = x ®1
4
3p (D)  None of these
(D)  f(x) is discontinuous at x =
4
ì x 2 + 3 x - 10
ï , when x ¹ -5
ì p 107. I f f ( x ) = í x 2 + 2 x - 15 is continuous at x = -5,
ïï x sin x , when 0 < x £
2 , then ïa , when x = -5
101. If f ( x ) = í î
ï p sin(p + x ), when p < x < p then the value of ‘a’ will be
ïî 2 2
3 7 8 2
(A)  f(x) is discontinuous at x = p/2 (A)  (B)  (C)  (D) 
(B)  f(x) is continuous at x = p/2 2 8 7 3
(C)  f(x) is continuous at x = 0
(D)  None of these ìx + l, x < 3
ï
ì 108. If f ( x ) = í 4 , x = 3 is continuous at x = 3, then λ =
ï 1- cos 4 x ï3 x - 5, x > 3
, when x < 0 î
ï x2
102. If f ( x ) = ïí a, when x = 0 , (A)  4 (B) 3 (C) 
2 (D) 1
ï
ï x
, when x > 0 ì 1
ï (16 + x ) - 4 ïsin , x ¹ 0
î 109. The value of k which makes f ( x ) = í x continuous
is continuous at x = 0, then the value of ‘a’ will be ïî k , x = 0
(A)  8 (B)  -8 (C)  4 (D)  None of these at x = 0 is
ax - b , when 0 ≤ x < 1
2 -1 (D)  None of these
(A)  8 (B) 1 (C) 

103. If f ( x ) =  2, when x = 1 is continuous at x = 1, then ì x 2 + 1, x ¹ 0 , 2
 x + 1, when1 < x ≤ 2 ìsin x , x ¹ np , n Î Z ï
 110. I f f ( x ) = í and g( x ) = í 4, x = 0 ,
the most suitable value of a, b are î 2, otherwise ï 5, x = 2
î
(A)  a = 2, b = 0 (B)  a = 1, b = -1 then lim g [f ( x )] is
x ®0
(C)  a = 4, b = 2 (D)  All the above (A)  5 (B) 6 (C) 
7 (D) 1
ì x- | x |
ï , when x ¹ 0 ì x -4
104. If f ( x ) = í x , then ï| x - 4 | + a, x < 4
ïî 2, when x = 0 ïï
111. Let f ( x ) = í a + b , x = 4 . Then f(x) is continuous at x = 4
(A)  f(x) is continuous at x = 0 ï x -4 when
ï + b, x > 4
(B)  f(x) is discontinuous at x = 0
îï| x - 4 |
(C)  lim f ( x ) = 2
x ®0 (A)  a = 0, b = 0 (B)  a = 1, b = 1
(D)  None of these (C)  a = -1, b = 1 (D)  a = 1, b = -1

Mathematical Problem Book for JEE.indb 835 07-06-2018 13:11:14


836 Mathematics Problem Book for JEE

ì x 4 - 5x2 + 4 (A)  A = 0, B = 1 (B)  A = 1, B = 1


ï , x ¹ 1, 2 (C)  A = -1, B = 1 (D)  A = -1, B = 0
ï| ( x - 1)( x - 2)|
ï
112. Let f ( x ) = í 6, x = 1 x 2 - 10 x + 25
ï 119. If f ( x ) = for x ≠ 5 and f is continuous at x = 5,
12, x = 2 x 2 - 7 x + 10
ï
ïî then f(5) =
Then f(x) is continuous on the set (A)  0 (B) 5 (C) 
10 (D) 25
(A)  R (B)  R - {1} (C) 
R - {2} (D)  R - {1, 2} 120. I n order that the function f(x) = (x + 1)cot x is continuous at
ì x - 1, x < 2 x = 0, f(0) must be defined as
113. Function f ( x ) = í is a continuous function
î2 x - 3, x ³ 2 (A)  f (0 ) =
1
(B)  f (0 ) = 0
( A)  For all real values of x e
(B) For x = 2 only (C)  f (0 ) = e (D)  None of these
(C)  For all real values of x such that x ≠ 2
(D)  For all integral values of x only 121. The function f(x) = sin|x| is
ì px (A)  Continuous for all x
ï1+ sin 2 , for - ¥ < x £ 1 (B)  Continuous only at certain points
ï (C)  Differentiable at all points
114. If the function f ( x ) = í ax + b , for 1 < x < 3 is contin-
ï (D)  None of these
xp
ï 6 tan , for 3 £ x < 6
î 12 122. If f(x) = |x|, then f(x) is
uous in the interval (-∞, 6), then the values of a and b are (A)  Continuous for all x
respectively (B)  Differentiable at x = 0
(A)  0, 2 (B)  1, 1 (C) 
2, 0 (D)  2, 1 (C)  Neither continuous nor differentiable at x = 0
 sin[ x ] (D)  None of these
 [ x ] + 1 , for x > 0
 p
 cos p [ x ] ì1- sin x
ïï - , x¹
123. I f f ( x ) = í p 2 x
 2 , for x < 0; where [x] denotes the greatest 2 , be continuous at x = p/2, then
115. I f f ( x ) = 
[ x ] ï p
 l, x=
 k , at x = 0 ïî 2

 value of λ is

(A)  -1 (B) 1 (C) 
0 (D) 2
integer less than or equal to x, then in order that f be contin-
uous at x = 0, the value of k is ì sinp x
ï , x ¹0
(A)  Equal to 0 (B)  Equal to 1 124. Let f ( x ) = í 5 x . If f(x) is continuous at x = 0,
(C)  Equal to -1 (D) Indeterminate ïî k , x =0
ì x + 2, 1 £ x £ 2 then k =
ï
116. The function f ( x ) = í4 , x =2 is continuous at p 5
ï3 x - 2, x > 2 (A)  (B)  (C) 
1 (D) 0
î 5 p
(A)  x = 2 only (B)  x ≤ 2 2- x +4
(C)  x ≥ 2 (D)  None of these 125. If f ( x ) = , ( x ¹ 0 ), is continuous function at x = 0,
sin 2 x
ìï 5 x - 4 , if 0 < x £ 1 then f(0) equals
117. If the function f ( x ) = í 2 is continuous
îï4 x + 3bx , if 1 < x < 2 1 1 1 1
(A)  (B)  - (C)  (D)  -
at every point of its domain, then the value of b is 4 4 8 8
(A)  -1 (B) 0 (C)  1 (D)  None of these
ì x, if x is rational
118. The values of A and B such that the function 126. If function f ( x ) = í , then f(x) is contin-
î1- x , if x is irrational
ì p
ï -2 sin x , x£-
2
uous at _____ number of points.
ï
ï p p (A)  ∞ (B) 1 (C) 
0 (D)  None of these
f ( x ) = í A sin x + B , - < x < , is continuous every where
ï 2 2 ì x2 - 9
ï p ï , if x ¹ 3
x³ 127. If f ( x ) = í x - 3
ï cos x , 2 ï2 x + k , otherwise, is continuous at x = 3, then
î î
are k equals to

Mathematical Problem Book for JEE.indb 836 07-06-2018 13:11:32


Chapter 19 | Limit, Continuity and Differentiability 837

-6 (D) 1/6
(A)  3 (B) 0 (C)  2 x - sin-1 x
136. If the function, f ( x ) = , ( x ¹ 0 ) is continuous at
2 x + tan-1 x
ìæ 1 ö -1
ïïç x 2 + e 2 - x ÷ , x ¹2 each point of its domain, then the value of f(0) is
The function defined by f ( x ) = íç
128.  ÷ , is
ïè ø (A)  2 (B) 1/3 (C) 2/3 (D)  -1/3
îï k, x =2
|x|
continuous from right at the point x = 2, then k is equal to 137. The function f ( x ) = | x | + is
x
-1/4 (D)  None of these
(A)  0 (B) 1/4 (C)  (A)  Continuous at the origin
loge (1+ x ) - loge (1- x ) (B) Discontinuous at the origin because |x| is discontinuous
129. For the function f ( x ) = to be continu- there
x
|x|
ous at x = 0 the value of f(0) should be (C) Discontinuous at the origin because is discontinu-
x
(A)  -1 (B) 0 (C) 
-2 (D) 2 ous there
|x|
ì 1+ kx - 1- kx (D) Discontinuous at the origin because both |x| and
, for - 1 £ x < 0 x
ï are discontinuous there
130. If f ( x ) = í x , is continuous at
ï 2 x 2 + 3 x - 2, for 0 £ x £ 1 138. The value of f at x = 0 so that the function
î
x = 0, then k = 2 x - 2- x
f (x) = , x ¹ 0, is continuous at x = 0 is
(A)  - 4 (B)  - 3 (C) 
- 2 (D)  -1 x
(A)  0 (B)  log 2 (C) 
4 (D)  log 4
1- sin x + cos x
131. The function f ( x ) = is not defined at x = p.
1+ sin x + cos x 2x2 + 7
139. The function f ( x ) = is discontinuous for
The value of f(p), so that f(x) is continuous at x = p, is x + 3x2 - x - 3
3

(A)  x = 1 only
1 1 (B)  x = 1 and x = -1 only
(A)  - - 1 (D) 1
(B)  (C) 
2 2 (C)  x = 1, x = -1, x = -3 only
(D)  x = 1, x = -1, x = -3 and other values of x
1- cos x
 ,x ≠0
132. If f ( x ) =  x is continuous at x = 0, then k = 1 - x
 k, x = 0  , x ≠ -1
140. If f (x) =  1+ x , then the value of f ([2x]) will be
 1, x = -1
1 1 1 
(A)  0 (B)  (C)  (D)  - (where [ ] shows the greatest integer function)
2 4 2
133. A function f on R into itself is continuous at a point a in R, iff (A)  Continuous at x = -1 (B)  Continuous at x = 0
for each ∈ > 0, there exists, δ > 0 such that 1
(C)  Discontinuous at x = (D)  All of these
(A) | f(x) - f(a) | < ∈ ⇒ | x - a | < δ 2
(B) | f(x) - f(a) | > ∈ ⇒ | x - a | > δ 1- cos 4 x
141. If the function f ( x ) = , where x ≠ 0 and f(x) = k where
(C) | x - a | > δ ⇒ | f(x) - f(a) | > ∈ 8x2
(D) | x - a | < δ ⇒ | f(x) - f(a) | < ∈ x = 0 is a continuous function at x = 0, then the value of k
will be
ì e1/ x - 1
ï , x ¹0 (A)  k = 0 (B)  k = 1 (C) 
k = -1 (D)  None of these
134. For the function f ( x ) = í e1/ x + 1 , which of the follow-
ï0 , x =0
î
ïì e ; x £ 0
x
ing is correct 142. If f ( x ) = í , then
lim f ( x ) does not exist
(A)  îï| 1- x |; x > 0
x ®0 (A)  f(x) is differentiable at x = 0
(B)  f(x) is continuous at x = 0 (B)  f(x) is continuous at x = 0
lim f ( x ) = 1
(C)  (C)  f(x) is differentiable at x = 1
x ®0 (D)  f(x) is continuous at x = 1
(D)  xlim
®0
f ( x ) exists but f(x) is not continuous at x = 0
143. Which of the following statements is true:

135. T he function ‘f  ’ is defined by f(x) = 2x - 1, if x > 2, f(x) = k if x = 2 (A)  A continuous function is an increasing function.
and x2 - 1, if x < 2 is continuous, then the value of k is equal to (B)  An increasing function is continuous.
(C)  A continuous function is differentiable.
4 (D)  -3
(A)  2 (B) 3 (C)  (D)  A differentiable function is continuous.

Mathematical Problem Book for JEE.indb 837 07-06-2018 13:11:52


838 Mathematics Problem Book for JEE

ì x + 1, when x < 2 153. Let [x] denotes the greatest integer less than or equal to x. If
144. If f ( x ) = í , then f ′(2) equals f (x) = [x sin p x], then f (x) is
î2 x - 1, when x ³ 2
(A)  Continuous at x = 0 (B)  Continuous in (-1, 0)
(A)  0 (B) 1 (C)  2 (D)  Does not exist
(C)  Differentiable in (-1,1) (D) 
All the above
 e(1/ x ) - e(-1/ x )
x ,x ≠0 ì| x - 3 |; x ³1
145. If f ( x ) =  e(1/ x ) + e( -1/ x ) then which of the following ï
154. The function defined by f ( x ) = í 1 2 3 13 is
 0 , x = 0 ïî 4 x - 2 x + 4 ; x < 1

is true
(A)  Continuous at x = 1 (B)  Continuous at x = 3
(A)  f is continuous and differentiable at every point
(B)  f is continuous at every point but is not differentiable (C)  Differentiable at x = 1 (D)  All the above
(C)  f is differentiable at every point ìï e x + ax , x < 0
(D)  f is differentiable only at the origin 155. If f ( x ) = í is differentiable at x = 0, then (a, b) is
2
146. If f(x) = |x - 3|, then f is îïb( x - 1) , x ³ 0
(A)  Discontinuous at x = 2 (A) (-3, -1) (B) (-3, 1) (C) 
(3, 1) (D) (3, -1)
(B)  Not differentiable x = 2 156. The function y = | sin x | is continuous for any x but it is not
(C)  Differentiable at x = 3 differentiable at
(D)  Continuous but not differentiable at x = 3
(A)  x = 0 only
147. Let h(x) = min{x, x2}, for every real number of x. Then
(B)  x = p only
(A)  h is continuous for all x (C)  x = k p (k is an integer) only
(B)  h is differentiable for all x
(D)  x = 0 and x = k p (k is an integer)
(C)  h′(x) = 1, for all x > 1
(D)  h is not differentiable at two values of x 157. The function y = e- | x | is
148. T here exists a function f(x) satisfying f(0) = 1, f ′(0) = -1, (A)  Continuous and differentiable at x = 0
f(x) > 0 for all x and (B)  Neither continuous nor differentiable at x = 0
(A)  f(x) < 0, ∀x (B)  -1 < f ′′(x) < 0, ∀x (C)  Continuous but not differentiable at x = 0
(D)  Not continuous but differentiable at x = 0
(C)  -2 < f ′′(x) ≤ -1, ∀x (D)  f ′′(x) < -2, ∀x
ì x , if 0 £ x £ 1 ì 1+ x , x £ 2
149. The function f ( x ) = í is 158. A function f (x) = í is
î1, if1 < x £ 2 î5 - x , x > 2
(A) Continuous at all x, 0 ≤ x ≤ 2 and differentiable at all x, (A)  Not continuous at x = 2
except x = 1 in the interval [0, 2] (B)  Differentiable at x = 2
(B)  Continuous and differentiable at all x in [0, 2] (C)  Continuous but not differentiable at x = 2
(C)  Not continuous at any point in [0, 2] (D)  None of these
(D)  Not differentiable at any point [0, 2] 159. T he left-hand derivative of f (x) = [x] sin(p x) at x = k, k is an
150. The function f (x) = | x | at x = 0 is integer and [x] = greatest integer ≤ x, is
(A)  Continuous but non-differentiable (A)  (-1)k (k - 1)p (B) (-1)k - 1 (k - 1)p
(B)  Discontinuous and differentiable (C)  (-1)k k p (D) (-1)k - 1 k p
(C)  Discontinuous and non-differentiable
(D)  Continuous and differentiable ïì 0 , x <0
160. Let f (x) = í 2  , then for all values of x
îï x , x ³0
ì x2
ï ,x ¹0 (A)  f is continuous but not differentiable
151. Consider f ( x ) = í| x |
(B)  f is differentiable but not continuous
ï 0, x = 0
î (C)  f ′ is continuous but not differentiable
(D)  f ′ is continuous and differentiable
(A) f (x) is discontinuous everywhere
(B) f (x) is continuous everywhere 161. Which of the following is not true:
(C) 
f ′(x) exists in (-1, 1) (A)  A polynomial function is always continuous
(D) 
f ′(x) exists in (-2, 2) (B)  A continuous function is always differentiable
152. At the point x = 1, the given function (C)  A differentiable function is always continuous
(D)  ex is continuous for all x
ìï x 3 - 1; 1 < x < ¥
f (x) = í is 1
162. The function f ( x ) = x 2 sin , x ¹ 0 , f (0 ) = 0 at x = 0
ïî x - 1; - ¥ < x £ 1 x
(A)  Continuous and differentiable (A)  Is continuous but not differentiable
(B)  Continuous and not differentiable (B)  Is discontinuous
(C)  Discontinuous and differentiable (C)  Is having continuous derivative
(D)  Discontinuous and not differentiable (D)  Is continuous and differentiable

Mathematical Problem Book for JEE.indb 838 07-06-2018 13:12:03


Chapter 19 | Limit, Continuity and Differentiability 839

ì x -1 ì 1 "x < 0
ïï 2 x 2 - 7 x + 5 for x ¹ 1 175. Let f (x) = í . Then what is the value
163. If f ( x ) = í , then f ′(1) = î1 + sin x "0 £ x £p /2
ï 1
- for x = 1 of f ′(x) at x = 0?
ïî 3
(A)  1 (B)  -1 (C) 
∞ (D)  Does not exist
(A)  -1/9 (B)  -2/9 (C) 
-1/3 (D) 1/3
f (5) - f (1)
x 176. I f f (x) = x2 - 2x + 4 and = f ¢(c) . Then value of c will
164. If f (x) = for x ∈ R, then f ′(0) = . 5 -1
1+ | x | be
(A)  0 (B) 1 (C) 
2 (D) 3
(A)  0 (B) 1 (C) 
2 (D) 3
mx 2 , x ≤1
165. The value of m for which the function f ( x ) =  is 177. L et f(x + y) = f (x) + f (y) and f (x) = x2g(x) for all x, y ∈ R, where
 2 x , x >1 g(x) is a continuous function. Then f ′(x) is equal to
differentiable at x = 1 is
g (0) + g ′(x) (D) 0
(A)  g ′(x) (B)  g (0) (C) 
(A)  0 (B) 1 (C) 
2 (D)  Does not exist
178. The function f(x) = (x2 - 1) |x2 - 3x + 2 | + cos(| x |) is not differ-
ì sin x , for x > 0 entiable at
166. Let f (x) = í and g(x) = ex. Then (gof  ) (0) is
î1 - cos x , for x £ 0 (A)  -1 (B) 0 (C) 
1 (D) 2
(A)  1 (B)  -1 (C) 
0 (D)  None of these 179. T he function which is continuous for all real values of x and
differentiable at x = 0 is
1
167. Suppose f (x) is differentiable at x = 1 and lim f (1+ h) = 5, 1
h®0 h (A)  | x | (B) log x (C) 
sin x (D)  x 2
then f ′(1) equals
180. Which of the following is not true:
(A)  5 (B) 6 (C) 
3 (D) 4
(A)  Every differentiable function is continuous.
168. If f is a real-valued differentiable function satisfying (B) If derivative of a function is zero at all points, then the
| f(x) - f(y) | ≤ (x - y)2, x, y ∈ R and f (0) = 0, then f (1) is equal to function is constant.
-1 (D) 0
(A)  2 (B) 1 (C)  (C)  If a function has maximum or minima at a point, then
169. Let f be differentiable for all x. If f (1) = -2 and f ′(x) ≥ 2 for the function is differentiable at that point and its deriv-
x ∈ [1, 6], then ative is zero.
(D) If a function is constant, then its derivative is zero at all
(A)  f (6) < 5 (B)  f (6) = 5 (C) 
f (6) ≥ 8 (D)  f (6) < 8
points.
170. f (x) = | | x | - 1| is not differentiable at
ì x + 2, -1 < x < 3
(A)  0 (B)  ±1, 0 (C) 
1 (D)  ±1 ï
181. If f (x) = í 5, x = 3 , then at x = 3, f ′(x) =
171. If f (x) is twice differentiable polynomial function such that ï8 - x , x > 3
î
f (1) = 1, f (2) = -4, f (3) = 9, then
(A)  f ′′(x) = 2, ∀ x ∈ R (A)  1 (B)  - 1 (C) 
0 (D)  Does not exist
(B)  There exist at least one x ∈ (1, 3) such that f ′′(x) = 2 0 £ x £1
ìx,
(C) There exist at least one x ∈ (2, 3) such that f ′(x) = 5 = f ′′(x) 182. If f (x) = í , then
(D)  There exist at least one x ∈ (1, 2) such that f (x) = 3 î2 x - 1, 1 < x

172. If f (x) is a differentiable function such that f : R → R and (A)  f is discontinuous at x = 1
(B)  f is differentiable at x = 1
 1
f   = 0 ∀ n ≥ 1, n ∈l , then (C)  f is continuous but not differentiable at x = 1
 n
(D)  None of these
(A)  f (x) = 0 ∀ x ∈ (0, 1)
(B)  f (0) = 0 = f ′(0) ìïax 2 + b ; x £0
(C)  f (0) = 0 but f ′(0) may or may not be 0 183. If f (x) = í possesses derivative at x = 0, then
2
ïî x ; x >0
(D) |f (x)| ≤ 1 ∀ x ∈ (0, 1)
173. Let f be continuous on [1, 5] and differentiable in (1, 5). (A)  a = 0, b = 0 (B)  a > 0, = 0
If f (1) = -3 and f ′(x) ≥ 9 for all x ∈ (1, 5), then (C)  a ∈ R, = 0 (D)  None of these
(A)  f (5) ≥ 33 (B)  f (5) ≥ 36
x
(C)  f (5) ≤ 36 (D)  f (5) ≥ 9 184. The set of all those points, where the function f (x) =
1+ x
174. Let f(x + y) = f (x) f (y) and f (x) = 1 + sin(3x)g(x) where g(x) is is differentiable, is
continuous. Then f ′(x) is
(A) (-∞, ∞) (B) [0, ∞]
(A)  f (x)g(0) (B) 3g(0)
(C)  f (x)cos 3x (D) 3f (x)g(0) (C)  (-∞, 0) ∪ (0, ∞) (D) (0, ∞)

Mathematical Problem Book for JEE.indb 839 07-06-2018 13:12:16


840 Mathematics Problem Book for JEE

æ 2x ö 197. lim( -1)[ x ] , where [x] is the greatest integer function, is


185. Function y = sin-1 ç ÷ is not differentiable for x ®2
è 1+ x 2 ø
equal to
(A) | x | < 1 (B)  x = 1, -1
(A) 1 (B)  -1 (C) 
±1 (D) 
None of these
(C)  | x | > 1 (D)  None of these
1+ sin x - cos x + log(1- x )
186. If f (x) = x( x - x +1), then 198.   lim equals
x ®0 x3
(A)  f (x) is continuous but non-differentiable at x = 0
(A) 1/2 (B)  -1/2 (C) 0 (D)  none of these
(B)  f (x) is differentiable at x = 0
(C)  f (x) is not differentiable at x = 0 sin(e x -2 - 1)
(D)  None of these 199. If f ( x ) = , then lim f ( x ) is given by
log( x - 1) x ®2

187. T he number of points at which the function f(x) = |x - 0.5| (A) –2 (B) 0 (C) 1 (D) –1
+ | x - 1 | + tan x does not have a derivative in the interval
(0, 2), is x cos x - log(1+ x )
200.   lim is equal to
(A)  1 (B) 2 (C) 
3 (D) 4   x ®0 x2
(A) 1/2 (B) 1 (C) 0 (D) 3
x l - 5l
188. If lim  = 500, then positive values of λ is
x ®5 x - 5 x +1 + x -1 - 2
201.   lim equals
(A) 3 (B) 4 (C) 5 (D) 6 x ®0 x
(A) 1 (B) 
–1 (C) 2 (D) 0
ìx, x <0
ï
189. If f (x)  = í1, x = 0 , then lim f(x) is equal to a tan x - asin x
ï 2 x ®0 202.   lim is equal to (a > 0) 
x >0 x ® 0 tan x - sin x
îx ,
(A) 0 (B) 1 (C) 2 (D) 
Does not exist (A)  a (B) ln a (C)  ln (1/a) (D) 1/a

190. Let f(x) = [a + b sin x], x ∈ (0, p), a ∈ I, b is a prime number æ 1+ e1/ n + e 2 / n +  + e ( n -1) / n ö
and [.] denotes G.I.F. The number of points at which f(x) is not 203.   lim ç ÷ is equal to
n ®¥ ç n ÷
differentiable is è ø
(A)  b (B) 2b + 1 (C) 2b - 1 (D) 
b+1 e -1 (D) 
(A) 0 (B) 1 (C)  e

p - cos -1 x
191.   lim éë x + [ x ]ùû , n ∈ I, is equal to ([.] denotes greatest integer 204.   lim is given by
x ®n x ®-1+ x +1
function)
1 1
(A) 0 (B) 1 (C)  -1 (D) 
Does not exist (A) = =
(B)  (C) 1 (D) 0
p 2p
2m / x
æ x 3x ö 205. Let f : R → R be such that f(1) = 3 and f ′(1) = 6. Then
192.   lim ç sin + cos ÷ is
x ®0 è m mø 1/ x
æ f (1+ x ) ö equals
lim ç ÷
e2 (C) 
(A) 1 (B)  e6m (D)  log 6m x ® 0 è f (1) ø

193. The function max {1 - x, 1 + x, 2} is e1/2 (C) 


(A) 1 (B)  e2 (D) 
e3
(A) 
Continuous at all points
(B) 
Differentiable at all points
 x 2 + 1, x ≠ 0 , 2
Continuous at all points except at - 1, 1
(C)  sin x , x ≠ np , n ∈I 
(D) 
None of these 206. I f f (x) =  and g( x ) = 2, x=0 ,
2, otherwise 4 ,
 x =2
1 æ 1+ ax ö
194. The value of lim log ç ÷
x ®0 x è 1- bx ø then lim g[f (x)] is
x ®0

(A) 0 (B) 
a/b (C) 
a + b (D)  ea/b (A) 0 (B) 1 (C) 2 (D) 6
1
 p x
195.   lim (3n + 5n + 7n ) n is equal to  sin   , x <1
n ®¥ 207. The function f(x) =  2
 2 x - 3 [ x ], x ≥1
(A)  e3 (B) 
e5 (C) 5 (D) 7 

196. W
 hich of the following functions have finite number of (A)  Is continuous at x = 1
points of discontinuity? (B)  Is differentiable at x = 1
x (C)  Is continuous but not differentiable at x = 1
(A) tan x (B)  x[x] (C)  (D)  sin [np x]
x (D)  None of these

Mathematical Problem Book for JEE.indb 840 07-06-2018 13:12:37


Chapter 19 | Limit, Continuity and Differentiability 841

(cos x - 1)(cos x - e x ) (A)  Equal to zero each


208. The integer n for which lim is a finite (B) Non-existent
x ®0 xn (C)  Equal to 1 and -1, respectively
non-zero number is (D)  None of these
(A) 1 (B) 2 (C) 3 (D) 4 x
æ x2 + 5x + 3 ö
209.   lim {logn -1(n) × logn (n + 1)lognk -1(n )} , where k ∈ N.
k 217. If f ( x ) = çç 2 ÷÷ , then lim f ( x ) is
n ®¥ è x + x +2 ø x ®¥

(A)  k/2 (B) 2k (C) 


k (D) 
None of these (A)  e (B)  e2 (C) 
e3 (D) 
e4
x
210. Let f (x) = ò | t |  dt, x ≥ - 1. Then ì-1, x <0
ï
-1 218. If f ( x ) = í0 , x = 0 and g(x) = sin x + cos x, then points of
(A)  f and f ′ are continuous for x + 1 > 0 ï1, x >0
î
(B)  f is continuous but f ′ is not continuous for x + 1 > 0 discontinuity of f {g(x)} in (0, 2p) is
(C)  f and f ′ are derivable at x = 0
(D)  f is continuous at x = 0 but f ′ is not ì p 3p ü ì 2p 5p ü
(A)  í , ý (B) 
í , ý
î 2 4 þ î 3 3 þ
211. If f(x) = (x - x0) φ(x) and φ(x) is continuous at x = 0, then f ′(x0)
is equal to ì 3p 7p ü ì 5p 7p ü
(C)  í , ý í ,
(D)  ý
(A)  φ ′(x0)  (B) φ (x0) (C) 
x0φ (x0) (D) 
None of these î4 4 þ î4 3 þ

212. If f(x) = {| x | - | x - 1 |}2, then f ′(x) equals 219. The value of the derivative of |x - 1| + |x - 3| at x = 2 is
(A)  0 for all x (A)  -2 (B) 0 (C) 2 (D)  None of these
(B) 2{| x | - | x - 1 |}
220. g: R → R, g(x) = cos-1[sin f(x)] has exactly two elements in
ì0 for x < 0 and for x > 1 range set. Then
(C)  í
î4 (2 x - 1) for 0 < x < 1 (A)  f(x) must be discontinuous function
(B)  f(x) may be continuous function
ì0 for x < 0
(D)  í (C)  It’s not possible to have such a function
î4 (2 x - 1) for x > 0 (D)  f(x) is discontinuous at finite points only

213. If a and b be the roots of the equation (ax2 + bx + 1) = 0, then æ cos x - cos a ö
221.   lim ç ÷ is equal to
x ® a è cot x - cot a ø
lim (1 + ax2 + bx + c)1/(x-a) is
x ®a
sin3 a cosec3a
(A)  a (a - b) (B)  ln |a (a - b)| (A) sin3a (B) cosec3a (C)  (D) 
2 2
(C)  ea(a - b) (D) 
None of these
222. f(x + y) = f(x) ⋅ f(y) ∀x and y. If f(3) = 3 and f ′(0) = 11, then
214. If f(x) is differentiable and strictly increasing function and f ′(3)  is given by
f ( x2 ) - f ( x) (A) 33 (B) 28 (C) 44 (D) 68
f ′(0) ≠ 0, then the value of lim is
x ®0 f ( x ) - f (0 )
cosecx
æ 1+ tan x ö
(A) 1 (B) 0 (C)  -1 (D) 2 223.   lim ç ÷ is equal to
x ® 0 è 1+ sin x ø

ì1, x ³1
ï 1
(A)  (B) 1 (C) 2 (D) 
e
ï1 1 1 e
215. The function f ( x ) = í 2 , < x < , n = 2 , 3,  .
ïn n n -1
224. Let f : R → R be a differentiable function and f(1) = 4. Then the
ï0 , x =0
î f(x)
2t
(A)  Is discontinuous at infinitely many points value of lim
x ®1 ò x -1
dt is
4
(B)  Is continuous every where
1 (A) 8f ′(1) (B) 4f ′(1) (C) 2f ′(1) (D) 
f ′(1)
(C)  Is discontinuous only at x =
n
225. The value of f(0), so that the function
(D)  None of these
a2 - ax + x 2 - a2 + ax + x 2
ì| x | f (x) = becomes continuous
ï , x ¹0 a+ x - a- x
216. If f(x) = 1 + x - [x], g(x) = í x . If h(x) = g(f(x)), then
ïî0 , for all x is given by
x =0
h′(1) and h′(-1) are (A) a (B)  - a (C)  a3/2 (D)  -a3/2

Mathematical Problem Book for JEE.indb 841 07-06-2018 13:12:58


842 Mathematics Problem Book for JEE

lncos x
æ xö ì
226. The lim ç 2 sin2 ÷ is equal to ï a(1- x sin x ) + b cos x + 5
x ®0 è 2ø ï x <0
ï x2
(A)  Does not exist (B) 1 (C) 1/2 (D) 2 ï
5. Let f ( x ) = í 3 x = 0 . If f is contin-
ï 1
ïé æ
Practice Exercise 2 ï ê1+ ç cx + dx 3 öù x
÷ú x >0
ï êë çè x 2 ÷ø úû
î
Single/Multiple Correct Choice Type Questions uous at x = 0, then
(A)  a = -1 (B)  b = -4 (C) 
c = 0 (D) 
d = loge5
 1 32 1
1. Let Sn = lim  6 + 6 +  + 6  and 6. If x cos a + y sina = x cos b + y sin b = 2 lim (an + b n )1/ n (where
n→∞  n n n  n ®¥

æ 1 32 ( n - 1)5 ö÷ . Then which of the following x2 + y2 ≠ 0 and 0 < b < a), then
Tn = lim ç 6 + 6 +  + (A) cos a + cos b = cos a cos b is a parabola
n ®¥ ç n n n6 ÷ø
è (B)  cos a + cos b = sin a + sin b is a straight line
is/are true? (C)  cos a + cos b = sin a sin b are pair of line
(D)  cos a cos b + sin a sin b  = 0 is a circle
1+ 1 æ x ö
2x
(A)  Sn ® (B)  ( Sn + Tn ) <
6 3 7. f(x) = ç ÷ , then
è2+ x ø
1 1- (A)  lim f(x) = - 4 (B) 
lim f(x) = 2
(C)  ( Sn + Tn ) > (D)  Tn ® x® ¥ x® ¥
3 6
 e x + e2 x + e3 x 
ln  1
3  (C)  lim f(x) = e-4 (D)  lim f(x) =
2 1/ x   9
2. Let f ( x ) = | x |, g(n, x ) = sin[p ([n] + [n] ) ] h (x) =
and . x ®¥ x ®1
x
 e x + e2 x + e3 x  8. Which of the following is/are true?
ln  
 3      (A) If lim {f(x) + g(x)} exists, then both lim f(x) and lim g(x) exist.
= sin[p ([n] + [n]2 )1/ x ] h( x ) = . Then (where [.] represent greatest x®a x®a x®a
x
integer function)     (B) If lim f(x) and lim g(x) exist, then lim {f(x) + g(x)} exists.
x®a x®a x®a

(A)  nlim lim f { g[n, h( x )]} = 1


®¥ x ® 0     (C) If lim f(x) and lim g(x) exist, then lim f(x) g(x) exists.
x®a x®a x®a

(B)  lim lim f { g[n, h( x )]} does not exist


n ®¥ x ®0     (D) If lim {f(x) g(x)} exists, then both lim f(x) and lim g(x) exist.
x®a x®a x®a

(C)  lim lim g[n, h( x )] does not exist


n ®¥ x ®0 9. If x is a real number in [0, 1], then the value of
lim lim [1 + cos2m(n! px)] is
(D)  lim lim g[n, h( x )] = 1 m ®¥ n ®¥
n ®¥ x ® 0
(A)  1 if x ∉ Q (B)  2 if x ∉ Q
(C)  1 if x ∈ Q (D)  2 if x ∈ Q
æ 1ö æ 1ö
x tan ç ÷ ln ç ÷ f ( x ) dx 10. Which of the following limits tend to unity?
3. If f ( x ) = lim e
n ®¥
ènø ènø and ò 3 sin11 x cos x = g( x ) + C , then
sin(tan t ) sin(cos x )
(A)  lim (B)  lim
æp ö 3 t ®0 sin t x →p / 2 cos x
(A)  g ç ÷ =
è4ø 2 x2 æ 1- cos x ö
(C)  lim (D)  lim ç ÷
(B)  g(x)  is continuous for all x x ®0 x x ®p / 2 è x2 ø

æ p ö -15 ì 1
(C)  g ç ÷ = ïï x + 2 , x < 0
è4ø 8 11.  f (x) = í , identify the correct statement(s) ([ ]
ï2 x + 1 , x ³ 0
(D)  g (x)  is non-differentiable at infinitely many points ïî 3

denotes greatest integer function)
é 1ù
4.   lim x 5 ê 3 ú is (where [.] represents greatest integer function) (A)  lim [f(x)] = 0 (B) 
lim f ( x ) does not exist
x ®0 ëx û x® 0 x ®0

(A)  A non-zero real number (B)  A rational number é ù [f ( x )]


(C)  ê lim f ( x )ú exists (D)  lim does not exist
(C)  An integer (D) Zero ë x ® 0 û x® 0 x

Mathematical Problem Book for JEE.indb 842 07-06-2018 13:13:41


Chapter 19 | Limit, Continuity and Differentiability 843

12. Which of the following function(s) has/have removable dis- 1


continuity at x = 1? (A)  (B)  3 (C) 0 (D)  None of these
3
1 x2 - 1
(A)  f(x) =  (B) 
f(x) =  3 18. If tangent at C intersect extended PA at Q, then the area of
ln| x | x -1 ∆CPQ is
1
1-x x +1 - 2 x 1 ïì sin2 q (1+ q cot q ) ïü
(C)  f(x) =  2-2 (D)  f(x) =  (A)  ítanq - ý
x2 - x 2 ïî q - sinq ïþ
13. A function f(x) satisfies the relation f(x + y) = f(x) + f(y) + xy 1 ïì sin2 q (1+ q cot q ) ïü
(x + y) ∀ x, y ∈ R. If f ′(0) = -1, then (B)  ítanq + ý
2 ïî q - sinq ïþ
(A)  f(x) is a polynomial function.
(B)  f(x) is an exponential function. 1 ïì sin2 q (1- q cot q ) ïü
(C)  f(x) is twice differentiable for all x ∈ R. (C)  ítanq + ý
2 îï q - sinq þï
(D)  f ′(3) = 8.
1 ïì sin2 q (1+ q cot q ) ïü
x
(D)  ítanq - ý
14. Let f(x) =  ò | t + 1| dt . Then 2 îï q - sinq þï
-2

(A)  f(x) is continuous in [-1, 1]. Area( ∆CPQ )


19. The value of lim+ is
(B)  f(x) is differentiable in [-1, 1]. q →0 sin2 q
(C)  f ′(x) is continuous in [-1, 1]. 1
(D)  f ′(x) is differentiable in [-1, 1]. (A)  (B) 3 (C) 0 (D)  Not defined
3
[x] +1 é 5ö æ1 ù n
15. f(x)  =  for f : ê0 , ÷ → ç , 3ú , where [ . ] represents æ xö
{x} +1 ë 2ø è2 û Paragraph for Questions 21–23: Let f(x) = lim ç cos ÷÷ , g(x) =
n ® ¥ç n
greatest integer function and { . } represents fractional part of è ø
lim (1- x + x n e )n . Now, consider the function y = h(x), where h(x)
x , then which of the following is true. n ®¥

(A) f(x) is injective discontinuous function. = tan-1 [g-1 f -1(x)].


(B)  f(x) is surjective non-differentiable function. ln[f ( x )]
20.   lim + is equal to
(C) min [ lim f ( x ), lim f ( x )] = f (1). x® 0 ln[ g( x )]
x ®1- x ®1+

(D)  max (x values of point of discontinuity) = f(1). 1 1


(A)  (B)  - (C) 0 (D) 1
16. If f(x) = 0 for x < 0 and f(x) is differentiable at x = 0, then for 2 2
x > 0, f(x) may be 21. Domain of the function y = h(x) is
(A)  x2 (B)  x (C) sin x (D)  -x3/2 (A)  (0, ∞) (B)  R (C)  (0, 1) (D)  [0, 1]
22. Range of the function y = h(x) is
Comprehension Type Questions
æ pö æ p ö æ p pö
Paragraph for Questions 18–20: A tangent line is drawn to a (A)  ç 0 , ÷ (B)  ç - , 0 ÷ (C)  R (D)  ç- , ÷
è 2ø è 2 ø è 2 2ø
­circle of radius unity at the point A and a segment AB is laid off
whose length is equal to that of the arc AC, a straight line BC is Paragraph for Questions 24–26: Let f(x) = max {a, b, c}, where
drawn to intersect the extension of the diameter AO at the point P. a n | sin x | + a - n | cos x |
a = lim lim+
B
n → ∞ a →1 a n + a -n
C
a - n | sin x | + a n | cos x |
b = lim lim-
n → ∞ a →1 a n + a -n
p  p 2p (n - 1)p 
θ c = lim 1+ cos + cos +  + cos . Then
P O D A
n→∞ 4 n  2n 2n 2n 
23. The value of a is
1
(A)  2 |sin x| (B) |cos x| (C) |sin x| (D) 
2
1
24. The value of b + c - is
Figure 19.26 2
(A) |cos x| (B)  2 | cos x| - 1
17. The value of lim+ PA is (C) |sin x| + 1 (D) |sin x| + |cos x|
q ®0

Mathematical Problem Book for JEE.indb 843 07-06-2018 13:14:03


844 Mathematics Problem Book for JEE

25. Range of f(x) is 31. Match the following:


é1 ù é 1 ù 1
(A)  [0, 1] (B)  ê , 1ú (C)  ê , 1ú (D)  éê , 2 ùú List I List II
ë2 û ë 2 û ë2 û
Let f: R → R be a differentiable function and f(1) = 1,
(A) 
Paragraph for Questions 27–29: Let a function is defined as x2
( f (t ) - t ) (p)  0
x ®1 ò
f ′(1) = 3. Then the value of lim dt is
ì 1 ( x - 1)2
ïï[ x ], -2 £ x £ - 1

f (x) = í 2 , where [ . ] denotes greatest integer n


ï2 x 2 - 1, - 1 < x £ 2 æ 1+ n 4 ö
(B)  lim ç ÷÷ is equal to (q)  -1
ïî 2 n®¥ç
è 2 ø
function.
2x
(C) If f(x) =  lim . tan-1 (nx), x > 0, then lim+ [f(x) -1]
26. The number of points of discontinuity of f(x) is p
n ®¥ x ®0 (r) 2
(A) 1 (B) 2 (C) 3 (D)  None of these is {where [.] represents greatest integer function}

27. The function f(x - 1)  is discontinuous at the points é n 1ù


(D)  nlim ê å r ú  = 
1 1 1 ®¥
ë r =1 2 û (s) 1
(A)  -1, - (B)  - , 1 (C) 0, (D)  0, 1
2 2 2 (where [.] denotes the greatest integer function)
28. Number of points where |f(x)| is not differentiable is (t) 4
(A) 1 (B) 2 (C) 3 (D) 4
32. Match the following:

List I List II
Matrix Match Type Questions
Let f : R → R be such that f(a) = 1, f′(a) = 2
(A) 
29. Match the following: 1/ x
æ f 2 (a + x ) ö (p)  0
List I List II and lim çç ÷÷  = ek. Then k =
x® 0
è f (a) ø
æ n 4 öæ n 5 ö
ç å x ÷ç å x ÷ cos[tan-1(tan x )]
4 lim+ =
(A)  If lim è nx =1 ø è nx =1 ø = , then t can be (p) 6 (B)  x®
p
x-
p (q)  1
n ®¥ æ ö æ ö 5
ç å x ÷ç å x ÷
t 9 -t 2 2
è x =1 ø è x =1 ø
sin(cos x + 1)
lim =
(B)  If m be the slope of tangent to the curve xy = yx (C)  x ®p æxö (r)  4
(q) 2 cos ç ÷
at (e, e), then (3 - m) can be è2ø
æ 1 1ö xe sin x - e x sin-1(sin x )
(C)  If f : R - {0} ® R , f ( x )f ( y ) = f ( xy ) + 3 ç + ÷ , = (s)  3
èx yø (D)  xlim
(r) 3 ®0 sin2 x - x sin x
æ 1ö
then 2 f ç - ÷ can be (t)   Does not exist
è 2ø

{ }
(D)  If lim ( 2 + 1)n !+ K = 1 , then K can be (where {.} (s) 4
n ®¥
Integer Type Questions
is a fractional part function) 33. f : R → R be a twice differentiable function satisfying
f ′′(x) - 5f ′(x) + 6f (x) ≥ 0 ∀ x ≥ 0, if f (0) = 1, f ′(0) = 0, If f (x) satisfies
(t) 7 f (x) ≥ ah(bx) - bh(ax), ∀ x ≥ 0, then find (a + b) h(0).
1
30. If lim ( x 4 + ax 3 + 3 x 2 + bx + 2 - x 4 + 2 x 3 - cx 2 + 3 x - d ) = 4 , (1+ a3 ) + 8e x
x →∞ 34. If a and b are positive numbers and lim 1
= 2,
x ®0
2
then match the values of a, b, c and d. 1+ ( 2 + b + b )e x
then find the value of a2 + b2.
List I List II 35. Let f (x)be a differentiable function such that f ′(x) + f (x) = 4xe-x
(A)  a (p)  5 n
- pp ep
· sin 2x and f (0) = 0. If lim å f (kp ) = p , then find value of p.
(B)  b (q)  0 n ®¥
k =1 (e - 1)2
(C)  c (r)  1 1
tan x
(D)  d (s)  2 36. Let f ( x ) = and lim ([f ( x )] + x 2 ) { f ( x )} = e l . Then find λ.
x x →0
1 (where [.] and {.} denotes greatest integer and fractional part
(t) 
2 function respectively)

Mathematical Problem Book for JEE.indb 844 07-06-2018 13:14:25


Chapter 19 | Limit, Continuity and Differentiability 845

Answer Key
Practice Exercise 1
 1. (B)  2. (A)  3. (B)  4. (D)   5.  (A)  6. (B)   7.  (C)  8. (B)  9. (C)  10.  (A)
11.  (A) 12.  (C) 13.  (C) 14.  (C)  15. (C) 16.  (B) 17.  (B) 18.  (A) 19.  (B) 20.  (C) 
21.  (B) 22.  (A)  23.  (A) 24.  (D) 25.  (C) 26.  (B)  27.  (C) 28.  (A)  29.  (A) 30.  (C)
31.  (C) 32.  (D) 33.  (D) 34.  (C) 35.  (B) 36.  (D) 37.  (B) 38.  (B) 39.  (C) 40.  (B)
41.  (C) 42.  (D) 43.  (D) 44.  (B) 45.  (D) 46.  (D) 47.  (C) 48.  (B) 49.  (B) 50.  (D)
51.  (C) 52.  (A) 53.  (C) 54.  (B) 55.  (B) 56.  (C)  57.  (A)  58.  (A) 59.  (B) 60.  (C) 
61.  (A)  62.  (A) 63.  (B) 64.  (D) 65.  (B) 66.  (B) 67.  (D) 68.  (B) 69.  (B) 70.  (A)
71.  (A) 72.  (B) 73.  (D) 74.  (B) 75.  (C) 76.  (C) 77.  (B) 78.  (C) 79.  (C) 80.  (D)
81.  (C) 82.  (B) 83.  (D) 84.  (C) 85.  (C) 86.  (A) 87.  (B) 88.  (B) 89.  (B) 90.  (B)
91.  (C) 92.  (B) 93.  (C) 94.  (D) 95.  (B) 96.  (B) 97.  (B) 98.  (C) 99.  (A) 100.  (C)
101.  (A) 102.  (A) 103.  (D) 104.  (B) 105.  (B) 106.  (B) 107.  (B) 108.  (D) 109.  (D) 110.  (D)
111.  (D) 112.  (D) 113.  (A) 114.  (C) 115.  (A) 116.  (C) 117.  (A) 118.  (C) 119.  (A) 120.  (C)
121.  (A) 122.  (A) 123.  (C) 124.  (A) 125.  (D) 126.  (C) 127.  (B) 128.  (B) 129.  (D) 130.  (C)
131.  (C) 132.  (A) 133.  (A) 134.  (D) 135.  (B) 136.  (B) 137.  (C) 138.  (D) 139.  (C) 140.  (D)
141.  (B) 142.  (B), (D) 143.  (D) 144.  (D) 145.  (B) 146.  (D) 147.  (A), (C ), (D) 148.  (D) 149.  (A) 150.  (A)
151.  (B) 152.  (B) 153.  (D) 154.  (D) 155.  (B) 156.  (D) 157.  (C) 158.  (C) 159.  (A) 160.  (C)
161.  (B) 162.  (D) 163.  (B) 164.  (B) 165.  (D) 166.  (C) 167.  (A) 168.  (D) 169.  (C) 170.  (B)
171.  (B) 172.  (B) 173.  (B) 174.  (D) 175.  (D) 176.  (D) 177.  (D) 178.  (D) 179.  (C) 180.  (C)
181.  (D) 182.  (C) 183.  (C) 184.  (A) 185.  (B) 186.  (C) 187.  (C) 188.  (B) 189.  (D) 190.  (C)
191.  (D) 192.  (B) 193.  (A)  194.  (C) 195.  (D) 196.  (C)  197.  (D) 198.  (D) 199.  (C) 200.  (A)
201.  (D) 202.  (B)  203.  (C) 204.  (B) 205.  (C) 206.  (B) 207.  (C) 208.  (B) 209.  (C)  210.  (A)
211.  (B)  212.  (C) 213.  (C)  214.  (C)  215.  (A) 216.  (A) 217.  (D) 218.  (C) 219.  (B) 220.  (A)
221.  (A) 222.  (A) 223.  (B) 224.  (A) 225.  (B) 226.  (B)

Practice Exercise 2
 1. (A), (C), (D)  2. (A), (C)  3. (C), (D)  4. (B), (C), (D)  5. (A), (B), (C)  6. (A), (B), (C), (D)
 7. (C)  8. (B), (C)  9. (A), (D) 10.  (A), (B) 11.  (A), (B) 12.  (B), (D)
13.  (A), (C), (D) 14.  (A), (B), (C), (D) 15.  (A), (B), (D) 16.  (A), (D) 17.  (B) 18.  (C)
19.  (D) 20.  (B) 21.  (C) 22.  (D) 23.  (C) 24.  (A)
25.  (C) 26.  (B) 27.  (C) 28.  (C) 29.  (A) → (q), (t); (B)→ (q), (s); (C)→ (q), (r); (D)→ (p), (q), (s)
30.  (A)→ (s); (B)→ (p), (q), (r), (s), (t); (C)- (p); (D)→ (p), (q), (r), (s), (t) 31.  (A)→ (t), (B)→ (r), (C)→ (q), (D)→ (p)
32.  (A)→ (r), (B)→ (q), (C)→ (p), (D)→ (q) 33. 5 34. 2 35. 2 36. 3

Solutions

Practice Exercise 1 (2 x - 3)( x - 1) ´ ( x + 1) -1 -1


2.  lim = =
1. Here f (0) = 0 x ®1 ( x - 1)(2 x + 3) ´ ( x + 1) 5 . 2 10
Since
3. Applying L’Hospital’s rule,
1 1
-1 £ sin £ 1Þ - | x | £ x sin £ | x |
x x 1 f ¢(9 ) 4
× f ¢( x )
We know that lim x = 0 and lim - x = 0. 2 f (x) f (9) 3
x ®0 x →0 lim = = =4
x ®9 1 1 1
In this way lim f ( x ) = 0. 3
x →0 2 x 9

Mathematical Problem Book for JEE.indb 845 07-06-2018 13:14:30


846 Mathematics Problem Book for JEE

2
x x æ q qö
4. Since lim- = -1 and lim+ = 1, hence limit does not exist. ç cos - sin ÷
x ®0 x x ®0 x 1- sinq è 2 2ø
11.   lim = lim =0
q ®p / 2 cosq q ®p / 2 æ q q öæ q qö
x +h - x ( x + h )2 - ( x )2 1 ç cos - sin ÷ç cos + sin ÷
5.   lim = lim = è 2 2 øè 2 2ø
h®0 h h ® 0 h( x + h + x ) 2 x
ì 2 tan 2 x ü
Alternate solution: Apply L’Hospital’s rule, ï - 1ï
tan 2 x - x 1
12. lim = lim í 2 x ý=
x +h - x 1 1 x ® 0 3 x - sin x x ®0
ï 3- sin x ï 2
lim = lim = î x þ
h®0 h h®0 2 x + h 2 x
Alternate solution: Apply L’Hospital‘s rule,
2x -1 2 x log 2
6.   lim = lim = 2 log 2 = log
g4
x → 0 (1+ x )1/ 2
- 1 x → 0 1 (1+ x )-1/ 2 tan 2 x - x 2 sec2 2 x - 1 2 - 1 1
2 lim = lim = =
x ®0 3 x - sin x x ®0 3 - cos x 3 -1 2
 f (x) f ′( x ) 
As lim = lim   p 
 x → a g( x ) x → a g′( x )   x- 
13. lim 2  2  = -2
x -1 1 x →p / 2  p 
7.  lim =-  sin  2 - x  
x ®1 ( x - 1)(2 x - 5) 3
Alternate solution: Apply L’Hospital’s rule, Alternate solution: Apply L’Hospital’s rule.
x -1 1 1 2x - p 2
lim = =- lim = lim = -2
x →1 2x2 - 7x + 5 4 x - 7 3 x →p / 2 cos x x →p / 2 - sin x

sin x 1 1 14. Apply L’Hospital’s rule,


8.   lim Let x = or y = .
x →∞ x y x
 (a + h)2 sin(a + h) - a2 sin a
sin x 1 1 lim
lim Let x = or y = . Then h®0 h
x →∞ x y x
2(a + h)sin(a + h) + (a + h)2 cos(a + h)
x→∞⇒y→0 = lim
h®0 1
Hence,
= 2a sin a + a2 cos a
æ sin x ö æ 1ö 1
lim ç ÷ = lim ç y × sin ÷ = ylim y ´ lim sin = 0 ´ = 0
x ®¥ è x ø y ®0 è y ø ®0 y ®0 y 15. Apply L’Hospital’s rule,

(1+ x )1/ 2 - (1- x)1/ 2 1 1


2 ´ 9 sin2 3 x lim = lim + =1
9.   lim = 18 x ®0 x x ® 0 2 1+ x 2 1+ x
x ®0 (3 x )2
a + 2x - 3x
16.   lim
sina - cos a x ®a 3a + x - 2 x
10. lim
a ®p / 4 a -p / 4
a + 2x - 3x a + 2x + 3x 3a + x + 2 x
= lim ´ ´
ì æ 1 1 öü x ®a 3a + x - 2 x a + 2x + 3x 3a + x + 2 x
ï 2 ç sina × - cos a × ÷ï
ï è 2 2 øï
= lim í ý 3a + x + 2 x 2
a ®p / 4
ï æ pö ï = lim =
ç a- ÷ x ®a 3( a + 2 x + 3 x ) 3 3
ïî è 4ø ïþ

æ pö x ⋅ (2 x - 1) 2x -1 x2
sin ç a - ÷ 17.   lim = lim ⋅
è 4ø x → 0 1 - cos x x →0 x 1- cos x
= 2 lim = 2 ´1 = 2
a ® p/ 4 æ pö
ça - ÷ x2
è 4ø = log 2 × lim = (log 2) × 2 = 2 log 2 = log 4
x ®0 x
Alternate solution: Apply L’Hospital’s rule, 2 sin2
2
sin a - cos a cos a + sin a 1 1 tan x - sin x sin x - sin x cos x
lim = lim = + = 2 18.   xlim = lim
a ®p / 4 a - (p / 4 ) a ®p / 4 1 2 2 ®0 x3 x ®0 x 3 cos x

Mathematical Problem Book for JEE.indb 846 07-06-2018 13:14:54


Chapter 19 | Limit, Continuity and Differentiability 847

æ xö é x úù æ x3 x5 ö æ x2 x3 x4 ö
sin x ç 2 sin2 ÷ ê sin2 çç x - + -  ÷÷ çç - x - - - -  ÷÷
è 2ø sin x 2 2 × 1ú = 1 3 ! 5 ! 2 3 4
= lim = lim ê × × = lim è ø + lim è ø
x ®0 x 3 cos x x ®0 ê x cos x æ x ö2 4 ú 2 x ®0 x2 x ®0 x2
ê ç ÷ ú
êë è2ø úû æ x3 x5 x2 x3 ö
çç∵ sin x = x - + -  and log(1- x ) = - x - - -  ÷÷
19. Let y = xx. Then log y = x log x. è 3! 5! 2 3 ø
Hence,
-x2 æ 1 1 ö x4
x
- x3 ç + ÷ - 
lim log y = lim x log x = 0 = log 1Þ lim x = 1 2 è3 ! 3ø 4 1
y ®0 x ®0 x ®0 = lim =-
x ®0 x2 2
40 5
æ 1ö æ 1ö æ sin x ö
40
(2 x + 1) ( 4 x - 1) ç2 + ÷ ç4 - ÷
5
æ x + sin x ö ç 1+ x ÷
x xø
20.   lim = lim è ø è 26.   lim ç ÷ = xlim ç cos x ÷ = lim 1 = 1
(2 x + 3)45 45 x ®¥ è x - cos x ø ®¥
x ®¥ x ®¥
æ 3ö çç 1- ÷÷ x ®¥
ç2 + ÷ è x ø
è xø 
2 ×440 5  sin x cos x 
= = 32 As xlim
→∞ x
and lim
x →∞ x
both are equal to 0 
245  
1/ x
1 æ 1+ tan x ö
21. Let tan-1 2 x = q . Then x = tanq and as x → 0, q → 0. So, 27. Given limit = lim ç ÷
2 x ® 0 è 1- tan x ø

1 {(1+ tan x )1/ tan x }(tan x )/ x e


tanq 1
x = lim = -1 = e 2
lim = lim 2 = x → 0 {(1 - tan x )1/ tan x }(tan x )/ x e
- 1
x → 0 tan 2 x q →0 q 2
2

22. Apply L’Hospital’s rule, æ 1+ 5 x 2 ö


1/ x 2 lim [(1+ 5 x 2 )1/ 5 x ]5 e5
28.   lim çç ÷ = x ®0
= = e2
x ® 0 1+ 3 x 2 ÷ e3
2
2
2 xe x + sin x 2 sin x 1 3 è ø lim [(1+ 3 x 2 )1/ 3 x ]3
lim = lim e x + lim = 1+ = x ®0
x ®0 2x x ® 0 x ® 0 2x 2 2
[As lim (1 + x )1/ x = e ]
x →0
1
23. Let f ( x ) = log x Þ f ¢(x) =
x 4 x2 + 5x + 8 4( -1/ h)2 + 5( -1/ h) + 8
29.   lim = lim
Therefore, given function = f ′(a) + kf ′(e) = 1 x ®-¥ 4x +5 h®0 4( -1/ h) + 5
1 k æ a - 1ö
Þ + = 1Þ k = e ç ÷ (1/ h) 4 - 5h + 8h2 4 1
a e è a ø = lim = =-
h®0 (1/ h)( -4 + 5h) -4 2
Alternate solution: Apply L’Hospital’s rule to find both the
limits. x tan 2 x - 2 x tan x
30.   lim
x ®0 (1- cos 2 x )2
1 1 1 k æ a - 1ö
lim + k lim = 1 + = 1Þ k = e ç ÷
x ®0 a + x x ®e x a e è a ø x (tan 2 x - 2 tan x ) 1 x (tan 2 x - 2 tan x )
= lim 2 2
= lim
x ®0 (2 sin x ) x ® 0 4 sin4 x
1 1 1 k æ a - 1ö
lim + k lim = 1 + = 1Þ k = e ç ÷
x ®0 a + x x ®e x a e è a ø ïìæ 1 2 ö æ x3 2 5 ö ïü
x íç 2 x + (2 x )3 + (2 x )5 +  ÷ - 2 çç x + + x +  ÷÷ ý
1 îï è 3 15 ø è 3 15 ø þï
= lim
1 x ®0 4 4
(1- cos 2 x ) æ x 2
x 4 ö
2 sin x x 4 çç 1- + +  ÷÷
24.   lim = lim 3 ! 5 !
x ®0 x x ®0 x è ø
sin x sin x 1 æ8 2ö 2 1
So, lim+ = 1 and lim- = -1. = ×ç - ÷ = =
x →0 x x →0 x 4 è3 3ø 4 2
Hence, limit does not exist. -
5x
é x +2 ù x +2
x
25. Apply L’Hospital’s rule, we get æ x +2-5ö êæ 1- 5 ö -5 ú
1 31.   lim ç ÷ = lim ç ÷ = e -5
1 - sin x -   x ®¥ è x + 2 ø x ®¥ êè x +2ø ú
cos x - êë úû
lim 1- x = lim (1- x )2 = - 1
x ®0 2x x ®0 2 2  
Alternate solution:  -5 x -5 
 As xlim = lim = - 5
sin x + log(1- x) →∞ x + 2 x →∞ 2
lim  1+ 
x ®0 x2 x

Mathematical Problem Book for JEE.indb 847 07-06-2018 13:15:20


848 Mathematics Problem Book for JEE

2x
sin nx æ tan x ö  a b
32.   lim n × lim ç (a - n)n - ÷=0 38. Since, lim  1+ + 2  = e2
x ®0 nx x ®0 è x ø x →∞  x x 
1 Hence,
Þ n((a - n)n - 1) = 0 Þ (a - n)n = 1Þ a = n + 2( ax + b )
n
é x ù 2
x

f (2h + 2 + h2 ) - f (2) f ¢(2h + 2 + h2 )(2 + 2h) ê æ ax + b ö ax + b ú


lim ç 1+ ÷ = e2
33.   lim 2
= lim x ®¥ êè x2 ø ú
h ® 0 f ( h - h + 1) - f (1) h ® 0 f ¢( h - h2 + 1)(1- 2h) êë úû
6´2 2( ax + b )
2(ax + b )
= =3 Þ lim e x = e 2 Þ lim = 2 ⇒ 2a = 2 ⇒ a = 1
4 ´1 x ®¥ x ®¥ x
e x - e- x Thus, a =1 and b ∈ R.
34.   y = xlim
→0 sin x -1
39. Using L’Hospital’s rule, lim =1
é x x2 ù é x x2 ù q→
p - cosec 2q
ê1+ + + ú - ê1- + - ú 2
1! 2 ! û ë 1! 2 !
Þ y = lim ë û æ -4 ö
( 3 x -1)
x ®0 sin x é æ x -1ö ù çè x -1÷ø
3 x -1 ç ÷
æ 4 ö ê æ ( - 4 ) öè - 4 ø ú
é x x3 x5 ù 40.   lim ç 1- ÷ = lim êç 1+ ÷ ú
2ê + + + ú x ®¥ è x - 1ø x ®¥ è x - 1ø
ê ú
1 ! 3 ! 5 ! ë û
Þ y = lim ë û
x ®0 sin x   1 
 -4  3 - x  
 
é x2 x4 ù   1 
2 ê1+ + + ú lim   1- x  
3! 4 ! =e x →∞   = e -12
Þ y = lim ë û
x ®0 sin x
é e x - e sin x ù æ 0 ö
x 41.   lim ê ú , ç form ÷
x ® 0 ê x - sin x ú è 0 ø
ë û
é x2 ù
lim 2 ê1+ + ú Using L’Hospital’s rule three times, then
x ®0
ë 2 ! û Þ y = 2 =2
Þy= e x - e sin x × cos x e x - e sin x cos2 x + sin x × e sin x
sin x 1 lim = lim
lim
x ®0 x    x ®0 1- cos x x ®0 sin x
Alternate solution: Applying L’Hospital’s rule, e x - e sin x × cos3 x + e sin x 2 cos x sin x + e sin x × cos x sin x + e sin x × cos x
= lim
1 x ®0 cos x
e0 + 0
e x - e- x e x + e- x e 1+ 1 =1
lim = lim = = =2
x ® 0 sin x x ® 0 cos x cos 0 1
x2 + 3x + 2 x2 + 2x + x + 2
42.   lim 2
= lim 2
35. Using L’Hospital’s rule, x →-1 x + 4 x + 3 x →-1 x + 3 x + x + 3

3 1 ( x + 1)( x + 2) x +2 1
3× + 3× = lim = lim =
3 cos x + 3 sin x 2 2= 1 x →-1 ( x + 1)( x + 3) x →-1 x + 3 2
lim =
x ®p / 6 6 6 3
1
2
ö 2 æ sin x 43.   lim log(1+ x ) = lim 2 log(1+ x) x
-2 sin ç ÷ x ®0 x x ®0
cos(sin x ) - 1 è 2 ø 1 -1
36.   lim = lim = -2 × =
x ®0 x 2 x ®0 x 2
4 2  1 
= lim 2 loge e = 2 As lim (1+ x ) x = loge e = 1
x →0 x →0
1  
1 1
n é 3n ù n
37.   lim (3 + 4n )n = lim ( 4 n ) n ê n + 1ú x +1 x +1
n ®¥ n ®¥ êë 4 úû æ 3x - 4 ö 3 æ 3x + 2 - 6 ö 3
44.   lim ç ÷ = lim ç ÷
1/ n 1/n x ®¥ è 3 x + 2 ø x ®¥ è 3 x + 2 ø
é ù é ù
ê ú ê ú -6 x +1
1 1 ×
= lim 4 ê1+ ú = 4 lim ê1+ ú x +1 é 3 x +2 ù 3 x +2 3
n ®¥ ê 4ö ú n ®¥ ê 4ö ú æ 6 ö 3 æ 6 ö -6 ú
= lim êç 1-
n n
æ
ê ç ÷ ú æ
ê ç ÷ ú = lim ç 1- ÷ ÷
x ®¥ è 3x + 2 ø x ®¥ êè 3x + 2 ø ú
êë è 3 ø úû êë è 3 ø úû êë úû
0 -2( x +1)
é 1ù  -2( x + 1) -2 
= 4 ê1+ ú = 4 ´(1)0 = 4 ´ 1 = 4 = lim e 3 x +2 = e -2 / 3 As lim = 
ë ¥û x ®¥  x →∞ 3 x + 2 3

Mathematical Problem Book for JEE.indb 848 07-06-2018 13:15:49


Chapter 19 | Limit, Continuity and Differentiability 849

é æ 1ö æ 4 öù 1
ê x ç 1+ x ÷ x ç 3 + x ÷ ú +2(27 x log 27 - 9 x log 9 - 3 x log 3) ( - sin x )
45.   ( x + 1)(3 x + 4 ) è ø è øú 2 4 + cos x
lim = lim ê
x ®¥ x 2 ( x - 8) x ®¥ ê æ 8 ö ú cos x
êë x 3 ç1- ÷ úû
è x ø Applying limit, we have

æ æ 1öæ 4öö 2[(log 27)2 - (log 9 )2 - (log 3)2 ]
ç 1 ç 1+ ÷ ç 3 + ÷ ÷ = 4 +1
è x ø è x ø÷ 1
= lim ç =0
x ®¥ ç x æ 8ö ÷
ç ç 1 - ÷ ÷ é 9(log 3)2 - 4(log 3)2 - (log 3)2 ù
è è xø ø = 2ê ú 5
êë 1 úû
46. In closed interval of x = 0 at right hand side [x] = 0 and at left
hand side [x] = -1. Also [0] = 0. Therefore, function is defined as = 5 .8(log 3)2
ì sin[ x ]
ï ( -1 £ x < 0) = 8 5 (log 3)2
f ( x) = í [x]
ï0 (0 £ x < 1)
î xn 1
51.   lim = lim =1
Hence, n ®¥ næ 1 ö n ®¥ æ 1 ö
x ç 1+ n ÷ ç 1+ n ÷
sin[ x ] è x ø è x ø
left hand limit = lim f ( x ) = lim
x ®0 - x ®0 - [x]
sin( -1) 1 éæ 1 ö æ 1 1 ö æ 1 1 ö
= = sin1 52.   lim
n®¥ 2 ëè
êç 1- 3 ÷ + ç 3 - 5 ÷ + ç 5 - 7 ÷ + 
-1 ø è ø è ø
Right hand limit = 0. Hence, limit does not exist. æ 1 1 öù
+ç -
 1  ç ( 2n - 1) ( 2n + 1) ÷÷ ú
è ø úû
(10 )n  - 1
 (10 )
n
1- (10 )n  1 1é 1 ù 1
47.   lim = lim =- = lim 1- =
n ®¥ 2 ê 2n + 1úû 2
n + 1 1 
n→∞ 1+ (10 ) x →∞ n +1  10 ë
(10 )  1+ n +1 
 10 
 x2 +1 
Hence, a = 1. 53. lim  -a x - b  = 0
x →∞  x + 1 
log(1+ x 3 ) 3 x 2 / (1+ x 3 )
48.   lim = lim
x ® 0 sin3 x x ® 0 3 sin2 x cos x x 2 (1 - a ) - x (a + b ) + 1- b
⇒ lim =0
x →∞ x +1
[By using L’Hospital’s rule]
Since the limit of the given expression is zero, therefore
é 1 æ x ö2 1 ù 1 1 degree of the polynomial in numerator must be less than
= lim ê ç ÷ × ú = × (1)2 × = 1 denominator. Hence,
x ® 0 ê 1+ x 3 è sin x ø cos x ú 1 + 0 1
ë û 1 - a = 0 and a + b = 0
⇒ a = 1 and b = -1
4q (tanq - sinq ) 4q sinq (1- cosq )
49.   lim = lim f(x)
q ®0 (1- cos 2q )2 q ® 0 4 sin4 q cosq
ò 4t 3dt
4[f ( x )]3 ´ f ’( x )
6
 q  2 sin2 q / 2 54.   lim (0 / 0 form) = lim
= lim  x ®2 x -2 x ®2 1
q → 0  sin q 
 sin2 q cos q
= 4(f(2))3 × f ′(2) = 18
2
2 sin q / 2 1 é 1 2 n ù
= lim 55.   lim ê + + +
q → 0 [2 sin(q / 2)cos(q / 2)]2 cosq n ®¥ ë 1- n2 1- n2 1- n2 úû
1 1 1
= lim = ån 1 n2 + n 1
q ®0 2 cos2 (q / 2) × cosq 2 lim 2
= lim =-
n ®¥ 1- n 2 n®¥ 1- n2 2
50. Applying L’Hospital’s Rule, we have
12 + 22 + 32 +  + n2 Sn2
56. Given limit = lim = lim
27 x log 27 - 9 x log 9 - 3 x log 3 n ®¥ 1+ n 3 n ®¥ 1+ n3
lim
x →0 1
- ( - sin x ) æ 1 öæ 1ö
1+ 2+ ÷
2 4 + cos x 1 n(n + 1)(2n + 1) 1 çè n ÷ø çè nø
= lim = lim
2(27 x log 27 - 9 x log 9 - 3 x log 3) 4 + cos x n ®¥ 6 1+ n3 n ®¥ 6 æ 1 ö
Þ lim ç 3 + 1÷
x ®0 sin x èn ø
1 2 æ 1ö
Þ lim 2[27 x (log 27)2 - 9 x (log 9 )2 - 3 x (log 3)2 ] 4 + cos x = ×1× = ç ÷
x ®0 6 (1) è 3 ø

Mathematical Problem Book for JEE.indb 849 07-06-2018 13:16:14


850 Mathematics Problem Book for JEE

57. We have 1 - 2 + 3 - 4 + 5 - 6 +  - 2n
63.   lim
sn +1 - sn an +1 n ®¥
n2 + 1 + 4 n2 - 1
lim = lim =0
n ®¥ n n ®¥ n(n + 1) -2 -2
åk 2
f (x) = y = =
1+ 2 3
k =1

(Since n → ∞, numerator → a while denominator → ∞) ( x + 1)10 + ( x + 2)10 +  + ( x + 100 )10
64.   lim
58. We have x ®¥ x 10 + 1010
4 + 3an éæ 1 ö10 æ 2 ö10 æ 100 ö ù
10
an +1 = x 10 êç 1+ ÷ + ç 1+ ÷ +  + ç 1+
3 + 2an ÷ ú
êëè x ø è xø è x ø ûú
= lim = 100
4 + 3an x ®¥ é 1010 ù
Þ lim an +1 = lim x 10 ê1+ 10 ú
n ®¥ n ®¥ 3 + 2a
  n ë x û
4 + 3a
Þa= Þ 2a 2 = 4 Þ a = 2 65. We have,
  3 + 2a
1+ 2 + 3 +  + n
a¹- 2 lim
n ®¥ n2 + 100
because each an > 0, therefore lim an = a > 0.
n→∞
59. We know that æ 1ö
n2 ç 1+ ÷
n(n+ 1) è nø = 1
sin 2n A = lim = lim
cos A cos 2 A cos 4 A cos 2n -1 A = n ®¥ 2( n2 + 100 ) n ®¥ 2æ 100 ö 2
2n ç 1 + 2 ÷
2n sin A è n ø
x
Taking A = , we get x
2n
∫ cos t
2
dt
66.   lim 0
æ x ö æ x ö æxö æxö sin x x
x →0
cos ç n ÷ cos ç n -1 ÷ cos ç ÷ cos ç ÷ =
è2 ø è2 ø è4ø è 2 ø 2n sin æ x ö Applying L’Hospital’s rule, we get
ç n÷
è2 ø
x
ò cos t
2
d cos x 2
æxö æxö æ x ö æ x ö lim 0 = lim =1
Hence, lim cos ç ÷ cos ç ÷ cos ç n -1 ÷ cos ç n ÷ x ®0 x x ®0 1
n ®¥ 2
è ø 4
è ø è 2 ø è2 ø
sin x sin x ( x / 2n ) sin x 67. Here f(2) = 0
= lim = lim =
n ®¥ n æ x ö n®¥ x sin( x / 2n ) x lim f ( x ) = lim f (2 - h) = lim½2 - h - 2½= 0
2 sin ç n ÷ x ®2 - h®0 h®0
è2 ø
lim f ( x ) = lim f (2 + h) = lim ½2 + h - 2½= 0
é æ 1 ön ù x ®2 + h®0 h®0
ê1- ç ÷ ú Hence, it is continuous at x = 2.
1 1 1 1 1 êë è 2 ø úû
60.   y = lim + 2 + 3 +  + n = lim 68. f (p/2) = 3. Since f (x) is continuous at x = p/2, so
n ®¥ 2 2 2 2 n®¥ 2 æ 1- 1 ö
ç ÷  k cos x  p  k
è 2ø lim 
x →p / 2  p
 = f   ⇒ = 3 ⇒ k = 6
- 2x  2 2
 1
⇒ y = lim 1- n  = 1- 0 = 1
n→∞  2  69. Since limit of the function is a + b as x → 0, therefore to be
continuous at a function, its value must be
æ 1 2 3 nö
61.   lim ç 2 + 2 + 2 +  + 2 ÷ a + b at x = 0 ⇒ f(0) = a + b
n ®¥ è n n n n ø
70. For, f(x) to be continuous
n lim f ( x ) = f (2) = k
(n + 1)
æ 1+ 2 + 3 +  + n ö 2 x ®2
= lim ç ÷ = lim
n ®¥ è n2 ø n®¥ n
2
x 3 + x 2 - 16 x + 20
Þ k = lim
1 n +1 1  1 1 x ®2 ( x - 2)2
= lim = lim  1+  =
2 n→∞ n 2 n→∞  n  2 ( x 2 - 4 x + 4 ) ( x + 5)
= lim =7
1- n2 (1- n)(1+ n) 2(1- n) x ®2 ( x - 2)2
62.   lim = lim = lim
n ®¥ Sn n ®¥ 1 n ®¥ n 71. Here, lim f ( x ) = k , lim f ( x ) = -k and f(0) = k.
n(n + 1) x ®0 + x ®0 -
2
But f(x) is continuous at x = 0, therefore k must be zero.
æ1 ö
= lim 2 ç - 1÷ = 2(0 - 1) = -2 72. lim f ( x ) = f (0 ) = lim (1+ x )1/ x = e
n ®¥ è n ø x ®0 x ®0

Mathematical Problem Book for JEE.indb 850 07-06-2018 13:16:46


Chapter 19 | Limit, Continuity and Differentiability 851

æ 1ö
73. Since lim f ( x ) ¹ f ç ÷ é sin x 2 ù
x ®1/ 2 è2ø 84. f (0 ) = lim f ( x ) = lim f ( x ) = lim x ê 2 ú = 0
x ®0 + x ®0 - x ®0 ê x úû
Hence, the correct answer is option (D). ë
74. f(a) = 0 85. Clearly from curve (Fig. 19.26) drawn of the given function f(x)
æ x2 ö ïì (a - h)
2
ïü is discontinuous at x = 0.
lim f ( x ) = lim çç - a ÷÷ = lim í - aý = 0
x ®a - x ®a - a h®0 ï a ïþ
è ø î
ïì (a + h) ïü
2
and, lim f ( x ) = lim ía - ý=0
x ®a + h®0 ï
î a þï (0,1/4)
Hence, it is continuous at x = a. O
75. f(0) = 0
lim f ( x ) = lim e -1/ h = 0 and lim f ( x ) = lim e1/ h = ¥ (0, −1)
x ®0 - h®0 x ®0 + h®0
Hence, function is discontinuous at x = 0.
ïì x - 4 x + 3 ïü
2
Figure 19.26
76. f ( x ) = í 2 ý, for x ≠ 1
îï x - 1 þï 86. It is obvious that the correct answer is option (A).
f(x) = 2, for x = 1 87.  lim f ( x ) = lim x + 1 = 2 = k
x ®1 x ®1
x2 - 4 x + 3 ( x - 3)
f (1) = 2, f (1+ ) = lim = lim = -1 88. (i) When 0 ≤ x < 1, f(x) doesn’t exist as [x] = 0 here.
x ®1+ x2 -1 x ®1+ ( x + 1)

(ii) Also lim f ( x ) and lim f ( x ) does not exist.


x2 - 4 x + 3 x ®1+ x ®1-
f (1- ) = lim = -1Þ f (1) ¹ f (1- )
x ®1- x2 -1 Hence, f(x) is discontinuous at all integers and also in (0, 1).
Hence, the function is discontinuous at x = 1. sin2 ax 2
89.   lim f ( x ) = a = a2 and f(0) = 1
x +1 x ®0 (ax )2
77. f ( x ) = . Hence, the points are 3, -4.
( x - 3)( x + 4 ) Hence, f(x) is discontinuous at x = 0 , when a ≠ ±1.
78. f(0+) = f(0-) = 2 and f(0) = 2 90.   lim f ( x ) = 0 ,
x ®0 -
Hence, f(x) is continuous at x = 0. f (0 ) = 0 , lim f ( x ) = - 4
x ®0 +
1 2 1
79. xlim f ( x ) = x sin , but - 1 £ sin £ 1 and x ® 0 f(x) discontinuous at x = 0.
®0 + x x
and lim f ( x ) = 1 and lim f ( x ) = 1, f (1) = 1
Therefore, x ®1- x ®1+

lim f ( x ) = 0 = lim- f ( x ) = f (0 ) Hence, f(x) is continuous at x = 1.


x ®0+ x ®0
Hence, f(x) is continuous at x = 0. Also, lim f ( x ) = 4(2)2 - 3.2 = 10
x ®2 -
2
80.   f (0 - ) = lim k (2 x - x ) = 0 ; f (0 + ) = lim cos x = 1 f(2) = 10 and lim f ( x ) = 3(2) + 4 = 10
x ®0 - x ®0 + x ®2 +

Hence, f(0) = cos x = 1 Hence, f(x) is continuous at x = 2.


Hence, no value of k can make f(0-) = 1. 91.   lim f ( x ) = sin-1(0 ) = 0 and f(0) = 0
x ®0
81. f (0 ) = 0 ; f (0 - ) = lim -h -h
= lim =0 Hence, f(x) is continuous at x = 0.
h®0 e + 1 h ® 0 1+ 1
-1/ h

e1/ h 2 sin 2 x 2
92.   xlim f ( x ) = lim = =k
h ®0 x ®0 2x .5 5
f (0 + ) = lim 1/ h =0
h®0 e +1 93.   lim f ( x ) = 0 and lim f ( x ) = 1+ 1 = 2.
x ®1+ x ®1-
1/ 2 x 2 2
82. lim f ( x ) = lim [(1+ 2 x ) ] =e Hence, f(x) is discontinuous at x = 1.
x ®0 - x ®0
94.   lim f ( x ) = -2 and f(-1) = -2
x ®( -1)
83. lim f ( x ) = lim 21/ h = ¥
x ®0 + h®0
1 1
95.   lim f ( x ) = and lim f ( x ) = and f (2) = 1
-1/ h 1 x ®2 - 2 x ®2 + 2
lim f ( x ) = lim 2 = lim =0
x ®0 - h®0 h ® 0 21/ h
96. Obviously lim f ( x ) = f (b ) = 0
x ®b

Mathematical Problem Book for JEE.indb 851 07-06-2018 13:17:22


852 Mathematics Problem Book for JEE

 97.   lim f ( x ) = -1, lim f ( x ) = 1, f ( a) = 1 and f(4) = a + b


x ®a - x ®a + Since f(x) is continuous at x = 4.
 98.  lim f ( x ) = 1, f (1) = 2 Therefore,
x ®1 lim f ( x ) = f ( 4 ) = lim f ( x )
x ®4 - x ®4 +
 99.   lim f ( x ) = 3, lim f ( x ) = 3 and f(2) = 3 ⇒ a - 1 = a + b = b + 1 ⇒ b = -1 and a = 1
x ®2 - x ®2 +

112. F or any x ≠ 1, 2, we find that f(x) is the quotient of two polyno-


æ 3p ö
100. Here, f ç ÷ = 1 and lim f (x) =1 mials and a polynomial is everywhere continuous. Therefore,
è 4 ø x ®3p / 4 -
f(x) is continuous for all x ≠ 1, 2. Check continuity at x = 1, 2.
2 æ 3p ö p 113. Since lim f ( x ) = lim f ( x ) = f (2) = 1 .
lim f ( x ) = lim 2 sin ç + h ÷ = 2 sin = 1 x ®2 - x ®2 +
x ®3p / 4 + h®0 9è 4 ø 6
Also it is continuous for all values of x, less than 2 and greater
3p
Hence, f(x) is continuous at x = . than 2.
4
114. G
 iven function is continuous at all point in (-∞, 6) and at
p -p æp ö p
101.   lim f ( x ) = , lim f ( x ) = and fç ÷= x = 1, x = 3 function is continuous.
x ®p / 2 - 2 x ®p / 2 + 2 è2ø 2
If function f(x) is continuous at x = 1, then
æ 2 sin2 2 x ö p
102.   lim f ( x ) = lim çç 2 ÷ ÷4 = 8 lim f ( x ) = lim f ( x ) Þ 1+ sin = a + b
x ®0 - x ®0 -
è (2 x ) ø x ®1- x ®1+ 2
   ⇒ a + b = 2 (1)
lim f ( x ) = lim 16 + x + 4 = 8. Hence, a = 8.
x →0 + x →0 + If at x = 3, function is continuous, then
103.   lim f ( x ) = a - b , lim f ( x ) = 2 Þ a - b = 2 3p
x ®1- x ®1+
lim f (3) = lim+ f ( x ) Þ 3 a + b = 6 tan
x ® 3- x ®3 12
All the given sets of a, b make f(x) continuous at x = 1.    ⇒ 3a + b = 6 (2)
104.   lim f ( x ) = 1+ 1 = 2 lim f ( x ) = 0 , f (0 ) = 2 From Eqs. (1) and (2), a = 2, b = 0
x ®0 - x ®0 +
115. If f is continuous at x = 0, then
105.   lim f ( x ) = lim ( x + 2) ( x 2 + 4 ) = 32, f (2) = 16
x ®2 x ®2 lim f (x) = lim+ f (x) = f (0 ) Þ f (0 ) = lim- f (x)
x ®0- x ®0 x ®0
106.   lim f ( x ) = 1, lim f ( x ) = 6
x ®1- x ®1+ p
cos [0 - h]
( x - 2) ( x + 5) -7 7 k = lim f (0 - h) = lim 2
107.   lim f ( x ) = = = =a h®0 h®0 [0 - h]
x→ -5 ( x + 5)( x - 3) -8 8
p p
108. By definition of continuity, we know that cos [ - h] cos {- h - 1}
k = lim 2 = lim 2
lim f ( x ) = f (3) = lim f ( x )
x ®3 + x ®3 - h→ 0 [ - h] h→ 0 {-h - 1}
Þ lim f ( x ) = 4 or lim 3 - h + l = 4 æ pö
x ®3 - h®0 cos ç - ÷
⇒ 3+ l = 4 ⇒ l =1 k = lim è 2ø ⇒k=0
h®0 -1
1 116. Clearly the function is defined only in the interval [1, ∞)
109. If x → 0, then the value of sin passes through [-1, 1]
x hence option (B) cannot even apply. For x > 2, y = 3x - 2
infinitely many ways, therefore limit of the function does not which is a straight line, hence continuous. Further y = 4 at
exist at x = 0. Hence, there is no value of k for which the func- x = 2. Hence, the function is continuous at x = 2 also (but not
tion is continuous at x = 0. at x = 2 only).
110. As we are given f(x) = sin x, if x ≠ np, that is, x ≠ 0, p, 2p, … 117. f(x) is continuous at every point of its domain, so
and f(x)= 2 otherwise. Hence, lim+ g {f ( x )} = lim+ g {sin x } = lim+ (sin2 x + 1) lim f ( x ) = lim+ f ( x ) = f (1)
x ®0 x ®0 x ®0 x →1- x →1
lim+ g {f ( x )} = lim+ g {sin x } = lim+ (sin2 x + 1) = 1.
x ®0 x ®0 x ®0    ⇒ 5 × 1 - 4 = 4 × 1 + 3 × b × 1
Similarly, lim g {f ( x )} = 1    ⇒ 1 = 4 +3b ⇒ 3b = -3 ⇒ b = -1
x ®0 -
118. For continuity at all x ∈ R, we must have
4-h-4
111.   lim- f ( x ) = lim f ( 4 - h) = lim +a æ pö
x ®4 h®0 h®0 4-h-4 f ç - ÷ = lim - ( -2 sin x ) = lim + ( A sin x + B )
è 2 ø x ® ( -p / 2 ) x ® ( -p / 2 )
h
= lim - + a = a - 1 ⇒ 2 = -A + B(1)
h→ 0 h
æp ö
4+h-4 and f ç ÷ = lim - ( A sin x + B ) = lim + (cos x )
lim f ( x ) = lim f ( 4 + h) = lim + b = b +1 è 2 ø x ®( p / 2 ) x ®( p / 2 )
x →4 + h→ 0 h→ 0 4+h-4
⇒ 0 = A + B(2)

Mathematical Problem Book for JEE.indb 852 07-06-2018 13:17:57


Chapter 19 | Limit, Continuity and Differentiability 853

From Eqs. (1) and (2), we get A = -1 and B = 1. é 1 ù -1


128. f ( x ) = ê x 2 + e 2 - x ú and f (2) = k
x 2 - 10 x + 25 êë úû
119.  f (5) = lim f ( x ) = lim
x ®5 x ®5 x 2 - 7 x + 10
If f(x) is continuous from right at x = 2, then
( x - 5)2 5-5
= lim = =0 lim f ( x ) = f (2) = k
x ®5 ( x - 2 )( x - 5) 5-2 x ®2+

-1
120. For continuity at x = 0, we must have é 1 ù
Þ lim+ ê x 2 + e 2 - x ú = k Þ k = lim f (2 + h)
f (0 ) = lim f ( x ) x ®2 ê úû h®0
x ®0 ë
x cot x -1
é 1ù é 1 ù
cot x
= lim ( x + 1) = lim ê(1+ x ) x ú Þ k = lim ê(2 + h)2 + e 2 -( 2 + h ) ú
x ®0 x ®0 ê úû h®0 ê úû
ë ë
lim
æ x ö
1 ù x ®0 çè tan x ÷ø
Þ k = lim[ 4 + h2 + 4 h + e -1/ h ]-1
é h®0
= lim ê(1+ x ) x ú = e1 = e
x ®0 ê ú 1
ë û Þ k = [ 4 + 0 + 0 + e -¥ ]-1 Þ k =
4
121. It is obvious that the correct answer is option (A).
122. It is obvious that | x | is continuous for all x. Now, 129. By L’Hospital’s rule, lim f ( x ) is 2. Therefore, for f(x) to be con-
x ®0

0+h -0 tinuous, the value of function should be 2.


Rf ¢( x ) = lim =1
h®0 h 1+ kx - 1- kx
0-h -0 130.   L.H.L. = lim- =k
Lf ¢( x ) = lim = -1 x ®0 x
h®0 -h
2
R.H.L. = lim+ (2 x + 3 x - 2) = -2
Hence, f(x) = | x | is not differentiable at x = 0. x ®0
p Since it is continuous,
123. f(x) is continuous at x = , then
2 L.H.L. = R.H.L. ⇒ k = -2
p  1- sin x  0 
lim f ( x ) = f   or l = lim ,  form x x x x x
x →p / 2  2 x →p / 2 p - 2 x  0  2 cos2- 2 sin cos cos - sin
131. f ( x ) = 2 2 2 = 2 2
Applying L’Hospital’s rule, x x x x x
2 cos2 + 2 sin cos cos + sin
2 2 2 2 2
- cos x cos x
l = lim Þ l = lim =0
x ®p / 2 -2 x ®p / 2 2 æp x ö p
= tan ç - ÷ at x = p , f (p ) = - tan = -1
è4 2ø 4
124. Since f(x) is continuous at x = 0, therefore
lim f ( x ) = f (0 ) Þ lim sinp x = k 1- cos x
 , x≠0
x ®0 x ®0 5 x 132. f ( x ) =  x continuous at x = 0
 k , x=0
æ sinp x ö p p p
Þ lim ç ÷ × = k Þ (1) × = k Þ k =
x ®0 è p x ø 5 5 5
2 ⋅ sin2 ( x / 2)
lim+ f ( x ) = f (0 ) ⇒ lim =k
125. If f(x) is continuous at x = 0, then x →0 x →0 x
2- x +4 0  2 sin2 x / 2 x
f (0 ) = lim f ( x ) = lim ,  form Þ lim × =kÞk =0
x →0 x →0 sin 2 x 0  x ® 0 ( x / 2 )2 4
Using L’Hospital’s rule, 133. It is obvious that the correct answer is option (A).
æ 1 ö
ç- ÷ 1 ì 1
2 x+4 ø
f (0 ) = lim è =- ï e x -1
x ®0 2 cos 2 x 8 ï , x ¹0
134. Given f ( x ) = í 1
126. At no point, function is continuous. ïe +1
x
ïî 0 , x =0
x2 - 9
127.   lim f ( x ) = lim = lim( x + 3) = 6 and f(3) = 2(3) + k = 6 + k
x ®3 x ®3 x - 3 x ®3 1 1
1-
e x -1 1/ x
e
As f is continuous at x = 3 ; Therefore Þ lim+ = lim+ =1
x ®0 1 x ®0 1
6+k=6⇒k=0 ex +1 1+ 1/ x
e

Mathematical Problem Book for JEE.indb 853 07-06-2018 13:18:30


854 Mathematics Problem Book for JEE

1 1 1
1- Therefore, f ([2x]), for all values of x where x <
a continu-
e x -1 e1/ x 1- e ¥ 2
lim- 1 = lim+ = =1 1
x ®0 x ®0 1 1+ e ¥ ous function and for x = and x =1, f(x) be a discontinuous
e +1
x 1 +
e1/ x function. 2
So, lim f ( x ) exists at x = 0, but at x = 0 it is not continuous.
x ®0
1- cos 4 x
135. We have  , x≠0
141.  f ( x ) =  8 x 2
f(x) = 2x - 1, if x > 2,  k, x=0
f(x) = k, If x = 2 and If f(x) is a continuous function at point x = 0, then
x2 - 1, if x < 2, lim f ( x ) = lim [f ( x )] = lim[f ( x )] = lim [f (0 + h)]
x →0 + x →0 - x →0 h→ 0 -
Therefore,
1- cos 4 h sin2 2h
lim f ( x ) = f (2) Þ lim(2 x - 1) = k Þ k = 3 = lim[f (h)] = lim = lim
x ®2 x ®2 h→ 0 h→ 0 8h2 h→ 0 4 h2

2
 2 x - sin-1 x  0  æ sin 2h ö 2
136.  f ( x ) = lim  = lim ç ÷ = (1) = 1
 = f (0),
x → 0  2 x + tan-1 x 
 form
0 h ® 0 è 2h ø

Applying L’Hospital’s rule, 1- cos 4( -h)


= lim- f ( x ) = lim[f (0 - h)] = lim[f ( -h)] = lim
x ®0 h®0 h®0 h®0 8( -h)2
 1 
2 -  = lim
1- cos 4 h
=1
 1- x 2  2 - 1 1
f (0 ) = lim = = h®0 8h2
x →0  1  2 +1 3
 2 +  ⇒ f ( 0 ) = 1⇒ k = 1
1+ x 2 
sin-1 x ì ex ; x £0
2- ï
x 2 -1 1 f ( x ) = 1 - x ; 0 < x £1
Alternate solution: f (0 ) = lim -1
= = 142.   í
x ®0 tan x 2 + 1 3 ï x - 1; x >1
2+ î
x
|x| f ( 0 + h) - f ( 0 ) 1- h - 1
137. |x| is continuous at x = 0 and is discontinuous at x = 0. Rf ¢(0 ) = lim = lim = -1
x h®0 h h®0 h

Hence, f ( x ) = | x | +
|x|
is discontinuous at x = 0. f ( 0 - h) - f ( 0 ) e -h - 1
Lf ¢(0 ) = lim = lim =1
x h®0 -h h ® 0 -h

æ 2 x - 2- x ö é (2 x + 2- x )loge 2 ù So, it is not differentiable at x = 0.


138.  f (0 ) = lim f ( x ) = lim çç ÷÷ = lim ê ú Similarly, it is not differentiable at x = 1.
x ®0 x ®0
è x ø x ®0 ëê 1 úû But it is continuous at x = 0, 1.
= (20 + 20) loge 2 143. S tatement (D) is true, because differentiable function is always
= (1 + 1) loge 2 continuous.
= 2loge2 = loge4 144. As Lf ′(2) ≠ R f ′(2), hence the correct answer is option (D).
2 2
2x + 7 2x + 7 145.  f (0 + 0 ) = lim f ( x ) = lim f (0 + h)
139.  f ( x ) = 2
= 2
x ( x + 3) - 1( x + 3) ( x - 1)( x + 3) h®0 h®0

1/ 0 + h
2x2 + 7 e - e -1/ 0 + h e1/ h - e -1/ h
= = lim(0 + h) = lim h =0
( x - 1)( x + 1)( x + 3) h®0 e1/ 0 + h + e -1/ 0 + h h®0 e1/ h + e -1/ h
Hence, points of discontinuity are x = 1, x = -1 and x = -3 only. e -1/ h - e1/ h
and f (0 - 0 ) = lim f (0 - h) = lim - h =0
ì1- | x | ì 1 x <0 h®0 h®0 e -1/ h + e1/ h
ï , x ¹ -1 ï
140.  f ( x ) = í 1+ x and f ( x ) = í1- x and f(0) = 0; hence, f(0 + 0) = f(0 - 0) = f(0)
ïî 1, ïî1+ x , x ³0
x = -1 Hence, f is continuous at x = 0.
At remaining points f(x) is obviously continuous.
ì 1, x <0 Thus, it is everywhere continuous. Again,
ï
ï 1, 0 £ x < 1 f ( 0 - h) - f ( 0 )
ì 1, x <0 ï 2 Lf ¢(0 ) = lim
ï ï h®0 -h
f ( [ 2 x ]) = í 1 - [ 2 x ] Þ f ( [ 2 x ]) = í 1
ï 1 + [2 x ] , x >0 ï 0, £ x £1
î 2 e -1/ h - e1/ h
ï h
-1/ h
ï- 1 , 1£ x < 3 = lim e + e1/ h = -1
ïî 3 2 h®0 -h

Mathematical Problem Book for JEE.indb 854 07-06-2018 13:19:00


Chapter 19 | Limit, Continuity and Differentiability 855

Hence, function is continuous in [0, 2]


e1/ h - e -1/ h
h Clearly, from graph (Fig. 19.28) it is not differentiable at x = 1.
f ( 0 + h) - f ( 0 ) 1/ h -1/ h
Rf ′(0 ) = lim = lim e + e =1
h→ 0 h h→ 0 h 150. Since this function is continuous at x = 0.
Since L f ′(0) ≠ R f ′(0). Now for differentiability
Hence, f is not differentiable at x = 0. f(x) = |x| = |0| and f(0 + h) = f(h) = |h|
146.   lim f ( x ) = lim f (3 - h) = lim | 3 - h - 3 |= 0 Hence,
x ®3 h®0 h®0 f ( 0 + h) - f ( 0 ) | h|
lim = lim- = -1
lim+ f ( x ) = lim f (3 + h) = lim | 3 + h - 3 |= 0 h®0- h h®0 h
x ®3 h®0 h®0

Since f ( 0 + h) - f ( 0 ) | h|
and lim+ = lim+ =1
lim- f ( x ) = lim+ f ( x ) = f (3) h®0 h h®0 h
x ®3 x ®3
Hence, f is continuous at x = 3. (See Fig. 19.27.) Therefore, it is continuous and non-differentiable.

 x2
 = x, x>0
 x2 x
 , x ≠ 0 
|x − 3| 151. We have f ( x ) = | x | = 0, x=0
 0, x = 0  x2
 
O x=3 = -x, x<0
 - x
Figure 19.27
We have
Now lim- f ( x ) = lim- - x = 0 , lim+ f ( x ) = lim x = 0 and f (0 ) = 0
f ( 3 - h ) - f ( 3) | 3 - h - 3 | -0 h x ®0 x ®0 x ®0 x ®0
Lf ¢(3) = lim = lim = lim = -1
h®0 -h h®0 -h h ® 0 -h So, f(x) is continuous at x = 0.
f ( 3 + h ) - f ( 3) | 3 + h - 3 | -0 Also f(x) is continuous for all other values of x.
Rf ¢(3) = lim = lim =1 Hence, f(x) is continuous everywhere. Clearly, Lf ′(0) = -1 and
h®0 h h®0 h
Rf ′(0) = 1. Therefore f(x) is not differentiable at x = 0.
Since L f ′(3) ≠ R f ′(3). Hence f is not differentiable at x = 3.
152. We have
o Trick: Can be seen by graph it is continuous but tangent is not
defined at x = 3. f (1+ h) - f (1) [(1+ h)3 - 1] - 0
Rf ¢(1) = lim = lim =3
147. x ≤ x2 ⇒ x(1 - x) ≤ 0 ⇒ x(x - 1) ≥ 0 h®0 h h®0 h
é x: x £0 f (1- h) - f (1) [(1- h) - 1] - 0
ê 2 Lf ¢(1) = lim = lim =1
Þ x £ 0 or x ³ 1; therefore h( x ) = ê x : 0 < x < 1 h®0 -h h ® 0 -h
ê x: x ³1
ë Hence,
h(x) is continuous for every x but not differentiable at x = 0 R f ′(1) ≠ L f ′(1) ⇒ f (x) is not differentiable at x = 1
and 1. Also Now,
é 1 x <0 f (1+ 0 ) = lim f (1+ h) = 0
h®0
ênot exists x =0
ê and    f (1- 0 ) = lim f (1- h) = 0
h¢( x ) = ê 2x 0 < x <1 h®0
ê
ênot exists x =1 Hence,
ê 1 x >1 f(1 + 0) = f(1 - 0) = f(0) ⇒ f (x) is continuous at x = 1
ë
Hence, at x = 1, f(x) is continuous and not differentiable.
Therefore, h′(x) = 1 for all x > 1.
153. Here, when -1 ≤ x ≤ 1, 0 ≤ x sin π x < 1. So,
148. It is obvious that the correct answer is option (D).
f (x) = [x sin π x] = 0 for -1 ≤ x ≤ 1,
ìx, 0 £ x £1
149.  f ( x ) = í That is, f (x) is constant function (equal to zero) in [-1, 1].
î 1, 1 £ x £ 2
Therefore, f (x) is differentiable in (-1, 1).
lim- f ( x ) = lim f (1- h) = lim(1- h) = 1 =
x
x ®1 h®0 h®0 y y=1 154. Since |x - 3| = x - 3, if x ≥ 3;
lim+ f ( x ) = lim f (1+ h) = 1 |x - 3| = -x + 3, if x < 3
x ®1 h®0 O x=1
Hence, the given function can be defined as
Hence, function is continuous in (0, 2).
Now ì1 2 3 13
ï4 x - 2 x + 4 , x <1
lim f ( x ) = lim(0 + h) = 0 = f (0 ) Figure 19.28 ï
x ®0+ h®0 f (x) = í 3 - x, 1£ x < 3
ï x - 3, x ³3
lim f ( x ) = lim(2 - h) = 1 = f (2) ï
x ®2- h®0 î

Mathematical Problem Book for JEE.indb 855 07-06-2018 13:19:24


856 Mathematics Problem Book for JEE

Now proceed to check the continuity and differentiability at Hence, f is continuous at x = 2.


x = 1. Now,
lim f ( x ) = f (1) = 2 5 - (2 + h) - 3
x ®1 Rf ¢( x ) = lim = -1
h®0 h
and
lim f ( x ) = - 1 1+ ( 2 - h ) - 3
x ®1
Lf ¢( x ) = lim =1
h®0 -h
So, f(x) is continuous and differentiable at x = 1. Since, R f ′(x) ≠ L f ′(x); therefore, f is not differentiable at x = 2.
Also,
lim f ( x ) = f (3) = 0 [k - h]sinp (k - h) - [k ]sinp k
x ®3
159.  f ¢(k - 0 ) = lim
h®0 -h
So, f(x) is also continuous at x = 3. ( -1)k -1(k - 1)sinp h - k ´ 0
= lim
155. Given f(x) is differentiable at x = 0. Hence, f(x) will be contin- h®0 -h
uous at x = 0. Therefore,
( -1)k -1(k - 1)sinp h
x
lim (e + ax ) = lim b( x - 1) 2 = lim = ( -1)k × (k - 1)p
x ®0 - x ®0 +
h®0 -h

⇒ e0 + a × 0 = b(0 - 1)2 ⇒ b = 1 (1) ïì0 , x <0


160.  f ( x ) = í 2 ; lim- f ( x ) = lim f (0 - h) = 0
But f(x) is differentiable at x = 0, then îï x , x ³0 x ®0 h®0

d x d ïì0 , x <0
Lf ¢( x ) = Rf ¢( x ) Þ (e + ax ) = b( x - 1)2 f (x) = í 2 ; lim- f ( x ) = lim f (0 - h) = 0
dx dx x ³0
îï x , x ®0 h®0
⇒ ex + a = 2b(x - 1)
and lim f ( x ) = lim f (0 + h) = lim(0 + h)2 = 0
At x = 0, x ®0+ h®0 h®0

e0 + a = -2b ⇒ a + 1= -2b ⇒ a = -3 Þ lim- f ( x ) = lim+ f (0 )


x ®0 x ®0
⇒ (a, b) = (-3, 1)
Hence, f (x) is continuous function at x = 0.
156. I t can be easily seen from the graph (Fig. 19.29) of f(x) = |sin x| f ( x ) - f (0) f ( 0 - h) - 0 0-0
that it is everywhere continuous but not differentiable at Lf ¢( x ) = lim- = lim = lim =0
x ®0 x -0 h®0 -h h ® 0 -h
integer multiples of π and at x = 0.
f ( x ) - f (0) f ( 0 + h) - f ( 0 ) 0-0
Rf ¢( x ) = lim+ = lim = lim =0
x ®0 x -0 h→ 0 h h→ 0 h

⇒ L f ′(x) = R f ′(x)
Hence, f (x) is differentiable at x = 0.
Now
ì0 , x <0
f ¢( x ) = í ; lim- f ¢( x ) = lim f ¢(0 - h) = 0
î2 x , x ³ 0 x ®0 h®0

ì0 , x <0
Figure 19.29 f ¢( x ) = í ; lim f ¢( x ) = lim f ¢(0 - h) = 0
î2 x , x ³ 0 x ®0- h®0

ìïe - x , x ³0 and
157. We have, f ( x ) = í lim f ¢( x ) = lim f ¢(0 + h) = lim 2(0 + h) = 0
x
ïî e , x <0 x ®0+ h®0 h®0

Now Þ lim- f ¢( x ) = lim+ f ¢( x ) = 0


x ®0 x ®0
lim- f ( x ) = lim- e x = 1, lim+ f ( x ) = lim+ f ( x )e - x = 1 Hence, f ′(x) is continuous function at x = 0.
x ®0 x ®0 x ®0 x ®0
f ′( x ) - f ′( 0 ) f ′( 0 - h ) - f ′ ( 0 )
Also, f(0) = e0 = 1. So, f(x) is continuous for all x. Lf ′′( x ) = lim- = lim
h→ 0 x -0 h→ 0 -h
éd ù
(LHD at x = 0) = ê (e x )ú =1 0-0
ë dx û x =0 = lim =0
h®0 -h
é d -x ù f ′( x ) - f ′( 0 ) f ′( 0 + h ) - f ′ ( 0 )
(RHD at x = 0) = ê (e )ú = -1 Rf ′′( x ) = lim+ = lim
ë dx û x =0 h→ 0 x -0 h→ 0 h

So, f(x) is not differentiable at x = 0. 2(0 + h) - 0 2h


= lim = lim = 2 Þ Lf ¢¢( x ) ¹ Rf ¢¢( x )
Hence, f (x) = e-| x |
is everywhere continuous but not differ- h®0 h h®0 h

entiable at x = 0. 2(0 + h) - 0 2h


= lim = lim = 2 Þ Lf ¢¢( x ) ¹ Rf ¢¢( x )
158.   lim- 1+ (2 - h) = 3, lim+ 5 - (2 + h) = 3, f (2) = 3
h®0 h h®0 h
h® 0 h® 0
Hence, f ′(x) is not differentiable at x = 0.

Mathematical Problem Book for JEE.indb 856 07-06-2018 13:19:52


Chapter 19 | Limit, Continuity and Differentiability 857

161. A
 continuous function may or may not be differentiable. f (1+ h)
So, (B) is not true. is continuous as it is given that lim = 5 and hence
h®0 h
æ 1ö æ 1ö f(1) = 0. Therefore,
162.   lim f ( x ) = x 2 sin ç ÷ , but - 1 £ sin ç ÷ £ 1 and x ® 0
x ®0 èxø èxø f (1+ h)
f ¢(1) = lim =5
Hence, h®0 h
lim f ( x ) = 0 = lim- f ( x ) = f (0 )
x ®0+ x ®0
f (x) - f (y)
Therefore, f (x) is continuous at x = 0. Also, the function 168.   lim £ lim x - y or f ¢( x ) £ 0
x®y x-y x®y
1
f ( x ) = x 2 sin is differentiable because ⇒ f ′(x) = 0 ⇒ f (x) is constant
x
1 As f(0) = 0, hence f(1) = 0.
h2 sin - 0
h h2 sin(1/ -h)
Rf ¢( x ) = lim = 0 , Lf ¢( x ) = lim =0 169. As f(1) = -2 and f ′(x) ≥ 2 ∀x ∈ [1, 6]
h®0 h h®0 -h
Applying Lagrange’s mean value theorem,
163. By definition, f (6 ) - f (1)
f (1+ h) - f (1) = f ′(c) ≥ 2
f ¢(1) = lim 5
h®0 h
⇒ f(6) ≥ 10 + f(1) ⇒ f(6) ≥ 10 - 2 ⇒ f(6) ≥ 8
1 æ -1 ö æ 1 1ö
-ç ÷ ç + ÷
2(1+ h) - 5 è 3 ø è 2h - 3 3 ø | x | - 1, | x | -1 ≥ 0
= lim = lim 170.  f ( x ) = 
h®0 h h®0 h  - | x | + 1, | x | - 1< 0
é 3 + 2h - 3 ù é 2h ù | x | -1, x ≤ -1 or x ≥ 1
= lim ê = lim ê
h ® 0 ë 3h(2h - 3) ú ú
û h®0 ë 3h(2h - 3) û f (x) = 
 - | x | +1, -1< x < 1
2 2 -2
= lim = =  - x - 1, x ≤ -1
h®0 3(2h - 3) 3( -3) 9  x + 1,
 -1 < x < 0
f (x) = 
164. Let x < 0. Then - x + 1, 0 ≤ x <1

d æ x ö 1  x - 1, x ≥1
| x | = -x ⇒ f ¢( x ) = ç ÷=
dx è 1- x ø (1- x )2
Y
⇒ [f ′(x)]x = 0 = 1 (0,1)
Again
x > 0 ⇒ | x | = x
X
d æ x ö 1 (−1, 0) (1, 0)
f ¢( x ) = ç ÷= Þ [f ¢( x )]x = 0 = 1
dx è 1+ x ø (1+ x )2
⇒ f ′(0) = 1 Figure 19.30
f (1- h) - f (1) From the graph (Fig. 19.30), it is clear that f (x) is not differen-
165.  Lf ¢(1) = lim
h®0 -h tiable at x = -1, 0 and 1.
171. L et a function be g(x) = f(x) - x2. Then g(x) has at least 3 real
m(1- h)2 - m m[1+ h2 - 2h - 1]
= lim = lim roots which are x = 1, 2 , 3, so g′(x) has at least 2 real roots
h®0 -h h®0 -h in x ∈ (1, 3) and g′′(x) has at least 1 real roots in x ∈ (1, 3)
f (1+ h) - f (1) Therefore, f ′′(x) = 2 for at least one x ∈ (1, 3).
= lim m(2 - h) = 2m Rf ¢(1) = lim
h® 0 h® 0 h
 1  1  1
172.  f (1) = f   = f   =  = lim f   = 0
f (1+ h) - f (1) 2(1 + h) - m  2  3 n→∞  n 
= lim m(2 - h) =and
2m Rf ¢(1) = lim = lim
h® 0 h® 0 h h→ 0 h Since there are infinitely many points in x ∈ (0,1) where
For differentiability, L f ′(1) = R f ′(1). æ 1ö
f(x) = 0 and lim f ç ÷ = 0
But for any value of m, R f ′(1) = L f ′(1) not possible. n ®¥ è n ø

166. (gof )(x) = g[f (x)] = g[1 - cosx] = e1-cos x, for x ≤ 0 ⇒ f (0) = 0


(gof )′(x) = e1-cos x · sin x, for x ≤ 0 And since there are infinitely many points in the neighbour-
(gof )′(0) = 0 hood of x = 0. Such that f(x) remains constant in the neigh-
bourhood of x = 0. Therefore,
f (1+ h) - f (1)
167.  f ¢(1) = lim ; as function is differentiable so it f ′(0) = 0
h®0 h

Mathematical Problem Book for JEE.indb 857 07-06-2018 13:20:17


858 Mathematics Problem Book for JEE

173. f(1) = -3; f ′(x) ≥ 9 for all x ∈ (1, 5); hence, f(5) ≥ 36. ì x + 2, -1 < x < 3
ï
174. We know, 181. If f (x) = í5, x = 3 and f(3) = 5
f ( x + h) - f ( x ) ï8 - x , x >3
f ′( x ) = lim î
h→0 h f ( x ) - f ( 3) f ( 3 - h ) - f ( 3)
Given f(x + y) = f(x) f(y), so L.H.D = lim- = lim
x ®3 x -3 h®0 -h
f ( x )f (h) - f ( x )
f ′( x ) = lim (3 - h + 2 ) - 5 -h
h→0 h = lim = lim =1
h®0 -h h ® 0 -h
f ( x )( f ( h) - 1)
⇒ f ′( x ) = lim f ( x ) - f ( 3) f ( 3 + h ) - f ( 3)
h→0 h R.H.D = lim+ = lim
f ( x )(1+ sin(3h)g( h) - 1) x ®3 x -3 h ® 0 h
⇒ f ′( x ) = lim
h→0 h 8 - (3 + h) - 5 -h
= lim = lim = -1
⇒ f ′( x ) = lim 3f ( x )g(h) = 3f ( x )g(0 ) h®0 h h®0 h
h→0
L.H.D ≠ R.H.D f (x) is not differentiable.
1 ∀x < 0 ìx, 0 £ x £1
 182.  f ( x ) = í
175.  f ( x ) =  p
1+ sin x , ∀0 ≤ x ≤ 2 î2 x - 1, x > 1
lim f ( x ) = lim f (1- h) = lim (1- h) = 1
x ®1- h®0 h®0
0 , ∀ x < 0 (LHD)
⇒ f ′( x ) =  lim f ( x ) = lim f (1+ h) = lim 2(1+ h) - 1 = 1
cos x , 0 ≤ x ≤ p / 2, (RHD) x ®1+ h®0 h®0

Lf ′(0) = 0, Rf ′(0) = 1 Since,


So, derivative does not exist at x = 0. lim f ( x ) = lim+ f ( x ) = 1
x ®1- x ®1
176. f (x) = - 2x + 4; f ′(x) = 2x - 2
x2 Hence, function is continuous at x = 1
At x = c, f ′(c) = 2c - 2 f (1- h) - f (1) (1- h) - 1
f(5) = 52 - 2(5) + 4 = 19; f(1) = 12 - 2(1) + 4 = 3 Lf ¢(1) = lim = lim =1
h®0 -h h®0 -h
f (5) - f (1) 19 - 3 16
= f (c ) Þ = 2c - 2 Þ = 2c - 2 f (1+ h) - f (1) 2 + 2h - 1 - 1
5 -1 5 -1 4 Rf ¢(1) = lim = lim =2
h®0 -h h®0 h
⇒ 4 = 2c - 2 ⇒ 2c = 6 or c = 3
Therefore, L f ′(1) ≠ R f ′(1)
f ( x + h) - f ( x ) f ( x ) + f ( h) - f ( x ) Hence, function is not differentiable at x = 1.
177. We have f ¢( x ) = lim = lim
h®0 h h®0 h 183. f  (x) possesses derivative at x = 0, so it is both continuous and
[As f ( x + y ) = f ( x ) + f ( x )] differentiable at x = 0. Now f (0 + 0) = 0, f (0 - 0) = b, f (0) = b.
Therefore,
f ( h) h2 g(h) b=0
= lim = lim = 0. g(0 ) = 0
h→ 0 h h→ 0 h Also R f ′(0) = 0, L f ′(0) = 0,∀a ∈ R.
[As g is continuous therefore lim g(h) = g(0 )] Therefore, f ′(0) = 0 if b = 0.
h® 0
184. Let h(x) = x, x ∈ (-∞, ∞); g(x) = 1 + | x |, x ∈ (-∞, ∞)
178. Since function| x | is not differentiable at x = 0. So, Here h is differentiable in(-∞, ∞) but | x | is not differentiable
|  x2 - 3x + 2| = | (x - 1)(x - 2) | at x = 0
Hence, the function is not differentiable at x = 1 and 2. Therefore, g is differentiable in (-∞, 0) ∪ (0, ∞) and g(x) ≠ 0,
Now f (x) = (x2 -1)| x2 - 3x + 2| + cos(| x |) is not differentiable
h( x ) x
at x = 2. ∀x ∈ (-∞, ∞), therefore f ( x ) = =
For 1 < x < 2, f (x) = -(x2 - 1)(x2 - 3x + 2) + cos x g( x ) 1+ | x |
For 2 < x < 3, f (x) = +(x2 - 1)(x2 - 3x + 2) + cos x
It is differentiable in (-∞, 0) ∪ (0, ∞) for x = 0
L f ′(x) = -(x2 - 1)(2x - 3) - 2x(x2 - 3x + 2) - sin x
L f ′(2) = -3 - sin 2 h
-0
R f ′(x) = (x2 - 1)(2x - 3) + 2x(x2 - 3x + 2) - sin x f ( h) - f ( 0 ) 1+ | h | 1
lim = lim = lim =1
R f ′(2) = (4 - 1)(4 - 3) + 0 - sin 2 = 3 - sin 2 h®0 h-0 h®0 h h ® 0 1+ | h |
Hence, L f ′(2) ≠ R f ′(2).
Therefore f is differentiable at x = 0, so f is differentiable in
dy (-∞, ∞).
179. Since = cos x which is defined at x = 0 and no other
dx
1 2(1 + x 2 ) - 4 x 2 2(1 - x 2 )
differential coefficient is defined at x = 0. 185.   y ¢ = × =
2 2
æ 2x ö
2 (1 + x ) (1 - x 2 )2 (1 + x 2 )
180. It is a fundamental concept. Hence, the correct answer is 1- ç ÷
option (C). è 1+ x 2 ø

Mathematical Problem Book for JEE.indb 858 07-06-2018 13:20:42


Chapter 19 | Limit, Continuity and Differentiability 859

ì 2 y
ïï 2
for x <1 = x
Þ y ¢ = í 1+ x
1 +
− 1
x =
ï -2 for x >1 y
ïî1+ x 2 y=2

Hence for | x | = 1, the derivative does not exist.


186. S ince the function is defined for x ≥ 0, that is, not defined for
x < 0. Hence, the function neither continuous nor differentia- −1 0 1
ble at x = 0.

187. Function f (x) = | x - 0.5| + | x - 1| + tan x does not have a
p
derivative at the points x = 0.5, 1, Î (0 , 2).
2
188. Applying L’Hospital’s Rule, Figure 19.32
l x l -1 1 æ 1+ ax ö 1
lim = 500 194.   lim log ç ÷ = lim [log(1+ ax ) - log(1- bx )]
x ®5 1 x ®0 x è 1- bx ø x ®0 x
⇒ λ · 5λ -1 = 500
1  (ax )2 (ax )3    (bx )2 (bx )3 
⇒ λ = 4 = lim  ax - + +   -  -bx - - -  
x → 0 x  2! 3!    2! 3!  

189. L.H.L. =  lim f (0 - h) = lim( -h) = 0
h®0 h®0
1é ( a2 - b 2 ) x 2 ( a3 + b 3 ) x 3 ù
2 lim ê(a + b ) x - + + ú
R.H.L. =  lim f (0 + h) = lim(h ) = 0 x ®0 x ê
ë 2 ! 3 ! úû
h®0 h®0

But f(0) = 1 é ( a2 - b 2 ) x ( a3 + b 3 ) 2 ù
So, limit does not exist. lim ê(a + b ) - + x + ú = (a + b )
x ®0 ê
ë 2! 3! úû
190. f(x) = [a + b sin x] x ∈ (0, p)
1/ n
é æ 3 ön æ 5 ön ù
β 195.   lim (3n + 5n + 7n )1/ n = lim 7 ê1+ ç ÷ + ç ÷ ú
ë è 7 ø è 7 ø úû
n ®¥ n ®¥ ê
4
1/ n
3 é æ 3 ön æ 5 ön ù
2 = lim 7 ê1+ ç ÷ + ç ÷ ú = 7 ´ e0 = 7
ë è 7 ø è 7 ø úû
n ®¥ ê
1

x
196.  f ( x ) =
x
Figure 19.31
ì-1 x < 0
Þ f (x) = í
Total number of points of discontinuous are 2β - 1 (Fig. 19.31). î1 x > 0
191. [x + [x]] = [x] + [x] = 2[x] So, this function is discontinuous at only 1 point.
2m / x
æ x 3x ö 197.   lim+ ( -1)[ x ] = ( -1)2 = 1, lim- ( -1)[x] = ( -1)1 = -1
192.   A = lim ç sin + cos ÷ x ®2 x ®2
x ®0 è m mø
lim( -1)[ x ] does not exist.
2m æ x 3x ö æ0 ö x ®2
ln A = lim ln ç sin + cos ÷ ç form ÷
x ®0 x è m mø è 0 ø
1+ sin x - cos x + log(11-+xsin
) æ 0x ö- cos x + log(1- x ) æ 0 ö
Applying L’Hospital’s rule 198.   lim 3
lim ç ÷ form3 ç ÷ form
x è 0 ø x è0ø
x ®0 x ® 0

2m 1æ x 3x ö 1 Applying L’Hospital’s form, we get


lim × cos - 3 sin ÷ = 2 (1- 0)) = 2
x ®0 æ x 3 x ö m çè m mø 1+ 0
ç sin + cos ÷ 1
è m mø cos x + sin x + ( -1) 0
1- x  
lim 2  
Therefore, A = e2. x →0 3x 0

193. D
 rawing the graph, we can see that function is continuous 1
- sin x + cos x +
at all points. (See Fig. 19.32) Hence, the correct answer is = lim ( x - 1)2  0  form
 
option (A). x →0 6x 0

Mathematical Problem Book for JEE.indb 859 07-06-2018 13:21:02


860 Mathematics Problem Book for JEE

2 Applying L’Hospital’s rule


- cos x - sin x -
= lim ( x - 1)3
1
x →0 6
-3 -1
2 cos -1
( -1+ h) ⋅ h(2 - h) 1
= = lim =
h→0 1 2p
6 2
2 h
sin(e x -2 - 1) æ 0 ö
199.   lim ç ÷ 205. The form of the limit is 1∞, so
x ®2 log( x - 1) è 0 ø
1/ x  f (1+ x )  1  f (1+ x ) - f (1) 
cos(e x - 2 - 1) ⋅ e x - 2  f (1+ x )   f (1) -1 x lim 
f (1)⋅( x ) 
= lim e   x →0 
= lim =1 lim =e
x →2 1 x →0 
 f (1) 
 x →0

x -1 f ′ (1) 6

= e f (1) = e 3 = e 2
x cos x - log(1+ x ) æ 0 ö
200. lim ç ÷
x ®0 x2 è0ø 206. LHL RHL
1 lim g[f ( x )] lim g[f ( x )]
cos x - x sin x - x ®0 - x ®0 +

= lim 1+ x æ 0 ö
ç ÷ Þ lim g[f (0 - h)] = lim g[f (0 + h)]

x ®0 2x è0ø h®0 h®0

1 Þ lim g[sin(0 - h)] = lim g[sin(0 + h)]


- sin x - sin x - x cos x + h®0 h®0
= lim (1+ x )2 = 1
x →0 2 2 Þ lim [sin(0 - h)] + 1 2 = lim [sin(0 + h)]2 + 1
h®0 h®0

x +1 + x -1 - 2 = 1 =1
201. lim Therefore, LHL = RHL = 1.
x →0 x
x + 1- ( x - 1) - 2 207. LHL RHL
= lim
x ®0 x p lim 2(1+ h) - 3 [1+ h]
lim sin (1- h) h®0
2-2 h®0 2
= lim =0
x →0 x = 1 = |-1| × 1
= 1
a tan x - asin x Function is continuous but not differentiable at x = 1.
202. lim
x ® 0 tan x - sin x
(cos x - 1)(cos x - e x ) 2 sin2 ( x / 2)(e x - 1 + 1- cos x )
a sin x
(a tan x - sin x
- 1) 208.   lim = lim
= lim x ®0 xn x ®0 xn
x ®0 tan x - sin x
2 sin2 ( x / 2)(e x - 1+ 2 sin2 ( x / 2)]
= a0⋅log
a lim
x ®0 xn
= log a 2
1 é sin( x / 2) ù (e x - 1+ 2 sin2 ( x / 2)]
= lim ê
1 2 n -1 x ®0 2 ë x / 2 úû x n -2
1+ e n + e n +  + e n
It is non-zero, if
203.   lim
n→∞ n n-2=1⇒ n= 3
n -1 r Alternative method:
1
=å en ×
n
r =0 (cos x - 1)(cos x - e x )
lim
1 x →0 xn
= ò e x dx
 x2 x 4  2
0
 - 2! + 4 ! +  ( - x - x + ))
 
= [e x ]10 = lim is non-zero
x →0 xn
=e-1 ⇒n=3
p - cos -1 x log n log(n + 1) log(n + 2) log nk
204. lim+ 209.   lim × 
x →-1 x +1 n ®¥ log( n - 1) log n log(n + 1) log nk - 1

p - cos -1 ( -1+ h) log nk log n


= lim ⇒ lim = k lim =k
h→0
( -1+ h) + 1 n→∞ log( n - 1) n→∞ log( n - 1)

Mathematical Problem Book for JEE.indb 860 07-06-2018 13:21:35


Chapter 19 | Limit, Continuity and Differentiability 861

210. Here, 1+ { x }
x x Þ h( x ) = {x2} ≥ 0
ét | t |ù x|x| 1 1+ { x }
f ( x ) = ò | t | dt = ê ú = 2 +2
-1 ë 2 û -1
1+ { x }
1 Þ h( x ) = =1
Þ f ¢( x ) = (| x | + | x |) =| x | 1+ { x }
2
Both f and f ′ are continuous for x + 1 > 0. h(x) = 1
ª h′(x) = 0 for both x = 1 and -1.
f ( x ) - f ( x0 )
211.   f ′( x 0 ) = lim
x → x0 x - x0 217. 1∞ form
( x - x 0 )f ( x ) - 0 f ( x ) = ( x - x 0 ) f ( x )
= lim   lim é x + 5 x + 3
2 ù
x → x0 x - x0  f ( x0 ) = 0  L = e x ®¥ ê 2 - 1ú x
212. f(x) = {|x| - |x - 1|}2 êë x + x + 2 úû

⇒ f(x) = x2 + (x - 1)2 - 2|x| |x - 1| lim x2 + 5x + 3 - x2 - x - 2


L = e x ®¥ ×x
 x 2 + ( x - 1)2 - 2( x )( x - 1), x ≤ 0 x2 + x + 2

⇒ f ( x ) =  x 2 + ( x - 1)2 + 2 x ( x - 1), 0 ≤ x ≤1 lim ( 4 x + 1) x
= e x ®¥
 2 2 x2 + x + 2
 x + ( x - 1) - 2 x ( x - 1), x ≥1
æ 1ö
 x 2 + x 2 - 2 x + 1- 2 x 2 + 2 x , x2 ç 4 + ÷
x≤0 lim è xø
 = e x ®¥
⇒ f ( x ) =  x 2 + ( x - 1)2 + 2 x 2 - 2 x , 0 ≤ x ≤1 2æ 1 2 ö
x ç 1+ + 2 ÷
 2 2 2 è x x ø
 x + x - 2 x + 1- 2 x + 2 x , x ≥1
= e4
ì1 x £0 218. g(x) = sin x + cos x
ï 2
f ( x ) = í4 x - 4 x + 1 0 £ x £ 1 æ pö
ï1 = 2 sin ç x + ÷
î x ³1 è 4ø
+ - +
ì0 for x < 0 and for x > 1
So, f ¢( x ) = í 3p 7p
î4(2 x - 1) for 0 < x < 1 2π
4 4
3p 7p
213.  lim (1+ ax 2 + bx + c )1/( x -a ) [Since ax2 + bx + c = a(x - a)(x - b)] f [g(x)] is discontinuous at and  .
x ®a 4 4
a( x - b ) 3p

+1, x < 4
lim a( x - b )
= lim [1+ a( x - a )( x - b )] a( x -a )( x - b ) = e x ®a = e a (a - b )
x ®a 
0 , x = 3p
f ( x2 ) - f ( x) æ 0 ö  4
214.   lim ç form ÷ f [ g( x )] = 
x ®0 f ( x ) - f (0 ) è 0 ø  -1, 3p < x < 7p
 4 4
2 xf ′( x 2 ) - f ′( x )  7p
= lim = 0 - 1 = -1 1, x>
x →0 f ′( x )  4
215. The function f is clearly continuous for | x | > 1 219. f(x) = |x - 1| + |x - 3|
1 1 1  -2 x + 4 , x ≤ 1
lim f ( x ) = 1, lim- f ( x ) = = , lim f ( x ) = 1, lim - f ( x ) = 
x ®1+ x ®1 22 4 x ®-1+ x ®( -1) 4 ⇒ f ( x ) = 2, 1< x < 3
2 x - 4 , x ≥3
1 1 
Also, lim+ f ( x ) = 2
, lim- f ( x ) = .

1 n x® 1 (n + 1)2 ⇒ f ′(x) at x = 2 is zero
n n
220. O
 bviously function must be discontinuous. Hence, the
1 correct answer is (A).
Thus, f is discontinuous for x = ± , n = 1, 2, 3, …
n
cos x - cos a æ 0 ö
216. h (x) = g[f(x)] 221.   lim ç ÷
x ®a cot x - cot a è 0 ø
1+ x - [ x ] - sin x
h( x ) = Þ lim = sin3a
1+ x - [ x ] x ® a -cosec 2 x

Mathematical Problem Book for JEE.indb 861 07-06-2018 13:22:00


862 Mathematics Problem Book for JEE

f ( 3 + h ) - f ( 3) 1 1
222.  f ¢(3) = lim r5
n
x6 1
h®0 h Sn = lim å 6 = ò x 5 dx = =
n ®¥
r =1 n 0
6 0
6
f ( 3) × f ( h ) - f ( 3)
= lim
h®0 h
[f (h) - f (0 )]
= lim f (3)
h®0 h
= f (3) × f ¢(0 ) = 3 ´ 11 = 33
cosec x
 1+ tan x 
223.  L = lim  
x → 0  1+ sin x 
0 1
lim æ 1+ tan x - 1- sin x 1 ö
L = e x ®0 ç ´ ÷
è 1+ sin x sin x ø
lim tan x - sin x 1
L = e x ®0 ´ Figure 19.33
1+ sin x sin x
+ -
æ 1 ö æ 1ö æ 1ö
lim ç cos x
- 1÷ Þ Sn = ç ÷ ; Tn = ç ÷
L = e x ®0 ç ÷ è6ø è6ø
è 1+ sin x ø
But since x5 is concave upward (Fig. 19.34) the area included
= e0 = 1 in Sn for any two consecutive values of r is more than area
f(x) excluded in Tn for same values of r. So,
ò 2t dt
æ0ö 1 1 1
224. lim 4 Sn - > - Tn ⇒ Sn + Tn >
x ®1 x - 1 è 0 ø
ç ÷ form 6 6 3

2f ( x ) × f ¢( x )
= lim
x ®1 1
= 2f(1) f ΄(1) included in Sn excluded in Tn
= 8f ΄(1)
a2 - ax + x 2 - a2 + ax + x 2
225. lim
x ®0 a+ x - a- x
On rationalising, we get
Figure 19.34
a2 - ax + x 2 - ( a2 + ax + x 2 ) a+ x + a- x
= lim ×
x →0 a + x - ( a - x) a - ax + x 2 + a2 + ax + x 2
2 x
æ e x + e2 x + e3 x ö
l nç ÷
2 ç ÷
2.  f { g[n, h( x )]} = sinp ([n] + [n] )
3
è ø
-2ax a+ a
= lim ´
x ®0 2x a2 + a2
a 1
=- =- a
a lim f { g[n, h( x )]} = sinp ([n] + [n]2 ) 2
x ®0
lncos x
 2 x lncos x ln x
226.   y = lim  2 sin  = lim (1- cos x ) = lim(1- x ) æ 1 ö ö
1/ 2
p 1 p ö
x →0  2 æ æ
x →0 x →1
Þ lim sin ç p [n] ç 1+ ÷ ÷ = lim sin ç p [n] + - ´ ¼÷ = 1
n ®¥ ç è [n] ø ø ÷ n ®¥ è 2 [n] 8 ø
è
ln(1- x )
Þ ln y = lim ln x ln(1- x ) = lim = 0 (using L ’Hospital rule) æ p 1 ö
x ®1 x ®0 1 lim lim g[n, h( x )] = lim sin ç p [n] + - + ÷
n ®¥ x ® 0 n ®¥ è 2 8[n] ø
ln x
Does not exist as value can be 1 or -1.
Practice exercise 2 æ 1ö æ 1ö
lim tan ç ÷ ln ç ÷ = 0 Þ f ( x ) = 1
3.  
 1 32 243 1
n ®¥ ènø ènø
1.   Sn = lim  6 + 6 + 6 +  + 6  (see Fig. 19.33)
n→∞  n n n n  dx -3 3
Þò 11/ 3 1/ 3
= (tan x )-8 / 3 - (tan x )-2 / 3 + C
sin x cos x 8 2
æ 1 32 243 ( n - 1)5 ö÷
Tn = lim ç 6 + 6 + 6 +  + æp ö 15
n ®¥ ç n n n n6 ÷ø gç ÷ = -
è 4
è ø 8

Mathematical Problem Book for JEE.indb 862 07-06-2018 13:22:28


Chapter 19 | Limit, Continuity and Differentiability 863

2
é 1ù æ 1ö 1
y = lim x 5 ê 3 ú
4.   Also lim f ( x ) = ç ÷ =
x ®0 ëx û x®1 è3ø 9
æ 1 ö é 1ù 1 8. By standard results, the correct answers are options (B) and (C).
x 5 ç 3 - 1÷ < x 5 ê 3 ú < x 5 3
èx ø ëx û x
9. If x ∈ Q, then n!px → multiple of π
x2 - x5 < y < x2 ⇒ y → 0
cos(n!px) → ±1
1/ h
  ch + dh  
3 1+1=2
5.  RHL = lim f ( x ) = lim 1+  2  = e d to exist the limit If x ∉ Q then cos(n! px) be any number between -1 and 1.

x →0 h→ 0 
  h  
lim [1 + [any no between -1 & 1]2m]
m®¥
c = 0,
LHL = lim f (0 - h) [1] = 1
h®0
sin(tan t ) tan t
a(1- h sin h) + b cos h + 5 a + b cos h + 5 ah sin h 10.   lim = lim =1
= lim 2
= lim - t ®0 sin t t ®0 t
h®0 h h ® 0 h2 h2
b sin(cos x )
Limit is possible if a + b + 5 = 0 and -a - = 3 lim =1
2
x →p / 2 cos x

On solving, |x|
lim = DNE
a = -1, d = loge 3, c = 0, b = - 4 x →0 x
1- cos x 4
6. x cos α + y sin α = x cos b + y sin b lim = 2
1/ n
x ®p / 2 x2 p
ïì æ b ö ïü
n
=  2 × lim a í1+ ç ÷ ý = 2a (1) é 1ù é 1ù
11. (A)  lim ê x + ú = 0 , lim ê2 x + ú = 0

n ®¥
ïî è a ø ïþ x ® 0- ë 2û x ® 0 + ë 3û
é b æbö
n ù Hence, lim [f(x)] = 0.
êAs 0 < < 1 Þ ç ÷ ® 0 as n ® ¥ ú x ®0
êë a èaø úû
æ 1ö 1 æ 1ö 1
(B)  lim ç x + ÷ = , lim ç 2 x + ÷ =
Equation (1) shows that α and b are the roots of x cos q + y sin q x ® 0- è 2 ø 2 x ® 0+ è 3ø 3
= 2a(2)
Therefore, Since, f(0-) ≠ f(0+).

x2cos2q = (2a - y sinq )2 Therefore, lim f(x) does not exist.


x® 0
⇒ x2(1 - sin2 q ) = 4a2 - 4ay sin q + y2 sin2 q 
⇒ (x2 + y2) sin2 q - 4ay sin q + 4a2 - x2 = 0 (C) Since, lim f(x) does not exist so é lim f ( x )ù does not exist.
x® 0 êë x ®0 úû
4 ay 4 a2 - x 2
Þ sina + sin b = & sina × sin b = 2 é 1ù é 1ù
êë x + 2 úû êë2 x + 3 úû
2 2
x +y x + y2
(D)  lim = 0 , lim =0
(A)  cos a + cos b = cos a cos b ⇒ 4ax = 4a2 - y2 x ® 0- x x ® 0+ x
⇒ y2 = -4a(x - a) is a parabola. Therefore,
(B) cos a + cos b = sin a + sin b ⇒ 4ax = 4ay [f ( x )]
lim =0
⇒ x = y is a straight line. x® 0 x
(C)  cos a + cos b = sin a sin b ⇒ 4ax = 4a2 - x2 12. (A)  lim f(x) does not exist
⇒x2 + 4ax + 4a2 = 8a2 x ®1

⇒ x + 2 = ±2 2a are pair of lines. 2


(B)  lim f (x) =
x ®1 3
(D) cos a cos b + sin a sin b = 0
⇒ 4a2 - y2 + 4a2 - x2 = 0 ⇒ x2 + y2 = 8a2 is a circle. Hence, f(x) has removable discontinuity at x = 1.
2x (C)  lim f(x) does not exist
æ x ö x ®1
7.  f ( x ) = ç ÷
è2+ x ø -1
(D)  lim f ( x ) =
æ x ö
2x
æ x ö
2x x ®1 2 2
Þ lim f ( x ) = lim ç ÷ = lim ç 1+ - 1÷
x® ¥ x® ¥ è 2+ x ø x® ¥ è 2+ x ø Hence, f(x) has removable discontinuity at x = 1.
æ 2 ö
lim 2 x ç - ÷
æ x ö
lim - 4 ç ÷
13. f(x + y) = f(x) + f(y) + xy (x + y)
=e è 2+ x ø =e è 2+ x ø = e -4 f(0) = 0
x ®¥ x ®¥

Mathematical Problem Book for JEE.indb 863 07-06-2018 13:22:57


864 Mathematics Problem Book for JEE

f ( h) 16. Both x2, -x3/2 have their RHL = 0 and RHD = 0.


Þ lim = -1 Hence, the correct answers are options (A) and (D).
h®0 h

f ( x + h) - f ( x ) f ( x ) + f ( h) + xh( x + h) - f ( x ) AB PA q PA PA
Þ lim = lim 17.   = ⇒ = =
h®0 h h®0 h DC PD sinq PA - DA PA - (1- cosq )
f ( h) ⇒ q · PA - q (1 - cos q ) = PA · sin q
= lim + lim x (x + h) = -1 + x2
h®0 h h®0 q (1- cosq )
Þ PA =
⇒ f ′(x) = -1 + x2 (q - sinq )
x3 q (1- cosq )
⇒ f (x) = -x+c lim =3
3 q ®0+ q - sinq

Hence, f(x) is a polynomial function, f(x) is twice differentiable
OC 1
for all x ∈ R and f ′(3) = 32 - 1 = 8. 18. In DOCQ, cos θ =   =  (Fig. 19.36)
OQ OQ
x -1 x x
1 æ t2 ö 1
14.  f ( x ) = ò | t + 1| dt = - ò (t + 1) dt + ò (t + 1) dt = 2 + çç 2 + t ÷÷ OQ = 
cosq
-2 -2 -1 è ø -1

x2 C
= + x + 1 for x ≥ -1
2
f(x) is a quadratic polynomial.
Therefore, f(x) is continuous as well as differentiable in [-1, 1].
Also f ′(x) is continuous as well as differentiable in [-1, 1]. P O D A Q
ì 1
ï x + 1, 0 £ x < 1
ï
ï2
15.  f (x) = í , 1£ x < 2
ïx Figure 19.36
ï 3 5
ï x - 1, 2 £ x < 2 Area of ∆CPQ = 
1
⋅ PQ⋅DC =
1
(PO + OQ)⋅DC
î
2 2
Clearly, f(x) is discontinuous and bijective function (see
Fig. 19.35). 1
= {PA - OA + OQ}⋅DC
2

1 ìq (1- cosq ) 1 ü
= í - 1+ ý ⋅sin θ
2 î q - sinq cosq þ
3
1 ìï sin2 q (1- q cot q ) üï
= ítanq + ý
2 ïî (q - sinq ) ïþ
2
∆CPQ 1 q sinq - 1+ cosq  1 1
19.   lim+ = lim   = lim
1
q →0 sin2 q q →0+ 2  cosq (q - sinq )  2 h→0 cos h

½ ∆CPQ 1 q sinq - 1+ cosq  1 1  h sin h - 1+ cos h 


lim = lim   = lim lim  
q →0+ sin2 q q →0+ 2  cosq (q - sinq )  2 h→0 cos h h→ 0  h - sin h 
O 1 2 5/2
1  h cos h + sin h - sin h 
= ⋅1⋅ lim   L-Hospital rule
Figure 19.35 2 h→ 0  1- cos h 
1 1  h cos h 
lim f ( x ) = = lim  
x ®1- 2 2 h → 0 1- cos h 
lim f ( x ) = 2
x ®1+ 1  - h sin h + cos h 
= h lim   L-Hospital rule
2 h→ 0  sin h 
min é lim f ( x ), lim+ f ( x )ù  =  1 ≠ f(1)
ëê x ®1- x ®1 ûú 2 1
⇒  = not defined
max (1, 2) = 2 = f(1) 0

Mathematical Problem Book for JEE.indb 864 07-06-2018 13:23:20


Chapter 19 | Limit, Continuity and Differentiability 865

Common Solution for Questions 21–23: 1


Hence, b + c - = |cos x|
n
 
n 2
 x x 
f ( x ) = lim  cos = lim 1+  cos - 1 
n→ ∞  n   
n→ ∞  n   ì 1ü
25. f(x) = max í|sin x |, | cos x |, ý
î 2 þ
 x  1 x
lim  cos -1 n - lim 2 sin2  n
n  n 
Hence, range of f(x) is 
=e
n→∞ 
=e
n→∞ 2 1 
, 1 .
2
 2 
1 x x
2 n 
 1n
-2 lim -2 lim
n→∞ 1 n→∞ 4 1 x
-
=e n =e n =e 2

y = f(x) = e - x/2, x ≥ 0 (range = (0, 1])


n n
g(x) = lim (1- x + x e )
n→ ∞
26.  
( e1/ n -1)
lim x
1/ n
= e = ex "x Î R
n→∞

h(x) = tan-1 (g-1[f -1(x)]


x 1 1
- = ln y ⇒ x = 2 ln ⇒ f -1(x) = 2 ln ; 0 < x ≤ 1
2 y x Figure 19.37
y = g(x) = ex
-1, -1/2 are two points of discontinuity (see Fig. 19.37)
x = ln y ⇒ g-1(x) = ln x
æ 1ö é æ 1 öù
⇒ g-1 ç 2 ln ÷ = ln ê2 ln ç x ÷ ú for 0 < x < 1
è xø ë è øû

é 1 ù
⇒ h(x) = tan-1 êln æç ln ö÷ ú for 0 < x < 1
2
ë è x øû
27.  
ln f ( x ) -x / 2 1
20.   lim+ = lim+ =-
x ® 0 ln g( x ) x ®0 x 2

21. Domain of h(x) is (0, 1).


22. h(x) = tan-1 [ln (ln 1/x2)] (0 < x < 1)
Figure 19.38
1
1 < 1 < ∞ ⇒ 0 < ln <∞ Discontinuous at 0, 1/2 (see Fig. 19.38)
x2 x2
28.  
Hence,
- ∞ < ln [ln (1/x2)] <∞
Therefore, range of h(x) is (-π/2, π/2). 2
n -n
a | sin x | + a | cos x | 1
23.   a = lim lim+
n → ∞ a →1 a n + a -n
| sin x | + a -2n | cos x |
= lim lim+ = | sin x |
n → ∞ a →1 1+ a -2n 2 1 1/2 1
2
a - n | sin x | + a n | cos x |
24.  b = lim lim- Figure 19.39
n → ∞ a →1 a n + a -n
a -2n | sin x | + | cos x | At -1, -1/2, 1/ 2 the function is not differentiable
= lim lim- = | cos x |
n → ∞ a →1 a 2n + 1 (see Fig. 19.39).
p  p 2p (n - 1)p  æ n 4 öæ n 5 ö 1 1
c = lim 1+ cos + cos +  + cos
ç å ÷ç å ÷ ò ò y dy (t + 1)(10 - t ) 4
4 5
n → ∞ 4n 
 2n 2n 2n  x x y dy
29. (A)  lim è nx =1 ø è nx =1 ø = 10 0
= =
 p (n - 1)p  n ®¥ æ
t öæ 9 -t ö
1
30 5
p  sin 4 cos 4 n ç å x ÷ ç å x ÷ ò y dy ò y dy
t 9 -t
 p p 1
= lim  = sin 4 cos 4 = 2 è x =1 ø è x =1 ø 0
n → ∞ 4n  p 0
 sin 
4n ⇒ t2 - 9t + 14 = 0 ⇒ (t - 2) (t - 7) = 0 ⇒ t = 2, 7

Mathematical Problem Book for JEE.indb 865 07-06-2018 13:23:40


866 Mathematics Problem Book for JEE

(B)  x y = y x ⇒ y In x = x In y 1/ x æ f 2 ( a + x )- f ( a ) ö


é f 2 (a + x ) ù lim ç
xf ( a )
÷
Differentiating with respect to x, we get 32. (A)  lim ê ú
x ®0
=e è ø

ë f (a) úû
x® 0ê
( x ln y - y ) y 2 (ln x - 1)
y¢ = = lim
2f ( a + x ) - f ( a + x )
( y ln x - x ) x 2 (ln y - 1) = e x ®0 f (a) = e4
2
y (ln x - 1) y Hence, k = 4.
y ¢ = lim = lim
x ®e 2
x (ln y - 1) x ® e x y¢ cos[tan-1(tan x )]
y ®e y ®e (B)  lim +
p p
⇒ y ′ = ±1 x®
2
x-
2
So, m = ±1 and 3 - m is equal to 2 or 4.
(C)  Put x = y = 1, then é æp öù
cos tan-1 ê tan ç + h ÷ ú
[f  (1)]2 = f  (1) + 6 ⇒ f  (1) = 3, -2 ë è2 øû
lim
3 æ 1+ x ö æ 1+ x ö h®0+ h
Put y = 1, we get f ( x ) = ç ÷ or f ( x ) = - ç ÷.
2è x ø è x ø  p
cos  h - 
m m  2 sin h
(D) If m is even ( 2 + 1) + ( 2 - 1) = 2I , then lim = lim+ =1
h→ 0 + h h→ 0 h
lim {( 2 + 1)n !+ k } = lim {2I - ( 2 - 1)n !+ k }
n→∞ n→∞ é æ x öù
n !+ k sin ê2 cos2 ç ÷ ú
= lim { -( 2 - 1) } =1 sin(cos x + 1) ë è 2 øû æ xö
n→∞ (C)  lim = lim × ç 2 cos ÷
x ®p æxö x ®p æ 2 xö è 2ø
⇒ n! + k must be even integer ⇒ k is even cos ç ÷ ç 2 cos ÷
è2ø è 2ø
30. After rationalizing, we get = 1´ 0 = 0
( a - 2 ) x 3 + ( 3 + c ) x 2 + ( b - 3) x + ( 2 + d )
lim =4 xe sin x - e x sin-1(sin x ) xe x e sin x - x - 1
x ®¥
x 4 + ax 3 + 3 x 2 + bx + 2 + x 4 + 2 x 3 - cx 2 + 3 x - d (D)  lim 2
= lim × =1
x® 0 sin x - x sin x x ® 0 sin x sin x - x
Since limit is finite, so degree of numerator must be 2. So,
a-2=0⇒a=2 33. Given inequality can be written as
Now, dividing numerator and denominator by x2 , we get
f ′′(x) - 2f ′(x) ≥ 3[f ′(x) - 2f (x)]
b -3 2+d
(3 + c ) + + 2 Let f ′(x) - 2f (x) = g(x). Then
lim x x =4
x ®¥ a 3 b 2 2 c 3 d g ′(x) - 3g(x) ≥ 0 (Multiply e-3x)
1+ + 2 + 3 + 4 + 1+ - 2 + 3 - 4 -3x -3x
⇒ [g(x)e  ] ≥ 0 ⇒ g(x)e is non-decreasing
x x x x x x x x
Now
3+c g(0) = f ′(0) - 2f (0) = -2
Þ =4Þc =5
2 g(x)e-3x ≥ -2, ∀x ≥ 0
Hence, a = 2, c = 5 and b, d can be any real number. f ′(x) - 2f (x) ≥ -2e3x, ∀x ≥ 0 (Multiply e-2x )
Hence, the correct answer is (A) → (s); (B) → (p), (q), (r), (s), (t); -2x
⇒ [f (x)e ] ≥ -2e , ∀x ≥ 0
x
(C) → (p); (D) → (p), (q), (r), (s), (t). ⇒ [f (x)e-2x + 2ex] ≥ 0
x2 x2 ⇒ f (x)e-2x + 2ex ≥ 3
x 2
x 2

f (t ) -ft(t ) - t
ò (f (tò) -(ft()tdt) - t ) dt ⇒ f (x) ≥ 3e2x - 2e3x, ∀x ≥ 0
31. (A)  lim òlim ò 2 dt =2 lim dt = 1lim 1 2 Comparing ah(bx) - bh(ax) with 3e2x - 2e3x, we get
1 x -1 1()x - 1 )
(
x ®1 x ® 1 x ®1 x ®( x1 - 1()x - 1)2
h(x) = ex, a = 3, b = 2
2 x (f ( x 2 ) - x 2 ) f ( x 2 ) - x 2 + 2 x 2f ¢( x 2 ) - 2 x 2 ⇒ (a + b) h(0) = 5
= lim = lim =4
x ®1 2( x - 1) x ®1 1 (1+ a3 ) + 8e1/ x
34. RHL = lim+ =2
1+ n 4 
n
 n 4 - 1
n x ®0 1+ (2 + b + b2 )e1/ x
(B)  lim   = lim  1+
n → ∞ 2  n → ∞ 2  ⇒ 2 + b + b2 = 4 ⇒ b2 + b - 2 = 0 ⇒ b = 1
41/ n -1 1 41/ n -1 1 (1+ a3 ) + 8e1/ x
lim ⋅n lim ln 4 LHL = lim- =2
=e n→∞ 2 =e n→∞ 2 1/ n = e2 =2 x ®0 1+ (2 + b + b2 )e1/ x

2x ⇒ 1 + a3 = 2 ⇒ a = 1
(C) f ( x ) = lim tan-1(nx ) = x , x > 0
n®¥ p
35. Let f(x) = y. Then
Hence,
dy
lim [f ( x ) - 1] = lim + [ x - 1] = -1 + y = 4 xe - x × sin 2 x
x ®0 + x ®0 dx
é n 1ù I.F = ex
é 1ù
(D)  lim ê å r ú = lim ê1- n ú = 0
ye x = 4 ò x sin 2 x dx
ë r =1 2 úû ë 2 û
n ® ¥ê n ® ¥
I II

Mathematical Problem Book for JEE.indb 866 07-06-2018 13:24:03


Chapter 19 | Limit, Continuity and Differentiability 867

é æ cos 2 x ö 1 ù e -p 1
ye x = 4 ê x ç - ÷ + ò cos 2 xdx ú S(1- e -p ) = -p
= p
ë è 2 ø 2 û 1- e e -1
x æ x cos 2 x sin 2 x ö 1 ep
= ye = 4 ç - + ÷+c S= = p
è 2 4 ø p -p
(e - 1)(1- e ) (e - 1)2
⇒ yex = (sin 2x - 2x cos 2x) + c
f(0) = 0 ⇒ c = 0 Therefore,
Therefore, y = e-x (sin 2x - 2x cos 2x)
n
-2p ep
lim å f (kp ) =
Now, n ®¥
k =1 (ep - 1)2
f (kp) = e-kp
(sin 2kp - 2kp cos 2kp) = e-kp (0 - 2kp) Hence, p = 2.
f (kp) = -2p(ke-kp ) 1
¥ 1 tan x x
-1
å f (kp ) = 2p å ke - kp
36.   lim ([f ( x )] +
x →0
x 2 ) { f ( x )} = lim (1+ x
x →0
2
) x = lim(1+ x 2 ) tan x - x
x →0
k =1

S = 1× e -p + 2e -2p + 3e -3p +  ¥ lim


x3
lim
3x2
lim
6x
= e x ®0 tan x - x = e x ®0 sec = e x ®0 2 sec = e3
2 2
x -1 x tan x
-2p -3p
S = e + 2e + ¥
_____________________________ Therefore, l = 3.
-p -p -2p -3p
S(1- e )=e +e +e + ¥

Mathematical Problem Book for JEE.indb 867 07-06-2018 13:24:12


868 Mathematics Problem Book for JEE

Solved JEE 2017 Questions


JEE Main 2017 3. lim
3x - 3
is equal to
x →3 2x - 4 - 2
cot x - cos x
1. lim equals 3
x→
p (p - 2 x )3 (A) 3 (B)
2 2
1 1 1 1
(A) (B) (C) (D)
16 8 2 2 2
1 1 (ONLINE)
(C) (D)
4 24 Solution: It is given that
(OFFLINE)
3x - 3
Solution: We have lim
x →3 2x - 4 - 2
cot x (1- sin x ) Rationalizing it, we get
lim 3
p p
x→ 
2 -8  x -  3x + 3 1 2x - 4 + 2
 2 lim 3 x - 3 × × ×
x →3 3x + 3 2x - 4 - 2 2x - 4 + 2
p   p  p   2p 
tan  - x 
2   1- cos  2 - x   tan  - x 
2   sin  2 - x   ( 3 x )2 - 32 ( 2x - 4 + 2)
lim = lim ⇒ lim ×
p  p  p 
2 p  p  p 
2 x →3 3x + 3 ( 2 x - 4 )2 - ( 2 )2
x→
2 8 - x - x x→
2 8 - x 2  - x
2   2  2  2  3x - 9 ( 2x - 4 + 2 )
⇒ lim ×
1 1 1 x →3 3x + 3 2x - 4 - 2
         = × 1× =
8 2 16
3( x - 3) 2x - 4 + 2
Hence, the correct answer is option (A). ⇒ lim ×
x →3 3x + 3 2x - 6

 1  6x x  3( x - 3) 2x - 4 + 2
2. If for x ∈ 0 ,  , the derivative of tan-1   is x ⋅ g( x ), ⇒ lim ×
 4  1- 9 x 3  x →3 3x + 3 2( x - 3)
then g(x) equals
3 2x - 4 + 2 3 2×3-4 + 2
3x x 3x ⇒ lim = ×
(A) 3 (B) x →3 2 3x + 3 2 3×3 +3
1- 9 x 1- 9 x 3
3 9 3 ( 2 + 2)
(C) 3 (D) = ×
3 2 3+3
1+ 9 x 1+ 9 x
(OFFLINE) 3 2 2 2 2 2 1
= × = = =
Solution: We have 2 6 4 2 2

 6x x   2(3 x x )  Therefore,
y = tan-1  3
= tan-1   3x - 3 1
 1- 9 x   1- (3 x x )2  lim =
x →3 2x - 4 - 2 2
   = 2 tan-1(3 x x )
Hence, the correct answer is option (D).
Now, differentiating w.r.t. x, we get

1a + 2a +  + na 1
dy 2  1  4. If lim = for
= 3( x) + x (1) n→∞ [( n + 1) a -1
[(na + 1) + (na + 2) +  + (na + n)] 60
dx 1+ (3 x x )2  2 x 
some positive real number a, then a is equal to
6  x  9 x  9 
    =  2 + x = = x = x ⋅ g( x ) 17
3
1+ 9 x   1+ 9 x 3  1+ 9 x 3  (A) (B) 8
2
9 15
Therefore, g( x ) = . (C)
7 (D)
1+ 9 x 3 2
Hence, the correct answer is option (D). (ONLINE)

Mathematical Problem Book for JEE.indb 868 07-06-2018 13:24:43


Chapter 19 | Limit, Continuity and Differentiability 869

Solution: The given limit is 17 3


(A) (B)
a a a a 20 5
1 + 2 + 3 ++ n 1
lim = 2 2
n→∞ ( n + 1)a -1[( na + 1) + ( na + 2) +  + ( na + n)] 60 -
(C) (D)
5 5
1a + 2a +  + na 1
⇒ lim = (ONLINE)
n→∞ a -1  2 n(n + 1)  60
(n + 1) n a +
 2  Solution: We have
  n tan 4 x
2 n∑ k a   4  tan 5 x
 k =1  1 f (x) =  
⇒ lim =  5
n→∞ ( n + 1)a -1[2n2a + n2 + n] 60
tan 4 x
n a  4  tan 5 x 2
 k lim   =k+
2∑   x  5 5
k =1
 n 1 x→
⇒ lim = 2
n→∞  a -1 60
1 tan 4 x cot 5 x
 1+  [2na + n + 1]  4 2
a ⇒ lim   =k+
x  5 5
x→
a 2
2 n  k
∑ 
a k =1 n  1  4  x→ x
lim(tan 4 x . cot( 5 x ))
2
⇒ lim = ⇒  2 =k+
n→∞  n -1 60  5 5
1  1
 1+  2a + 1+ n   x
n 0 × cos  2 x + 
 4  2 2
⇒  =k+
1 n a 1  5 5
 k
⇒ lim
n→∞ n
∑  n  = ∫ x adx   (derivative as limit of a sum) 0
k =1 0  4 2
⇒  =k+
 5 5
Therefore,
1 2 2 3
⇒k+ = 1 ⇒ k = 1- ⇒ k =
2∫ x adx 5 5 5
0 1
= Hence, the correct answer is option (B).
2a + 1 60

2( x a +1)]10 1 JEE Advanced 2017


⇒ =
(a + 1)(2a + 1) 60 1. Let [x] be the greatest integer less than or equals to x. Then, at
2 1 which of the following point(s) the function f(x) = xcos(p (x +
⇒ =
(a + 1)(2a + 1) 60 [x])) is discontinuous?
⇒ (a + 1) (2a + 1) = 120 (A)
x = −1 (B) x = 0
(C)
x = 1 (D) x = 2
⇒ 2a2 + 3a + 1 = 120
Solution: It is given that
⇒ 2a2 – 3a = 119 = 0
f(x) = xcos (p (x + [x]))
-3 ± 9 + 952 -3 ± 961 -3 ± 31 and    f(x) = (−1)[x] xcos p x
⇒a= = =  (since a > 0)
4 4 4
Hence, the value of a is This function is discontinuous at all integral points except x = 0.
At x = z; z = 0, ±1, ±2, …
-3 + 31 28
= =7⇒a=7 f(z) = zcos p (2z) = z
4 4
f(z+) = zcos p (2z) = z
Hence, the correct answer is option (C).
f(z−) = zcos p (2z – 1) = −z
5. The value of k for which the function f(0) = 0
 tan 4 x
Hence, the correct answers are options (A), (C) and (D).
 4  tan 5 x , 0 < x < p
 
f ( x ) =  5  2 2. Let f:  →(0,1) be a continuous function. Then, which of the
 2 p following function(s) has(have) the value zero at some point
 k+ , x=
in the interval (0, 1)?
 5 2
p
p -x
   is continuous at x = is x9 – f(x)
(A) (B) x - ∫ 2 f (t )cos t dt
2 0

Mathematical Problem Book for JEE.indb 869 07-06-2018 13:25:08


870 Mathematics Problem Book for JEE

x p /2 Therefore,
e x - ∫ f (t )sin t dt
(C) (D) f ( x ) + ∫ f (t )sin t dt
0 0 p /2
d
Solution: We discuss the options as follows: g( x ) = ∫ dt
(f (t )cosec t )dt
x
Option (A): Let g(x) = x9 – f(x).
p /2
⇒ g( x ) = f (t ) cosec t x
          g(0) = −f(0) < 0 [as f ∈(0 ,1)]
p  p 
g(1) = 19 – f(1) = 1 – f(1) > 0   [as f ∈(0 ,1)] g( x ) = f   cosec   - f ( x ) cosecx
 2  2
Hence, option (A) is correct. It is given that f(p/2) = 3 and cosec(p/2) = 1. Therefore,
p /2- x
Option (B): Let g( x ) = x - ∫ f (t )cos t dt.
g( x ) = 3 - f ( x ) cosec x =
3 - f (x)
0 sin x
p /2-0
Now,
g(0 ) = 0 - ∫ f (t )cos t dt < 0
 3 - f ( x ) f (x)
0 lim g( x ) = lim   = 3 - lim
x →0 x → 0  sin x  x → 0 sin x
p / 2 -1
and        g(1) = 1- ∫ f (t )cos t dt > 0 Since f(0) = 0 and sin 0 = 0, we get
0
Hence, option (B) is correct. f (x) 0
lim =
x x →0 sin x 0
Option (C): Let g( x ) = e x - ∫ f (t )sin t dt .
d
0 Taking derivative: Using sin x = cos x, we get
Differentiating w.r.t. x, we get dx

f ′( x ) 1
g′( x ) = e x - f ( x )sin x (1) lim =        [as f ′(0 ) = 1 given]
Now x →0 cos x 1
g(0) = 1
Therefore, lim g( x ) = 3 - 1 = 2.
x →0
Also, we know that f ( x ) ∈(0 ,1) ⇒ 0 < f(x) < 1 ⇒ 0 < f(x)sinx < 1
Hence, the correct answer is (2).
Therefore, from Eq. (1), we get

g ′( x ) > 0 4. If f: R → R is a twice differentiable function such that f ′′( x ) > 0


 1 1
Thus, g(x) is strictly an increasing function. for all x ∈R, and f   = , f (1) = 1, then
 2 2
Hence, option (C) is incorrect.
p /2 1
f ′(1) ≤ 0
(A) (B) 0 < f ′(1) ≤
Option (D): Let g( x ) = f ( x ) + ∫ f (t )sin t dt
1
2
0 (C) < f ′(1) ≤ 1 (D) f ′(1) > 1
2
Since f ( x ) ∈(0 , 1), we get g(x) > 0.
Hence, option (D) is incorrect. Solution: It is given that
Hence, the correct answers are options (A) and (B).
 1 1
f ′′( x ) > 0 and f  2  = 2 , f(1) = 1
3. Let f:  ®  be a differentiable function such that f(0) = 0,
p  p /2 Using Lagrange’s mean value theorem, let us consider that
f   = 3 and f ′(0 ) = 1. If g( x ) = ∫ [f ′(t )cosec t - cot t cosec t f (t )]dt
 2 x é1 ù 1 
f : ê , 1ú ®  be continuous and differentiable on  , 1 , then
p /2  p ë2 û 2 
= ∫ [f ′(t )cosec t - cot t cosec t f (t )]dt for x ∈ 0 ,  , then x→0
lim g(x) = ______.
x  2 1 
there exists c ∈ , 1 such that
Solution: It is given that 2 
p /2
 1
g( x ) = ∫ [f ′(t ) cosec t - cot t cosec t f (t )]dt f (1) - f   1- 1
 2 2 =1⇒
x f ′( c ) = = f ′( c ) = 1
1 1
1-
We know that 2 2
d Since f ′( x ) is increasing function for x ∈R, we get f ′(1) > 1.
(f (t ) cosec t ) = f ′(t ) cosec t - f (t ) cosec t cos t
dt Hence, the correct answer is option (D).

Mathematical Problem Book for JEE.indb 870 07-06-2018 13:25:49


Chapter 19 | Limit, Continuity and Differentiability 871

sin( 2 x ) Using sin2x + cos2x = 1, we get


5. If g( x ) = ∫ sin-1(t )dt , then
sin x
f ( x ) = cos 2 x - cos 2 x ( - cos2 x + sin2 x ) + sin 2 x ( -2 sin x cos x )
p   p
g′   = -2p
(A) (B) g′  -  = -2p = cos 2 x - cos 2 x ( - cos 2 x ) + sin 2 x ( - sin 2 x )
 2  2
p   p = cos 2 x + cos2 2 x - sin2 2 x
g′   = 2p
(C) (D) g′  -  = -2p
 2  2 Using cos2x + sin2x = 1 and sin22x = 1 – cos22x, we get
Solution: It is given that
f ( x ) = cos 2 x + cos2 2 x - (1- cos2 2 x )
sin( 2 x )
g( x ) = ∫ sin-1(t ) dt     = cos 2 x + cos2 2 x - 1+ cos2 2 x
sin x Therefore,
Differentiating this equation, we get
f ( x ) = 2 cos2 2 x + cos 2 x - 1 = cos 4 x + cos 2 x
g′( x ) = [sin-1(sin 2 x )]2 cos 2 x - [sin-1(sin x )]cos x Differentiating this, we get
Therefore,
f ′( x ) = 4( - sin 4 x ) + 2( - sin 2 x )

 f ′( x ) = -4 sin 4 x - 2 sin 2 x
p   p  p
g′   = [sin-1(sinp )]2 cos p - sin-1  sin   cos
 2   2 2 Now f ′( x ) = 0 gives
−4sin4x – 2sin2x = 0
p 
⇒ g′   = 0 Using sin2x = 2sinx cosx, we get
 2
 p   p p -4(2 sin 2 x cos 2 x ) - 2 sin 2 x = 0
and g′  -  = [sin-1( - sinp )]2 cos p - sin-1  - sin  cos  = 0
 2   2 2 -8 sin 2 x cos 2 x - 2 sin 2 x = 0
There is no correct option. -2 sin 2 x ( 4 cos 2 x + 1) = 0
p -p
cos(2 x ) cos(2 x ) sin(2 x ) ⇒ 2sin2x = 0 ⇒ sin2x = 0 ⇒ 2x = 0, p, −p ⇒ x = 0 , ,
2 2
6. If f ( x ) = - cos x cos x - sin x , then
and
sin x sin x cos x -1  -1
4cos2x + 1 = 0 ⇒ cos2 x = ⇒ 2 x = cos -1   ⇒ 2x = 1.8 + 2p n
f ′( x ) = 0 at exactly three points in (−p, p ).
(A) 4  4
(B) f ′( x ) = 0 at more than three points in (−p, p). where n = …, −2, −1, 0, 1, 2, …, which gives 4 points in the range
(C)
f(x) attains its maximum at x = 0. (−p, p ).
(D)
f(x) attains its minimum at x = 0. Thus, the total points in (−p, p ) range are 7.
Solution: It is given that Now,

cos 2 x cos 2 x sin 2 x f ′( x ) = [ -4(cos 4 x )4 ] - [2 cos(2 x )2] = -16 cos 4 x - 4 cos 2 x


f ( x ) = - cos x cos x - sin x At x = 0, we get
sin x sin x cos x f ′′( x ) = -16 - 4 = -20       (maxima)
2 2 2 2 Thus, at x = 0, the function f(x) attains maximum at x = 0.
⇒ f ( x ) = cos 2 x (cos x + sin x ) - cos 2 x ( - cos x + sin x )
+ sin 2 x ( - sin x cos x - sin x cos x ) Hence, the correct answers are options (B) and (C).

Mathematical Problem Book for JEE.indb 871 07-06-2018 13:26:18


Mathematical Problem Book for JEE.indb 872 07-06-2018 13:26:18
20 Differentiation

20.1  Introduction tan 3( x + h) - tan 3 x


f ′( x ) = lim
The rate of change of one quantity with respect to some another h h→ 0
where h is small increment in x.
quantity has a great importance in mathematics. The rate of
change of a quantity y with respect to another quantity x is called Multiplying and dividing by  tan(3 x + 3h) + tan 3 x  , we get
the derivative or differential coefficient of y with respect to x. tan(3 x + 3h) - tan 3 x
f ′( x ) = lim
h→ 0 h  
 tan(3 x + 3h) + tan 3 x 
20.2 Differentiation from First Principle tan(3 x + 3h - 3 x )[1+ tan(3 x + 3h)tan 3 x ]
When the derivative of a function is calculated directly by using = hlim
→0 h  tan(3 x + 3h) + tan 3 x 
the definition of derivative, it is called differentiation from first
∵ tan A - tan B 
principle. This method is also known as ab-initio method or delta   = tan( A - B )(1+ tan A tan B )
method.  
f ( x + h) - f ( x ) tan 3h [1+ tan(3 x + 3h)tan 3 x ]
f ′( x ) = lim = lim ⋅3⋅
h→ 0 h h→ 0 3h tan(3 x + 3h) + tan 3 x
-
3 + tan2 3 x 3 sec2 3 x
1+
Illustration 20.1  Differentiate x 2, with respect to x, from the = 1× 3 × =
2 tan 3 x 2 tan 3 x
first principle. 3 x
- Illustration 20.3  Differentiate e , with respect to x, from the
Solution: Let f ( x ) = x 2 . Then
first principle.
3 3
- - x
( x + h) 2 -x 2 Solution: Let f ( x ) = e . Then
f ′( x ) = lim
h→ 0 h x +h x
e -e
f ′( x ) = lim
where h is small increment in x. Now, h→ 0 h
where h is small increment in x.
 3  e - 1
3 - x x +h - x
-
  h 2  e 
x 2
 1+  - 1 f ′( x ) = lim 
 x  
  h→ 0 h
f ′( x ) = lim x +h - x
h→ 0 h x e -1 x + h - x
= lim e ⋅ ⋅
  3  5  h→ 0 x+h - x h
- -
3 h  2   2  h2
3
-  
x 1- ⋅ +
2 +  - 1 x x +h- x x 1 e x
 2 x 2! x 2
 =e ⋅1⋅ lim =e ⋅ lim =
h→ 0 h[ x + h + x ] h→ 0 x+h+ x 2 x
 
= lim Illustration 20.4  Find the derivative of tan−1x with the first
h→ 0 h
principle.
  3  5   p p
Solution: Let tan−1x = q, q ∈ - ,  . Then
-
3  3  - 2   - 2  h  -
3
 3 3 -
5  2 2
= lim x 2 - + ⋅ 2 +  = x 2 ×  -  = - x 2 x = tanq(1)
h→ 0  2x 2! x   2x  2
  and tan−1 (x + h) = q + ∆q
⇒ x + h = tan (q + ∆q )(2)
Illustration 20.2  Differentiate tan3x , with respect to x, from tan-1( x + h) - tan-1 x
the first principle. Let lim = L . Then
h→ 0 h
Solution: Let f ( x ) = tan 3 x . Then q + ∆q - q ∆q
L = lim = lim  [from Eqs. (1) and (2)]
h→ 0 h h→ 0 h

Mathematical Problem Book for JEE.indb 873 07-06-2018 13:26:39


874 Mathematics Problem Book for JEE

∆q ∆q In general
  = lim = lim
h→ 0 tan(q + ∆q ) - tanq ∆q → 0 tan(q + ∆q ) - tanq d du dv dw
(u ± v ± w ± ) = ± ± ±
( ∆q ) cos(q + ∆q ) cosq 1 1 dx dx dx dx
  = lim = cos2q = =
∆q → 0 sin ∆q sec q 1+ x 2
2 Illustration 20.5  Differentiate 5 sin x - 2 loge x .
Solution:
20.3 Derivatives of Some of the d d d 2
Frequently Used Functions
dx
(5 sin x - 2 loge x ) = (5 sin x ) - (2 loge x ) = 5 cos x -
dx dx x

Function Derivative 2. Product rule:


c (constant) 0 d (uv ) dv du
=u +v
sinx cosx dx dx dx
cosx −sinx d [(sin x ) e x ] de x d (sin x )
= sin x + ex = (sinx) ex + (cosx) ex
tanx sec2x dx dx dx
In general
cosecx −cosecx cotx
d  1 du1 1 du2 1 du3 
cotx −cosec2x (u1u2u3 …) = (u1u2u3 …)  + + + 
dx u
 1 dx u2 dx u 3 dx 
secx secx tanx
1 Illustration 20.6  Differentiate x 2e x sin x .
sin−1x
1- x 2 Solution: First we differentiate x 2e x , that is,
d 2 x d d 2
-1 ( x e ) = x 2 (e x ) + e x ( x ) = x 2e x + 2 xe x
cos−1x dx dx dx
1- x 2
Now,
1 d 2 x d d
tan−1x ( x e sin x ) = x 2e x (sin x ) + sin x ( x 2e x )
1+ x 2 dx dx dx
-1 = x 2e x cos x + sin x ( x 2 + 2 x ) e x
cot−1x
1+ x 2 = e x ( x 2 cos x + x 2 sin x + 2 x sin x ) = xe x ( x cos x + x sin x + 2 sin x )
1 3. Quotient rule: Here v(x) ≠ 0
sec−1x
| x | x2 -1 du dv
v -u
d  u dx dx
-1  =
cosec−1x dx  v  v2
| x | x2 -1 d (tan x ) dx
x - (tan x )
d  tan x  dx dx
logex 1/x  =
dx  x  x2
logax (1/x) loga e
x sec2 x - tan x
xn nxn−1 =
x2
ax ax logea
ex
ex ex Illustration 20.7  Differentiate .
1+ sin x
Solution:
The above written derivatives can be easily found by using the
d d
definition of differentiation. (1+ sin x ) (e x ) - e x (1+ sin x )
d  ex  dx dx
 =
dx  1+ sin x  (1+ sin x )2
20.4  Rules to Find Out Derivatives
(1+ sin x ) e x - e x cos x e x (1+ sin x - cos x )
Let u and v are differentiable functions of x. Then following are the = =
(1+ sin x )2 (1+ sin x )2
rules to find derivatives:
4. Chain rule: The chain rule is probably the most widely used
1. Sum rule: differentiation rule in mathematics. Chain rule says that the
d du dv derivative of the composition of two differentiable functions
(u + v ) = +
dx dx dx is the product of their derivatives evaluated at appropriate
points. The formula is
d de x d (loge x ) 3
(2e x + 3 loge x ) = 2 +3 = 2e x +
dx dx dx x {f [ g( x )]}′ = f ′[ g( x )] ⋅ g′( x )

Mathematical Problem Book for JEE.indb 874 07-06-2018 13:27:10


Chapter 20 | Differentiation 875

Illustration 20.8  Differentiate sin x 2 .  1


  = e - ax cos( x loge x ) ⋅ 1⋅ loge x + x ⋅  + sin( x loge x )e - ax ( -2ax )
2 2

 x
Solution: Put y = x 2 and z = sin y . Then
  = e - ax cos( x loge x )(loge x + 1) - 2axe - ax sin( x loge x )
2 2

dy dz
= 2 x and = cos y cot x dy
dx dy Illustration 20.11  If y = 1- x 2 + , find .
x dx
Therefore, Solution:
d dz dz dy dy d d  cot x 
(sin x 2 ) = = ⋅ = (cos y ) (2 x ) = 1- x 2 +  
dx dx dy dx dx dx dx  x 
= (cos x 2 )(2 x ) = 2 x cos x 2 d d
(cot x ) - cot x ⋅
x (x)
d 1- x 2 d (1- x 2 )
dx dx
This solution can be rewritten using a more convenient notation in
= ⋅ +
d (1- x 2 ) dx ( x) 2
the following manner: 2 1
x ( - cosec x ) - cot x ⋅
d d (sin x 2 ) d ( x 2 ) 1 2 x
(sin x 2 ) = ⋅ = cos x 2 ⋅ 2 x = 2 x cos x 2 = ( -2 x ) +
dx d( x 2 ) dx 2 1- x 2 x
. Differentiation of parametrically defined functions:
5 x 2 x cosec2 x + cot x
= - -
• Working rule: 1- x 2 2x3 2
dx
(a) If x and y are functions of parameter q , then find and
dq
dy
respectively. Your Turn 1
dq
(b) Now dy x2 + 1
dy dy dq 1. Find , if y = .
= × dx x
dx dq dx dy x 2 - 1
For example, Ans. =
dx x2
x = sint + cost dy x3 -1
y = cost 2. Find , if y = .
dx x
dx dy
= cos t - sin t ; = - sin t dy 2 x 2 + 1
dt dt Ans. =
dy dx x2
dy - sin t 3. Find of
⇒ = dx
dx cos t - sin t (a) y = (x + 2)(x + 3)
dy 3x + 4
Illustration 20.9  Find , if x = a(q - sinq ); y = a(1- cosq ) . (b) y =
dx 4x + 5
Solution: Consider
x = a(q - sinq ); y = a(1- cosq ) (c) y = ax 2 + 2bx + c
dy dy 1
dx dy Ans. (a) = 2x + 5; (b) =- ;
⇒ = a(1- cosq ); = a(0 + sinq ) = a sinq dx dx ( 4 x + 5)2
dq dq
dy
(c) = (ax 2 + 2bx + c )-1/ 2 (ax + b )
dx
Therefore, dy
4. If x = e -t and y = tan−1 (2t + 1), find
2
q q .
a sinq 2 sin cos dx
dy dy dx
= ÷ = = 2 2 = cot q 2

dx dq dq a(1- cosq ) 2q 2 dy -et


2 sin Ans. =
2 dt 2t (2t 2 + 2t + 1)
dy . Find dy/dx:
5
Illustration 20.10  If y = e - ax sin( x loge x ), find
2
.  x 
dx (a) sin cos x (b) sin  log
Solution:  x + 1
dy d - ax 2
=
dx dx
{e sin( x loge x )} (
(c) log x + e
x
)
dy sin x ⋅ cos x cos x
- ax 2
Ans. (a) =- ;
d sin( x loge x ) d (e ) dx 2 cos x
= e - ax
2
       + sin( x loge x ) ⋅
dx dx dy 1  x  1
2 d sin( x log x ) d  (b) = cos  log  ;
= e - ax e
⋅ ( x loge x ) dx 2  x + 1 x ( x + 1)
d ( x loge x ) dx
dy 2 x +e x
d (e - ax ) d (c) =
2

+ sin( x loge x ) ⋅ ( - ax 2 )
d ( - ax 2 ) dx (
dx 2 x x + e x
)

Mathematical Problem Book for JEE.indb 875 07-06-2018 13:27:57


876 Mathematics Problem Book for JEE

6. Differentiation of implicit function: If a relation between x Hence,


and y is such that y cannot be expressed in terms of x, then y dy
= 2 x 2 + y 2 - 3 xy
is called an implicit function of x. Here we will give method to dx
dy
find if y is an implicit function of x. dy
dx Illustration 20.14  If x 3 y 2 = loge ( x + y ) + sin(e x ), find .
•• Working rule: dx
Solution:
(a) Differentiate the given relation between x and y with
respect to x. x 3 y 2 = loge ( x + y ) + sin(e x )(1)
dy
(b) Bring all the terms containing on left-hand side and Differentiating with respect to x, we get
dx dy
remaining terms on right-hand side and then find . dy 1  dy 
dx 3x2 y2 + x3 ⋅ 2y = x
 1+  + cos(e ) ⋅ e
x
dx x + y  dx 
(c) Use the given relation between x and y to get the result in
simplified form. dy 1 1 dy
or 3 x 2 y 2 + 2 x 3 y = + ⋅ + e x cos(e x )
dy dx x + y x + y dx
Illustration 20.12  If y = tan( x + y ), find .
dx  1  dy 1+ ( x + y )e x cos(e x ) - 3 x 2 y 2 ⋅ ( x + y )
Solution: or 2 x 3 y -  =
 x + y  dx x+y
y = tan( x + y ) (1)
dy 1+ ( x + y )e x cos(e x ) - 3 x 3 y 2 - 3 x 2 y 3
Differentiating with respect to x, we get or =
dx 2x 4 y + 2x3y2 -1
dy d d d
= [tan( x + y )] = tan( x + y ) ⋅ ( x + y ) 7. Logarithmic differentiation: When u and v both are func-
dx dx d( x + y ) dx
tions of x, then derivative of a function of the form uv cannot
 dy  dy be found directly by using standard formula and hence in such
= sec2 ( x + y ) ⋅  1+  = sec2 ( x + y ) + sec2 ( x + y )
 dx  dx cases both sides are differentiated after taking logarithm. This
dy process is called logarithmic differentiation.
or [1- sec2 ( x + y )] = sec2 ( x + y ) dy
dx Illustration 20.15  If y = x x , then find .
dx
Hence, Solution: [Here power is variable]

dy sec2 ( x + y ) 1+ tan2 ( x + y ) 1+ y 2 Given,


= = = y = x x (1)
2 2
{
dx 1- sec ( x + y ) 1- 1+ tan ( x + y ) 1-- (1+ y 2 ) } Taking logarithm we get,
2
1+ y loge y = loge ( x x )
=- 2
[from Eq. (1), y = tan( x + y )]
y or loge y = x loge x
1 dy Differentiating with respect to x, we get
Illustration 20.13  If x = y + , prove that = 2 x 2 + y 2 - 3 xy d d
dy y+1 dx (loge y ) = ( x loge x )
= 2 x 2 + y 2 - 3 xy .
dx y +1
y + ... to ∞
dx dx
Solution: d d( x ) d
or    (loge y ) = ⋅ loge x + x (loge x )
1 dy dx dx
x= y+
y+1 1 dy 1
y +1 or ⋅ = 1⋅ loge x + x ⋅ = loge x + 1
y + ... to ∞ y dx x
Hence,
Hence,
1
x = y + (1) dy
x = y (1+ loge x ) = x x (1+ loge x ) [from Eq. (1), y = x x ]
Differentiating with respect to x, we get dx
dy 1 dy 1 dy
1= - or = 1+ 2 Illustration 20.16  If x m y n = ( x + y )m + n , then find .
dx x 2 dx x dx
dy 1 Solution:
 
or = 1+ ( x - y )2  As from Eq. (1), = x - y 
dx  x  x m y n = ( x + y )m + n
= 1+ x 2 + y 2 - 2 xy (2) Taking logarithm, we get

From Eq. (1), loge ( x m ) + loge ( y n ) = ( m + n)loge ( x + y )


2 2
x = xy + 1, 1 = x - xy
or m loge x + n loge y = ( m + n)loge ( x + y )
Putting in Eq. (2), we get Differentiating with respect to x, we get,
dy m n dy m + n  dy  m + n m + n dy
= x 2 - xy + x 2 + y 2 - 2 xy + =  1+  = +
dx x y dx x + y  dx  x + y x + y dx

Mathematical Problem Book for JEE.indb 876 07-06-2018 13:28:41


Chapter 20 | Differentiation 877

 n m + n  dy m + n m  5
or  y - x + y  dx = x + y - x where f = cos -1  
 13 
nx + ny - my - ny dy mx + nx - mx - my  5   5
= = f − q as x ∈ , 1 = cos -1   - cos -1 x
y( x + y ) dx x( x + y )  13   13 
nx - my dy nx - my
or = Hence,
y ( x + y ) dx x ( x + y )
1  1 2
f ′( x ) = ⇒ f ′  =
Hence, 2  2  3
dy y 1- x
=
dx x
Illustration 20.19  Differentiate cos -1( 4 x 3 - 3 x ), where 0 ≤ x ≤ 1.
dy
Illustration 20.17  If y = e x sin x 3 + (tan x ) x, then find . Solution: Put x = cosq , then
dx
x 3 x
Solution: Let u = e sin x and v = (tan x ) .   p
 3q , q ∈0 , 
Now,   3
cos -1( 4 x 3 - 3 x ) = 
u = e x sin x 3 2p - 3q , q ∈ p , p 

Differentiating with respect to x, we get  3 2 

du d[sin( x )3 ] d d 3 1
= ex⋅ + sin x 3 ⋅ (e x ) [cos -1( 4 x 3 - 3 x )] = - if < x ≤ 1
dx dx dx dx 1- x 2 2
3 1
= e x ⋅ cos x 3 ⋅ 3 x 2 + sin x 3 ⋅ e x = if 0 ≤ x <
1- x 2 2
Hence,
du
= 3 x 2e x cos x 3 + e x sin x 3 20.5 Derivative of Second Order y or y2
dx
and v = (tan x ) x d2 y d  dy 
=   is the derivative of second order and is denoted
Hence, dx 2 dx  dx 
by y ′′or y 2
loge v = x loge (tan x )
Illustration 20.20  If y = loge (loge x ), find y 2.
Differentiating with respect to x, we get
1 dv 1 Solution: Let y = loge (loge x ). Then
= 1⋅ loge (tan x ) + x ⋅ sec2 x 1 1 1
v dx tan x y1 = ⋅ =
Hence, loge x x x loge x
dv d  1  d d
= v [loge (tan x ) + x cot x ⋅ sec2 x ] y2 = = ( x loge x )-1 = -1( x loge x )-2 ⋅ ( x loge x )
dx dx  x loge x  dx dx
= (tan x ) x [loge (tan x ) + x cot x sec2 x ]
1  1  -(1+ loge x )
=- ⋅  x ⋅ + loge x ⋅1 =
Now 2
( x loge x )  x  ( x loge x )2
y =u+v
Illustration 20.21  If y = a cos(loge x ) + b sin(loge x ), prove that
Hence, x 2 y ′′ + xy ′ + y = 0 .
dy du dv
= +
dx dx dx Solution: Consider
2 x 3 x 3 x 2
= 3 x e cos( x ) + e sin( x ) + (tan x ) [loge (tan x ) + x cot x sec x ] y = a cos(loge x ) + b sin(loge x ) (1)
8. Differentiation by substitution: Sometimes, it is easier to Differentiating with respect to x, we get
differentiate, by making substitutions. Usually these examples
 1  1
involve inverse trigonometric functions. y ′ = a ⋅  - sin(loge x ) ⋅  + b cos(loge x ) ⋅ 
 x  x
 5 x + 12 1- x 2  5
Illustration 20.18  If f ( x ) = cos -1  , < x < 1, ⇒ xy ′ = - a sin(loge x ) + b cos(loge x )
 13  13
Again differentiating with respect to x, we get
 1  1  1
then find f ′   . xy ′′ + y ′ ⋅1 = - a ⋅ cos(loge x ) ⋅  + b ⋅  - sin(loge x ) ⋅ 
 2  x  x
Solution: Put x = cosq , then
⇒ x 2 y ′′ + xy ′ = -[a cos(loge x ) + b sin(loge x )] = - y [from Eq. (1)]
 5 cosq + 12 sinq 
  f ( x ) = cos -1  -1
 > 0 = cos [cos(q - f )] > 0,
 13 ⇒ x 2 y ′′ + xy ′ + y = 0

Mathematical Problem Book for JEE.indb 877 07-06-2018 13:29:30


878 Mathematics Problem Book for JEE

20.6 Differentiation of a Function with (c)


d
sec -1 x =
1
, for | x | > 1
Respect to Another Function dx | x | x2 -1
d -1
Let u = f ( x ) and v = g( x ) be two functions of x. Then to find deriv- ( d) cosec -1x = , for | x | > 1
dx | x | x2 -1
du
ative of f ( x ) with respect to g( x ), that is, to find , we use the d 1
dv (e) tan-1 x = , for x ∈R
formula dx 1+ x 2
du du dx d -1
= (f) cot -1 x = , for x ∈R
dv dv dx dx 1+ x 2
. Differentiation of hyperbolic functions:
5
Illustration 20.22  Differentiate loge sin x with respect to cos x. d
(a) sinh x = cosh x
Solution: Let u = loge sin x and v = cos x . Then dx
du dv - sin x d
= cot x and = (b) cosh x = sinh x
dx dx 2 cos x dx
Hence, d
du du dx (c) tanh x = sec h2 x
= = -2 cos x cot x ⋅ cosecx dx
dv dv dx d
Some Standard Differentiations: (d) coth x = - cosech2 x
dx
d n
1. Differentiation of algebraic functions: x = nx n -1 d
dx (e) sech x = - sech x tanh x
In particular dx
d d
(a) [f ( x )]n = n [f ( x )] n -1f ′( x ) (f) cosech x = - cosech x coth x
dx dx
d 1 d
(b) ( x)= (g) sinh-1 x = 1/ (1+ x 2 )
dx 2 x dx
d  1 n d
(c)  n  = - n +1 (h) cosh-1 x = 1/ ( x 2 - 1)
dx x x dx
2. Differentiation of trigonometric functions: d
(i) tanh-1 x = 1/ ( x 2 - 1)
d dx
(a) sin x = cos x
dx d
( j) coth-1 x = 1/ (1- x 2 )
d dx
(b) cos x = - sin x
dx d
(k) sec h-1x = -1/ x (1- x 2 )
d dx
(c) tan x = sec2 x
dx d
d (l) cosech-1x = -1/ x (1+ x 2 )
(d) sec x = sec x tan x dx
dx Euler’s Theorem on Homogeneous Functions
d If f ( x , y ) is a homogeneous function in x, y of degree n, then
(e) cosec x = - cosec x cot x
dx ∂f ∂f
d +y = nfx
(f) cot x = -cosec2 x ∂x ∂y
dx Deduction of Euler’s theorem
. Differentiation of logarithmic and exponential functions:
3 If f ( x , y ) is a homogeneous function in x, y of degree n, then
d 1
(a) log x = , for x > 0 ∂2f ∂2f ∂f
dx x 1. x +y = (n - 1)
d x ∂x 2 ∂x ∂y ∂x
x
(b) e =e
dx ∂2f ∂2f ∂f
2. x + y 2 = (n - 1)
d x ∂y ∂x ∂y ∂y
(c) a = a x log a, for a > 0
dx
∂2f ∂2f ∂2f
d 1 3. x 2 + 2 xy + y 2 2 = n(n - 1) f ( x , y )
(d) loga x = , for x > 0, a > 0, a ≠ 1 ∂x 2 ∂x ∂y ∂y
dx x log a
4. Differentiation of inverse trigonometrical functions:
d 1 Your Turn 2
(a) sin-1 x = , for -1 < x < 1
dx 1- x 2 dy
of ln(xy) = x2 + y2.
1. Find
d -1 dx dy y (2 x 2 - 1)
(b) cos -1 x = , for -1 < x < 1 Ans. =
dx 1- x 2 dx x (1- 2 y 2 )

Mathematical Problem Book for JEE.indb 878 07-06-2018 13:30:25


Chapter 20 | Differentiation 879

dy y
2. Find of x + y = sin (xy). ⇒ - 1 = tan log x
dx dy y cos( xy ) - 1 x2
Ans. = y
dx 1- x cos( xy ) ⇒ = 1 + tan log x
. Differentiate (log x )
3
tan x
with respect to sin(m cos–1x). x2
⇒ y = x2 + x2 tan log x
 tan x  Now,
-(log x )tan x  sec2 x ⋅ loglog x + 1- x 2 dy 1
 x log x  = 2x + 2x tan log x + x sec2 (log x) ×
Ans. dx x
m cos( m cos -1 x ) = 2x + 2x tan log x + x sec2 (log x)
dy 2 2 2 2y
4. If xy.yx = 1, then find . = (x + x tan log x) + x sec2 (log x) = + 1 + tan2 (log x)
dx x x
dy ( y + x log y ) y
Ans. =- ⋅   y   2y
2
dx ( x + y log x ) x 2y y2 2y
⇒ + x 1+  2 - 1  = + 1+ 4 + 1 - 2
5. Differentiate sin2x with respect to (logx)2. x  x  x x x
 
x sin x cos x
Ans. xy 2 2 yx 2y
log x = - + + 2x
2 2 dy x4 x2 x
6. If x + y + xy = 2, then find .
dx dy (2 x + y )
Ans. =- 2y 2y y2 y2
dx (2 y + x ) += + 2 x -
= + 2x
x3 x x x3
dy Hence, the correct answer is option (A).
7. If y = sin x + sin x + sin x +  ∞ , then find .
dx
dy cos x 3. If y = ( x + 1+ x 2 )m , then (1+ x 2 ) y 2 + xy1 - m2 y = ____.
Ans. =
dx 2 y - 1 ( A) 0 (B) 1
 1 dy (C) – 1 (D) 2
8. If 5f(x) + 3f   = x + 2 and y = xf(x), then find at x = 1.
 x dx Solution: Substituting the value of y in the given equation, we
have
 dy  7 m -1  1⋅ 2 x 
Ans.   =
 dx  at x =1 8 y1 = m  x + 1+ x 2  ⋅  1+ 
   2 1+ x 2 
Additional Solved Examples
( )
m
m x + 1+ x 2
=
1. If f(x) = sinx, g(x) = x2, h(x) = logx and F(x) = (hogof )(x), then 1+ x 2
F’’(x) is
my
(A) 2cosec3x (B) 2cotx2 − 4x2cosec2x2 =
2
(C) 2x cotx (D) − 2cosec2x 1+ x 2
Solution: ⇒ y12 (1+ x 2 ) = m2 y 2
F ( x ) = (hogof )( x ) = h{ g[f ( x )]} Differentiating with respect to x, we get

= h[ g(sin x )] = h(sin2 x ) = loge (sin2 x ) 2 y1y 2 (1+ x 2 ) + y12 (2 x ) = 2m2 yy1

= 2 loge (sin x ) Cancelling 2y1, we get

⇒ F ’( x ) = 2 cot x ⇒ F ’’( x ) = -2 cosec x


Therefore, 2 (1+ x 2 ) y 2 + xy1 = m2 y

Hence, the correct answer is option (D). ⇒ (1+ x 2 ) y 2 + xy1 - m2 y = 0


 y - x2 
tan-1  2  Hence, the correct answer is option (A).
 x  dy
2. x = e , then is d2 y
dx 4. If x + y + y - x = c , then is
 dx 2
y2 2y  2y  y2 2y  2 -2
(A) x  2 + 4 - 2  + (B) x  2 + 4 - 2  (A) (B)
 x x  x  x x  c c2
 2
y2 2y  (C) (D) None of these
(C)  2 + 4 - 2  (D) None of these c2
 x x 
Solution: We are given that
Solution:
tan-1 (( y - x 2 )/ x 2 ) x + y + y - x = c (1)
x=e
Also
Taking log on both sides, we get
( x + y )2 - ( y - x )2 = x + y - ( y - x )
 y 
log x = tan−1  2 - 1
x ⇒ ( x + y + y - x )( x + y - y - x ) = 2 x

Mathematical Problem Book for JEE.indb 879 07-06-2018 13:31:09


880 Mathematics Problem Book for JEE

By Eq. (1), we have Solution:


2x dx
x + y - y - x = (2) = -2 sin t + 2 sin 2t
c dt
Adding Eqs. (1) and (2), we have dy
2x = 2 cos t - 2 cos 2t
2 x+y =c+ dt
c
Now,
Squaring both the sides, we get
dy 2 cos t - 2 cos 2t cos t - cos 2t
4 x2 = =
4( x + y ) = c 2 + + 4x dx -2 sin t + 2 sin 2t sin 2t - sin t
c2 3t t
After cancelling 4x from both the sides, we get 2 sin sin 3t
= 2 2 = tan
4 x2 3t t 2
4 y = c2 + 2 2 cos sin
c 2 2
dy 8 x d2 y 2 Therefore,
⇒ 4 = 2 ⇒ 2= 2
dx c dx c d2 y 3t 3 dt
Hence, the correct answer is option (C). = sec2 × ×
dx 2 2 2 dx
f ′′(0 ) 1
5. If f ( x ) = (1+ x )n, then the value of f (0 ) + f ′(0 ) + +  + f n (0) 3 3t 1
2! n! = sec2 ⋅
f ′′(0 ) 1 n 2 2 2 sin 2t - 2 sin t
f ( 0 ) + f ′( 0 ) + +  + f (0 ) is
2! n! d2 y 3
(A) n (B) 2n ⇒ 2 =-
dx t = p 2
(C) 2n-1 (D) None of these 2
Solution: Hence, the correct answer is option (D).
f (0 ) = 1, f ′( x ) = n(1+ x )n -1  dy 
2
8. If x = secq - cosq , y = sec n q - cosn q , then   is equal to
f ′′( x ) = n(n - 1)(1+ x )n - 2 , … ,  dx 

f (n) ( x ) = n(n - 1) …1 = n ! n2 ( y 2 + 4 ) n2 ( y 2 - 4 )
(A) (B)
⇒ f ′(0 ) = n, f ′′(0 ) = n(n - 1), … , f n (0 ) = n ! x2 + 4 x2
2
y2 - 4  ny 
Therefore, given expression is (C) n (D)   - 4
x2 - 4  x
n n(n - 1) n!
1+ + ++ Solution:
1 2! n!
dy
= n C 0 + n C1 + nC2 +  + nC n = 2 n = n sec n -1q ⋅ secq ⋅ tanq - n ⋅ cosn -1q ⋅ ( - sinq )
dq
Hence, the correct answer is option (B).
 sinq 
d2 y dy = n sec n q + cosn -1q ⋅ sinq 
6. If y = sin (sinx), and + tan x + f ( x ) = 0 , then f(x) is  cos q 
2 dx
dx n sinq  n n 
= sec q + cos q 
( A) sin2x sin(cosx) (B) sin2x cos(sinx) cosq 
(C) cos2x sin(cosx) (D) cos2x sin(sinx) = n tanq (sec n q + cosn q )
Solution: and
dy dx sinq
= cos(sin x ) ⋅ cos x = secq tanq + sinq = secq + sinq
dx dq cosq
d2 y sinq
⇒ = - cos(sin x ) ⋅ sin x + cos x [ - sin(sin x )cos x ] = (secq + cosq ) = tanq (secq + cosq )
dx 2 cosq
d2 y dy Therefore,
⇒ + tan x = - cos(sin x ) ⋅ sin x - cos2 x ⋅ sin(sin x )
dx 2 dx dy n tanq (sec n q + cosn q )
+ cos(sin x )⋅⋅ cos x ⋅ tan x =
dx tanq (secq + cosq )
= - cos2 x ⋅ sin(sin x ) n(secn q + cosn q )
2
=
⇒ f ( x ) = cos x ⋅ sin(sin x ) secq + cosq
Hence, the correct answer is option (D). Hence,
7. If x = 2 cos t − cos 2t and y = 2 sin t − sin 2t, then the value of 2
 dy  n2 (secn q + cosn q )2
d2 y p   =
2
at t = is dx (secq + cosq )2
dx 2
(A) 3/2 (B) −5/2 n2 (secn q - cosn q )2 + 4 
=
(C) 5/2 (D) −3/2 (secq - cosq )2 + 4

=
(
n2 y 2 + 4 )
2
x +4

Mathematical Problem Book for JEE.indb 880 07-06-2018 13:31:48


2
 dy  n2 (secn q + cosn q )2
  =
dx (secq + cosq )2
Chapter 20 | Differentiation 881
n2 (secn q - cosn q )2 + 4 
= 
(secq - cosq )2 + 4
Solution:
=
(
n2 y 2 + 4 )
x 2 x - 2 x x cot y - 1 = 0(1)
x2 + 4
Hence, the correct answer is option (A). Now at x = 1,
p
 2 x - 1 dy 1- 2 cot y - 1 = 0 ⇒ cot y = 0 ⇒ y =
9. If y = f  2  and f ′(x) = sin x2, then is 2
 x + 1 dx
On differentiating Eq. (1) with respect to x, we get
(A) cos x2.f ′(x) (B) − cos x2.f ′(x)
2  dy 
2(1+ x - x 2 )  2 x - 1 2 x 2 x (1+ log x ) - 2  x x ( - cosec2 y ) + cot y x x (1+ log x ) = 0
(C) sin  2  (D) None of these  dx 
( x 2 + 1)2  x + 1
2x -1  p
Solution: Let 2 = z .⇒
Then y = f (z) At  1,  , we have
x +1  2
2x -1
= z ⇒ y = f (z)  
x2 + 1   dy 
2(1+ log1) - 2 1( -1)   + 0 = 0
Therefore,   dx   1, p  
  2 

dy dz
= f ′( z ) ⋅
dx dx  dy   dy 
⇒ 2 + 2  =0⇒  = -1
dz  dx   1, p   dx   1, p 
= sin z 2 ⋅ [because f ′(z)=sin z2]  
2  
2
dx
2
 2 x - 1 d  2 x - 1 Hence, the correct answer is option (A).
= sin  2 
 x + 1 dx  x 2 + 1 d2 x
2 2
2. equals
 2 x - 1 2(1+ x - x ) dy 2
= sin  2  .
 x + 1 ( x 2 + 1)2 -1 -3
 d2 y   dy   d 2 y   dy  -2
Hence, the correct answer is option (C). (A) -  2    (B)
 2   
 dx  dx  dx  dx
d  3 d2 y 
10. If y2 = p(x), a polynomial of degree 3, then 2 y  is
dx  dx 2   d 2 y   dy  -3  d2 y 
-1

equal to (C) -  2    (D)  2


 dx   dx   dx 
(A) p’’’(x) + p’(x) (B) p’’’(x) + p’’(x)
 [AIEEE 2011]
(C) p(x) p’’’(x) (D) a constant
Solution: Solution:
 
y 2 = p( x ) (1) 
d  dx 
d  dx  d d  1 
1 11 dd  ddyy
⇒ 2 y y1 = p ’( x )  
 ==   == -- 2 dy  
(2) dy  dy
dy dy dy dy
dy   dy    dy2 dy
 dy  dx
dx

  dx 
⇒ 2( y y2 + y1 y1) = p ’’( x ) dx   dx
dx
1 -2
-2
⇒ y y 2 = [ p ’’( x ) - 2 y12 ] dy
 dy 11 dd  dy
dy
2 == --   

Multiplying both sides by y2, we have  dx
 dx dy
 dy  dx
dx  dx
dx
 
1  dx
dx
⇒ y 3 y 2 = [ p ’’( x ) y 2 - 2( y y1)2 ] -3
2 dy -3
 dd22yy  dy
2 == -- 22   

1 1 
= p ’’( x ) ⋅ p( x ) -  p ’( x ) [using Eqs. (1) and (2)] dx   dx
 dx dx

2 2 
Now, Hence, the correct answer is option (C).
d 3 1 1  1  dy
( y y 2 ) = [ p ’’’( x )p( x ) + p ’( x )p ’’( x )] - 2 ×  p ’( x ) ×  p ’’( x ) 3. If y = sec(tan-1 x ), then at x = 1 is equal to
dx 2 2  2  dx
1
d 3 (A) (B) 1
⇒ 2 ( y y 2 ) = p( x )p ’’’( x ) 2
dx 1
Hence, the correct answer is option (C). (C) 2 (D)
2
 [AIEEE 2013]
  Previous Years' Solved JEE Main/AIEEE Solution: We have,
Questions
y = sec(tan-1 x )
1. Let y be an implicit function of x defined by x2x − 2xx cot y − 1 = 0. Therefore,
Then y′(1) equals
dy 1 dy 1 1
(A) − 1 (B) 1 = sec(tan-1 x )tan(tan-1 x ) ⋅ 2
⇒ = 2 × 1× =
(C) log 2 (D) − log 2 dx 1+ x dx x =1 2 2
 [AIEEE 2009] Hence, the correct answer is option (D).

Mathematical Problem Book for JEE.indb 881 07-06-2018 13:32:21


882 Mathematics Problem Book for JEE

 d2 y   d2 x  Put x = 0, we get
4. If y = enx, then  2   2  is equal to 1
 dx   dy  g′(1) == 2.
f ′( 0 )
(A) nenx (B) ne−nx
Hence, the correct answer is (2).
(C) 1 (D) −ne−nx
 [JEE MAIN 2014 (ONLINE PAPER SET 1)]   sinq   p p
3. Let f (q ) = sin  tan-1    , where - 4 < q < 4 . Then the
  cos 2q 
Solution:
d
y = e nx value of [f (q )] is _____. [IIT-JEE 2011]
d (tanq )
dy d2 y Solution:
= e nx (n) ⇒ 2 = n2e nx (1)
dx dx   sinq    p p
sin  tan-1    , where q ∈ - 4 , 4 
Now   cos 2q 
d2 x  
dy 2
=
d  dx  d  1  1 d - nx
=  =
dy  dy  dy  ne nx  n dy
e ( ) 

sin  tan-1 
sinq
 2 cos q - 1 
2


1 dx  1  1 = sin[sin-1(tanq )] = tanq
= e - nx ( - n) = - e nx  nx  = - e -2nx (2)
n dy  ne  n d (tanq )
=1
Now from Eqs. (1) and (2), we get d (tanq )
 d2 y   d2 x  Hence, the correct answer is (1).
2 nx  1  -2 nx
 2   2  = n e ×  -  e = − n e−n x
 dx   dy  n
Hence, the correct answer is option (D).
Practice Exercise 1
dy
  Previous Years' Solved JEE Advanced/ 1. If cos( x + y ) = y sin x , then is equal to
dx
IIT-JEE Questions sin( x + y ) + y cos x sin( x + y ) + y cos x
(A) - (B)
d2 x sin x + sin( x + y ) sin x + sin( x + y )
1. equals y cos x - sin( x + y )
dy 2 ( C) (D) None of these
-1 -1 -3 sin x - sin( x + y )
 d2 y   d2 y   dy 
(A)  2  (B) -  2    dy
 dx   dx  dx 2. If sin2 x + 2 cos y + xy = 0 , then is equal to
dx
 d 2 y   dy  -2  d 2 y   dy  -3 y + 2 sin x y + sin 2 x
(C)  2    (D) -  2    (A) (B)
 dx   dx   dx   dx  2 sin y + x 2 sin y - x
 [IIT-JEE 2007] y + 2sin x
dy dx (C) (D) None of these
Solution: Since × = 1, we get sin y + x
dx dy
dy
-1   3. If x 3 + 8 xy + y 3 = 64 , then is equal to
dx 1  dy  dx
= = 
dy dy/dx  dx 
3x2 + 8 y 3x2 + 8 y
-1 -
(A) (B)
d  dx  d  dy  dx 8 x + 3y2 8 x + 3y2
⇒ =   ×
dy  dy  dy  dx  dy 3x + 8 y2
(C) 2 (D) None of these
-2 8 x + 3y
d2 x
 dy  d2 y dx 
⇒ 2 = -  × 2 × 
dy  dx  dx  dy  dy
  4. If sin(x + y) = log(x + y), then is equal to
dx
-3
 dy  d2 y (A)
2 (B) −2
×  =-
dx  dx  2 (C)
1 (D) −1
Hence, the correct answer is option (D).   5. If ln( x + y ) = 2 xy ,then y ’(0 ) =
3 x/2 -1 (A) 1 (B) −1
2. If the function f ( x ) = x + e and g( x ) = f ( x ), then the value (C) 2 (D) 0
of g′(1) is _____. [IIT-JEE 2009]
dy
  6. If x y = e x - y , then is equal to
Solution: dx
1
f (0 ) = 1, f ′( x ) = 3 x 2 + e x / 2 log x ⋅ [log(ex )]-2
(A) (B) log x ⋅ [log(ex )]2
2
⇒ f ′[ g( x )]g′( x ) = 1 log x ⋅ (log x )2
(C) (D) None of these

Mathematical Problem Book for JEE.indb 882 07-06-2018 13:33:05


Chapter 20 | Differentiation 883

p dy 2x
16. The differential coefficient of tan-1
tan x
  7. y = (tan x )(tan x ) , then at x = , the value of is equal to with respect to
4 dx 2 x 1 - x2
(A)
0 (B) 1 sin-1 is
(C)
2 (D) None of these 1+ x 2
(A) 1 (B) − 1
dy
  8. If y = (sin x )tan x, then is equal to (C) 0 (D) None of these
dx
(sin x )tan x ⋅ (1+ sec2 x ⋅ logsin x )
(A)  x  d2 y
17. If y = x log   , then x 3 2 is equal to
 a + bx  dx
tan x ⋅ (sin x )tan x -1 ⋅ cos x
(B)
2
(sin x )tan x ⋅ sec2 x ⋅ logsin x
(C) dy  dy 
(A)
x - y (B)  x - y
tan x ⋅ (sin x )tan x -1
(D) dx  dx 
2
dy  dy 
  9. If y = 21/logx 4 , then x is equal to (C)
y - x (D)  y - x
dx  dx 
(A) y (B) y
18. If y = x 3 logloge (1+ x ), then y ′′ (0 ) equals
y2
(C) (D) y 4
(A)
0 (B) − 1
10. The derivative of y = x ln x is (C)
6 log e 2 (D) 6

x ln x ln x
(A) (B) x ln x -1 ln x d2 x
ln x -2 19. is equal to
(C) ln x -12x ln x (D) x dy 2

dy 1 (d 2 y/dx 2 )
-1
11. If y = sin(2 sin x ), then is equal to (A) (B)
dx (dy/dx )2 (dy/dx )2
2 - 4 x2 2 + 4 x2 d2 y ( -d 2 y/dx 2 )
(A) (B) (C) 2 (D)
1- x 2 1- x 2 dx (dy/dx )2
2 - 4 x2 2 + 4 x2 20. If fn ( x ), gn ( x ),  hn ( x ), n = 1, 2, 3 are polynomials in x such that
(C) (D)
1+ x 2 1+ x 2
f1( x ) f2 ( x ) f3 ( x )
fn (a) = gn (a) = hn (a), n = 1, 2, 3 and F ( x ) = g1( x ) g2 ( x ) g3 ( x ) ,
3 cos x + 4 sin x  dy
12. If y = cos -1   , then h1( x ) h2 ( x ) h3 ( x )
 5 dx
then F ′(a) is equal to
(A) 0 (B) 1
1 (A)
0 (B) f1(a)g2 (a)h3 (a)
(C) -1 (D)
2 (C)
1 (D) None of these
d x - x -1
13. cos -1 is equal to
dx x + x -1 x3 sin x cos x
1 -1 21. Let f ( x ) = 6 -1 0 , where p is a constant. Then
(A) 2 (B)
1+ x 1+ x 2 p p2 p3
2 -2 d3
(C) 2 (D) [f ( x )] at x = 0 is
1+ x 1+ x 2 dx 3
d 1+ x 2 (A) p (B) p + p2
14. cos -1 is equal to
dx 2 (C) p + p3 (D) Independent of p
-1 1
(A) (B) x3 x2 3x2
2 1- x 4 2 1- x 4 d 3f ( x )
22. f ( x ) = 1 -6 4 , here p is a constant, then is
-x x dx 3
(C) (D) p p 2
p 3
1- x 4 1- x 4
(A) Proportional to x 2 (B) Proportional to x
x -1 1-
15. Differential coefficient of sin with respect to x is (C) Proportional to x 3 (D) A constant
1+ x
1 x y y1 y2
(A) (B) 23. If y = sin px and y n is the nth derivative of y, then y3 y4 y5
2 x 1- x
is equal to y6 y7 y8
(C)
1 (D) None of these

Mathematical Problem Book for JEE.indb 883 07-06-2018 13:34:05


884 Mathematics Problem Book for JEE

(A)
1 (B) 0 1 1
(C)
– 1 (D) None of these -
(A) (B)
4 2
d2 y
24. If y 2 = ax 2 + bx + c , then y 3 2 is 1
-
(C) (D)
1
dx 2 4
(A) A constant
(B) A function of x only d 1- sin 2 x
(C) A function of y only 33. is equal to
dx 1+ sin 2 x
(D) A function of x and y
p 
d2 y sec2 x
(A) (B) - sec2  - x 
x
25. If y = a ⋅ b 2 x -1
, then is 4 
dx 2
p  p 
sec2  + x 
(C) (D) sec2  - x 
y 2 ⋅log ab2
(A) (B) y ⋅log ab2 4  4 
y2
(C) (D) y ⋅(log ab2 )2
34. If f ( x ) = x tan-1 x , then f ’(1) is equal to
d
26. log(log x ) is equal to p 1 p
dx 1+
(A) (B) +
x log x 4 2 4
(A) (B) 1 p
log x x (C) - (D) 2
2 4
( x log x )-1
(C) (D) None of these
35. If f ( x ) = log x (log x ), then f ’( x )at x = e is
2
d  1 
27.  x+  is equal to 1
dx  x (A)
e (B)
e
1 1
(A) 1- 2 (B) 1+ (C) 1 (D) None of these
x x2
1  p
(C) 1- (D) None of these 36. If f ( x ) = 1+ cos2 ( x 2 ) , then f ’   is
2x  2 
x2 x3 dy
28. If y = 1+ x + + +  ∞ , then is equal to (A) p /6 (B) - (p / 6 )
2! 3! dx
(A)
y (B) y -1 (C)
1/ 6 (D) p / 6
y +1
(C) (D) None of these d
37. tan-1(sec x + tan x ) is equal to
d  -1 cos x  dx
29.  tan  is equal to
dx  1+ sin x  (A) 1 (B) 1/2
1 1 (C)
cos x (D) sec x
(A) - (B)
2 2
d x
(C) -1 (D) 1 38. (e logsin 2 x )
dx
d 2 2
30. [cos(1- x ) ] is equal to
dx e x (logsin 2 x + 2 cot 2 x )
(A)
-2 x (1- x 2 )sin(1- x 2 )2
(A) e x (logcos 2 x + 2 cot 2 x )
(B)
-4 x (1- x 2 )sin(1- x 2 )2
(B) e x (logcos 2 x + cot 2 x )
(C)
2 2 2 (D)
None of these
4 x (1- x )sin(1- x )
(C)
-2(1- x 2 )sin(1- x 2 )2
(D)  x + 1 -1  x - 1 dy
39. If y = sec -1   + sin   , then dx is equal to
dy  x - 1  x + 1
31. If y = sin-1( x 1- x + x 1- x 2 ), then is equal to
dx 1
(A)
0 (B)
-2 x 1 -1 1 x +1
(A) + (B) -
2
1- x 2 x - x2 1- x 2
2 x - x2 (C)
1 (D) None of these
1 1 d
(C) + (D) None of these 40. sin-1(3 x - 4 x 3 ) is equal to
2 dx
1- x 2 x - x2
3 -3
(A) (B)
 x 1- x 2 1- x 2
d  1+ cos 
32. -1 2 is equal to
 tan x 1 -1
dx  1- cos  (C) (D)
 2 1- x 2 1- x 2

Mathematical Problem Book for JEE.indb 884 07-06-2018 13:35:02


Chapter 20 | Differentiation 885

1+ tan x dy e2 x cos x dy
41. If y = , then is equal to 48. If y = , then is equal to
1- tan x dx x sin x dx
1 1- tan x p  e2 x [(2 x - 1)cot x - x cosec2 x ]
(A) ⋅ sec2  + x  (A)
2 1+ tan x 4  x2
2x
e [(2 x + 1)cot x - x cosec2 x ]
1- tan x p  (B)
(B) ⋅ sec2  + x  x2
1+ tan x 4 
2x
e [(2 x - 1)cot x + x cosec2 x ]
1 1- tan x p  (C)
(C) ⋅ sec  + x  x2
2 1+ tan x 4  (D) None of these
(D)
None of these d - ax 2
49. [e log(sin x )]
d  sec x + tan x  dx
42.   is equal to e - ax (cot x + 2ax logsin x )
2

dx  sec x - tan x  (A)


e - ax (cot x + ax logsin x )
2
2 cos x cos x (B)
(A) (B)
(1- sin x )2 (1- sin x )2 (C) e - ax (cot x - 2ax logsin x )
2

2 cos x (D) None of these


(C) (D) None of these
1- sin x dy
50. If y = log x ⋅ e(tan x + x ) , then
2
is equal to
d  1- cos x  dx
43. log  is equal to
dx  1+ cos x  2  1 
e(tan x + x )  + (sec2 x + x )log x 
(A)
(A)
sec x (B) cosec x x 
2  1 
x x e(tan x + x )  + (sec2 x - x )log x 
(B)
(C)
cosec (D) sec
2 2  x 
d  -1 1- cos x  (tan x + x 2 )  1 2 
44.  tan  is equal to
(C)
e  x + (sec x + 2 x )log x 
dx  1+ cos x   
2  1 
1 e(tan x + x )  + (sec2 x - 2 x )log x 
(D)
-
(A) (B) 0  x 
2
1 1+ e x dy
(C) (D) 1 51. If y = , then is equal to
2 1- e x dx
 sin x  p 
45. If f ( x ) = tan-1  , then f ’   is equal to ex ex
 1+ cos x   3 (A) (B)
1 1 (1- e x ) 1- e2 x (1- e x ) 1- e x
(A) (B)
2(1+ cos x ) 2 ex ex
1 (C) (D)
(C) (D) None of these (1- e x ) 1+ e2 x (1- e x ) 1+ e x
4
d  e ax  d x
46. 52. [e log(1+ x 2 )] is equal to
  is equal to dx
dx  sin(bx + c ) 
 2x 
e ax [a sin(bx + c ) + b cos(bx + c )] e x log(1+ x 2 ) +
(A)
(A)  1+ x 2 
sin2 (bx + c )
 2x 
e x log(1+ x 2 ) -
(B) 
e ax [a sin(bx + c ) - b cos(bx + c )]  1+ x 2 
(B)
sin(bx + c )  x 
e x log(1+ x 2 ) +
(C)
e ax [a sin(bx + c ) - b cos(bx + c )]  1+ x 2 
(C)
sin2 (bx + c )  x 
e x log(1+ x 2 ) -
(D)
(D)
None of these  1+ x 2 
e x log x dy e2 x + e -2 x
dy
47. If y = 2
, then is equal to 53. If y = , then is equal to
x dx e2 x - e -2 x dx
e x [1+ ( x + 2)log x ] e x [1- ( x - 2)log x ] -8 8
(A) (B) (A) -2 x 2
(B) 2 x
x3 x4 2x
(e - e ) (e - e -2 x )2
e x [1- ( x - 2)log x ] e x [1+ ( x - 2)log x ] -4 4
(C) (D) (C) (D)
x3 x 3 (e - e -2 x )2
2x
(e 2x
- e -2 x )2

Mathematical Problem Book for JEE.indb 885 07-06-2018 13:35:59


886 Mathematics Problem Book for JEE

2( x - sin x )3 / 2 dy 2 xy + sec2 x + y sec x tan x


54. If y = , then is equal to (C)
-
x dx x 2 + sec x
2( x - sin x )3 / 2  3 1- cos x 1  (D)
None of these
(A)  2 ⋅ 1- sin x - 2 x  x dy
x   64. If sin( xy ) + = x 2 - y , then is equal to
2( x - sin x ) 3/ 2 y dx
 3 1- cos x 1 
(B)  2 ⋅ x - sin x - 2 x  y [2 xy - y 2 cos( xy ) - 1] [2 xy - y 2 cos( xy ) - 1]
x  
(A) 2 (B)
2( x - sin x )1/ 2  3 1- cos x 1  xy cos( xy ) + y 2 - x xy 2 cos( xy ) + y 2 - x
(C)  2 ⋅ x - sin x - 2 x 
x   y [2 xy - y 2 cos( xy ) - 1]
(D)
None of these -
(C) (D) None of these
xy 2 cos( xy ) + y 2 - x
d  1+ cos x 
55. cos -1  is equal to
dx 
x
2   1 dy
1 65. If y =  1+  , then is equal to
(A) 1 (B)  x dx
2
1 x
(C) (D) None of these  1   1 1 
3  1+  log  1+  -
(A)
x  x 1+ x 
1+ cos x dy
56. If y = tan-1 , then is equal to x
1- cos x dx  1   1 
 1+  log  1+  
(B)
1 x  x 
(A)
0 (B) -
2 1 
x
x 
(C)
1/2 (D) 1 
 x +  log( x - 1) -
(C)
x  x + 1
dy
57. If y x 2 + 1 = log( x 2 + 1 - x ), then ( x 2 + 1) + xy + 1 is equal x
to dx  1   1 1 
 1+  log  1+  +
(D)
(A) 0 (B) 1 x  x 1+ x 
(C) 2 (D) None of these
58. The derivative of f ( x ) = | x 2 - x | at x = 2 is Practice Exercise 2
(A) − 3 (B) 0
(C) 3 (D) Not defined
Single/Multiple Correct Choice Type Questions
59. The derivative of f ( x ) = 3 | 2 + x | at the point x 0 = -3 is
(A) 3 (B) −3 d 2  cos 4 x + cos2 x + 1 2
1. If   = a cos x + b cos x + c , then
(C) 0 (D) Does not exist dx 2  cos2 x + cos x + 1 
60. Derivative of the function f ( x ) = log5 (log7 x ), x > 7 is (A) a = − 4 (B) b = −1
1 1 (C) b = 1 (D) c = −2
(A) (B) dy
x (In5)(In7)(log7 x ) x(ln5)(ln7) 2. If y = sec (tan−1 x) , then at x = 1 is equal to
dx
1 1  1 
(C) (D) None of these (A) (B) sin−1  sin 
x (In x ) 2  2
dy (C) 1 (D) None of these
61. If x = a(t - sin t ) and y = a(1- cos t ), then is equal to
dx 10 x 1 dy
. If y = 10 and
3 .
= 10x l , then value of l  is
t t y dx
(A)
tan   (B) - tan  
 2  2 (A) ln 10 (B) (ln 10)2
t t (C) eln(ln10 )
2
(D) (log10e)2
(C)
cot   (D) - cot  
 2  2
x3
 t dy   4. Given f(x) = − + x2 sin 1.5 a − x sin a. sin 2a − 5 sin–1
62. If x = a  cos t + logtan  , y = a sin t , then  is equal to 3
 2  dx (a2 − 8a + 17), then
(A)
tant (B) - tant (A) f ′(x) = − x2 + 2x sin6 − sin4 sin8
(C)
cot t (D) - cot t (B) f ′ (sin 8) > 0
(C) f ′ (x) is not defined at x = sin 8
dy
63. If y sec x + tan x + x 2 y = 0, then is equal to (D) f ′ (sin 8) < 0
dx
2 xy + sec2 x + y sec x tan x Comprehension Type Questions
(A)
x 2 + sec x Paragraph for Questions 5–7: The graph of y = f(x) is given with
2 xy + sec2 x + sec x tan x six labelled points (see Fig. 20.1). Out of these points, answer the
-
(B)
x 2 + sec x following questions.

Mathematical Problem Book for JEE.indb 886 07-06-2018 13:36:46


Chapter 20 | Differentiation 887

1
13. The abscissa of the point of contact of tangent for which is
C greatest, is a
1
(A) (B) 1
3
1
E
(C) − 1 (D) –
B D 3
x
A Matrix Match Type Questions
F 14. Match the following:
Figure 20.1 Column I Column II
  5. The point which has the greatest value of f ′(x) is (p)  Does not exist
(A) B (B) C x - cos(sin-1 x )
(A)  lim is equal to
(C) D (D) E x→
1 1- tan(sin-1 x )
2
  6. The point where f ′ and f ′′ are non-zero and of the same sign
are  1 1
(A) B and D (B) D and E (B) If f(x) = log x 2 (log x ) , then f ′   is (q) –
 2 2
(C) B and E (D) None of these equal to
  7. The points where at least two of f, f ′ and f ′′ are zero, For the function f(x) = ln tan
(C)  (r) 28
(A) C and D (B) A and D
p x
(C) A and F (D) None of these  + 
4 2
Paragraph for Questions 8–10: In certain problems, the differ-
entiation of {f(x).g(x)} appears. One student commits mistake and dy
  if = sec x + p, then p is equal to
df dg dx
differentiates as ⋅ , but he gets correct result if f(x) = x3 and
1 dx dx (s) 1
g(0) = . 1 1- cos 2 x
(D)  lim is equal to
3 x → 0 x 1+ cos 2 x
  8. The function g(x) is
3 4 (t) 0
(A) 3 (B) 3
| x - 3| | x - 3|
15. Match the following:
9 27
(C) 3 (D) Column I Column II
| x - 3| | x - 3 |3
(A) If y = cos–1 (cos x), then y′ at x = 5
9. Derivative of {f(x − 3)⋅g(x)} with respect to x at x = 100 is (p)  Does not exist
is equal to
(A) 0 (B) 1
(C) – 1 (D) 2 (B) For the function f(x) = ln |tan x| (q) 2
 p
f ( x ) ⋅ g( x )     f ′  -  is equal to
10. lim will be  4
x [1+ g( x )]
x→0
(A)
0 (B) − 1  1+ x  1
(C) The derivative of tan–1  at (r) 
(C)
1 (D) 2  1- x  2
1 x = − 1 is
Paragraph for Questions 11–13: Let f ( x ) = 2
. Let m be the
1+ x
log x (s)  1
slope, a be the x-intercept and b be the y-intercept of a tangent (D) The derivative of at x = − 1
to y = f(x), then is x
11. Abscissa of the point of contact of the tangent for which m is (t)  –1
greatest
1
(A) (B) 1 Integer Type Questions
3
1 4x 2+3x 2
− 1
(C) (D) − 16. If y = tan–1 + tan–1  , where 0 < x < and
3 1+ 5 x 2 3-2x 3
12. The greatest value of b is dy l
= , then find l.
9 3 dx 1+ 25 x 2
(A) (B)
8 8
17. The function y = f(x) defined by the parametric equations
1 5 x = et sin t, y = et cos t satisfies the relation y′′ (x + y)2 = l(xy′
(C) (D)
8 8 − y), then find l.

Mathematical Problem Book for JEE.indb 887 07-06-2018 13:37:07


888 Mathematics Problem Book for JEE

Answer Key
Practice Exercise 1
1. (A) 2. (B) 3. (A) 4. (D) 5. (A) 6. (A)
7. (C) 8. (A) 9. (C) 10. (C) 11. (A) 12. (B)
13. (D) 14. (C) 15. (D) 16. (A) 17. (B) 18. (A)
19. (D) 20. (A) 21. (D) 22. (D) 23. (B) 24. (A)
25. (D) 26. (C) 27. (A) 28. (A) 29. (A) 30. (C)
31. (C) 32. (A) 33. (B) 34. (B) 35. (B) 36. (B)
37. (B) 38. (A) 39. (A) 40. (A) 41. (A) 42. (A)
43. (B) 44. (C) 45. (B) 46. (C) 47. (D) 48. (A)
49. (C) 50. (C) 51. (A) 52. (A) 53. (A) 54. (B)
55. (B) 56. (B) 57. (A) 58. (C) 59. (B) 60. (A)
61. (C) 62. (A) 63. (C) 64. (A) 65. (A)

Practice Exercise 2
1. (A), (C) 2. (A), (B) 3. (B), (C) 4. (A), (D) 5. (A) 6. (C)
7. (B) 8. (C) 9. (A) 10. (A) 11. (D) 12. (A)
13. (A) 14. (A) → (q), (B) → (p), (C) → (t), (D) → (p) 15. (A) → (t), (B) → (q), (C) → (r), (D) → (s) 16. 5
17. 2

Solutions
Practice Exercise 1 At x = 0 , y = 1 [from ln( x + y ) = 2 xy ]
1. cos( x + y ) = ( y sin x )
Hence,
 dy  dy 1- 2
⇒ - sin( x + y )  1+  = y cos x + sin x y ’(0 ) = =1
 dx  dx -1
dy y cos x + sin( x + y ) x
⇒ =- 6. x y = e x - y ⇒ y log x = x - y ⇒ y =
dx sin( x + y ) + sin x 1+ log x
dy
2. sin2 x + 2 cos y + xy = 0 ⇒ = log x (1+ log x )-2 = log x [log ex ]-2
dx
dy dy
⇒ 2 sin x cos x - 2 sin y +y+x =0 7. log y = (tan x )tan x logtan x (1)
dx dx
Hence, Taking log again in Eq. (1), we get
dy y + sin 2 x
= log(log y ) = tan x logtan x + log(logtan x )
dx 2 sin y - x
 dy  dy Differentiating with respect to x, we get
3. x 3 + 8 xy + y 3 = 64 ⇒ 3 x 2 + 8  y + x  + 3 y 2 =0
 dx  dx 1 1 dy

Hence, log y y dx
dy 3x2 + 8 y
=- sec2 x 1 1
dx 8 x + 3y2 = sec2 x logtan x + tan x ⋅ + ⋅ ⋅ sec2 x
tan x logtan x tan x
4. It is an implicit function, so Therefore,
1  
cos( x + y ) - dy 1
dy ∂f / ∂x x+y = y log y sec2 x ⋅ logtan x + 1+ 
=- =- = -1 dx  tan x logtan x 
dx ∂f / ∂y 1
cos( x + y ) -  1 
x+y   = y (tan x )tan x logtan x ⋅ sec2 x (logtan x + 1) + 
 tan x logtan x 
5. ln( x + y ) = 2 xy
Differentiating both sides with respect to x, we get    = y (tan x )tan x sec2 x [logtan x (logtan x + 1) + cot x ]

 1   dy   dy  p p 
Now at x = , y = 1, logtan   = log1 = 0
 x + y   1+ dx  = 2  x dx + y  4  4

dy 1- 2 xy - 2 y 2 Therefore,
⇒ = dy
dx 2 x 2 + 2 xy - 1 = 1⋅1⋅ 2[0 + 1] = 2
dx

Mathematical Problem Book for JEE.indb 888 07-06-2018 13:37:37


Chapter 20 | Differentiation 889

  8. Given y = (sin x )tan x ; log y = tan x ⋅ logsin x 2x 2x .


16. Let y1 = tan-1 and y 2 = sin-1
1- x 2 1+ x 2
Differentiating with respect to x, we get
Differentiating y1 and y 2 with respect to x, we get
1 dy
⋅ = tan x ⋅ cot x + logsin x ⋅ sec2 x
y dx dy1 d  -1 2 x 
=  tan 
dy dx dx 1- x 2 
= (sin x )tan x [1+ logsin x ⋅ sec2 x ]
dx dy 2 d  -1 2 x 
=  sin 
  9. Given dx dx  1+ x 2 
1 Putting x = tanq , we have
y = 21/logx 4 ⇒ log y = (log 2)
log x 4
y1 = tan-1 tan 2q = 2q = 2 tan-1 x
log 2 loge 4 loge 2 2 log 2 log 2
⇒ log x 4 = ⇒ = ⇒ =
log y loge x loge y log x log y and y 2 = sin-1 sin 2q = 2 tan-1 x
⇒ log x = log y 2 ⇒ x = y 2 Again
1 dy 2 ln x dy1 d 2
10. y = x ln x ⇒ ln y = (ln x )2 ⇒ = = (2 tan-1 x ) = (1)
y dx x dx dx 1+ x 2
dy 2 d 2
dy 2 ln x 2( x ln x )ln x dy and = (2 tan-1 x ) = (2)
⇒ =y = ⇒ = 2 x ln x -1 ln x dx dx 1+ x 2
dx x x dx

11. Let x = sinq . Then Hence,


dy1
=1
2 sin-1 x = 2q ⇒ y = sin 2q dy 2
17. From the given relation
dy dy /dq 2 cos 2q 2(1- 2 sin2 q ) 2 - 4 x 2
⇒ = = = = y
dx dx /dq cosq 1- sin2 q 1- x 2 = log x - log(a + bx )
x
3 4  Differentiating, we get
12. y = cos -1  cos x - sin x 
5 5   dy 
 x - y
3 4 dx  1 1 a
Putting = r cosq , = r sinq, we get = - b=
5 5 2 x a + bx x (a + bx )
x
r =1
dy Hence,
⇒ y = cos -1[cosq cos x - sinq sin x ] = q + x ⇒ =1 dy ax
dx -y= x
(1)
13. Putting x = cotq , we have dx a + bx
 x - x -1   2  Differentiating again with respect to x, we get
-1 x - 1
y = cos -1   = cos  2 
 x + x -1   x + 1 d2 y dy dy (a + bx )a - ax .b d2 y a2
x + - = ⇒ x =
dy -2 dx 2 dx dx (a + bx )2 dx 2 (a + bx )2
= cos -1(cos 2q ) = 2q ⇒ =
dx 1+ x 2 2
d2 y a2 x 2  dy 
2 ⇒ x3 = =x - y  [by Eq. (1)]
14. Putting x = cos 2q , we have dx 2 (a + bx )2  dx 
d  -1 1+ x 2  d 18. y = x 3 logloge (1+ x )
cos  = [cos -1 cosq ]
dx  2  dx
  x3 1
⇒ y ′ = 3 x 2 logloge (1+ x ) + ⋅
d d 1  -x 1+ x loge (1+ x )
= [q ] =  cos -1 x 2  =
dx dx  2  1- x 4 3x2 1
⇒ y ′′ = 6 x logloge (1+ x ) + ⋅
1- x loge (1+ x ) (1+ x )
15. Let y = sin-1 . Then
1+ x
dy -1 x3 1 x3 3x2
= (1) - ⋅ - +
dx x (1+ x ) (1+ x )2 loge (1+ x ) (1+ x )2 [loge (1+ x )]2 (1+ x )loge (1+ x )
dz 1 ⇒ y ′′(0 ) = 0
and z= x ⇒ = (2)
dx 2 x  
Therefore, by Eqs. (1) and (2), we have d2 x
d  dx  d  1  -1 d 2 y
19. =   =   = ⋅
dy dy/dx -2 dy 2 dy  dy  dy  dy   dy  2 dx 2
= =  dx   
dz dz/dx 1+ x dx

Mathematical Problem Book for JEE.indb 889 07-06-2018 13:38:30


890 Mathematics Problem Book for JEE

20. We have dy
f1( x ) f2 ( x ) f3 ( x ) 24. y 2 = ax 2 + bx + c ⇒ 2 y = 2ax + b
dx
F ( x ) = g1( x ) g2 ( x ) g3 ( x ) 2 2
 dy  d2 y d2 y  dy 
h1( x ) h2 ( x ) h3 ( x ) ⇒ 2   + 2 y 2 = 2a ⇒ y 2 = a -  
Hence,  dx  dx dx  dx 
2
f1′ ( x ) f2′ ( x ) f3′ ( x ) f1( x ) f2 ( x ) f3 ( x ) d2 y  2ax + b  d 2 y 4 ay 2 - (2ax + b )2
⇒y 2
= a-  ⇒y 2=
F ’( x ) = g1( x ) g2 ( x ) g3 ( x ) + g1′ ( x ) g2′ ( x ) g3′ ( x ) dx  2y  dx 4 y2
h1( x ) h2 ( x ) h3 ( x ) h1( x ) h2 ( x ) h3 ( x )
d2 y
⇒ 4 y3 = 4 a(ax 2 + bx + c ) - ( 4 a2 x 2 + 4 abx + b2 )
f1( x ) f2 ( x ) f3 ( x ) dx 2
+ g1( x ) g2 ( x ) g3 ( x ) d2 y d2 y 4 ac - b2
⇒ 4 y3 = 4 ac - b2 ⇒ y 3 = = a constant
h1′ ( x ) h2′ ( x ) h3′ ( x ) dx 2 dx 2 4
⇒ F ’(a) = 0 (since fn (a) = gn (a) = hn (a), n = 1, 2, 3) 25.   y = a x b2 x -1
Therefore, two rows in each determinant become identical
on putting x = a. dy
= a x b2 x -1 log a + 2a x b2 x -1 log b = a x b2 x -1(log a + 2 log b )
dx
d3 d3 d3
x3 sin x cos x d2 y
dx 3
dx 3
dx 3 6 - cos x sin x = a x b2 x -1(log a + 2 log b )2 = a x b2 x -1(log ab2 )2
21. f ′′′( x ) = 6 -1 0 = 6 -1 0 dx 2
2 = y (log ab2 )2
p p p3 p p2 p3
d 1 1
26. log(log x ) = ⋅ = ( x log x )-1
Hence, dx x log x
6 -1 0 2
d  1  d  1  1
27.  x+  = x + + 1 = 1- 2
f ′′′(0 ) = 6 -1 0 = 0 , which is independent of p. dx  x dx  x  x
p p2 p3 x2 x3
28. y = 1+ x +
+ + ∞ ⇒ y = e x
2! 3!
x3 x2 3x2 Differentiating with respect to x, we get
22. f ( x ) = 1 -6 4
dy
= ex = y
p p2 p3 dx
⇒f ( x ) = x 3 ( -6 p3 - 4 p2 ) - x 2 ( p3 - 4 p ) + 3 x 2 ( p2 + 6 p ) d  -1  cos x  
29. tan 
3 3 2 3
⇒ f ( x ) = -6 p x - 4 p x - x p + 4 px + 3 p x + 18 px 2 3 2 2 2 2 dx   1+ sin x  
Hence,   x x 
d  -1  cos2 - sin2
d
f ( x ) = -18 p3 x 2 - 12 p2 x 2 - 2 xp3 + 8 px + 6 p2 x + 36 px 2 2 
=
       tan  
dx dx  x x x x
 cos + sin + 2 sin cos  
2 2

d2  2 2 2 2 
and f ( x ) = -36 p3 x - 24 p2 x - 2 p3 + 8 p + 6 p2 + 36 p
dx 2    x  
  1- tan    
d  -1  2   d  -1 p x 1
d 3f ( x )   = tan   =  tan tan  4 - 2   = - 2
and = -36 p3 - 24 p2 = a constant dx  1+ tan  x    dx  
dx 3    2   
sin px p cos px - p2 sin px d d
30. [cos(1- x 2 )2 ] = - sin(1- x 2 )2 (1- x 2 )2
23. D = - p3 cos px p 4 sin px p5 cos px dx dx
- p 6 sin px - p7 cos px p 8 sin px = 4 x (1- x 2 )sin(1- x 2 )2
sin px p cos px - p2 sin px
31. Putting x = sin A and x = sin B , we have
= p 9 - cos px p sin px p2 cos px
y = sin-1(sin A 1- sin2 B + sin B 1- sin2 A )
- sin px - p cos px p2 sin
n px
sin px p cos px - p2 sin px = sin-1[sin( A + B )] = A + B = sin-1 x + sin-1 x
= - p 9 cos px p sin px p2 cos px = 0 dy 1 1
⇒ = +
sin px p cos px 2
- p sin px dx 1- x 2
2 x - x2

Mathematical Problem Book for JEE.indb 890 07-06-2018 13:39:12


Chapter 20 | Differentiation 891

32. Let d x 1
38. (e logsin 2 x ) = e x logsin 2 x + 2e x cos 2 x
x x dx sin 2 x
1+ cos 2 cos2
y = tan -1 2 = tan -1 4 = e x logsin 2 x + e x 2 cot 2 x = e x (logsin 2 x + 2 cot 2 x )
x x
1- cos 2 sin2  x + 1
2 4 -1  x - 1
39. y = sec -1   + sin  
-1 x -1 p x p x  x - 1  x + 1
y = tan cot = tan tan  -  = -
4  2 4 2 4  x - 1 -1  x - 1 p
= cos -1   + sin  =
Therefore,  x + 1  x + 1 2
dy 1
=- dy  -1 -1 p
dx 4 ⇒ =0 Assin x + cos x = 
dx  2
1- sin 2 x cos x - sin x 40. Put x = sinq , we get
33. y = =
1+ sin 2 x cos x + sin x d d 3
1- tan x p  dy p  sin-1(3 x - 4 x 3 ) = sin-1(sin 3q ) =
= = tan  - x  ⇒ = - sec2  - x  dx dx 1- x 2
1+ tan x  4  dx  4 
34. f ( x ) = x tan-1 x  1+ tan x  p 
41. y=  or y = tan  + x 
 1- tan x  4 
Differentiating with respect to x, we get
dy 1 p 
1 = sec2  + x 
f ’( x ) = x + tan-1 x dx p  4 
1+ x 2 2 tan  + x 
4 
Now put x = 1, then
1 p 1 1 1- tan x  p 
f ’(1) = + tan-1(1) = +    =   sec2  + x 
2 4 2 2 1+ tan x   4 
log(log x ) d  sec x + tan x  d  1+ sin x  2 cos x
35. f ( x ) = log x (log x ) = 42.  =  =
log x dx  sec x - tan x  dx  1- sin x  (1- sin x )2
1 1 1 d  1- cos x  d   x
- log(log x ) -0
1 43. log = log  tan   = cosec x
⇒ f ’( x ) = x x 2
⇒ f ’( e ) = e = dx  1+ cos x  dx   2 
(log x ) 1 e
d  -1 1- cos x  d  -1 x 1
36. f ( x ) = 1+ cos2 ( x 2 ) 44.  tan =  tan tan  =
dx  1+ cos x  dx  2 2
1
f ’( x ) = ⋅ (2 cos x 2 ) ⋅ ( - sin x 2 ) ⋅ (2 x )
2 2  sin x   x x
2 1+ cos ( x ) 45. f ( x ) = tan-1  = tan-1  tan  =
2  1+ cos x   2 2
- x sin 2 x
f ’( x ) = 1
1+ cos2 ( x 2 ) ⇒ f ’( x ) =
2
p
At x = , Hence,
2 p  1
f ’  =
 3 2
p 2p p
 p  - 2 ⋅ sin 4 - ⋅1
f ’ = = 2 d  e ax  ae ax sin(bx + c ) - be ax cos(bx + c )
 46.  =
 2  p 3 dx  sin(bx + c )  [sin(bx + c )]2
1+ cos2
4 2
e ax [a sin(bx + c ) - b cos(bx + c )]
=
Hence, sin2 (bx + c )
 p p  ex 
dy 1+ ( x - 2)log x 
f ’  =- 6 47. = -2 x -3e x log x + x -2  e x log x +  = e x  
 2  dx  x  x3 
d d  1+ sin x  Aliter: Taking log we have,
37. tan-1(sec x + tan x ) = tan-1 
dx dx  cos x  log y = x + loglog x - 2 log x
  x  x  1 dy 1 2
d -1
 sin  2  + cos  2   d p x

y dx
= 1+ -
x log x x
= tan  = tan-1 tan  + 
dx  cos  x   x   dx  4 2
  - sin    dy e x log x  x log x + 1- 2 log x 
 2 2  ⇒ = , 
dx x2  x log x 
d p x 1
=  + = e x [( x - 2)log x + 1]
dx  4 2  2 =
x3

Mathematical Problem Book for JEE.indb 891 07-06-2018 13:39:58


892 Mathematics Problem Book for JEE

e2 x cos x x p x p x
48. y = ⇒ log y = 2 x + logcos x - log x - logsin x = tan-1 cot = tan-1 tan  -  = -
x sin x 2  2 2 2 2
1 dy  - sin x  1 cos x dy 1
= 2+ - - ⇒ =-
y dx  cos x  x sin x dx 2
dy 2 1 1 cot2 x  57. y x 2 + 1 = log[ x 2 + 1 - x ]
⇒ = e2 x  cot x - - 2 cot x - 
dx  x x x x  Differentiating both sides with respect to x, we get

e2 x dy 1 1  1 2 x 
= [(2 x - 1)cot x - x cosec2 x ] x2 + 1+ y ⋅ ⋅2x = ⋅ - 1
x2 dx 2 x2 + 1 x 2 + 1 - x  2 x 2 + 1 
d - ax 2 dy -1
log(sin x )} = e - ax ( -2ax ).log(sin x ) + e - ax cot x
2 2
49. {e ⇒ ( x 2 + 1) + xy = x 2 + 1⋅
dx dx x2 + 1
= e - ax [cot x - 2ax log(sin x )]
2
dy
⇒ ( x 2 + 1) + xy + 1 = 0
dx
y = log x ⋅ e(tan x + x
2
)
50.
58. f ( x ) = | x 2 - x | ⇒ f ′( x ) = 2 x - 1 ⇒ f ′(2) = 3
Therefore,
dy 1  - x , x < 0
= e(tan x + x ) ⋅ + log x ⋅ e(tan x + x ) (sec2 x + 2 x ) 59. f ( x ) = 3 | 2 + x |; f ′( x ) = -3, ∵| x |= 
2 2

dx x   x, x > 0 
2  1 
= e(tan x + x )  + (sec2 x + 2 x )log x   log x 
x  60. f ( x ) = log5 (log7 x ) ⇒ f ( x ) = log5  e 
 loge 7 
1+ e x 1+ e x ⇒f ( x ) = log5 loge x - log5 loge 7
51. y= or y 2 =
1- e x 1- e x loge loge x
⇒f ( x ) = - log5 loge 7
dy (1- e x )e x + (1+ e x )e x 2e x loge 5
2y = =
dx (1- e x )2 (1- e x )2 dy dy/dt a sin t sin t
Hence, 61. = = =
dx dx/dt a(1- cos t ) 1- cos t
dy ex  1- e x   1- e x  ex t t
=     = 2 sin   .cos  
dx (1- e x )2 1+ e x  1- e x  (1- e x ) 1- e2 x  2  2 t
= = cot  
2 t  2
d x 1 2 sin  
52. [e log(1+ x 2 )] = e x log(1+ x 2 ) + e x 2x  2
dx (1+ x 2 )
 t
 2x  62. Given that x = a  cos t + logtan  and y = a sint .
= e x log(1+ x 2 ) +  2
 1+ x 2  Differentiating with respect to t, we get
e2 x + e -2 x dy
53. y=            = a cos t (1)
e2 x - e -2 x dt
Hence, 
dx  t   1  t 
and    = a  - sin t + cot     sec2   
dy (e 2x
-e -2 x
)2(e 2x
-e -2 x
) - (e 2x
+e -2 x
)2(e 2x
+e -2 x
) dt   2  2  2 
= 2
dx (e2 x - e -2 x )2 
= a  - sin t +
1  cos t

  =a = a cos t cot t (2)
-8 sin t  sin t
= From Eqs. (1) and (2), we get
(e2 x - e -2 x )2
3 1 dy
54. log y = log 2 + log( x - sin x ) - log x = tan t
2 2 dx

dy  3 1- cos x 1  63. y sec x + tan x + x 2 y = 0


⇒ = y ⋅ - 
dx  2 x - sin x 2 x  dy dy
⇒ sec x + y sec x tan x + sec2 x + 2 xy + x 2 =0
d  1+ cos x  d  -1  x 1 dx dx
55.  cos -1  =  cos  cos   =
dx  2  dx   2 2 dy 2 xy + sec2 x + y sec x tan x
⇒ =-
dx x 2 + sec x
x
1+ cos x 2 cos2 x
56. y = tan -1
= tan -1 2 64. sin( xy ) + = x 2 - y
1- cos x x y
2 sin2
2 Differentiating both sides, we get

Mathematical Problem Book for JEE.indb 892 07-06-2018 13:40:51


Chapter 20 | Differentiation 893

 1  dy 1 = sin 8 [ 2 sin 6 − (sin 8 + sin 4)]


d dy
cos( xy ) ( xy ) + x  - 2  + = 2x - = sin 8 [2 sin 6 − 2 sin 6 cos 2] = 2 sin 6 sin 8 (1 − cos 2)
dx  y  dx y dx
sin 6 < 0, sin 8 > 0, 1 − cos 2 > 0
 x  dy 1
⇒  x cos( xy ) - 2 + 1 = 2 x - - y cos( xy ) Therefore,
 y  dx y
f ′ (sin 8) < 0
dy  2 xy 2 - y - y 3 cos( xy ) 
⇒ =    5. f ′(x) (slope) is positive at B and E but f ′(x) has greatest value at
dx  xy 2 cos( xy ) - x + y 2  B relative to E.
x   6. f ′(x) (slope) is positive at B and E and f ′′(x) has also positive
 1  1
65. y =  1+  ⇒ log y = x log  1+  sign (concavity up) at E.
 x  x
  7. f(x) has equal root at A, so f(x) and f ′(x) is zero at A point. f ′(x)
1 dy  1 1 (slope) is zero at D and concavity of graph is not changing at
⇒ = log  1+  -
y dx  x  1+ x D so f ′′(x) is also at D.
dy  1    1 
x
1    8. f(x) g(x) = x3 g(x)
⇒ =  1+  log  1+  - ⇒ 3x2 g′(x) = 3x2 g(x) + x3 g′(x)
dx  x    x  1+ x 
⇒ 3g′(x) = 3g(x) + xg′(x)
⇒ (3 − x) g′(x) = 3g(x)
Practice Exercise 2
g ′( x ) 3
⇒∫ dx = ∫ dx
d2 d g( x ) 3- x
1. 2
(cos2 x - cos x + 1) = ( -2 sin x cos x + sin x ) ⇒ ln g(x) = − 3 ln |3 − x| + ln c
dx dx
Therefore,
d
= ( - sin2 x + sin x ) = ( -2 cos 2 x + cos x ) c
dx g(x) =
| 3 - x |3
= -2(2 cos2 x - 1) + cos x = -4 cos2 x + 2 + cos x Now,
= -4 cos2 x + cos x + 2 c 1
g(0) = = ⇒c=9
⇒ a = −4, b = 1, c = 2 27 3
Hence,
2. y = sec (tan−1 x) 9
g(x) =
| 3 - x |3
Hence,
  9. f(x – 3). g(x) = (x – 3)3. g(x) = 9
dy sec(tan-1 x ) ⋅ tan(tan-1 x )
= Therefore, derivative of f(x – 3)⋅ g(x) is 0.
dx 1+ x 2
9
dy 1  1  x3
= = sin-1  sin  f ( x ) ⋅ g( x ) | 3 - x |3
dx x =1 2  2 10. lim = lim =0
x → 0 x [1+ g( x )] x→0  9 
x  1+ 
x
  3. y = 1010  | 3 - x |3 
Hence,
2x
dy x 11. f ′(x) = -
= 1010 ln10 ⋅10 x ln10 = y10 x (ln10 )2 (1+ x 2 )2
dx
1 dy -2 + 6 x 2
= 10 x (ln10 )2 f ′′(x) =
y dx (1+ x 2 )3
1
Now, f ′′(x) = 0 if x = ±
3
1
2
l = (ln 10)2 = eln(ln10 ) Hence, f ′(x) is greatest at x = – .
3
x3
  4. f(x) = − + x2 sin 1.5 a − x sin a. sin 2a − 5 sin–1 (a2 − 8a + 17) 12. Equation of tangent at x = a  is
3
1 -2a
f(x) is defined when y – = (x – α)
1+ a 2 (1+ a 2 )2
–1 ≤ a2 – 8a + 17 ≤ 1
⇒ −1 ≤ (a − 4)2 + 1 ≤ 1 ⇒ a = 4 1 2a 2 1+ 3a 2
⇒b= 2
+ 2 2
=
Hence, 1+ a (1+ a ) (1+ a 2 )2
3 Hence,
x 5p
f(x) = − + x2 sin 6 − x sin 4 sin 8 –
3 2 db (1+ a 2 )2 ⋅ 6a - 2(1+ 3a 2 )(1+ a 2 ) 2a 2a (1- 3a 2 )
Hence, = =
da (1+ a 2 )4 (1+ a 2 )3
f ′(x) = − x2 + 2x sin 6 − sin 4 sin 8
db
f ′(sin 8) = − sin28 + 2 sin 8 sin 6 − sin 4 sin 8 =0
da

Mathematical Problem Book for JEE.indb 893 07-06-2018 13:41:21


894 Mathematics Problem Book for JEE

Now, (1- x )2 1
1 2
a = 0, ± = ⋅ =
3
2
2(1+ x ) (1- x ) 2
1+ x 2
At x = –1, we have
1 9 d  1+ x  1
At a = ± ,b= . tan-1  =
3 8 dx  1- x  2

1+ 3a 2 1
13. a= d ln| x | x ⋅ x - ln| x | 1- ln| x | ln| x |
2a (D) = =
1 2a dx x x2 x2 x
⇒ =
a 1+ 3a 2 d ln| x |
⇒ = 1 (at x = –1)
1 dx x
Hence, its greatest value is .
3 2
+x
14. (A) -1 4x -1 2 + 3x -1 5 x - x -1 3
16. y = tan + tan = tan + tan
1+ 5 x 2 3 - 2x 1+ 5 x ⋅ x 2
x - cos(sin-1 x ) x - 1- x 2 2 1 1- ⋅ x
lim = lim = lim ( - 1 - x ) = - 3
1 1- tan(sin-1 x ) 1 x 1 2
x→ x→ 1- x→ 2
2 2 2 = tan–1 5x – tan–1 x + tan–1 + tan–1 x
1- x 2
3
x - cos(sin-1 x ) x - 1- x 2 2 1 –1 –1 2
lim = lim = lim ( - 1- x ) = - = tan 5x + tan
1 1- tan(sin-1 x ) 1 x 1 2 3
x→ x→ 1- x→
2 2 2 2
1 - x Therefore,
1  1 dy 5
(B) x = is not in the domain. Hence, f ′   does not exist. =
2  2 dx 1+ 25 x 2
p x Hence, l = 5
(C) y = f(x) = ln tan  + 
 4 2
17. x = et sin t and y = et cos t
Therefore,
p x ⇒ x2 + y2 = e2t ⇒ et = x 2 + y 2 (1)
sec2  + 
dy  4 2 1 1 1
= ⋅ = x  x
dx p x 2 2 p x p x and tan t = ⇒ t = tan–1   (2)
tan  +  sin  +  cos  +  y  y
 4 2  4 2  4 2
From Eqs. (1) and (2), we have
1
= = sec x
p   x
sin  + x  tan-1  y 
2  e = x2 + y2  (3)
Thus, p = 0
    Taking log on both sides, we get
1 1- cos 2 x | tan x |  x 1
(D) lim = lim = Does not exist tan–1   = ln (x2 + y2)
x →0 x 1+ cos 2 x x →0 x  y 2

15. (A) y = cos–1(cos x) Differentiating both sides with respect to x, we have


 
-1 sin x
y′ = ⋅(– sin x) =  1  
1- cos2 x | sin x |  x 2  y ⋅1 - x ⋅ y ′ = 1 ⋅ (2 x + 2 yy ′ )
 1+ 2   y2

 2 ( x2 + y2 )
Therefore, y ′ at x = 5 is –1.  y 
 
(B) y = f(x) = ln |tan x|
y-x
Therefore, ⇒ y′ = (4)
x+y
 tan x 
f ′(x) = (1/tanx) (sec2x) · 
 tan x  Again differentiating Eq. (4) with respect to ′x’, we have
( x + y ) ( y ′ - 1) - ( y - x ) (1+ y ′ )
 p y′′ =
⇒ f ′-  = 2 ( x + y )2
 4
⇒ y′′ (x + y)2 = y′ (2x) – 2y
d  1+ x  1 d  1+ x 
(C) tan–1  = ⋅
dx  1- x   1+ x 
2  
dx  1- x  ⇒ y′′ (x + y)2 = 2 (xy′ – y)
1+ 
 1- x  Hence proved and l = 2.

Mathematical Problem Book for JEE.indb 894 07-06-2018 13:41:52


Chapter 20 | Differentiation 895

Solved JEE 2017 Questions


JEE Main 2017   f ′( x ) =
9 2
x + 2a1x + a2
2
1. Let f be a polynomial function such that f (3 x ) = f ′( x ) ⋅ f ′′( x ),
for all x ∈. Then f ′′( x ) = 9 x + 2a1
f (2) - f ′(2) + f ′′(2) = 10
(A) (B) f ′′(2) - f (2) = 4 81 3
f (3 x ) = x + 9a1x 2 + 3a2 x + a3
(C) f ′′(2) - f ′(2) = 0 (D) f (2) - f ′(2) = 28 2
(ONLINE) Now,
Solution: Let f(x) = a0xn + a1xn−1 + a2xn−1 + … + an−1x + an.
f (3 x ) = f ′( x ) ⋅ f ′′( x )
f ′( x ) = a0nx n -1 + a1(n - 1) x n - 2 +  + an -1
81 3 9 
⇒ x + 9a1x 2 + 3a2 x + a3 =  x 2 + 2a1x + a2  (9 x + 2a1)
2 2 
f ′′( x ) = a0n(n - 1) x n - 2 + a1(n - 1)(n - 2) x n - 3 +  + an - 2
81 3 81
Now, ⇒ x + 9a1x 2 + 3a2 x + a3 = x 3 + [9a1 + 18a1] x 2
2 2
f(3x) = 3na0xn + 3n−1a1xn−1 + 3n−2a2xn−2 + … + 3an−1 + an +[ 4 a12 + 9a2 ] x + 2a1a2
f ′( x ) ⋅ f ′′( x ) = [a0nx n -1 + a1(n - 1) x n - 2 +  + an -1][a0n(n - 1) x n - 2 Comparing the coefficients, we get
n -3
+ a1(n - 1)(n - 2) x  + an - 2 ] 9a1 = 27a1
Comparing highest powers of x, we get ⇒ a1 = 0, 3a2 = 4 a12 + 9a2 = 9a2
3n a0 x n = a02 (n - 1) x n -1+ n - 2 = a02n2 (n - 1) x 2n - 3 ⇒ a2 = 0
Therefore, Therefore,
2n – 3 = n 3 3
f (x) = x
n 2
⇒ n = 3 and 3 a0 = a02n2 (n - 1) 9
f ′( x ) = x 2
3 2
⇒ a0 = 27 =
2 f ′′( x ) = 9 x
Therefore,
3 3 Hence, f ′′(2) - f ′( x ) = 18 - 18 = 0.
   f ( x ) = x + a1x 2 + a2 x + a3
2 Hence, the correct answer is option (C).

Mathematical Problem Book for JEE.indb 895 07-06-2018 13:42:15


Mathematical Problem Book for JEE.indb 896 07-06-2018 13:42:15
21 Applications of
Derivatives
21.1 Geometrical Interpretation x3 = 3 (−3)2 = 27
⇒x = 3
of Derivative Therefore, the point of intersect is (3, −3). Hence, the slope of the
Let us consider a curve in the form y = f(x) and two points P(x, y) tangent at this point is
and Q(x + ∆x, y + ∆y) that lie on the curve (Fig. 21.1). dy 32 3
= =-
dx (3, - 3) 2( -3) 2
Q (x + ∆x, y + ∆y)

y T
21.2  Tangent and Normal
A tangent to a point is a line which touches the curve at that point.
P(x, y) A normal to a point is the line which is perpendicular to the tan-
gent at that point.
If the equation of a curve is y = f(x) and a point A(x1, y1) lies on it,
then the equation of the tangent at point A is
 dy 
y – y1 =   ( x − x1)
 dx  A
Y
x and the equation of the normal at point A is
O 1
y - y1 = - ( x - x1)
(dy / dx ) A

Figure 21.1 Illustration 21.2   Find the equation of tangent and normal for
Then illustration 21.1.
dy Dy Solution: The equation of the tangent is
= lim
dx Dx ® 0 Dx 3
y - ( -3) = - ( x - 3)
 y + ∆y − y  2
= lim 
∆x → 0  x + ∆x − x 
 ⇒2y + 6 = −3x + 9
= lim (slope of the chord PQ ) ⇒3x + 2y − 3 = 0
Dx ® 0
The equation of the normal is
Now, this is equal to the slope of the tangent PT at point P(x, y) 2
which, in turn, is equal to tany. Here, y is the angle that the tan- y - ( -3) = + ( x - 3)
3
gent at point P makes with the positive direction of x-axis.
⇒3y + 9 = 2x − 6
Illustration 21.1   Find the slope of tangent at the point that has ⇒2x − 3y − 15 = 0
the ordinate −3 on the curve x3 = 3y2.

Solution: Differentiating the equation of the given curve w.r.t. x, Key Point:
we get When the curve is given in parametric form, that is, when x =
g(t) and y = h(t), the equation of tangent at the point t = t1 is
æ dy ö
3x2 = 3´ ç 2y ÷ h′(t1)
è dx ø y − h(t1) = [ x − g (t1)]
g′(t1)
dy x 2 and the equation of normal is
⇒ =
dx 2 y
g¢(t1)
Now, to obtain this value, we require abscissa as well. Substituting y - h (t1) = - [ x - g (t1)]
h¢(t1)
y = −3 in the equation of curve, we have

Chapter 21.indd 897 12-06-2018 19:02:49


898 Mathematics Problem Book for JEE

Illustration 21.3   Find the points on the curve y = x3 − x2 − x + 3 where m = dy / dx . Similarly, we can conclude with the following
where the tangent is parallel to the x−axis. results:

Solution: Given curve is 1.  PN is called the ‘length of the normal’, which is expressed as
y= − −x+3
x3 x2 2
(PM)secy = y1 1 + m (from ∆MNP)
dy
⇒ = 3x2 − 2x −1
dx 2.  TM is called the ‘subtangent’ which is expressed as
Since the tangent is parallel to the x−axis, the slope is y1 y
(PM)cot ψ = = 1 (from ∆TMP)
tan 0° = 0 tanψ m
That is, 3.  MN is called the ‘subnormal’ which is expressed as
dy (PM)tan y = | y1m | (from ∆MNP)
=0
dx
Hence, Illustration 21.4   Find the length of tangent, normal, subtan-
3x2 − 2x −1 = 0 or (3x + 1)(x − 1) = 0 x
gent and subnormal to the curve y = at the point having
Therefore, 1− x 2
1 abscissa 2 .
x= − or 1
3
For the first point, we have Solution: At x = 2 and y = − 2 , the point is P ( )
2 , − 2 . Now,
3 2
1  1  1  1 86 dy (1- x 2 ) - x ( -2 x ) 1+ x 2
x = − and y =  −  −  −  −  −  + 3 = = =
3  3   3   3  27 dx (1- x 2 )2 (1- x 2 )2
For the second point, we have Therefore,
x = 1 and y = 13 − 12 − 1 + 3 = 2 dy 1+ 2
Hence, the points on the given curve are = =3 =m (say)
dx P (1- 2)2
 1 86  The equation of tangent is
 − ,  and (1, 2)
3 27 
y + 2 = 3( x − 2 )
21.2.1 Length of  Tangent, Normal, Subtangent It intersects x−axis at point
and Subnormal æ4 2 ö
T çç , 0 ÷÷
Let the tangent and the normal at point P(x1, y1) meet x−axis at
è 3 ø
points T and N, respectively (Fig. 21.2). Here, PT is called the length
of the tangent, which is equal to The length of the tangent is
y æ4 2 ö
2
(PM)cosecy = 1 (from ∆TMP) PT = çç - 2 ÷÷ + (0 + 2 )2 =
2 5
sinψ
è 3 ø 3
y
The perpendicular drawn from point P on x−axis meets at
M( 2, 0). The subtangent is
4 2 2
MT = - 2=
3 3
The equation of normal is
P (x1, y1) 1
y + 2 = - (x - 2)
ψ 3
ψ which intersects x−axis at point N( -2 2, 0). The length of the
x
O T N normal is PN = 20 and of the subnormal is MN = 3 2.
M
Aliter:  The length of the tangent is
y 1 + m2 ( - 2 ) 1+ 9 2 5
Figure 21.2 = =
m 3 3
Hence, the length of tangent PT is
The length of the normal is
y1 sec ψ y 1 + m2
= 1
tanψ m y 1+ m2 = − ( 2 )× 10 = 2 5

Chapter 21.indd 898 10-06-2018 18:10:45


Chapter 21 | Applications of Derivatives 899

The length of the subtangent is Now,

y − 2 2 tanψ 2 − tanψ 1 m2 − m1
= = tanq  = tan(y2 − y1) = =
m 3 3 1 + tanψ 2 tanψ 1 1 + m1 m2
The length of the subnormal is If q  is the acute angle between the two curves, we have

y × m = − 2 ×3 = 3 2 m2 − m1
tan q =
1 + m1m2
Note (Tangent and Normal):  Let y = f(x) be the given curve. The
equation of the tangent at (x1, y1) would be where m1 = f ′(x) at P and m2 = g′(x) at point P.
 dy 
y − y1 =   ( x − x1) Remarks:
 dx ( x1 , y1 )
(i)  The curves intersect orthogonally if m1m2 = −1.
(ii)  The curves touch each other if m1 = m2.
æ dy ö
or y − f(x1) = ç ÷ ( x - x1)
è dx ø( x1 , y1 ) Illustration 21.5   Find the angle of intersection of the curves
Similarly, the equation of the normal at (x1, y1) would be y = x3 and 6y = 7 − x2.

1 Solution: The point of intersection is obtained by solving the


y - y1 = - ( x - x1)
æ dy ö equations simultaneously
ç ÷
è dx ø( x1 , y1 ) 7 x2
y = x3 and y = −
6 6
provided that
That is,
 dy 
  ≠0 6x3 = 7 − x2 or 6x3 + x2 − 7 = 0
dx ( x , y )
1 1
⇒(x − 1)(6x2 + 7x + 7) = 0
This gives x = 1 and the other factor gives complex roots. If x = 1,
21.3  Angles Between Two Curves then y = 1 (by using equation y = x 3). Now, from curve C1, we have
dy
Given two curves C1: y = f(x) and C2: y = g(x) intersecting at some = 3x2 = 3
dx x =1
point P(x1, y1) (Fig. 21.3).
From curve C2, we have
y C1
dy −2 x −1
T2 T1 = =
dx 6 3 x =1

Since the product of the slopes results to be −1, the curves intersect
at right angles.
P(x1, y1)
θ Illustration 21.6   Prove that the curves xy = 4 and x2 + y2 = 8
C2 touch each other.

Solution: First, we must find the common points. Solving the two
Ψ1 Ψ2 equations simultaneously, we get
x 16
O A B x2 + =8 or x4 − 8x2 + 16 = 0
x2
That is,
(x2− 4) 2 = 0 or x2 = 4 or x = ± 2
Figure 21.3 Correspondingly, y = ±2. Hence, the common points are (2, 2) and
(−2, −2).
Let PT1 be the tangent at point P to curve C1 and let PT1 make an dy dy −y
angle y1 with OX. Let PT2 be the tangent at P to curve C2 and let 1.  For curve C1: x + y = 0 and hence =
dx dx x
PT2 make an angle y2 with OX. The angle between two curves is
defined to be the angle between the two tangents at the point dy dy −x
2.  For curve C2: 2x + 2y = 0 and hence =
of intersection. Therefore, from ∆ABP, q, the angle between the dx dx y
curves is At points (2, 2) and (−2, −2), m1 = −1 and m2 = −1, respectively.
∠APB = T1PT2 = y2 − y1 Hence, the curves touch each other at both points.

Mathematical Problem Book for JEE.indb 899 07-06-2018 13:44:55


900 Mathematics Problem Book for JEE

Note (Angle of Intersection of Two Curves):  Let y = f(x) and y = We should find dy / dt when y = 6 in. Now,
g(x) be two given intersecting curves. The angle of intersection of
1  1 2 
these two curves is defined as the acute angle between the tan- V=

3  volume of a cone = 3 p r h (1)
(p r2)(y)
gents that can be drawn to the given curves at the point of inter-  
section. Now, from similar ∆ONM and ∆OCB, we get

MN ON r y y
21.4  dy/dx as Rate Measures =
BC OC
or =
5 10
or r =
2
In this section, we discuss about how dy / dx is useful in
determination of rates of change related to physical situations. Substituting r = y /2 in Eq. (1), we get

Illustration 21.7   A spherical balloon is pumped with air into it 1 y2 p


V= p ( y) = y3
at the rate of 10 in3/min. Find the rate of increase of radius of the 3 4 12
balloon when its radius is 15 in. Differentiating w.r.t. t, we get
Solution: Let y be the volume and x the radius of the balloon at
dV p dy
any time t. It is given that dy / dt = 10 in3/min. To find dx / dt when = (3 y 2 )
dt 12 dt
x = 15 in, since the balloon is spherical
4 3 Therefore,
y = p x (1)
3 dy 4 dV
=
dy 4 dx dx dt p y 2 dt
= p (3 x 2 ) = 4p x 2 (2)
dt 3 dt dt When y = 6 in, we get
Therefore, dy 4  4 
= 4 =  in/min
dx dy / dt 10 dt (p )6 2
 9p 
= =
dt 4p x 2 4p x 2
Hence, when the depth of water is 6 in, the water level is rising at
Therefore, when x = 15 in, the rate (4/9p ) in/min.
dx 10  1 
= =  in/min
dt 4p (15)2  90p  21.5 Errors and Approximations
Hence, the rate of increase of the radius of the balloon when its Let y = f ( x ). Then we know that
radius is 15 in is (1/ 90p ) in/min.
f (x +δ x) − f (x)
lim = f ′( x )
Illustration 21.8   The diameter of a cone is 10 in and its depth is
δ x →0 δx
10 in. Water is poured into it at the rate of 4 in3/min. At what rate is Therefore,
the water level rising at the instant when the depth is 6 in? f (x +δ x) − f (x)
= f ′( x ) + ε
Solution: See Fig. 21.4. Let OAB be the cone and LM be the level of δx
water at any time t. where ε → 0 , when δ x → 0 . So,
Let ON = y, volume OLM = V and radius MN = r. f ( x + δ x ) − f ( x ) = f ′( x )δ x + ε δ x
Given AB = 10 in, OC = 10 in and dV / dt = 4 in3/min. or f ( x + δ x ) − f ( x ) = f ′( x )δ x (approximately)
or    δ y = f ′( x ) δ x [As f ( x + δ x ) − f ( x ) = δ y ]
C
A B Thus, if d x is an error in x, then the corresponding error d y in y can
be calculated.

Note:
1.  d x and d y are known as differentials.
N r
L M If y = f ( x ) and d y is an increment in y, corresponding to an
2. 
increment d x in x, then we have
dy
y δy = (δ x )
dx
3.  The error d x in x is called an absolute error.
δx δ x 
O 4.  is called the relative error and  × 100  is called the
x  x 
Figure 21.4 percentage error.

Mathematical Problem Book for JEE.indb 900 07-06-2018 13:44:57


Chapter 21 | Applications of Derivatives 901

Illustration 21.9   Find the approximate value of 0.037 . Your Turn 1


Solution: Let f ( x ) = x . Then, Find the slopes of the curve y = (x + 2)(x − 3) at the points
 1. 
where it meets x−axis. Ans.  −5, 5
f ( x + δ x ) − f ( x ) = f ′( x )δ x
Find the points on the curve y = x3 − 2x2 + x − 2 when the gra-
 2. 
 1  dient is zero.
or f (x +δ x) − f (x) =  δ x
2 x   1 50 
Ans.  (1, −2) and  , − 
We may write  3 27 
0.037 = (0.04 − 0.003)  3. 
Find the equation of tangent and normal to the curve
x3 = y2 at the point (1, 1). Also find the length of tangent,
Taking x = 0.04 and d x = −0.003, we have normal, subtangent and subnormal.
 1  13 13 2 3
f (0.037) − f (0.04) =  ( −0.003) Ans.  3x − 2y − 1 = 0, 2x + 3y − 5 = 0, , , ,
 2 0.04  3 2 3 2
 4. Find the angle of intersection of the curves y = x and y = x3.
2

0.003  3   3  77  1
or f (0.037) = f (0.04) − =  0.04 −  =  0.2 − = Ans.  tan−1   , 0
2 × 0.2  400   400  400 7
Therefore, State true or false: The curves x2 − y2 = 16 and xy = 25 cut each
 5. 
77 other at right angles.
0.037 = = 0.1925 Ans.  True
400
 6. 
If the radius of a circle is increasing at a constant rate of 2 ft/
sec, then find the rate of increase of its area when the radius
Illustration 21.10   Find the approximate value of tan 46° if it is
is 20 ft.
given that 1° = 0.01745 rad.
Ans.  (80p  ) sq.ft/sec
Solution: Let f ( x ) = tan x. Then f ′( x ) = sec2 x . Now, Water is poured at the rate of 1 ft3/min into a cylindrical tub. If
 7. 
f ( x + δ x ) − f ( x ) = f ′( x )δ x the tub has a circular base of radius a ft, then the rate at which
water is rising in the tub is _______.
or f ( x + δ x ) − f ( x ) = (sec2 x )δ x  1 
Ans.   2  ft/min
Taking x = 45° = (p /4)° and δ x = 1° = 0.01745, we get  pa 

f (46°) − f (45°) = (sec2 45°) × 0.01745 21.6  Monotonicity of Function


or tan46° = tan45° + (sec2 45°) × 0.01745 In this section, the behaviour of function is discussed. Generally,
= (1+ 2 × 0.01745) = 1.03490 there are four types of behaviours shown in function in the inter-
vals of its domains.
Illustration 21.11   The time T of oscillation of a simple pendulum
l 21.6.1  Increasing Behaviour of Function
of length l is given by T = 2p . Find the percentage error in T
g If in an interval I, for any two points (x1, y1) and (x2, y2) (Fig. 21.5),
corresponding to an error of 2% in the value of l. we have
x2 > x1 ⇔ y2 > y1
Solution:
The function is said to be monotonically increasing or simply
l
T = 2p increasing in I. If the function is differentiable in the required
g
interval (which is, normally, true for most of the functions), it can
1 1 be inferred that (dy / dx ) > 0 for all points in the interval.
⇒ logT = log2 + logp + log l − log g
2 2
y
1  dT  1
⇒  =
T  dl  2l
1  dT  y2
 1
⇒   δ l =   δ l
T  dl  2l
y1
 1  1   dT  
⇒  δ T =   δ l since δ T =  dl  δ l 
T   2l     
 δT  1 δl  1
⇒  T × 100  = 2  l × 100  = 2 × 2 = 1 x1 x2 x

Therefore, percentage error in T = 1%. Figure 21.5

Mathematical Problem Book for JEE.indb 901 07-06-2018 13:44:59


902 Mathematics Problem Book for JEE

For example, y
y= x3

dy
⇒ = 3x2
dx

Now, (dy / dx ) > 0 for all real values of x except x = 0. Here,


(dy / dx ) ≥ 0 for the entire domain, but still the function is
increasing. For any two points such that x2 > x1, we have y2 > y1
x
certainly. a x1 x2 b

Note (Increasing Function): The function f(x) is said to be increas- Figure 21.7
ing function in D1 if for every x1, x2 ∈ D1,
Note (Non-Decreasing Function): The function f(x) is said to be
x1 > x2 ⇒ f(x1) > f(x2) non-decreasing in D1 if for every x1, x2 ∈ D1,
f(x) is increasing function in [a, b] if f '( x ) > 0 ∀ x ∈ (a , b ). x1 > x2 ⇒ f(x1) ≥ f(x2)
The function f(x) is non-decreasing in [a, b] if f ′ ( x ) ≥ 0 ∀ x ∈ (a , b ).
21.6.2  Decreasing Behaviour of Function
21.6.4 Non-Increasing Behaviour
If in an interval I, for any two points (x1, y1) and (x2, y2) (Fig. 21.6),
See Fig. 21.8. In this case,
we have
x2 > x1 ⇔ y2 ≤ y1
x2 > x1 ⇔ y2 < y1
for all points in that interval. This implies that
The function is said to be monotonically decreasing function or
dy
simply decreasing function in I. Also, for a differentiable function, ≤0
here (dy / dx ) < 0 for all points in the interval. dx
dy
where = 0 for a continuous set of points in the interval.
y dx
y

y2

y1

x x
x1 x2 a x1 x2 b

Figure 21.6 Figure 21.8


Note: These conditions normally are sufficient to find the intervals
Note (Decreasing Function): The function f(x) is said to be a de- of increasing function. However, when the derivative may include
creasing function in D1 if for every x1, x2 ∈ D1, zero also at specific points in the interval, the function may be still
increasing.
x1 > x2 ⇒ f(x1) < f(x2)
Non-Increasing Function: The function f(x) is said to be a non-­
f(x) is decreasing in [a, b] if f '( x ) < 0 ∀ x ∈ (a , b). increasing function in D1 if for every x1, x2 ∈ D1,
x1 > x2 ⇒ f(x1) ≤ f(x2)
21.6.3 Non-Decreasing Behaviour The function f(x) is non-increasing in [a, b] if f '( x ) ≤ 0 ∀ x ∈ (a , b ).
See Fig. 21.7. In this case,
x2 > x1 ⇔ y2 ≥ y1 Key Points:
1. If f ′( x ) ≥ 0 ∀ x ∈ (a , b ) and the points which make
for all points in that interval. This implies that f ′ ( x ) = 0 [in between (a, b)], do not form an interval, then
dy f(x) would be increasing in [a, b].
≥0
dx 2. If f ′( x ) ≤ 0 ∀ x ∈ (a , b ) and the points which make
dy f ′( x ) = 0 [in between (a, b)], do not form an interval, then
where = 0 for a continuous set of points in the interval. f(x) would be decreasing in [a, b].
dx

Mathematical Problem Book for JEE.indb 902 07-06-2018 13:45:00


Chapter 21 | Applications of Derivatives 903

point b is called the point at which the function is a minimum


3. If f(0) = 0 and f ′( x ) ≥ 0 ∀x ∈ R , then f(x) ≤ 0 ∀ x ∈ (−∞, 0) and f(b) is the corresponding minimum value of the function.
and f(x) ≥ 0 ∀ x ∈ (0, ∞). (iii) The maximum and minimum values of f(x) as defined above
4. If f(0) = 0 and f ′( x ) ≤ 0 ∀ x ∈ R , then f(x) ≥ 0 ∀ x ∈ (−∞, 0) are not necessarily the greatest and least values of f(x). They
and f(x) ≤ 0 ∀ x ∈ (0, ∞). are maximum and minimum in the immediate neighbour-
5. A function is said to be monotonic if the function is either hood of x = a and x = b. Hence, these are also referred as ‘local
increasing or decreasing. maximum’ or ‘local minimum’.
6. The points for which f' ( x ) = 0 (or the function does not ex- (iv) The points of maximum or minimum of a function are also
ist) are called ‘critical points’. Here, it should also be noted called the ‘points of extremum’.
that the critical points are the interior points of an interval. (v) A necessary condition for the existence of an extremum (max-
7. The stationary points are the points where f' ( x ) = 0 in the imum or minimum) for a function f(x) is either f ′(x) = 0 or f ′(x)
domain. does not exist.
Y

Illustration 21.12   Find the values of x for which the function A C


f(x) = 2x3− 21x2 + 72 x + 30 is (a)  increasing and (b)  decreasing.
B
Solution: D
f ′(x) = 6x2 − 42x + 72 = 6(x2 − 7x + 12) = 6(x − 3)(x − 4)
O
The function f(x) is increasing if f ′(x) > 0. That is, if 6(x − 3)
(a)  a b c d X
(x − 4) > 0 or if either x > 4 or x < 3 or if x ∈ (−∞, 3) ∪ (4, ∞)
(a)
The function f(x) is decreasing if f ′(x) < 0. That is, if 6(x − 3)
(b) 
(x − 4) < 0 or if x ∈ (3, 4). Y
A
Illustration 21.13   Separate the intervals in which the function
f(x) = x − ex is increasing or decreasing.

Solution:
f ′(x) = 1 − ex O
Now, f ′(x) > 0 ⇒ 1 − ex > 0 ⇒ ex < 1 ⇒ x < 0 a X
and f ′(x) < 0 for all x > 0. Therefore, f(x) is increasing in the interval
(b)
(−∞, 0) and decreasing in the interval (0, ∞). Y

Illustration 21.14  If x > 0, show that log(1 + x) < x.


Solution: The method is to construct the function f(x) = x − log(1
+ x) and show that (a)  it is increasing and (b)  f(0) = 0 or positive.
Let f(x) = x − log (1 + x). Then B
1 x O
f ′(x) = 1− = a X
1+ x 1+ x
is positive for all x > 0. Therefore, f(x) is increasing for all x > 0 and (c)
f(0) = 0 − log (1) = 0 − 0 = 0 Figure 21.9
That is, f(0) = 0 and f(x) is increasing for all x > 0 implies that f(x) is In Fig. 21.9(a), at the maximum and minimum points on the
positive for all x > 0. That is, graph, the tangent is parallel to the x−axis and hence f ′(x) = 0. In
x − log(1 + x) > 0 Fig. 21.9(b), the function is increasing as x approaches a from the
or log(1 + x) < x left and the function is decreasing as x increases beyond a. The
graph is not smooth and hence it has no tangent at x = a. In Fig.
21.7 Maxima and Minima of Functions 21.9(c), x = a is a point of local maxima ( f ( x ) is discontinuous at
of a Single Variable x = a).
Note: It is very important to note that for maxima and minima,
Let the function f(x) be defined on an interval A. Let a and b ∈ A. we use the basic definition at all the critical points (i.e. the points
Then where f ( x ) is discontinuous).
(i)  f(x) is said to have a maximum value at x = a if f(a)  > f(a + h) and 1.  Second Derivative Test (for Continuous Functions)
f(a)  > f (a − h) for all sufficiently small positive values of h. The If f(x) has a maximum value at x = a and f ′(a)  exists, then
(i) 
point a is called the point at which the function is a maximum f ′(a)  must be zero. Similarly, if f(x) has a minimum value
and f(a)  is the corresponding maximum value of the function. at x = b and f ′(b)  exists, then f ′(b)  must be zero.
(ii) f(x) is said to have a minimum value at x = b if f(b) < f(b + h) and If c be a point in the domain of f(x) such that f ′(c)  = 0 and
(ii) 
f(b)  < f(b − h) for all sufficiently small positive values of h. The f ′′(c)  ≠ 0, then

Mathematical Problem Book for JEE.indb 903 07-06-2018 13:45:01


904 Mathematics Problem Book for JEE

(a)  x = c is a point of local maxima if f ′′(c)  < 0 21.7.1.1  Test for Local Maximum/Minimum
(b)  x = c is a point of local minima if f ′′(c)  > 0
 1.   Test for local maximum/minimum at x = a if f(x) is differentiable
2.  First Derivative Test (For Continuous Functions) at x = a: If f(x) is differentiable at x = a and if it is a critical point
If f ′(A)  = 0 (or it does not exist) and f ′(x) changes its sign
(i)  of the function; if f ′(a) = 0 and f ′( x ) changes its sign while
from plus to minus as x passes through the point a from left passing through the point x = a, then
to right, then f(x) is maximum at x = a.  (i) f(x) would have a local maximum at x = a if f ′(a − 0) > 0 and
If f ′(B)  = 0 or does not exist and f ′(x) changes its sign from
(ii)  f ′(a + 0) < 0 . It means that f ′( x ) should change its sign from
minus to plus as x passes through the point b from left to positive to negative.
right, then f(x) is minimum at x = b.
(ii) f(x) would have local minimum at x = a if f ′(a − 0) < 0 and
(iii) 
If the derivative does not change its sign in moving from left f ′(a + 0) > 0 . It means that f ′( x ) should change its sign from
to right through the point a, then f(x) has neither maximum negative to positive.
nor minimum at x = a.
(iii) If f(x) does not change its sign while passing through x = a,
3.  nth Derivative Test then f(x) would have neither a maximum nor minimum at
(i)  It can be applied to x = c only if f ′(c)  = 0 and f ′′(c)  = 0. x = a.
(ii) By differentiation, find nth derivative of f(x) denoted by Second−order derivative test for maxima and minima: Let f(x)
f n(x), n ∈ N. be a differentiable function on an interval I. Let a ∈ I and f ′′(x) is
(iii)  Step−by−step, find the earliest non−zero f n(c), n = 3, 4, 5, continuous at x = a. Then
6, 7, … (i)  x = a is a point of local maximum if f ′(a)  = 0 and f ′′(a)  < 0.
(iv)  In this process, (ii)  x = a is a point of local minimum if f ′(a)  = 0 and f ′′(a)  > 0.
(a) if n is odd, then x = c is neither local maximum nor local (iii)  If f ′(a)  = f ′′(a)  = 0 and f ′ ′′(a)  ≠ 0 exists, then x = a is neither a
minimum point. point of local maximum nor a point of local minimum.
(b) if n is even, and if  2.  Test for local maximum/minimum at x = a if f(x) is not
Positive, ⇒ x = c is local minimum point differentiable at x = a:
f n (c ) = 
Negative, ⇒ x = c is local maximum point Case I: When f(x) is continuous at x = a and also f ′(a − h) and

f ′(a + h) exist which are non−zero, then f(x) has a local
4.  Absolute Maximum/Minimum Points maximum or minimum at x = a if f ′(a − h) and f ′(a + h)
(i) 
To find the absolute maximum/minimum values of f(x) in are of opposite signs.
open interval (a, b), we proceed as follows: (a)  If f ′(a − h) > 0 and f ′(a + h) < 0, then x = a will be a
(a) Find all extremum points of f(x) by using critical points. point of local maximum.
Let these extremum points be c1, c2, c3 … (b)  If f ′(a − h) < 0 and f ′(a + h) > 0, then x = a will be a
(b)  Compare the lengths of ordinates f(c1), f(c2), f(c3) , … point of local minimum.
(c) The greatest value of these ordinates is called absolute Case II: W  hen f(x) is continuous and f ′(a − h) and f ′ (a + h) exist,
maximum value of f(x). but one of them is zero, we should infer the information
(d) The least value of these ordinates is called absolute min- about the existence of local maximum/minimum from
imum value of f(x). the basic definition of local maximum/minimum.
Absolute maximum value/absolute minimum value can oc- Case III: If f(x) is not continuous at x = a and f ′ (a − h) and/or
cur at more than one extremum point. Absolute maximum/ f ′(a + h) are not finite, then compare the values of f(x) at
minimum value is also called global maximum/minimum the neighbouring points of x = a.
value of f(x).
(ii) 
To find the absolute maximum/minimum value in closed 21.7.1.2  Concept of Global Maximum/Minimum
interval [a, b], include the values of ordinates at the end points,
namely, f(a) and f(b)  in the above procedure of comparison of Let y = f(x) be a given function with domain D. Let [a, b] ⊆ D. The
the lengths of the ordinates at the extremum points. global maximum/minimum of f(x) in [a, b] is basically the greatest/
least value of f(x) in [a, b]. The global maximum and minimum in
21.7.1 Concept of Local Maximum and Local [a, b] would always occur at critical points of f(x) within [a, b] or at
Minimum the end points of the interval.
Let y = f(x) be a function defined at x = a and also in the vicinity of Illustration 21.15   Find the local maximum and minimum val-
the point x = a. Then, f(x) is said to have a local maximum at x = a
ues of f(x) = 2x3 − 15x2 + 36x +11.
if the value of the function at x = a is greater than the value of the
function at the neighbouring points of x = a. Similarly, f(x) is said Solution: Let y = 2x3 − 15x2 + 36x + 11. Therefore,
to have a local minimum at x = a if the value of the function at
x = a is less than the value of the function at the neighbouring dy
= 6 x 2 − 30 x + 36 = 6(x2 − 5x + 6)
points of x = a. dx

Mathematical Problem Book for JEE.indb 904 07-06-2018 13:45:02


Chapter 21 | Applications of Derivatives 905

d2 y Therefore,
= 12 x - 30 d2 y
dx 2 = - e (e )1/ e < 0
dx 2
For extremum,
dy / dx = 0 Hence, y is maximum when x = 1/ e and the maximum value of
1/ e
⇒ x2 − 5x + 6 = 0 y= e .
Þ (x −2) (x − 3) = 0
Illustration 21.17  If y = a log | x | + bx2 + x has its extreme values
That is, x = 2 or x = 3. Now,
at x = −1 and x = 2, find a and b.
d2 y
= 12(2) − 30 = − 6 < 0 Solution:
dx 2 x =2 y = a log | x | + bx2 + x
Therefore, y or f(x) is a maximum when x = 2 and the maximum Therefore,
value of f(x) = f(2). Therefore, dy a
= + 2bx + 1
f(2) = 2(23) − 15(22) + 36(2) + 11 = 39 dx x

d2 y For extreme values, dy / dx = 0.


= 12 × 3 − 30 = 6 > 0
dx 2 dy a
x =3 =0⇒ + 2b ( −1) + 1 = 0
Therefore, y or f(x) is a minimum when x = 3 and the minimum dx x = −1 −1
value of or a + 2b −1 = 0
f(x) = f(3) = 2(3)3 − 15(3)2 + 36(3) + 11 = 38
dy a
x and      dx = 0 Þ + 2b (2) + 1 = 0
æ 1ö x=2 2
Illustration 21.16   Show that the maximum value of ç ÷ is
èxø or a + 8b + 2 = 0
1
ee . Solving these two equations simultaneously, a = 2 and b = -(1/ 2).
x
1
Solution: Let y =   . Then Illustration 21.18   Find the maximum and minimum values of
x f(x) = x + sin2x in [0, 2p ].
 1
log y = x loge   = − x loge x(1) Solution:
x
f ′(x) = 1 + 2cos 2x; f ′′(x) = − 4 sin2x
Therefore, 1 2p 4p
1 dy é 1 ù f ′(x) = 0 ⇒ cos2 x = - ⇒ 2 x = ,
= - ê x × + loge x ú = - (1 + loge x ) (2) 2 3 3
or
y dx ë x û p 2p
x x= ,
dy æ 1ö 3 3
⇒ = - ç ÷ (1+ loge x )
dx èxø æp ö 2p 4 3
Differentiating, we get f ¢¢ ç ÷ = - 4 sin =- = -2 3 < 0
è 3ø 3 2
2
1 d2 y 1 æ dy ö 1
- 2 ç ÷ = - (3) p
y dx 2
y è dx ø x ⇒x=
3
From Eq. (2), we get which is local maximum point.
x
dy æ 1ö æ 2p ö 4p 3
= - y (1 + loge x ) = - ç ÷ (1 + loge x ) f ¢¢ ç ÷ = - 4 sin =4 =2 3>0
dx èxø è 3 ø 3 2

For the maximum or minimum values of y, dy / dx = 0. Therefore, 2p


⇒x=
x
3
æ 1ö
ç ÷ (1 + loge x ) = 0 which is local minimum point.
èxø
Now, for absolute maximum/minimum, we compare the values of
However, (1/ x ) x ¹ 0 for any value of x. Therefore,
æπ ö π 3 æ 2π ö 2π 3
1 f (0) = 0, f ç ÷ = + ,fç ÷= - , f (2π ) = 2π
1 + loge x = 0; logex = −1; x = e−1 = (4) è3ø 3 2 è 3 ø 3 2
e
Therefore, the absolute maximum is
When x = 1/e, Eq. (3) gives
f(x) = f(2p) = 2p
1 d2 y and the absolute minimum is
-0 = -e
y dx 2 f(x) = f(0) = 0

Chapter 21.indd 905 11-06-2018 13:02:26


906 Mathematics Problem Book for JEE

Y
Illustration 21.19  Find the points of local extremum of the E
function f(x) = (2x − 1)2/5(x + 2).
Solution: C
2
f ′(x) = (2x − 1)2/5(1) + (x + 2) (2x − 1)−3/5 (2)
5
4( x + 2)
= (2 x - 1)2/5 + A
5(2 x - 1)3/5 B

(2 x − 1) + 4 x + 8
= D
5(2 x − 1)3/5
X
That is, a c d e b
6x + 7 Figure 21.11
f ′(x) =
5(2 x - 1)3/5 Fig. 21.11 shows that the graphical interpretation of Rolle’s
For critical points, f ′(x) = 0 or the function is not defined. Therefore, theorem. The slope of tangent is zero at points C, D and E.
7 1
x =- , Note (Rolle’s Theorem): Let y = f(x) be a given function and satis-
6 2
fies the following conditions:
1.  f(x) be continuous in [a, b].
+ – + 2.  f(x) be differentiable in (a, b).
1/2
If f(a)  = f(b), then f ′(c)  = 0 at least once for some c ∈ (a, b).
–7/6

Figure 21.10 Illustration 21.21   Verify Rolle’s theorem for f(x) = x3(x − 1)2 in
the interval 0 ≤ x ≤ 1. Also find the value of c in between a and b
See Fig. 21.10. Near x = -7 / 6, the sign of f ′(x) changes from where f ′(x)= 0
positive to negative. Therefore, the local maxima is at x = −7 / 6.
Near x = 1/2, the sign of f ′(x) changes from negative to positive. Solution: We have
Therefore, the local minima is at x = 1/2. f(x) = x3 (x − 1)2(1)
Therefore,
Illustration 21.20   Find the local maximum/minimum points of
f(x) = (x − 2)3 (x − 3). f ′(x) = 3x2 (x − 1)2 + x3[2(x − 1)] = x2(x − 1)[3(x − 1) + 2x]
or f ′(x) = x2(x − 1)(5x − 3) (2)
Solution:
f ′(x) = (x − − 11) = 0
2)2(4x Clearly, f ′(x) is finite for all x and hence f(x) is differentiable at all x.
⇒x = 2, 2, 11/4 Therefore,
are critical points.  (i) f(x) is continuous at all x and hence also continuous in the
f ′′(x) = 2(x − 2) (4x − 11) + 4(x − 2)2 closed interval [0, 1].
f ′′(11/4) > 0 ⇒ x = 11/4  (ii)  f(x) is differentiable in the open interval (0, 1).
is local minimum point. from Eq. (1), f (0) = 0 and f (1) = 0. Therefore, f(0) = f(1).
(iii) 
f ′′(2) = 0 Hence, all conditions of Rolle’s theorem are satisfied. Now from
Therefore, the test for second derivative fails in this case. Eq. (2),
f ′′′(x) = 2(4x − 11) + 16 (x − 2) f ′(c)  = 0 ⇒ c2(c − 1)(5c − 3) = 0
f ′′′(2) = − 6 ≠ 0 3
At the earliest non−zero derivative at x = 2 is of odd order, x = 2 is or c = 0, 1,
5
neither local maximum nor local minimum point. Hence, x = 11/4
However, 0 < c < 1. Therefore, c = 3/5. Thus, there exists at least
is the one and only local minimum point of f(x).
one c, that is, c = 3 / 5 between 0 and 1 such that f ′(c)  = 0. Hence,
Rolle’s theorem has been verified.
21.8  Mean Value Theorems
Illustration 21.22   Taking the functions f(x) = (x − 3)logx, prove
21.8.1  Rolle’s Theorem that there is at least one value of x in (1, 3) which satisfies xlogx
= 3 − x.
If f(x) is continuous in the interval [a, b] and differentiable in (a, b)
and further f(a)  = f(b), then there is at least one point x = c on the Solution: We have
interval (a, b), where f ′(c)  = 0. f(x) = (x − 3)logx(1)

Mathematical Problem Book for JEE.indb 906 07-06-2018 13:45:05


Chapter 21 | Applications of Derivatives 907

Therefore, Solution: We have


1 f(x) = 3x2 + 5x + 7 (1)
f ′(x) = (x − 3) + 1(logx)(2) Therefore,
x
Clearly, f ′(x) is finite for all positive values of x and hence f(x) is f(1) = 3 + 5 + 7 = 15
differentiable for all x > 0. Therefore, f(x) is differentiable in (1, 3); and f(3) = 27 + 15 + 7 = 49
and therefore f(x) is also continuous in [1, 3]. Also Now, f ′(x) = 6x + 5 (2)
f(1) = (1 − 3)(log 1) = 0 and f(3) = (3 − 3)(log 3) = 0 Here a = 1 and b = 3. Now from Lagrange’s mean value theorem,
f (b ) − f ( a )
Therefore, f '(c ) =
b−a
f(1) = f(3)
Therefore,
Thus, by Rolle’s theorem, there will be at least one value of x in
f (3) − f (1) 49 − 15
(1, 3) such that f ′(x)=0. Therefore, from Eq. (2), we get 6c + 5 = = = 17
3 −1 2
x −3
+ log x = 0 or 6c = 12 and thus c = 2.
x
or    x(logx) = 3 − x Illustration 21.24   Using mean value theorem, show that |cosa
− cosb| ≤ |a − b|.

21.8.2  Lagrange’s Mean Value Theorem Solution:


Case I: When a = b,
If f(x) is continuous in the interval [a, b] and differentiable in (a, b),
|cosa − cosb| = 0 = | a − b|(1)
then there exists at least one point x = c in the interval (a, b), where
f (b ) − f ( a ) Case II: When a ≠ b, let a < b. Let
f ′(c)  =
b−a f(x) = cosx
Y
B Then
E
C f ′ (x) = −sinx
Clearly, f(x) is differentiable and continuous at all x. Therefore, by
D Lagrange’s mean value theorem, there will be at least one c, a < c
< b such that
A
f (b ) − f ( a )
= f ′( c )
b−a
cos b − cos a
X
or = − sin c
O a c d e b b−a
Therefore,
Figure 21.12
cos a − cos b
The geometrical meaning is clearly depicted in the graph shown = − sin c
a−b
in Fig. 21.12. Here,
f (b ) − f ( a ) cos a − cos b
or = − sin c ≤ 1  [∴ |sinq  | ≤ 1]
b−a a−b
is the slope of the chord AB. The tangents at C, D and E are parallel or |cos a − cos b| ≤ |a − b|(2)
to this chord. Rolle’s theorem is a special case of Lagrange’s mean From Eqs. (1) and (2), for all values of a and b,
value theorem.
|cos a − cos b| ≤ |a − b|
Note (Lagrange’s Mean Value Theorem): If y = f(x) be a given func-
tion, which is (i) continuous in [a, b] and (ii) differentiable in (a, b),
then 21.9 Geometrical Problems
f (b ) − f ( a ) In this section, we will use differential calculus in optimization
f' (c ) =
b−a problems.

at least once for some c ∈ (a, b). Illustration 21.25   Find the cone of maximum volume that can
be inscribed in a sphere of radius R.
Illustration 21.23  Find c of the Lagrange’s mean value theorem
for the function f(x) = 3x2 + 5x + 7 in the interval [1, 3]. Solution: Let ABC be the cone with radius R (Fig. 21.13).

Mathematical Problem Book for JEE.indb 907 07-06-2018 13:45:06


908 Mathematics Problem Book for JEE

A D x C

A B
B
M C Figure 21.14
The volume (V) of the box is
(a − 2x)(b − 2x)x
That is,
Figure 21.13
V = abx − 2(a + b) x2 + 4x3
Here, BM = MC = x and the height AM = y. In ∆OMB,
Therefore,
BM2 + OM2 = OB2 dV
= ab − 4x (a + b) + 12x2
x2 + (y − R)2 = R2 dx
Therefore, d 2V
x2 = 2Ry − y2 ⇒ = 24x − 4 (a + b)
dx 2
The volume (V ) of the cone is
Now, dV /dx = 0, when 12x2− 4(a + b)x + ab = 0 or when
1 p
p x 2 y = y (2 R y − y 2 )
3 3 4 (a + b ) ± 16 (a + b )2 − 48ab
x =
24
p
= (2 Ry 2 − y 3 )
3 (a + b ) ± a2 − ab + b2
Therefore, =
6
dV p p
= (4 Ry − 3 y 2 ) = y (4 R − 3 y ) The plus sign gives a value of x greater than b /2 and hence not
dy 3 3
admissible.
dV 4R When
=0 ⇒ y = (y = 0 is meaningless in this context)
dy 3
(a + b ) − a2 − ab + b2
x=
Now, 6
d 2V p d 2V / dx 2 is negative. Therefore, V is maximum when
= (4 R − 6 y )
dy 2 3
(a + b ) − a2 − ab + b2
4 R d 2V p x=
When y = , is (4R − 8R) which is negative. Therefore, 6
3 dy 2 3
4R
V is maximum when y = . The cone has maximum volume Your Turn 2
3
4R 2 2R
  1. Separate the intervals in which the function f(x) = sin x + cos x
when height is and radius is . sin x + cos x is increasing and decreasing.
3 3
æ pö p p 
Ans.  Increasing in ç 0, ÷ ; decreasing in  , 
Illustration 21.26   A rectangular sheet of metal has four equal è 4 ø 4 2
square portions removed from the four corners and the sides are  2. Prove that x > sinx for all x ∈ (0, ∞).
then turned up so as to form an open rectangular box. Show that State true or false: The function f(x) = sinx +
 3.  3 cos x has a
when the volume contained in the box is maximum, the depth will maximum value at x = p / 6.
be (1/ 6)[(a + b ) − a2 − ab + b2 ] where a and b, (a > b) are the Ans.  True
sides of the original rectangle. The global maximum value of f(x) =
 4.  x2 − 4x + 20 in the interval
Solution: Let ABCD be the given rectangular sheet of metal with AB [0, 5] is ______.
= a, BC = b and x be the side of the four squares cut off (Fig. 21.14). Ans.  25

Mathematical Problem Book for JEE.indb 908 07-06-2018 13:45:07


Chapter 21 | Applications of Derivatives 909

State true or false: The function f(x) = x2/3 does not have any
 5.   2. 
Find the coordinates of the points on the curve
local extrema. y = ( x 2 − 1) / ( x 2 + 1), x > 0 such that tangent at these point(s)
Ans.  False have the greatest slope.
 6. Find the minimum value of xx. Solution:
1/ e 2
Ans.  (1/ e ) y=1−
x2 +1
Verify Rolle’s theorem for f(x) = (x − 1)2 (x − 2) in the interval
 7.  The slope(s) is(are)
[1, 2]. dy 4x
s= = 2
2
Find c of mean value theorem for f ( x ) = x − 4 , a = 2 and b
 8.  dx ( x + 1)2
= 3. Therefore,
d 2 y ds 4 ( x + 1) 1− ( x )2( x + 1)2 x 
2 2 2
Ans.  5 = =
 9. 
Prove that among all rectangles with the given perimeter, the dx 2 dx ( x 2 + 1)4
square has maximum area.  x 2 + 1− 4 x 2 
= 4  2 3  
10. Find the range of the function f(x) = x3 − 3x2 + 6x − 2 where x  ( x + 1) 
∈ [−1, 1].
 1  1
Ans.  [−12, 2] −12  x +  x− 
4(1− 3 x 2 )  3  3
= =
(1+ x 2 )3 (1+ x 2 )3
Additional Solved Examples ds 1 1  d 2s 
Therefore, for = 0, x = , − and  2  <0
3 x ( x + 1) π
dx 3 3  dx  x =1 3
Prove that sinx + 2x ≥
 1.  ∀ x∈ 0,  . (Justify the 1
π  2 1
Then the maximum is at x = and y = − . Hence, the point is
inequality, if any used). 3 2
Solution:  1 1
 , − .
y  3 2

 3. 
Prove that there exist exactly two non−similar isosceles
triangles ABC such that tanA + tanB + tanC = 100.
Solution: Let A = B. Then
/2 2A + C = 180°
O x and 2tanA + tanC = 100
Now,
2A + C = 180°
⇒ tan2A = −tanC(1)
Also,
y = f (x) 2tanA + tanC = 100
  ⇒ 2tanA − 100 = −tanC(2)
Figure 21.15 From Eqs. (1) and (2),
See Fig. 21.15. Let us consider 2 tan A
2tanA − 100 =
f(x) = 3x2 + (3 − 2p)x − p sinx 1− tan2 A
Let tanA = x. Then
When f(0) = 0, f (π / 2) is negative. Therefore, 2x
= 2x − 100
f ′(x) = 6x + 3 − 2p − p cosx 1− x 2
f″(x) = 6 + p sinx > 0 ⇒x3 − 50x2 + 50 = 0
 π Let f(x) = x − 50x + 50. Then f ′(x) = 3x2 − 100x. Thus, f ′(x) = 0 has
3 2
It is clear that f ′(x) is increasing function in 0,  and there is no 100 100 
 2 roots 0, . Also f(0), f   < 0. Thus, f(x) = 0 has exactly three
3  3 
 π distinct real roots. Therefore, tanA has three distinct values; how-
local maxima of f(x) in 0,  . The graph of f(x) always lies below
 2 ever, one of them will be obtuse angle. Hence, there exists exactly
 π  π two non−similar isosceles triangles.
the x−axis in 0,  . Therefore, f(x) ≤ 0 in x ∈ 0, 2  . Now,
 2   Let A(p2, −p), B(q2, q), C(r2, −r) be the vertices of a triangle ABC.
 4. 
3x2 + 3x ≤ 2px + p sinx A parallelogram AFDE is drawn with D, E and F on the line
segments BC, CA and AB, respectively. Show that the maximum
3 x ( x + 1)
⇒ sinx + 2x ≥ area of the parallelogram is (1/ 4)[( p + q )(q + r )( p − r )] , given
π
p > r.

Chapter 21.indd 909 10-06-2018 18:14:45


910 Mathematics Problem Book for JEE

Solution: See Fig. 21.16. Let AF = lAB; AE = mAC. The area of the
d 2L
parallelogram = AF·AEsinA. =0
dt 2 t = 0
A(p 2, −p)
d 3L
However, ≠ 0 implies that there is neither maxima nor
dt 3 t = 0

d 2L
minima at t = 0. Also, > 0 at t = 1, which implies that L is
F E dt 2
minimum at t = 1. So, the shortest distance is
13
(Value of I at t = 1) − (Radius of circle) = −1
3
 6. Find a polynomial f(x) of degree 5 which increases in the
interval (−∞, 2] and [6, ∞) and decreases in the interval [2, 6].
B(q 2, q) D C(r 2, −r ) Given that f(0) = 3 and f ′(4) = 0.
Figure 21.16 Solution: See Fig. 21.17. The wavy curve of the derivative will be
In similar triangles, namely, ABC and FBD, we have f ′(x)= k(x− 2)(x − 4)2(x − 6)
and k > 0.
FB BD FD
= = ⇒1−l=m
AB BC AC
The area is
lm(AB)·(AC)sinA
That is,
l(1 − l)AB·(AC)sinA 4
The area is maximum when l is 1/2 (the vertex of parabola y = l − l2),
2 6
which implies that
1
m=l=
2
That is, F and E are the mid−points of AB and AC, respectively.
1
Areamax. = AB ⋅ AC sin A Figure 21.17
4
p2 − p 1 f(x) = k ∫ ( x 2 − 8 x + 12)( x 2 − 8 x + 16) dx
1 1 2
= (Area of ∆ABC ) = q q 1
2 2 2 = k ∫ [ x 4 − 16 x 3 + 64 x 2 + 28( x 2 − 8 x ) + 192]dx
r −r 1
 x5 x3 
1 = k  − 4 x 4 + 92 − 112 x 2 + 192 x  + c
= (p + q)(q + r)(p − r)  5 3 
2
Find the shortest distance between the curves 9x2 + 9y2 − 30y
 5.  Now, since f(0) = 3, we have
+ 16 = 0 and y2 = x3.  x5 92 3 
f(x) = k  − 4 x 4 + x − 112 x 2 + 192 x  + 3, k > 0
Solution: The equation 9x2 + 9y2 − 30y + 16 = 0 can be rewritten as  5 3 
5
2   7. A conical vessel is to be prepared out of a circular sheet of gold

x2 +  y −  = 1 of unit radius. How much sectorial area is to be removed from
 3
the sheet so that the vessel has maximum volume?
Any point on the curve y2 = x3 can be taken to be (t2, t3). Let l be
Solution: See Fig. 21.18.
the distance between the centre of the given circle and the point
(t2, t3). Then Lateral height of the cone = Radius of the circle = 1
2
 5
L = l2 = t 4 +  t3 − 
 3
Now, we calculate the minimum value of l. The required distance is
l − Radius of given circle O
dL  5
Now, = 4t 3 + 2  t 3 −  ·3t2 = 0
dt  3 2θ
For maximum or minimum, t = 0 or 1. Now,
B
d 2L A
= 12t2 + 30t4 − 20t
dt 2
Figure 21.18

Mathematical Problem Book for JEE.indb 910 07-06-2018 13:45:10


Chapter 21 | Applications of Derivatives 911

The lateral area of the cone is the area of the circle with sectorial real values of b such that f(x) has the smallest value at x = 1.
area removed, that is, Solution: At x = 1, f(x) = −1.
p (1)2 Smallest value of f(x) = −1. At all other points of the interval, f(x) > −1.
p r(1) = (2p − 2q )
2p Now, for x ≥ 1, f(x) = 2x −3. So,
That is,
f ′(x) = 2 > 0
p −q
r =  (here r is radius of the cone) ⇒f(x) is an increasing function
p
Height h of the cone is ⇒Least value exists at x =1
Now, for x < 1, f ′(x) = −3x2 < 0. Therefore,
12 − r 2 f(x) is decreasing function in the interval 0 ≤ x < 1.
Volume of the cone is Therefore, f(x) is the smallest at x =1 and

3  b − b + b − 1
3 2

1  p −q 
2 2 f(1 − 0) = lim −(1− h) +  2  ≥ −1
1 2 2  p −q 
V= p r h = p   × 1 − 
h→0
 b + 3b + 2 
3 3  p   p 
(b2 + 1)(b − 1)
⇒ −1 + ≥ −1
Upon maximizing the volume V, we get (b + 1)(b + 2)
p −q 2 (b2 + 1)(b − 1)
= ⇒ ≥0
p 3 (b + 1)(b + 2)
 2 (b − 1)
⇒ q  = p  1− ⇒ ≥ 0 (since b2 +1 is positive)
  (b + 1)(b + 2)
 3 
The area of the sectorial area removed is ⇒ b ∈ (−2, −1) ∪ [1, ∞)
1 2  2 10. Find all the possible values of the parameter a so that the
(1) (2q ) = p  1− 
2  3  function, f(x) = x3 − 3(7 − a)x2 − 3 (9 − a2)x + 2, has a negative

point of local minimum.
 8. Let a + b = 4, where a < 2, and let g(x) be a differentiable
a b Solution: We have
function. If (dg / dx ) > 0 for all x, prove that ∫ g( x )dx + ∫ g( x )dx f(x) = x3 − 3(7 − a)x2 − 3(9 − a2)x + 2
0 0 f ′(x) = 3x2 − 6(7 − a)x − 3(9 − a2)
increases as (b − a) increases.
For the distinct real roots, D > 0
Solution: Let b − a = t. Given that 36(7 − a)2 + (4 × 3 × 3)(9 − a2) > 0
a+b=4
⇒ 49 + a2 − 14a + 9 − a2 > 0
Thus,
29
t t ⇒ 14a < 58 ⇒ a <
a=2− ;b=2+ 7
2 2 For local minima:
Let us consider f ′′(x) = 6x − 6(7 − a) > 0
a b
⇒x−7+a>0
f(t) = ∫ g( x )dx + ∫ g( x )dx
0 0 Now, 7− a < x as x must be negative. So,
2 −( t /2) 2 + ( t /2) 7−a<0⇒a>7
Now, f(t) = ∫ g( x )dx + ∫ g( x )dx Thus, by contradiction, that is, for real roots, a < (29/7) and for
0 0
negative point of local minimum a > 7. No possible value of a.
 t   1  t   1
f ′(t) = g  2 −   −  + g  2 +   
 2  2  2  2  Previous Years' Solved JEE Main/AIEEE
1
f ′(t) = [ g(b ) − g(a)] Questions
2
 1. 
A value of C for which the conclusion of Mean Value Theorem
dg
Since > 0 for all values of x, g(x) is increasing since b > a holds for the function f ( x ) = loge x on the interval [1, 3] is
dx
1
g(b)  > g(a) (A)  2log3 e (B)  loge 3
2
Hence, f ′(t) > 0, that is, f(t) increasing as t increases. Therefore, f(t)
(C)  log3 e (D)  loge 3
increases as (b − a) increases.
Solution: Using mean value theorem, we get
 3 b3 − b2 + b − 1 f (3) − f (1) 1 log3 − log1 2
Let f(x) =  − x + 2
 9.  , 0 ≤ x < 1 . Find all possible f ′(c ) = ⇒ = ⇒c = = 2log3 e.
 b + 3b + 2 3 −1 c 2 loge 3
 2 x − 3, 1≤ x ≤ 3
 Hence, the correct answer is option (A).

Mathematical Problem Book for JEE.indb 911 07-06-2018 13:45:13


912 Mathematics Problem Book for JEE

The function f(x) = tan−1(sin x + cos x) is an increasing function


 2.  How many real solutions does the equation x7 + 14x5 + 16x3
 4. 
in + 30x − 560 = 0 have?
p p   p p
(A)   ,  (B)   − ,  (A)  7 (B)  1
4 2  2 4
(C)  3 (D)  5
 p  p p
− , 
(C)   0,  (D)  [AIEEE 2008]
 2  2 2
[AIEEE 2007] Solution:
Solution: x 7 + 14 x 5 + 16 x 3 + 30 z − 560 = 0
 p
2 cos  x + 
1  4 Let f ( x ) = x 7 + 14 x 5 + 16 x 3 + 30 x. Then
f ′( x ) = (cos x − sin x ) =
1+ (sin x + cos x )2 1+ (sin x + cos x )2
p p p f ′( x ) = 7 x 6 + 70 x 4 + 48 x 2 + 30 > 0 ∀x
p 
f(x) is increasing if f ′ ( x ) > 0 for cos  + x  > 0 ⇒ − < x + <
4  2 4 2 Therefore, f(x) is a strictly increasing function for all x.
3p p So, it can have at the most one solution.
⇒− <x<
4 4 Hence, the correct answer is option (B).
 p p
Hence, f(x) is increasing when x ∈ − ,  Given P( x ) = x 4 + ax 3 + bx 2 + cx + d such that x = 0 is the
 5. 
 2 4
only real root of P′(x) = 0. If P(−1) < P(1), then in the interval
Hence, the correct answer is option (B). [−1, 1]
 3. Suppose the cubic x3 − px + q has three distinct real roots (A)  P(−1) is the minimum and P(1) is the maximum of P
where p > 0 and q > 0. Then which one of the following holds (B)  P(−1) is not minimum but P(1) is the maximum of P
true?
(C)  P(−1) is the minimum and P(1) is not the maximum of P
p p
(A)  The cubic has minima at and maxima at − (D)  neither P(−1) is the minimum nor P(1) is the maximum of P
3 3
p p [AIEEE 2009]
(B)  The cubic has minima at − and maxima at
3 3 Solution: P (x) = x 4 + ax 3 + bx 2 + cx + d ; P ′( x ) = 4 x 3 + 3ax 2 + 2bx + c
p p
(C)  The cubic has minima at both and − As x = 0 is a solution for P ′( x ) = 0, we have c = 0. Therefore, P( x ) = x 4 + ax 3 + bx 2 + d
3 3
P( x ) = x 4 + ax 3 + bx 2 + d
p p
(D)  The cubic has maxima at both and −
3 3 Further, we have P( −1) < P(1) , which implies that,
[AIEEE 2008]
1− a + b + d < 1+ a + b + d ⇒ a > 0
Solution: Let f ( x ) = x 3 − px + q , from Fig. 21.19, for maxima and
As P′( x ) = 0 , only when x = 0 and P(x) is differentiable in (−1, 1), we
p shall have the maximum and minimum at the points x = −1, 0 and
minima, f ′( x ) = 0. This implies that 3 x 2 − p = 0 ⇒ x 2 =
3 1 only. Also, we have, P(−1) < P(1). Therefore,
p
Therefore, x = ± . Now, Maximum value of P(x) = Maximum value of {P (0), P (1)};
3
 Minimum value of P(x) = Minimum value of {P(−1), P(0)}.
p  p
f ′′ ( x ) = 6 x ⇒ f ′′  −  < 0 ⇒ f ′′   > 0
 3  3 In the interval [0, 1], we have,

P ′( x ) = 4 x 3 + 3ax 2 + 2bx = x (4 x 2 + 3ax + 2b)

p As P′( x ) has only one root, that is, x = 0, 4x2 + 3ax + 2b = 0 has
3 3a2
no real roots. Therefore, (3a)2 − 32b < 0 ⇒ < b. Thus, b > 0.
32
p
− Therefore, we have a > 0 and b > 0. That is,
3
P′( x ) = 4 x 3 + 3ax 2 + 2bx > 0 ∀x ∈ (0, 1)

Hence, P(x) is increasing in [0, 1]. Therefore, maximum value of


Figure 21.19 P( x ) = P(1)

p p Similarly, P(x) is decreasing in [−1, 0]. Therefore, minimum value of


Therefore, there is a maxima at − and minima at . P(x) does not occur at x = − 1.
3 3
Hence, the correct answer is option (A). Hence, the correct answer is option (B).

Mathematical Problem Book for JEE.indb 912 07-06-2018 13:45:16


Chapter 21 | Applications of Derivatives 913

 6. The shortest distance between the line y − x =1 and the curve That is, at x = p,
x = y2 is f’′(x) < 0 ⇒ maxima;
3 2 2 3 at x = 2p,
(A)  (B)  f’′(x) > 0 ⇒ minima.
8 8
3 2 3 Hence, the correct answer is option (C).
(C)  (D)  4   9. A spherical balloon is filled with 4500p  cubic meters of helium
5
gas. If a leak in the balloon causes the gas to escape at the
[AIEEE 2009] rate of 72p cubic meters per minute, then the rate (in meters
Solution: Let us consider that line (1) be x − y + 1 = 0 and line (2) per minute) at which the radius of the balloon decreases
be the tangent to the curve x = y 2 . Therefore, 49 minutes after the leakage began is
dy dy 1 9 7 2 9
1= 2 y ⇒ = = Slope of given line (2) (A)  (B)  (C)   (D) 
dx dx 2 y 7 9 9 2
In order to find the shortest distance, these two lines should be [AIEEE 2012]
parallel, therefore equating their slopes, 4 3
Solution: Volume of the sphere is given by: v = p r
1 1
2 3
 1 1  1 1
= 1⇒ y = ⇒ x =   = ⇒ ( x , y ) =  ,  After 49 minutes of leakage, the volume is: 4500p − 49(72p) = 972p.
2y 2 2 4 4 2 Therefore,
This is the point on the curve from which if a perpendicular is 4 3
drawn on to the given line, then the length of that perpendicular π r = 972π ⇒ r 3 = 729 ⇒ r = 9
3
will be the shortest distance between the line and the curve. Therefore, the rate (in meters per minute) at which the radius of
1 1 the balloon decreases 49 min after the leakage began is
− +1
4 2 3 3 2 4 dv 4 dr dr dr 72 2
Therefore, the shortest distance is = = v = πr3 ⇒ = π 3r 2 ⇒ 72π = 4π r 2 ⇒ = =
1+ 1 4 2 8 3 dt 3 dt dt dt 4 ⋅ 9 ⋅ 9 9
Hence, the correct answer is option (A). Hence, the correct answer is option (C).
4 10. 
The population p(t) at time t of a certain mouse species
The equation of the tangent to the curve y = x + 2 , that is,
 7. 
parallel to the x−axis is x dp(t )
satisfies the differential equation = 0.5 p(t ) − 450. If
dt
(A)  y = 1 (B)  y = 2
p(0) = 850, then the time at which the population becomes
(C)  y = 3 (D)  y = 0
zero is 1
[AIEEE 2010] (A)  2 ln 18   (B)  ln 9 (C)  ln18  (D)  ln 18
2
Solution: Since the equation is parallel to x−axis, we have, [AIEEE 2012]
dy 8 Solution: We have,
= 0 ⇒ 1− 3 = 0
dx x d ( p(t )) 1
= p(t ) − 450
Therefore, dt 2

x =2 ⇒ y =3 d ( p(t )) p(t ) − 900 d ( p(t ))


⇒ 2∫
p(t ) − 900 ∫
⇒ = = dt ⇒ 2ln p(t ) − 900 = t + c
dt 2
Thus, the equation of tangent is That is, t = 0 ⇒ 2ln50 = 0 + c ⇒ c = 2ln50
y − 3 = 0(x − 2) ⇒ y − 3 = 0 ⇒ y = 3
Therefore, 2ln p(t ) − 900 = t + 2ln50
Hence, the correct answer is option (C).
⇒ P(t ) = 0 ⇒ 2ln900 = t + 2ln50
x
 5π  900 
 8. For x ∈  0,  , define f ( x ) = ∫ t sin t dt . Then f has ⇒ t = 2(ln900 − ln50) = 2ln   = 2ln18.
 2  0  50 
(A)  local minimum at p  and 2p Hence, the correct answer is option (A).
(B)  local minimum at p  and local maximum at 2p
(C)  local maximum at p  and local minimum at 2p 11. 
Let a, b ∈ R be such that the function f given by
(D)  local maximum at p  and 2p 2
f ( x ) = ln x + bx + ax , x ≠ 0 has extreme values at x = −1 and
[AIEEE 2011]
x = 2.
Solution: We have, Statement 1: f  has local maximum at x = −1 and at x = 2.
f ′( x ) = x sin x ⇒ f ′( x ) = 0 1 −1
Statement 2: a = and b =
2 4
This implies that x = 0 or sin x = 0 and x = 2p, p. Therefore, (A)  Statement 1 is false, statement 2 is true.
1 1
f ′′( x ) = x cos x + sin x ⇒ (2 x cos x + sin x ) (B) Statement 1 is true, statement 2 is true; statement 2 is a
2 x 2 x correct explanation for statement 1.

Chapter 21.indd 913 10-06-2018 18:15:47


914 Mathematics Problem Book for JEE

(C)  Statement 1 is true, statement 2 is true; statement 2 is not Therefore, one tangent passes through the point (2, 2) and has
a correct explanation for statement 1. slope 2
(D)  Statement 1 is true, statement 2 is false. y − 2 = 2( x − 2) ⇒ y = 2 x − 2
[AIEEE 2012] The other tangent passes through the point (−2, −2) and has slope 2
Solution:
1 y + 2 = 2( x + 2) ⇒ y = 2 x + 2
f ′( x ) = + 2bx + a
x Substituting y = 0, we get x−intercepts as, x = 1 and −1.
It is given that f has extreme values and hence differentiable. Hence, the correct answer is option (D).
Therefore, The real number k for which the equation 2x3 + 3x + k = 0 has
14. 
1 1 1 two distinct real roots in [0, 1]
f ′( −1) = 0 ⇒ a − 2b = 1; f ′(2) = 0 ⇒ a + 4b = − ⇒ a = ; b = −
2 2 4
(A)  lies between 2 and 3 (B)  lies between −1 and 0
Therefore, f ′′( −1), f ′′(2) are negative and f has local maxima at x (C)  does not exist (D)  lies between 1 and 2
= −1 and 2. [JEE MAIN 2013]
Hence, the correct answer is option (B).
Solution: When the given equation, 2x3 + 3x + k = 0 has two dis-
12.  Consider the function f ( x ) = x − 2 + x − 5 , x ∈ R. tinct real roots in [0, 1], then f ′(x) will change sign, but f ′(x) = 6x2 +
3 > 0, for all values of x ∈ R. Therefore, no value of k exists.
Statement 1: f ′(4) = 0.
Hence, the correct answer is option (C).
Statement 2: f is continuous in [2, 5], differentiable in (2, 5) 1
and f(2) = f(5). 15. If g is the inverse of a function f and f '( x ) = , then g′(x)
is equal to 1 + x5
(A)  Statement−1 is false, statement−2 is true.
(B) Statement−1 is true; statement−2 is true; statement 2 is a 1
(A)  (D)  1+ { g( x )}5
correct explanation for statement−1. 1+ { g( x )}5
(C) Statement−1 is true; statement−2 is true; statement 2 is (C) 1 + x5 (D) 5x4
not a correct explanation for statement−1. [JEE MAIN 2014 (OFFLINE)]
(D)  Statement−1 is true, statement−2 is false.
[AIEEE 2012] Solution: Given g (x) = f  −1 (x). Therefore,
f (g(x)) = x
Solution: f ′(g(x)) g′(x) = 1
f (x)= 7 − 2x; x > 2
1 1
= 3; 2≤ x ≤5 Thus, g' ( x ) = =
f ′ ( g( x )) 1
= 2 x − 7; x > 5 1+ { g( x )}5
f(x) is constant function in [2, 5]. f is also continuous in [2, 5] and Thus, g′ (x) = 1+ {g(x)}5.
differentiable in (2, 5) and f(2) = f(5); by Rolle’s theorem f ′(4) = 0. Hence, the correct answer is option (B).
Therefore, both Statement 2 and Statement 1 are true and
16. If f and g are differentiable functions in [0, 1] satisfying f(0) = 2
Statement 2 is correct explanation for Statement−1.
Hence, the correct answer is option (B). = g(1), g(0) = 0 and f(1) = 6, then for some c ∈[0, 1]
(A)  f ′(c)  = g′(c)  (B)  f ′(c) = 2g′(c)
13. 
The intercepts on x−axis made by tangents to the curve,
x (C) 2f ′(c)  = g′(c)  (D) 2f ′(c) = 3g′(c)
y = ∫ t dt , x ∈R , which are parallel to the line y = 2x, are [JEE MAIN 2014 (OFFLINE)]
0
equal to Solution: Let h (x) = f (x) − 2g(x). Then
(A)  ±2 (B)  ±3 h (0) = f (x) − 2 g(x) = 2 − 2 × 0 = 2
and h (1) = f (1) − 2 g(1) = 6 − 2 × 2 = 2
(C)  ±4 (D)  ±1
[JEE MAIN 2013] Now h(x) is a differentiable function in [0, 1] and h(0) = h(1), so by
Rolle’s theorem h′(c)  = 0 for some c ∈ (0, 1). Therefore,
Solution: Slope of the tangent to the curve will be 2. So we can
equate the slope as,   0 = f ′ (c) − 2 g′ (c) ⇒ f ′ (c) = 2 g′ (c)
dy Hence, the correct answer is option (B).
= x = 2 ⇒ x = ±2
dx
For x = 2, we have, 17. If x = −1 and x = 2 are extreme points of f (x) = a log |x| + b x2
2 + x, then
y = ∫ t dt = 2 1 1
0
(A)  a = 2, b = − 2 (B)  a = 2, b = 2
For x = −2, we have, 1 1
−2 (C)  a = −6, b = 2 (D)  a = −6, b = −
2
y= ∫ t dt = −2 [JEE MAIN 2014 (OFFLINE)]
0

Mathematical Problem Book for JEE.indb 914 07-06-2018 13:45:19


Chapter 21 | Applications of Derivatives 915

Solution: 3
f (x) = a log(x) + b x2 + x = θ × (cosθ − sinθ )
α e
f '( x ) = + 2β x + 1 Since tangent is parallel to x−axis, we have
x
dy π
Now f ′(− 1) = − a − 2b + 1 = 0 = 0 ⇒ sinθ = cosθ (since, eθ > 0) ⇒ θ =
dx 4
α
f '(2) = + 4 β x + 1= 0 Hence, the correct answer is option (C).
2
Thus, the equations are 20. 
The volume of the largest possible right circular cylinder that
α + 2 β − 1 = 0 and α + 8 β + 2 = 0 can be inscribed in a sphere of radius = 3 is
On solving the above equations, we get 4 8
1 (A)  3 π (B)  3π
β = − ,α = 2 3 3
2
(C)  4p (D)  2p
Hence, the correct answer is option (A).
[JEE MAIN 2014 (ONLINE SET-2)]
If the Rolle’s theorem holds for the function f(x) = 2x3 + ax2
18. 
1 Solution: See Fig. 21.20.
+ bx in the interval [−1, 1] for the point c = , then the value Volume of required cylinder = V = p r2 h  (1)
2
of 2a + b is
(A)  1 (B)  −1 (C)  2 (D)  − 2
[JEE MAIN 2014 (ONLINE SET-1)]
 1
Solution: Since Rolle’s theorem holds, so f ′   = 0 . Now
2
f ’ (x) = 6x2 + 2ax + b
2 O h
 1  1 1
f ′   = 6   + 2a × + b
2 2 2 3
6 −3
or   +a+b =0⇒a+b = (1)
4 2
A
Since, f (− 1) = f (1) B
⇒ 2 (− 1)3 + a (− 1)2 + b (− 1) = 2 + a + b
⇒ −2 + a − b = 2 + a + b Figure 21.20
Therefore, Now, by Pythagoras theorem, we have
2b=−4⇒b=−2 2
( 3) h
2
1 = r2 +  
Therefore, from Eq. (1), we get a = . Thus, 2
2
h2
 1 ⇒ r2 = 3 − (2)
2a + b = 2   − 2 = −1 4
2 Therefore,
Hence, the correct answer is option (B).
 h2   h3 
For the curve y = 3 sinq cosq, x = eq sin q, 0 ≤ q ≤ p, the tangent
19.  V = p  3 −  h = p  3h −  (3)
 4   4 
is parallel to x−axis when q  is
3π π  1   3h2 
(A)  (B)  V ′ = p  3 − × 3h2  = p  3 − 
4 2  4   4 
π π For Extreme V,
(C)  (D) 
4 6 3 2
V′ = 0 ⇒ 3 = h ⇒ h2 = 4 ⇒ h = ±2
[JEE MAIN 2014 (ONLINE SET-2)] 4
Now
Solution: 3p 3p
V ′′ = − × 2h = − × 2 × 2 = −3p < 0
3 d 4 4
sin2q
dy dy / dq 2 dq
= = q Therefore, volume is maximum when h = 2. From Eq. (1), required
dx dx / dq e cosq + (sinq )eq volume is
3  1 
(cos2θ )2 V = π  3 × 2 − × 23  = π (6 − 2) = 4π
2 3{cos2 θ − sin2 θ }  4 
= θ = θ
e (cosθ + sinθ ) e (cosθ + sinθ ) Hence, the correct answer is option (C).

Chapter 21.indd 915 10-06-2018 18:17:13


916 Mathematics Problem Book for JEE

1 Therefore,
If f ( x ) = x 2 − x + 5, x >
21.  and g(x) is its inverse function, then
2 f(q  ) = 1 (1 + sin q cos q ) − cos q (− sinq − cos q ) + 1 (− sin2 q + 1)
g′(7) equals
= 1 + sin 2 q + 2 cos2 q = 1 + sin 2 q + 1 + cos 2 q
1 1 1 1
(A)  − 3     (B)  13     (C)      
(D)  −   p 
3 13 = 2 + 2 sin  2q +  
  4 
[JEE MAIN 2014 (ONLINE SET-3)]
Solution: Therefore, max. f(q  ) = 2 + 2 and min. f (q  ) = 2 − 2 since,
1  p
f ( x ) = x 2 − x + 5, x > −1 ≤ sin  2q +  ≤ 1
2  4
Now
Hence, the correct answer is option (C).
(f )
(7) ′ =
−1 1
=
1
=
1
=
1
( −1
f ′ f (7) ) f ′(2) 2(2) − 1 3 24. 
If the volume of a spherical ball is increasing at the rate of
4p cc/sec, then the rate of increase of its radius (in cm/sec),
Since f ′ (x) = 2x − 1 and because if f is differentiable and non−
when the volume is 288p cc, is
zero at x = a, then f  −1 is differentiable at x = f(a) = b and we have
1 1
1 (A)  6 (B)  9
(f −1)(b ) =
(
f ′ f −1(b ) ) (C) 
1
(D)  24
1

Hence, the correct answer is option (C). 36


[JEE MAIN 2014 (ONLINE SET-4)]
22. 
Let f and g be two differentiable functions on R such that
Solution:
f ‘(x) > 0 and g’(x) < 0, for all x ∈ R. Then for all x
dV d 4  4p dr dr 1
(A)  f(g(x)) > f (g(x − 1)) = 4p ⇒  p r 3  = 4p ⇒ × 3 r 2 = 4p ⇒ = 2
dt dt  3  3 dt dt r
(B)  f(g(x)) > f (g(x + 1))
dr  dr 
(C)  g(f(x)) > g (f(x − 1)) =
dt  Whenvol. =288 dt  When r = 6
(D)  g(f(x)) < g (f(x + 1))
[JEE MAIN 2014 (ONLINE SET-3)]  4 3 72 3 
∵ 3 p r = 288 ⇒ r = 216 ⇒ r = 6 
 
Solution:
1 1
f ‘(x) > 0 and g’(x) < 0 for all x ∈ R = ⇒
Now 62 36
x > x − 1 and x + 1 > x Hence, the correct answer is option (C).
Therefore,
p 
g (x) < g (x − 1) and g (x + 1) < g (x) The equation of a normal to the curve, sin y = x sin  + y  at
25. 
 3 
Thus,
x = 0, is
f (g (x)) < f (g (x − 1) and f (g (x + 1) < f (g (x)) true
Also (A)  2 x + 3 y = 0 (B)  2 y − 3 x = 0
f (x) > f (x − 1) and f (x + 1) > f (x) (C)  2 y + 3 x = 0 (D)  2 x − 3 y = 0
Therefore, [JEE MAIN 2015 (ONLINE SET-2)]
g (f (x)) < g (f (x − 1)) and g (f (x + 1) < g( f (x))
Solution: Given value curve is
Hence, the correct answer is option (B).
p  dy p  dy p 
1 cosq 1 sin y = x sin  + y  ⇒ cos y = x cos  + y  + sin  + y 
1 2 x 3  dx 3  dx 3 
If f (q ) = − sinq
23.  1 − cosq and A and B are
3 −1 2  dy  3  −dx  −2
−1 sinq 1 ⇒   = ⇒  =
dx 2 dy
respectively the maximum and the minimum values of f(q  ),   x =0   x =0 3
then (A, B) is equal to Therefore, Equation of normal at (0, 0) is
(A)  (3, − 1) (B)  (4,2 − 2 ) −2 −2
( y − 0) = ( x − 0) ⇒ y = x
3 3
(C)  (2 + 2 , 2 − 2 ) (D)  (2 + 2 , − 1)
or 2x + 3y = 0
[JEE MAIN 2014 (ONLINE SET-3)]
Hence, the correct answer is option (A).
Solution: 26. 
Let k and K be the minimum and the maximum values of
1 cos q 1 (1+ x )0.6
the function f ( x ) = in [0, 1], respectively, then the
f (q ) = − sinq 1 − cosq 1+ x 0.6
−1 sinq 1 ordered pair (k, K) is equal to

Mathematical Problem Book for JEE.indb 916 07-06-2018 13:45:24


Chapter 21 | Applications of Derivatives 917

(A)  (1, 20.6) (B)  (2−0.4, 20.6) 4


= 2 x - (1- 2 x ) = 0
(C)  (2−0.6, 1) (D)  (2−0.4, 1) p
[JEE MAIN 2015 (ONLINE SET-2)] Hence,
0.6
(1+ x ) d2A 4
Solution: f ( x ) = , x Î [0,1] ; (k, k) = ?; where k = f(x) min and = 2 x + (2) > 0
1+ x 0.6 dx 2
p
k = f(x) max. 2
(1+ x )3/5 Now, x- (1- 2 x ) = 0
f (x) = p
(1+ x 3/5 ) p   x − 2 + 4x = 0
3 3 æ 2 ö
(1+ x 3/5 ). (1- x )-2/5 - (1+ x )3/5 . x -2/5 Þ x =ç ÷
⇒ f ¢( x ) = 5 5 èp +4 ø
(1+ x 3/5 )2
The minimum value occurs at
é 3/5 2/5 ù
⇒ f ¢( x ) = 3 ê (1+ x ) x - (1+ x ) ú 2
2/5 2/5 3/5 2
5 ëê x (1+ x ) (1+ x ) ûú x=
π +4
3é x 2/5 + x - 1- x ù That is,
⇒ f ¢( x ) = ê
2/5 2/5 3/5 2 ú
5 ëê x (1+ x ) (1+ x ) ûú 4 π
π r = 1- =
3/5 π +4 π +4
Þ f ¢( x ) = 0 for x = 1, −1 \ f(−1) = 0, f (1) = (2)
2 1
⇒r =
(p + 4)
(2)3/5
⇒ f ′( x ) = 0 for x = 1, −1 ∴ f(−1) = 0, f (1) = Þ x = 2r
2
Hence, the correct answer is option (D).
Also f(0) = 1, so in [0, 1], f(x) has
Minimum value = f(1) = (2)−2/5 = (2)0.4 = k 28. 
If m and M are the minimum and the maximum value of
and maximum value = f(0) = 1 = k 1
4 + sin2 2 x - 2cos 4 x , x ÎR , then M − m is equal to
2
Therefore, (k , k ) º (2 -0.4 ,1).
9 15
Hence, the correct answer is option (D). (A)  (B) 
4 4
27. 
A wire of length 2 units is cut into two parts which are bent 7 1
respectively to form a square of side x units and a circle of (C)  4 (D) 
4
radius r units. If the sum of the areas of the square and the
circle so formed is minimum, then [JEE MAIN 2016 (ONLINE SET−1)]

(A) 2x = r (B) 2x = (p + 4)r Solution: We have


(C) (4 − p)x = p r (D)  x = 2r 1
y = 4 + 4(1- cos2 x )cos2 x - 2cos 4 x
[JEE MAIN 2016 (OFFLINE)] 2
Solution: Side length of the square = x = 4 + 2cos2x − 4cos4x
Radius of circle = r = 2(2 + cos2x − 2cos4x)
= 2(−2cos4x + cos2x + 2)
Perimeter of square + Perimeter of circle = 2
4x + 2p r = 2  cos2 x 
= −4  cos 4 x − − 1
Þ 2x + p r = 1  2 
The sum of the area is æ cos2 x 1 1 ö
= -4 çç cos 4 x - + - - 1÷÷
x2 + p r2 =A è 2 16 16 ø
p r = 1 − 2x
é æ 1 ö 17 ù
2

æ 1- 2x ö = ê 4 ç cos2 x - ÷ - ú
⇒ r =ç ÷ êë è 4 ø 16 úû
è π ø
2
p (1- 2 x )2 17 æ 1ö
Now, A= x + 2 = - 4 ç cos2 x - ÷
p2 4 è 4ø
(1- 2 x )2 Therefore,
= x2 + 17
p ymax = M =
4
Therefore,
dA 1 17 æ 9 ö 17 9 8
= 2 x + 2(1- 2 x )( -2) ymin = m = -ç4´ ÷ = - =
dx p 4 è 16 ø 4 4 4

Chapter 21.indd 917 12-06-2018 19:04:52


918 Mathematics Problem Book for JEE

Hence, 2
17 8 9 f ′( x ) = − sin2 x cos2 x (2) = − sin4 x
M−m= − = 2
4 4 4
That is,
Hence, the correct answer is option (A).
4 x ∈ (p ,2p ) ∪ (3p , 4p ) ∪ (5p ,6p )
The minimum distance of a point on the curve y = x2 − 4 from
29. 
the origin is  p p   3p   5p 3p 
x ∈ ,  ∪  ,p  ∪  , 
15 19 15 19 4 2  4   4 2 
(A)  (B)  (C)   (D)  Hence, the correct answer is option (C).
2 2 2 2
[JEE MAIN 2016 (ONLINE SET−1)]

Solution: See Fig. 21.21. We have


  Previous Years' Solved JEE Advanced/
OP2 = x2 + (x2 − 4)2 IIT-JEE Questions
d (OP )2
= 2 x + 2( x 2 − 4)(2 x ) = 0 Paragraph for Questions 1−3: If a continuous function f defined
dx on the real line R, assumes positive and negative value in R, then
Now, x = 0, 1 + 2(x2 − 4) = 0 the equation f(x) = 0 has a root in R. For example, if it is known
that a continuous function f on R is positive at some point and its
7
2x2 − 7 = 0 ⇒ x = ± minimum value is negative then the equation f(x) = 0 has a root
2 in R.
Therefore, from the origin, the minimum distance of the point on
the curve is Consider f(x) = kex − x for all real x where k is a real constant.

2  1.  The line y = x meets y = kex for k ≤ 0 at


7 7  7 1 15 (A)  no point (B)  one point
+ −4 = + =
2 2  2 4 2 (C)  two points (D)  more than two points
[IIT−JEE 2007]
y
Solution: Line y = x intersects the curve y = kex(k ≤ 0) at exactly one
point as shown in Fig. 21.22.

y=x
(0, 0)
x
O
(−2, 0) (2, 0)
x

P (x1, x 2 − 4)
(0, 4)
y = ke x

y
Figure 21.21
Figure 21.22
Hence, the correct answer is option (A).
Hence, the correct answer is option (B).
Let f(x) = sin4x + cos4x. Then, f is an increasing function in the
30. 
interval  2.  The positive value of k for which kex − x = 0 has only one root is
1
 5p 3p   p 5p  (A)  (B)  1
e
(A)   8 , 4  (B)   , 
  2 8  (C)  e (D)  loge2
[IIT−JEE 2007]
p p   p
(C)   4 , 2  (D)  0,  Solution: Let f ( x ) = ke x − x .Then
   4
[JEE MAIN 2016 (ONLINE SET−2)] f ′( x ) = ke x − 1
Substituting f ′( x ) = 0 ⇒ x = − log k , we get
Solution: We have
f ′′( x ) = ke x
f(x) = sin4x + cos4x = 1 − 2sin2x cos2x
f ′′( − log k ) = 1 > 0
1
f ( x ) = 1− sin2 2 x which implies that f ( x ) has one minima at point
2 x = −logk

Mathematical Problem Book for JEE.indb 918 07-06-2018 13:45:27


Chapter 21 | Applications of Derivatives 919

Since the equation has only one root, we get ( x − 1)n


f ( − log k ) = 0  5. Let g( x ) = ; 0 < x < 2, m and n are integers, m ≠
logcos m ( x − 1)
1
⇒ 1+ log k = 0 ⇒ k = 0, n > 0, and let p be the left hand derivative of |x − 3| at x = 1.
e
Hence, the correct answer is option (A). If lim+ g( x ) = p , then
x →1

 3. For k > 0, the set of all values of k for which kex − x = 0 has two (A)  n = 1, m = 1
distinct roots is (B)  n = 1, m = −1
 1 1  (C)  n = 2, m = 2
(A)   0,  (B)   ,1
 e e  (D)  n > 2, m = n
1  [IIT−JEE 2008]
(C)   , ∞  (D)  (0, 1)
e  Solution: Let f ( x ) = x − 1 . Then
[IIT−JEE 2007]
Solution: We have f(x) = kex − x. As discussed in the Solution of  x − 1, x ≥ 1
f (x) = 
Question 2, we can show that f(x) has a minima at x = −logk. There- 1− x , x < 1
fore, if f(x) has two distinct roots, then f(−logk) < 0. That is,  1, x ≥ 1
f ′( x ) = 
1  −1, x < 1
k<
e
Therefore, p = −1.
Hence,
( x − 1)n
 1 Now, g( x ) =
k ∈  0,  logcos m ( x − 1)
 e
Hence, the correct answer is option (A). hn 0 
lim+ g( x ) = lim  form 
x →1 h → 0 logcos h  0 
 4. The total number of local maxima and local minima of the
(2 + x )3 , −3 < x ≤ −1 nhn −1
function f ( x ) =  is = lim
2/3 h → 0 m ⋅ ( − tan h)
 x , −1 < x < 2
(A)  0 (B)  1 n ⋅ hn − 2
= lim
h→0  − tan h 
m⋅ 
(C)  2 (D)  3  h 
[IIT−JEE 2008]
n ⋅ hn − 2
Solution: See Fig. 21.23. We have = lim
h→0 ( − m)
(2 + x )3 , −3 < x ≤ −1
f (x) =  As lim+ g( x ) = −1. Therefore,
2/3 x →1
 x , −1 < x < 2
n.hn −2
y lim = −1
h→ 0 ( −m)
⇒ n = 2 and m = 2
Hence, the correct answer is option (C).
1
 p p
  6. Let the function g : ( −∞ , ∞ ) →  − ,  be given by
 2 2
X p
g(u ) = 2 tan−1(e u ) −. Then, g is
−2 −1 0 1 2 2
(A)  even and is strictly increasing in (0, ∞)
(B)  odd and is strictly decreasing in (−∞, ∞)
(C)  odd and is strictly increasing in (−∞, ∞)
(D)  neither even nor odd, but is strictly increasing in (−∞, ∞)
[IIT−JEE 2008]
Figure 21.23
Solution: We have
Clearly x = −1 is point of local maxima and x = 0 is a point of local p
minima. Therefore, g(u ) = 2 tan−1(e 4 ) −
2
Total no. of local maxima and minima = 2 p
−1 −4
g( −u ) = 2 tan (e ) −
Hence, the correct answer is option (C). 2
1 p
−1 
= 2 tan  4  −
e  2
p
= 2cot −1(e 4 ) −
2
Mathematical Problem Book for JEE.indb 919 07-06-2018 13:45:29
p
g(u ) = 2 tan−1(e 4 ) −
920 2
Mathematics Problem Book for JEE
p
g( −u ) = 2 tan−1(e −4 ) −
2
1 p
−1 
(D) 
f(x) is decreasing on (−1, 1) but has neither a local
= 2 tan  4  − maximum nor a local minimum at x = 1
e  2
[IIT−JEE 2008]
p
= 2cot −1(e 4 ) −
2 Solution: We have
2a( x 2 − 1)
p  p f ′( x ) =
= 2  − tan−1(e 4 ) − ( x 2 + ax + 1)2
2  2
p Clearly,
= − 2 tan−1(e 4 ) f (x) is Decreasing in (−1, 1)
2
f (x) is Increasing in ( −∞ , −1) ∪ (1, ∞ )
 p Therefore, x = 1 is point of minima.
= −  2 tan−1 e 4 − 
 2 Hence, the correct answer is option (A).
= − g(4) ex
f ″(t )
⇒ g(4) is an odd function
 9. Let g( x ) = ∫ 1+ t 2 dt which of the following is true?
0
Now, (A)  g′( x ) is positive on (−∞, 0) and negative on (0, ∞)
2e 4 (B)  g′( x ) is negative on (−∞, 0) and positive on (0, ∞)
g′(4) =
>0 (C)  g′( x ) changes sign on both (−∞, 0) and (0, ∞)
1+ e 24
Therefore, g(4) is strictly increasing function. (D)  g′( x ) does not change sign on (−∞, ∞)
[IIT−JEE 2008]
Hence, the correct answer is option (C).
Solution: We have
Paragraph for Questions 7−9: Consider the function f: (−∞, ∞) → ex
f ″(t )
x 2 − ax + 1
(−∞, ∞) defined by f ( x ) = 2 , 0 < a < 2.
g( x ) = ∫ 1+ t 2 dt
0
x + ax + 1
f ′( e x ) x
g ′( x ) = ⋅e
 7.  Which of the following is true? 1+ e 2 x
(A)  (2 + a)2 f ′′(1) + (2 − a)2 f ′′( −1) = 0 g′( x ) > 0 ⇒ x ∈ (0, ∞ )
(B)  (2 − a)2 f ′′(1) − (2 + a)2 f ′′( −1) = 0 g′( x ) < 0 ⇒ x ∈ ( −∞ ,0)

(C)  f ′(1)f ′( −1) = (2 − a)2 Hence, the correct answer is option (B).
10. Match the statements/expressions in Column I with the values
(D)  f ′(1)f ′( −1) = (2 − a)2
[IIT−JEE 2008] given in Column II.
Column I Column II
Solution: We have
2
x + 2x + 4 (p) 0
x 2 − ax + 1 (A) The minimum value of is
f (x) = 2 ,0 < a < 2 x +2
x + ax + 1
( x 2 + ax + 1)(2 x − a) − ( x 2 − ax + 1)(2 x + a) Let A and B be 3 × 3 matrices of real (q)  1
(B) 
f ′( x ) =
( x 2 + ax + 1)2 numbers, where A is symmetric, B is skew−
2a( x 2 − 1) symmetric, and (A + B)(A − B) = (A − B)(A
=
( x 2 + ax + 1)2 + B). If ( AB )t = ( −1)k AB , where ( AB )t is
the transpose of the matrix AB, then the
4 ax ( x 2 + ax + 1)2 − 4 ax ( x 2 − 1)(2 x + a)( x 2 + ax + 1)
f ″( x ) = possible values of k are
( x 2 + ax + 1)4
4a −4 a Let a = log3 log3 2. An integer k is satisfying (r) 2
(C) 
f ′(1) = , f ″( −1) = −a

(2 + a )2
(2 − a)2 1 < 2( − k + 3 )
< 2, must be less than
    
Therefore, (D) If sinq = cos φ , then the possible values of (s) 3
2 2
(2 + a) f ″(1) + (2 − a) f ″( −1) = 0 1 p
 q ± φ −  are
p 2
Hence, the correct answer is option (A).
 8.  Which of the following is true? [IIT−JEE 2008]

(A) 
f(x) is decreasing on (−1, 1) and has a local minimum at Solution:
x=1 (A) → ( r )
f(x) is increasing on (−1, 1) and has a local maximum at x = 1
(B)  2
(C) 
f(x) is increasing on (−1, 1) but has neither a local maximum Let y = x + 2 x + 4 . Then
x +2
nor a local minimum at x = 1

Mathematical Problem Book for JEE.indb 920 07-06-2018 13:45:30


Chapter 21 | Applications of Derivatives 921

xy + 2 y = x 2 + 2 x + 4 Column I Column II
2
x + (2 − y ) x + 2(2 − y ) = 0 (A) 
Interval contained in the domain of defini-  π π
tion of non−zero solutions of the differen- (p)   − 2 , 2 
As x is real, therefore,  
tial equation (x − 3)2y′ + y = 0
D≥0 (B) 
Interval containing the value of the inte-  π
2 5 (q)   0, 
(2 − y ) − 4 ⋅ 2(2 − y ) ≥ 0 gral ∫ ( x − 1)( x − 2)( x − 3)( x − 4)( x − 5)dx  2
y 2 + 4 y − 12 ≥ 0 1

y ≤ −6 or y ≥ 2
(C) 
Interval in which at least one of  π 5π 
Minimum value is 2. the points of local maximum of (r)   8 , 4 
 
(B) → (q , s) cos2 x + sin x lies
Interval in which tan−1(sin x + cos x) is in-
(D)   π
( A + B )( A − B ) = ( A − B )( A + B )
creasing (s)   0, 
⇒ AB = BA  8
As A is symmetric and B is skew-symmetric, so (t)  ( −π , π )
( AB )t = − AB [IIT−JEE 2009]
⇒ k = 1, and k = 3 Solution:
2 dy
(C) → ( r , s) (A)  ( x − 3) +y =0
dx
a = log3 log3 2 dx dy
∫ ( x − 3)2 = −∫ y
⇒3 −a
= 3− log3 (log3 2) = log2 3
1
Now, ⇒ = ln y + c
x −3
1 < 2− k +log2 3 < 2
so domain is R − {3}.
1< 3 ⋅ 2− k < 2 (B) Put x = t + 3
 3 2 2
⇒ log2   < k < log2 3
∫ (t + 2)(t + 1)t (t − 1)(t − 2)dt = ∫ t (t
2
 2 − 1)(t 2 − 4)dt = 0 (being odd
−2 −2
⇒ k = 1 or k < 2 and k < 3
function)
(D) → ( p , r ) 2
5  1
(C)  f ( x ) = −  sin x − 
We have 4  2
sinθ = cos φ 1
Maximum value occurs when sin x =
2
π  (D)  f ′(x) > 0 if cos x > sin x.
⇒ cos  − θ  = cos φ
2 
Hence, the correct matches are (A)−(p, q, s); (B)−(p, t, s); (C)−(p,
π q, r, t); (D)−(s).
⇒ − θ = 2nπ ± φ
2 1
12.  For function f ( x ) = x cos , x ≥ 1,
π x
⇒ −2nπ = θ ± φ − (A)  for atleast one x in interval [1, ∞), f(x + 2) − f(x) < 2
2
1 π (B)  lim f ′( x ) = 1
x →∞
⇒ −2n =  θ ± φ − 
π 2 (C)  for all x in the interval [1, ∞), f(x + 2) − f(x) > 2
(D)  f ′(x) is strictly decreasing in the interval [1, ∞)
1 π [IIT−JEE 2009]
Therefore,  θ ± φ −  is even number.
π 2 1
Solution: For f ( x ) = x cos   , x ≥ 1
1 π x
Hence,  θ ± φ −  = 0,2
π 2  1 1  1
f ′( x ) = cos   + sin   → 1 for x → ∞
x x x
Hence, the correct matches are (A)−(r); (B)−(q, s); (C)−(r, s);
Also  1 1  1 1  1 1  1
(D)−(p, r). f ′( x ) =   + sin   − 2 sin   − 3 cos  
x x x x x x x
11. 
Match the statements/expressions in Column I with the open 1  1
intervals in Column II. =− cos   < 0 for x ≥ 1
x3 x

Chapter 21.indd 921 12-06-2018 19:06:51


922 Mathematics Problem Book for JEE

⇒ f ′(x) is decreasing for [1, ∞ ) 1


a=b=c =e+ ⇒a=b=c
⇒ f ′( x + 2) < f ′( x ). e
1 1 Hence, the correct answer is option (D).
Also, lim f ( x + 2) − f ( x ) = lim ( x + 2)cos − x cos  = 2
x →∞ x →∞  x +2 x 16. Let f be a function defined on R (the set of all real numbers)
∴ f ( x + 2) − f ( x ) > 2∀x ≥ 1 such that f ′(x) = 2010 (x − 2009) (x − 2010)2 (x − 2011)3
Hence, the correct answers are options (B), (C) and (D). (x − 2012)4, for all x ∈ R. If g is a function defined on R with
values in the interval (0, ∞) such that f(x) = ln (g(x)), for all
The maximum value of the function f(x) = 2x3 − 15 x2 + 36x − 48
13.  x ∈ R , then the number of points in R at which g has a local
{
on the set A = x x 2 + 20 ≤ 9 x is _____. } [IIT−JEE 2009]
maximum is
[IIT−JEE 2010]

Solution: Solution:
f ′(x) = 6(x − 2)(x − 3) f ( x ) = ln{ g( x )}
So, f (x) is increasing in (3, ∞). g( x ) = e f ( x )
Also A = {4 ≤ x ≤ 5}. Therefore, g′( x ) = e f ( x ) ⋅ f ′( x )
fmax = f (5) = 7 g′( x ) = 0 ⇒ f ′( x ) = 0 as e f ( x ) ≠ 0
Hence, the correct answer is (7). ⇒ 2010( x − 2009)( x − 2010)2 ( x − 2011)3 ( x − 2012)4 = 0
14. Let p(x) be a polynomial of degree 4 having extremum at x = 1, So, there is only one point of local maxima.
p( x ) 
2 and lim  1+  = 2. Then the value of p(2) is _____.
Hence, the correct answer is (1).
x →0  x2 
Let f : R → R be defined as f ( x ) = x + x 2 − 1 . The total number
17. 
[IIT−JEE 2009]
of points at which f attains either a local maximum or a local
Solution: Let P( x ) = ax 4 + bx 3 + cx 2 + dx + e . Then minimum is _____.
P′(1) = P′(2) = 0 [IIT−JEE 2012]
 x 2 + p( x )  Solution:
lim   = 2
x →0
 x2 
x x2 −1
⇒ p(0) = 0 ⇒ e = 0 f ′( x ) = + 2 ⋅ (2 x )
x x −1
 2 x + p '( x )  2 x − 1, x < −1
lim 
x →0 
 = 2  −(2 x + 1),
2x  − 1< x < 0
=
⇒ p '(0) = 0 ⇒ d = 0 1− 2 x , 0 < x <1
 2 + p "( x )  2 x + 1, x >1
lim  =2
x →0  2 
−ve +ve −ve +ve −ve +ve
⇒ c =1
On solving, a = 1/4, b = − 1
So, −1 −1/2 0 −1/2 1
4
x So, f ′(x) changes sign at points
p( x ) = − x3 + x2
4 1 1
x = −1, − ,0, ,1
⇒ p(2) = 0 2 2
Hence, the correct answer is (0). So, total number of points of local maximum or minimum is 5.
Hence, the correct answer is (5).
15. Let f, g and h be real−valued 2
functions
2
defined
2
on2 the
interval [0, 1] by f ( x ) = e x + e − x , g( x ) = xe x + e − x and 18. 
Let p(x) be a real polynomial of least degree which has a local
2 2
h( x ) = x 2e x + e − x . If a, b and c denote, respectively, the maximum at x = 1 and a local minimum at x = 3. If p(1) = 6 and
absolute maximum of f, g and h on [0, 1], then p(3) = 2, then p′(0) is _____.
(A)  a = b and c ≠ b (B)  a = c and a ≠ b [IIT−JEE 2012]
(C)  a ≠ b and c ≠ b (D)  a = b = c Solution: Let p′( x ) = k ( x − 1)( x − 3) . Then
[IIT−JEE 2010]
 x3 
Solution: p( x ) = k  − 2 x 2 + 3 x  + c
2 2 2 2
f ( x ) = e x + e − x ⇒ f ′( x ) = 2 x (e x − e − x ) ≥ 0 ∀ x ∈ [0,1]  3 

Clearly for 0 ≤ x ≤ 1, f ( x ) ≥ g( x ) ≥ h( x ) Now,


1 4
As f (1) = g(1) = h(1) = e + and f (1) is the greatest. Therefore, p(1) = 6 ⇒ k + c = 6
e 3

Chapter 21.indd 922 10-06-2018 18:19:24


Chapter 21 | Applications of Derivatives 923

also, (D)  g is decreasing on (1, 2) and increasing on (2, ∞)


p(3) = 2 ⇒ c = 2 Solution:
So, k = 3, and  2( x − 1) 
g ′( x ) =  − ln x  f ( x )
p′(0) = 3k = 9  x +1 
Hence, the correct answer is (9). For x ∈(1, ∞ ), f ( x ) > 0

Paragraph for Questions 19 and 20: Let f ( x ) = (1− x )2 sin2 x + x 2  2( x − 1) 


Let h( x ) =  − ln x  . Then
x  x +1 
 2(t − 1) 
for all x ∈R , and let g( x ) = ∫  − 1n t  f (t ) dt for all x ∈ (1, ∞ ).
1  t + 1   4 1  −( x − 1)2
h′ ( x ) =  − = <0
[IIT−JEE 2012]  ( x + 1)2
x  ( x + 1)2 x
19.  Consider the statements:
Also h(1) = 0 so, h(x) < 0 ∀x > 1
P:  There exists some x ∈R , such that f ( x ) + 2 x = 2(1+ x 2 )
Q:  There exists some x ∈R , such that 2f ( x ) + 1 = 2 x (1+ x ) Therefore, g(x) is decreasing on (1, ∞).
Then Hence, the correct answer is option (B).
(A)  both P and Q are true x
21.  If f ( x ) = ∫ e t (t − 2)(t − 3) dt for all x ∈ (0, ∞ ), then
2

(B)  P is true and Q is false 0

(C)  P is false and Q is true (A)  f  has a local maximum at x = 2


(D)  both P and Q are false. (B)  f  is decreasing on (2, 3)


(C)  there exists some c ∈ (0, ∞ )such that f ′′(c ) = 0


Solution:
(D)  f  has a local minimum at x = 3
f ( x ) = (1− x )2 sin2 x + x 2 ∀ x ∈ R

x
 2(t − 1)  0 + 1 − 3 +
g( x ) = ∫  − ln t  f (t ) dt ∀ x ∈ (1, ∞ )
1
 t + 1  [IIT−JEE 2012]
For statement P: Solution:
f ( x ) + 2 x = 2(1+ x )  (1)
2
2
f ′( x ) = e x ( x − 2)( x − 3)

(1− x )2 sin2 x + x 2 + 2 x = 2 + 2 x 2 Clearly, maxima at x = 2, minima at x = 3 and decreasing in x ∈ (2,3).



f ′( x ) = 0 for x = 2 and x = 3 (Rolle’s theorem)
(1− x )2 sin2 x = x 2 − 2 x + 2 = ( x − 1)2 + 1
So, there exist c ∈ (2,3) for which f ′′(c ) = 0.
(1− x )2 (sin2 x − 1) = 1
Hence, the correct answers are options (A), (B), (C) and (D).
−(1− x )2 cos2 x = 1
22. A rectangular sheet of fixed perimeter with sides having their
2. 2
(1− x ) cos x = −1 lengths in the ratio 8 : 15 is converted into an open rectangular

box by folding after removing squares of equal area from all
So, equation (1) will not have real solution. four corners. If the total area of removed squares is 100, the
Therefore, P is wrong. resulting box has maximum volume. Then the lengths of the
For statement Q: sides of the rectangular sheet are
(A)  24 (B)  32
2(1− x )2 sin2 x + 2 x 2 + 1 = 2 x + 2 x 2  (2)
(C)  45 (D)  60
2(1− x )2 sin2 x = 2 x − 1
[JEE ADVANCED 2013]
2x −1 2x −1
2sin2 x = ; Let h( x ) = − 2sin2 x Solution: We have
(1− x )2 (1− x )2
V = (8λ − 2 x )(15λ − 2 x ) x
Clearly, h(0) = − ve, lim− h( x ) = +∞
x →1 = 4 x 3 − 46λ x 2 + 120λ 2 x
So, by IVT, Eq. (2) will have solution.
Differentiating with respect to x, we get
Therefore, Q is correct.
dV
Hence, the correct answer is option (C). = 12 x 2 − 92λ x + 120 λ 2 = 0 at x = 5
dx
20.  Which of the following is true?
(A)  g is increasing on (1, ∞) ⇒ 60 λ 2 − 230 λ + 150 = 0

(B)  g is decreasing on (1, ∞) ⇒ 6 λ 2 − 23λ + 15 = 0


(C)  g is increasing on (1, 2) and decreasing on (2, ∞) ⇒ (6 λ − 5)( λ − 3) = 0

Mathematical Problem Book for JEE.indb 923 07-06-2018 13:45:36


924 Mathematics Problem Book for JEE

For l = 3, the lengths of sides (as shown in Fig. 21.24) are obtained 1
as 45, 24. (C)  − 4 < f ( x ) < 1 (D)  −∞ < f ( x ) < 0
[JEE ADVANCED 2013]
x 15l
Solution: We have
d2 y dy
− 2 + y ≥ ex
dx 2 dx
2
d y dy
8l
⇒ e − x 2 − 2e − x + e− x y ≥ 1
dx dx
d2
⇒ 2 ( ye − x ) ≥ 1
dx (1)

Figure 21.24 ye −x
Hence, the correct answers are options (A) and (C).

0 1
The function f ( x ) = 2 x + x + 2 – x + 2 – 2 x has a local min-
23. 
imum or a local maximum at x equals
−2
(A)  −2 (B) 
3
2 Figure 21.26
(C)  2 (D)  −x
3 From Eq. (1) and Fig. 21.26, ye is concave up. Hence, −∞ < f(x) < 0.
[JEE ADVANCED 2013] Hence, the correct answer is option (D).

Solution: We have If the function e−x f(x) assumes its minimum in the interval
25. 
xx <
<−−2,
2, ff (( xx )) =
=− 2 xx −
−2 −44 1
[0, 1] at x = , which of the following is true?
2 4
−2
−2≤ ≤ xx <
<−− 2 ,, =2
ff (( xx )) = +4
2 xx + 4 1 3 1
3
3 (A)  f ′( x ) < f ( x ), < x < (B)  f ′( x ) > f ( x ),0 < x <
2 4 4 4
− 2
−3≤ ≤ xx ≤ 0,
≤ 0, ff (( xx )) =
=− 4 xx
−4 1 3
3 (C)  f ′( x ) < f ( x ),0 < x < (D)  f ′( x ) < f ( x ), < x < 1
≤ xx <<2 =4 4 4
0≤
0 2 ff (( xx )) = 4 xx
[JEE ADVANCED 2013]
xx ≥ 2,
≥ 2, ff (( xx )) == 2 xx +
2 +44
d
Solution: See Fig. 21.27. We know that ( ye − x ) is an increasing
It is clear from Fig. 21.25 that x = −2 and x = 0 are the points of min- dx
2 function. Therefore,
ima. Therefore, x = − is point of maxima.
3
1 1
0< x < x>
4 4
d d
( ye − x ) < 0 ( ye − x ) > 0
dx dx
dy − x dy − x
e− x − e y < 0 e− x −e y >0
dx dx
8/3 dy dy
<y >y
dx dx
f ′( x ) < f ( x ) f ′( x ) > f ( x )
−2 2 2

3

Figure 21.25
Hence, the correct answers are options (A) and (B).
1/4
Paragraph for Questions 24 and 25: Let f: [0, 1] → R (the set of O 1
all real numbers) be a function. Suppose the function f is twice
differentiable, f(0) = f(1) = 0 and satisfies f″(x) − 2f ′(x) + f(x) ≥ ex,
x ∈ [0, 1].

24.  Which of the following is true for 0 < x < 1?


1 1 Figure 21.27
(A)  0 < f ( x ) < ∞ (B)  − 2 < f ( x ) < 2 Hence, the correct answer is option (C).

Mathematical Problem Book for JEE.indb 924 07-06-2018 13:45:38


Chapter 21 | Applications of Derivatives 925

æ 1ö
x -ç t + t ÷ dt Then ìmax {f ( x ), g( x )} if x £ 0,
26.  Let f : (0, ¥ ) ® R be given by f ( x ) = 1 e h( x ) = í
ò è ø
t
.
îmin {f ( x ), g( x )} if x > 0.
x
(A)  f(x) is monotonically increasing on [1, ∞)
(B)  f(x) is monotonically decreasing on (0, 1) |x | + 1
æ 1ö
(C)  f ( x ) + f ç ÷ = 0, for all x ∈ (0, ∞)
èxø (0, 2)
(D)  f(2x) is an odd function of x on R Sharp edge Sharp edge
1×1
[JEE ADVANCED 2014]
1
Solution:
x
æ 1ö
-ç t + ÷
x e è tø
f (x) = ò dt (−1, 0) (1, 0)
1
x t
 1 1 
− x +  − + x
d e  x
d e x 
d  1 Figure 21.28
⇒ f (x) = x− ×  
dx x dx 1 dx  x  Points of Intersection are, (1, 2) and (−1, 2). [See Fig. 21.28.]
x h(x) = x2 + 1, in (− ∞,−1)
 1
− x + 
x2 + 1= |x| + 1 at −1
 x  1
e − x + 
 x  1 |x| + 1, in (−1, 0)
= + xe ×− 2
x  x  |x| + 1 = x2 + 1 at 0
 1 = x2 + 1, in (0, 1)
− x +  1
e  x
1 − x + 
  x2 + 1 = |x| + 1 at 1
= + e  x = |x| + 1, in (1, ∞)
x x
 1
At sharp edges i.e. at −1, 0 and 1, there is no smooth turn, so no
− x + 
 x derivative exits there. Elsewhere, function is continuous and differ-
2e
= >0 entiable. Hence, there are three such points.
x Hence, the correct answer is (3).
Therefore, f(x) is strictly increasing in (0, ∞).  (1)
28. 
A cylindrical container is to be made from certain solid
æ 1ö æ 1ö
-ç t + ÷
1
-ç t + ÷ material with the following constraints: It has a fixed inner
1 x e è tø e è tø
Now   f ( x ) + f æç ö÷ = ò 1 dt +ò x dt volume of V mm3, has a 2 mm thick solid wall and is open at
èxø x t x t the top. The bottom of the container is a solid circular disc of
æ 1ö
-ç t + ÷
thickness 2 mm and is of radius equal to the outer radius of
1
e è tø the container.
   = ò x
1 dt = 0 (2) If the volume of the material used to make the container is
t
x minimum when the inner radius of the container is 10 mm,
æ 1ö V
Now f (2 x ) + f ç x ÷ = f (2 x ) + f (2- x ) = 0 then the value of is _____.
è2 ø 250p
Therefore, f(2x) is an odd function.  (3) [JEE ADVANCED 2015]
Note: Let 2x = m as log2m = x. For m ∈ (0, ∞), x ∈ (−∞, ∞) Solution:
We can say f(2x) = h(x) an odd function. Then h(−x) = −h(x).
­Therefore, from Eqs. (1), (2) and (3), we can conclude that the
r
correct options are (A), (C) and (D).
Hence, the correct answers are options (A), (C) and (D).
Let f : R → R and g : R → R be respectively given by f(x) = |x|
27. 
+ 1 and g(x) = x2 + 1. Define h : R ® R by
ìmax {f ( x ), g( x )} if x £ 0, V h
h( x ) = í 2 mm
îmin {f ( x ), g( x )} if x > 0.
The number of points at which h(x) is not differentiable is _____.
[JEE ADVANCED 2014]

Solution: 2 mm
f :R®R g:R®R
(x) = |x| +1
f  g(x) = x2 + 1
h:R®R Figure 21.29

Chapter 21.indd 925 10-06-2018 18:22:47


926 Mathematics Problem Book for JEE

Vm = Volume of material used is minimum when r = 10 mm, é x4 1 ù é x4 1 ù


æ 1ö
Þ 64 ê - ú £ f ( x ) - f ç ÷ £ 96 ê - ú
V êë 4 64 úû è2ø êë 4 64 úû
=?
250p 1 1
1
é16 x 5 ù é 24 x 5 3 ù
Here V is constant but r is variable and h is variable such that Þ ê - x ú £ ò f ( x ) dx £ ê - xú
ëê 5 úû1/2 1/2 ëê 5 2 ûú1/2
V = p r2h(1)
1
Volume of material used is 26 39
Þ ≤ ∫ f ( x ) dx ≤
(r + 2)2 h -
Vm = p r 2h
p r + 2)2 (2)
+ ( 10 1/2 10
volume of volume of Volume of
outer cylinder inner cylinder Base disc é x4 1 ù é x4 1 ù
without base without base ⇒ 64 ê - ú £ f ( x ) £ 96 ê - ú
ëê 4 64 ûú ëê 4 64 ûú
= 4p h + 4rp h + 2p r2 + 8p r + 8p
1 1 1
4π v 4 rπ .v  3
∫ (16 x ∫ f ( x )dx ≤ ∫  24 x
4 4
Vm = + + 2π r 2 + 8π 2 + 8π ⇒ − 1)dx ≤ −  dx
π r2 π r2 2
1/2 1/2 1/2

æ -2 ö æ -1 ö æ 26 39 ö
Now, Vm = 4V ç 3 ÷ + 4V ç 2 ÷ + 4p r + 8p = 0 Clearly, ç , ÷ Ì (1,12)
dr èr ø èr ø è 10 10 ø
r =10
Hence, the correct answer is option (D).
8V 4V
Þ - - + 40p + 8p = 0
1000 100 Let f : R ® (0, ¥ ) and g: R → R be twice differentiable function
30. 
48V such that f ¢¢ and g′′ are continuous functions on R. Suppose
Þ − = −48π
1000 f ( x )g( x )
f ¢(2) = g(2) = 0, f ′′(2) ≠ g′(2) ≠ 0. If lim = 1, then
x ® 2 f ¢ ( x )g ¢ ( x )
V
⇒ =4
250p (A)  f has a local minimum at x = 2.
Hence, the correct answer is (4). (B)  f has a local maximum at x = 2.
(C)  f ¢¢(2) > f (2).
192 x 3 æ 1ö
29. 
Let f ′( x ) = for all x ∈R with f ç ÷ = 0. If (D)  f ( x ) - f ¢¢( x ) = 0 for at least one x ÎR.
2 + sin4 π x è2ø
1 [JEE ADVANCED 2016]
m£ ò f ( x )dx £ M , then the possible values of m and M are Solution: Let f : R ® (0, ¥ ) and g: R → R
1/2
(A)  m = 13, M = 24 f(x) > 0 "x ÎR
1 1
(B)  m = , M = It is given that f ¢(2) = 0, g(2) = 0, f ¢¢(2) ¹ 0 and g¢(2) ¹ 0.
4 2
It is also given that
(C)  m = −11, M = 0
(D)  m = 1, M = 12 f ( x )g( x ) æ0 ö
lim =1 ç form ÷
x ®2 f ¢( x )g¢( x ) è0 ø
[JEE ADVANCED 2015]
Solution: Applying L’Hospital rule, we get
f ′( x )g( x ) + g′( x )f ( x )
192 x 3 lim =1
f ¢( x ) = "x ÎR , x →2 f ′′( x )g′( x ) + f ′( x )g′′( x )
2 + sin4 π x
For finite limit, we get
1 1
f ¢(2)g(2) + g¢(2)f (2)
Here ò f ( x ) = ò (ò f ¢( x )dx ) (1) f ¢¢(2)g¢(2) + f ¢(2)g¢¢(2)
=1
1/2 1/2
3 g¢(2)f (2)
192 x 192 x 3 192 x 3 =1
Here, ≤ ≤ f ¢¢(2)g¢(2)
3 2 + sin4 p x 2
x x x f (2)
192 192 x 3 192 = 1 ⇒ f ¢¢(2) = f (2) > 0 and f ¢(2) = 0
∫ ∫ ∫ x dx
3 3
Þ x dx ≤ 4
dx ≤ f ′′(2)
3 1/2 1/2 2 + sin p x 2 1/2
which means that f(x) has local minima at x = 2.
x
192 æ x 4 1 ö 192 æ x 4 1 ö Hence, the correct answer is option (A).
Þ çç - ÷÷ £ ò f ¢( x )dx £ ç - ÷
3 è 4 64 ø 1/2 2 çè 4 64 ÷ø
f (2) - f ¢¢(2) = 0

Chapter 21.indd 926 10-06-2018 18:23:26


Chapter 21 | Applications of Derivatives 927

Therefore, we can say that f ( x ) - f ¢¢( x ) = 0 has at least one solution 10.  The angle between curves y 2 = 4 x and x 2 + y 2 = 5 at (1, 2) is
in x ÎR. p
Hence, option (D) is correct. (A)  tan-1(3) (B)  tan−1(2) (C)  (D)  p / 4
2
Hence, the correct answers are options (A) and (D).
For the curve by 2 = ( x + a)3 , the square of the subtangent is
11. 
proportional to
Practice Exercise 1 1/2
(A)  (Subnormal) (B)  Subnormal
3/2
The points on the curve y = 12x − x3 at which the gradient is
 1.  (C)  (Subnormal) (D)  None of these
zero are 2
The tangent to the curve y = ax + bx at (2, − 8) is parallel to
12. 
(A)  (0, 2), (2, 16) (B)  (0, −2), (2, −16) x−axis. Then
(C)  (2, −16), (−2, 16) (D)  (2, 16), (−2, −16) (A)  a = 2, b = -2 (B)  a = 2, b = -4
 2. 
The area of the triangle formed by the coordinate axes and a (C)  a = 2, b = -8 (D)  a = 4, b = -4
2
tangent to the curve xy = a at the point ( x1, y1) on it is 13. 
The sum of intercepts on the coordinate axes made by the
2
a x1 2
a y1 tangent to the curve x + y = a is
(A)  (B)  (C)  2a2 (D)  4a2
y1 x1 (A)  a (B)  2a (C)  2 a  (D)  None of these
2 2
 3.  The slope of tangent to the curve x = t + 3t − 8, y = 2t - 214. 
t - 5 The coordinates of a point on the curve y = x log x at which
y = 2t 2 - 2t - 5 at the point (2, −1) is the normal is parallel to the line 2 x - 2 y = 3 are
(A)  22 / 7 (B)  6 / 7 (C)  − 6 (D)  None of these (A)  (0, 0) (B)  (e , e )
−2 −2
The point of the curve y 2 = 2( x - 3) at which the normal is
 4.  (C)  (e 2 , 2e 2 ) (D)  (e , −2e )
parallel to the line y - 2 x + 1 = 0 is If normal to the curve y = f ( x ) is parallel to x−axis, then the
15. 
correct statement is
æ 1 ö
(A)  (5, 2) (B)  ç - , -2 ÷ dy dy
è 2 ø (A)  = 0 (B)  =1
dx dx
æ3 ö
(C)  (5, −2) (D)  ç ,2 ÷ dx
è2 ø (C)  = 0 (D)  None of these
dy
The line x + y = 2 is tangent to the curve x 2 = 3 - 2 y at its
 5.  The length of normal to the curve x = a (q + sinq ),
16. 
point y = a(1- cosq ) at the point q = p / 2 is
(A)  (1, 1) (B)  (−1, 1)
(A)  2a (B)  a / 2  
(C)  2 a (D)  a / 2
(C)  ( 3 , 0) (D)  (3, −3)
17. 
The normal of the curve x = a(cos q + q sinq ),
2
 6. If x = t and y = 2t, then the equation of the normal at t = 1 is y = a(sinq - q cosq ) at any q is such that
(A)  x + y - 3 = 0 (B)  x + y - 1 = 0 (A)  It makes a constant angle with x−axis
(C)  x + y + 1 = 0 (D)  x + y + 3 = 0 (B)  It passes through the origin
 7. 
The equation of the normal to the curve y = sin(p x/2) at the (C)  It is at a constant distance from the origin
point (1, 1) is (D)  None of these
(A)  y = 1 (B)  x = 1 The slope of the tangent to the curve x = 3t 2 + 1, y = t 3 − 1 at
18. 
-2 x = 1 is
(C)  y = x (D)  y - 1 = p ( x - 1)
(A)  0 (B)  1/ 2 (C)  ∞ (D)  −2
The equation of tangent to the curve y = 2cos x at x = p /4 is
 8.  An equation of the tangent to the curve y = x from the point
19.  4

 p æ pö (2, 0) not on the curve is


(A)  y − 2 = 2 2  x −  (B)  y + 2 = 2 ç x + ÷
 4 è 4ø (A)  y = 0 (B)  x = 0
 p æ pö (C)  x + y = 0 (D)  None of these
(C)  y − 2 = − 2  x −  (D)  y - 2 = 2 ç x - ÷
 4 è 4ø 2
The angle of intersection of the curves y = x 2 and x = y at
20. 
x y (1, 1) is
 9. At which point the line + = 1, touches the curve y = be − x / a
a b 4
(A)  (0, 0) (B)  (0, a) (A)  tan−1   (B)  tan−1(1)
3
(C)  (0, b) (D)  (b, 0) -1 æ 3 ö
(C)  90° (D)  tan ç ÷
è4ø

Mathematical Problem Book for JEE.indb 927 07-06-2018 13:45:47


928 Mathematics Problem Book for JEE

21. 
The abscissae of the points, where the tangent to curve −1 −1
(A)  tan (2 2 ) (B)  tan (3 2 )
y = x 3 − 3 x 2 − 9 x + 5 is parallel to x−axis, are
(C)  tan−1(3 3) (D)  tan−1(5 2 )
(A)  0 and 0 (B)  x = 1 and −1
If the normal to the curve y 2 = 5 x − 1, at the point (1, −2) is of
34. 
(C)  x = 1 and −3 (D)  x = −1 and 3
the form ax − 5 y + b = 0, then a and b are
If the curve y = a x and y = b x intersect at angle a , find the
22.  (A)  4, − 14   (B)  4, 14   (C)  −4, 14   (D)  −4, −14
value of tanα . If a tangent to the curve y = 6 x − x 2 is parallel to the line
35. 
4 x − 2 y − 1 = 0, then the point of tangency on the curve is
a−b log a − log b
(A)  (B)  1+ log a log b (A)  (2, 8)    (B)  (8, 2)    (C)  (6, 1)    (D)  (4, 2)
1+ ab
The normal to the curves x = a (1+ cosθ ) and y = a sinθ at
36. 
a+b log a + log b angle θ always passes through the fixed point
(C)  (D)  1− log a log b
1− ab (A)  (a, a)    (B)  (0, a)    (C)  (0, 0)    (D)  (a, 0)
23.  The equation of tangent at ( −4, − 4) on the curve x 2 = −4 y is 37. 
If ST and SN are the lengths of the subtangent and
the subnormal at the point θ = π /2 on the curve
(A)  2 x + y + 4 = 0 (B)  2 x − y − 12 = 0
x = a(θ + sinθ ), y = a(1− cosθ ), where a ≠ 1 , then
(C)  2 x + y − 4 = 0 (D)  2 x − y + 4 = 0
(A)  ST = SN (B)  ST = 2 SN
The point at which the tangent to the curve y = 2 x 2 − x + 1 is
24.  2 3 3
parallel to y = 3x + 9 will be (C)  ST = a SN (D)  ST = a SN
(A)  (2, 1) (B)  (1, 2) (C)  (3, 9) (D)  (−2, 1) The equation of the tangent to the curves x = 2cos3 θ and
38. 
y = 3sin3 θ at the point θ = π /4 is
At what point on the curve x 3 − 8a2 y = 0, the slope of the
25. 
normal is −2 / 3? (A)  2 x + 3 y = 3 2 (B)  2 x − 3 y = 3 2
(A)  (a, a) (B)  (2a, − a) (C)  3 x + 2 y = 3 2 (D)  3 x − 2 y = 3 2
(C)  (2a, a) (D)  None of these The curve given by x + y = e
39.  xy
has a tangent parallel to the
The length of the normal at point t of the curve x = a(t + sin t ),
26.  y-axis at the point
y = a(1− cos t ), is (A)  (0, 1)    (B)  (1, 0)   (C)  (1, 1)    (D)  (−1, −1)
(A)  a sin t (B)  2a sin3 (t /2)sec(t /2) 40. 
For which of the following intervals, the given function
(C)  2a sin(t /2) tan(t /2) (D)  2a sin(t /2) f ( x ) = −2 x 3 − 9 x 2 − 12 x + 1 is decreasing?
The tangent drawn at the point (0, 1) on the curve y = e 2 x
27.  (A)  ( −2, ∞ ) (B)  ( −2, − 1)
meets x−axis at the point (C)  ( −∞ , − 1) (D)  ( −∞ , − 2) and ( −1, ∞ )
(A)  (1/ 2,0)   (B)  ( −1/ 2, 0)   (C)  (2, 0)    (D)  (0, 0) 3
41.  f ( x ) = x − 27 x + 5 is an increasing function, when
(B)  | x | > 3
2
The equation of the tangent to the curve (1+ x ) y = 2 − x ,
28.  (A)  x < −3
where it crosses the x−axis, is
(C)  x ≤ −3 (D)  | x | < 3
(A)  x + 5 y = 2 (B)  x − 5 y = 2 x
42.  If f ( x ) = sin x −
is increasing function, then
(C)  5 x − y = 2 (D)  5 x + y − 2 = 0 2
π π
The equation of the tangent to curve y = be − x / a at the point
29.  (A)  0 < x < (B)  − 3 < x < 0
3
where it crosses y−axis is π π π
(C)  − < x < (D)  x =
(A)  ax + by = 1 (B)  ax − by = 1 3 3 2
x y x y If x tends 0 to p, then the given function f ( x ) = x sin x + cos x
43. 
(C)  − = 1 (D)  + =1
a b a b + cos2 x is
(A)  Increasing
The angle of intersection of curves y = x 2 and 6 y = 7 − x 3 at
30. 
(B)  Decreasing
(1, 1) is
(C)  Neither increasing nor decreasing
(A)  p / 4    (B)  p / 3    (C)  p / 2    (D)  p (D)  None of these
The tangent to the curve y = 2 x 2 − x + 1 at point P is parallel
31.  Let y = x 2e − x . Then the interval in which y increases with
44. 
to y = 3 x + 4, the coordinates of point P are respect to x is
(B)  (1, 2)          (C)  (− 1, 2)     (D)  (2, − 1)
(A)  (2, 1)      (A)  ( −∞ , ∞ )   (B)  ( −2,0)   (C)  (2, ∞ )    (D)  (0,2)
2
32.  For the curve xy = c , the subnormal at any point varies as The function y = 2 x 3 − 9 x 2 + 12 x − 6
45.  is monotonically
2 3 2 3
decreasing, when
(A)  x     (B)  x          (C)  y       
(D)  y
(A)  1 < x < 2 (B)  x > 2
33.  The angle between the curves y = sin x and y = cos x is (C)  x < 1 (D)  None of these

Chapter 21.indd 928 10-06-2018 18:23:58


Chapter 21 | Applications of Derivatives 929

For which value of x, the function f ( x ) = x 2 − 2 x is decreasing?


46.  57.  The function f defined by f ( x ) = ( x + 2)e − x is
(A)  x > 1   (B)  x > 2 (C)  x < 1  (D)  x < 2 (A)  Decreasing for all x

The function f ( x ) = cos x − 2 px is monotonically decreasing


47.  (B)  Decreasing in ( −∞ , − 1) and increasing in ( −1, ∞ )
for (C)  Increasing for all x
1 1 (D)  Decreasing in ( −1, ∞ ) and increasing in ( −∞ , − 1)
(A)  p < (B)  p > (C)  p < 2  (D)  p > 2
2   2
58.  If f ( x ) = x 3 − 10 x 2 + 200 x − 10, then
If f ( x ) = kx 3 − 9 x 2 + 9 x + 3 is monotonically increasing in each
48. 
interval, then (A)  f ( x ) is decreasing in [ −∞ ,10] and increasing in [10, ∞]
(A)  k < 3 (B)  k ≤ 3 (B)  f ( x ) is increasing in [ −∞ ,10] and decreasing in [10, ∞]
(C)  k > 3 (D)  None of these (C)  f ( x ) is increasing throughout real line
I n which interval is the given function f ( x ) = 2 x 3 − 15 x 2
49.  (D)  f ( x ) is decreasing throughout real line
+ 36 x + 1 is monotonically decreasing? x x
If f ( x ) =
59.  and g( x ) = , where 0 < x ≤ 1, then in this
(A)  [2, 3]   (B)  (2, 3) (C)  ( −∞ ,2)  (D)  (3, ∞ ) sin x tan x
50.  The function f ( x ) = tan x − x interval
(A)  Always increases (A)  Both f ( x ) and g( x ) are increasing functions
(B)  Always decreases (B)  Both f ( x ) and g( x ) are decreasing functions
(C)  Never decreases (C)  f ( x ) is an increasing function
(D)  Sometimes increases and sometimes decreases (D)  g( x ) is an increasing function
2x
51.  The function f ( x ) = log(1+ x ) − is increasing on Function f ( x ) = 2 x 3 − 9 x 2 + 12 x + 29 is monotonically decreas­
60. 
2+ x
(A)  (0, ∞ ) (B)  ( −∞ , 0) ing, when
(C)  ( −∞ , ∞ ) (D)  None of these (A)  x < 2 (B)  x > 2 (C)  x >1 (D)  1< x < 2
3 2
The values of a, for which the function (a + 2) x − 3ax + 9ax − 1
52.  61.  2 x 3 + 18 x 2 − 96 x + 45 = 0 is an increasing function when
decreases monotonically throughout for all real x, are
(A)  x ≤ −8, x ≥ 2 (B)  x < −2, x ≥ 8
(A)  a < −2 (B)  a > −2
(C)  −3 < a < 0 (D)  −∞ < a ≤ −3 (C)  x ≤ −2, x ≥ 8 (D)  0 ≤ x ≤ −2

53.  If f ( x ) = 2 x + cot −1 x + log( 1+ x 2 − x ), then f ( x ) a sin x + b cos x


62.  The function is decreasing if
(A)  Increases in [0 , ∞ ) c sin x + d cos x
(B)  Decreases in [0 , ∞ ) (A)  ad − bc > 0 (B)  ad − bc < 0
(C)  Neither increases nor decreases in (0, ∞ ) (C)  ab − cd > 0 (D)  ab − cd < 0
(D)  Increases in (− ∞ , ∞ ) 2

54.  For all real values of x, the increasing function f(x) is 63.  The function f ( x ) = 1− e − x /2
is
4 (A)  Decreasing for all x
(A)  x −1  
(B)  x 2 (C)  x 3    (D)  x
(B)  Increasing for all x
The function f ( x ) = x 3 − 3 x 2 − 24 x + 5
55.  is an increasing (C)  Decreasing for x < 0 and increasing for x > 0
function in the interval
(D)  Increasing for x < 0 and decreasing for x > 0
(A)  ( −∞ , − 2) ∪ (4, ∞ ) (B)  ( −2, ∞ )
64.  Consider the following statements:
(C)  (−2, 4) (D)  ( −∞ , 4)
π 
56. 
Which one is the correct statement about the function S: Both sin x and cos x are decreasing functions in  , π 
2 
f(x) = sin2x?
R: If a differentiable function decreases in (a, b), then its
 π π 
(A)  f ( x ) is increasing in  0,  and decreasing in  , π  derivative also decreases in (a, b).
 2 2 
Which of the following is true?
 π π 
(B)  f ( x ) is decreasing in  0,  and increasing in  , π  (A)  Both S and R are wrong
 2 2  (B) 
Both S and R are correct but R is not the correct
 π π π  explanation for S
(C)  f ( x ) is increasing in  0,  and decreasing in  , 
 4 4 2 (C)  S is correct and R is the correct explanation for S
(D)  The statements (A), (B)  and (C)  are correct. (D)  S is correct and R is wrong.

Chapter 21.indd 929 10-06-2018 18:24:35


930 Mathematics Problem Book for JEE

65. 
The function which is neither decreasing nor increasing in 75.  The function f ( x ) = 1− x 3 − x 5 is decreasing for
 p 3p 
 ,  is (A)  1 ≤ x ≤ 5 (B)  x ≤ 1
2 2 
(C)  x ≥ 1 (D)  All values of x
(C)  x    (D)  | x − 1|
2
(A)  cosec x  (B)  tan x
l sin x + 6 cos x 76.  The function x x is increasing, when
66.  Function f ( x ) = is monotonically increasing, if
2sin x + 3cos x 1 1
(A)  x > (B)  x <
(A)  l > 1  
(B)  l < 1 (C)  l < 4  (D)  l > 4 e e
(C)  x < 0 (D)  For all real x
2
67.  On the interval (1, 3), the function f ( x ) = 3 x + is
x 77.  2 x 3 − 6 x + 5 is an increasing function if
(A)  Strictly decreasing
(A)  0 < x < 1 (B)  −1 < x < 1
(B)  Strictly increasing
(C)  x < −1 or x > 1 (D)  −1 < x < −1/ 2
(C)  Decreasing in (2, 3) only
(D)  Neither increasing nor decreasing 78. 
The length of the longest interval, in which the function
68.  If f ( x ) = sin x − cos x , the function decreasing in 0 ≤ x ≤ 2p is 3sin x − 4 sin3 x is increasing, is
(A)  [5p /6, 3p /4] (B)  [p /4, p /2] p p 3p
(A)  (B) 
  (C)   (D)  p
(C)  [3p /2,5p /2] (D)  None of these 3 2 2

log x 79.  Let f ( x ) = x 3 + bx 2 + cx + d ,0 < b2 < c. Then f(x)


69.  The function f ( x ) = is increasing in the interval (A)  is bounded (B)  has a local maxima
x
(A)  (1,2e ) (B)  (0, e) (C)  has a local minima (D)  is strictly increasing
(C)  (2, 2e) (D)  (1/e, 2e) 80.  If f ( x ) = x , −1 ≤ x ≤ 1, then function f ( x ) is
70.  If f ( x ) = xe x (1− x )
, then f ( x ) is (A)  Increasing (B)  Decreasing
(C)  Stationary (D)  Discontinuous
 1 
(A)  Increasing on  − ,1
 2  81.  For all x ∈ (0,1), which is the correct one?
(B)  Decreasing on R x
(A)  e < 1+ x (B)  loge (1+ x ) < x
(C)  Increasing on R
 1  (C)  sin x > x (D)  loge x > x
(D)  Decreasing on  − ,1
 2  The function f ( x ) = 2 x 3 − 3 x 2 + 90 x + 174 is increasing in the
82. 
3 2
If f ( x ) = x − 6 x + 9 x + 3 be a decreasing function, then x lies
71.  interval
in 1 1
(A)  < x < 1 (B)  < x < 2
(A)  ( −∞ , −1) ∩ (3, ∞ ) (B)  (1, 3) 2 2
59
(C)  (3, ∞ ) (D)  None of these (C)  3 < x < (D)  −∞ < x < ∞
4
1 −1
72.  If f ( x ) = − log(1+ x ), x > 0, then f ( x ) is The function f ( x ) = tan (sin x + cos x ), x > 0 is always an
83. 
x +1 increasing function on the interval
(A)  An increasing function
(A)  (0, p ) (B)  (0, p / 2)
(B)  A decreasing function
(C)  (0, p /4) (D)  (0,3p / 4)
(C)  Both increasing and decreasing functions
(D)  None of these  e2 x − 1
84.  The function f ( x ) =  2 x  is
73.  The function f ( x ) = x + cos x is  e + 1
(A)  Always increasing (A)  Increasing (B)  Decreasing
(B)  Always decreasing (C)  Even (D)  None of these
(C)  Increasing for certain range of x What is the value of x if the function x 5 − 5 x 4 + 5 x 3 − 10 has
85. 
(D)  None of these a maximum?
(A)  3  
(B)  2 (C)  1  
(D)  0
74.  The function f ( x ) = x 1/ x is
(A)  Increasing in (1, ∞ ) log x
86.  The local maximum value of the function is
(B)  Decreasing in (1, ∞ ) x
(C)  Increasing in (1, e ), decreasing in (e , ∞ ) 1
(D)  Decreasing in (1, e ), increasing in (e , ∞ ) (A)  e  
(B)  1 (C)   (D)  2e
e

Chapter 21.indd 930 12-06-2018 19:11:28


Chapter 21 | Applications of Derivatives 931

87.  The function x 5 − 5 x 4 + 5 x 3 − 1 is (A)  4  


(B)  3 (C)  2   (D)  1
(A)  Maximum at x = 3 and minimum at x = 1 2
102.  Maximum value of x (1− x ) , when 0 ≤ x ≤ 2, is
(B)  Minimum at x = 1
(A)  2 / 27  (B)  4 / 27 (C)  5   (D)  0
(C)  Neither maximum nor minimum at x = 0
(D)  Maximum at x = 0 103. 
If from a wire of length 36 m, a rectangle of greatest area is
88. 
The adjacent sides of a rectangle with given perimeter as made, then its two adjacent sides (in metre) are
100 cm and enclosing maximum area are (A)  6, 12   (B)  9, 9 (C)  10, 8    (D)  13, 5
(A)  10 cm and 40 cm (B)  20 cm and 30 cm 2
104.  The minimum value of 2 x + x − 1 is
(C)  25 cm and 25 cm (D)  15 cm and 35 cm
9
89. 
The area of a rectangle will be maximum for the given (A)  −1/ 4  (B)  3 / 2 (C)  −9 / 8   (D) 
perimeter, when rectangle is a 4
(A)  Parallelogram (B)  Trapezium The minimum value of the function y = 2 x 3 − 21x 2 + 36 x − 20
105. 
(C)  Square (D)  None of these is
 90. 
When 36 factorises into two factors in such a way that the (A)  −128 (B)  −126
sum of factors is minimum, then the factors are (C)  −120 (D)  None of these
(A)  2, 18 (B)  9, 4 106. 
The sum of two non−zero numbers is 4. The minimum value
(C)  3, 12 (D)  None of these of the sum of their reciprocals is
(A)  3 / 4 (B)  6 / 5
If f ( x ) = 2 x 3 − 3 x 2 − 12 x + 5 and x ∈ [ −2, 4] , then the
 91. 
maximum value of function is at the following value of x (C)  1 (D)  None of these
(B)  −1
(A)  2   (C)  −2    (D)  4 The minimum value of [(5 + x )(2 + x )] / [1+ x ] for non-negative
107. 
 92. The point for the curve y = xe x real x is
(A)  x = −1 is minimum (B)  x = 0 is minimum (A)  12  
(B)  1 (C)  9   (D)  8
(C)  x = −1 is maximum (D)  x = 0 is maximum
108.  One maximum point of sinp x cosq x is
x
 93. The maximum value of (1/ x ) is −1 −1
(A)  x = tan ( p / q ) (B)  x = tan (q / p )
e e −e e
(A)  (e )  
(B)  (e ) (C)  (e )    (D)  (1/ e ) −1 −1
(C)  x = tan ( p / q ) (D)  x = tan (q / p )
 94. 
The number that exceeds its square by the greatest amount
is 109. 
When 20 is divided into two parts so that the product of the
cube of one quantity and the square of the other quantity is
(A)  −1  
(B)  0 (C)  1/ 2    
(D)  1
maximum. The parts are
If for a function f ( x ), f ’(a) = 0 , f "(a) = 0 , f ′′′(a) > 0 , then at
 95.  (A)  10, 10   (B)  16, 4 (C)  8, 12    D)  12, 8
x = a, f ( x ) is
2 2
(A)  Minimum (B)  Maximum If f ( x ) = ( x − 1) / ( x + 1), for every real number x, then the
110. 
(C)  Not an extreme point (D)  Extreme point minimum value of f
(A)  does not exist because f is unbounded
 96. 
The least value of the sum of any positive real number and
its reciprocal is (B)  is not attained even though f is bounded
(C)  is equal to 1
(A)  1  
(B)  2 (C)  3    (D)  4
(D)  is equal to −1
x
 97.  x has a stationary point at The number of values of x where the function f ( x ) = cos x +
111. 
(A)  x = e (B)  x = 1/ e   cos( 2x ) attains its maximum is
(C)  x = 1 (D)  x = e (A)  0  
(B)  1 (C)  2   (D)  Infinite
2
− 2 x +1)sin2 x
x
 98. When x is positive, the minimum value of x is 112.  The minimum value of e(2 x is
−1 −1/e 1/e e (A)  e  
(B)  1/e (C)  1    (D)  0
(B)  e
(A)  e   (C)  e (D)  e
   
 99. The maximum value of xy subject to x + y = 8 is If x and y be two variables such that x > 0 and xy = 1, then
113. 
the minimum value of x + y is
(A)  8  
(B)  16 (C)  20    (D)  24
(A)  2  
(B)  3 (C)  4   (D)  0
x
∫0 te
−t 2
100.  A minimum value of dt is 114.  x /(1+ x tan x ) is maxima at
(A)  1  
(B)  2 (C)  3    (D)  0 (A)  x = sin x (B)  x = cos x
101. 
If the sum of two numbers is 3, then the maximum value of (C)  x = p /3 (D)  x = tan x
the product of the first number and the square of the second 115. 
If x is real, then the greatest and the least values of
number is ( x 2 − x + 1) / ( x 2 + x + 1) are

Mathematical Problem Book for JEE.indb 931 07-06-2018 13:46:08


932 Mathematics Problem Book for JEE

1 1 (A)  No maxima and minima


(A)  3, − (B)  3, (B)  One maximum and one minimum
2 3
(C)  Two maxima
1
(C)  −3, − (D)  None of these (D)  Two minima
3
128.  If f ( x ) = 1/ (4 x 2 + 2 x + 1), then its maximum value is
log x
116.  The minimum value of in the interval [2, ∞ ) is (A)  4/3   (B)  2/3 (C)  1   (D)  3/4
x
log2 1
(A)  (B)  Zero 129.  If f ( x ) = x +
, x > 0, then its greatest value is
2 x
1 (A)  − 2 (B)  0
(C)  (D)  Does not exist
e (C)  3 (D)  None of these
117.  The maximum value of x e 4 − x2
is 130. 
The perimeter of a sector is p. The area of the sector is
maximum when its radius is
2
(A)  e (B)  e −2
(A)  (B)  1/ p
p   (C)  p /2    (D)  p /4
(C)  12e −2 (D)  4e −2
If y = a log x + bx 2 + x has its extremum value at x = 1 and
131. 
118.  If A + B = π / 2, the maximum value of cos A cos B is
x = 2, then what is (a, b )?
(A)  1/2 (B)  3/4
1 1 
(C)  1 (D)  4/3 (A)   1,  (B)   ,2 
 2 2 
119. 
A cone of maximum volume is inscribed in a given sphere,
then ratio of the height of the cone to diameter of the  −1   −2 −1 
(C)   2,  (D)   , 
sphere is  2  3 6
(A)  2/3 (B)  3/4 x

∫ (t − 4)e −4 t dt has
4
(C)  1/3 (D)  1/4 132.  In ( −4, 4) the function f ( x ) =
−10
120. 
The ratio of height of cone of maximum volume inscribed in
(A)  No extrema (B)  One extremum
a sphere to its radius is
(C)  Two extrema (D)  Four extrema
(A)  3/4 (B)  4/3
(C)  1/2 (D)  2/3 133.  On [1, e], the greatest value of x 2 log x is
121.  The function f ( x ) = x + sin x has 2 1 1
(A)  e (B)  log
(A)  A minimum but no maximum e e
(B)  A maximum but no minimum 2
(C)  e log e (D)  None of these
(C)  Neither maximum nor minimum
(D)  Both maximum and minimum At what value of x, the function f ( x ) = x − x , ( x ∈ R ) attains a
134. 
maximum value?
The function f ( x ) = ax + (b/ x ); a, b , x > 0 takes on the least
122. 
(A)  2   (B)  3 (C)  1/e   (D)  1
value at x equal to
(A)  B  
(B)  a (C)  b (D)  b/a 135.  If ab = 2a + 3b , a > 0, b > 0 then the minimum value of ab is
(A)  12 (B)  24
123.  If xy = c 2 , then the minimum value of ax + by is (C)  1/ 4 (D)  None of these
(A)  c ab (B)  2c ab
136. 
If PQ and PR are the two sides of a triangle, then the angle
(C)  −c ab (D)  −2c ab between them which gives maximum area of the triangle is
(A)  p   (B)  /3 (C)  p /4    (D)  p /2
2 4 2 4 6
124.  If a x + b y = c , then the maximum value of xy is
For what value of x, the function y = a(1− cos x ) is maximum?
137. 
(A)  c 2 / ab (B)  c 3 /ab
(A)  π (B)  p /2
(C)  c 3 / 2ab (D)  c 3 /2ab
(C)  −p /2 (D)  −p /6
3 2
125.  The function f ( x ) = 2 x − 15 x + 36 x + 4 is maximum at 138.  The minimum value of [ x 2 + (250/ x )] is
(A)  x = 2 (B)  x = 4 (A)  75 (B)  50
(C)  x = 0 (D)  x = 3 (C)  25 (D)  55
1/ x
139.  The maximum value of x is
126.  The maximum slope of the curve y = − x 3 + 3 x 2 + 9 x − 27 is
1/e
(A)  0  
(B)  12 (C)  16    (D)  32 (A)  1/ e (B)  e
(C)  E (D)  1/ e e
127.  The function f ( x ) = 2 x 3 − 3 x 2 − 12 x + 4 has

Chapter 21.indd 932 12-06-2018 19:13:24


Chapter 21 | Applications of Derivatives 933

2x
140.  The minimum value of 4 e + 9e
−2 x
is (C)  a < x1 < b (D)  a ≤ x1 ≤ b
(A)  11 (B)  12 Consider the function f ( x ) = e −2 x sin 2x over the interval
153. 
(C)  10 (D)  14  p  p
 0,  . A real number c ∈ 0,  , as guaranteed by Rolle’s
2 2
141.  The point (0,5) is closest to the curve x 2 = 2 y at
theorem, such that f ′ (c ) = 0 is
(A)  (2 2 ,0) (B)  (0, 0)
(A)  p /8 (B)  p /6
(C)  (2,2) (D)  None of these
(C)  p /4 (D)  p /3
142.  The maximum value of xy when x + 2 y = 8 is
Let f ( x ) = x − 1 + x + 24 − 10 x − 1; 1 < x < 26 be real-
154. 
(A)  20  
(B)  16 (C)  24   (D)  8
valued function. Then f '( x ) for 1 < x < 26 is
143.  The minimum value of P(1, 1) is
(A)  0 (B)  1/ x − 1
(A)  15 / 2  (B)  11/ 2 (C)  −13 / 2   (D)  71/ 8
If P = (1, 1), Q = (3,2) and R is a point on x-axis, then the value
144.  (C)  2 x − 1 − 5 (D)  None of these
of PR + RQ will be minimum at 155. 
If f ( x ) satisfies the conditions of Rolle’s theorem in [1,2] and
2
5  1 
(A)   ,0    (B)   ,0  (C)  (3, 0)    (D)  (1, 0)
f ( x ) is continuous in [1, 2] , then ∫1 f '( x )dx is equal to
3  3  (A)  3  
(B)  0 (C)  1   (D)  2
2 2 4 10 20 3 2
145.  Let f ( x ) = 1+ 2 x + 2 x +  + 2 x . Then f ( x ) has If the function f ( x ) = x − 6 x + ax + b satisfies Rolle’s
156. 
(A)  More than one minimum  2 3 + 1
theorem in the interval [1,3] and f '   = 0, then
(B)  Exactly one minimum  3 
(C)  At least one maximum
(A)  a = −11 (B)  a = −6
(D)  None of these
(C)  a = 6 (D)  a = 11
For which interval, the function ( x 2 − 3 x )/( x − 1) satisfies all
146. 
the conditions of Rolle’s theorem? f (b ) − f ( a ) 1
157. 
In mean value theorem = f '(c ), if a = 0, b = and
(A)  [0, 3] (B)  [− 3, 0] b−a 2
f ( x ) = x ( x − 1)( x − 2), the value of c is
(C)  [1.5, 3] (D)  For no interval
x
(A)  1− ( 15 /6) (B)  1+ 15
For the function f ( x ) = e , a = 0, b = 1, the value of c in mean
147. 
value theorem will be (C)  1− ( 21/6) (D)  1+ 21
(A)  log x (B)  log(e − 1) (C)  0
(D)  1 The abscissa of the points of the curve y = x 3 in the interval
158. 
Rolle’s theorem is not applicable to the function f ( x ) =| x |
148.  [−2, 2], where the slope of the tangents can be obtained by
defined on [−1, 1] because mean value theorem for the interval [−2, 2], are
(A)  f is not continuous on [ −1, 1] (A)  ±(2/ 3) (B)  ± 3
(B)  f is not differentiable on (−1,1) (C)  ±( 3 /2) (D)  0
(C)  f ( −1) ≠ f (1)
(D)  f ( −1) = f (1) ≠ 0 In mean value theorem, f (b ) − f (a) = (b − a)f '(c ) if a = 4,
159. 
b = 9 and f ( x ) = x , the value of c is
If f ( x ) = cos x ,0 ≤ x ≤ (π / 2) , then the real number c of the
149. 
mean value theorem is (A)  8.00 (B)  5.25 (C)  4.00 (D)  6.25

(A)  p /6 (B)  p /4 Practice Exercise 2


−1 −1
(C)  sin (2/ p ) (D)  cos (2/ p )

From mean value theorem f (b ) − f (a) = (b − a)f '( x1); a < x1 < b
150.  Single/Multiple Correct Choice Type Questions
if f ( x ) = 1/ x, then what is the value of x1?  1. For the curve represented parametically by the equations,
(A)  ab (B)  (a + b )/2 x = 2 ln cot t + 1 and y = tan t + cot t
(A)  tangent at t = p/4 is parallel to x-axis
(C)  2ab /(a + b ) (D)  (b − a)/(b + a)
(B)  normal at t = p/4 is parallel to y-axis
For the function x + (1/ x ), x ∈ [1,3], the value of c for the
151.  (C)  tangent at t = p/4 is parallel to the line y = x
mean value theorem is
(D)  tangent and normal intersect at the point (2, 1)
(A)  1 (B)  3
  2. Let g′(x) > 0 and f ′(x) < 0, ∀ x∈ R. Then
(C)  2 (D)  None of these (A) g(f(x +1)) > g(f(x − 1))
f (b ) − f ( a ) (B) f(g(x − 1)) > f(g(x + 1))
152.  If from mean value theorem, f '( x1) = , then
b−a (C) g(f(x + 1)) < g(f(x − 1))
(A)  a < x1 ≤ b (B)  a ≤ x1 < b (D) g(g(x + 1)) < g(g(x − 1))

Chapter 21.indd 933 10-06-2018 18:27:00


934 Mathematics Problem Book for JEE

 3. If f(x) = x3 − x2 + 100x + 1001, then Paragraph for Questions 12−14: If a continuous function f
 1   1  defined on the real line R, assumes positive and negative values
(A)  f(2000) > f(2001) (B)  f   > f  2000  in R then the equation f(x) = 0 has a root in R. For example, if it is
 1999 
(C)  f(x + 1) > f (x − 1) (D)  f (3x − 5) > f (3x) known that a continuous function f on R is positive at some point
and its minimum value is negative then the equation f(x) = 0 has
  4. The abscissa of a point on the curve xy = (a + x)2, the normal a root in R.
at which the cuts off numerically equal intercepts from the
coordinate axes is Consider f(x) = kex − x for all real x where k is a real constant.
a 12.  The line y = x meets y = kex for k ≤ 0 at
(A)  − (B)  2 a
2
(A)  no point (B)  one point
2a (C)  two points (D)  more than two points
(C)  (D)  − 2 a
2 13. The positive value of k for which kex − x = 0 has only one root is
In x
  5. For the function f(x) = , which of the following statements (A)  1 e (B)  1
x
are true?
(C)  e (D)  loge 2
(A)  f(x) has the horizontal tangent at x = e
(B)  f(x) cuts the x−axis only at the one point
For k > 0, the set of all the values of ‘k’ for which kex − x = 0 has
14. 
(C)  f(x) is the many−one function two distinct roots is
(D)  f(x) has the one vertical tangent
 1 1  1 
 6. The equation of the tangent drawn to the curve y = (x + 1)3 (A)   0,   (B)   , 1 (C)   , ∞    (D)  (0, 1)
from the origin is  e e  e 
(A)  y = 3x (B)  y = − 3x
Paragraph for Questions 15−17: Let f and g are the two functions
(C)  4y = 27x (D)  y = 0
such that f(x) and g(x) are continuous in [a, b] and differentiable in
  7. If the derivative of an odd cubic polynomial vanishes at two (a, b). Then at least one c ∈ (a, b), such that
different values of ‘x’, then
f (b ) − f ( a )
(A) the coefficient of x3 and x in the polynomial must be the f ′(c)  =
same in sign b−a
(B) the coefficient of x3 and x in the polynomial must be the (i) If f(a)  = f(b), then f ′(c)  = 0 (RMVT)
different in sign (ii) If f(a)  ≠ f(b)  and a ≠ b (LMVT)
(C) the values of ‘x’ where derivative vanishes are closer to the f (b ) − f ( a ) f ′(c )
origin as compared to the respective roots on the either (iii) If g′(x) ≠ 0, then = (Cauchy’s theorem)
g(b ) − g(a) g ′(c )
side of origin.
(D) the values of ‘x’ where derivative vanishes are far from the The set of the values of k, for which the equation x3 − 3x + k
15. 
origin as compared to the respective roots on the either = 0 has two distinct roots in (0, 1) is
side of the origin. (A)  (1, 4)   (B)  (0, ∞) (C)  (0, 1)    (D)  f
  8. Let f(x) = (x − 1)4 (x − 2)n , n∈N. Then f(x) has
(A)  Local minimum at x = 2 if n is even 16.  Which of the following is true?
(B)  Local minimum at x = 1 if n is odd
(A)  |tan−1x − tan−1y| ≤ |x − y| ∀ x, y ∈ R
(C)  Local maximum at x = 1 if n is odd
(B)  |tan−1x − tan−1y| ≥ |x − y| ∀ x, y ∈ R
(D)  Local minimum at x = 1 if n is even
(C)  |sin x − sin y| ≥ |x − y| ∀ x, y ∈ R
Comprehension Type Questions (D)  None of these
p sinα − sin β
Paragraph for Questions 9−11: Let a(t) is a function of t such 17.  If 0 < a < q < b <
, then is equal to
da 2 cos α − cos β
that = 2 for all the values of t and a = 0 when t = 0. Further (A)  tan q  
(B)  − tan q (C)  cot q    (D)  − cot q
dt
y = m(t) x + c(t) is the tangent to the curve y = x2 − 2ax + a2 + a at the
point whose abscissa is 0. Then Paragraph for Questions 18−20: Let y = a x + bx be curve, (2x
 9. If the rate of change of the distance of the vertex of y = x − 2ax
2 − y) + l (2x + y − 4) = 0 be family of lines.
+ a2 + a from the origin with respect to t is k, then k = 1
18. If the curve has slope − at (9, 0), then a tangent belonging
2 2
(A)  2 (B)  2 2 (C)  (D)  4 2
to the family of lines is
If the rate of change of c(t) with respect to t, when t = k, is ,
10.  (A)  x + 2y − 5 = 0 (B)  x − 2y + 3 = 0
then (C)  3x − y − 1 = 0 (D)  3x + y − 5 = 0
(A)  16 − 2 2 (B)  8 2 + 2 19. A line of the family cutting positive intercepts on the axes and
(C)  10 2 + 2 (D)  16 2 + 2 forming the triangle with the coordinate axes, then the mini-
mum length of the line segment between the axes is
11.  The rate of change of m(t), with respect to t, at t =  is (A) (22/3 − 1)3/2   (B) (22/3 + 1)3/2
(A)  −2  
(B)  2 (C)  − 4     (D)  4 (C) 73/2   (D) 27

Chapter 21.indd 934 10-06-2018 18:27:02


Chapter 21 | Applications of Derivatives 935

Two perpendicular focal chords of the curve y2 − 4x − 4y + 4


20.  (B) 
The area of the triangle formed by the normal
= 0 form the diagonals of a quadrilateral. Minimum area of a 1
at the point (1, 0) on the curve x = esiny with (q) 
quadrilateral is 2
the axes is
(A)  16 (B)  32
If the angle between the curves x2y = 1 and y = (r)  7
(C) 
(C)  64 (D)  50 e2(1−x) at the point (1, 1) is q , then tanq  is equal
to
Paragraph for Questions 21−23: A function f(x) having the fol-
lowing properties: The length of the sub−tangent at any point on (s)  3
(D) 
the curve y = bex/3 is equal to
(i)  f(x) is the continuous except at x = 3
(ii)  f(x) is the differentiable except at x = − 2 and x = 3 (t)  0

(iii)  f(0) = 0, lim f(x) → − ∞, lim f(x) = 3, lim f(x) = 0


x →3 x →− ∞ x→ ∞
26.  Match the following:
(iv)  f ′(x) > 0 ∀ x ∈ (−∞, − 2) ∪ (3, ∞) and f ′(x) ≤ 0 ∀ x ∈ (− 2, 3)
List I List II
(v) f ′′(x) > 0 ∀ x ∈ (− ∞, − 2) ∪ (− 2, 0) and f ′′(x) < 0 ∀ x ∈ (0, 3) ∪ A function f is differentiable in [0, 5] and f(0) = 4 (p)  3
(A) 
(3, ∞) f (x)
Then, answer the following questions: and f(5) = −1. If g(x) = and c ∈ (0, 5), then
x +1
21.  Maximum possible number of solutions of f(x) = | x | is g′(c)  is equal to
(A)  2 (B)  1 (C)  3 (D)  4 Let f(x) and g(x) be differentiable for 0 ≤ (q)  − 5/6
(B) 
22.  Graph of function y = f (− | x |) is x ≤ 1, f(0) = 2, g(0) = 0, f(1) = 6. Let there
(A)  differentiable for all x, if f ′(0) = 0 exists a the real number c ∈ (0, 1) such that
f ′(c)  = 2g′(c). Then g(1) is equal to
(B)  continuous but not differentiable at two points, if f ′(0) = 0
(C)  continuous but not differentiable at one points, if f ′(0) = 0 (C) 
The length of the longest interval in which (r)  15
p
(D)  discontinuous at two points, if f ′(0) = 0 f(x) = 3 sinx − 4 sin3x is increasing, is , then
λ
23.  f(x) + 3x = 0 has five solutions if l is
(A)  f(−2) > 6 (B)  f ′(0) < − 3 and f(− 2) > 6 (D) 
If Lagrange’s mean value theorem is satisfied (s)  2
(C)  f ′ (0) > − 3 (D)  f ′(0) > − 3 and f(− 2) > 6 for f(x) = 25 − x 2 and c ∈ (1, 5), then the value
of c2 is
Matrix Match Type Questions (t)  10
24.  Match the following:
27.  Match the following:
List I List II
List I List II
(A) 
Circular plate is expanded by the heat from (p) 2
the radius 5 cm to 5.06 cm. Approximate  p (p) 0
increase in the area is (A) 
Number of values of  ‘x’  lying in  0,  , at which
 2
If an edge of a cube increases by 1%, then the (q) 0.6 p
(B)  f ( x ) = ln(sin x ) is not monotonic, is
percentage increase in the volume is (B) 
If the greatest interval in which the function f(x) (q) 2
2 (r) 3 = x3 − 3x + 2 is decreasing is [a, b], then a + b =
x
If the rate of decrease of y =
(C)  − 2x + 5 is
2 x2 + 2 (r)  − 3
twice the rate of decrease of x, then x is equal If f(x) =
(C)  , 1 ≤ x ≤ 3 (where [⋅] greatest
[x]
to (given that the rate of decrease is non−zero)
integer function), then the least value of f(x) is
(D) 
Rate of increase in the area of the equilateral (s) 3
3 3 Set of all possible values of ‘a’ such that f(x) =
(D) 
triangle of side 15 cm, when each side is (s) 
4 e2x − (a + 1) ex + 2x is monotonically increasing
increasing at the rate of 0.1 cm/sec; is
for all x ∈ R is (− ∞, k], then k eqvals
(t) 4
(t)  − 2

25.  Match the following: 28.  Match the following:


List I List II
List I List II
(A) 
If a portion of the tangent at any point on (p) 1
the curve x = at3, y= at4 between the axes is (A) 
Number of points which are the local extrema (p) 1
(2 + x ) ;
3
divided by the point of contact in the ratio m:n – 3 ≤ x ≤ –1
externally, then | n + m | is equal to (m and n of f(x) =  2/3
 x ; –1 < x < 2
are coprime)

Mathematical Problem Book for JEE.indb 935 07-06-2018 13:46:19


936 Mathematics Problem Book for JEE

List I List II 32. 


A cone is made from a circular sheet of the radius 3 by
If a + b = 1; a > 0, b > 0, then the minimum value (q) 2
(B)  cutting out a sector and keeping the cut edges of the
remaining piece together. The maximum volume attainable
of  1  1  is λp
 1+  1+  for the cone is , then find l.
 a  b  3
If the possible values of ‘a’ such that the inequality 3 − x2 >
33. 
(C) 
The maximum value attained by (r) 3
   y = 10 − |x − 10|, −1 ≤ x ≤ 3, is  13 
|x − a| has at least one negative solution is a ∈  − , l  , then
 4 
If P(t2, 2t), t ∈ [0, 2] is an arbitrary point on the (s) 4
(D)  find l.
parabola y2 = 4x and Q is foot of the perpen­
 xe ax , x ≤0
dicular from focus S on the tangent at P, then 34. Let f(x) =  , where a is positive constant and
2 3
maximum area of the triangle PQS is  x + ax − x ,x >0
(t) 5  λ1 a 
the interval in which f ′ (x) is increasing is  − ,  , then find
 a λ2 
Integer Type Questions (l1 + l2).
x2 y 2 A cubic f(x) vanishes at x = −2 and has relative minimum/
35. 
29. 
Let α be the angle in radians between + = 1 and the 1
36 4 1 14
circle x + y = 12 at their points of the intersection. If a = tan−1
2 2 maximum at x = −1 and x =
3
. If ∫ f ( x ) dx =
3
, the cubic
−1
k f(x) = l1x3 + l2x2 − x + 2, then find (l1 + l2).
, then find the value of k . 2

2 3 2 If f(x) is a twice differentiable function such that f(a)  = 0, f(b)  =


36. 
 9 
Find the minimum value of (x1 − x2)2 +  2 − x12 −
30.  , 2, f(c)  = −1, f(d)  = 2, f(e) = 0, where a < b < c < d < e, then the
 x 2 minimum number of the zeroes of g(x) = (f ′(x))2 + f ′′(x) f(x) in
where x1 ∈ (0 , 2 ) and x2 ∈ R+. the interval [a, e] is _____.
1 The chord of the parabola y = −p2x2 + 5px − 4 touches the
37. 
The values of ‘a’ for which the function f(x) = sinx − a sin2x −
31.  1
3 curve y = at the point x = 2 and is bisected by that
sin 3x + 2ax increases throughout the number line is [ l, ∞), (1− x )
then find l. point. Find the number of the values of ‘p’.

Answer Key
Practice Exercise 1
  1. (D)   2. (C)   3. (B)   4. (C)   5. (A)   6. (A)
  7. (B)   8. (C)   9. (C)  10. (A)  11. (B)  12. (C)
 13. (A)  14. (D)  15. (C)  16. (C)  17. (C)  18. (A)
 19. (A)  20. (D)  21. (D)  22. (B)  23. (D)  24. (B)
 25. (C)  26. (C)  27. (B)  28. (A)  29. (D)  30. (C)
 31. (B)  32. (D)  33. (A)  34. (A)  35. (A)  36. (D)
 37. (A)  38. (C)  39. (B)  40. (D)  41. (B)  42. (C)
 43. (B)  44. (D)  45. (A)  46. (C)  47. (B)  48. (C)
 49. (B)  50. (A)  51. (A)  52. (D)  53. (A)  54. (C)
 55. (A)  56. (C)  57. (D)  58. (C)  59. (C), (D)  60. (D)
 61. (A)  62. (B)  63. (C)  64. (D)  65. (A)  66. (D)
 67. (B)  68. (D)  69. (B)  70. (A)  71. (B)  72. (B)
 73. (A)  74. (C)  75. (D)  76. (A)  77. (C)  78. (A)
 79. (D)  80. (A)  81. (B)  82. (D)  83. (C)  84. (A)
 85. (C)  86. (C)  87. (C)  88. (C)  89. (C)  90. (D)
 91. (D)  92. (A)  93. (C)  94. (C)  95. (C)  96. (B)
 97. (B)  98. (B)  99. (B) 100.  (D) 101.  (A) 102.  (B)

Mathematical Problem Book for JEE.indb 936 07-06-2018 13:46:21


Chapter 21 | Applications of Derivatives 937

103.  (B) 104.  (C) 105.  (A) 106.  (C) 107.  (C) 108.  (A)
109.  (D) 110.  (D) 111.  (A) 112.  (C) 113.  (A) 114.  (B)
115.  (B) 116.  (D) 117.  (D) 118.  (A) 119.  (A) 120.  (B)
121.  (C) 122.  (D) 123.  (B) 124.  (C) 125.  (A) 126.  (B)
127.  (B) 128.  (A) 129.  (D) 130.  (D) 131.  (D) 132.  (C)
133.  (A) 134.  (C) 135.  (B) 136.  (D) 137.  (A) 138.  (A)
139.  (B) 140.  (B) 141.  (D) 142.  (D) 143.  (D) 144.  (A)
145.  (D) 146.  (D) 147.  (B) 148.  (B) 149.  (C) 150.  (A)
151.  (B) 152.  (A) 153.  (A) 154.  (A) 155.  (B) 156.  (D)
157.  (C) 158.  (A) 159.  (D)

Practice Exercise 2
1. (A), (B) 2. (B), (C) 3. (B), (C) 4. (A) 5. (A), (B), (C) 6. (C), (D)
7. (B), (C) 8. (A), (C), (D) 9. (B) 10. (D) 11. (C) 12. (B)
13. (A) 14. (A) 15. (D) 16. (A) 17. (D) 18. (B)
19. (B) 20. (B) 21. (C) 22. (B) 23. (D) 24.(A)→(q),(B)→(r),
 (C)→ (t), (D)→ (s) 25. (A)→(r), (B)→ (q), (C)→(t), (D)→ (s) 26. (A)→ (q), (B)→ (s), (C)→ (p), (D)→ (r)
27. (A)  → (p), (B)  → (p), (C)  → (s), (D)  →(s) 28. (A)  →(q), (B)  →(r), (C)  → (r), (D)  → (t) 29. 16 30. 8
31. 1 32. 2 33. 3 34. 5 35. 2 36. 6 37. 2

Solutions

Practice Exercise 1 or x = 2 x1

 1. We have Therefore, the point on x-axis is (2 x1,0). Now, the tangent
dy meets y-axis where x = 0. Since
= 12 − 3 x 2 = 0 ⇒ x = ± 2
dx x12 y = 2a2 x1
Hence, the points are (2, 16) and (−2, −16).
2a2
we have y=
 2. We have x1
a2 So, the point on the y-axis is
y=
x  2a 2 
Therefore,  0, 
 x1 
dy a2 The required area is
=− 2
dx x 1  2a 2 
(2 x1)  = 2a 2
Now, at ( x1 , y1). 2  x 
 1 
At the point (x1, y1), we have  3. We have t = 2 for the point (2,−1). Therefore, for t = 2, we get
dy −a2 dy 4t − 2 6
= = =
dx x12 dx 2t + 3 7
Thus, the tangent to the curve is  4. It is given that
−a 2 y 2 = 2( x − 3) (1)
y − y1 = ( x − x1)
x12 Differentiating w.r.t. x, we get
 dy  dy 1
⇒ yx12 − y1x12 = −a2 x + a2 x1 2y   = 2 ⇒ =
 dx  dx y
1 2(1+ x 2 ) − 4 x 2 (∵ x1y1 = a2 ) The slope of the normal is
⇒ y' . 
2 2 (1+ x 2 )2 −1
1− [(2 x ) / (1+ x )] = −y
dy/dx
This meets the x-axis where y = 0. Therefore,
The slope of the given line is 2. Therefore,
a2 x = 2a2 x1 y = −2

Mathematical Problem Book for JEE.indb 937 07-06-2018 13:46:22


938 Mathematics Problem Book for JEE

From Eq. (1), we get Therefore,


x =5  dy  −b − x1 / a − y1
  = e =  [by Eq. (1)]
Thus, the required point is (5, − 2) .  ( x1 , y1 ) a
dx a

 5. The given curve is Now, the equation of tangent of given curve at point ( x1, y1) is
2
x = 3 − 2 y (1) −y x y x1
y − y1 = 1 ( x − x1) ⇒ + = + 1
Differentiating w.r.t. x, we get a a y1 a
dy dy x y
2x = 0 − 2 ⇒ = −x Comparing with + = 1, we get y1 = b and
dx dx a b
The slope of the tangent of the curve is − x. From the given x
1+ 1 = 1 ⇒ x1 = 0
line, the slope is −1 and hence x = 1 and from Eq. (1), y = 1. a
Therefore, the coordinate of the point is (1, 1). Hence, the point is (0, b).
2
 6. We have x = t and y = 2t . At t = 1, x = 1 and y = 2, we have 10.  For the curve
 dy  dy / dt 2 1  dy  dy 4
  = = = ⇒   =1 y2 = 4 x ⇒ =
dx dx / dt 2t t  dx  t =1 dx 2 y
we have
Therefore, the equation of the normal at (1, 2) is  dy 
  =1
 1  dx (1,2)
y − 2 = −  ( x − 1)
 dy / dx  and for the curve
⇒ y − 2 = −1( x − 1) dy − x
x2 + y2 = 5 ⇒ =
⇒ x + y −3 = 0 Therefore, dx y

 7. We have  dy  −1
  =
px  dx  (1,2) 2
y = sin
2 The angle between the curves is
dy p p ( −1/2) − 1
⇒ = cos x q = tan−1 = tan−1(3)
dx 2 2 1+ ( −1/2)
dy
⇒   =0 11.  We have
 dx (1, 1)
by 2 = ( x + a)3
Therefore, the equation of the normal is
 dy 
1 ⇒ 2by   = 3( x + a)2
y − 1 = ( x − 1)  dx 
0
dy 3
⇒ x =1 ⇒ = ( x + a)2
dx 2by
 8. We have Therefore, the subnormal is
y = 2cos x
dy 3
At x = p /4, y = 2/ 2 = 2 and therefore, y = ( x + a)2
dx 2b
dy Therefore, the subtangent is
= −2(sin x )
dx
y y 2by 2
 dy  = =
⇒  =− 2 dy / dx 3( x + a) / 2by 3( x + a)2
2
 dx  x = p / 4
p  2b[( x + a)3 / b] 2
Therefore, the equation of the tangent at  , 2  is = = ( x + a)
4  3( x + a)2 3
 p  4
y − 2 = − 2x −  or (Subtangent)2 = ( x + a)2
 4 9
9.  Let the point be ( x1, y1). Therefore, (Subtangent)2 (4 / 9)( x + a)2 8b
Also = =
Subnormal (3 / 2b )( x + a)2 27
y1 = be − x1 / a (1)

⇒(Subtangent)2 = Constant × (Subnormal)
Also, the curve is
Therefore, (Subtangent)2 ∝ (Subnormal)
y = be − x / a
dy −b − x / a 12.  Now,
⇒ = e y = ax 2 + bx
dx a

Mathematical Problem Book for JEE.indb 938 07-06-2018 13:46:24


Chapter 21 | Applications of Derivatives 939

That is, 16.  The length of the normal is


dy  dy   dy 
2
= 2ax + b ⇒   = 4a + b y 1+  
dx  dx  (2, − 8)
 dx 
Since the tangent is parallel to x−axis, we have Now,
dy dy dy / dq a sinq sinq 2sin(q /2)cos(q /2)
= 0 ⇒ b = −4 a (1) = = = =
dx dx dx / dq a(1+ cosq ) 1+ cosq 2cos2 (q /2)

Now, the point (2, −8) is on the curve of y = ax 2 + bx. Therefore, Therefore,
−8 = 4 a + 2b (2)  dy   q  p
   p  =  tan   p  = 1[ y ]  p  = a  1− cos  = a
dx  θ =   2  θ =  
θ =  2
 2  2 2
From Eqs. (1) and (2), we get a = 2, b = −8 .
Hence, the length of the normal is
13.  We have
a 1+ (1)2 = 2a
1 dy 1
x + y = a; + =0
2 x 2 y dx 17.  We have
y = a(sinθ − θ cosθ ), x = a(cosθ + θ sinθ )
Therefore,
Therefore,
dy y
=− dy
dx x = a[cosθ − cosθ + θ sinθ ] = aθ sinθ

Hence, the tangent at (x, y) is dx
= a( − sinθ + sinθ + θ cosθ ) = aθ cosθ
y dθ
Y−y =− (X − x)
x That is,
dy dy / dq aq sinq
= = = tanq
or X y + Y x = xy ( x + y ) = axy dx dx / dq aq cosq

X Y Therefore, slope of the tangent is tanθ and the slope of the


or + =1 normal is − cot θ . Hence, the equation of normal is
a x a y cosθ
[ y − a sinθ + aθ cosθ ] = − [ x − a cosθ − aθ sinθ ]
Clearly, its intercepts on the axes are a x and a y. sinθ
Sum of the intercepts is ⇒ y sinθ − a sin2 θ + aθ sinθ cos θ

a( x + y ) = a ⋅ a = a = − x cosθ + a cos2 θ + aθ sinθ cosθ


⇒ x cosθ + y sinθ = a(sin2 θ + cos2 θ )
⇒ x cosθ + y sinθ = a
14.  We have
Therefore, the distance from the origin is
dy
y = x log x ⇒ = 1+ log x a
dx = Constant
The slope of the normal is sin2 θ + cos2 θ
1 −1 18.  We have
− =
(dy / dx ) 1+ log x x = 3t 2 + 1, y = t 3 − 1
The slope of the line 2 x − 2 y = 3 is 1. Therefore, Therefore,
dy
−1 = 3t 2
= 1 ⇒ log x = −2 ⇒ x = e −2 dt
1+ log x
dy  dy / dt  3t 2 t
Now, =  = =
⇒ y = −2e −2 dx  dx / dt  6t 2
−2 −2 For x = 1 ,
Therefore, the coordinate of the point is (e , − 2e ).
0
3t 2 + 1 = 1⇒ t = 0 ⇒ Slope = =0
15.  The slope of the normal is 2
1 Let the point of contact be (h, k ), where k = h4 . The tangent is
19. 

(dy / dx )
 dy 3
y − k = 4 h3 ( x − h)  As dx = 4 x 
This is parallel to x−axis. Therefore,  
1 dx It passes through (2, 0). Therefore,
− =0⇒ =0
(dy/dx ) dy −k = 4 h3 (2 − h)

Chapter 21.indd 939 10-06-2018 18:27:41


940 Mathematics Problem Book for JEE

8 The slope of the tangent of the second curve is


⇒ h = 0 or
3 dy
m2 = = b x log b
8
4 dx
⇒ k = 0 or  
3  dy 
⇒ m2 =   = log b
 8 8 4 dx (0,1)
Since points of contact are (0, 0) and  ,    .
 3  3  Therefore,
The equation of tangents are m1 − m2 log a − log b
tanα = =
4 3 1+ m1m2 1+ log a log b
 8  8  8
y = 0 and y −   = 4    x − 
 3  3  3

20.  We have 23.  We have


x 2 = −4 y
y = x2
dy
⇒ 2 x = −4
dy dx
⇒ = m1 = 2 x
dx dy − x
⇒ =
 dy  dx 2
⇒  = 2 = m1 and x = y 2
 dx  (1,1)  dy 
⇒  =2
 dx  ( −4, − 4)
dy
⇒ 1= 2 y We know that the equation of tangent is
dx
dy 1  dy 
⇒ = m2 = ( y − y1) =   ( x − x1) ⇒ y + 4 = 2( x + 4)
 dx  ( x , y )
dx 2y 1 1

 dy  1 ⇒ 2x − y + 4 = 0
⇒  =
 dx  (1,1) 2
Therefore, the angle of intersection is 24.  We have
2 − (1/ 2) 3 dy
m1 − m2 y = 2x2 − x +1 ⇒ = 4 x −1
tanθ = = = dx
1+ m1m2 1 + 2 × (1/ 2) 4
We know that this equation gives the slope of tangent to
⇒ θ = tan−1(3 / 4) the curve. Since this tangent is parallel to y = 3 x + 9, the
slope of the tangent is 3 and so 4 x − 1 = 3 or x = 1. Therefore
21.  We have y = 2 x 2 − x + 1 = 2 − 1+ 1 = 2. Thus, the point ( x , y ) is (1, 2).
y = x3 − 3x2 − 9 x + 5
25.  We have
dy
⇒ = 3x2 − 6 x − 9 x 3 − 8 a2 y = 0
dx
We know that this equation gives the slope of the tangent to ⇒ 3 x 2 − 8a2  dy  = 0
the curve. The tangent is parallel to the x-axis. Therefore,  dx 
dy dy
dx
=0 ⇒ 3 x 2 = 8a2  
 dx 
Therefore, dy 3 x 2
2 ⇒ =
3x − 6 x − 9 = 0 dx 8a2
Therefore, the slope of the normal is
⇒ x = −1, 3
1 1 8 a2
22. 
Clearly, the point of intersection of curves is (0, 1). Now, the − = − 2 =− 2
(dy / dx ) 3 x / 8 a2 3x
slope of tangent of the first curve is
Given that
dy
m1 = = a x log a −8a2 −2
dx =
3x2 3
 dy 
⇒  = m1 = log a Therefore,
 dx  (0,1)
( x , y ) = (2a, a)

Chapter 21.indd 940 10-06-2018 18:27:43


Chapter 21 | Applications of Derivatives 941

26.  We have  dy  −b
x = a(t + sin t ), y = a(1− cos t )   =
dx (0, b ) a
Therefore,
Therefore, the equation of tangent is
dy dy / dt a(sin t ) t
= = = tan −b
dx dx / dt a(1+ cos t ) 2 y −b = ( x − 0)
a
The length of the normal is
x y
2 ⇒ + =1
 dy  a b
y 1+  
 dx 
30.  We have
= a(1− cos t ) 1+ tan2 (t /2) = a(1− cos t )sec(t /2) y = x2
= 2a sin2 (t /2)sec(t /2) = 2a sin(t /2)tan(t /2)  dy 
⇒  = m1 = 2 x = 2
 dx  (1,1)
27.  We have
y = e2 x dy
and 6 y = 7 − x 3 ⇒ 6. = −3 x 2
dx
dy
⇒ = 2e 2 x  dy  1
dx ⇒  = m2 = −
 dx  (1,1) 2
 dy 
⇒  =2 Clearly, m1m2 = −1. Therefore, the angle of intersection is
 dx (0,1)
p /2.
Therefore, the equation of tangent is
y − 1 = 2( x − 0) ⇒ y = 2 x + 1
31.  We have
This tangent meets x−axis. Therefore,
y =0 y = 2x2 − x +1
Let the coordinates of P be (h, k). Then
⇒ 0 = 2 x + 1⇒ x = −1/ 2
 dy 
 1  = 4h − 1
Therefore, the coordinates of the point are  − ,0  .  dx  ( h , k )
 2 
28.  We have Clearly, P is parallel to y = 3x + 4. Therefore, slopes are equal
2
(1+ x ) y = 2 − x (1) 4h − 1 = 3 ⇒ h = 1
It meets x-axis, where y = 0. That is, Therefore, P is (1, 2).
0=2− x ⇒ x =2 32.  We have
So, Eq. (1) meets x-axis at the point (2, 0). Also from Eq. (1), xy = c 2 (1)
2− x Subnormal is y (dy / dx ) . Therefore, from Eq. (1), we get
y=
1+ x 2
c2 dy − c 2
2 y= ⇒ = 2
dy (1+ x )( −1) − (2 − x )(2 x ) x dx x
⇒ =
dx (1+ x 2 )2 Thus, the subnormal is
2
dy x − 4 x − 1 y ( −c 2 ) − yc 2 − yc 2 y 2 − y 3
⇒ = 2
= 2 2
= = 2
dx (1+ x 2 )2 x (c / y ) c4 c
The slope of tangent at (2, 0) is Therefore, the subnormal varies as y 3 .
4 − 8 − 1 −5 −1 33.  If sin x = cos x , then x = p /4.
= =
(1+ 4)2 25 5
 dy  1
Therefore, the equation of the tangent at (2, 0) is If y = sin x , then   = .
 dx  x = p / 4 2
1  dy  −1
y − 0 = − ( x − 2) ⇒ x + 5 y = 2 If y = cos x , then   = .
5  dx  x = p / 4 2
m − m2
29.  The curve is y = be − x / a . tanq = 1 = 2 2 ⇒ q = tan−1(2 2 )
1+ m1m2
Since the curve crosses y−axis (i.e. x = 0), y = b. Now,
dy −b − x / a 34.  We have
= e
dx a
y 2 = 5 x − 1 (1)
At point (0, b), we have

Mathematical Problem Book for JEE.indb 941 07-06-2018 13:46:34


942 Mathematics Problem Book for JEE

At (1, −2) , we have  3  −3  1 


dy  5  −5  y − = x− 
=  = 2 2 2  2
dx  2 y (1, − 2) 4 ⇒ 3 2x + 2 2y = 6
Therefore, the equation of normal at the point (1, −2) is ⇒ 3x + 2y = 3 2
 −5  39.  The curve is
[ y − ( −2)]   + x − 1 = 0
4 x + y = e xy
Therefore,
Differentiating with respect to x, we get
4 x − 5 y − 14 = 0 (2)
dy  dy 
As the normal is of the form ax − 5 y + b = 0 and on comparing 1+ = e xy  y + x 
dx  dx 
this with Eq. (2), we get a = 4 and b = −14 .
dy ye xy − 1
35.  Given that or =
dx 1− xe xy
y = 6 x − x 2 (1) As tangent is parallel to y−axis.
dy Thus,
= 6 − 2x
dx dy
= ∞ ⇒ 1− xe xy = 0 ⇒ 1− x ( x + y ) = 0
Since, the tangent is parallel to the line dx
4 x − 2 y − 1= 0 This holds for x = 1 , y = 0.
Therefore,
dy −4 40.  We have
= 6 − 2x = ⇒ 6 − 2x = 2 ⇒ x = 2
dx −2
f ( x ) = −2 x 3 − 9 x 2 − 12 x + 1
Substituting the value of x in Eq. (1), we get y = 8. Hence, the
required point of tangency will be (2,8). ⇒ f '( x ) = −6 x 2 − 18 x − 12

36.  The slope of the normal is To be decreasing, we need to have f '( x ) < 0 , that is,

− dx − d[a(1+ cosq )]/ dq a sinq −6 x 2 − 18 x − 12 < 0


= = = tanq
dy d (a sinq )/ dq a cosq ⇒ x2 + 3x + 2 > 0
Now, the equation of normal at q is ⇒ ( x + 2)( x + 1) > 0
y − a sinq = tanq [ x − a(1+ cosq )] Therefore, either x < −2 or x > −1. So,
Clearly, this line passes through (a, 0). x ∈( −1, ∞ ) or ( −∞ , −2)
37.  We have 41.  To be increasing, we need to have
dx dy f '( x ) = 3 x 2 − 27 > 0
= a(1+ cos q ), = a (sinq )
dq dq
⇒ x2 > 9 ⇒ | x | > 3
dy dy/dq a sinq 42.  We have
= = = 1 = y q = p /2 = a
dx q =p /2 dx/dq a(1+ cosq ) x 1
f ( x ) = sin x −
⇒ f '( x ) = cos x −
The length of the subtangent is 2 2
y a We know that f '( x ) > 0 for increasing function. Obviously, it
ST = = =a is increasing for
dy/dx 1
p p
and the length of the subnormal is − <x<
3 3
dy
SN = y = a (1) = a
dx 43.  We have
Hence, ST = SN
f ( x ) = x sin x + cos x + cos2 x
38.  We have
Therefore,
2 1
x q =p / 4
= = f '( x ) = sin x + x cos x − sin x − 2cos x sin x = cos x ( x − 2sin x )
2 2 2
Hence, x → 0 to p , then f '( x ) < 0. That is, f ( x ) is decreasing
3 dy 9 sin2 q cos q −3 function.
y q =p / 4
= , =
2 2 dx q =p / 4 −6 cos2 q sinq q =p / 4
2 44.  We know that
dy −1
Therefore, the equation of the tangent is = x ( x − 2)
dx e x

Mathematical Problem Book for JEE.indb 942 07-06-2018 13:46:36


Chapter 21 | Applications of Derivatives 943

is positive when x ( x − 2) is negative, that is, x lies in the 50.  We have


interval (0, 2). An exponential function, as we know, is always dy
y = tan x − x ⇒ = sec2 x − 1 = tan2 x ≥ 0
positive. dx
45.  Here,
51.  We have
f ( x ) = y = 2 x 3 − 9 x 2 + 12 x − 6 2x
f ( x ) = log(1+ x ) −
⇒ f '( x ) = 6 x 2 − 18 x + 12 2+ x
Since f ( x ) is increasing or decreasing in (a, b ) according as 1 (2 + x ) ⋅ (2) − (2 x )
⇒ f ′( x ) = −
f '( x ) > 0 or < 0 for every x ∈(a, b ). Hence, f '( x ) = 6( x − 2)( x − 1) 1+ x (2 + x )2
which is obviously decreasing if x ∈(1, 2), that is, x2
⇒ f ′( x ) =
1< x < 2 ( x + 1)( x + 2)2
46.  We have Obviously,
2
f ( x ) = ( x − 1) − 1 f '( x ) > 0 for all x > 0
Hence, f ( x ) is increasing on (0, ∞ ) .
Hence, decreasing in x < 1 (Fig. 21.30).
3 2
If f ( x ) = (a + 2) x − 3ax + 9ax − 1 decreases monotonically for
52. 
Y
all x ∈R , then f ′( x ) ≤ 0 for all x ∈R .
  3(a + 2) x 2 − 6ax + 9a ≤ 0 for all x ∈R
y = f (x)
⇒ (a + 2) x 2 − 2ax + 3a ≤ 0 for all x ∈R
⇒ a + 2 < 0 and discriminant ≤ 0
⇒ a < −2, −8a2 − 24 a ≤ 0 ⇒ a < −2 and a(a + 3) ≥ 0
(1,0) ⇒ a < −2, a ≤ −3 or a ≥ 0 ⇒ a ≤ −3 ⇒ −∞ < a ≤ −3
X
O
53.  We have
f ( x ) = 2 x + cot −1 x + log( 1+ x 2 − x )
(1, –1) Therefore,
1 1  x 
f '( x ) = 2 − +  − 1
1+ x 2 2 2 
Figure 21.30 1+ x − x  1+ x 
Aliter: f '( x ) = 2 x − 2 = 2( x −1). 1+ 2 x 2 1 1+ 2 x 2 (1+ x 2 )
= − = −
To be decreasing, we need to have 1+ x 2 1+ x 2 1+ x
2
1+ x 2
2( x − 1) < 0 ⇒ ( x − 1) < 0 ⇒ x < 1
=
x 2 + 1+ x 2 ( 1+ x − 1) ≥ 0 for all x
2

47.  We know that f ( x ) is monotonically decreasing if f '( x ) < 0 . 1+ x 2


1 Hence, f(x) is an increasing function on ( −∞ , ∞ ) and in
f '( x ) = − sin x − 2 p < 0 ⇒ sin x + p > 0
2 particular on [0, ∞ ).
1
⇒p> [∵ −1 ≤ sin x ≤ 1].
2 Since f ( x ) = x 3 ⇒ f '( x ) = 3 x 2 , which is non−negative for all
54. 
48.  We have real values of x.
f '( x ) = 3kx 2 − 18 x + 9 = 3 [kx 2 − 6 x + 3] > 0, ∀x ∈R
55.  We have
Therefore,
f ( x ) = x 3 − 3 x 2 − 24 x + 5
2
∆ = b − 4 ac < 0, k > 0
For increasing function, we need to have f '( x ) > 0, that is,
That is,
3 x 2 − 6 x − 24 > 0
36 − 12k < 0 or k > 3
⇒ x2 − 2x − 8 > 0
49.  We have That is,
y = f ( x ) = 2 x 3 − 15 x 2 + 36 x + 1 x 2 − 4 x + 2 x − 8 > 0 ⇒ ( x + 2)( x − 4) > 0
Therefore, x ∈( −∞ , − 2) ∪ (4, ∞ )
dy
= f '( x ) = 6 x 2 − 30 x + 36 = 6( x 2 − 5 x + 6)
dx 56.  Since
or f '( x ) = 6( x − 2)( x − 3) f ( x ) = sin2 x ⇒ f '( x ) = 2cos2 x
To be monotonically decreasing, we need to have f '( x ) < 0, obviously,
that is,  p
⇒ ( x − 2)( x − 3) < 0 ⇒ x ∈(2,3) f '( x ) > 0 in  0, 
 4

Mathematical Problem Book for JEE.indb 943 07-06-2018 13:46:40


944 Mathematics Problem Book for JEE

π π Therefore, g '( x ) < 0 ⇒ g( x ) is decreasing.


and f '( x ) < 0 in  , 
 4 2 60.  A function is monotonically decreasing, when f ′( x ) < 0
Hence, the result.   6 x 2 − 18 x + 12 < 0
⇒ x2 − 3x + 2 < 0
57.  We have
⇒ x2 − 2x − x + 2 < 0
f ( x ) = ( x + 2)e − x ⇒ ( x − 2)( x − 1) < 0
f '( x ) = e − x − e − x ( x + 2) Therefore, x ∈1 < x < 2.

f '( x ) = − e − x [ x + 1] 61.  We have f '( x ) = 6 x 2 + 36 x − 96 > 0 for increasing function


  f '( x ) = ( x + 8)( x − 2) ≥ 0
For increasing function,
⇒ x ≥ 2, x ≤ −8
− e − x ( x + 1) > 0 or e − x ( x + 1) < 0
62.  Let us consider
e − x > 0 or ( x + 1) < 0 a sin x + b cos x
y=
x ∈( −∞ , ∞ ) and x ∈( −∞ , −1) c sin x + d cos x
Therefore, The function will be decreasing when dy/dx < 0.
(c sin x + d cos x )(a cos x − b sin x ) − (a sin x + b cos x )(c cos x − d sin x )
x ∈( −∞ , −1) <0
(c sin x + d cos x )2
Hence, the function is increasing in ( −∞ , − 1).
⇒ ac sin x cos x − bc sin2 x + ad cos2 x
For decreasing function,
−bd sin x cos x − ac sin x cos x + ad sin2 x
− e − x ( x + 1) < 0 or e − x ( x + 1) > 0 , x ∈( −1, ∞ )
−bc cos2 x + bd sin x cos x < 0
Hence, the function is decreasing in ( −1, ∞ ).
⇒ ad (sin2 x + cos2 x ) − bc (sin2 x + cos2 x ) < 0
58.  We have
⇒ (ad − bc ) < 0
f ( x ) = x 3 − 10 x 2 + 200 x − 10
63.  We have
That is, 2

f '( x ) = 3 x 2 − 20 x + 200 f ( x ) = 1− e − x /2

For increasing function, we need to have Therefore,


2 2
2
f '( x ) > 0 ⇒ 3 x − 20 x + 200 > 0 f ′( x ) = − e − x /2
( − x ) = xe − x /2

For f ( x ) to be increasing, then f ′( x ) > 0. So,


 20 200 100 100 
3  x2 − x+ + − >0
9 
2

 3 3 9 xe − x /2 > 0 ⇒ x > 0
and f ( x ) to be decreasing for x < 0.
 10 
2
500 
⇒ 3  x −  + >0 64. 
See Fig. 21.31. From the trend of value of sin x and cos x ,
 3 9 
 we know sin x and cos x decrease in (p /2) < x < p. So,
2 statement S is correct.
 10  500
⇒3 x −  + >0 Y
 3 3
This is always increasing throughout the real line.
59.  We have
sin x − x cos x cos x (tan x − x )
f '( x ) = =
sin2 x sin2 x ψ1 ψ2
X
0 < x ≤ 1⇒ x ∈Q1 ⇒ tan x > x , cos x > 0 O a x1 x2 b

Therefore,
Figure 21.31
f '( x ) > 0 for 0 < x ≤ 1
Statement R is incorrect which is depicted in the graph
Thus, f ( x ) is an increasing function. Now, (Fig. 21.31). Clearly, f ( x ) is differentiable in (a, b). Also,
a < x1 < x 2 < b. However,
tan x − x sec2 x sin x cos x − x sin2 x − 2 x
g '( x ) = = =
tan2 x sin2 x 2sin2 x f ′( x1) = tanθ1 < tanθ2 = f ′( x 2 )

(sin2 x − 2 x )' = 2cos2 x − 2 = 2[cos2 x − 1] < 0 π 3π 


65.  The graph of cosec x is opposite in  ,  (Fig. 21.32).
⇒ sin2 x − 2 x is decreasing ⇒ sin2 x − 2 x < 0 2 2 

Chapter 21.indd 944 10-06-2018 18:29:28


Chapter 21 | Applications of Derivatives 945

Y 70.  We have
f ′( x ) = e x (1− x ) + x (e x (1− x ) )(1− 2 x )

1 = e x (1− x ) [1+ x (1− 2 x )] = e x (1− x ) ( −2 x 2 + x + 1)

2
Now, by the sign−scheme (Fig. 21.33) for −2 x 2 + x + 1,
X  1 
O /2 3 /2 f ′( x ) ≥ 0, if x ∈ − ,1 because e x (1− x ) is always positive.
 2 
 1 
So, f ( x ) is increasing on  − ,1 .
 2 
− + −
Figure 21.32
66.  The function is monotonically increasing if
f ′( x ) > 0
−1/2 1
(2sin x + 3cos x )( λ cos x − 6 sin x )

(2sin x + 3cos x )2 Figure 21.33
( λ sin x + 6 cos x )(2cos x − 3sin x ) 71.  We have
− >0
(2sin x + 3cos x )2
f ( x) = x3 − 6x2 + 9x + 3
⇒ 3λ (sin2 x + cos2 x ) − 12(sin2 x + cos2 x ) > 0
For the function to be decreasing, we need to have
⇒ 3λ − 12 > 0 ⇒ λ > 4
f ′( x ) < 0
67.  We have
2 ⇒ 3 x 2 − 12 x + 9 < 0
f ( x ) = 3x +
x ⇒ x2 − 4 x + 3 < 0
2
⇒ f ′( x ) = 3 − ⇒ ( x − 3) ( x − 1) < 0
x2
Clearly, f ′( x ) > 0 on the interval (1, 3); therefore, f ( x ) is Therefore, x ∈(1,3).
strictly increasing.
72.  We have
68.  We have
f ( x ) = sin x − cos x 1
f (x) = − log(1+ x )
Therefore, x +1
  π  π ⇒ f ′( x ) = −
1 1
f ′( x ) = cos x + sin x = 2 cos  x −   = 2 cos  x −  −
  4  4 ( x + 1)2 1+ x
That is,
For f ( x ) to be decreasing,
f ′( x ) < 0  1 1 
f ′( x ) = −  + 2
π π  3π  x + 1 ( x + 1) 

<x−  < (within 0 ≤ x ≤ 2p )
2  4 2 Now, f ′( x ) is negative when x > 0 or f ′( x ) < 0, ∀x > 0.
3π 7π Therefore, f ( x ) is decreasing function.
⇒ <x≤
4 4
69.  We have 73.  We have
log x f ( x ) = x + cos x
f (x) =
x
⇒ f ′( x ) = 1− sin x
Therefore,
1 log x 1− log x Now, f ′( x ) > 0 for all values of x. Therefore, f ( x ) is an
f ′( x ) = − 2 = increasing function.
x2 x x2
For f ( x ) to be increasing, 74.  Let us consider
f ′( x ) > 0
y = x 1/ x
⇒ 1− log x > 0
⇒ 1 > log x Taking log both sides, we have
⇒e> x  1
⇒ log y =   log x
 x
Therefore, f ( x ) is increasing in the interval (0, e ).

Chapter 21.indd 945 10-06-2018 18:29:31


946 Mathematics Problem Book for JEE

1 dy 1 log x 1− log x 80.  It is always increasing (Fig. 21.34).


⇒ = − 2 =
y dx x 2 x x2 Y


dy  1− log x 
= x 1/ x 
dx  x 2 
f(x) = x
1/ x 1− log x
Now, x > 0 for all x and > 0 in (1, e) and
x2
1− log x X
< 0 in (e , ∞ ). Therefore, f ( x ) is increasing in (1, e)
x2
and decreasing in (e , ∞ ).

75.  We have
f ( x ) = 1− x 3 − x 5

⇒ f ′( x ) = −3 x 2 − 5 x 4
Figure 21.34
That is, f ′( x ) < 0 for all values of x. x
Both e and 1+ x are increasing and
81.  e ≥ 1+ (1/ 2), because

76.  Let us consider y = x x . Then e = 1.65 (approximately) and so option (A)  is not correct.
Since
dy  1 22 
= x x (1+ log x ) p p
dx sin <   < 
6 6  2 42

For (dy / dx ) > 0; option (C)  is not correct. Now,
1 1  1 
x x (1+ log x ) > 0 log <   log is negative
2 2 2
1
⇒ 1+ log x > 0 ⇒ loge x > loge
e Therefore, Option (D) is not correct. Thus, by elimination,
For this to be positive, x should be greater than 1/ e. option (B)  is correct.

82.  f ′ ( x ) = 6( x 2 − x + 15) > 0 ∀ x .


77.  Let us consider f ( x ) = 2 x 3 − 6 x + 5 . Then
83.  We have
  f ′( x ) = 6 x 2 − 6 > 0
  p 
f ( x ) = y = tan−1  2 sin  x +  
⇒ x 2 − 1> 0   4
⇒ ( x − 1) ( x + 1) > 0
 p dy  p
⇒ x > 1 or x < −1 ⇒ tan y = 2 sin  x +  ⇒ sec2 y = 2 cos  x + 
 4 dx  4
78.  We have dy p

Now, > 0 ⇒ cos  x +  > 0
3sin x − 4 sin3 x = sin3 x dx  4
which is increasing when −p / 2 ≤ 3 x ≤ p / 2, that is,  p
−p / 6 ≤ x ≤ p / 6 . Therefore, x ∈ 0,  .
 4
The length of the interval is
84.  We have
p  p p e2 x − 1
−−  = f (x) =
6  6 3 e2 x + 1

79.  Given that e −2 x − 1 1− e 2 x


⇒ f (− x ) = =
e −2 x + 1 1+ e 2 x
f ( x ) = x 3 + bx 2 + cx + d
e2 x − 1
⇒ f (x) = − = −f ( x )
Therefore, e2 x + 1
f '( x ) = 3 x 2 + 2bx + c Thus, f ( x ) is an odd function. Now,
Now, its discriminant is 4(b2 − 3c ), so e2 x − 1
f (x) =
4(b2 − c ) − 8c < 0 e2 x + 1
4e2 x
since b2< c and c > 0. Therefore, f '( x ) > 0 for all x ∈R . ⇒ f ′( x ) = > 0 ∀ n ∈R
(1+ e 2 x )2
Hence, f is strictly increasing. which implies that f ( x ) is an increasing function.

Mathematical Problem Book for JEE.indb 946 07-06-2018 13:46:48


Chapter 21 | Applications of Derivatives 947

85.  Obviously, it has a maximum at x = 1. 89.  We know that the perimeter of a rectangle is

86.  Let us consider S = 2( x + y )


where x and y are adjacent sides.
log x
f (x) = S - 2x
x ⇒y=
1 log x 2
Þ f '( x ) = 2 - 2 Now, the area of a rectangle is
x x
x 1
For maximum or minimum value of f ( x ) f '( x ) = 0 A = xy = ( S - 2 x ) = ( Sx - 2 x 2 )
2 2
f ( x ) f '( x ) = 0
Differentiating A w.r.t. x, we get
1- loge x dA 1
⇒ f '( x ) = =0 = (S - 4 x ) = 0
x2 dx 2
1- loge x Therefore,
or =0
x2 S S
x= and y =
4 4
Therefore, loge x = 1 or x = e, which lie in (0, ∞ ). For x = e , Now, d 2 A / dx 2 is negative. Hence, the area of rectangle is
maximum when the rectangle is a square.
d2 y 1
2
=- 3 90. Let the two factors of 36 be x and 36/x. Now, solving
dx e
x + (36 / x ) to be minimum. The factors will be 6, 6.
which is negative. Hence, y is maximum at x = e and its 91.  We have
maximum value is f '( x ) = 6 x 2 - 6 x - 12
log e 1
= That is,
e e
f '( x ) = 0 Þ ( x - 2)( x + 1) = 0 Þ x = -1,2
87.  Let us consider
f ( x ) = x5 - 5x 4 + 5x3 - 1 Here, f (4) = 128 - 48 - 48 + 5 = 37. That is,
f ( -1) = -2 - 3 + 12 + 5 = 12
⇒ f '( x ) = 5 x 4 - 20 x 3 + 15 x 2 = 0
Therefore, f (2) = 16 - 12 - 24 + 5 = -15

( x - 3)( x - 1) = 0 or x = 3,1 f ( -2) = -16 - 12 + 24 + 5 = 1


Therefore, the maximum value of the function is 37 at x = 4 .
Now, f ¢¢( x ) = 20 x 3 - 60 x 2 + 30 x
x
92.  Given equation [of the curve (Fig. 21.35)] is y = xe . Therefore,
Substituting x = 3 and 1, we get f ¢¢(3) is positive, f ¢¢(1) is
negative and f ¢¢(0) = 0 . Hence, f ( x ) is neither maximum nor dy d2 y
= xe x + e x = e x (1+ x ) and = ( x + 2) e x
minimum at x = 0. dx dx 2
A
88.  We have
2 x + 2 y = 100 Þ x + y = 50 (1)
a
Let the area of rectangle be A. Therefore,
A
A = xy Þ y =
x O
Substituting this in Eq. (1), we have a a
x
A
x+ = 50
x B M C
Þ A = 50 x - x 2
dA Figure 21.35
⇒ = 50 - 2 x
dx
For the maximum or minimum value of f ( x ), we have
For maximum area, dA / dx = 0. Therefore, dy
= 0 Þ x = -1
50 - 2 x = 0 dx

Therefore, x = 25 and y = 25. Hence, the adjacent sides are Therefore, {f ′′( x )} x = −1 is positive. Hence, f ( x ) is minimum at
25 and 25 cm. x = −1.

Mathematical Problem Book for JEE.indb 947 07-06-2018 13:46:52


948 Mathematics Problem Book for JEE

93.  We have 1
x Therefore, y is minimum at x = and minimum value
 1 e
f (x) =  
 x 1/ e
 1
=  = e −1/ e .
x  1  e
1 x log  
Since,   = e x
x  99. We have
dy  1 x+y=8
Thus, = e u where, u = x ⋅ log  
du x  Therefore,
On solving this equation, we have
x y = 8 − x (1)
1 1
⇒ f '( x ) =    log − 1  Now, f ( x ) = xy = x (8 − x ) = 8 x − x 2
 x  x 
 Therefore,
1 1 1
Now, f '( x ) = 0 ⇒ log = 1 = log e ⇒ = e ⇒ x = f ′( x ) = 8 − 2 x
x x e
1/e
Therefore, the maximum value of function is e .  For maximum value of f ( x ), f '( x ) = 0. Therefore, x = 4 and
y = 4 and hence the maximum value of xy is 4 × 4 = 16.
94.  Let the number be x. Then
100.  We have
y = x − x2 x
f ( x ) = ∫ te − t dt ⇒ f '( x ) = xe − x = 0 ⇒ x = 0
2 2

2 0
dy d y
⇒ = 1− 2 x and = −2(< 0)
dx dx 2
2
f ′′( x ) = e − x (1− 2 x 2 ); f ′′ (0) = 1 > 0
1
⇒ 1− 2 x = 0 ⇒ x =   Therefore, the minimum value is f (0) = 0.
2
95.  This is a fundamental property. Let the first number be 3 − x and the second number be x.
101. 
96.  We have Accordingly, we have to maximize (3 − x ) x 2. Let us consider
1 1 f ( x ) = (3 − x ) x 2 = 3 x 2 − x 3 ⇒ f '( x ) = 6 x − 3 x 2
f (x) = x + ⇒ f '( x ) = 1− 2
x x
Therefore,
⇒ f '( x ) = 0 ⇒ x 2 − 1 = 0 ⇒ x = 1, −1
f '( x ) = 0 ⇒ x = 0, 2
However, it is given that x is positive; hence, at x = 1, we have
1 Also
f ( x ) = 1+ = 2 f ′′ ( x ) = 6 − 6 x
1

97.  Let us consider Obviously, f ′′ (2) = −6 < 0. Therefore, the required maximum
2
y = x x ⇒ log y = x (log x ), ( x > 0) value is (3 − 2)2 = 4.
On differentiating, we get
102.  Given that
dy
= x x (1+ log x ) f ( x ) = x (1− x )2
dx
Therefore, That is,

dy f ( x) = x3 − 2x2 + x
=0
dx
Now, f '( x ) = 3 x 2 − 4 x + 1
1
⇒ log x = −1 ⇒ x = e −1 = Substituting f ′( x ) = 0 , we have
e
Therefore, the stationary point is x = 1/e . 3x 2 − 4 x + 1= 0
98.  We have, 3 x 2 − 3 x − x + 1 = 0 ⇒ x = 1, 1/ 3
d2 y 1 Now, f ′′( x ) = 6 x − 4
= x x (1+ log x )2 + ( x x )
dx 2 x
When x = 1/ e Therefore, f ′′ (1) = 2 is positive and f ''(1/ 3) = −2 is negative.
(1/ e ) −1 Hence, the maximum value is x = 1/ 3. The maximum is
d 2 y  1
=  >0
dx 2  e   1 4
f =
 3  27

Chapter 21.indd 948 12-06-2018 19:15:12


Chapter 21 | Applications of Derivatives 949

103.  Given that 107.  Given that


2(a + b ) = 36 [(5 + x )(2 + x )]
f (x) =
a + b = 18 [1+ x ]

The area of the rectangle is 4 4


f ( x ) = 1+ + (5 + x ) = (6 + x ) +
1+ x (1+ x )
ab = a(18 − a)
4
⇒ f '( x ) = 1− = 0 ; x 2 + 2 x − 3 = 0 ⇒ x = −3, 1
Now, A = 18a − a2 (1+ x )2
That is,
8
Now, f ′′ ( x ) =
dA (1+ x )3
= 18 − 2a
da
That is, f ′′( −3) is negative and f ′′(1) is positive. Hence, the
Substituting dA / da = 0 , we get minimum value at x = 1 is

18 − 2a = 0 ⇒ a = 9; b = 9 (5 + 1)(2 + 1) 6 × 3
f (1) = = = 9.
(1+ 1) 2
104.  We have
108.  Let us consider that
f ( x) = 2x2 + x −1
y = sinp x (cosq x )
1
⇒ f '( x ) = 4 x + 1⇒ f '( x ) = 0 ⇒ x = − dy
4 Now, = p sinp −1 x (cos x )(cosq x ) + q cosq −1 x ( − sin x )sinp x
dx
Therefore, f ′′ ( x ) = 4 is positive. Therefore,
dy
= p sinp −1 x (cosq +1 x ) − q cosq −1 x (sinp +1 x )
2 1 −9 dx
[f ( −1/ 4)]min = − − 1=
16 4 8 Substituting dy/dx = 0, we get
105.  Given that p p
tan2 x = ⇒ tan x = ±
f ( x ) = 2 x 3 − 21x 2 + 36 x − 20 q q
and f '( x ) = 6 x 2 − 42 x + 36 Therefore, the point of maxima is
On substituting, we get p
x = tan−1
f '( x ) = 0 ⇒ 6 x − 42 x + 36 = 0 ⇒ x − 7 x + 6 = 0
2 2 q

⇒ x − 6 x − x + 6 = 0 ⇒ ( x − 1)( x − 6) = 0 ⇒ x = 1,6
2
109.  Let us consider
Now, f ′′ ( x ) = 12 x − 42 x + y = 20 ⇒ y = 20 − x
That is, f ′′ (1) = −30 is negative and f ′′ (6) = 30 is positive. and x 3( y2 ) = z ⇒ z = x3( y2 )
Hence, x = 6 is the point of minima and the minimum value
is z = x 3 (20 − x )2 ⇒ z = 400 x 3 + x 5 − 40 x 4
f (6) = 2(6)3 − 21(6)2 + 36 × 6 − 20 dz
Now, = 1200 x 2 + 5 x 4 − 160 x 3
dx
f (6) = −128
Now, dz/dx = 0, then x = 12, 20. Also
106.  Let us consider
x + y = 4 or y = 4 − x d2z
= 2400 x + 16 x 3 − 480 x 2
That is, dx 2
1 1 x+y 4 4  d2z 
+ = or f ( x ) = =
x y xy xy x (4 − x ) and  2
 dx  x =12
4
Now, f ( x ) = is negative. Hence, x = 12 is the point of maxima. Therefore,
4 x − x2
−4 x = 12, y = 8
and f '( x ) = (4 − 2 x )
(4 x − x 2 )2 110.  We have
Substituting f '( x ) = 0, we get 4 − 2 x = 0. Therefore, x 2 − 1 x 2 + 1− 2 2
f (x) = 2
= 2
= 1− 2
x = 2 and y = 2 x +1 x +1 x +1
Therefore, Therefore,
 1 1 1 1  2 
min  +  = + = 1 f ( x ) < 1 ∀x and ≥ −1    2 ≤ 2
 x y 2 2 x +1 

Chapter 21.indd 949 12-06-2018 19:17:38


950 Mathematics Problem Book for JEE

Therefore,  d2z  2
−1 ≤ f ( x ) < 1 Now,  2 = =2
 dx  x =1 1
Hence, f ( x ) has minimum value −1 and also there is no
maximum value. which is positive. Hence, x = 1 is the point of minima and
x = 1 and y = 1. Therefore, minimum value is
Aliter: We have x+y =2

( x 2 + 1)2 x − ( x 2 − 1)2 x 4x 114.  If x / (1+ x tan x ) is maxima, then its reciprocal
f ′( x ) = 2 2
= 2
( x + 1) ( x + 1)2
1+ x tan x
f ′( x ) = 0 ⇒ x = 0 x
is a minima. Let us consider
( x 2 + 1)2 4 − 4 x .2( x 2 + 1)2 x 1+ x tan x
Now, f ′′ ( x ) = 1
( x 2 + 1)4 y= = + tan x
x x
( x 2 + 1)4 − 16 x ( x ) −12 x 2 + 4 Therefore,
= =
( x 2 + 1)3 ( x 2 + 1)3 dy 1
= − 2 + sec2 x
Therefore, f ′′(0) > 0 and there is only one critical point that dx x
has minima. Hence, f ( x ) has the least value at x = 0.
d2 y 2
−1 = + 2sec x sec x tan x
fmin = f (0) = = −1 dx 2 x 3
1
111.  We have On substituting dy/dx = 0, we get
f ( x ) = cos x + cos( 2 x ) 1
− + sec2 x = 0
f '( x ) = − sin x − 2 sin( 2 x ) = 0 x2
Hence, x = 0 is the only solution. 1
⇒ sec2 x =
f ''( x ) = − cos x − 2cos( 2 x ) < 0 at x = 0 x2
Hence, the maxima occurs at x = 0. ⇒ x 2 = cos2 x
⇒ x = cos x
112.  Given that
2
− 2 x +1)sin2 x
Therefore,
y = e(2 x
d2 y 2
For minima or maxima, = + 2sec2 x tan x = 2sec2 x (sec x + tan x )
dx 2 cos3 x
dy
=0 which is positive. At x = cos x ,
dx
1+ x tan x
Therefore, x
2
− 2 x +1)sin2 x
e(2 x [(4 x − 2)sin2 x + 2(2 x 2 − 2 x + 1)sin x cos x ] = 0 is minimum and hence, x / (1+ x tan x ) is maximum.
⇒ [(4 x − 2)sin x + 2(2 x − 2 x + 1)sin x cos x ] = 0
2 2 115.  Let us consider
⇒ 2sin x [(2 x − 1)sin x + (2 x 2 − 2 x + 1)cos x ] = 0 x2 − x +1
y= 2
x + x +1
⇒ sin x = 0
Therefore, y is minimum for sin x = 0 . Thus, the minimum dy ( x 2 + x + 1)(2 x − 1) − ( x 2 − x + 1)(2 x + 1)
⇒ =
value of y is dx ( x 2 + x + 1)2
2
− 2 x +1)(0)
y = e(2 x = e0 = 1
dy 2x2 − 2
113.  We have ⇒ = 2 = 0 ⇒ 2 x 2 − 2 = 0 ⇒ x = −1, + 1
dx ( x + x + 1)2
1
xy = 1⇒ y = Therefore,
x
d 2 y 4( − x 3 + 3 x + 1)
and let z = x + y. Then =
dx 2 ( x 2 + x + 1)3
dz 1
= 1− 2 d2 y
dx x At x = −1 , < 0, the function occupies the maximum
dz 1 dx 2
Now, = 0 ⇒ 1− 2 = 0 d2 y
dx x value; therefore, f ( −1) = 3 and at x = 1 , > 0, the
dx 2
Thus, function occupies the minimum value. Therefore,
d2z 2 1
x = −1, + 1 and = f (1) =
dx 2 x 3 3

Mathematical Problem Book for JEE.indb 950 07-06-2018 13:47:00


Chapter 21 | Applications of Derivatives 951

116.  Let us consider A


log x dy x (1/ x ) − log x 1− log x
y= ⇒ = =
x dx x2 x2

Substituting dy / dx = 0, we get y

1− log x
=0
x2 x
B D C
2
d y −3 x + 2 x log x
⇒ 1− log x = 0 ⇒ x = e and =
dx 2 x4
E
At x = e,
Figure 21.36
d2 y 1
2
= 3 <0 or
2
x = y (2r - y ) (1)
dx −e
The volume of the cone is
Therefore, in [2, ∞), the function p2 = q is maximum and the
minimum value does not exist. 1 1 1
V = p x 2 y = p y (2r - y ) y = p (2ry 2 - y 3 )
3 3 3
117.  We have
dV 1 dV
3 −x 2
4 −x
f ( x ) = x 4 e − x ⇒ f ′( x ) = 4 x e + x e ( −2 x )
2 2
⇒ = p (4 ry − 3 y 2 ) ⇒ =0
dy 3 dy
For maximum value
1 4
2 2 ⇒ p (4 ry - 3 y 2 ) = 0 ⇒ y (4 r - 3 y ) = 0 ⇒ y = r ,0
f ′( x ) = 0 ⇒ 4 x 3 e − x − 2 x 5 e − x = 0 3 3
⇒ x2 = 2 ⇒ x = ± 2 d 2V 1
Now, = p (4 r - 6 y )
Now, dy 2 3
2 2 2 2 Substituting y = (4 / 3)r, we get
f ′′( x ) = 12 x 2e − x + 4 x 2e − x ( −2 x ) − 10 x 4 e − x − 2 x 5e − x ( −2 x )

⇒ f ¢¢( 2 ) = 24 e -2 - 32e -2 - 40e -2 + 32e -2 d 2V 1 æ 4 ö


= p ç 4r - 6 ´ r ÷
dy 2 3 è 3 ø
which is negative. Hence, f ( x ) is maximum at x = 2 and
which holds a negative value. So, the volume of the cone is
therefore the maximum value is 4e -2 . maximum at
118.  Let us consider 4
y= r
æp ö 3
f ( A) = cos A cos B = cos A cos ç - A÷ = cos A sin A
è2 ø Height y 2
⇒ = =
Therefore, Diameter 2r 3

f ¢( A) = cos2 A - sin2 A = cos2 A See Fig. 21.37. Let the diameter of sphere be AE = 2r. Let the
120. 
radius of the cone be x and its height be y. Therefore, AD = y
p p since BD 2 = AD(DE ).
Now, f ¢( A) = 0 Þ cos2 A = 0 ⇒ 2 A = ÞA=
2 4 A
p
and f ¢¢( A) = -2sin2 A = -2sin = -2 (negative)
2
Hence, f ( A) is maximum at p / 4. Therefore, maximum value
is y
p p 1
cos sin =
4 4 2
119.  Let the diameter of the sphere be x
B D C
AE = 2r
Let radius of cone (Fig. 21.36) be x and height be y. There-
fore, AD = y since E
2
BD = AD(DE ) Figure 21.37

Mathematical Problem Book for JEE.indb 951 07-06-2018 13:47:03


952 Mathematics Problem Book for JEE

or x 2 = y (2r − y ) (1) At x = + c b / a, ax + by will be minimum. The minimum


The volume of the cone is value is
1 1 1  a a bc 2 b
V = p x 2 y = p y (2r − y ) y = p (2ry 2 − y 3 ) f  c  = (a)c + = 2c ab
3 3 3  b b c a
dV 1 dV 124.  We have
⇒ = p (4 ry − 3 y 2 ) ⇒ =0 1/ 4
dy 3 dy  c 6 − a2 x 4 
a2 x 4 + b2 y 4 = c 6 ⇒ y =  
1 4  b2 
⇒ p (4 ry − 3 y 2 ) = 0 ⇒ y (4 r − 3 y ) = 0 ⇒ y = r ,0
3 3
Hence,
d 2V 1
Now, = p (4 r − 6 y )  c 6 − a2 x 4 
1/ 4
dy 2 3 f ( x ) = xy = x  
Substituting y = (4 / 3)r, we get  b2 
d 2V 1  4   c 6 x 4 − a2 x 8 
1/ 4
= p  4r − 6 × r  ⇒ f (x) = 
dy 2 3  3   b2 
 
which is of negative value and hence the volume of the On differentiating f ( x ) w.r.t. x, we get
cone is maximum at −3/ 4
4 1  c 6 x 4 − a2 x 8   4 x 3c 6 8 x 7 a2 
y= r f ′( x ) =    2 − 
3 4 b2   b b2 
Height y 4 Substituting f ′( x ) = 0, we get
⇒ = =
Radius r 3 4 x 3c 6 8 x 7 a2
− =0
121.  We have b2 b2
f ( x ) = x + sin x ⇒ f ′( x ) = 1+ cos x c6 c 3/2
⇒ x4 = 2
⇒ x = ± 1/ 4
Now, f ′( x ) = 0 ⇒ 1+ cos x = 0 ⇒ cos x = −1 ⇒ x = p 2a 2 a
c 3/2
and f ′′( x ) = − sin x, f ′′(p ) = 0, f ′′′( x ) = − cos x At x = , f ( x ) is maximum and hence
21/ 4 a
f ′′′(p ) = 1 ≠ 0 1/ 4 1/ 4
 c 3/2   c12 c12   c12  c3
Therefore, neither maximum nor minimum. f  1/ 4  =  2 2 − 2 2  =  2 2  =
2 a   2a b 4a b   4a b  2ab
122.  We have
b b 125.  We have
f ( x ) = ax + ⇒ f ′( x ) = a −
x x2 f ( x ) = 2 x 3 − 15 x 2 + 36 x + 4 ⇒ f ′( x ) = 6 x 2 − 30 x + 36  (1)
We know that for its maximum value, f ′( x ) = 0.
b 6 x 2 − 30 x + 36 = 0 ⇒ ( x − 2)( x − 3) = 0 ⇒ x = 2,3
⇒ f ′( x ) = 0 ⇒ x =
a On differentiating Eq. (1), we get
b f ′′( x ) = 12 x − 30
2b
Now, f ′′( x ) = which implies that at x = , f ′′( x ) is
x 3 a ⇒ f ′′(2) = 24 − 30 = −6 < 0
positive. Therefore, f ( x ) has the least value at Therefore, f ( x ) is maximum at x = 2.
b 126.  We have
x=
a y = f ( x ) = − x 3 + 3 x 2 + 9 x − 27

123.  We have The slope of this curve is


c2 bc 2 f ′( x ) = −3 x 2 + 6 x + 9
xy = c 2 ⇒ y = ⇒ f ( x ) = ax + by = ax +
x x
Let us consider
Differentiating w.r.t. x, we get
g( x ) = f ′( x ) = −3 x 2 + 6 x + 9
bc 2
f ′( x ) = a − 2
x On differentiating w.r.t. x, we get
Substituting f ′( x ) = 0, we get g ′( x ) = −6 x + 6
2 2
ax − bc = 0 Substituting
bc 2
g ′( x ) = 0 ⇒ x = 1
⇒ x2 = ⇒ x = ± c b/a
a Now, g′′( x ) = −6 < 0

Chapter 21.indd 952 10-06-2018 18:31:11


Chapter 21 | Applications of Derivatives 953

and hence at x = 1, g( x ), that is, the slope will have its Length of the arc is
maximum value. Therefore, s
[ g(1)]max. = -3 ´ 1+ 6 + 9 = 12 s = rq or q =
r
Therefore, the perimeter of the sector is
127.  We have
f ( x ) = 2 x 3 - 3 x 2 - 12 x + 4 p = r + s + r = 2r + s (2)
s
and f ¢( x ) = 6 x 2 - 6 x - 12 Substituting q = in Eq. (1), we have
r
Now, f ¢( x ) = 0 ⇒ x 2 - x - 2 = 0 ⇒ x = 2, - 1 æ 1 ö æ sö 1 2A
A = ç r 2 ÷ ç ÷ = rs ⇒ s =
and f ¢¢( x ) = 12 x - 6 è2 ø è rø 2 r

That is f ′′(2), which is positive and f ¢¢( -1) is negative. Now, substituting the value of s in Eq. (2), we get
Therefore, the given function has one maximum and one æ 2Aö
p = 2r + ç ÷ or 2 A = pr - 2r 2
minimum. è r ø
128.  We have Differentiating w.r.t. r, we get
1 dA
f (x) = 2 2 = p - 4r
4 x + 2x +1 dr
-(8 x + 2) We know that for the maximum value of area is
⇒ f ¢( x ) =
(4 x 2 + 2 x + 1)2 dA p
= 0 or p - 4 r = 0 or r =
Substituting f ¢( x ) = 0, we get dr 4
131.  We have
8 x + 2 = 0 ⇒ x = -1 dy a
4 = + 2bx + 1
dx x
-[(4 x 2 + 2 x + 1)2 8 - (8 x + 2)2(4 x 2 + 2 x + 1)(8 x + 2)]
f ¢¢( x ) = æ dy ö
(4 x 2 + 2 x + 1)4 ⇒ç ÷ = a + 2b + 1 = 0
è dx ø x =1
Now, f ¢¢( -1/ 4) is negative (i.e. point of maxima). Therefore,
⇒ a = -2b - 1
1 4 æ dy ö a
f ( -1/ 4)max. = = and = + 4b + 1 = 0
4 × (1/ 16) − 2 × (1/ 4) + 1 3 çè ÷ø
dx x = 2 2
129.  We have −2b − 1
⇒ + 4b + 1 = 0
1 1 2
f ( x ) = x + ⇒ f ′( x ) = 1 −
x x2 1
⇒ -b + 4b + =0
Substituting f ′( x ) = 0, we get x = −1, 1. Since x > 0, no 2
maximum value can be found. -1 -1
⇒ 3b = ⇒ b=
130.  See Fig. 21.38. 2 6

A 1 -2
and a = - 1=
3 3
r 132.  We have
x
f (x) = ò (t 4 - 4)e -4 t dt ⇒ f ¢( x ) = ( x 4 - 4)e -4 x
s -10
O θ
Now, f ¢( x ) = 0 Þ x = ± 2 , ± 2

r and f ¢¢( x ) = - 4( x 4 - 4)e -4 x + 4 x 3e -4 x


At x = 2 and x = − 2 , the given function has extreme
B value.

Figure 21.38 133.  We have


The perimeter of a sector is p. Let AOB be the sector with f ( x ) = x 2 log x ⇒ f ¢( x ) = (2log x + 1) x
radius r. If the angle of the sector be q  radians, then the area Now, f ¢( x ) = 0 ⇒ x = e -1/2 ,0
of the sector is
1 Since 0 < e -1/2 < 1 and none of these critical points lies in
A = r 2q (1) the interval [1, e], we only complete the value of f ( x ) at the
2

Mathematical Problem Book for JEE.indb 953 07-06-2018 13:47:08


954 Mathematics Problem Book for JEE

2 3
end points 1 and e. We have f (1) = 0, f (e ) = e . Therefore, the 2 x 3 - 250 = 0 ⇒ x = 125 ⇒ x = 5
greatest value is e 2 . Also

134.  We have d2 y 500


2
= f ¢¢( x ) = 2 + 3
f ( x ) = y = x − x ⇒ log y = - x log x dx x
Differentiating w.r.t. x, Now,
500
1 dy é 1 ù f ¢¢(5) = 2 + >0
. = - ê x . + log x ú 125
y dx ë x û
Hence, at x = 5 , the function will be minimum. The minimum
1 dy value is
⇒ . = -[1+ log x ]
y dx
f (5) = 25 + 50 = 75
dy 139.  We have y = x 1/ x. Taking log on both side, we have
⇒ = - x - x [1+ log x ]
dx
1
dy  1  log y = log x
⇒ = x − x log − 1 x
dx  x 
Differentiating on both sides w.r.t. x, we get
Substituting dy/dx = 0,
1 dy 1 log x
1 = - 2
= loge e
loge y dx x 2 x
x
1 1 dy 1
⇒ =eÞ x = ⇒ = (1- log x ) x 1/ x
x e dx x 2
135.  We have For maximum, we need to have
2a
ab = 2a + 3b ⇒ (a - 3)b = 2a ⇒ b = dy
a-3 =0 ⇒ x=e
2a 2 dx
Now, z = ab =
a-3 Therefore,
dz 2[(a - 3)2a - a2 ] 2[a2 - 6a] ymax = e1/ e
⇒ = =
da (a - 3)2 (a - 3)2
140.  Let us consider
Substituting dz/da = 0, we get
f ( x ) = 4 e 2 x + 9e -2 x
2
a - 6a = 0
Therefore,
d2z
That is, a = 0,6. Now, at a = 6, is positive and when f ¢( x ) = 8e 2 x - 18e -2 x
da2
a = 6, b = 4 Substituting f ¢( x ) = 0, we get
(ab)min = 6 × 4 = 24
8e 2 x - 18e -2 x = 0
136.  Let PQ = a and PR = b. Then
Taking log both sides and solving, we have
1
D = ab sinq e 2 x = 3/2 Þ x = log(3/2)1/2
2
(As -1 £ sinq £ 1 ). Since, the area is maximum when sinq = 1, Also
we have
p f ¢¢( x ) = 16e 2 x + 36e -2 x > 0
q=
2 Now,
137.  We have 3 2
= 4´ +9´
1/2 1/2

y = a (1- cos x ) ⇒ y ¢ = a sin x f (log(3/2)1/2 ) = 4 e 2.(log(3/2) )


+ 9e -2(log(3/2) )
2 3
⇒ y ¢ = 0 Þ sin x = 0 ⇒ x = 0, p = 6 + 6 = 12
Now, y ′′ = a cos x ⇒ y ¢¢(0) = a and y ¢¢(p ) = - a Hence, the minimum value is 12.
Hence, y is maximum when x = p .
141.  Let a point on the curve be (h, k). Then
138.  Let us consider h2 = 2k (1)
æ 250 ö The distance is
y = f ( x) = ç x2 + ÷
è x ø
D = h2 + (k − 5)2
dy 250
and = f ′( x ) = 2 x − 2 From Eq. (1),
dx x
Substituting f ′( x ) = 0, we get D = 2k + (k - 5)2

Mathematical Problem Book for JEE.indb 954 07-06-2018 13:47:10


Chapter 21 | Applications of Derivatives 955

dD 1 Also, 1 < x < 3. Therefore, R = (5 / 3,0) .


= ´ 2(k - 5) + 2 = 0 x Î( -1, ¥ )
dk 2 2k + (k - 5)2
145.  We have
So, at k = 4 , the function D must be minimum. Then, the f ( x ) = 1+ 2 x 2 + 22 x 4 + 23 x 6 + ...210 x 20
point is ( ± 2 2 , 4) .
f ′( x ) = x [4 + 4(2)2 x 2 +  + 20(2)10 x 18 ]
142.  We have
8- x Therefore,
x + 2 y = 8, y =
2 f ¢( x ) = 0 ⇒ x = 0
2
(8 - x ) x Also, f ¢¢(0) > 0.
Now, f ( x ) = xy = x . = 4x -
2 2 146.  Here,
And f ′( x ) = 4 − x
x2 − 3x 2 -1 æ 2 ö
For extremum, we need to have f ′( x ) = 0. Therefore, x = 4 f (x) = ⇒ - sin c = - Þ c = sin ç ÷
x −1 p èpø
and y = 2. Also,
Obviously, it is not derivable at x = 1 , that is, in (0,3). Also,
f ¢¢( x ) = -1 < 0
So, the maximum value of xy is 4 × 2 = 8. f (a) = f (b ) does not hold for [ −3,0] and [1. 5, 3]. Hence the
answer is (D).
143.  We have
147.  Here,
f (a) = 2a2 - 3a + 10
f (b ) − f ( a )
f ¢( a ) = 4 a - 3 = f '(c )
b−a
f ¢¢(a) = 4 > 0
eb - e a
⇒ = f '(c )
For extremum, b-a
3 e -1 c
f ¢( a ) = 0 Þ a = ⇒ = e Þ c = log(e - 1).
4 1- 0

Therefore, f (a) is minimum at a = 3 / 4. 148.  We have


ì - x , when –1 £ x < 0
æ 3ö
2
æ 3ö 71 f (x) = í
f (a)min = 2 ´ ç ÷ - 3 ´ ç ÷ + 10 = î x , when 0 £ x £ 1
è 4ø è 4ø 8
Clearly, f ( -1) =| -1|= 1 = f (1) . However,
144. 
Let the coordinates of R be (x, 0). Given that P (1,1) and f (0 + h) - f (0) |h| h
Rf ¢(0) = lim = lim = lim = 1
Q (3,2). h® 0 h h ® 0 h h → 0 h

PR + RQ = ( x - 1)2 + (0 - 1)2 + ( x - 3)2 + (0 - 2)2 f (0 - h) - f (0) | -h | h


Lf ¢(0) = lim = lim = lim = −1
h® 0 h h® 0 - h h→ 0 − h
= x 2 - 2 x + 2 + x 2 - 6 x + 13 Therefore,
For minimum value of PR + RQ, we get Rf ¢(0) ¹ Lf ¢(0)
d
(PR + RQ ) = 0 Hence, it is not differentiable on ( −1, 1).
dx
149.  We know that
d d
⇒ ( x 2 - 2 x + 2 ) + ( x 2 - 6 x + 13) = 0 f (b ) - f ( a )
dx dx f ¢( c ) =
b-a
( x - 1) ( x - 3)
⇒ =- 0 -1 2
2 2
x - 2x + 2 x - 6 x + 13 Þ f ¢( c ) = =-
p /2 p (1)
Squaring both sides, we get
However,
( x − 1)2 ( x − 3)2 f ¢( x ) = - sin x Þ f ¢(c ) = - sin c (2)
2
= 2
( x − 2 x + 2) x − 6 x + 13
From Eqs. (1) and (2), we get
⇒ 3x2 - 2x - 5 = 0 2 æ 2ö
- sin c = - Þ c = sin-1 ç ÷
⇒ (3 x - 5)( x + 1) = 0 p èp ø
That is, 150.  We have
5 −1
x = , -1 f ′( x1) =
3 x12

Mathematical Problem Book for JEE.indb 955 07-06-2018 13:47:13


956 Mathematics Problem Book for JEE

Therefore, 1 3
a = 0, f (a) = 0 ⇒ b = , f (b ) =
−1 (1/ b ) − (1/ a) 1 2 8
= = − ⇒ x1 = ab
x12 b−a ab f ′( x ) = ( x − 1)( x − 2) + x ( x − 2) + x ( x − 1)
151.  We have f ′(c ) = (c − 1)(c − 2) + c (c − 2) + c (c − 1)
1 1
f ′( x ) = 1 − 2 ⇒ f ′( c ) = 1 − 2 = c 2 − 3c + 2 + c 2 − 2c + c 2 − c
x c
That is,
1 (10 / 3) − 2 1 2
1− 2 = ⇒ 1− 2 = ⇒ c 2 = 3 f' (c ) = 3c 2 − 6c + 2
c 2 c 3
According to mean value theorem, we have
⇒c = 3
f (b ) − f ( a )
152. 
According to mean value theorem, in an interval [a, b] for f(x), f ′( c ) =
b−a
we have
f (b ) − f ( a ) (3 / 8) − 0 3
= f ′( c ) ⇒ 3c 2 − 6c + 2 = =
b−a (1/ 2) − 0 4
5
where a < c < b. Therefore, a < x1 < b . ⇒ 3c 2 − 6c + =0
4
153.  We have 6 ± 36 − 15 6 ± 21 21
c= = = 1±
f ( x ) = e −2 x sin2 x 2×3 6 6

⇒ f ′( x ) = 2e −2 x (cos2 x − sin2 x ) 158.  Given that the equation of curve is


Now, y = x3 = f ( x)

f ′( c ) = 0 So, f (2) = 8 and f ( −2) = −8. Now,


p f (2) − f ( −2)
⇒ cos2c − sin2c = 0 ⇒ tan2c = 1 ⇒ c = f' ( x ) = 3 x 2 ⇒ f '( x ) =
8 2 − ( −2)

From Rolle’s theorem in (1, 26), f (1) = f (26) = 5. In the given


154.  8 − ( −8)
⇒ = 3x2
interval, the function satisfies all conditions of Rolle’s theo- 4
Therefore,
rem. Therefore, in [1, 26], at least, there is a point for which
2
f ′( x ) = 0. x=±
3
2 159.  We have
∫1 f ′( x )dx = [f ( x )]1 = f (2) − f (1) = 0
2
155. 
We have because f ( x )
f (x) = x
satisfies the conditions of Rolle’s theorem. Therefore,
Therefore,
f (2) = f (1)] ).
f (a) = 4 = 2, f (b ) = 9 = 3
156.  We have
1
Now, f ′( x ) =
f ( x ) = x 3 − 6 x 2 + ax + b 2 x
⇒ f ′( x ) = 3 x 2 − 12 x + a f (b ) − f ( a ) 3 − 2 1
Also, f ′( c ) = = =
b−a 9−4 5
 1 
⇒ f ′( c ) = 0 ⇒ f ′  2 +  =0 Therefore,
 3
1 1 25
2 = ⇒ c= = 6.25
 1   1  2 c 5 4
⇒ 3 2 +  − 12  2 +  +a=0
 3 3
Practice Exercise 2
 1 4   1
⇒ 3 4 + +  − 12  2 +  +a=0
 3 3 3 dx 2 −2
 1.  = (− cosec2 t) =
dt cot t sin t cos t
That is,
12 + 1+ 4 3 − 24 − 4 3 + a = 0 ⇒ a = 11 dy sin2 t − cos2 t
= sec2 t − cosec2 t =
dt sin2 t cos2 t
157.  From mean value theorem, Now,
f (b ) − f ( a ) dx p −2
f ′( c ) = at  t =  = =−4
b−a dt  4 1/ 2

Chapter 21.indd 956 10-06-2018 18:31:40


Chapter 21 | Applications of Derivatives 957

dy  p  a + 1
at  t =  = 2 − 2 = 0 ⇒ (a + 1)2 3 –  =0
dt  4  a 

dy dx y
Here, =0⇒− =∞
dx dy
P(α , β )
 2. g(x) is increasing and f(x) is decreasing. So,
  g(x + 1) > g(x − 1) and f(x + 1) < f(x − 1)
x
⇒ f {g(x + 1)} < f {g(x − 1)} and g {f(x + 1)} < g {f(x − 1)} −1 O (0, 0)

 3.  f(x) = x3 − x2 + 100x + 1001


f ′(x) = 3x2 − 2x + 100 > 0 ∀ x ∈ R
Therefore, f(x) is increasing (strictly).
Figure 21.39
Therefore,
⇒ a = −1
 1  æ 1 ö
f  > fç ÷ 1
 1999  è 2000 ø or a=
2
⇒ f(x + 1) > f(x − 1) Therefore, equation of tangent is
  4.  xy = (a + x)2 y=0
y + xy′ = 2(a + x) 27
y′ = ±1 or y = x
4
y ± x = 2(a + x)
  7.  See Fig. 21.40.
(a + x )2 (a + x )2
± x = 2(a + x) ⇒ ±x = 2(a + x) − f(x) = ax3 + bx2 + cx + d
x x
± x2 = (2 + x) [x − a] ⇒ ± x2 = x2 − a2 Now, f(x) is odd. Therefore,
a    f(−x) = − f(x)
2x2 = a2 ⇒ x = ±
2 ⇒ −ax3 − bx2 − cx − d = −ax3 + bx2 − cx + d
In x It gives b = 0 = d
 5.  f(x) = (1)
x
f(x) = ax3 + cx = x (ax2 + c)
Since domain is R+, we have
Therefore,
1
x × - In x f ′(x) = 3ax2 + c = 0
x 1- In x
f ′(x) = 2
= c
x x2 Only when x2 = − is positive.
3a
(A)  For horizontal tangent
Therefore, c and a are of different signs.
f ′(x) = 0 ⇒ In x = 1 ⇒ x = e(True)
(B)  If Eq. (1) cuts the x−axis, then c
Let − = k.
In x a
= 0 ⇒ x = 1 (True)
x
(C)  f  ′(x) is +ve if x ∈ (0, e) and f ′(x) is −ve if x ∈ (e, ∞). Therefore,
f(x) is not monotonic.
0 k k
Hence, f(x) is many−one.  (True)
(D)  For vertical tangent f ′(x) = ∞, so 3
1- In x x2 Figure 21.40
2 =∞⇒ =0 ⇒x=0
x 1- In x So, non−zero root of f(x) is ± k .
which is not in the domain of f(x).(False)
  6.  See Fig. 21.39. k
Also ± is closer to origin than ± k .
Note: Curve is not passing through origin. 3
Let (a, b ) be the point of contact. Then
 8.  f(x) = (x − 1)4 (x − 2)n , n ∈ N(1)
æ dy ö b –0 b
ç ÷ = a – 0 ⇒ 3(a + 1) = a (1)
2
Therefore,
è dx øP
Also (a, b ) lies on the curve. Therefore, f ′(x) = 4 (x − 1)3 (x − 2)n + (x − 1)4 n (x − 2)n − 1
(a + 1)3 = (x − 1)3 (x − 2)n − 1 (4x − 8 + nx − n)
3(a + 1)2 =
a = (x − 1)3 (x − 2)n − 1 [(n + 4) x − (n + 8)]

Mathematical Problem Book for JEE.indb 957 07-06-2018 13:47:17


958 Mathematics Problem Book for JEE

If n is odd, then f ′(x) > 0 if x < 1 and sufficiently close to 1 and 12.
f ′(x) < 0 if x > 1 and sufficiently close to 1. Therefore, x = 1 is
y
point of local maximum.
y=x
Similarly, if n is even, then x = 1 is a point of local minimum.
Further if n is even, then f ′(x) < 0 for x < 2 and sufficiently close
to 2 and f ′(x) > 0 for x > 2 and sufficiently close to 2. x
(0, 0)
Therefore, x = 2 is a point of local minimum. y = ke x

da
 9.  = 2 ⇒ a = 2t + c Figure 21.41
dt
With the help of graph shown in Fig. 21.41, we can see that
Since c = 0 {Because a = 0, when t = 0}
there is only one solution.
Therefore, 13. Consider y = kex and y = x.
a = 2t Let (a, kea ) be a point on y = kex.
Therefore, the curve y = x − 2ax + a2 + a becomes
2 If it lies on y = x also, then α = kea.
Now,
y = x2 − 4tx + 4t2 + 2t dy
= kex
If x = 0, then dx
Therefore,
y = 4t2 + 2t
dy
= kea = a = 1
Now, dx
x =a
dy y = x is tangent to y = kex at one point.
= 2x − 4t
dx Therefore,
1 = ke
Therefore, That is,
dy k = 1/e
= − 4t
dx at x = 0 14.  See Fig. 21.42. Consider y = kex and y = x.
x
Therefore, equation of the tangent is From the above question, ex =
k
y − (4t2 + 2t) = − 4t (x − 0) 1 x
If we decrease the value of k from , then slope of y =
That is, e k
increases.
y = −4t x + 4t2 + 2t x
Therefore, y = ex and y = intersect at two distinct points
Vertex of y = x − 4t x + 4t + 2t is (2t, 2t).
2 2 k
(See Fig. 21.42).
Therefore, distance of vertex from the origin = 2 2 t. y

Therefore, rate of change of distance of vertex from origin


with respect to t = 2 2 .
That is, k = 2 2 x
O
10.  c(t) = 4t2 + 2t
Therefore,
dc Figure 21.42
= 8t + 2
dt æ 1ö
Therefore, k ∈ ç 0, ÷
dc è eø
Þ = 16 2 + 2
dt at t = 2 2 15. Let 0 < a < b < 1, and a, b are the roots of f (x) = x3 − 3x + k = 0.
Then
⇒ l = 16 2 + 2
f(a ) = f(b ) = 0
11.  m(t) = −4t ⇒ f(x) satisfies RMVT
Therefore, ⇒ f ′(c)  = 0
dm ⇒ 3c2 = 3
= −4
dt ⇒ c = ±1
dm
Þ = −4 But c must be lies between a and b.
dt at t = l
Hence, k ∈ f .

Mathematical Problem Book for JEE.indb 958 07-06-2018 13:47:18


Chapter 21 | Applications of Derivatives 959

16. Let f(x) = tan−1x. Then for some a ∈ (x, y), we have y


-1 -1
tan y - tan x B
f ′(a) = (LMVT)
y-x θ P (1, 2)
L 1
-1 -1
1 tan x - tan y æ 1 ö 2
⇒ 2 = ç 2
£ 1÷ θ
1+ a x-y è 1+ a ø x
O M A
⇒ |tan−1x − tan−1y| ≤ |x − y|
Figure 21.43
17. Let f(x) = sinx and g(x) = cos x.
BP = sec q
p
Also, sin x ≠ 0 for x ∈ æç 0 , ö÷ AB = AP + BP = 2 cosec q + sec q
è 2ø
Therefore, minimum value of AB = (22/3 + 1)3/2.
Then, by Cauchy’s theorem, we have
20. Let y − 2 = m (x − 1) be a focal chord. Then
f ( b ) - f (a ) f ¢(q ) y = mx + 2 − m
=
g( b ) - g(a ) g¢(q ) ⇒ (mx + 2 − m) − 4x − 4 (mx + 2 − m) + 4 = 0
2

sin b - sina cosq That is, (mx − m)2 − 4x = 0


⇒ = = − cot q
cos b - cos a - sinq ⇒ m2x2 − (2 m2 + 4) x + m2 = 0
18.  Putting x = 9, y = 0 in the given equation of curve, we have Now,
2( m2 + 2)
1 a x1 + x2 = ; x1 x2 = 1
0 = 3a + 9b − = +b m2
2 2 ´ 3
⇒ a = −3b(1) |x2 − x1| = ( x1 + x 2 )2 - 4 x1x 2

dy a ( m2 + 2)2 2
= +b 4 - 4 = 2 m 4 + 4 m2 + 4 - m 4
dx 2 x =
m4 m
dy a 1 4
= + b = − (2) |x2 − x1| = m2 + 1
dx (9, 0) 6 2 m2
4
Length of diagonal = ( x 2 - x1)2 + m2 ( x 2 - x1)2 = (m2 + 1)
Using Eqs. (1) and (2), we get m2
b = − 1 and a = 3 Length of diagonal of perpendicular chord is
Therefore,
 1 
y=3 x −x 4m2  2 + 1 = 4 (1 + m2)
m 
Point (1, 2) lies on curve as well as it is point of intersection of
family of lines.  2 1 
1. 4
dy 3 Area = 2
(1 + m2) 4 (1 + m2) = 8  m + 2 + 2 
= −1 2 m  m 
dx 2 x
⇒ Minimum area = 16 + 8 × 2 = 32
dy 1
at (1, 2) is
dx 2 21.  Graph of y = f(x). See Fig. 21.44.

1 (−2, f(−2))
y−2= (x − 1)
2 Y
y=3
⇒ x − 2y + 3 = 0

19.  See Fig. 21.43. X


x = −2 x=3
2
sinq =
PA
PA = 2 cosec q
1
cos q =
BP Figure 21.44

Mathematical Problem Book for JEE.indb 959 07-06-2018 13:47:20


960 Mathematics Problem Book for JEE

(−2, f(−2)) 25.  See Fig. 21.48.

dy 4t
y = |x | (A)  =
dx 3
4t
Tangent is y − at4 = (x − at3)
3
at 3 at 4
x-intercept = ; y-intercept = −
4 3
If P divides AB in the ratio l:1, we have
Figure 21.45 at 3
l ⋅0 +
Graph of f(x) = |x| has 3 points of intersection, so equation has at3 = 4 ⇒ l = −3
3 solutions (See Fig. 21.45.). Hence, (C) is the correct answer. l +1 4

22.  See Fig. 21.46. Therefore,


m 3
(−2, f(−2)) (2, f(2)) =−
n 4

y
P
(at 3, at 4)

B
x
Figure 21.46 O
 at 
4  at 3 
Hence, (B) is the correct answer. 0, −  A  ,0
3  
  4 
23.  See Fig. 21.47. Figure 21.48
(−2, f(−2)) ⇒ m = 3, n = 4
⇒m+n=7

dx
(B)  = esiny cos y; slope of normal = −1
dy
−2
Equation of normal is x + y = 1.

y = −3x 1
Area =
2
1 dy 2
Figure 21.47 (C)  y = : = − 3 : slope of tangent = −2
x 2 dx x
Hence, (D) is the correct answer.
dy
24. (A) r = 5 cm, d dr = 0.06 y = e2−2x : = e2−2x⋅(−2) : slope of tangent = −2
dx
A = p r2dA = 2p rd r = 10p × 0.06 = 0.6p
Therefore,
(B)  v = x3, d v = 3x2dx
tan q = 0
δv δx
× 100 = 3 × 100 = 3 × 1 = 3
v x
y be x /3
(D)  Length of sub−tangent = = =3
dx dx y′ 1
(C)  (x − 2) =2 ⇒x=4 b e x /3
dt dt 3
26. (A) Using LMVT, we get
3 2
(D)  A = x g(5) − g(0) −1/ 6 − 4 5
4 g′(c)  = = =−
5 5 6
Now,
(B)  Let f(x) = f(x) − 2g(x), x ∈ [0, 1]. Then
dA 3 dx 3 1 3 3
= × = ⋅15⋅ =
dt 2 dt 2 10 4 f(0) = 2, f(1) = 6 − 2g(1)

Chapter 21.indd 960 10-06-2018 18:33:42


Chapter 21 | Applications of Derivatives 961

Now, f ′(x) = f ′(x) − 2g ′(x) ⇒f(x) satisfies condition of Therefore,


Rolle’s theorem on [0, 1], so 1 1
ab < ⇒ >4
f(0) = f(1) ⇒ 2 = 6 − 2g(1) 4 ab

⇒ g(1) = 2
(C)  f(x) = sin 3x −2 −1
π π π
Clearly, longest length = e −  −  =
6  6  3 Figure 21.49
Therefore, l = 3. Therefore,
 1 1
f (5) − f (1) −c − 6  1+   1+  ≥ 1+ 8 = 3
(D)  f ′(c)  = ⇒ = ⇒ c2 = 15  a b
5 −1 25 − c 2 2 (C)  y = 10 − (10 − x) = x
Therefore, the maximum value is y = 3.
27. (A)  f ( x ) = ln(sin x )
(D)  Equation of tangent at P is ty = x + t2.
cos x It intersects the line x = 0 at Q. Therefore, coordinates of Q
f ′(x) = >0
sin x are (0, t). Therefore,
Therefore, the required number of values of x is 0. 0 t 1
1 1
Area of ∆PQS =
 1 0 1 = [− t (1 − t2) + 2t]
(B)  f ′(x) = 3x − 3 ≤ 0 if −1 ≤ x ≤ 1
2
2 2
Therefore, a = −1, b = 1 t2 2t 1
1
Therefore, a + b = 0 = (t + t3)
2
Now,

 x 2 + 2, 1 ≤ x < 2 dA 1 2
 2 = (3t + 1) > 0 ∀ t ∈ [0, 2]
dt 2
x +2
(C)  f(x) =  , 2≤ x <3 Therefore, area is maximum for t = 2.
 2
 x2 + 2 Hence,
 , x =3 1
 3 maximum area = [2 + 8] = 5
2
2 x , 1 < x < 2
⇒ f ′(x) =  29.  For the points of intersection, we have
x, 2 < x < 3
12 − y 2 y2
Therefore, least value of f(x) in [1, 2) is 3. + =1
36 4
Least value of f(x) in [2, 3) is 3. So,
⇒ y = ± 3 and x = ±3
11
f(3) = Consider the point P (3, 3 ). Equation of the tangent at P to
3
Therefore, the least value of f(x) is 3. the circle is 3x + 3 y = 12.
(D)  f(x) = e2x − (a + 1) ex + 2x Therefore, slope of this tangent is − 3 .
f ′(x) = 2e2x − (a + 1) ex + 2 Equation of the tangent at P to the ellipse is
Now, x 3
+ y=1
  2e2x − (a + 1) ex + 2 ≥ 0 for all x ∈ R 12 4
 1 1
⇒ 2  e x + x  − (a + 1) ≥ 0 for all x ∈ R
 e  Therefore, slope of this tangent is  − .
3 3
⇒ 4 − (a + 1) ∈ 0
⇒a∈3 If a  is angle between these tangents, then
⇒a=3 2
tan a =
28. 
(A) See Fig. 21.49. By graph, it is clear that at x = −1 is local 3
max. and x = 0 is local min. Therefore,
(B)  a + b = 1 2
a = tan−1
3
 1 1 1 1 1 2
 1+   1+  e = 1+ + + = 1+ Therefore,
 a b a b ab ab
k=4
a+b 1 Hence,
ab < =
2 2 k2 = 16

Chapter 21.indd 961 10-06-2018 18:33:45


962 Mathematics Problem Book for JEE

30.  See Fig. 21.50.


2
 9 
d 2 = (x1 − x2)2 +  2 − x12 −  3
 x2 
Let y12 = 2 − x12. Then

x12 + y12 = 2 Figure 21.52


9 2π
y2 = ⇒ x2y2 = 9 ⇒ Vmax =
x2 3
Therefore, l = 2.

33. See Fig. 21.53. If a < amin, then the curve y = | x − a | will not
3)

(1, 1)
,
(3

intersect the curve y = −x2 + 3.


Similarly, if a > amax, then the curve y = |x − a| will not intersect
d
(
the curve y = 3 − x2 for any x ∈ − 3, 0 . )

L1 L2
(0, 3)
Figure 21.50
amin amax
d → shortest distance between two curves will be along
the common normal y = x
Therefore, Figure 21.53
d 2 = 4 + 4 = 8 Case Ι:
31.  See Fig. 21.51. L1 is tangent to y = − x2 + 3 and its equation is y = x − a
f ′(x) = cos x − 2a cos 2x − cos 3x + 2a ≥ 0 ∀ x ∈ R Therefore,
⇒ cos x − cos 3x + 2a (1 − cos 2x) ≥ 0 dy
= −2x = 1
⇒ 2 sin2x + sin x + 4a sin2x ≥ 0 dx
That is,
⇒ 2 sin2x (cos x + a) ≥ 0
1
⇒ a ≥ −cos x ⇒ a ≥ 1 x=−
2

 1 11 
Therefore,  − ,  lies on y = x − a
h 3  2 4

r  1 11 
Thus, point of contact is  − ,  .
Figure 21.51  2 4

32.  See Fig. 21.52. Since, it lies on y = x − a. Therefore


3=h +r 2 2
13
⇒ r2 = 3 − h2 amin = −
4
Now, 13
So, the inequality has a negative solution if − < a < 0. (1)
1 1 4
V= p r2 h = p (3 − h2) h
3 3 Case ΙΙ:
Therefore, Line L2 is y = a − x and passes through (0, 3) if a = 3.

dV 1 Thus, the inequality has a negative solution if − 3 < a < 3. (2)
= p (3 − 3h2)
dh 3 From Eqs. (1) and (2), we get that the inequation has at least
13
dV one negative solution if − < a < 3.
= 0 at h = 1 4
dh
ïì xe
ax
, x £0
d 2V 34.  f(x) = í
< 0 at h = 1 2 3
dh2 ïî x + ax - x , x > 0

Chapter 21.indd 962 10-06-2018 18:33:47


Chapter 21 | Applications of Derivatives 963

Now,
ïì e + axe
ax ax
, x £0
⇒ f ′(x) = í h(x) = 0 and f(x) = 0
2
ïî1 + 2 ax - 3 x , x >0
or f ′(x) = 0
and As f(x) = 0 has 4 minimum solutions and f ′(x) = 0 has minimum
ïì(2a)e + a xe , x £ 0
ax 2 ax 3 solutions, h(x) = 0 has minimum 7 solutions and h′(x) = g(x) =
f ′′ (x) = í 0 has minimum 6 solutions.
ïî 2a - 6 x , x >0
37. Given
For x < 0, y = −p2x2 + 5 px − 4 (1)
f ′′(x) > 0 1
y= (2)
⇒ (2a + a2x) eax > 0 (1− x )
−2 Chord touches curve (2) at x = 2 which gives y = −1.
⇒ a (2 + ax) > 0 ⇒ x > e
a Let (x1, y1) and (x2, y2) are ends of chord.
a æ x1 + x 2 y1 + y 2 ö
For x > 0, 2a − 6x > 0, x < . Touching point is middle point ç , ÷
3 è 2 2 ø
Also f is continuous at 0.
x1 + x 2
æ 2 aö = 2; x1 + x2 = 4
Therefore, f(x) is increasing in ç - , ÷ . 2
è a 3ø
and y1 + y2 = −2
1
35.  x = − 1 and x = are roots of f ′(x) = 0. Therefore, (x1, y1) and (x2, y2) satisfy the curve.
3
f ′(x) = a(3x − 1) (x + 1) = a(3x2 + 2x − 1) Therefore,
⇒ f(x) = a (x3 + x2 − x + b) y1 = −p2x21 + 5px1 − 4 (3)
f(−2) = 0 ⇒ b = 2 ⇒ f(x) = a (x3 + x2 − x + 2)
and y2 = −p x + 5px2 − 4
2 2
2 (4)
1 1
14 14
∫ f ( x ) dx = 3 ⇒ ò a( x + x - x + 2) = 3
3 2 Subtracting Eq. (4) from Eq. (3), we get
−1 -1 y1 − y2 = − p2x21 + 5px1 − 4 + p2x22 − 5px2 + 4
1 14 æ 1 ö 14 ⇒ y1 − y2 = −p2(x21 − x22) + 5p(x1 − x2)
⇒a +2 = ⇒ 2a ç + 2 ÷ = ⇒a=1
òx
2
3 è3 ø 3
-1 y1 − y 2
⇒ = − p2 (x1+ x2) + 5p = −4p2 + 5p (5)
Therefore, x1 − x 2
f(x) = x3 + x2 − x + 2
d æ dy ö 1 1
36.  g(x) = (f(x)⋅f ′(x)) Again, ç ÷ = =
dx è dx øat x =2 (1− x )2 1
To get the zero of g(x), we take function Therefore, from Eq. (5), we get
h(x) = f(x)⋅ f ′(x) 1 = −4p2 + 5p
Between any two roots of h(x), there lies at least one root of ⇒ 4p2 − 5p + 1 = 0
h′(x) = 0. That is, 1
⇒ p = 1,
g(x) = 0 4

Mathematical Problem Book for JEE.indb 963 07-06-2018 13:47:28


964 Mathematics Problem Book for JEE

Solved JEE 2017 Questions


JEE Main 2017 Now, the area is
1 2 1  20 − 2r 
1. The normal to the curve y(x – 2)(x – 3) = x + 6 at the point r q = r2  
2 2  r 
where the curve intersects the y-axis passes through the
That is,
point:
1
 1 1  1 1 z = (20r − 2r 2 )
(A)
 ,  (B)  , −  2
2 2  2 3
Differentiating w.r.t. r, we get
 1 1  1 1
(C)
 ,  (D)  − , −  dz 1
2 3  2 2 = (20 − 4 r ) = 0
dr 2
(OFFLINE) ⇒ r=5
Solution: It is given that d2z
At r = 5, we get q = 2; therefore, < 0 (hence, it is maxima).
y(x – 2)(x – 3) = x + 6 dr 2
Therefore, the maximum area is
At y-axis, we know that x = 0. Therefore,
1 1
y(−2)(−3) = 0 + 6 z = r 2q = × 52 × 2 = 25 m2
2 2
Now,
y(x2 – 5x + 6) = x + 6 Hence, the correct answer is option (B).
x+6
⇒y= 3. The tangent at the point (2, −2) to the curve, x2y2 – 2x =
x2 − 5x + 6
4(1 – y) does not pass through the point
Differentiating w.r.t. x, we get
(A) (−2, −7) (B) (−4, −9)
dy [( x 2 − 5 x + 6 )(1)] − [( x + 6 )(2 x − 5)]  1
= (C)
 4 ,  (D) (8, 5)
dx ( x 2 − 5 x + 6 )2 3
At x = 0, we have y = 1 as follows: (ONLINE)
6 − [(6 )( −5)] Solution: The given curve is
y= 2
=1
6 x2y2 – 2x = 4(1 – y)
Therefore, the equation of normal is
Differentiating this equation, we get
y – 1 = −1(x – 0)
dy  − dy 
That is, y + x − 1 = 0 or y + x = 1. 2x2y + 2 xy 2 − 2 = 4 
dx  dx 
 1 1
Thus, the normal to the given curve line passes through  ,  . dy dy
 2 2 ⇒ 2x2y +4 = 2 − 2 xy 2
dx dx
Hence, the correct answer is option (A).
dy
⇒ (2 x 2 y + 4 ) = 2 − 2 xy 2
2. Twenty meters of wire is available for fencing off a flower-bed dx
in the form of a circular sector. Then, the maximum area (in dy 2 − 2 xy 2
sq.m) of the flower-bed, is: ⇒ =
dx 2 x 2 y + 4
(A) 10 (B) 25
(C) 30 (D) 12.5 The tangent at point (2, −2) is
(OFFLINE)
Solution: It is given that r + r + rq = 20 meters. Therefore, dy 2 − 2(2)( −2)2 2 − 2(2)( 4 ) 2 − 16 −14
   = = =
20 − 2r dx ( 2 , −2 ) 2(2)2 ( −2) + 4 2( 4 )( −2) + 4 −16 + 4 −12
q=
r dy 14 7
⇒ = =
dx ( 2 , −2 ) 12 6
r q r The equation of tangent is given by
dy
( y − y1) = ( x − x1)
dx (x,y)

Here, x1 = 2 and y1 = 2. Therefore,


7
rq y − ( −2) = ( x − 2)
6

Mathematical Problem Book for JEE.indb 964 07-06-2018 13:47:52


Chapter 21 | Applications of Derivatives 965

7 Solution of integral has the form


⇒ y + 2 = ( x − 2)
6
x (I.F.) = ∫ Q × (I.F.)dx
⇒ 6(y + 2) = 7(x − 2)
1 1
y ∫
⇒ 6y + 12 = 7x – 14 ⇒x× = 3 y × dx
y
⇒ 7x – 6y = 26 1
y ∫
⇒x× = 3 dy
 1
All the given points (8, 5), (−4, −9) and  4 ,  satisfy the equation
 3
except the point (−2, −7): 1  x n +1 
⇒x× = 3 y + C        since ∫ x ndx = 
y  n + 1
7 x − 6 y ( −2 , −7 ) = 28
⇒ x = 3y2 + Cy
Therefore, the tangent at point (2, −2) of the given curve does not
pass through the point (−2, −7). For point (1, 1): 1 = 3 + C ⇒ C = −2
Hence, the correct answer is option (A). Therefore, x = 3y2 – 2y.
 −1 1
This equation is satisfied by point  ,  .
 3 3
4. The curve satisfying the differential equation, ydx – (x + 3y2)dy
2
= 0 and passing through the point (1, 1) also passes through −1  1  1 −1 1 2 −1 −1
= 3 ×   = 2  ⇒ = − ⇒ =
the point 3  3  3 3 3 3 3 3
 1 1  1 −1 Hence, the correct answer is option (D).
(A)
 ,  (B)  , 
4 2 4 2 
d2 y dy
5. If 2 x = y1/ 5 + y −(1/ 5) and ( x 2 − 1) +l x + ky = 0 , then
 1 −1  −1 1 dx 2 dx
(C)
 ,  (D)  , 
3 3  3 3 l + k is equal to
(ONLINE) (A) −23 (B) −24
(C) 26 (D) −26
Solution: The given differential equation is
(ONLINE)
ydx – (x + 3y2)dy =0 Solution: It is given that
⇒ ydx = (x + 3y2)dy
2x = y1/5 + y−1/5
2
dx x + 3 y ⇒ 2x = y1/5 + 1/y1/5
⇒ =
dy y
Therefore,
dx x
⇒ = + 3y 1
dy y 2x = a + ⇒ a2 + 2ax + 1 = 0
a
dx  1  
⇒ − x = 3y 2x ± 4 x2 − 4 2b ± b2 − 4 ac 
dy  y  a=       since x = 
2  2 
The above differential equation is of the form 2
2x ± 2 x −1
⇒a=
dx 2
+ P ( y ) x = Q( y )
dy ⇒ a = x ± x2 −1
−1
with P( y ) = and Q(y) = 3y. ⇒ y1/ 5 = x ± x 2 − 1
y
To solve the differential equation of this form, let us find the inte- ⇒ y = ( x ± x 2 − 1)5
grating factor: Therefore,
−1
∫ y dy   x2 −1± x 
I.F = e ∫
P . dy
=e = e − log y dy 2x 
= 5( x ± x 2 − 1)4  1±  = 5( x + x 2 − 1)4  
dx  2 x 2 − 1  x 2 − 1 
1
Using standard integral ∫ dy = log y, we get
y dy −5 y
⇒ = (1)
dx x2 −1
  a 
I.F. = elog(1/ y )     since log   = log a − log b
  b   2  dy  1 2x 
 x − 1 −5  − 5( −5 y ) 
1 d y 2  dx  2 x 2 − 1 
       I.F. =
y ⇒ 2=
dx ( x 2 − 1)

Mathematical Problem Book for JEE.indb 965 07-06-2018 13:48:26


966 Mathematics Problem Book for JEE

Therefore, 1
y=
d2 y dy x x3
( x 2 − 1) 2 = −5 x 2 − 1 + 5 y
dx dx 2
x −1 When we substitute the values from the given options, only option
2 (C) satisfies the above equation.
d y dy Hence, the correct answer is option (C).
⇒ ( x 2 − 1) 2
= 25 y − x
dx dx
d2 y dy 7. The function f defined by f(x) = x3 – 3x2 + 5x + 7 is
⇒ ( x 2 − 1) + 1x − 25 y = 0 (A) decreasing in R.
dx 2 dx
(B) increasing in R.
Therefore, l = 1, k = −25; hence, (C) decreasing in (0 , ∞ ) and increasing in ( −∞ , 0 ).
(D) increasing in (0 , ∞ ) and decreasing in ( −∞ , 0 ).
l + k = −24 (ONLINE)
Hence, the correct answer is option (B). Solution: The given function is
    f(x) = x2 – 3x2 + 5x + 7
6. A tangent to the curve, y = f(x) at P(x, y) meets x-axis at A and
y-axis at B. If AP : BP = 1 : 3 and f(1) = 1, then the curve also f ′( x ) = 3 x 2 − 6 x + 5
passes through the point
The discriminant of the above quadratic equation is
1  1 
(A)
 , 4 (B)  , 24 Δ = 36 – 4(3)(5) = 36 – 60 < 0
2 3 
 1  1 Therefore,
(C)
 2,  (D)  3,  f ′( x ) > 0 ∀x ∈R +
8  28 
Also,
(ONLINE)
f ′( x ) > 0 ∀x ∈R −
Solution: We have

( y − y2 ) Therefore, the given function f is increasing in R.


= f ′ ( x1) ⇒ y − y1 = f ′( x1)( x − x1) Hence, the correct answer is option (B).
( x − x1)

− y1 y JEE Advanced 2017


When y = 0: = x − x1 ⇒ x = x1 − 1 .
f ′( x1) f ′( x1) Directions for Questions 1–3: Answer the questions by appropri-
  ately matching the information given in the three columns of the
y
Therefore, point A is A  x1 − 1 , 0 , following table:
 f ′ ( x1 ) 
Let f(x) = x + logex – x logex, x ∈(0 , ∞ ).
When x = 0: y − y1 = f ′( x ) ⋅ ( − x1) ⇒ y = y1 − x1f ′( x1) . • Column 1 contains information about zeros of
Therefore, point B is B(0 , y1 − x1f ′( x1)) . f ( x ), f ′( x ) and f ′′( x ).
Point P divides AB in the ratio 1 : 3. • Column 2 contains information about the limiting behaviour
of f ( x ), f ′( x ) and f ′′( x ) at infinity.
  y1  
3  x1 −  • Column 3 contains information about increasing/decreasing
 f ′ ( x1)  
x1 =  nature of f ( x ) and f ′( x ).
4
y1 − x1f ′( x1) Column 1 Column 2 Column 3
y1 =
4 (I) f (x) = 0 for some    (i) lim f ( x ) = 0 (P) f is increasing
Therefore, x ∈(1, e2 ) x →∞ in (0, 1)
4 y1 = y1 − x1f ′( x1) (II) f ′( x ) = 0 for   (ii) lim f ( x ) = −∞ (Q) f is decreasing
some x ∈(1, e ) x →∞ in (e, e2)
−3 y1 −3 y
⇒ f ′( x1) = ⇒ f ′( x ) =
x1 x (III) f ′( x ) = 0 for  (iii) lim f ′( x ) = −∞ (R) f ′ is increasing
x →∞
some x ∈(0 , 1) in (0, 1)
Now,
dy −3 y dy −3dx (IV) f ′′( x ) = 0 for (iv) lim f ′′( x ) = 0 (S) f ′ is decreasing
= ⇒ = x →∞
dx x y x some x ∈(1, e ) in (e, e2)

On integrating, we get
1. Which of the following options is the only CORRECT
combination?
ln y = −3ln x + C ⇒ y = kx−3
(A) (I) (i) (P) (B) (II) (ii) (Q)
y(1) = 1 ⇒ k = 1 (C) (III) (iii) (R) (D) (IV) (iv) (S)

Mathematical Problem Book for JEE.indb 966 07-06-2018 13:49:08


Chapter 21 | Applications of Derivatives 967

Solution: It is given that (A)


(I) (iii) (P) (B) (II) (iv) (Q)
(C)
(III) (i) (R) (D) (II) (iii) (P)
f(x) = x + logex – xlogex, x ∈(0 , ∞ )
Solution: Following explanation of Question 1, we have the
d d 1 1 following conclusions:
⇒ f ′( x ) = f ( x ) = ( x + log x − x log x ) = 1+ − log x − x ⋅
dx dx x x • In Column 2, option (i) is false.
1 •• In Column 3, options (P) and (R) are false.
  f ′( x ) = − log x
x
Thus, from given options, only (III) (i) (R) combination is INCORRECT.
d d 1  −1 1
⇒ f ′′( x ) = f (x) =  − log x  = 2 − Hence, the correct answer is option (C).
dx dx  x x x
•   lim f ( x ) = lim ( x + log x − x log x ) = −∞ . Hence, option (ii) is 4. If f: R→R is a differentiable function such that f ′( x ) > 2f ( x ) for
x →∞ x →∞ all x ∈R, and f(0) = 1, then
correct. (A) f(x) is increasing in (0 , ∞ ) .
1  (B) f(x) is decreasing in (0 , ∞ ) .
•   lim f ′( x ) = lim  − log x  = −∞. Hence, option (iii) is correct.
x →∞ x →∞  x  f(x) > e2x in (0 , ∞ ) .
(C)
 −1 1 
•   lim f ′′( x ) = lim  2 −  = 0. Hence, option (iv) is correct. f ′( x ) < e2 x in f ′( x ) < e2 x .
(D)
x →∞ x →∞  x x
Also, we have Solution: It is given that f: R→R is differentiable function such that
f ′( x ) > 2f ( x ) for all x ∈R and f(0) = 1.
1 
lim f ′( x ) = lim+  − log x  = ∞
x →0+ x →0  x  f ′( x ) > 2f ( x ) ⇒ f ’ ( x ) − 2f ( x ) > 0

Thus, lim+ f ′( x ) = ∞ and lim f ′( x ) = −∞ implies f ′( x ) is decreas- Multiplying this with e−2x, we get
x →0 x →∞
d −2 x
ing function. e −2 x f ′( x ) − 2e −2 x f ( x ) > 0 ⇒ [e f ( x )] > 0
So, options (Q) and (S) are correct. Therefore, from the given dx
options, (II) (ii) (Q) is correct.
Therefore, e −2 x f ( x ) is an increasing function.
Hence, the correct answer is option (B).
Let e −2 x f ( x ) = g( x ) .
2. Which of the following options is the only CORRECT
combination? •  for x = 0: e −20f (0 ) = 1 = g(0 ) .
(A) (I) (i) (R) (B) (II) (iii) (S) •  for x > 0: g(x) > g(0).
(C) (III) (iv) (P) (D) (IV) (i) (S) That is,
Solution: Following equations in the solution of Question 1, we 1
e −2 x f ( x ) > f (0 ) ⇒ e −2 x f ( x ) > 1. ⇒ f ( x ) >
have the following conclusions: e −2 x
Thus,
•• In Column 2, option (i) is false and (ii) (iii) and (iv) are correct.
•• In Column 3, options (Q) and (S) are correct and options (P) f ( x ) > e2 x [in (0, ∞ ] (1)
and (R) are false.
It is given that f ′( x ) > 2f ( x ) . Now, using (1), we get
Thus, from the given options, only (II) (iii) (S) is correct.
Hence, the correct answer is option (B). f ′( x ) > 2 f ( x ) > 2 e 2 x

3. Which of the following options is the only INCORRECT Thus, f(x) is an increasing function [in (0, ∞].
combination? Hence, the correct answers are options (A) and (C).

Mathematical Problem Book for JEE.indb 967 07-06-2018 13:49:37


Mathematical Problem Book for JEE.indb 968 07-06-2018 13:49:37
22
<

Indefinite Integration

22.1 Primitive or Anti-Derivative of 22.2.2  Fundamental Formulas on Integration


a Function   1. ∫ 1dx = x + c
A function f( x ) is called a primitive or an anti-derivative of a x n +1  d  x n +1  
∫x
n n
function f ( x ) if f ′( x ) = f ( x ).   2. dx = + c , n ≠ −1   since,  n + 1 = x 
n +1  dx   
x5 d  x5  1  d 1
For example, is a primitive of x 4 , because = x4.
5 dx  5 
  3. ∫ x dx = ln( x ) + c since, dx (ln x ) = x 
L et f( x ) be a primitive of a function f ( x ) and let c be any constant.
1 (ax + b )n +1
∫ (ax + b)
n
Then   4. dx = ⋅ + c , n ≠ −1
a n +1
d
(f ( x ) + c ) = f ′( x ) = f ( x ) [since f ′( x ) = f ( x )] 1 1
dx   5. ∫ (ax + b) dx = a ⋅ ln ax + b + c
So, f( x ) + c is also a primitive of f ( x ).
 d x 
∫e
x
T hus, if a function f ( x ) possesses a primitive, then it possesses   6. dx = e x + c since, (e ) = e x 
 dx 
infinitely many primitives that are contained in the expression
f( x ) + c where c is a constant. ax  d  ax  
∫a
x
  7. dx = + c since,   = ax 
ln a  dx  ln a  
x5 x5 x5
For example, , − 2, + 1, etc. are primitives of x4.
5 5 5  d 
  8. ∫ sin x dx = − cos x + c since, dx ( − cos x ) = sin x 
22.2 Indefinite Integral and Indefinite d
 
Integration   9. ∫ cos x dx = sin x + c since, dx (sin x ) = cos x 

Let f ( x ) be a function. Then the collection of all its primitives is  d 


called the indefinite integral of f ( x ) and is denoted by ∫ f ( x )dx . 10. ∫ sec
2
x dx = tan x + c since, (tan x ) = sec2 x 
 dx 
d
Thus, (f ( x ) + c ) = f ( x ) ⇒ ∫ f ( x )dx = f ( x ) + c .  d 
dx 11. ∫ cosec
2
x dx = − cot x + c since, ( − cot x ) = cosec2 x 
 dx 
where f( x ) is the primitive of f ( x ) and c is an arbitrary constant
known as the constant of integration.  d 
Here ∫ is the integral sign, f ( x ) is the integrand, x is the variable 12. ∫ sec x ⋅ tan x dx = sec x + c since, dx (sec x ) = sec x ⋅ tan x 
of integration and dx is the element of integration.
The process of finding an indefinite integral of a given function 13. ∫ cosec x ⋅ cot x dx = −cosec x + c
is called integration of the function.
It follows from the above discussion that integrating a function  d 
d
since, dx ( − cosec x ) = cosec x ⋅ cot x 
f ( x ) means finding a function f( x ) such that f( x ) = f ( x ).
dx
14. ∫ tan x dx = ln sec x + c = − ln cos x + c
22.2.1  Fundamental Properties of Integration
 d 
since, dx (lncos x ) = − tan x 
1. ∫ c ⋅ f ( x )dx = c ∫ f ( x )dx
2. ∫ (f ( x ) ± g( x ))dx = ∫ f ( x )dx ± ∫ g( x )dx 15. ∫ cot x dx = ln sin x + c = − ln cosec x + c
1
3. ∫ f ( x )dx = g( x ) + c ⇒∫ f (ax + b)dx = a g(ax + b) + c  d 
Note:  Every continuous function is integrable. since, dx (lnsin x ) = cot x 

Mathematical Problem Book for JEE.indb 969 07-06-2018 13:50:30


970 Mathematics Problem Book for JEE

p x x a2
16. ∫ sec x dx = ln sec x + tan x + c = lntan  4 + 2  + c 29. ∫ x 2 − a2 dx =
2
x 2 − a2 − ln x + x 2 − a2 + c
2
 d 
since, dx (ln(sec x + tan x )) = sec x  x a2
30. ∫ x 2 + a2 dx =
2
x 2 + a2 + ln x + x 2 + a2 + c
2
 x
17. ∫ cosec x dx = ln cosec x − cot x + c = lntan  2  + c Key points:
1. The signum function has an anti-derivative on any interval
 d  which does not contain the point x = 0, and does not possess
since, dx (ln(cosec x − cot x )) = cosec x  an anti-derivative on any interval which contains the point.
2. The anti-derivative of every odd function is an even function
dx
18. ∫ = sin−1 x + c = − cos −1 x + c and vice versa.
1− x 2 sin x
Illustration 22.1  Evaluate ∫ dx.
 d 1  1+ sin x
since, (sin−1 x ) =  Solution:
 dx 1− x 2 
sin x sin x (1− sin x ) sin x − sin2 x
dx −1
x + c = − cot −1 x + c
∫ 1+ sin x dx = ∫ (1+ sin x ) ⋅ (1− sin x ) dx = ∫ cos2 x
dx
19. ∫ 1+ x 2 = tan
d −1 1 d −1 1  (
= ∫ sec x ⋅ tan x − tan2 x dx )
 dx (tan x ) = 1+ x 2 , dx (cot x ) = − 1+ x 2 
= ∫ (1− sec2 x + sec x ⋅ tan x ) dx = x − tan x + sec x + c
dx
20. ∫ = sec −1 x + c = − cosec −1x + c
x x2 −1 ( x + 1)2
Illustration 22.2  Evaluate ∫ dx.
d 1 d 1  x ( x 2 + 1)
−1
 (sec x ) = , (cosec −1x ) = −  Solution:
 dx x x 2 − 1 dx x x 2 − 1 
( x + 1)2 ( x 2 + 2 x + 1)
22.2.2.1  Some Standard Results on Integration ∫ x ( x 2 + 1) dx = ∫ x ( x 2 + 1)
dx

dx x x x2 + 1
21. ∫ = sin−1 + c = − cos −1 + c =∫ dx + ∫
2x
dx
2
a −x 2 a a
x ( x 2 + 1) x ( x 2 + 1)
 d  −1 x  1 
1 2
since  sin  = 2  = ∫ dx + ∫ 2 dx = ln x + 2 tan−1 x + c
 dx a 2
a − x  x ( x + 1)
dx 1 −1 x 1 x
22. ∫ a2 + x 2 = a ⋅ tan + c = − ⋅ cot −1 + c ax 3 + bx 2 + c
a a a Illustration 22.3  Evaluate ∫ dx.
x4
 d  −1 x  a  Solution:
since dx  tan a  = 2 
 a + x2  ax 3 + bx 2 + c a b c  b c
∫ x 4
dx = ∫  + 2 + 4  dx = a ln x − − 3 + k
x x x  x 3x
dx 1 x 1 x
23. ∫ = ⋅ sec −1 + c = − ⋅ cosec −1 + c
x⋅ x −a 2 a 2 a a a
1+ x + x + x 2
 d   Illustration 22.4  Evaluate ∫ dx.
−1 x  a x + 1+ x
since  sec  = 
Solution:
 dx a 2 2
x ⋅ x − a 
dx 1 x 1 x −a 1+ x + x + x 2 x + 1( x + x + 1)
24. ∫ 2
x −a 2
= − ⋅ cot h−1 + c = ⋅ ln
a a 2a x+a
+ c, x > a ∫ x + x + 1 dx = ∫ x + x +1
dx

dx 1 x 1 a+ x 3
−1 2
25. ∫ a2 − x 2 = − a ⋅ tan h a
+ c = ⋅ ln
2a a− x
+ c, x < a = ∫ x + 1dx = ( x + 1) 2 + c
3
dx x
26. ∫ = ln x + x 2 − a2 + c = cos h−1 + c
2 2 a Illustration 22.5  Evaluate ∫ (sin4 x − cos 4 x ) dx .
x −a
dx x Solution:
27. ∫ = ln x + x 2 + a2 + c = sin h−1 + c
a
∫ (sin x − cos 4 x ) dx = ∫ (sin2 x − cos2 x )(sin2 x + cos2 x )dx
2 2 4
x +a

x 2 a2 x 1
28. ∫ a2 − x 2 dx = a − x 2 + sin−1 + c = ∫ (sin2 x − cos2 x ) dx = − ∫ cos 2 x dx = − sin 2 x + c
2 2 a 2

Mathematical Problem Book for JEE.indb 970 07-06-2018 13:51:07


Chapter 22 | Indefinite Integration 971

 x (C) x sin x − a − sina lnsin( x − a ) + c


Illustration 22.6  Evaluate ∫ 1+ sin   dx.
 4 (D) None of these
Solution:  Ans. (B)
cos x − 1
 x  x  x  x  x   2. ∫ dx =
∫ 1+ sin   dx = ∫ sin2   + cos2   + 2 sin   cos   dx
 4  8  8  8  8
cos x + 1
x 1 x
(A) 2 tan − x + c (B) tan − x + c
  x
2 2 2 2
 x
= ∫ sin   + cos    dx 1 x x
  8   8  (C) − tan + x + c (D) −2 tan + x + c
2 2 2
  x  x cos( x/8 ) sin( x/8 )  Ans. (D)
= ∫ sin   + cos    dx = − + +c
  8   8   1 8 18 1
= 8[sin( x/8 ) − cos( x/8 )]+ c
  3. ∫ 1− sin x dx =
( A) x + cos x + c (B) 1+ sin x + c
2x (C) sec x − tan x + c (D) sec x + tan x + c
Illustration 22.7  Evaluate ∫ (2 x + 1)2 dx . Ans. (D)
Solution: 1
  4. If ∫ (sin 2 x − cos 2 x ) dx = sin(2 x − a ) + b, then
2x 2 x + 1− 1 2
∫ (2 x + 1)2 dx = ∫ (2 x + 1)2 dx p
(A) a = , b = 0
4
1 1 p
=∫ dx − ∫ dx (B) a = − , b = 0
(2 x + 1) (2 x + 1)2 4
1 1 5p
= ln 2 x + 1 + +c (C) a = , b = any constant
2 2(2 x + 1) 4
5p
(D) a = − , b = any constant Ans. (D)
sin2 x − cos2 x 4
Illustration 22.8  Evaluate ∫ sin2 x ⋅ cos2 x dx .
 x2 x3 x4 
Solution:   5. ∫  1+ x + + + + … dx =
 2! 3! 4 ! 
sin2 x − cos2 x
∫ sin2 x ⋅ cos2 x dx = ∫ sec2 x dx − ∫ cosec2 x dx (A) − e x + c (B) e x + c
(C) e + c (D) − e − x + c
−x
= tan x + cot x + c
Ans. (B)
Illustration 22.9  Evaluate ∫ (3 cosec2 x + 2 sin 3 x ) dx . cot x ⋅ tan x
Solution:
  6. ∫
sec2 x − 1
dx =

2 (A) cot x − x + c (B) − cot x + x + c


∫ (3 cosec
2
x + 2 sin 3 x ) dx = −3 cot x − cos 3 x + c (C) cot x + x + c (D) − cot x − x + c
3
Ans. (D)
1
Illustration 22.10  Evaluate ∫ 1+ x + x
dx. 2
 7. ∫ (sec x + tan x ) dx =
Solution:
1
1 ( 1+ x − x ) (A) 2(sec x + tan x ) − x + c (B) (sec x + tan x )3 + c
∫ 1+ x + x
dx = ∫
( 1+ x + x ) ⋅ ( 1+ x − x )
dx 3
(C) sec x (sec x + tan x ) + c (D) 2(sec x + tan x ) + c
Ans. (A)
( x + 1)3 / 2 ( x )3 / 2
= ∫ ( 1+ x − x ) dx = − +c
3/ 2 3/ 2   8. ∫x (tan−1 x + cot −1 x ) dx =
51

2 x 52 x 52
= [( x + 1)3 / 2 − ( x )3 / 2 ] + c
3 (A) (tan−1 x + cot −1 x ) + c (B) (tan−1 x − cot −1 x ) + c
52 52
p x 52 p p x 52 p
(C) + + c (D) + +c
Your Turn 1 Ans. 104 2 52 2 (A)
sin x

9. ∫ 5sin x dx =
1. ∫ sin( x − a ) dx =
(A) 5cos x + c (B) −5cos x + c
(A) x cos a − sina lnsin( x − a ) + c
(C) 5sin x + c (D) −5sin x + c
(B) x cos a + sina lnsin( x − a ) + c Ans. (B)

Mathematical Problem Book for JEE.indb 971 07-06-2018 13:52:09


972 Mathematics Problem Book for JEE

tan x 2. I = ∫ f ( x ) ⋅ f ′( x ) dx : In this case, we put f ( x ) = t ⇒ f ′( x ) dx = dt


10. ∫ sec x + tan x dx =
(A) sec x + tan x − x + c (B) sec x − tan x + x + c Illustration 22.14  Evaluate ∫ sin x ⋅ cos x dx.
(C) sec x + tan x + x + c (D) − sec x − tan x + x + c
Ans. (B) Solution: We have
I = ∫ sin x ⋅ cos x dx
22.3  Methods of Integration Let sin x = t . Then cos xdx = dt .
22.3.1  Integration by Substitution
I = ∫ sin x ⋅ cos x dx = ∫ t dt
1. I = ∫ f (f ( x ))f ′( x ) dx : Here, we put f( x ) = t , so that f ′( x ) dx = dt
t2 (sin x )2
and in that case ⇒I = +c = +c
2 2
∫ f (f ( x ))f ′( x ) dx = ∫ f (t ) dt Illustration 22.15  Evaluate
Illustration 22.11  Evaluate ∫ x sin x dx. 3 4
∫(x
3m
+ x 2 m + x m ) ⋅ (2 x 2 m + 3 x m + 6 )1/ m dx , ( x > 0 )
Solution: We have
Solution:
I = ∫ x 3 sin x 4 dx
I = ∫ ( x 3m −1 + x 2 m −1 + x m −1) ⋅ (2 x 3m + 3 x 2 m + 6 x m )1/ m dx
Let x 4 = t . Then
Put (2 x 3m + 3 x 2 m + 6 x m ) = t . Then
3 dt
4 x dx = dt ⇒ dx =
4x3 6 m( x 3m −1 + x 2 m −1 + x m −1)dx = dt
4
sin t cos t cos x
⇒I =∫ dt = − +c = +c 1 1
+1
m +1
4 4 4 tm tm t m
I=∫ dt = +c = +c
sin(ln x ) 6m 1  6( m + 1)
Illustration 22.12  Evaluate ∫ dx . 6 m  + 1
x m 
Solution: We have m +1

sin(ln x ) (2 x 3 m + 3 x 2 m + 6 x m ) m
I=∫ dx ⇒I = +c
x 6( m + 1)

Let ln x = t . Then x 2 tan−1 x 3


Illustration 22.16  Evaluate ∫ dx.
dx
= dt 1+ x 6
Solution:
x
⇒ I = ∫ sint dt = − cos t + c − cos(ln x ) + c x 2 tan−1 x 3
I=∫ dx
1+ x 6
x
Illustration 22.13  Evaluate ∫ dx. Put tan−1 x 3 = t . Then
x 4 + x2 + 1
Solution: We have 3x2
dx = dt
x 1+ x 6
I=∫ dx
x 4 + x2 + 1
1 1 t2 (tan−1 x 3 )2
2
Let x = t . Then 2 xdx = dt .
⇒I =
3 ∫ tdt = ⋅ + c =
3 2 6
+c

x 1 1 f ′( x )
I=∫
2 ∫ t2 + t +1
4 2
dx = dt 3. I = ∫ dx : In this case, we put f ( x ) = t and f ′( x )dx = dt . So,
x + x +1 f (x)
1 1
I= ∫ dt f ′( x ) 1
2  1  3  2
2 I=∫ dx = ∫ dt = ln(f ( x )) + c
f (x) t
 t +  +  
2  2 
x3
    Illustration 22.17  Evaluate ∫ dx.
 1  2 1 1+ x 4
 t +   x + 
1  2  1  2  Solution:
⇒I = tan−1  +c = tan−1  +c
3   3  3   3  x3
        I=∫ dx
  2     2   1+ x 4

Mathematical Problem Book for JEE.indb 972 07-06-2018 13:52:54


Chapter 22 | Indefinite Integration 973

Put 1+ x 4 = t. Then ⇒ 4 x 3dx = dt Illustration 22.22  Evaluate ∫ cos 3 x ⋅ 2 + sin 3 x dx.


1 1 1 1 Solution:
4∫t
⇒I = dt = ln t + c = ln(1+ x 4 ) + c
4 4 I = ∫ cos 3 x ⋅ 2 + sin 3 x dx
sin 2 x Put 2 + sin 3 x = t . Then
Illustration 22.18  Evaluate ∫ dx.
a2 sin2 x + b2 cos2 x 3 cos 3 x dx = dt
Solution: 1 3
1 +1
sin 2 x 1 1 t2 2(2 + sin 3 x ) 2
I=∫ dx ⇒ I = ∫ t 2 dt = 1 + c = +c
a2 sin2 x + b2 cos2 x 3 3 +1 9
2

Put a2 sin2 x + b2 cos2 x = t . Then sin2 x ( a2 − b2 )dx = dt


5. Standard substitutions:
11 1
⇒ I = 2 2 ∫ dt = 2 2 ln t + c Integrand form Substitution
(a − b ) t (a − b )
1 1 x = a sinq or
= 2 2 ln(a sin2 x + b2 cos2 x ) + c
2
(a) a2 − x 2 , , a2 − x 2
(a − b ) a2 − x 2 a cosq

1 1 x = a tanq or
Illustration 22.19  Evaluate ∫ dx . (b) a2 + x 2 , , a2 + x 2
1+ e x 2
a +x 2
a cotq
Solution:
1 x = a secq or
1 e− x (c) x 2 − a2 , , x 2 − a2
I=∫ dx = ∫ dx x − a2
2
1+ e x
1+ e −x a cosecq

Put 1+ e − x = t . Then − e − x dx = dt x x+a


, , x = a tan2 q or
1 (d) x+a x
⇒ I = − ∫ dt = − ln t + c = − ln(1+ e − x ) + c
t 1 a cot2 q
x ( x + a) ,
x ( x + a)
4. I = ∫ (f ( x ))n ⋅ f ′( x )dx : In this case, we put f ( x ) = t and f ′( x )dx = dt
x a− x
f ( x ) = t
and f ′( x )dx = dt . So, , ,
a− x x x = a sin2 q or
(e)
1 x = a cos2 q
(f ( x ))n +1 x (a − x ),
I = ∫ (f ( x ))n ⋅ f ′( x )dx = +c x (a − x )
n +1

(ln x )5 x x −a
Illustration 22.20  Evaluate ∫ x dx. (f) x −a
,
x
,
x = a sec2 q or
Solution: 1 a cosec2q
x ( x − a) ,
(ln x )5 x ( x − a)
I=∫ dx
x
(g) a+ x a− x x = a cos2q
dx ,
Put ln x = t . Then = dt a− x a+ x
x
x −a
t6 (ln x )6 , ( x − a )( b − x ), x = b sin2 q +
⇒I = ∫ t 5dt = +c = +c (h) b −x
6 6 a cos2 q
(b > a )
Illustration 22.21  Evaluate ∫ sin10 x ⋅ cosx dx.
dx
Illustration 22.23  Evaluate ∫ .
Solution: (1+ x ) x − x 2
I = ∫ sin10 x ⋅ cosx dx Solution: Let x = sin2 q . Then

Put sin x = t . Then cos x dx = dt dx = 2sinq ⋅ cosq ⋅ dq

t 11 (sin x )11
⇒ I = ∫ t 10dt = +c = +c 2 sinq ⋅ cosq ⋅ dq 2dq 2(1− sinq )dq
I=∫ =∫
(1+ sinq ) ∫
11 11 =
2
(1+ sinq ) sin q − sin q 4 cos2 q

Mathematical Problem Book for JEE.indb 973 07-06-2018 13:53:44


974 Mathematics Problem Book for JEE

  ⇒ I = 2∫ (sec2 q − secq ⋅ tanq ) dq = 2(tanq − secq ) + c a sec2 q dq dq


⇒I =∫ 3 3
=∫ 2

  ⇒ I = 2
sinq − 1
+c=
2 x −1 (
+c
) dq
1
a sec q
1
a secq
 1
 cosq  1− x ⇒I = 2 ∫ = 2 ∫ cosq dq = sinq + c
a secq a a2
1− x x
Illustration 22.24  Evaluate ∫ dx. ⇒I = 2 2 2 12 +c
1+ x a (x + a )
Solution:
1− x
1− x (1− x )2 1 x Illustration 22.27  Evaluate ∫ dx.
I=∫ dx = ∫ dx = ∫ dx − ∫ dx 1+ x
1+ x 1 − x 1− x 1− x Solution:

Let x = sin2 q . Then 1− x


I=∫ dx
1+ x
dx = 2sinq ⋅ cosq ⋅ dq Put x = cos 2q . Then dx = −2 sin 2q ⋅ dq.
I = ∫ 2 sinq dq − ∫ 2 sin q dq = 2∫ sinq dq − ∫ (1− cos 2q ) dq
2
1− cos 2q
I = −2∫ sin 2q ⋅ dq
sin2q 1+ cos 2q
= −2 cosq − q + +c
2
2 sin2 q
−1 ⇒ I = −2∫ sin 2q ⋅ dq = −4 ∫ tanq ⋅ sinq ⋅ cosq ⋅ dq
I = x ⋅ 1− x − 2 1− x − sin x +c 2 cos2 q

⇒ I = −4 ∫ sin2 q dq = −2∫ (1− cos 2q )dq


t2 +1
Illustration 22.25  Evaluate ∫ t 2 dt .
t −1 sin2q 
⇒ I = −2  q −  + c = −2q − sin2q + c
 2 
Solution: Put s = t 2 . Then ds = 2tdt .

Now, ⇒ I = − cos −1 x + 1− x 2 + c
1 s +1 1 1+ s   6. Some more substitution:
2 ∫ s −1
I= ds = ∫ ds
( ) or ( x ± )
n n
2 s2 − 1   1. 
For the type x 2 + a2 ± x x 2 − a2 , put the
1 1 1 s 1 1 2sds expression within the bracket = t.
2 ∫ s2 − 1
= ds + ∫ ds = ln s + s2 − 1 + ∫
2 s2 − 1 2 4 s2 − 1 1 1 −1+
1

  2.  For the type ( x + a)


−1−
n ⋅ ( x + b)
−1+  ( x + b) n 1
n or  ⋅
Let s2 = x ⇒ 2 sds = dx . Then  ( x + a)  ( x + a)2
11
( )
1 2sds 1 dx 1 1 2 11 11 −−11++
((xx++bb)) nn ( x + b)
4 ∫ s2 − 1 4 ∫ x − 1 2
⇒ = = x −1 = 1+aa))−−11−−nn ⋅⋅((xx++bb   
s ((x−x+ −−11++ 11
2 )) nn or
or   ⋅⋅ (n ∈ N, n > 1), put = t.
((xx++aa)) 2
((xx++aa))2 ( x + a)
So, 1
  
3. 
For ,  n1,  n2 ∈ N (and >1),  again put (x + a) =
1 1 2 ( x + a)n1 ( x + b )n2
I = ln s + s2 − 1 + s −1
2 2 t (x + b).
1 1 4 dx
= ln t 2 + t 4 − 1 +
  t −1+ c Illustration 22.28  Evaluate ∫ 65
2 2 ( x + 1) ( x − 3)4 5
Solution:
dx
Illustration 22.26  Evaluate ∫ . dx dx
(a2 + x 2 )3 2 I=∫ 65 45
= ∫ 45
( x + 1) ( x − 3)  ( x − 3) 
Solution: ( x + 1)2 
dx  ( x + 1) 
I=∫  ( x − 3) 
(a2 + x 2 )3 2 Put  = t. Then
 ( x + 1) 
Put x = a tanq . Then 4
dt = dx
dx = a sec2 q dq ( x + 1)2
Therefore, Hence,
2
a sec q dq dt 5 1/ 5 5  ( x − 3) 
1/ 5
I=∫ I=∫ = t +c =  +c
2
(a + (a tanq ) ) 2 32 
4t 4 / 5 4 4  ( x + 1) 

Mathematical Problem Book for JEE.indb 974 07-06-2018 13:54:27


Chapter 22 | Indefinite Integration 975

dx æ 2x ö
Illustration 22.29  Evaluate ∫ . (A) ∫x
6
tan−1x 3 dx (B) ò tan-1 ç ÷ dx
( x + 1)2 ( x − 3)3 è 1- x 2 ø
Solution:
∫x
3
(C) cos x 2 dx (D) None of these Ans. (C)
dx
I=∫
( x + 1)2 ( x − 3)3  
∫ tan x ⋅ sec
2
4. x ⋅ 1− tan2 x dx =
 ( x − 3)  4
Put   = t. Then dt = dx
 ( x + 1)  ( x + 1)2 1 1
(A) − (1− tan2 x )3 2 + c (B) (1− tan2 x )3 2 + c
4 3 3
⇒ ( x + 1) =
(1− t ) 2
(C) − (1− tan2 x )2 3 + c (D) None of these Ans. (A)
(1− t )3 dt (1− t 3 + 3t 2 − 3t )dt 1 3
I = ∫ =∫ = ∫ (t
−3
− 1+ 3t −1
− 3t −2
)dt
44 t 3 44 t 3 44 sin 2 x
1  1 
= 4  − 2 − t + 3 ln t + 3t −1 + c
  5. ∫ sin 5 x ⋅ sin 3 x dx =
4  2t 
−2 −1
(A) lnsin 3 x − lnsin 5 x + c
1  1  ( x − 3)   ( x − 3)   ( x − 3)   ( x − 3)  
I = 4 −   −   + 3 ln   + 3    +c 1 1
4  2  ( x + 1)  ( x + 1)  ( x + 1)  ( x + 1)   (B) lnsin 3 x + lnsin 5 x + c
3 5

( ) dx.
3
1 1
Illustration 22.30  Evaluate ∫ x 2 + 22 + x (C)
3
lnsin 3 x − lnsin 5 x + c
5
Solution: (D) 3 lnsin 3 x − 5 lnsin 5 x + c Ans. (C)
( ) dx
3
I= ∫ x 2 + 22 + x
ex

( )
  6. dx =
Put x 2 + 22 + x = t . Then 1− e 2 x

x 2 + 22 (A) cos −1(e x ) + c (B) − cos −1(e x ) + c


dx =
( )
dt
x 2 + 22 + x
(C) cos −1(e 2 x ) + c (D) 1− e2 x + c  Ans. (B)
2 2 2 2
t +2 t +2 a x cos(a x )
⇒ x 2 + 22 = ⇒ dx = dt
2t 2t 2   7. ∫ ln a dx =
( t 2 + 22 ) ⋅ t 3 1 3 2 1  t4 t2 
I= ∫ dt = ∫ (t + 2 t )dt =  + 22  + c (A) sina x + c (B)
a x sin a x + c
2 2 2 4 2
2t
sin(a x )
( ) + 2(
 4  (C) lnsina x + c 
+ c (D) Ans. (C)
2
x +2 + x 2
ln2 a
1
)
2
2 2
=  x +2 + x  +c
2 4  sin x
    8. ∫ (a + b cos x )2 dx =
Your Turn 2 (A)
1
(a + b cos x ) + c (B) + c
1
b b(a + b cos x )
dx
  1. ∫ = (C)
1
ln(a + b cos x ) + c (D)
None of these Ans. (B)
x 1− (ln x )2 b

(A) cos −1(ln x ) + c (B) x ln(1− x 2 ) + c


sec2 x
1
(C) sin−1(ln x ) + c (D) cos −1(ln x ) + c Ans. (C)
  9. ∫ dx =
tan2 x + 4
2

  2. ∫
f ′( x )dx
(f ( x ))2
= (
(A) ln tan x + tan2 x + 4 + c )
(A) −(f ( x ))−1 + c (B) ln(f ( x )) + c (B)
1
2 (
ln tan x + tan2 x + 4 + c )
f(x)
(C) e + c (D) None of these Ans. (A) 1 1 
(C) ln  tan x + tan2 x + 4  + c
2 2 
  3. For which of the following functions, the substitution x 2 = t is
applicable? (D) None of these Ans. (A)

Mathematical Problem Book for JEE.indb 975 07-06-2018 13:55:23


976 Mathematics Problem Book for JEE

2 x ⋅ tan−1 x 2 Illustration 22.34  Evaluate ∫ (f ( x )g′′( x ) − g( x )f ′′( x )) dx .


10. ∫ x4 +1
dx =
Solution:
−1 2 2 1
(A) (tan x ) + c (B) (tan−1 x 2 )2 + c
2   I = ò [(f ( x )g¢¢( x ) - g( x ) f ¢¢( x ))] dx = ∫ f ( x )g′′( x ) dx − ∫ g( x )f ′′( x ) dx
−1 2 2
(C) 2(tan x ) + c (D) None of these Ans. (B)
  I = [f ( x )g′( x ) − ∫ f ′( x )g′( x )dx ] − [ g( x )f ′( x ) − ∫ f ′( x )g′( x )dx ]

22.3.2  Integration by Parts = f ( x )g′( x ) − g( x )f ′( x )


  

If F and G are two functions of x, then integral of the product of Illustration 22.35  Evaluate ∫ x 2 + a2 dx.
these two functions is given by
 dF  Solution:
∫ F ⋅ G dx = F ∫ G dx − ∫  dx ∫ G dx  dx 2x2
   I = ∫ x 2 + a2 dx = x 2 + a2 ∫ 1dx − ∫ dx
Or we can say that the integral of the product of two functions 2 x 2 + a2
= (First function) × (Integral of second function) − Integral of x 2 + a2 a2
  = x x 2 + a2 − ∫ dx + ∫ dx
{(Differentiation of first function) × (Integral of second function)}. 2
x +a 2
x + a2
2

Note: In applying the above rule, care has to be taken in the selec-
tion of the first function (F ) and the second function (G). ⇒ I = x x 2 + a2 − I + a2 In x + x 2 + a2 + c
Normally, we use the following methods:
1. In the product of two functions, one of the function is not ⇒ 2I = x x 2 + a2 + a2 In x + x 2 + a2 + c
directly integrable (that is, ln x , sin−1 x , cos −1 x , tan−1 x , etc.),
x 2 a2
then we take it as the first function and the remaining function ⇒I = x + a2 + In x + x 2 + a2 + c
2 2
is taken as the second function.
2. If there is no other function, then unity is taken as the
second function. For example, in the integration of sin−1 x − cos −1 x
−1 Illustration 22.36  Evaluate ∫ dx.
∫ ln x dx , ∫ sin x dx , 1 is taken as the second function. sin−1 x + cos −1 x
3. If both of the functions are directly integrable, then the first Solution:
function is chosen in such a way that the derivative of the func-
tion thus obtained under integral sign is easily integrable.
sin−1 x − cos −1 x
In the above stated order, the function on the left is always cho- I = ∫ −1
sin x + cos −1 x
2
dx = ∫ sin−1 x − cos −1 x dx
p
( )
sen as the first function. This rule is known as ILATE (Inverse,
Logarithmic, Algebraic, Trigonometric, Exponential).  −1 p
 sin x + cos −1 x = 
For example, in the integration of ∫ x sin x dx , x is taken as the 2
first function and sin x is taken as the second function.
For first expression, ∫ sin−1 x dx
Illustration 22.31  Evaluate ∫ sec3 q dq.
Put x = sin2 q . Then
Solution: 1− 2 x = cos 2q ⇒ dx = sin 2q dq
I = ∫ sec3 q dq = secq ∫ sec2 q dq − ∫ tanq (secq tanq ) dq q cos 2q 1
∫ q sin 2q dq = − 2
+ ∫ cos 2q dq
2
= secq ⋅ tanq − ∫ secq (sec q − 1) dq 2
q cos 2q sin 2q
=− + +c
= secq ⋅ tanq − ∫ sec3 q dq + ∫ secq dq 2 4
(2 x − 1) −1 1
⇒ I = secq ⋅ tanq − I + ∫ secq dq = sin x+ x (1− x ) + c
2 2
1 1
⇒ I = [secq . tanq ] + ln| secq + tanq | + c
2 2 For second expression, ∫ cos −1 x dx

Illustration 22.32  Evaluate ∫ x ⋅ sin x dx. Put x = cos2 q . Then


dx = −2 sinq cosq dq ⇒ dx = − sin 2q dq
Solution:
−1 q cos 2q 1
I = ∫ x ⋅ sin x dx = − x cos x + ∫ cos x dx = − x cos x + sin x + c ∫ cos x dx = − ∫ q sin 2q dq =
2
− ∫ cos 2q dq
2
q cos 2q sin 2q
Illustration 22.33  Evaluate ∫ x sec2 x dx. = − +k
2 4
Solution: (2 x − 1) cos−1 1
= x− x (1− x ) + k
I = ∫ x sec x dx = x tan x − ∫ tan x dx = x tan x + lncos x + c
2 2 2

Mathematical Problem Book for JEE.indb 976 07-06-2018 13:56:03


2
 1− x 
I = ∫ ex  dx
 1+ x 2 
Chapter 22 | Indefinite
2
Integration 977
(1+ x − 2 x )
= ∫e x
dx
(1+ x 2 )2
Therefore,  1 2x 
= ∫ ex  −  dx
2
(
I = (2 x − 1)sin−1 x + x (1− x ) − x + a
p
)  (1+ x 2 ) (1
1+ x 2 ) 2 
ex
Illustration 22.37  Evaluate ∫ x 3 ln x dx. ⇒I = +c
(1+ x 2 )
Solution: e2 tan
−1
x
(1+ x )2
I = ∫ x ln x dx =
3 x 4
⋅ln x − ∫
x 1
⋅ dx
4 Illustration 22.43  Evaluate ∫ 1+ x 2
dx.
4 4 x
x4 x4 Solution: Put tan−1 x = t . Then
= ⋅ ln x − +c
4 16 1
dx = dt
x 2dx 1+ x 2
Illustration 22.38  Evaluate ∫ 2
.
( x sin x + cos x ) −1
e2 tan x
(1+ x )2
Solution: I=∫ dx = ∫ e2t (1+ tan t )2 dt = ∫ e2t (sec2 t + 2 tan t ) dt
1+ x 2
x2 x cos x x
  I = ∫ dx = ∫ ⋅ dx e 2t e 2t
( x sin x + cos x )2 ( x sin x + cos x )2 cos x ⇒ I = ∫ e 2t sec2 tdt + 2 tan t ⋅ − 2∫ sec2 tdt
−1 x 1 cos x + x sin x 2 2
I= ⋅ +∫ ⋅ dx
( x sin x + cos x ) cos x ( x sin x + cos x ) cos2 x
I = ∫ e2t sec2 tdt + tan t ⋅ e2t − ∫ e2t sec2 tdt + c
−1 x
( x sin x + cos x ) cos x ∫
  I = ⋅ + sec2 x dx −1
I = tan t ⋅ e2t + c ⇒ I = x ⋅ e2 tan x
+c
−1 x
  I = ⋅ + tan x + c æ x +3 ö x
( x sin x + cos x ) cos x Illustration 22.44  Evaluate ò ç 2÷
e dx .
è ( x + 4) ø
22.3.2.1  Some Important Results Solution:

∫e
x
1. (f ( x ) + f ′( x ))dx = e x f ( x ) + c  x +3  x  x + 4 − 1 x
I = ∫ e dx = ∫  e dx
 ( x + 4 )2   ( x + 4 )2 
Illustration 22.39  Evaluate ∫ e x (1− cot x + cot2 x ) dx.
 x+4  x  1  x
  = ∫ e dx − ∫  e dx
Solution:  ( x + 4 )2   ( x + 4 )2 
∫e (1− cot x + cot2 x )dx = ∫ e x ( − cot x + cosec2 x )dx = − e x cot x + c
x
ex ex
I=∫ dx − ∫ dx
æ 1- sin x ö ( x + 4) ( x + 4 )2
Illustration 22.40  Evaluate ò e x ç ÷ dx .
è 1- cos x ø  1  x  1  x  1  x
I= e +∫ e dx − ∫  e dx + c
Solution:  ( x + 4 )   ( x + 4 )2   ( x + 4 )2 
x x
2 sin cos  1  x
 1− sin x  1 2 2 x x
 = − = cosec2 − cot I= e +c
 1− cos x  2 x 2 x 2 2  ( x + 4 ) 
2 sin 2 sin
2 2
 1− sin x  x 2 x x x 2. ∫ (f ( x ) + xf ′( x ))dx = xf ( x ) + c
∫e  dx = ∫ e  cosec − cot  dx = − e cot + c
x x

1− cos x 2 2 2
Illustration 22.45  Evaluate ∫ ( x cos x + sin x )dx.
1 1
Illustration 22.41  Evaluate ∫ e  − 2  dx. x
x x  Solution:
Solution:
I = ∫ ( x cos x + sin x )dx = x sin x + c
1 1 ex
I = ∫ e x  − 2  dx ⇒ I = +c
x x  x  x + sin x 
Illustration 22.46  Evaluate ∫  dx .
2  1+ cos x 
 1− x 
Illustration 22.42  Evaluate ∫ e x  dx. Solution:
 1+ x 2 
 x + sin x  1  x x
Solution: I = ∫ dx = ∫  x sec2  dx + ∫ tan dx
 1+ cos x  2  2  2
2
 1− x 
I = ∫ ex  dx x
 1+ x 2  1 x tan 2 x x x
    I = − ∫ tan dx + ∫ tan dx + c = x tan + c
(1+ x 2 − 2 x ) 2 1 2 2 2
= ∫ ex dx
(1+ x 2 )2 2
 1 2x 
= ∫ ex  −  dx
 (1+ x 2 ) (1
1+ x 2 ) 2 

Mathematical Problem Book for JEE.indb 977 07-06-2018 13:56:42


978 Mathematics Problem Book for JEE

3. ∫ e ax sin bx dx , ∫ e ax cos bx dx : ⇒ bu + av = e ax sin bx  (1)


Similarly,
Working rule:  To evaluate ∫ e ax sin bx dx or ∫ e ax cos bx dx , proceed
as follows: bv − au = − e ax cos bx  (2)
(a) Put the given integral equal to I. Squaring Eqs. (1) and (2) and adding, we get
(b) Integrate by parts, taking e ax as the first function.
2 2 2 2 2 ax
(c) Again, integrate by parts taking e ax as the first function. ((aa2 ++bb2 )()(uu2 ++vv2 ))== ee2ax
This will involve I.
(d) Transpose and collect terms involving I and then obtain
the value of I. Your Turn 3
Let I = ∫ e ax sin bx dx . Then
∫ x sec
2
1. x dx =
cos bx  cos bx  x2
I = ∫ e ax sin bx dx = − e ax − ∫ ae ax  −  dx (A) tan x + lncos x + c (B) sec2 x + lncos x + c
b  b  2
cos bx a ax (C) x tan x + lnsec x + c (D) x tan x + lncos x + c
= − e ax + ∫ e cos bx dx
b b  Ans. (D)

= − e ax
cos bx a  ax sin bx a ax
+ e

− ∫ e sin bx dx 
  2. ∫ sin(ln x ) dx =
b b  b b 
1
cos bx a ax
2 (A) x (cos(ln x ) − sin(ln x )) + c
 a 2
∫e
ax ax
= −e + 2 e sin bx −   sin bx dx
b b  b ( B) cos(ln x ) − x + c
2 1
cos bx a ax  a (C) x (sin(ln x ) − cos(ln x )) + c
= − e ax + 2 e sin bx −   I
b b  b 2
2 ax
(D) − cos(ln x ) + c Ans. (C)
 a e
⇒ I +   I = 2 ( −b cos bx + a sin bx ) + c
 b b   3. If ∫ x sin x dx = − x cos x + A, then A =
ax
e
⇒I = ( −b cos bx + a sin bx ) + c (A) sin x + c (B) cos x + c
a2 + b 2 (C) Constant (D) None of these Ans. (A)
Thus,
e ax   4. ∫ x ln x dx =
∫e
ax
sin bx dx = 2 2
( −b cos bx + a sin bx ) + c
a +b
x2 x2 x2 x2
e ax  b (A) ln x − + c (B) ln x − +c
= sin  bx − tan−1  + c 2 2 2 4
2
a +b 2  a
x2 x2
Similarly, (C) ln x + + c (D) None of these Ans. (B)
ax
2 2
e
∫e
ax
cos bx dx = (b sin bx + a cos bx ) + c
a2 + b 2   5. ∫ x cos dx =
e ax b
= cos  bx − tan−1  + c (A) x sin x + cos x + c (B)
x sin x − cos x + c
2
a +b 2  a
(C) x cos x + sin x + c (D) x cos x − sin x + c  Ans. (A)
Illustration 22.47  Evaluate ∫ e sin x dx. x

∫ x cos
2
  6. x dx =
Solution:

ex ex x2 1 1
(A) − x sin 2 x − cos 2 x + c
∫ e sin x dx =
x
(1⋅ sin x − 1⋅ cos x ) + c = (sin x − cos x ) + c
12 + 12 2 4 4 8
x2 1 1
Illustration 22.48  If u = ∫ e cos bx dx and v = ∫ e sin bx dx ,
ax ax (B) + x sin 2 x + cos 2 x + c
4 4 8
2 2 2 2
then find (a + b )(u + v ). x2 1 1
(C) − x sin 2 x + cos 2 x + c
Solution: 4 4 8
sin bx  sin bx  sin bx a x2 1 1
u = ∫ e ax cos bx dx = e ax − ∫ ae ax  dx = e ax − v (D) + x sin 2 x − cos 2 x + c Ans. (B)
b  b  b b 4 4 8

Mathematical Problem Book for JEE.indb 978 07-06-2018 13:57:31


Chapter 22 | Indefinite Integration 979

−1 U( x ) P1 P2 Pk A x + B1 A2 x + B2 A x + Br
  7. ∫ tan x dx = = + +…+ + 1 + +…+ 2 r
V ( x ) ( x − a ) ( x − a )2 ( x − a )k x 2 + ax + b ( x 2 + ax + b )2 ( x + ax + b )
1
U( x(A) ) x tan P1 −1 x + P(ln( 1+ x 2 )) + c Pk A x + B1 A2 x + B2 A x + Br
= + 2 2 2 +…+ + 1 + +…+ 2 r +…
V(x) (x −a ) (x −a ) ( x − a )k x 2 + ax + b ( x 2 + ax + b )2 ( x + ax + b )r
1
(B) x tan−1 x − (ln(1+ x 2 )) + c where P1, P2, …, Pk, A1, A2, …, Ar, B1, B2, …, Br are real constants
2
to be determined. Reducing both sides of the above identity to
(C) ( x − 1)tan−1 x + c the integral form and equating the coefficients of equal powers
−1 2 of x, which gives a system of linear equations in the coefficient.
(D) x tan x − x ln(1+ x ) + c  Ans. (B)
Determine these coefficient. (This method is called the method of
−1 comparison of coefficients.) The constants can also be obtained by
  8. ∫ x tan x dx =
substituting suitably chosen numerical values of x in both sides of
2 the identity.
(1+ x ) x
(A) tan−1 x − + c Key point:
2 2
2 Before proceeding to write a rational function as a sum of partial
( x − 1) x
(B) tan−1 x − + c fractions, we should ascertain that it is either a proper rational frac-
2 2 tion or is rewritten as one.
( x 2 + 1) −1 x A rational function U(x)/ V(x) is proper if the degree of polyno-
(C) tan x + + c
2 2 mial V(x) is greater than the degree of the polynomial U(x). In case
( x 2 + 1) the degree of U(x) is greater than or equal to the degree of V(x), we
(D) tan−1 x − x + c  Ans. (A) U( x ) u( x )
2 first write = h( x ) + , where h(x) is a polynomial and u(x) is
V(x) v( x )
 a polynomial of degree less than the degree of polynomial V(x).
1 
  9. ∫  ln (ln x ) +  dx =
 (ln x )2  cos x dx
Illustration 22.49  Evaluate ∫ .
(1+ sin x )(2 + sin x )
x
(A) x ln(ln x ) + + c
(ln x ) Solution: Put sin x = t . Then
x
(B) x ln(ln x ) − +c cos x dx = dt
(ln x )
dt dt dt
(C) x ln(ln x ) +
ln x
+ c ∫ (1+ t )(2 + t ) = ∫ (1+ t ) − ∫ (2 + t ) = ln(1+ t ) − ln(2 + t ) + c
x
ln x (1+ sin x )
(D) x ln(ln x ) − + c Ans. (B) = ln +c
x (2 + sin x )

10. ∫ (sin(ln x ) + cos(ln x )) dx = Illustration 22.50  Evaluate ∫


dx
.
(A) x cos(ln x ) + c (B) sin(ln x ) + c sin x (2 + cos x − 2 sin x )

(C) cos(ln x ) + c (D) x sin(ln x ) + c  Ans. (D) x


Solution: Put tan = t . Then
2

22.4  Integration by Partial Fractions 1 2x


sec dx = dt ⇒ dx =
2dt
⇒ dx =
2dt
2 2 x 1+ t 2
sec2
A function of the form U(x)/V(x), where U(x) and V(x) are polyno- 2
mials, is called a rational function. Consider the rational function 2dt
x +7 1 1 1+ t 2
= − I=∫ 1 + t2 =∫ 2 dt
(2 x − 3)(3 x + 4 ) (2 x − 3) (3 x + 4 ) 2t  1− t 2
2t  t ( t − 4 t + 3)
2 + −2 
The two fractions on the RHS are called partial fractions. To inte- 1+ t 2  1+ t 2 1+ t 2 
grate the rational function on the LHS, it is enough to integrate the
two fractions on the RHS, which are easily integrable. This is known Expands into simple fractions
as a method of partial fractions.
1+ t 2 A B C
In case the degree of U(x) (numerator) is not less than that of = + +
t (t − 3)(t − 1) t (t − 3) (t − 1)
V(x) (denominator), we carry out the division of U(x) by V(x) and
reduce the degree of the numerator. ⇒ 1+ t 2 = A(t − 3)(t − 1) + Bt (t − 1) + C (t − 3)t
In order to write U(x)/V(x) in partial fractions, first of all we write
V(x) = ( x − a )k …( x 2 + ax + b )r where binomials are different, and 1 5
After solve the coefficients, A = ; B = ; C = −1.
then set 3 3

Mathematical Problem Book for JEE.indb 979 07-06-2018 13:58:08


980 Mathematics Problem Book for JEE

Hence, ìïæ b ö 4 ac - b2 üï
2
1 dt 5 dt dt Convert ax 2 + bx + c = a íç x + ÷ + ý
I= ò + ò
3 t 3 t - 3 ò t -1
-
îïè 2a ø 4 a2 þï

1 5 And then use formulas


= ln t + ln t - 3 - ln t - 1 + c
3 3 dx dx dx dx
(a) ∫ x 2 − a2 , ∫ x 2 + a2 , ∫ a2 − x 2 for ∫ ax 2 + bx + c
1 x 5 x x
= ln tan + ln tan - 3 - ln tan - 1 + c
3 2 3 2 2 dx dx dx dx
(b) ∫ 2 2
,∫
2 2
,∫
2 2
for ∫
2
3x + 1 x −a x +a a −x ax + bx + c
Illustration 22.51  Evaluate ∫ ( x − 2)2 ( x + 2) dx.
Solution:
(c) ∫ x 2 − a2 dx , ∫ x 2 + a2 dx , ∫ a2 − x 2 dx for ∫ ax 2 + bx + c dx

3x + 1 A ∫ x 2 −Ba2 dx , ∫ C x 2 + a2 dx , ∫ a2 − x 2 dx for ∫ ax 2 + bx + c dx .
2
= + +
( x − 2) ( x + 2) ( x − 2) ( x − 2)2 ( x + 2)
1
3 x + 1 = A( x − 2)( x + 2) + B( x + 2) + C ( x − 2)2 (1) Illustration 22.53  Evaluate ò x 2 + x + 1dx.
Putting x = 2 and −2 successively in Eq. (1), we get Solution:

7 5 1 1
B= ,C=− I=∫ 2
dx = ∫ 2
dx
4 16 x + x +1  1  3 
2
 x +  +  
5 2  2 
Now, we put x = 0 and get A= . Therefore,
16
2 æ 2 x + 1ö
3x + 1 = tan-1 ç ÷+c
I= ò dx 3 è 3 ø
( x -2)2 ( x + 2)
5 7 5 1
I= ò dx + ò dx -ò dx Illustration 22.54  Evaluate ò dx.
16( x -2) 4( x -2)2 16( x + 2) 2
2x + 3x + 2
5 7 5 Solution:
I= ln x − 2 − − ln x + 2 + c
16 4( x − 2) 16 1 1 1
I=∫
2∫
dx = dx
5 x −2 7 2
2x + 3x + 2 2 2
I = ln − +c  3  7 
16 x + 2 4( x − 2)  x +  +
4   4 

2x2 + 3 æ 2 2ö
Illustration 22.52  Evaluate ∫ ( x 2 − 1)( x 2 + 4 ) dx . =
1 çæ 3ö æ 3ö æ 7ö ÷
ln ç x + ÷ + ç x + ÷ + çç ÷ +c
2 çç è 4ø è 4 ø è 4 ÷ø ÷÷
Solution: è ø
2x2 + 3 A B
= + Illustration 22.55  Evaluate ò x 2 - x dx .
( x − 1)( x 2 + 4 )
2
( x 2 − 1) ( x 2 + 4 )
Therefore, Solution:
2 2 2
2 x + 3 = A( x + 4 ) + B( x − 1) æ 1ö æ 1ö
2 2
I = ò x 2 - x dx = ò ç x - ÷ - ç ÷ dx
Comparing the coefficient of x2 and constant terms è 2ø è2ø
æ 1ö
⇒ A + B = 2, 4 A − B = 3 çx- ÷
è 2ø 2 1 ææ 1ö ö
I= x - x - ln ç ç x - ÷ + x 2 - x ÷ + c
⇒ A = 1, B = 1 2 8 èè 2ø ø
1 1
I=∫ dx + ∫ dx
2
( x − 1) 2
( x + 4) 1
Illustration 22.56  Evaluate ò 2 x 2 + x + 1dx.
1 x -1 1 x
Þ I = ln + tan-1 + c Solution:
2 x +1 2 2
Integrals of the form: 1 1 1
I=∫
2∫
2
dx = 2
dx
dx dx 2x + x +1 2
1  7 
∫ ax 2 + bx + c , ∫ , ∫ ax + bx + c dx  x +  +
2
1.
4   4 
ax 2 + bx + c

Mathematical Problem Book for JEE.indb 980 07-06-2018 13:58:41


Chapter 22 | Indefinite Integration 981

æ ö Put ( x + 4 ) = t . Then dx = dt .
ç æç x + 1 ö÷ ÷
1 4 ç 4ø÷
I= × tan-1 ç è +c I = ∫ t 2 − 22 dt =
t 2 2 4
t - 2 - ln t + t 2 - 22 + c
2 7 æ 7ö ÷ 2 2
ç ç ÷ ÷
ç ç 4 ÷ ÷
è è ø ø
( x + 4) 4
I= ( x + 4 )2 − 22 − ln ( x + 4 ) + ( x + 4 )2 − 22 + c
2 æ ( 4 x + 1) ö 2 2
I= tan-1 ç ÷+c
7 è 7 ø ( x + 4) 2
I= x + 8 x + 12 − 2 ln ( x + 4 ) + x 2 + 8 x + 12 + c
2
cos x
Illustration 22.57  Evaluate ò sin2 x + 4 sin x + 5 dx .
Illustration 22.61  Evaluate ò 2ax - x 2 dx.
Solution:
cos x cos x Solution:
I=ò 2
dx = ò 2
dx
sin x + 4 sin x + 5 (sin x + 2) + 1 I = ò 2ax - x 2 dx = ò a2 - a2 + 2ax - x 2 dx
Put (sin x + 2) = t . Then cos x dx = dt .
= ∫ a2 − (a2 − 2ax + x 2 ) dx = ∫ a2 − ( x − a)2 dx
1
I=ò -1
dt = tan t + c = tan -1
( sin x + 2 ) + c
t2 +1 Put ( x − a) = t . Then dx = dt .
1 t a2 t
Illustration 22.58  Evaluate ò 2
dx. I = ∫ a2 − t 2 dt =
2
a2 - t 2 + sin-1 + c
2 a
2 - 3x - x
Solution:
( x − a) 2 a2 ( x − a)
1 1 I= a − ( x − a)2 + sin−1 +c
I=ò dx = ò dx 2 2 a
2 2
2 - 3x - x æ 17 ö æ 3ö
2
çç ÷÷ - ç x + ÷
I=
( x − a) 2ax − x 2 +
a2 −1 ( x − a)
sin +c
è 2 ø è 2ø
2 2 a
 3
Put  x +  = t . Then dx = dt . px + q px + q
 2 2. ∫ (ax 2 + bx + c ) dx , ∫ dx ,
(ax 2 + bx + c )
1 æ 2t ö -1 æ 2 x + 3 ö
I=ò dt = sin-1 ç ÷ + c = sin ç ÷+c ∫ ( px + q) (ax 2 + bx + c ) dx
æ 17 ö
2 è 17 ø è 17 ø
2
çç ÷÷ - t In these types of integrals, we write px + q =  (differential coef-
è 2 ø ficient of ax2 + bx + c) + m.
1  Find  and m by comparing the coefficient of x and constant
Illustration 22.59  Evaluate ò 2
dx. terms on both sides of the identity. In this way, the question
x - 4x +2 will reduce to the sum of two integrals, which can be inte-
Solution:
grated easily.
1 1 x +1
I=∫ dx = ∫ dx Illustration 22.62  Evaluate ∫ dx.
2
x − 4x + 2 ( x − 2) − ( 2 )2
2
( 2 x 2 + x − 3)

Put ( x − 2) = t . Then dx = dt . Solution: Let x + 1 = A (differential coefficient of 2x2 + x – 3) + B.


Then
1
I=∫ dt x +1 = A(4x + 1) + B = 4Ax + A + B
t − ( 2 )2
2

1 3
⇒ I = ln t + t 2 − ( 2 )2 + c Equating the coefficients, A = , B = . We get
4 4
x +1
⇒ I = ln ( x − 2) + x 2 − 4 x + 2 + c I=∫ dx
( 2 x 2 + x − 3)
1 4x +1 3 1
Illustration 22.60  Evaluate ò x 2 + 8 x + 12 dx. I= ∫ dx + ∫ dx
4 ( 2 x 2 + x − 3) 4 ( 2 x 2 + x − 3)
Solution:
1 4x +1 3 1
4 ∫ ( 2 x 2 + x − 3)
Let I1 = dx and I2 = ∫ dx
I = ∫ x 2 + 8 x + 12 dx = ∫ ( x + 4 )2 − (2)2 dx 4 (2 x 2 + x − 3)

Mathematical Problem Book for JEE.indb 981 07-06-2018 13:59:18


982 Mathematics Problem Book for JEE

Put (2 x 2 + x − 3) = t . Then ( 4 x + 1) dx = dt . Putting 6 + x − x 2 = t . Then (1− 2 x )dx = dt , we have

1 1 1 1 1
I1 = ò dt = 2 t + k1 = 2 6 + x - x 2 + k1
4∫ t
I1 = dt = t + k1 = (2 x 2 + x − 3) + k1
2 2 t
3 1 3 1 And
2ò 2ò
I2 = dx = dx dx dx
4 æ 2 x 3ö 4 ææ 1ö æ 5ö ö
2 2 I2 = ò =
çx + - ÷ çç x + ÷ -ç ÷ ÷ 6+ x - x 2
6 - ( x2 - x)
è 2 2ø çè 4 ø è 4 ø ÷ø
è
4 1 1
æ ææ 2 ö I2 = ∫ dx = ∫ dx = ∫ dx
1ö æ 5ö ö
2
æ3 1ö 2  1 1 2
= ln ç ç x + ÷ + ç ç x + ÷ - ç ÷ ÷ ÷ + k2 6 − (x − x) 6 −  x2 − x + −  25  1
−x − 
4 2 çç è 4ø çè
è 4 ø è 4 ø ÷ø ÷÷  4 4 4  2
è ø
1 4 1
Hence, =∫
dx   I2 = ∫ dx = ∫ dx
 2 1 1 2 2
6 − (x − x) 6 − x − x + − 25  1
1 3  1  x 3    −x − 
I= (2 x 2 + x − 3) + ln   x +  +  x 2 + −   + c 4 4 4  2
2 4 2   4   2 2  
æ2æ 1 öö æ 2 x - 1ö
= sin-1 ç ç x - ÷ ÷ + k2 = sin-1 ç ÷ + k2
3x + 2 è 5è 2 øø è 5 ø
Illustration 22.63  Evaluate ∫ dx.
( 4 x 2 + 4 x + 5)  2 x − 1
I1 + 4 I2 = 2 6 + x − x 2 + 4 sin−1  + k + 4 k2
Solution: Express  5  1
3x + 2 = l (differential coefficient of 4x2 + 4x + 5) + m  2 x − 1
⇒ I = 2 6 + x − x 2 + 4 sin−1  +c
⇒3x + 2 = l (8x + 4) + m = 8lx + 4l + m  5 
Comparing the coefficients, we get
x +2
8l = 3 and 4l + m = 2⇒ l = 3/8 and m = 2 − 4l = 1/2 Illustration 22.65  Evaluate ò 2
dx.
x - 2x + 4
3 8x + 4 1 1 Solution:
⇒I = ∫ 2
8 ( 4 x + 4 x + 5)
dx + ∫ 2
2 ( 4 x + 4 x + 5)
dx
x +2 x − 1+ 3
I=∫ dx = ∫ dx
3 1
= ln( 4 x 2 + 4 x + 5) + ∫
1
dx
2
x − 2x + 4 (x 2
− 2x + 4 )
8 8  2 5
 x + x +  x −1 3
4 I=∫ dx + ∫ dx
3 1  1 (x 2
− 2x + 4 ) ( x − 1) + ( 3 )2
2

= ln( 4 x 2 + 4 x + 5) + tan−1  x +  + c
8 8  2
Put x 2 − 2 x + 4 = t 2 . Then
5 - 2x (2 x − 2)dx = 2tdt ⇒ ( x − 1) dx = tdt
Illustration 22.64  Evaluate ò 2
dx.
6+ x - x t 3
Solution: ⇒ I = ∫ dt + ∫ dx
t ( x − 1) + ( 3 )2
2
5 - 2x
I=ò dx
6 + x - x2 I = x 2 − 2 x + 4 + 3 ln x + x 2 − 2 x + 4 + c
d
Let 5 − 2 x = M (6 + x − x 2 ) + N = M(1− 2 x ) + N
dx Illustration 22.66  Evaluate ∫ (2 x + 3) x 2 + 4 x + 3 dx.
Equating the coefficients of x and constant terms on both sides,
we get d 2
Solution: Let (2 x + 3) = M ( x + 4 x + 3) + N . Then
M = 1, M + N = 5 ⇒ N = 4 dx
( 2 x + 3 ) = M( 2 x + 4 ) + N
Therefore,
5 - 2 x = 1- 2 x + 4 Equating the coefficients of x and constant terms on both sides,
we get
Hence, M = 1, N = 3 − 4 M = −1
(1− 2 x ) + 4 (1− 2 x ) 4 Therefore,
I=∫ dx = ∫ dx + ∫ dx == II11++44I
I22
6+ x − x 2
6+ x − x 2
6+ x − x 2 ( 2 x + 3) = ( 2 x + 4 ) − 1
Now, I = ∫ ((2 x + 4 ) − 1) x 2 + 4 x + 3 dx
(1− 2 x )
I1 = ∫ dx
6 + x − x2 I = ∫ (2 x + 4 ) x 2 + 4 x + 3 dx − ∫ x 2 + 4 x + 3 dx = I1 - I2

Mathematical Problem Book for JEE.indb 982 07-06-2018 13:59:53


Chapter 22 | Indefinite Integration 983

To evaluate the integrals of the above form, divide the numera-


Putting x 2 + 4 x + 3 = t . Then (2 x + 4 )dx = dt , we have
tor by the denominator. Then, the integrals take the form given by
t 3/ 2 2 f (x) R( x )
I1 = ∫ t1/ 2 dt = + c1 = ( x 2 + 4 x + 3)3/ 2 + c1 = Q( x ) +
3/2 3 ax 2 + bx + c ax 2 + bx + c
I2 = ∫ x 2 + 4 x + 3 dx = ∫ ( x + 2)2 − 1dx where Q( x ) is a polynomial and R( x ) is a linear polynomial in x.
( x + 2) 2 1
= x + 4 x + 3 − ln ( x + 2) + x 2 + 4 x + 3 + c2 Then, we have
2 2
f (x) R( x )
I = I1 - I2 ∫ ax 2 + bx + c dx = ∫ Q( x ) dx + ∫ ax 2 + bx + c dx
( x 2 + 4 x + 3)3 2 ( x + 2) 2 1
= − x + 4 x + 3 + ln ( x + 2)+ x 2 + 4 x + 3 + c The integrals on RHS can be obtained by the methods dis-
32 2 2 cussed earlier.
( x 3 + 8 )( x - 1)
Illustration 22.67  Evaluate ∫ (2 x − 5) x 2 − 4 x + 3 dx. Illustration 22.68  Evaluate ò x2 - 2x + 4
dx.

Solution: Solution:

( x 3 + 8 )( x − 1) ( x − 1)( x 2 − 2 x + 4 )( x − 1)
I = ∫ (2 x − 5) x 2 − 4 x + 3 dx I=∫ 2
dx = ∫ dx
x − 2x + 4 x2 − 2x + 4
d 2
Let (2 x − 5) = M + Nd. Then
( x (2−x4−x 5+) 3=) M ( x 2 − 4 x + 3) + N . Then
dx dx x3 x2
I = ∫ ( x + 2)( x − 1) dx = ∫ ( x 2 + x − 2) dx = + − 2x + c
(2 x − 5) = M(2 x − 4(2) +x N
− 5 ) = M( 2 x − 4 ) + N 3 2
Equating the coefficients of x and constant terms on both sides,
2x3 - 3x2 + 5x + 6
we get Illustration 22.69  Evaluate ò x2 + 3x + 2
dx.

M = 1, N = 4 M − 5 = −1 Solution:
Therefore, 2x3 - 3x2 + 5x + 6
I=ò dx
x2 + 3x + 2
(2 x − 5) = (2 x − 4) − 1
 8x 
I = ∫ ((2 x − 4 ) − 1) x 2 − 4 x + 3 dx I = ∫  ( 2 x + 3) − 2  dx
 x + 3x + 2
I = ∫ (2 x − 4 ) x 2 − 4 x + 3 dx − ∫ x 2 − 4 x + 3 dx 2x + 3 1
I = ∫ (2 x + 3)dx − 4 ∫ dx + 12∫ dx
I = I1 - I2 x2 + 3x + 2 ( x + 1)( x + 2)

 1 1 
( x 2 − 4 x + 3)3 2 I = ( x 2 + 3 x ) − 4 ln x 2 + 3 x + 2 + 12∫  − dx
I1 = ∫ (2 x − 4 ) x − 4 x + 3 dx =
2
+ c1  x + 1 x + 2 
32
I = ( x 2 + 3 x ) - 4 ln ( x + 1)( x + 2) + 12 ln x + 1 - 12 ln x + 2 + c
2 2
I2 = ò x - 4 x + 3 dx = ò x - 4 x + 4 - 4 + 3 dx
I = ( x 2 + 3 x ) + 8 ln x + 1 - 16 ln x + 2 + c = x 2 - 3 x + 8 ln( x + 1) - 16 ln( x + 2) + c
= ∫ ( x − 2)I2=−( 1x22 dx
+ 3 x ) + 8 ln x + 1 - 16 ln x + 2 + c = x 2 - 3 x + 8 ln( x + 1) - 16 ln( x + 2) + c
    
( x − 2) 2 1
= x − 4 x + 3 − ln ( x − 2) + x 2 − 4 x + 3 + c2
2 2
ax 2 + bx + c ax 2 + bx + c
Now, 4. ∫ 2 dx , ∫ dx
px + qx + r px 2 + qx + r
I = I1 - I2
In this case, substitute
( x 2 − 4 x + 3)3/2 ( x − 2) 2
⇒I = − x − 4x + 3 + dd 22
3/ 2 2 ax22++bx
ax ( px22++qx
bx++cc==MM( px qx++r r) )++NN ( px
( px ++qx
qx++r r) )++RR
dx
dx
1
ln ( x − 2) + x 2 − 4 x + 3 + c Find M, N and R by comparing the coefficient of x2, x and constant
2
terms on both sides of the identity.
f (x)
3. ∫ ax 2 + bx + c dx where f ( x ) is a polynomial of degree greater x2 + x +1
Illustration 22.70  Evaluate ò x 2 + 3 x + 2 dx.
than 2.

Mathematical Problem Book for JEE.indb 983 07-06-2018 14:00:29


984 Mathematics Problem Book for JEE

Solution:
x2 1 x 2 + a2 x 2 − a2
2
x + x +1
5. ∫ x 4 + 1dx , ∫ x 4 + 1dx , ∫ x 4 + kx 2 + 1dx , ∫ x 4 + kx 2 + 1dx ,(k ∈R )
I=ò dx
x2 + 3x + 2 Rule for this form:
d
x + x + 1 = M( x + 3 x + 2 ) + N ( x 2 + 3 x + 2 ) + R
2 2
(a) To evaluate these types of integrals divide the numerator
dx
and denominator by x2.
x 2 + x + 1= Mx 2 + (3M + 2N ) x + (2M + 3N + R )
1 1 a2 a2
Comparing coefficients of variables and constant terms (b) Put x + = t or x - = t and x + = t or x - = t as
x x x x
M = 1, (3M + 2N ) = 1,(2M + 3N + R ) = 1 required.
M = 1, N = −1, R = 2 dx dx
d 2 ∫Similar
tan x form ∫ xtan
dx , ∫ is:cot dx ,x∫dx , 4∫ cot x 4dx ,, ∫ 4 4
,
( x 2 + 3 x + 2) −
( x + 3 x + 2) + 2 sin x + cos x sin x + cos x
I=∫ dx dx dx x ± cos x ) ( ± sin x ± cos x ) dx
( ± sindx
x2 + 3x + 2 ∫ sin6 x + cos6 x , ∫ sin6ax++bcos 6
sin x x
, ∫dx
a + b sin x
2x + 3 2
= ∫ 1dx − ∫ 2 dx + ∫ dx 5
x + 3x + 2 ( x + 1)( x + 2) Illustration 22.72  Evaluate ∫ x 4 + 1dx.
x +1
I = x - ln x 2 + 3 x + 2 + ln +c Solution:
x +2
5 5  ( x 2 + 1) − ( x 2 − 1) 
I=∫
2 ∫
x2 - x +1 dx =  dx 
Illustration 22.71  Evaluate ò dx. x4 +1 x4 +1 
x2 + 4 x + 3
Solution:   1  1 
1+  1− 2  
2
x - x +1 5   x 2  x
I=ò dx = ∫ dx − ∫ dx 
2  x2 + 1 x 2
+
1 
x2 + 4 x + 3  x2 x 2 
d 2
x 2 − x + 1 = M( x 2 + 4 x + 3 ) + N ( x + 4 x + 3) + R  
dx  1  1
  1+ 2   1− 2   5
x 2 − x + 1 = Mx 2 + ( 4 M + 2N ) x + (3M + 4 N + R ) 5  x   x 
= ∫ 2
dx − ∫ 2
dx  = (I1 − I2 )
2  1  1  2
Comparing coefficients of variables and constant terms  x −  +2 x +  −2 


  x x 
M = 1, ( 4 M + 2N ) = −1, (3M + 4 N + R ) = 1
5  1
M = 1, N = − , R = 8  1+ 2 
2 I1 = ∫ x dx
2
5 d 2  1
( x 2 + 4 x + 3) − ( x + 4 x + 3) + 8  x −  + 2
2 dx x
I=∫ dx
( x 2 + 4 x + 3)
For I1, we write
5 (2 x + 4 ) 1 1 æ 1 ö
=∫ ( x + 4 x + 3) dx − ∫
2
dx + 8∫ dx x- = t Þ ç 1+ 2 ÷ dx = dt
2 ( x 2 + 4 x + 3) 2
( x + 4 x + 3) x è x ø
5 1
I = I1 - I2 + 8I3 I1 = ò dt
2 t2 + 2
I1 = ∫ x 2 + 4 x + 3 dx = ∫ ( x + 2)2 − 1 dx  1
1 t 1  x − 
( x + 2) 2 1 x
= x + 4 x + 3 − ln ( x + 2) + x 2 + 4 x + 3 + c1 I1 = tan−1 +c = tan−1 + c1
2 2 2 2 2 2
(2 x + 4 )
I2 = ∫ dx = 2 x 2 + 4 x + 3 + c2 For I2, we write
x2 + 4 x + 3
1 1 æ 1 ö
I3 = ∫ dx = ln ( x + 2) + x 2 + 4 x + 3 + c3 x+ = t Þ ç 1- 2 ÷ dx = dt
x è x ø
( x + 2)2 − 1
5 5 1
I = I1I −
= I1 I−2 + 8I 2I3+ 8I3 1 1 t− 2 1 x+ − 2
2 2 I2 = ∫ 2 dt = ln +c = ln x + c2
( x − (8x) − 8)2 2 15 15 t −2 2 2 t+ 2 2 2 x+ 1+ 2
⇒ I=
⇒ I= x +x4 x++43x++ 3 +ln ( xln+ (2x) + 2) x+2 +x42x++43x ++c3 + c
2 2 2 2 x

Mathematical Problem Book for JEE.indb 984 07-06-2018 14:01:01


Chapter 22 | Indefinite Integration 985

Combining the two integrals, we get Put tan x = t 2 . Then


æ æ 1ö 1 ö 2t
5ç 1 çx- ÷ 1
x+ - 2 ÷ sec2 x dx = 2tdt ⇒ dx = dt
-1 è x ø x 1+ t 4
I= ç tan - ln ÷+k
2ç 2 2 2 2 x+ 1+ 2 ÷
ç x ÷  1  1
è ø  1+ 2   1+ 2 
 1 2t t t
I = ∫ t +  dt = 2∫ dt = 2∫ dt
 t  1+ t 4
∫  2 1  2
Illustration 22.73  Evaluate tan x dx . t + − 2 + 2  1
   t −  + 2
t2 t
Solution: Put tan x = t 2 . Then
2t  1  1
sec2 x dx = 2tdt ⇒ dx = dt Put  t −  = p. Then  1+ 2  dt = dp.
4  t  t 
1+ t
2t ⋅ t t2 +1 t2 −1  1
I = ∫ tan x dx = ∫ dt = ∫ dt + ∫ dt
1+ t 4
1+ t 4
1+ t 4  t − 
1 2 −1 p 2 −1  t
I = 2∫ 2 dp = tan +c = tan +c
 1  1 p +2 2 2 2 2
 1+ 2   1− 2 
I=∫ t dx + ∫ t dx
 1
2
 1
2  1 
 t −  + 2  t −  − 2  tan x − 
t t −1  tan x 
⇒ I = 2 tan +c
I = I1 + I2 2

6. Substitution for some irrational functions:


 1  1
For I1, we write  t −  = z ⇒  1+ 2  dx = dz
 t  t  n
∫ f ( x ,(ax + b)
1n
(a) ) dx, put (ax + b ) = t
1
I1 = ò dt dx
z2 + 2 (b) ∫ ( px + q) (ax + b )
, put (ax + b ) = t 2
æ 1ö
1 çt - ÷
-1 z 1 -1 è tø dx 1
I1 =
2
tan
2
+c =
2
tan
2
+c (c) ∫ 2
, put ( px + q ) =
t
( px + q ) (ax + bx + c )
æ 1 ö
1 ç tan x - ÷ dx 1
tan xø
I1 = tan-1 è +c (d) ∫ , first put x =
t
and then (a + bt 2 ) = z 2
2 2 ( px 2 + q ) (ax 2 + b )

1  1
For I2, we write t + = z ⇒  1− 2  dx = dz (ax 2 + bx + c )dx
t  t  (e) ∫ , here, we write
(dx + e ) (fx 2 + gx + h)
1
t+ − 2
1 1 z− 2 1 t
I2 = ∫ 2 dt = ln +c = ln + c1 ax2 + bx + c = A1 (dx + e) (2fx + g) + B1(dx + e) + C1
z −2 2 2 z+ 2 2 2 t + 1+ 2 where A1, B1 and C1 are constants which can be obtained by
t comparing the coefficient of like terms on both sides.
1
tan x + − 2 x dx
1 tan x Illustration 22.75  Evaluate ∫ .
I2 = ln + c2
2 2 1 ( x − 3) x + 1
tan x + + 2
tan x Solution: Put x + 1 = t 2 . Then dx = 2tdt , we get
Combining the two integrals, we get
2t (t 2 − 1)
æ æ 1 ö 1 ö I= ∫ (t 2 − 4 )t dt
ç
1 ç tan x - ÷ tan x + - 2 ÷
I =ç tan-1 è tan x ø + 1 ln tan x ÷+k  3  3 t −2
ç 2 2 2 2 1 ÷ = 2 ∫  1+ 2 dt = 2t + ln +c
ç tan x + + 2 ÷  (t − 4 )  2 t +2
è tan x ø
3 x +1− 2
= 2 x + 1 + ln +c
Illustration 22.74  Evaluate ∫ ( tan x + cot x ) dx. 2 x +1+ 2

Solution: dx
Illustration 22.76  Evaluate ∫ .
I = ∫ ( tan x + cot x ) dx ( x − 3) x + 1

Mathematical Problem Book for JEE.indb 985 07-06-2018 14:01:39


( x 2 − x + 1)
1 1
I2 = ∫ dx = ∫
2
( x − x + 1)  1
2
3
 x −  +
2 4
 1
= ln  x −  + ( x 2 − x + 1) + c2
 2
986 Mathematics Problem Book for JEE 1
I3 = ∫ dx (Same as Illustration 22.77)
( x + 1) ( x 2 − x + 1)
Solution: Put x + 1 = t 2 . Then dx = 2tdt , we get 2
1 1
2t 1 1 t −2 =− ln  1− x  +  1− x  + + c3
 
3  2(1+ x )    12
I= ∫ (t 2 − 4 )t dt = 2∫ (t 2 − 4 ) dt = 2 4 ln t + 2 + c  2(1+ x ) 

x +1− 2  1
1
= ln +c I = 2 ( x 2 − x + 1) + 2 ln  x −  + ( x 2 − x + 1) −
 2
2 x +1+ 2

dx 1 2 1
Illustration 22.77  Evaluate ∫ . ln  1− x  +  1− x  + +c
2
( x + 1) ( x − x + 1)  
3  2(1+ x )   
 2(1+ x )  12
Solution:
dx dx
I=∫ Illustration 22.79  Evaluate ∫ .
( x + 1) ( x 2 − x + 1) ( x + 2) x 2 + 1
2

1 1 Solution:
Put x + 1 = . Then dx = − dt , we get dx
t t2 I=∫
1 1 ( x + 2) x 2 + 1
2
I = −∫ dt = − ∫ dt
1 3t 2 − 3t + 1
t2. (1− t )2 − t (1− t ) + t 2 1 1
t2 Put x = . Then dx = − 2 dt , we get
t t
2
1 1 1 æ 1ö æ 1ö 1 dt tdt
3 ò æ 1 ö2 1
I=- dt = - ln ç t - ÷ + ç t - ÷ + +c I = −∫ = −∫
3 è 2ø è 2 ø 12  1  1 (1+ 2t 2 ) t 2 + 1
çt - ÷ + t 2  2 + 2 2 + 1
è 2 ø 12 t  t

1  1− x   1− x  1
2 Put (1+ t 2 ) = z 2 . Then tdt = zdz , we get
I= − ln   +   + +c
3  ( 2 1 + x )  ( )
2 1 + x 12 z dz dz
I = −∫ = −∫
1+ 2( z 2 − 1) z (2 z 2 − 1)
 
Illustration 22.78  Evaluate (2 x 2 + 3 x + 2)dx
∫ z-
1
( x + 1) ( x 2 − x + 1) 1 dz 1 2 +c
Solution: I=- ò =- ln
2 æ 2 1ö 2 2 1
çz - ÷ z+
è 2ø 2
(2 x 2 + 3 x + 2)dx
I=∫
( x + 1) ( x 2 − x + 1) 1 2z − 1 1 2(1+ t 2 ) − 1
I=− ln +c = − ln +c
d 2 2 2 2z + 1 2 2 2(1+ t 2 ) + 1
(2 x 2 + 3 x + 2) = a( x + 1) ( x − x + 1) + b( x + 1) + c
dx
a = 1, b = 2, c = 1 1 2(1+ x 2 ) − x
I=− ln +c
2 2 2(1+ x 2 ) + x
( x + 1)(2 x − 1) + 2( x + 1) + 1
I=∫ dx
( x + 1) ( x 2 − x + 1)
(2 x − 1) 1 1

I=∫
2
dx + 2∫
2
dx + ∫ dx Your Turn 4
( x − x + 1) ( x − x + 1) ( x + 1) ( x 2 − x + 1)
I = I1 + 2I2 + I3 dx
1. ∫ ( x − x2 ) =
(2 x − 1)
I1 = ∫ dx = 2 ( x 2 − x + 1) + c1
( x 2 − x + 1) (A) ln x − ln(1− x ) + c (B) ln(1− x 2 ) + c
1 1
I2 = ∫ dx = ∫ (C) − ln x + ln(1− x ) + c (D) ln( x − x 2 ) + c  Ans. (A)
2
( x − x + 1)  1
2
3
 x −  + dx
2 4 2. ∫ (1+ x + x 2 + x 3 ) =
 1
= ln  x −  + ( x 2 − x + 1) + c2
 2
1 1
1 (A) ln 1+ x - ln 1+ x 2 + tan-1 x + c
I3 = ∫ dx (Same as Illustration 22.77) 2 2
( x + 1) ( x 2 − x + 1) 1
(B) ln 1+ x - ln 1+ x 2 + tan-1 x + c
1 1
2 2
=− ln  1− x  +  1− x  + + c3
3  2(1+ x )  
2(1+ x )
 12

Chapter 22.indd 986 11-06-2018 10:13:09


Chapter 22 | Indefinite Integration 987

1 1 x 2 dx
(C) ln 1+ x 2 - ln 1+ x + tan-1 x + c 9.
2 2 ∫ ( x 2 + 2)( x 2 + 3) =
(D) ln 1+ x 2 + ln 1+ x + tan-1 x + c Ans. (A) (A) - 2 tan-1 x + 3 tan-1 x + c

( x − 1)dx x x
(B) - 2 tan-1 + 3 tan-1 +c
3. ∫ ( x − 2)( x − 3) = 2 3
x x
(A) ln( x − 3) − ln( x − 2) + c (C) 2 tan-1 + 3 tan-1 +c
2 3
(B) 2 ln( x − 3) − ln( x − 2) + c (D) None of theseAns. (B)
(C) ln( x − 3) + ln( x − 2) + c dx
10. ò 2 =
Ans. (B) ( x + 1)( x 2 + 4 )
(D) 2 ln( x − 3) + ln( x − 2) + c 
1 1 x
dx (A) tan-1 x - tan-1 + c
4. ∫ = 3 3 2
cos x (1+ cos x ) 1 1 x
(B) tan-1 x + tan-1 + c
x 3 3 2
(A) ln(sec x + tan x ) + 2 tan + c
2 1 1 x
(C) tan-1 x - tan-1 + c
x 3 6 2
(B) ln(sec x + tan x ) − 2 tan + c
2 x
x (D) tan-1 x - 2 tan-1 + cAns. (C)
(C) ln(sec x + tan x ) + tan + c 2
2 7. Substitution for trigonometric functions:
x
(D) ln(sec x + tan x ) − tan + c Ans. (D) dx dx
2 (a) ∫ (a + b cos x ) , ∫ (a + b sin x )
dx x x
5. ∫ (1+ x )(2 + x ) = 1− tan2
2
2 tan
2 x
Use cos x = , sin x = and put tan = t.
2 x 2 x 2
(A) ln( x + 2) − ln( x + 1) + c (B) ln( x + 2) + ln( x + 1) + c 1+ tan 1+ tan
2 2
(C) ln( x + 1) − ln( x + 2) + c (D) None of these Ans. (C) dx
Illustration 22.80  Evaluate ∫ .
(2 + cos x )
x dx
6. Correct evaluation of ∫ is Solution:
( x − 1)( x − 2) x
dx dx sec2
(A) 2 ln( x − 2) − ln( x − 1) + c (B) ln( x − 1) − ln( x − 2) + c I=∫ 2
(2 + cos x ) ∫  x ∫
= = dx
x
1− tan2 3 + tan2
( x − 1) ( x − 2)  2 2
(C)
( x − 2)
+ c (D) 2 ln
( x − 1)
+c Ans. (A) 2 + 2 x

 1+ tan 
2
dx x 1 x
= t . Then sec2 dx = dt .
7. ∫ ( x − 1)( x 2 + 1) = Put tan
2 2 2
 x
1 1 1 tan
(A) ln( x − 1) − ln( x 2 + 1) − tan−1 x + c dt 2 −1  t  2 −1  2 
I = 2∫ = tan  +c = tan   +c
2 4 2 (3 + t 2 ) 3  3  3 3
1 1 1  
(B) ln( x − 1) + ln( x 2 + 1) − tan−1 x + c
2 4 2
dx
1 1 1 Illustration 22.81  Evaluate ∫ .
(C) ln( x − 1) − ln( x 2 + 1) − tan−1 x + c (5 + 4 cos x )
2 2 2
Solution:
(D) None of theseAns. (A)
dx
I=∫
8. ( x 2 + x − 1) dx (5 + 4 cos x )
∫ ( x 2 + x − 6)
=
 2 x x
(A) x + ln( x + 3) + ln( x − 2) + c  1+ tan  dx ec2
se
2 2
I=∫ =∫ dx
  2 x  2 x 2 x
(B) x − ln( x + 3) + ln( x − 2) + c 5
  1 + tan  + 4 1 − tan 9 − tan
2  
2   2
(C) x − ln( x + 3) − ln( x − 2) + c x 1 x
Put tan = t . Then sec2 dx = dt .
(D) None of theseAns. (B) 2 2 2

Mathematical Problem Book for JEE.indb 987 07-06-2018 14:03:00


988 Mathematics Problem Book for JEE

 x æ 2 xö
dt 2 −1  t  2 −1 
tan
2  ç 1+ tan ÷ dx
I = 2∫ = tan   + c = tan  +c è 2ø
I=ò
(9 + t 2 ) 3  3 3 3  æ x x 2 xö
  2
ç 1+ tan - 2 tan - 1+ tan ÷
è 2 2 2ø
dx dx x
(b) ∫ (a sin x + b cos x + c ) , ∫ (a cos x + b sin x ) sec2 dx
2
I=ò
x x æ 2 x xö
1− tan2 2 tan ç 2 tan - 2 tan ÷
2 2 x è 2 2 ø
Use cos x = , sin x = and put tan = t
2 x 2 x 2 x 1 x
1+ tan 1+ tan = t . Then sec2 dx = dt . Therefore,
Put tan
2 2
2 2 2
b
Or a = r cos a and b = r sina ⇒ r = a2 + b2 and a = tan−1 . dt dt  1 1
I=∫ 2 =∫
t (t − 1) ∫  t − 1 t 
a =  −  dt
(t − t )
dx
Illustration 22.82  Evaluate ∫ .
(sin x + cos x + 2) t −1
I = ln(t − 1) − ln t + c = ln +c
t
Solution:
x
 2 x tan − 1
dx  1+ tan  dx ⇒ I = ln 2 +c
2 x
I=∫
(sin x + cos x + 2) ∫  
= tan
2 x  2 x x 2
 2  1+ tan 2  +  1− tan 2  + 2 tan 2 
dx dx
x
(c)  ò 2
(a + b cos x )

(a + b sin2 x )
,
sec2
dx
I=ò 2
æ 2x x ö dx dx dx
ç tan + 2 tan + 3 ÷ ∫ (a sin2 x + b cos2 x + c ) , ∫ (a cos2 x + b sin2 x ) , ∫ (a cos x + b sin x )2
è 2 2 ø
x 1 x Divide the numerator and the denominator by sin2 x or cos2 x and
Put tan = t . Then sec2 dx = dt .
2 2 2 put tan x = t.
dt dt 2  t + 1 dx
I = 2∫ = 2∫ = tan−1  +c Illustration 22.85  Evaluate ò .
(t 2 + 2t + 3) (t + 1)2 + 2 2  2  (1+ 2 cos2 x )
 
Solution:
x 
tan + 1 dx sec2 x dx sec2 x dx
2 
−1  I=∫ =∫ =∫
⇒ I = 2 tan  +c
2  (1+ 2 cos2 x ) (sec2 x + 2) (tan2 x + 3)
 
(Dividing the numerator and the denominator by cos2 x )
dx
Illustration 22.83  Evaluate ∫ . Put tan x = t . Then sec2 x dx = dt .
( 4 sin x + 3 cos x )
Solution: dt
I=∫
dx ( t 2 + 3)
I=∫
( 4 sin x + 3 cos x )
1 æ t ö 1 æ tan x ö
I= tan-1 ç ÷+c = tan-1 ç ÷+c
3 4  4 3 è 3ø 3 è 3 ø
3 cos x + 4 sin x = 5  cos x + sin x  = 5 cos( x − a ), tana =
5 5  3
dx
1 dx 1 1 Illustration 22.86  Evaluate ∫ .
(3 cos x + sin x )2
5 ∫ cos( x − a ) 5 ∫
I= = sec( x − a )dx = [sec( x − a ) + tan( x − a )] + c
5
Solution:
dx
Illustration 22.84  Evaluate ∫ . dx
(1− sin x − cos x ) I=∫
(3 cos x + sin x )2
Solution:
(Dividing the numerator and the denominator by cos2 x )
dx dx
I=∫
(1− sin x − cos x ) ∫ 
= dx sec2 x dx
x x I=∫ =∫
2 tan 1− tan2 2
 2 2  (3 cos x + sin x ) (3 + tan x )2
 1− 2 x

2 x

 1+ tan 1+ tan 
2 2 Put tan x = t . Then sec2 x dx = dt .

Mathematical Problem Book for JEE.indb 988 07-06-2018 14:03:36


Chapter 22 | Indefinite Integration 989

dt 1 (a cos x + b sin x )
I=∫ =− +c ò ( p cos x + q sin x ) dx
(3 + t )2 (3 + t )
1 (a cos x + b sin x + c ) = l ( p cos x + q sin x + r )
I=− +c
(3 + tan x ) d
+ m ( p cos x + q sin x + r ) + n
dx
cos x
Illustration 22.87  Evaluate ∫ cos 3 x dx . Compare both side coefficients of sin x, cos x and constant term,
and calculate the value of l, m and n.
Solution:
( 4 cos x + 5 sin x )
cos x cos x 1 Illustration 22.89  Evaluate ∫ dx .
I=∫ dx = ∫ dx = ∫ dx (2 cos x + 3 sin x )
cos 3 x ( 4 cos3 x − 3 cos x ) ( 4 cos2 x − 3) Solution:
( 4 cos x + 5 sin x )
(Dividing the numerator and the denominator by cos2 x ) I=∫ dx
(2 cos x + 3 sin x )
sec2 x sec2 x d
( 4 cos x + 5 sin x ) = a(2 cos x + 3 sin x ) + b
(2 cos x + 3 sin x )
I=∫ dx = ∫ dx
( 4 − 3(1+ tan x )) 2
(1− 3 tan2 x ) dx
= a(2 cos x + 3 sin x ) + b( −2 sin x + 3 cos x )

Put tan x = t . Then sec2 x dx = dt . Comparing the coefficients of cos x and sin x , we get
1 1 1 23 2
I=∫ a= ,b=

(1− 3 t )2
dt = ∫
3  1 2
dt 13 13
 − t  23 2 (3 cos x − 2 sin x )
3 I = ∫ 1dx + ∫ dx
13 13 (2 cos x + 3 sin x )
1 1+ 3t 1 1+ 3 tan x 23 2
I= ln +c = ln +c = x + ln 2 coss x + 3 sin x + c
2 3 1- 3t 2 3 1- 3 tan x 13 13

dx (cos x − 3 sin x + 4 )
Illustration 22.88  Evaluate ∫ 2 2 Illustration 22.90  Evaluate ∫ dx.
( 4 sin x + 5 cos x + 4 sin x cos x ) (cos x + sin x + 2)
Solution: Solution:
dx (cos x − 3 sin x + 4 )
I=∫ 2 2
I=∫ dx
( 4 sin x + 5 cos x + 4 sin x cos x ) (cos x + sin x + 2)

(Dividing the numerator and the denominator by cos2 x ) d


(cos x − 3 sin x + 4) = a(cos x + sin x + 2) + b (cos x + sin x + 2) + c
dx
dx sec2 xdx = a(cos x + sin x + 2) + b( − sin x + cos x ) + c
I=∫ 2 2
=∫ 2
( 4 sin x + 5 cos x + 4 sin x cos x ) ( 4 tan x + 5 + 4 tan x ) Comparing the coefficients of cos x , sin x and constant, we get
1 dx a = −1, b = 2, c = 6
4 ò ææ
I= (cos x − sin x ) dx

2 ö I = − ∫ 1dx + 2∫ dx + 6∫
ç ç tan x + ÷ + 1÷ (cos x + sin x + 2) (cos x + sin x + 2)
çè 2ø ÷
è ø
For IIIrd integral,
1  x
Put tan x + = t . Then sec2 x dx = dt .  1+ tan 
2
2 dx 2
6∫ = 6∫ dx
(cos x + sin x + 2) x x x
1 dx 1 1  1 1− tan2 + 2 tan + 2 + 2 tan2
= tan−1 t + c = tan−1  tan x +  + c
4 ∫ (t 2 + 1) 4
I= 2 2 2
4  2 x x
sec2 sec2
= 6∫ 2 dx = 6 ∫ 2 dx
( a cos x + b sin x ) x x 2
(d)  ò ( p cos x + q sin x ) dx tan + 2 tan + 3
2
2 2

 1 + tan
x 
 + 2
2
d
(a cos x + b sin x ) = l ( p cos x + q sin x ) + m
( p cos x + q sin x ) x
dx
(cos x − sin x ) sec2
⇒ I = − ∫ 1dx + 2∫ dx + 6∫ 2 dx
Compare both side coefficients of sin x and cos x, and calculate the (cos x + sin x + 2) 2
 x
value of l and m.  1+ tan  + 2
2

Mathematical Problem Book for JEE.indb 989 07-06-2018 14:04:04


990 Mathematics Problem Book for JEE

 x  Put cos x = t . Then − sin x dx = dt .


  1+ tan  
2
I = − x + 2 ln (cos x + sin x + 2) + 3 2 tan−1   +c t5 t3
 2  I = − ∫ (t 2 − t 4 ) dt = − +c
  5 3
sin5 x sin3 x
= − +c
dx 5 3
Illustration 22.91  Evaluate ò 1+ cot x .
dq
Solution: Illustration 22.93  Evaluate ∫ sinq ⋅ cos3 q .
dx sin x Solution:
I=ò
1+ cot x ò sin x + cos x
= dx
dq sec2 q sec2 q
d I=∫ 3
=∫ dq = ∫ dq
sin x = M (sin x + cos x ) + N(sin x + cos x ) sinq ⋅ cos q sinq ⋅ cosq tanq ⋅ cos2 q
dx
= M( − sin x + cos x ) + N(sin x + cos x ) Put tanq = t . Then sec2 q dq = dt .
Comparing the coefficients of sin x and cos x, we have 1+ t 2 æ1 ö
I=ò dt = ò ç + t ÷ dt
− M + N = 1, M + N = 0 t èt ø

1 1 t2 tan2 q
Solving these equations, we have M = − , N = . I = ln t + + c = ln tanq + +c
2 2 2 2
1 1 sin3 2 x
sin x = − ( − sin x + cos x ) + (sin x + cos x )
2 2
Illustration 22.94  Evaluate ∫ cos5 2 x dx.
sin x Solution:
I=∫ dx
sin x + cos x sin3 2 x
I=∫
dx
1 ( − sin x + cos x ) 1 (sin x + cos x ) cos5 2 x
=− ∫ dx + ∫ dx
2 (sin x + cos x ) 2 (sin x + cos x ) The given equation may be written as
1 1
= − ln (sin x + cos x ) + x + c sin3 2 x ⋅ sec2 2 x
2 2 I=∫ dx = ∫ tan3 2 x ⋅ sec2 2 x dx
cos3 2 x
∫ sin
m
(e)  x ⋅ cosn x dx , ( m, n ∈N )
Put tan 2 x = t . Then 2 sec2 2 x dx = dt .
If one out of m and n is odd, then substitute for term of even power.
If both are odd, then substitute either of the term. 1 3 t4 tan4 2 x

I= t dt = + c = +c
If both are even, then use trigonometric identities only. 8 8
The above substitution enables us to integrate any function of
the form R(sin x ,cos x ). However, in practice, it sometimes leads to sin nx
extremely complex rational function. In some cases, the integral
Illustration 22.95  Evaluate ∫ sin x
dx.

can be simplified by: Solution:


  (i) Substituting sin x = t , if the integral is of the form sin nx
In = ò dx
∫ R(sin x )cos x dx. sin x
  (ii) S ubstituting cos x = t , if the integral is of the form sin(n − 2) x
In− 2 = ∫ dx
∫ R(cos x )sin x dx . sin x
dt sin nx − sin(n − 2) x 2 cos(n − 1) x ⋅ sin x
(iii) Substituting tan x = t , that is, dx = , if the integral is In − In − 2 = ∫ dx = ∫ dx
1+ t 2 sin x sin x
dependent only on tan x .
2 sin(n − 1) x
(iv) Substituting cos x = t, if R( − sin x , cos x ) = − R(sin x , cos x ). In − In −2 =
(n − 1)
p
   (v) Substituting sin x = t , if R(sin x , − cos x ) = − R(sin x , cos x ).
∫x
m
8. Integrals of the form: (a + bx n ) dx
(a) If p ∈N (natural number). We expand the integral with the
(vi) Substituting tan x = t , if R( − sin x , − cos x ) = − R(sin x , cos x ).
help of binomial theorem and integrate.
Illustration 22.92  Evaluate ò sin3 x × cos2 x dx . Illustration 22.96  Evaluate ∫x
1/3
(2 + x1/2 )2 dx .

Solution: Solution:

I = ∫ sin x ⋅ (1− cos2 x )⋅ cos2 x dx I = ∫ x1/3 (2 + x1/ 2 )2 dx

Mathematical Problem Book for JEE.indb 990 07-06-2018 14:04:49


Chapter 22 | Indefinite Integration 991

Since P is a natural number. So


Your Turn 5
I = ∫ x1/ 3 ( 4 + x + 4 x1/ 2 ) dx
dx
= ∫ ( 4 x1/ 3 + x 4 / 3 + 4 x 5 / 6 ) dx
  1. ∫ 5 + 4 cos x =
2 1  1 1 
4 x 4 / 3 x 7 / 3 4 x11/ 6 (A) tan−1  tan x  + c (B) tan−1  tan x  + c
= + + +c 3 3  3 3 
4/3 7/3 11/ 6
2 1 x 1 1 x
(C) tan−1  tan  + c (D) tan−1  tan  + c
4 /3 3 24 11/ 6 3 3 2 3 3 2
I = 3x + x 7/3 + x +c
7 11 Ans. (C)
dx
  2. ∫ 2 2 2 2 =
(b) If p ∈I − (that is negative integer). Write x = t k where k is ( x + a )( x + b )
the LCM of the denominator of m and n.
1 1 −1  x  1 −1  x  
−2 3
(2 + x 2 3 )−1 dx.
(A)  tan  b  − a tan  a   + c
Illustration 22.97  Evaluate ∫x ( a2 − b 2 )  b

Solution: If we substitute x = t 3 (as we know p ∈ negative integer), 1 1 −1  x  1 −1  x  


(B) −  tan  b  − a tan  a   + c
then ( a2 − b 2 )  b

x = t 3 Þ dx = 3t 2dt 1  x 1  x
(C)  tan−1   − tan−1    + c
b  b a  a
3t 2 dt
I=∫ 2 2
dt = 3∫ = 3 tan−1 t + c 1  x 1  x
t (1+ t ) (1+ t 2 ) (D) −  tan−1   − tan−1    + c Ans. (A)
b  b a  a
I = 3 tan−1( x1 3 ) + c 1
 3.  ∫ dx =
m +1 1+ cos2 x
(c) If  is an integer and p∈ fraction, then put (a + bx n ) = t k ,
n 1 1 1 
(A) tan−1(tan x ) + c (B) tan−1  tan x  + c
where k is the denominator of the fraction p. 2 2  2 
 m +1  1  1 
(d) If  + p is an integer and p∈ fraction, then put (C) tan−1  tan x  + c (D)
None of these  Ans. (C)
 n  2  2 
( a + bx n ) = t k . x n, where k is the denominator of the frac- 1
tion P.
  4. ∫ 1+ 3 sin2 x dx =
−11
(1+ x 4 )−1/2 dx. 1 1
Illustration 22.98  Evaluate ∫x (A)
3
tan−1(3 tan2 x ) + c (B) tan−1(2 tan x ) + c
2
Solution: Here, (C) tan−1(tan x ) + c (D) None of these Ans. (B)
 m + 1   −11+ 1 1 1
 + p =  +  =−3
n   4 2  5.  ∫ dx equals
2x2 + x + 1
1 1
If we substitute 1+ = t12+, then
= t 2 , then 1  4 x + 1 1  4 x + 1
x4 (A) tan−1  + c (B) tan−1  +c
x4 7  7  2 7  7 
4 4
- 5 dx =-2tdt dx = 2tdt 1  4 x + 1
(C) tan−1  + c (D) None of these Ans. (D)
x x5 2  7 
1 1 dx
I=∫ dx = ∫ dx
x11(1+ x 4 )1/2 x11 ⋅ x 2 (1+ x −4 )1/2
  6. ∫ 7 + 5 cos x =
1 2t 1 1 1  1 x 1  1 x
I=− ∫ 8
4 x t
dt = − ∫ (t 2 − 1)2 dt = − ∫ (t 4 − 2t 2 + 1) dt
2 2
(A)
6
tan−1 
 6
tan  + c (B)
2 3
tan−1 
 3
tan  + c
2

1  t5 t3  1  x 1  x
I = −  − 2 + t + c (C) tan−1  tan  + c (D) tan−1  tan  + c
4  2 7  2
2 5 3 
Ans. (A)
1

( ) 
5/ 2
−4  7.  ∫ 2 dx is equal to
1  1+ x 2
( ) ( )
3/ 2 1/ 2 
=−  − 1+ x −4 + 1+ x −4 x + 4 x + 13
2 5 3  +c
  1  x + 2
  (A) ln|x 2 + 4 x + 13| + c (B) tan−1  tan  +c
3  3 

Mathematical Problem Book for JEE.indb 991 07-06-2018 14:05:40


992 Mathematics Problem Book for JEE

(2 x + 4 ) cot n −1 x
(C) ln 2 x + 4 + c (D) +c In = − − In − 2
( x 2 + 4 x + 13)2 n −1
 Ans. (B)
dx 1
  8. ∫ cos x − sin x is equal to (e) ∫ sec
n
x dx = [sec n − 2 x ⋅ tan x + (n − 2)∫ sec n − 2 x dx ]
n −1
If In = ∫ sec n x dx , then
1  x 3p 
(A) ln tan  + +c
2  2 8  (n − 1)In = sec n − 2 x ⋅ tan x + (n − 2)In − 2
1 x 1
(B) ln cot + c [ − cosec n − 2 x ⋅ cot x + (n − 2)∫ cosec n − 2 x dx ]
∫ cosec
n
(f)  x dx =
2 2 n −1
1  x 3p     If In = ∫ cosec n x dx , then
(C) ln tan  −  + c
2 2 8 
(n − 1)In = − cosec n − 2 x ⋅ cot x + (n − 2)In − 2
1 x p
(D) ln tan  −  + c Ans. (A)
2  2 8 sinq +1⋅ cos p −1 p − 1
(g) ∫ sin p x ⋅ cosq x dx = − sin p − 2 x ⋅ cosq x dx
p + q∫
+
1 p+q
  9. ∫ x 2 + 2 x + 2 dx =
 If I p , q = ∫ sin p x ⋅ cosq x dx , then
−1 −1
(A) sin ( x + 1) + c (B) sin h ( x + 1) + c
−1 −1
( p + q ) I( p , q ) = − sinq +1⋅ cos p −1 x + ( p − 1)I( p − 2 , q )
(C) tan h ( x + 1) + c (D) tan ( x + 1) + c Ans. (D)

dx x ( 2n − 3) dx
10. ∫
3 sin x + 2 cos x
dx =
(h) ∫ ( x 2 + k )n = k(2n − 2)( x 2 + k )n−1 + k(2n − 2) ∫ ( x 2 + k )n−1
3 cos x + 2 sin x
dx
12 5 If In = ∫ , then
(A) x − ln 3 cos x + 2 sin x + c ( x + k )n
2
13 13
12 5 x
(B) x + ln 3 cos x + 2 sin x + c k (2n − 2)In = + (2n − 3)In −1
13 13 ( x 2 + k )n −1
13 5 sinn x
(C) x + ln 3 cos x + 2 sin x + c (i) Reduction formulae for I(n, m) = ∫ dx is
12 13 cos m x
(D) None of these Ans. (A)
1 sinn −1 x n − 1 sinn −1 x
I(n , m) = ⋅ − ⋅ ⋅I
9. Reduction formulae: m − 1 cos m −1 x m − 1 cosm −1 x (n − 2 , m − 2)
cos x ⋅ sinn −1 x n − 1
∫ sin sinn − 2 x dx
n ∫
n
(a) x dx = − +
n Illustration 22.99  Evaluate ò tan5 x dx .
If In = ∫ sinn x dx , then Solution: Using
n −1
In = −
cos x ⋅ sin x
+
n −1
In − 2 tann−1 x
In = − In−2
n n n −1
sin x ⋅ cosn −1 x n − 1 Put n = 5, we get
(b) ∫ cosn x dx = cosn − 2 x dx
n ∫
+
n tan4 x
I5 = ò tan5 x dx Þ I5 = - I3
If In = ∫ cosn x dx , then 4

sin x ⋅ cosn −1 x n − 1 tan4 x tan2 x


In = + In − 2 = - + ln sec x + c
n n 4 2

tann −1 x
(c) ∫ tan
n
x dx = − ∫ tann − 2 x dx Illustration 22.100  Evaluate ∫ sin8 x dx .
n −1
If In = ∫ tann x dx , then Solution: Let cos x + i sin x = y . Then
n −1
tan x 1 1
In = − In − 2 and 2 cos nx = y n + n
2 cos x = y +
n −1 y y

cot n −1 x 1 1
(d) ∫ cot n x dx = − − ∫ cot n − 2 x dx 2i sin x = y − and 2i sin nx = y n − n
n −1 y y
If In = ∫ cot n x dx , then (Remember as the standard results)

Mathematical Problem Book for JEE.indb 992 07-06-2018 14:06:30


Chapter 22 | Indefinite Integration 993

Thus, 1
Let ln x = t. Then dx = dt. Therefore,
 1   1 
8
 1  x
(2i sin x )8 =  y −  =  y 8 + 8  − 8  y 6 + 6  dt dt 1 t +2− 5
 y   y   y  I= ∫ 2 =∫ = ln +c
(t + 4t − 1) ((t + 2)2 − 5) 2 5 t + 2 + 5
 1   1 
+28  y 4 + 4  − 56  y 2 + 2  + 70
 y   y  1 ln x + 2 − 5
⇒I= ln +c
2 5 ln x + 2 + 5
= 2 cos 8 x − 16 cos 6 x + 56 cos 4 x − 112 cos 2 x + 70
So Hence, the correct answer is option (D).

∫x
x
1   2. (1+ ln x ) dx is equal to
sin8 x = (cos 8 x − 8 cos 6 x + 28 cos 4 x − 56 cos 2 x + 35)
27 ( A) x x ln x + c (B) x x + c
(C) x ln x + c (D) None of these
and
1 Solution:
∫ sin x dx = ∫
8
(cos 8 x − 8 cos 6 x + 28 cos 4 x − 56 cos 2 x + 35) dx I  = ∫ x x (1+ ln x ) dx
27
Let xx  = t. Then
1  sin 8 x sin 6 x sin 4 x sin2 x 
∫ sin x dx = 27  8 − 8 6 + 28 4 − 56 2 + 35 x  + c
8
x ln x = ln t

1  sin 8 x 4  1 dt
 ⇒  1⋅ ln x + x ⋅  dx = ⇒ dx (1 + ln x)xx = dt
= 7 − sin 6 x + 7 sin 4 x − 28 sin2 x + 35 x  + c  x t
2  8 3 
Therefore,
Illustration 22.101  Evaluate ∫ cosec x dx . 4 I = ∫ dt ⇒ I = t + c = xx + c

Hence, the correct answer is option (B).


Solution:
sin x + cos x
3. ∫ dx is equal to
I = ∫ cosec 4 x dx = ∫ cosec2 x (1+ cot2 x ) dx 9 + 16 sin 2 x
cot3 x 1 5 + 4(sin x − cos x )
I = ∫ cosec2 x dx + ∫ cosec2 x cot2 x dx = − cot x − +c (A) ln +c
3 40 5 − 4(sin x − cos x )
1 5 − 4(sin x − cos x )
10. Some non-integral functions: (B) ln +c
40 5 + 4(sin x − cos x )
dx
∫ ln x (b) ∫ e x dx
2
(a) 1 5 + 4(sin x + cos x )
(C) ln +c
40 5 − 4(sin x + cos x )
x2 3
(c) ∫ 1+ x 5 dx (d) ∫ 1+ x 2 dx
(D)
1 5 − 4(sin x + cos x )
ln +c
40 5 + 4(sin x + cos x )
2 2
(e) ∫ 1+ x 3 dx (f) ∫ 1− k 2 sin2 x dx
Solution:
sin x sin x + cos x
−x I= ∫ dx
∫e (h) ∫
2
(g) dx dx 9 + 16 sin 2 x
x
Let t = sin x – cos x. Then t2 = 1 – sin 2x ⇒ sin 2x = (1 – t2)
∫ ∫ sin x
2
(i) sin x dx (j) dx
Also,
∫ cos x ∫ x tan x dx
2
(k) dx (l) dt = cos x+ sin x
Therefore,
Additional Solved Examples =∫
dt
=∫
dt
9 + 16 − 16t 2 25 − 16t 2
dx
  1. ∫ x (ln2 x + 4 ln x − 1) is equal to 5 
+t
1 dt 1 1  
= ∫ = ⋅ ln  4  + c
1 ln x + 2 − 5 1 ln x + 2 − 5 16  5  2 16 5 5
(A) ln + c (B) ln +c 2 2⋅  −t
2 ln x + 2 + 5   − t 4 4 
5 ln x + 2 + 5 4

1 ln x + 2 − 5 1 ln x + 2 − 5 1 5 + 4t 1 5 + 4(sin x − cos x )
(C) ln + c (D) ln +c = ln +c = ln +c
2 5 ln x − 2 + 5 2 5 ln x + 2 + 5 40 5 − 4t 40 5 − 4(sin x − cos x )

Solution: Hence, the correct answer is option (A).


dx 4. For what value of a and b, the equation
I= ∫
x (ln2 x + 4 ln x − 1) 1
∫(sin2x – cos 2x) dx = sin(2 x − a) + b holds good.
2

Mathematical Problem Book for JEE.indb 993 07-06-2018 14:07:18


994 Mathematics Problem Book for JEE

5p Solution:
(A) a = − , b is any arbitrary constant  x
4 I = ∫ cos  ln  dx
 a
5p
(B) a = , b is any arbitrary constant x
4 Let ln = t. Then
a
p x = a⋅et ⇒ dx = aet dt
(C) a = − , b is any arbitrary constant
4 aet
p I = a∫ et cos t dt = (cos t + sin t ) + c
(D) a = , b is any arbitrary constant 2
4
x  x  x
Solution: =  cos  ln  + sin  ln   + c
 sin2 x cos 2 x  2  a  a
∫(sin2x – cos 2x) dx = ∫ 2  −  dx =
 2 2  Hence, the correct answer is option (B).
 p ln( x + x 2 + 1)
− ∫ 2 cos  2 x +  dx 7. ∫ dx  is equal to
 4 x2 + 1
2  p 1  5p 
sin  2 x +  + c =
=− sin  2 x +  +c 1 2
2  4 2  4  (A) ln ( x − x 2 + 1) + c (B) ln2 ( x − x 2 + 1) + c
2
5p
a=− , b is any arbitrary constant. 1
4 (C) ln2 ( x + x 2 + 1) + c (D) ln2 ( x + x 2 + 1) + c
2
Hence, the correct answer is option (A).
5. For what value of a and b, the equation Solution:
dx x  ln( x + x 2 + 1)
∫ 1+ sin x = tan  2 + a + b holds good. I= ∫ dx
x2 + 1
5p
(A) a = − , b is any arbitrary constant Let ln (x + x 2 + 1) = t. Then
4
5p 1  2x  dx
(B) a = , b is any arbitrary constant  1+  dx = dt ⇒ = dt
4 ( x + x 2 + 1)  2 x 2 + 1 x2 + 1
p
(C) a = − , b is any arbitrary constant t2
4 ⇒ I = ∫ tdt = +c
p 2
(D) a = , b is any arbitrary constant 1 2
4 I= ln ( x + x 2 + 1) + c
Solution: 
2 

dx dx 1 p x Hence, the correct answer is option (C).
∫ 1+ sin x = ∫ p  2∫
= sec2  −  dx
 4 2 x cos x
1+ cos  − x 
2  8. ∫ ( x sin x + cos x )2 dx is equal to
p x 1 1
tan  −  (A) + k (B) − +k
1  4 2 p x x p ( x sin x + cos x )2 ( x sin x + cos x )
= + c = − tan  −  + c = tan  −  + c
2 1  4 2  2 4 1 1

2 (C) + k (D) +k
( x sin x + cos x )3 ( x sin x + cos x )4
p
  a = − , b any arbitrary constant. Solution:
4 x cos x
Hence, the correct answer is option (C). I= ∫ dx
( x sin x + cos x )2
 x
6. ∫ cos  ln a  dx is equal to Let
1
= t. Then
x sin x + cos x
  x  x
(A) x  cos  ln  − sin  ln   + c ( x sin x + cos x ) ⋅ 0 − 1( x cos x + sin x − sin x ) dt
  a  a =
( x sin x + cos x )2 dx
x  x  x
(B)  cos  ln  + sin  ln   + c − x cos x dt
2   a   a ⇒ =
( x sin x + cos x )2 dx
x  x  x
(C) cos  ln  − sin  ln   + c
2   a  a Therefore,
1
I = – ∫ dt = – +c
  x  x x sin x + cos x
( D) x  cos  ln  + sin  ln   + c
  a  a Hence, the correct answer is option (B).

Mathematical Problem Book for JEE.indb 994 07-06-2018 14:07:58


Chapter 22 | Indefinite Integration 995

Solution:
9. ∫( tan x + cot x ) dx is equal to
1 5 5
(A) 2sin–1 (sin x + cos x) + c (B) sin–1 (sin x – cos x) + c x5 +1 +1
5 + x10 x10 x10
2 I= ò x16
dx = ò
x16
dx = ò
x11
dx
1
(C) 2sin–1 (sin x – cos x) + c (D) sin–1 (sin x + cos x) + c 5
2 Let 1 + = t. Then
x10
Solution:  −10  1 1
5  11  dx = dt ⇒ 11 dx = − dt
  I = ∫ ( tan x + cot x ) dx  x  x 50
3/ 2
sin x + cos x 1 1 2 1 5 
  = 2 ∫
2 sin x ⋅ cos x
dx ⇒I=−
50 ∫ tdt = – × t3/2 = −  1+ 10 
50 3 75  x 
+c

(sin x + cos x ) Hence, the correct answer is option (A).


= 2∫ dx
sin 2 x
∫ cosec
6
12. xdx is equal to
Let t = sin x – cos x. Then
t2 = 1 − sin 2x ⇒ dt = (cos x + sin x) dx cot5 x 2 cot5 x
(A) − cot x – − + k
Therefore, 5 3
dt cot x 2 cot5 x
I = 2∫ = 2sin–1 (t) + c (B) − + + 2cot–3 x + k
1− t 2 3 5
tan3 x tan x
I = 2sin–1(sin x − cos x) + c. (C) − + 2 tan3 x + k
3 5
Hence, the correct answer is option (C). (D) None of these
1
10. ∫ x 2 ( x 4 + 1)3/ 4 dx is equal to Solution:

I = ∫ cosec 6 xdx = ∫ cosec4 x ⋅ cosec2 x dx


1/ 4 1/ 4
 1  1 = ∫ (1+ cot2 x )2 ⋅ cosec2 x dx = ∫ (1+ cot 4 x + 2 cot2 x ) ⋅ cosec2 x dx
(A)  1+ 4  + c (B) −  1+ 4  + c
 x   x 
Let cotx = t. Then
1/ 4 1/ 4
 1  1 - cosec2x dx = dt
(C) −  1− 4  + c (D)  1− 4  +c
 x   x 
t5 2 3
⇒ I = − ∫ (1+ t 4 + 2t 2 ) dt = −t − − t +c
Solution: s 3

1 dx cot5 x 2 3
I= ∫ 2 4 3/ 4
dx = ∫ 3/ 4 − cot x + c
⇒ I = − cot x −
x ( x + 1)  1 5 3
x 5  1+ 4 
 x  Hence, the correct answer is option (A).
( x 2 + 1){ln( x 2 + 1) − 2 ln x }
Let 1 + x – 4 = t. Then 13. ∫ dx is equal to
x4
−4 1 1 ( x 2 + 1) x 2 + 1   x 2 + 1 
5
dx = dt ⇒ dx = - dt
5 (A) 2 − 3 ln  2   + c
x x 4 x 3
  x  
1 dt 1 1
  ⇒ I =  ∫ 3 / 4 = − ∫ t −3 / 4 dt = − × 4t 1/ 4 + c ( x 2 + 1) x 2 + 1   x 2 + 1 
4 t 4 4 (B)  2 + 3 ln  2  + c
1/ 4
9 x3   x  
 1
  ⇒ I =   1+ 4  + c
 x  ( x 2 + 1) x 2 + 1   x 2 + 1 
(C)  2 + 3 ln  2  + c
Hence, the correct answer is option (B). x3   x  

5 + x10 ( x 2 + 1) x 2 + 1   x 2 + 1 
11. ∫ x 16
dx is equal to (D)
9x 3
2 − 3 ln  2   + c
  x  
3/ 2 3/ 2
1 5  1 5  Solution:
(A) −  1+  + c (B) −  1−  +c
75  x10  75  x10 
 x 2 + 1   x 2 + 1 
3/ 2 3/ 2   ln  2  
1 5  1 5   x 2 
(C)  1−  + c (D)  1+  +c ( x 2 + 1){ln( x 2 + 1) − 2ln x }   x  
75  x10  75  x10  I= ∫ dx = ∫ dx
x4 x3

Mathematical Problem Book for JEE.indb 995 07-06-2018 14:08:39


996 Mathematics Problem Book for JEE

x2 + 1 x
Let = t. Then Let t = tan . Then
x 2 2
2 1 x
– 3
dx = dt dt = sec2 dx
x 2 2
1 2dt
⇒ dx =
2∫
⇒I=− t ln t dt x
1+ tan2
2
1 2t 3 / 2 2 1 3 / 2 
= – (ln t ) ⋅ − ∫ ⋅ t dt  2dt
2  3 3 t  dx =
1+ t 2
1  1/2 2dt
=  ∫ t dt − t 3/2 (ln t )
  3 1 + t2 dt
1 ⇒I= ∫ = 2∫ 2
= t3/2 [2 – 3 ln t] + c  1− t 2  t +9
9 5+ 4 
 1+ t 2 
1 ( x 2 + 1) x 2 + 1   x 2 + 1 
= 3
2 − 3 ln  2   + c 2 1 x
9 x   x   = tan–1  tan  + c
3  3 2
Hence, the correct answer is option (D).
Hence, the correct answer is option (A).
1+ x 2 dx
14. ∫ x4
dx is equal to 16. ∫ ( 2 x + 3) is equal to
4x + 5
3/ 2 3/ 2
1 1 1 1
(A)  1+ 2  + c (B)  1− 2  +c (A) tan–1 4 x − 5 + c (B) tan–1 4 x + 5 + c
3 x 3 x
3/ 2 3/ 2 (C) tan–1 5 x + 4 + c (D) tan–1 5 x − 4 + c
1 1 1 1
(C) −  1+ 2  + c (D) -  1− 2  +c
3  x  3  x  Solution:
dx
Solution: I= ∫
1 ( 2 x + 3 ) 4x + 5
1+
1+ x 2 x 2 dx
I= ∫ dx = ∫ Put 4x + 5 = t. Then
x4 x 3

1 t −5 dt
Let 1 + = t . Then x=
⇒ dx =
2 4 4
x
1 dt 1 dt
2 ⇒I= ∫ = ∫
− dx = dt 4  2t − 10  2 (t + 1) t
x3  + 3 t
4 
1
2∫
  ⇒I= − t dt 1
Let t = u. Then dt = du.
2 t
1 2 Therefore,
= − . t3/2 + c du
2 3 I= ∫ 2 = tan−1 t + c
3/ 2 u +1
1 1
= −  1+ 2  +c ⇒ I = tan–1 4 x + 5 + c.
3  x 
Hence, the correct answer is option (B).
Hence, the correct answer is option (C).
∫e
ax
17. cos b xd x is equal to
dx
15. ∫ is equal to
5 + 4 cos x (A) Real part of ∫ e( a + bi ) x dx
2 1 x 2 1 x (B) Imaginary part of ∫ e( a + bi ) x dx
(A) tan–1  tan  + c (B) − tan–1  tan  + c
3 3 2 3 3 2 (C) Neither real nor imaginary part of ∫ e( a + bi ) x dx
1 1 x (D) None of these
(C) tan–1  tan  + c (D) None of these
3 3 2
Solution:
Solution:
I = ∫ e ax cos b xd x
dx dx
I= ò 5 + 4 cos x ò
=
é 2xù
= real part of ∫ e ax e ibx dx
ê 1 - tan ú
ê 2ú
5+ 4 ê = real part of ∫ e ax + ibx dx
2 xú
ê 1 + tan ú
êë 2 úû Hence, the correct answer is option (A).

Mathematical Problem Book for JEE.indb 996 07-06-2018 14:09:27


Chapter 22 | Indefinite Integration 997

2 x 2 + ln x 1 2
18. ∫e dx is equal to Let = t. Then – dx = dt. Therefore,
2 2
x2 x3
e2 x e2 x
(A) + c (B) +c 1 dt 1 1 1− x 2
4 2 I= - ò = ln |t – 1| + c = – ln
2 (t -1) 2 2 x2
+c
2 2
e2 x x2 xe 2 x
(C) + + c (D) +c
4 2 4 1 x2 1
= ln +c⇒A=
Solution: 2 1− x 2 2
2
+ln x 2
I = ∫ e2 x dx ⇒ xe2 x dx Hence, the correct answer is option (B).
Let x2 = t. Then −1  1+ x + x 2 
∫e
tan x
2x dx = dt 22.   dx is equal to
 1+ x 2 
2
1 2t e 2t e2 x −1 −1

2∫
e   ⇒ I =
dt = + c = +c (A) xe tan x + c (B) x 2e tan x + c
4 4
1 1 −1

Hence, the correct answer is option (A). (C) +c (D) 2 e tan x + c


tan−1 x x
xe
2 x +1 − 5 x −1 Solution:
19. ∫ 10 x
dx is equal to
 1+ x + x 2 
I = ∫ e tan x
−1

2 x 1 x −2 − x 1 −x  1+ x 2  dx
(A) 5 + 2 +c (B) 5 + 2 +c
ln 5 5 ln 2 ln5 5 ln2
1 -x 1 -x Let p = tan−1x. Then
( C) 5 - 2 +c (D) None of these
2 ln 5 5 ln 2 x = tanp ⇒ dx = sec2p dp
Solution: ⇒ I = ∫ e p (sec2 p + tan p ) dp
2 x +1 − 5 x −1   1 x
1  1
x −1
= ep tanp = x e tan x
+c
I= ∫ 10 x
dx = ∫ 2  5  −    dx
5  2  
 Hence, the correct answer is option (A).
x x 2
 1 1  1  1 
2 
 5  
5  2
23. ∫  x − x  dx is equal to
= − +c
1 1
ln ln x2 x2
5 2 (A) + log| x | + c (B) + log| x | + 2 x + c
−2 − x 1 −x 2 2
= 5 + 2 +c
ln 5 5 ln 2 1 1 
3
x2
Hence, the correct answer is option (B).
(C)  x+  + c (D) + log x − 2 x + c
3 x 2
20. If In = ò (ln x )n dx , then In + nIn –1 is equal to Solution:
( A) x (ln x)n –1 (B) x (ln x)n  1 
2
1  x2

(C) nx (ln x)n (D) None of these ∫  x−
x
 dx = ∫  x + − 2 dx =
x 2
+ log x − 2 x + c
Solution:
Hence, the correct answer is option (D).
In = ∫ (ln x )n dx
cos 2x
In = x(ln x)n – n ∫ (ln x )n−1dx 24. ∫ cos x + sin x dx is equal to
In = x(ln x)n – n In – 1 ( A) sin x − cos x + c (B) −sin x + cos x + c
In + nIn –1 = x(ln x)n (C) sin x + cos x + c (D) None of these

Hence, the correct answer is option (B). Solution:

dx x2 (cos x + sin x )(cos x − sin x )


cos 2x
21. If ∫ 3
= A ln 2
+ c , then A is equal to ∫ cos x + sin x dx = ∫
cos x + sin x
dx
x−x 1− x
 = sin x + cos x + c
( A) 2 (B) 1/2 Hence, the correct answer is option (C).
(C) 2/3 (D) 1/4
cot x
Solution: 25. If ∫ dx = A cot x + B , then A is equal to
dx dx sin x cos x
I= ∫ =∫
x − x3 1 
3 ( A) 1 (B) 2
x  2 − 1 (C) −1 (D) −2
x 

Mathematical Problem Book for JEE.indb 997 07-06-2018 14:10:16


998 Mathematics Problem Book for JEE

Solution: 1 sin2 x
= e (3 − sin2 x ) + c      (Option A)
cot x cot x 2
∫ sin x cos x dx = ∫ cot x ⋅cosec2x dx 2  1 
= e sin x  1+ cos2 x  + c     (Option B)
 2 
cosec2 x
= ∫
cot x
dx = −2 cot x + B = A cot x + B Hence, the correct answers are options (A) and (B).
⇒ A = −2
x3 − 3x + 7
Hence, the correct answer is option (D). 29. ∫ x2 + 4
dx is equal to
ln( x/ e )
26. The value of ∫ (ln x )2
dx is x2 7
(A) + ln( x 2 + 4 ) + c
x +1 x −1 2 2
(A) + c (B) +c x2 7 x 7
(Inx )2 (In x )2 (B) + tan−1 − ln( x 2 + 4 ) + c
2 2 2 2
x Inx
(C) + c (D) +c x2 7 x2 7
Inx x (C) − + tan−1 + ln x 2 + 4 + c
2 2 2 2
( )
Solution:
x 7 x
ln( x/ e ) ln( x ) − 1 ( D) + tan−1 + c
I=
(ln x )
∫ 2
dx = ∫ (ln x )2
dx 2 2 2
Solution:
Put ln x = t. Then
x = et ⇒ dx = etdt x3 − 3x + 7 7( x − 1)
2
=x–
x +4 x2 + 4
 t − 1 1 1 et x
I = ∫ et  2  dt = ∫ et  − 2  dt = + c = +c x  3x  7
3
( x  1)
 t  t t  t ln x  x 2  4 dx xdx  7x 2  4 dx
Hence, the correct answer is option (C).
x3 − 3x + 7 x2 ( x − 1)
9 x
(10 x + 10 loge 10 ) ∫ x +42
dx =
2
− 7∫ 2
x +4
dx
27. I = ∫ ( x10 + 10 x )
dx is equal to
2
x 7 x 7
= + tan−1 − ln( x 2 + 4 ) + c
( A) 10 x + x10 + c (B) 10  x − x10 + c
2 2 2 2
(C) 10 x + x10 + c (D) ln (10 x + x10) + c
Hence, the correct answer is option (B).
Solution: If p = x 10 + 10 x , then
e x −1 + x e −1
(10 x + 10 loge 10)dx = dp
9 x 30. ∫ ex + xe
dx is equal to

 I dp  p  c 1 1
(A) ln (x e + e x) + c (B) ln (x + e) + c
 I  ln( x  10 )  c
10 x e e
1
Hence, the correct answer is option (D). ( C) ln (x–e + e–x ) + c (D) None of these
e
2
28. The value of the integral ò e sin x (cos x + cos3 x ) sin x dx is Solution:
1 sin2 x e x −1 + x e −1
(A) e (3 -sin2 x ) + c I= ∫ dx
2 ex + xe
2  1  Let e x + x e = t. Then
(B) e sin x  1+ cos2 x  + c
 2  (e x + e⋅x e – 1) dx = dt ⇒ e(e x – 1 + x e – 1) dx = dt
sin2 x 2 2
(C) e (3 cos x + 2 sin x ) + c Therefore,
sin2 x 2 2 1 dt 1
e∫ t
(D) e (2 cos x + 3 sin x ) + c I= = ln(e x + x e) + c
e
Solution:
Hence, the correct answer is option (A).
I =e sin x (cos x  cos3 x ) sin xdx =e sin x (2  sin2 x )cos x sin xdx
2 2

Put t = sin2 x. Then Previous Years' Solved JEE Main/AIEEE


dt = 2sinx cosx dx Questions
The integral reduces to
1 t 3 t tet dx
⇒ I =
2∫
e ( 2 − t ) dt =
2
e −
2
+c 1. ∫ cos x + 3 sin x
equals to

Mathematical Problem Book for JEE.indb 998 07-06-2018 14:10:59


Chapter 22 | Indefinite Integration 999

1 x p  1 x p  4. If ò f ( x ) dx = y ( x ), then ∫ x 5f ( x 3 ) dx is equal to
(A) log tan  +  + c (B) log tan  −  + c
2  2 12  2  2 12 
1 3
x p  x p  (A) x y ( x 3 ) - 3ò x 3y ( x 3 ) dx + C
(C) log tan  +  + c (D) log tan  −  + c 3
 2 12   2 12 
1
 [AIEEE 2007] (B) x 3y ( x 3 ) - ò x 2y ( x 3 ) dx + C
3
Solution:
1
dx 1 dx (C)  x 3y ( x 3 ) − ∫ x 3y ( x 3 ) dx  + C
3
∫ cos x + 3 sin x
=
2∫ 1 3
cos x + sin x 1
(D)  x 3y ( x 3 ) − ∫ x 2y ( x 3 ) dx  + C  [JEE MAIN 2013]
4 4 3
1 dx
= ∫ Solution: We have
2  p
cos  x − 
 3 ∫ f ( x ) dx = y ( x )
1  p
2∫
= sec  x −  dx Let x3 = t and x2 dx = dt/3. Then
 3
1 1
1 x p p
= log tan  − +  + c x 5f ( x 3 ) dx  3tf (t )dt  3 tf (t )dt 1f (t )dt  dt 
2  2 6 4
1
1 x p   x 3y ( x 3 ) x 2y ( x 3 ) dx  C
= log tan  +  + c 3
2  2 12 
Hence, the correct answer is option (B).
Hence, the correct answer is option (A).
1
sin x dx  1 x+ x
2. The value of 2 is 5. The integral ∫  1+ x − e dx is equal to
 p  x
sin  x  
4 1 1
 x+ x+
(A) ( x + 1)e x + c (B) − xe x +c
 p  p
(A) x + log cos  x −  + c (B) x − log sin  x −  + c x+
1
x+
1
 4  4 (C) ( x − 1)e x + c (D) xe x +c
 p  p
(C) x + log sin  x −  + c (D) x − log cos  x −  + c  [JEE MAIN 2014 (OFFLINE)]
 4  4
 [AIEEE 2008] Solution:
Solution:  x + 1 1  x + 
1 1 1
 1 x+  x+
 p p ∫  1+ x −  e x dx = ∫ e x dx + x  1− 2  e x  dx = xe x + c
sin 
 x    dx  x  x 
sin x dx 4 4  
2  2 
 p  p
Since,xf ( x )  f ( x )dx  xf ( x )  c 
sin   
 x    x  
sin 
 4  4
  p p   p Hence, the correct answer is option (D).
p
 2 cos 4  cot   x   sin  dx dx cot  x   dx
  4  4   4 sin8 x − cos8 x
 p
6. ∫ (1− 2 sin2 x cos2 x ) dx is equal to
 x  log sin  x    c
 4
1 1
Hence, the correct answer is option (C). (A) sin 2 x + c (B) − sin 2 x + c
2 2
5 tan x 1
3. If the integral ∫ tan x − 2 dx = x + a ln sin x − 2 cos x + k , then a (C) − sin x + c (D) − sin2 x + c
2
is equal to
(A) −1 (B) −2  [JEE MAIN 2014 (ONLINE SET-1)]
(C) 1 (D) 2 [AIEEE 2012]
Solution:
Solution:
sin8 x  cos8 x (sin4 x  cos4 x )(sin4 x  cos 4 x )dx
5 tan x 5sinx 1 2 sin2 cos2 x dx 
tan x  2 dx sin x  2 cos x dx (1 2 sin2 x cos2 x )
2(cos x  2 sin x )  (sin x  2 cos x )  {(sin2 x + cos2 x )2 -2 sin2 x cos2 x }{sin2 x -cos2 x }dx
  dx =ò
 sin x  2 cos x  (1-2 sin2 x cos2 x))

 cos x  2 sin x 
 2  dx dx  2 log sin x  2 cos x  x  k ( 1 2 sin2 x cos2 x )(  cos 2 x )dx sin2 x
 sin x  2 cos x  ⇒ ⇒− +c
2
( 1 2 sin x cos x ) 2 2
Therefore, a = 2.
Hence, the correct answer is option (D). Hence, the correct answer is option (B).

Mathematical Problem Book for JEE.indb 999 07-06-2018 14:11:37


1000 Mathematics Problem Book for JEE

 1− x 2  x 5m −1 + 2 x 4 m −1
7. The integral ∫ x cos −1   dx ( x > 0 ) is equal to 9. If m is a non-zero number and ∫ ( x 2m + x m + 1)3 dx = f ( x ) + c ,
 1+ x 2  then f(x) is
(A) −x + (1 + x2) tan−1 x + c (B) x − (1 + x2) cot−1 x + c
(C) −x + (1 + x2) cot−1 x + c (D) x − (1 + x2) tan−1 x + c x 5m x 4m
(A) 2m m 2
(B) 2m
2m( x + x + 1) 2m( x + x m + 1)2
 [JEE MAIN 2014 (ONLINE SET-2)]
2m( x 5m + x 4 m ) ( x 5m − x 4 m )
Solution: (C) (D)
( x 2 m + x m + 1)2 2m( x 2 m + x m + 1)2
 x2 
1 1  dx , x  0
I x cos  2
 [JEE MAIN 2014 (ONLINE SET-4)]
 1  x 
Solution:
Put x = tan q. Then dx = sec2q dq.
x 5m −1 + 2 x 4 m −1 x 5m −1 + 2 x 4 m −1
I=∫ dx ⇒ ∫ dx
−1
I = ∫ tanq cos (cos 2q )sec q dq = 2∫ q tanq sec q dq
2 2
(x 2m m
+ x + 1) 3
{x 2m
(1+ x − m + x −2 m )}3

Again put tan q = t. Then sec2q dq = dt. x 5m−1 + 2 x 4 m−1 x − m−1 + 2 x −2m−1
⇒∫ dx ⇒ ∫ (1+ x −m + x −2m )3 dx
Therefore x 6m (1+ x − m + x −2m )3

I  2(tan1 t )tdt Now put 1+ x − m + x −2 m = t . Then

 t2 1 t2  ( − mx
− m −1
− 2mx −2 m −1) dx = dt ⇒ − m( x − m −1 + 2 x −2 m −1) dx = dt
= 2 (tan−1 t ) − ∫ dt 
 2 1+ t 2 2  Therefore,
 (t 2  1) 1dt 
 t 2 tan1 t   dt  2  1 dt 1 t −3+1 1 t −2
m ∫ t3
 1 t 2 I=− = − + c = − +c
 1 t  m −3 + 1 m −2
1 1 1
= t2 tan−1t − [t − tan −1 t] = t2 tan−1t − t + tan −1 t = (t2 + 1) tan−1 t − = +c = +c
2m t 2 2m(1+ x − m + x −2 m )2
t = {tan 2 q  + 1} tan−1 (tanq ) − tan q = q {tan 2 q + 1} − tan q
1 x 4m
Therefore, I = (1 + x2) tan −1 x − x + c. = 2
+c = 2m
+c
 1 1  2m( x + x m + 1)2
2 m  1+ m + 2 m 
Hence, the correct answer is option (A).  x x 
sin2 x cos2 x Hence, the correct answer is option (B).
8. The integral ∫ dx equal to
(sin3 x + cos3 x )2 dx
1 1 10. The integral ∫ equals
(A) +c (B) − +c x 2 ( x 4 + 1)3 / 4
(1+ cot3 x ) 3(1+ tan3 x ) 1 1
3 3 (A) ( x 4 + 1) 4 + c (B) −( x 4 + 1) 4 + c
sin x cos x
(C) 3
+c (D) − +c 1 1
(1+ cos x ) 3(1+ sin3 x )  x 4 + 1 4  x 4 + 1 4
 [JEE MAIN 2014 (ONLINE SET-3)] (C) −  4  + c (D)  4  + c
 x   x 
Solution:
 [JEE MAIN 2015 (OFFLINE)]
sin2 x cos2 x sin2 x cos2 x Solution:
I=∫ 3 3 2
dx = ∫ dx
(sin x + cos x ) cos6 x (1+ tan3 x )2 dx dx
I=∫ =∫
sin2 x tan2 x sec2 x x 2 ( x 4 + 1)3 / 4  1
3/ 4
=∫
      dx = ∫ dx x 5  1+ 4 
cos 4 x (1+ tan3 x )2 (1+ tan3 x )2  x 
Put x−4 = t. Then
Now put tan x = t. Then sec2 x dx = dt.
−4
dx = dt
Therefore, x5
t 2dt − dt 1 (1 t )1/ 4
I = −∫ ⇒ I=∫   c = − (1 + x-4)1/4 + c
(1+ t )3 2 4(1+ t )3 / 4 4 1/ 4
Hence, the correct answer is option (C).
Again substituting 1 + t3 = m, we get
3t2 dt = dm dx
11. The integral ∫ 3/4
is equal to
1 dm 1 1 1 1 1 ( x + 1) ( x − 2)5/4
  2      c
3 m 3 m 3 1 t 3 3(1 tan3 x ) 1/4 1/4
 x + 1  x − 2
(A) 4  +C (B) 4  +C
Hence, the correct answer is option (B).  x − 2   x + 1 

Mathematical Problem Book for JEE.indb 1000 07-06-2018 14:12:17


Chapter 22 | Indefinite Integration 1001

1/4 1/4
4  x + 1 4  x − 2  2 5
(C) −   +C (D) −   +C  3 + 6 
3  x − 2 3  x + 1 ⇒∫ x x dx
3
 [JEE MAIN 2015 (ONLINE SET-1)]  1 1
 1 + + 
Solution: x2 x5 
dx dx Put 1 + x−2 + x−5 = t. Then
I  =∫
3/ 4
( x  1) ( x  2)5/ 4
( x + 1) ( x − 2)2 ( x − 2)5 / 4 − 2
3/ 4
 2 5 
 3  6  dx  dt
3 / 4  x x 
dx ( x  1) 1
=∫   dx
( x + 1)3 / 4 ( x − 2)2 ( x − 2)−3 / 4 ( x  2)3 / 4 ( x  2)2 1 1
⇒ − 3 dt  2 + C
−3 / 4 t 2t
1  x + 1  −3 
−3 ∫  x − 2 
= ⋅ dx 1 x10
 ( x − 2)2  ⇒ 2
+C = +C
 1 1 2( x + x 3 + 1) 5
x 1 3 2 1+ 2 + 5 
Put t   dt  dx  x x 
x 2 ( x  2)2
Hence, the correct answer is option (C).
1 4 1/ 4
 I = (t )3 / 4 dt  (t )  C
3 3 dx
1/ 4
14. If ∫ cos3 x
2 sin 2 x
= (tan x ) A + C (tan x )B + k , where k is a
4 é x + 1ù
Þ- ê ú +C constant of integration, then A + B + C equals
3 êë x -2 úû
16 27
Hence, the correct answer is option (C). (A) (B)
5 10
log(t + 1+ t 2 )1 7 21
12. If ∫ dt = ( g(t ))2 + C , where C is a constant,
2
2
(C)
10
(D)
15
( 1+ t )
then g(2) is equal to  [JEE MAIN 2016 (ONLINE SET-1)]
(A) 2 log(2 + 5 ) (B) log(2 + 5 ) Solution: We have
1 1 dx
(C) log(2 + 5 ) log(2 + 5 )
(D) I=∫
5 2 cos3 x 4 tan x cos2 x
 [JEE MAIN 2015 (ONLINE SET-2)]
1 dx 1 sec 4 x dx
Solution: I= ∫ 4
= ∫
2 cos x tan x 2
( ) dt
tan x
log t + 1+ t 2 Put tan x = t. Then sec2x dx = dt. Therefore,
I=∫
1+ t 2 1 (1+ t 2 )dt 1 −1/ 2 3/ 2
2 ∫ t1/ 2
I= = ∫ (t + t )dt
( )

d 2  1 2
Since,  log t + 1+ t  = ,  we get
dt 1+ t 2 1 2 
=  2t 1/ 2 + t 5 / 2  + k
1 2 2 5 
I  log(t  1 t 2 )   C
2  1
= (tan x )1/ 2 + (tan x )5 / 2 + k
⇒ g(t ) = log(t + 1+ t 2 ) ⇒ g(2) = log(2 + 5 ) 5
Comparing this with the given equation, we get A = 1/2, B = 5/2
Hence, the correct answer is option (B). and C = 1/5. Therefore,

2 x12 + 5 x 9 1 5 1 1 16
13. The integral ò ( x 5 + x 3 + 1)3
dx is equal to A+B +C = + + = 3+ =
2 2 5 5 5
− x10 − x5 Hence, the correct answer is option (A).
(A) +C (B) +C dx
2( x 5 + x 3 + 1)2 ( x 5 + x 3 + 1)2 15. The integral ∫ is equal to (where C is a
x 10
x5 (1+ x ) x − x 2
(C) + C (D) +C constant of integration)
2( x 5 + x 3 + 1)2 2( x 5 + x 3 + 1)2
 [JEE MAIN 2016 (OFFLINE)] 1+ x 1− x
(A) −2 + C (B) − +C
Solution: We have 1− x 1+ x

2 x12 + 5 x 9 2 x12 + 5 x 9 1− x 1+ x
∫ ( x 5 + x 3 + 1)3 dx ⇒ ∫ 1 1
3
dx (C) −2
1+ x
+ C (D) 2
1− x
+C
15 
x  1+ 2 + 5 
 x x   [JEE MAIN 2016 (ONLINE SET-2)]

Mathematical Problem Book for JEE.indb 1001 07-06-2018 14:12:59


1002 Mathematics Problem Book for JEE

Solution: We have 1  1 1
Put t + = m ⇒ 1Put 2dt = d m 
− 2(tt + 1
dx dx t  t 1=)dt m ⇒ 1− 2  dt = d m
I=∫ =∫ t J  I 
  t 
(1+ x ) x − x 2 (1− x ) dm t 4  t 2 1
(1+ x )2 x ⇒ I − I = 2 d m
m − 1 ⇒ I −1I=mdt
 1
(1+ x ) 2
2
−1
  t 
1− x 1 m −1 1 m 2− 1
Put y = . Then = log = log 1
1+ x 2 m +1 2 t  tm +11
 
1
dy   dx 1  e x  e  x  1
x (1 x )2  log  x  x   c
2  e  e  1
Therefore,
1  e 2 x  e x  1
dy y ( −1/ 2 )+1  log  2 x x   c
I = −∫ =− + C = −2 y + C 2
y ( −1/ 2) + 1  e  e  1
Hence, the correct answer is option (C).
1− x
I = −2 +C sec2 x
1+ x 2. The integral ∫ dx equals (for some arbitrary
(sec x + tan x )9 / 2
Hence, the correct answer is option (C).
constant K)
  Previous Years' Solved JEE Advanced/ (A) −
1 1 1 2
 − (sec x + tan x )  + K
(sec x + tan x )11/ 2 11 7 
IIT-JEE Questions
1 1 1 2
(B)  − (sec x + tan x )  + K
e x
e −x (sec x + tan x )11/ 2 11 7 
1. Let I = ∫ dx , J = ∫ −4 x dx . Then, for an
e 4 x + e2 x + 1 e + e −2 x + 1 1 1 1 2
(C) − 11/ 2  + (sec x + tan x )  + K
arbitrary constant C, the value of J − I equals (sec x + tan x )  11 7 
1  e 4 x − e2 x + 1 1 1 1 2
(A) log  4 x  +C (D)  + (sec x + tan x )  + K
2  e + e2 x + 1 (sec x + tan x )11/ 2 11 7 
 [IIT-JEE 2012]
1  e2 x + e x + 1
(B) log  2 x  +C Solution:
2  e − e x + 1
sec2 x
1  e2 x − e x + 1 I=∫ dx
(C) log  2 x  +C (sec x + tan x )9 / 2
2  e + e x + 1
Let sec x + tan x = t . Then
1  e 4 x + e2 x + 1
(D) log  4 x  +C sec x  tan x  1/ t
2  e − e2 x + 1  [IIT-JEE 2008] Now,
Solution: We have (sec x tan x + sec2 x )dx = dt ⇒sec x (sec x + tan x )dx = dt
e x dx dt 1  1
I=∫ 4x sec x dx  , t    sec x
e + e2 x + 1 t 2 t 
e  x dx e3 x dx  1
J  4 x 2 x  4 x 2 x t  
1  t  dt 1 9 / 2 13 / 2
e e 1 e  e 1 I   9/2  (t t ) dt
2 t t 2
Now,
  9 1  13 1   
(e2 x − 1)e x dx 1 t 2 t 2  1 t 7 / 2 t 11/ 2 
J −I = ∫   9  13     11 
e 4 x + e2 x + 1 2   1  1  2   7  
2 2  2 2  
Put ex = t. Then  
exdx = dt 1 1 1 1 1 1
2
  t 7 / 2  t 11/ 2   7 / 2 
(t  1)dt 7 11 7t 11 t 11/ 2
J  I  4 2
t  t 1 1  1 t2 
 1   11/ 2   
1 2  dt t 11 7 
t 
 1  1 1 
 1 2  11/ 2 
 (sec x  tan x )2   k
t    1 (sec x  tan x ) 11 7 
 t
Hence, the correct answer is option (C).
1  e x  e  x  1
 log  x  x   c
2  e  e  1
1  e 2 x  e x  1
 log  2 x x   c
2  e  e  1
Mathematical Problem Book for JEE.indb 1002 07-06-2018 14:13:27
Chapter 22 | Indefinite Integration 1003

Practice Exercise 1 e x 1− x 2 + c
(C) (D) 1− x 2 sin−1 x + c
dx
1. ∫ is equal to x3
1+ x + x 9. The value of ∫ 1+ x 8 dx is equal to
2 2 3 3
(A) (1+ x )2 / 3 − x 2 / 3 + c (B) (1+ x )2 / 3 + x 3 / 2 + c 1 1
3 3 2 2 (A) tan−1 x 4 + c (B) tan−1 x 4 + c
4 2
3 3 2 2 1
(C) (1+ x )3 / 2 + x 3 / 2 + c (D) (1+ x )3 / 2 − x 3 / 2 + c (C) cot −1 x 2 + c (D) None of these
2 2 3 3 4
dx
2. The value of ∫ x is equal to
( e + 1)(2e x + 3) 10. The value of ∫ xe x dx is equal to
2
(A) x + In(e x + 1) − In(2e x + 1) + c (A) xe x + e x + c (B) xe x − e x + c
3 (C) − xe x + e x + c (D) None of these
1
( ) 2
(B) x − ln e x + 1 + ln 2e x + 3 + c
3 3
( ) 11. 
dx
is equal to
2 2x  x2
(C) x − ln(e x + 1) + ln(2e x + 3) + c
3 (A) sin−1(1− x ) + c (B) − cos −1(1− x ) + P
(D) None of these
(C) sin−1( x − 1) + c (D) cos −1( x − 1) + P
cos3 x dx
3. The value of ∫ 2 is equal to
sin x + sin x 1 p −1
12. If I = ∫ dp = f ( p ) + c , then f(p) is equal to
(A) log sin x − sin x + c (B) log |sin x| – sin x + c 2p p + 1
(C) log |sin x| + c (D) None of these
1 1 1 
dx (A) ln( p − p2 − 1) (B)  cos −1 p + sec −1 p
4. The value of ∫ is equal to 2 2 2 
x ( x n + 1)
1
xn 1 xn +1 (C) ln p + p2 − 1 − sec −1 p (D) None of these
(A) log + c (B) log +c 2
1+ x n n xn
sinq + cosq
xn +1 1 xn
13. ∫ (sin 2q ) dq is equal to
(C) log n
+ c (D) log n +c
x n x +1
(A) sin−1(sinq + cosq ) (B) sin−1q (sinq − cosq )

1+ x + x 2 −1 (C) sin−1(cosq − sinq ) (D) None of these


5. The value of ∫ 2
e tan x
dx is equal to
1+ x k
14. If ∫ x 6 sin(5 x 7 ) dx = cos(5 x 7 ), x ≠ 0, then
2 tan−1 x tan−1 x 5
( A) x e (B) e +c
−1
(C) xe tan x + c (D) None of these (A) k = 7 (B) k = –7
1 1
tan x (C) k = (D) k = −
6. ∫ cos x
dx is equal to 7 7

∫e
x
2 2 15. ( 4 x 2 + 8 x + 3) dx is equal to
(A) + c (B) +c
sin x cos x
(A) (2 x + 1) e x + k
2
(B) ( x + 1)2 e x + k
2 2
(C) +c (D) +c
tan x (sin x )3 / 2 (C) ( 4 x 2 + 3)e x + k (D) None of these

dx 2x
7. ∫ x ln x ln(ln x )
is equal to 16. The anti-derivative of w.r.t. x is
1− 4 x
(A) ln |(ln (ln x))| + c (B) |ln x| + c (A) log2 e ⋅ sin−1(2 x ) + k (B) sin−1(2 x ) + k
  1 
(C) ln  ln    + c (D) ln |ln x| + c (C) cos −1(2 x )log2 e + k (D) None of these
  x
dx
1+ 1− x 2 sin−1 x  17. ∫ is equal to
8. Value of the integral ∫ e  x  dx is equal to x x4 −1
 1− x 2 
  1
x
(A) sec −1 x 2 + c (B) sec −1 x 2 + c
e 2
(A) e x sin−1 x + c (B) + c
1− x 2 (C) tan−1 x 2 + c (D) cosec −1x 2 + c

Mathematical Problem Book for JEE.indb 1003 07-06-2018 14:14:31


1004 Mathematics Problem Book for JEE

dx 1
18. ∫ sec2 x + tan2 x is equal to (C)
4
sec 2q tan 2q + ln 4 sec 2q + tan 2q + c

(D) None of these


(A) 2 tan−1( 2 tan x ) + x + c (B) 2 tan−1( 2 tan x ) − x + c
cos 2 x + x + 1
(C) 2 tan−1(2 tan x ) + c (D) None of these 27. ∫ x 2 + sin 2 x + 2 x dx =
x 2 + sin2 x (A) log( x 2 + sin 2 x + 2 x ) + c (B) − log( x 2 + sin 2 x + 2 x ) + c
19. ∫ 1+ x 2
sec2 x dx is equal to
1
(A) tan x + c (B) tan x − tan−1 x + c (C) log ( x 2 + sin 2 x + 2 x ) + c (D) None of these
2
(C) tan x + tan−1 x + c (D) None of these
1+ tan x
20. ∫ sin 2x ⋅ logcos x dx is equal to
28. ò x + logsec x
dx is equal to

1  (A) log (x + log sec x) + c (B) −log (x + log sec x) + c


(A) cos2 x  + logcos x  + k (B) cos2 x logcos x + k
2  (C) log (x − log sec x) + c (D) None of these
1 
(C) cos2 x  − logcos x  + k (D) None of these (2 x + 1)e x
2  29. ∫ (2 x + 3)2 dx is equal to
3 + 2 cos x ex
21. ∫ (2 + 3 cos x )2 dx is equal to (A) + c (B) e x (2 x + 1) + c
2x +1
ex
sin x 2 cos x (C) e x (2 x + 3) + c (D) +c
(A) +c (B) +c 2x + 3
(3 cos x + 2) (3 sin x + 2) 2
 ln x − 1 
 2 cos x   2 sin x 
30. ∫  (ln x )2 + 1 dx is equal to
(C)  + c (D)  +c
 3 cos x + 2   3 sin x + 2  x ln x
(A) 2 + c (B)
x +1 (ln x )2 + 1
1
22. If ∫ f ( x )cos x dx = f 2 ( x ) + c , then f(x) can be x  x 
2 (C) + c (D) e x  2  + c
(A) x (B) 1 (ln x )2 + 1  x + 1
(C) cos x (D) sin x
( x 3 + x + 1)
31. The value of ∫ e x dx is equal to
23. If ∫ g( x )dx = g( x ), then ∫ g( x )(f ( x ) − f ′′ ( x ))dx is equal to (1+ x 2 )3 / 2
x
xe x 2e x
(A) g (x) (f (x) + f ′(x)) + c (B) g (x) (f (x) − f ′ (x)) + c (A) +C (B) +C
(1+ x 2 )1/ 2 (1+ x 2 )1/ 2
(C) g (x) f (x) f ′(x) + c (D) None of these ex
(C) +C (D) None of these
(1+ x 2 )1/ 2
∫e
ln(sin x )
24. dx is equal to

(A) sin x + c (B) −cos x + c cos 2x


logcos x
32. ∫ cos x
dx is equal to
(C) e +c (D) None of these
(A) 2sin x + log |(sec x − tan x)| + c
 x2 − x −1 
K +7
 x 2 + 1 (B) 2sin x − log |(sec x − tan x)| + c
5. ∫ 
2 + 1  2  dx is equal to (C) 2sin x + log |(sec x + tan x)| + c
 x   x  (D) 2sin x − log |(sec x + tan x)| + c
K +7
 x 2 − 1 K +8 sin x
 1 33. The value of ∫ dx is equal to
 x 
   x −  cos3 / 2 x
x
(A) + c (B) +c (A) 2 sin x + c (B) 2 cos x + c
k +7 k+8
K +8 (C) 2 sec x + c (D) 2 cosec x + c
 1
(C)  x −
  (K + 8) + c (D) None of these
x 2
 x +2
34. The value of ∫ e x   dx is equal to
 x + 4
∫ sec
3
26. 2q dq is equal to
ex x ex
1 (A) + c (B) +c
(A) secq tanq + ln secq + tanq + c x+4 x+4
2
1 ex e x x2
(B) sec 2q tan 2q + ln sec 2q + tan 2q + c (C) 2
+ c (D) +c
4 ( x + 4) x+4

Mathematical Problem Book for JEE.indb 1004 07-06-2018 14:15:28


Chapter 22 | Indefinite Integration 1005

x −a log( x /e )
35. The value of ∫
b−x
dx is equal to 43. The value of ∫ (log x )2
dx is equal to

 sin 2q  x +1 x −1
(A) (a + b )  + q  + c, where a sin2 q + b cos2 q = x (A) 2 + c (B) +c
 2  (log x ) (log x )2
 sin 2q  x log x
(B) (a − b )  + q  + c , where a sin2 q + b cos2 q = x (C) + c (D) +c
 2  log x x
 sin 2q 
(C) (a − b )  + q  + c , where a sin2 q − b cos2 q = x dx
 2  44. ∫ cos( x − a)cos( x − b) =
sin 2q 
(D) (a  b )   q   c , where a sin2 q − b cos2 q = x sin( x − a)
 2  (A) cosec(a − b )log +c
sin( x − b )
bx cos 4 x − a sin 4 x a sin 4 x cos( x − a)
36. If ∫ x 2
dx =
x
+ c, then a and b may be (B) cosec(a − b )log
cos( x − b )
+c
(A) a = 2, b = 2 (B) a = 1, b = 4
sin( x − b )
1 (C) cosec(a − b )log +c
(C) a = –1, b = 4 (D) a = , b = 2 sin( x − a)
4
cos( x − b )
( D) cosec(a − b )log +c
dx 1− x 3 − 1 cos( x − a)
37. If ∫ = a log + c , then
x 1− x 3 1− x 3 + 1
dx
(A) a =
1
(B) a =
2 45. ∫ x +a + x +b
=
2 3
1 2 2 
( C) a = (D) a = − (A) ( x + a)3 / 2 − ( x + b )3 / 2  + c
3 3 3(b − a) 
2 
 1 1 1 (B) ( x + a)3 / 2 − ( x + b )3 / 2  + c
38. If ∫ x loge (1+ 1/ x )dx = P( x )ln 1+ x  + 2 x − 2 ln(1+ x ) + c , 3(a − b ) 
then 2 
(C) ( x + a)3 / 2 + ( x + b )3 / 2  + c
x2 3(a − b ) 
(A) p( x ) = (B) p(x) = –1 (D) None of these
2
(C) p (x) = 1 (D) None of these
3 cos x + 3 sin x
cos x 46. ∫ 4 sin x + 5 cos x dx =
39. ∫ dx is equal to
x 27 3
cos x (A) x − log( 4 sin x + 5 cos x ) + c
(A) 2cos x + c (B) +c 41 41
x 27 3
(B) x + log( 4 sin x + 5 cos x ) + c
(C) sin x + c (D) 2sin x + c 41 41

1+ tan x 27 3
(C) x − log( 4 sin x − 5 cos x ) + c
40. ∫ x + log sec x dx = 41 41
(D) None of these
(A) log (x + log sec x) + c (B) −log (x + log sec x) + c
1
(C) log (x − log sec x) + c (D) None of these 47. If ∫ (sin 2 x + cos 2 x )dx = sin(2 x − c ) + a , then the value of a
2
( x 2 − 1) x2 + 1 and c is
41. If ∫ dx = k log tan−1 + c,
 x 2 + 1 x (A) c = p /4 and a = k (an arbitrary constant)
( x 4 + 3 x 2 + 1)tan−1   (B) c = −p /4 and c = p / 2 a = p / 2
 x 
(C) c = p /2 and a is an arbitrary constant
then k is equal to
(D) None of these
(A) 1 (B) 2
(C) 3 (D) 5
x3 − x − 2
42. The value of ∫
ln x
dx is equal to
48. ∫ (1− x 2 )
dx =
2
(1+ ln x ) 2 2
 x + 1 x  x − 1 x
x x ln x (A) log   − + c (B) log   + +c
(A) +c (B) +c  x − 1 2  x + 1 2
1− ln x 1+ ln x
2 2
x ln x  x + 1 x  x − 1 x
(C) +c (D) +c (C) log   + + c (D) log   − +c
1+ ln x x + x ln x  x − 1 2  x + 1 2

Mathematical Problem Book for JEE.indb 1005 07-06-2018 14:16:24


1006 Mathematics Problem Book for JEE

x 2dx (C) 2 sin x + cos x  + c (D) 2 sin x + x cos x  + c


49. ∫ (a + bx )2 =
x2
1  2a
(A) 2  x + log(a + bx ) −
a2 1 
+c
55. ∫ (9 − x 2 )3/ 2 dx =
b  b b a + bx 
x x x x
(A) − sin−1 + c (B) + sin−1 + c
1  2a a2 1  9− x 2 3 9− x 2 3
(B) 2  x − log(a + bx ) + +c
−1 x x
b  b b a + bx  (C) sin − + c (D) None of these
3 9 − x2
1  2a a2 1 
(C) 2
x + log(a + bx ) + +c
b  b b a + bx  1− x 2
56. ∫x 1+ x 2
dx =

1  a 2a a 1 
2
1
(D)  x + − log(a + bx ) − +c (A) sin−1 x 2 + 1− x 4  + c
b2  b b b a + bx  2  
1
(B) sin−1 x 2 − 1− x 4  + c
dx 2  
50. ∫ =
(1+ x ) p + q2 (tan−1 x )2
2 2
(C) sin−1 x 2 + 1− x 4 + c

(A)
1
log q tan−1 x + p2 + q2 (tan−1 x )2  + c (D) sin−1 x 2 + 1− x 2 + c
q  
1
57. If ∫ f ( x )sin x cos x dx = log(f ( x )) + c , then f ( x ) =
(B) log q tan−1 x + p2 + q2 (tan−1 x )2  + c 2(b − a2 )
2
 
1 1
2 2 (A) (B)
(C) ( p + q2 tan−1 x )3 / 2 + c 2 2
a sin x + b cos x 2 2
a sin x − b2 cos2 x
2 2
3q
(D) None of these 1 1
(C) (D)
a2 cos2 x + b2 sin2 x a2 cos2 x − b2 sin2 x
5
x
51. ∫ dx = dx
1+ x 3 58. ∫ ( x − a ) (b − x )
=
2
(A) (1+ x 3 )3 / 2 + c  x − a  x − a
9 (A) 2 sin−1  + c (B) sin−1  +c
 b − a   b − a 
2 2
(B) (1+ x 3 )3 / 2 + (1+ x 3 )1/ 2 + c
9 3  x + a
(C) 2 sin−1  + c (D) None of these
 b − a 
2 2
(C) (1+ x 3 )3 / 2 − (1+ x 3 )1/ 2 + c
9 3
59. If ∫ x e x cos x dx = ( x ) + c , then f(x) is equal to
(D) None of these
ex
52. ∫
dx
is equal to
(A) {(1− x ) sin x − x cos x }
2
sin x − cos x + 2
1 x p 1 x p ex
(A) −
2
tan  +  + c (B)
 2 8  2
tan  +  + c
 2 8
(B) {(1− x ) sin x + x cos x }
2
1 x p 1 x p ex
(C)
2
cot  +  + c (D) −
 2 8 2
cot  +  + c
 2 8 (C) {(1+ x ) sin x − x cos x }
2
( D) None of these
adx
53. ∫ b + ce x = dx
60. If I = ∫ = f(x) + c, then f(x) is equal to
a  e  x
a  b + ce x e x + 4e − x
(A) log   + c (B) log   +c 1  ex 
b x
 b + ce  b  ex  (A) 2 tan–1 (2 ex) (B) tan−1  
2  2
b  ex  b  b + ce x 
(C) log  x  + c (D) log   +c ex 1
a  b + ce  a  ex  (C) 2 tan−1 (D) tan−1(2 e2 x )
2 2
54. ∫ sin x dx =
61. ∫
dx
is equal to
(A) 2 sin x − cos x  + c (B) 2 sin x − x cos x  + c (
(2 x + 1) 1+ (2 x + 1) )

Mathematical Problem Book for JEE.indb 1006 07-06-2018 14:17:20


Chapter 22 | Indefinite Integration 1007

1
2x +1 2x +1
(A) tan-1 +c (B) loge +c ( x 4 − x )4
1+ 2 x + 1 1+ 2 x + 1 69. ∫ x5
dx is equal to

(C) loge  1+ 2 x + 1 + c (D) tan-1 1+ 2 x + 1 + c 5 5


  2x +1 4 1 4 4 1 4
 2x +1  (A)  1−  + c (B)  1−  + c
15  x 3  5  x3 
sin x 5
62. ∫ sin x − cos x dx is equal to 4 1 4
(C)  1+ 3  + c (D) None of these
15 x
x 1 x 1
(A) − log(sin x − cos x ) + c (B) + log(sin x − cos x ) + c
2 2 2 2 log( x + 1) − log x
x 1
70. ∫ x ( x + 1)
dx is equal to
( C) + log(sin x + cos x ) + c (D) None of these
2 2  x + 1   x + 1 
(A) − log  + c (B) − log log  +c
(1+ x )  x    x 
63. I = ∫ dx is equal to
x (1+ xe x )2 2
1   x + 1  1
(D) c − log( x + 1) − (logx ) 
2 2
(C) − log   + c
 xe   1  2   x  
x
2
(A) ln  x  − x +c
 1+ xe   1+ xe 
sin x − cos x sin x
 xe x   1 
71. ∫ 1− sin 2 x
e cos xdx is equal to
( B) ln  x + x +c
 1+ xe   1− xe  (A) e sin x + c (B) e sin x − cos x + c

 xe x   1  (C) e sin x + cos x + c (D) e cos x − sin x + c


(C) ln  x + x +c
 1− xe   1+ xe 
1 1− x 3 − 1
72. If ∫ dx = a log + b then a is equal to
 xe x   1  x 1− x 3 1− x 3 + 1
( D) ln  x + x +c
 1+ xe   1+ xe  1 2
(A) (B)
3 3
cos 5 x + cos 4 x
64.  ∫ 1− 2 cos 3 x
dx is equal to 1
(C) − (D) −
2
sin 2 x sin 2 x 3 3
(A) − sin x + c (B) − + sin x + c ax x
2 2 73. ∫ aa ⋅ aa ⋅ a x dx is equal to
sin 2 x sin 2 x
(C) − − sin x + c (D) + sin x + c aa
x
ax
2 2 (A) + c (B) aa (log a)3 + c
3
(log a)
e x (1+ x )
65. ∫ cos2 ( xe x ) dx =
aa
ax

(C) + c (D) None of these


x x (log a)3
(A) − cot( xe ) (B) tan( xe )
(C) tan(e x ) (D) None of these 74. ∫ e
log x
dx =
5

∫ tan
3
66. 2 x sec 2 xdx =
x log5 e x log5 5e
1 3 (A) (B)
(A) sec3 2x + 3 sec 2x (B) sec 2 x − 3 sec 2 x  log5 e log5 5e
6
x loge 5e + 1
(C) sec3 2 x − 3 sec 2 x  (D) None of these (C) (D) None of these
loge 5e + 1
sec x cosec x
67. ∫ dx
logtan x
75. ò x 2 sin xdx =
(A) log (tan x) + c (B) cot (log x) + c
(C) log (log tan x) + c (D) tan (log x) + c ( A) x2 sin x − 2 x cos x + c
(B) x2 sin x + c
68. ∫ cos3 x e ( )dx is equal to
log sin x
(C) − x2 cos x + 2 x sin x + 2 cos x + c
sin4 x cos 4 x (D) − x2 sin x − 2x cos x + sin x + c
(A) − + c (B) − +c
4 4
 (1− cos x ) 
(C)
e sin x
+ c (D) None of these
76. ∫ [cos x (1+ cos x )]  dx =
4  

Mathematical Problem Book for JEE.indb 1007 07-06-2018 14:18:16


1008 Mathematics Problem Book for JEE

 x ( A) x sin x + cos x − sin2 x + c


(A) log (sec x + tan x) − 2 tan   (B) x cos x − sin2 x + c
 2
(C) x sin x + cos x − (cos2 x)/2 + c
 x (D) x2 sin x + cos x − sin3 x + c
(B) log (sec x + tan x) + 2 tan  
 2 x + sin x
84. ∫ 1+ cos x dx is equal to
 x
(C) log (sec x − tan x) − 2 tan  
 2 x x
(A) x tan + c (B) tan + c
(D) None of these 2 2
x x
(C) x tan x + c (D) tan + c
77. ∫ (1− x ) x dx =
23
2 2
1
x 23 x 24 ( x − 1)23 ( x − 1)24 85. ∫ 4 cos3 x − 3 cos x dx is equal to
(A) + + c (B) + +c
23 24 23 24 1 1
(A) ln|sec 3x – tan 3x| + c (B) ln|sec 3x + tan 3x| + c
(1− x ) 25
(1− x ) 24 3 3
(C) − + c (D) None of these 1 1
25 24 (C) ln|sec 3x + tan 3x| + c (D) ln|sec 3x – tan 3x| + c
4 4
78. ∫ {sin(log x ) + cos(log x )} dx = 86. ∫ sin(log x )dx is equal to
( A) x sin (log x) + c (B) x cos (log x) + c
x x
(C) x log (sin x) + c (D) x log (cos x) + c (A) [sin (ln x) + cos (ln x)] + c (B) [cos (ln x) – sin (ln x)] + c
2 2
dx x
79. ∫ cos6 x + sin6 x is equal to (C) [sin (ln x) – cos (ln x)] + c (D) x[sin (ln x) – cos (ln x)] + c
2

(A) loge (tan x − cot x) + c (B) loge (cot x – tan x) + c


Practice Exercise 2
( C) tan–1 (tan x − cot x) + c (D) tan–1 (2 cot 2 x) + c
3 + 2 cos x Single/Multiple Correct Choice Type Questions
80. ∫ (2 + 3 cos x )2 dx is equal to
1. If Im, n = ∫ cos m x cos nxdx , then the value of (m + n)Im, n − m
 sin x   2 cos x  Im − 1, n − 1 (m, n ∈ N) is equal to
(A)  + c (B)  +c
 3 cos x + 2   3 sin x + 2  cos m x sin nx
(A) + c (B) cosmx sinnx + c
 2 cos x   2 sin x  n
(C)  + c (D)  +c
 3 cos x + 2   3 sin x + 2  cos m x cos nx
(C) + c (D) − cosmx cosnx + c
2 1
n
(x + x 3 + x 6 ) sin4 x sin4 (( p + q ) x )
81. I = ∫ 1
dx is equal to 2. If the anti-derivative of ∫ dx is f(x), then ∫ dx
x x
x (1+ x3) in terms of f(x) is
2  1 2  1 f (( p + q ) x )
3 3 (A) f((p + q)x) (B)
(A) x3 + 6 tan−1  x 6  + c (B) x3 − 6 tan−1  x 6  +c p+q
2   2  
(C) f((p + q)x) (p + q) (D) None of these
2  1
3 3 x ndx
(C) x + tan−1  x 6  + c (D) None of these 3. If I = ∫ , then
2   x x2 xn
1+ + ++
1! 2 ! n!
1
82. I = ∫ dx is equal to
1  x x2 xn  
1− e 2 x (A) I =  x + ln  1+ + ++   + c
n!   1! 2 ! n!  
sin−1 e x sin−1 e x
(A) ln + c (B) tan ln +c x 2 − e x ln x
2 2 (B) I = +c
n !(e x − 1)
 sin−1 e x  æ ex ö
(C) ln tan   + c (D) None of these (C) I = n !ln ç ÷+c
 2  ç
2 n
x x ... x ÷
ç 1 + + + + ÷
è 1! 2 ! n! ø
∫ (e
log x
83. + sin x )cos x dx is equal to (D) None of these

Mathematical Problem Book for JEE.indb 1008 07-06-2018 14:18:58


Chapter 22 | Indefinite Integration 1009

x 2 dx x dx p  p 
  4. Let f ( x ) = ∫ and g( x ) = ∫ , if (C) g′   is equal to 6
 3
(D) g′   is equal to 12
 3
2 2 4
(1+ x )(1+ 1+ x ) 1+ x
p 11. Let f(x) = [b2 + (a – 1)b + 2]x – ∫ (sin2 x + cos 4 x )dx be an
f(0) = g(0) = 0. Then the value of f (1) − 2g(1) + is
4
increasing function of x ∈ R and b ∈ R. Then a can take value(s)
(A) 0 (B) 1 ( A) 0 (B) 1
(C) 2 (D) None of these (C) 2 (D) 4
4
x10 + x 8 + 1 Comprehension Type Questions
  5. ∫ x 6 (3 x 10
)
+ 2 x 8 − 2 dx =
Paragraph for Questions 12 and 13: Let n be a positive integer
2( x10 + x 8 + 1)5 4 4( x10 + x 8 + 1)5 4 such that In = ∫ x n a2 − x 2 dx. Then answer the following questions:
(A) + c (B) +c
5x5 5
12. The value of I1 is
2( x 6 + x 4 + x −4 )5 4 2 2 1 2
(C) + c (D) None of these (A) ( a − x 2 )1/ 2 + C (B) ( a − x 2 )3/ 2 + C
5x5 3 3
2 1
  6. ∫ ( x 6 + x 4 ) (2 x 4 + 3 x 2 )dx , x > 0 = (C) – ( a2 − x 2 )3/ 2 + C (D) – ( a2 − x 2 )3/ 2 + C
3 3
3
1 1 6 2 − x n −1(a2 − x 2 )3 / 2
(A) (2 x 6 + 3 x 4 )3 / 2 dx + c (B) x ( 2 x + 3) 2 + c 13. If In = + kIn − 2, then the value of k is
6 18 n+2
3
1 1 2 2 n −1 n+2
(C) (2 x 6 + 3 x 4 )3 / 2 dx + c (D) x ( 2 x + 3) 2 + c (A) (B)
12 18 n+2 n −1
 n − 1 2  n + 2 2
x4 −1 (C)  a (D)  a
 n + 2   n − 1 
  7. ∫ ( x 5 + 1)( x + 1) dx =

1  ( x 5 + 1)  1  ( x 5 + 1) 
4
Matrix Match Type Questions
(A) ln   +c (B) ln   +c
5  ( x + 1)  5  ( x + 1)  14. Match the following:

 ( x 5 + 1)1 5  Column I Column II


(C) ln   +c (D) None of these
(A) If fr(x) = log log loglog x . Then (p) p
 ( x + 1)    
r times
e2 x − e x + 1 −1
  8. ∫ (e x sin x + cos x )(e x cos x − sin x ) dx = ∫ {xf1( x )f2 ( x )f100 ( x )} dx
= fk ( x ) + c , where k =
 e cos x − sin x 
x  e cos x + sin x 
x
(A) ln  x  + c (B) ln  x  +c x + sin x p
 e sin x + cos x   e sin x + cos x  (B) f ( x ) = ∫ dx and f (0) = 0, (q)
1+ cos x 4
 e x cos x − sin x  p 
(C) ln  x  + c (D) None of these then f   is
 e sin x − cos x   2
(C) Let (r) 101
  9. If ∫ (sin 3q + sinq )cosq e sinq dq = ( A sin3 q + B cos2 q + C sinq −1  x 
f ( x ) = ∫ e sin x
 1−  dx
+ D cosq + E )e sinq + F , then  1− x 2 
p 6
(A) A = 4 (B) B = –12  1 k 3 e
   and f (0) = 1 if f   = ,
(C) C = 20 (D) D = 0  2 p
then k is
1
10. Let f(x) = and its anti-derivative  tan x  p
4 − 3 cos x + 5 sin2 x dx and (s) 2
2
(D) Let f ( x ) = ∫  
1  sin x .cos x 
F(x) = tan−1( g( x )) + c, then
3  p  2k
p  f (0) = 0 if f   = ,then k is
(A) g(x) is equal to 3 tanx (B) g   is equal to 3  4 p
 4

Mathematical Problem Book for JEE.indb 1009 07-06-2018 14:19:44


1010 Mathematics Problem Book for JEE

Answer Key
Practice Exercise 1
1. (D) 2. (B) 3. (B) 4. (D) 5. (C) 6. (B)
7. (A) 8. (A) 9. (A) 10. (B) 11. (C) 12. (C)
13. (B) 14. (D) 15. (C) 16. (A) 17. (B) 18. (B)
19. (B) 20. (C) 21. (A) 22. (D) 23. (B) 24. (B)
25. (B) 26. (B) 27. (C) 28. (A) 29. (D) 30. (C)
31. (D) 32. (D) 33. (C) 34. (A) 35. (B) 36. (B)
37. (C) 38. (A) 39. (D) 40. (A) 41. (A) 42. (C)
43. (C) 44. (B) 45. (B) 46. (A) 47. (A) 48. (D)
49. (D) 50. (A) 51. (C) 52. (D) 53. (A) 54. (B)
55. (A) 56. (A) 57. (A) 58. (A) 59. (A) 60. (B)
61. (B) 62. (B) 63. (D) 64. (C) 65. (B) 66. (B)
67. (C) 68. (B) 69. (A) 70. (C) 71. (A) 72. (A)
73. (C) 74. (D) 75. (B) 76. (A) 77. (C) 78. (A)
79. (C) 80. (A) 81. (A) 82. (C) 83. (C) 84. (A)
85. (B) 86. (C)

Practice Exercise 2
1. (B) 2. (A) 3. (C) 4. (A) 5. (A) 6. (B)
7. (C) 8. (A) 9. (A), (B) 10. (B), (D) 11. (A), (B), (C) 12. (D)
13. (C) 14. (A) → (r), (B) → (s), (C) → (s), (D) → (p)

Solutions

Practice Exercise 1 1 n ⋅ x n1dx


4. (
n ∫ x n ( x n  1)
put x n  t )
1+ x − x
1. I = ∫ dx
(1+ x ) − ( x ) 1 dt 1 dt 1 dt
2 3/ 2 2
 ∫  ∫  ∫
n t (t  1) n t n (t  1)
= ( x + 1) − x 3 / 2 + c
3 3 1  t  1 xn
x log    c  log n c
e dx n  t  1 n x 1
∫ e x (e x + 1)(2e x + 3) (Let e
x
2. = t)
1 x  x 2 tan1 x
1 dt
5. I  e dx
1 x 2
2∫
I=
 3 dx
t (t + 1)  t +  Put tan1 x  t . Then  dt . So,
 2
1 x 2
1 dt dt 2 dt I (1 tan t  tan2 t )et dt (sec2 t  tan t )et dt
3 ∫ t ∫ t + 1 3 ∫  3
l= − +
1

 t +  I  et tan t  c  xe tan x
c
2
tan x sin x
=
x 2
− ln(e x + 1) + ln(2e x + 3) + c
6. ∫ cos x
dx = ∫
(cos x )3 / 2
dx (Let cos x = t )
3 3
= − ∫ t −3 / 2dt
(1− sin2 x )cos x dx
3. I = ∫
(sin x + sin2 x ) t −1/ 2
=− +c
1
 1− t 2  −
= ∫  dt [Put (sin x = t)] 2
 t + t2  2
= +c
cos x
 1− t 
= ∫ dt
 t  dx
7. ∫ x ln x ln(ln x ) (Put In x = t)
= log |sin x| − sin x + c

Mathematical Problem Book for JEE.indb 1010 07-06-2018 14:20:07


Chapter 22 | Indefinite Integration 1011

dt 1
∫ t lnt ⇒ k=−
7
1
∫e ( 4 x 2 + 8 x + 3)dx = ∫ e x [f ( x ) + f ’( x )] + 3∫ e x dx
x
  Let ln t = z. Then, dt = dz . So, 15.
t
dz = 4 x 2e x + 3e x + k
ò z = log z = log (ln t)
16. Put 2 x = t . Then
= ln |ln(ln x)| + c
2 x ln2 dx = dt

 2 −1 
x 1+ 1− x sin x  1 dt
8. ∫ e  
dx    log2 e  sin1 t   c
 1− x 2  ln 2 1 t 2

ex 2x
=∫
1− x 2
dx + ∫ e x ⋅ sin−1 x dx


1 4 x
dx  log2 e  sin1 2 x  k  
ex ex
=∫ dx + e x sin−1 x − ∫ dx 17. Put x 2 = t . Then
1− x 2 1− x 2
2x dx = dt
= e x sin−1 x + c
1 dt 1
x 3  
2 t t 1 2
2
 sec 1 t   c
9. ∫ 1+ x 8 dx

 
1 2 x dx 1
   sec 1 x 2  c
 
  Let x 4 = t . Then 4 x 3dx = dt . 2 2 2 2
x x2 1
1 dt 1
= tan−1 t + c
4 ∫ 1+ t 2 4
I=
dx (1− sin2 x ) 2 − (1+ sin2 x )
1
18. ∫ sec2 x + tan2 x = ∫ (1+ sin2 x ) dx = ∫ (1+ sin2 x )
dx
I = tan−1 x 4 + c
4 sec2 x
= 2∫ dx − ∫ 1 dx (Let tan x = t )
∫ xe dx = x ∫ e x dx − ∫ e x dx + c = xe x − e x + c 2 tan2 x + 1
x
10.
2 dt
dx dx = ∫ − x = 2 tan−1( 2 tan x ) − x + c
11. ò =ò = sin-1( x - 1) + c 2 2  1 2
2 2 t +
1- ( x - 2 x + 1) 1- ( x - 1)  2 

12. I  1 p  1 dp  1 p 1
dp  
1
dp x2 tan2 x dx
2 p p2  1 2 p p2  1 2 p p2  1 19. I = ∫ 2
sec2 x + ∫
1+ x 1+ x 2
1
f ( p )  ln p  p 2  1  sec 1 p sec2 x tan2 x dx
   2 = ∫ sec2 x − ∫ dx + ∫
1+ x 2 1+ x 2
13. From given equation, we have
= tan x − tan−1 x + c
(sinq + cosq )dq
∫ 1 1 sin x
20. I = − logcos x ⋅ cos 2 x − ∫ cos 2 x dx
1− (sinq − cosq )2
2 2 cos x
Put sin q − cos q = t. Then
1 1  2 cos2 x − 1
(cos q + sin q ) dq = dt = − cos 2 x log cos x − ∫  sin x dx
Therefore, 2 2  cos x 

dt Let cos x = t . Then -sin xdx = dt .


∫ = sin−1(sinq − cosq ) + c
1− t 2 1 1  2t 2 − 1
  = − cos 2 x logcos x + ∫  dt
2 2  t 
14. Put 5 x 7 = t . Then

35 x 6dx = dt 1 t2 1
= − cos 2 x logcos x + − log t
  Therefore, from given equation, we have 2 2 2
1 k 1 (cos x )2 1
35 ∫ sin t ⋅ dt = cos(5 x 7 )
5 = − cos 2 x ⋅ logcos x + − logcos x + k
2 2 2
− cos(5 x 7 ) k  1 
⇒ = cos(5 x 7 ) = cos2 x  − logcos x  + k
35 5 2 

Mathematical Problem Book for JEE.indb 1011 07-06-2018 14:20:53


1012 Mathematics Problem Book for JEE

3 + 2 cos x 1
21. I = ∫ dx I2 sec 2q . tan2 2q dq   sec2 2q  1. 2sec2q tan 2q dq
(2 + 3 cos x )2 2
sin x 1
  Let
3 cos x + 2
= t . Then  I2 
2
 t 2  1dt put sec 2q  t 

 
(3 cos x + 2)cos x − sin x ( −3 sin x ) 1 t 1 
dx = dt  I2   t 2  1  ln t  t 2  1   c2
2 2 2 
(3 cos x + 2)2
1 1

3 + 2 cos x
dx = dt
 I2  sec 2q sec2 2q  1  In sec 2q  sec2 2q  1  c
2   4  2  
(3 cos x + 2)
1 1
sin x I  I1  I2  sec 2q .tan2q  lnsec 2q  tan2q   c
⇒ I = ∫ 1 dt = t + c = +c 4 2
3 cos x + 2
27. Let x 2 + sin 2 x + 2 x = t. Then
1
22. ∫ f ( x )cos x dx = f 2 ( x ) + c ⇒ (2x + 2 cos 2x + 2 )dx = dt
2
dt
  By putting f (x) = sin x, we get ⇒ ( x + cos 2 x + 1)dx =
2
1 1 1 dt 1 1
   I = ∫ sin 2 x dx = cos 2 x + c ⇒ I = ∫ = log t = log( x 2 + sin 2 x + 2 x ) + k
2 4 2 t 2 2
1 d
= − (1− 2 sin2 x ) + c 28. As ( x + logsec x ) = 1+ tan x , so
4 dx
1 I = log( x + logsec x ) + c
= sin2 x + k
2 ex ex
1
29. ∫ (2 x + 3) dx − 2∫ (2 x + 3)2 dx
= f 2( x ) + k
2
ex
Hence, f(x) = sin x. = ∫ e x [f ( x ) + f ’( x )]dx = +c
2x + 3
23. I = ∫ g( x ) f ( x ) dx − ∫ g( x ) f ’’( x ) dx (ln x )2 − 2(ln x ) + 1  (ln x )2 + 1 ln x 
30. ∫ 2
((ln x ) + 1) 2
dx = ∫  2
 ((ln x ) + 1)
2
−2 2 2
((ln x ) + 1) 
dx
( )
= f ( x ) ⋅ ∫ g( x )dx − ∫ f ’( x ) ⋅ ∫ g( x )dx − ∫ g(x) f " ( x )dx
=∫
1
dx − 2∫
ln x
dx
= f ( x )g( x ) − [ g( x ) ⋅ f ’( x ) − ∫ f ’’( x ) ⋅ g( x )dx ] − ∫ g( x ) f "( x )dx (ln x )2 + 1 (ln x )2 + 1
x x 2 ln x ln xdx
= g(x) (f(x)) – f ’(x) + c = 2
+∫ 2 2
dx − 2∫ +c
(ln x ) + 1 x ((ln x ) + 1) (1+ (ln x )2 )2
∫e dx = ∫ sin x dx = −cos x + c
ln(sin x )
24. x
= +c
(ln x )2 + 1
k +7
 x2 − x −1   x 2 + 1
25. I = ∫  + 1  2  dx e x  x ( x 2 + 1) + 1
 x   x  31. ∫ ( x 2 + 1)3 / 2
dx

k +7
 x 2 − 1  x 2 + 1  x 1 
= ∫ = ∫ ex  2 + 2 3/ 2 
dx
  2  dx  ( x + 1)1/ 2
( x + 1) 
 x   x 
 Now, (Since, e x [f ’( x ) + f ( x )] = e x f ( x ) + c )
 1  1 xe x
 x −  = t ⇒  1+ 2  dx = dt = +c
x x ( x 2 + 1)1/ 2
k +8 2 cos2 x − 1
 1
 x − 
32. ∫ cos x
dx
k +7 t k +8 x
I = ∫ t dt = +c = +c
k+8 k+8 = 2∫ cos x − ∫ sec x dx
26. I sec (2q ) dq sec( 2q ) 1 tan2 2q  dq
3
= 2sin x − log|(sec x + tan x )| + c

 
sec 2q dq sec 2q  tan2 2q dq 33. cos x = t ⇒ − sin x dx = dt
  
t −3 / 2 +1 2
1 I = − ∫ t −3 / 2dt = − +c = + c = 2 sec x + c
I1 sec 2q dq  lnsec 2q  tan2q   c1 3 t
   2 − +1
2

Mathematical Problem Book for JEE.indb 1012 07-06-2018 14:21:42


Chapter 22 | Indefinite Integration 1013

2 2
x  x + 2 x 2   sec x tan x 
40. x + log sec x = t ⇒  1+  dx = dt
34. ∫ e  x + 4  dx = ∫ e 1− x + 4  dx  sec x 

 4 4   Hence, I = log( x + logsec x ) + c .


= ∫ e x 1− + 2
dx
 x + 4 ( x + 4)   2 
41. Let tan−1  x + 1 = q. Then
 4 4   x 
= e x + ∫ e x − + 2
dx
 x + 4 ( x + 4)  1 x (2 x ) − ( x 2 + 1)
x x    2
× dx = dq
4e xe  x 2 + 1 x2
= ex − = +c 1+ 
x+4 x+4 
 x 
x −a
35. ∫ b−x
dx
  ⇒
( x 2 − 1)
dx = dq
( x + 3 x 2 + 1)
4

Let x = a sin2 q + b cos2 q . Then  x 2 + 1


  dx = (2a sinq cosq − 2b cosq sinq )dq =( a − b )sin2q ⋅ dq   ⇒ I = log tan−1   +c
 x 
cos2 q (b − a)   Hence, the correct answer is (1).
= ∫ (b − a)sin q 2
× (a − b )sin 2q dq = 2∫ (a − b )(cos2 q )dq
ln x 1 1
42. ∫ dx = ∫ dx − ∫ dx
2
 1  (1+ ln x ) 1+ ln x (1+ ln x )2
= (a − b ) q + sin 2q  + c
 2  1
x x ⋅ dx 1
x
1+ ln x ∫ (1+ ln x )2 ∫ (1+ ln x )2
 sin 4 x − sin 4 x  a = + − dx
36.  −∫ 2
dx  − ∫ 2 sin 4 x dx
 4x 4x  x
x
b sin 4 x  b − a  sin 4 x = +c
= + dx 1+ ln x
4 x  4  x 2
Hence, a = 1, b = 4. log( x/ e )  1 1 
43. ∫ 2
dx = ∫  − 2
dx
dx x 2dx
(log x )  log x (log x ) 
37. ∫ =∫
=
x
+∫
x
dx − ∫
1
dx
x 1− x 3 x 3 1− x 3 2
log x x (log x ) (log x )2
 Let 1− x 3 = t 2 . Then − 3 x 2dx = 2tdt . x
= +c
log x
2 tdt 2 dt 2 dt
I=− ∫ =− ∫
3 1− t 2 3 ∫ t 2 − 1
=
3 (1− t 2 )t dx
44.  ∫
cos( x − a)cos( x − b )
1 t −1 1 1− x 3 − 1
log = log 1 sin (( x − b ) − ( x − a))
= 3
sin(a − b ) ∫ cos( x − a)cos( x − b )
t +1 3 1+ x 3 + 1 = dx
1
⇒ a= 1  sin( x − b ) sin( x − a) 
3 = ∫ −
sin( a − b )  cos( x − b ) cos( x − a) 
dx
 1  1 x 1
38. ∫ x log 1+ x  dx = P( x )ln 1+ x  + 2 − 2 ln(1+ x ) + c =
1
ln
cos( x − a)
+c
1 sin(a − b ) cos( x − b )
2 − 2
 1 x x x2
 LHS = log  1+  ⋅ − ∫ ⋅ dx dx ( x + a − x + b ) dx
 x 2  1 2
 1+ 
45. ∫ x +a+ x +b
=∫
( x + a − x − b)
x
1 2
(a − b ) ∫
 1 x
2
1 x x2  1 x 1 = ( x + a − x + b ) dx = (( x + a)3 2 − ( x + b )3 2 ) + c
  = log  1+  ⋅ + ∫ dx = ln  1+  + − ln(1+ x ) + c 3 (a − b )
 x  2 2 x +1 2  x 2 2
d
x2 46. 3 cos x + 3 sin x = a ( 4 sin x + 5 cos x ) + b ( 4 sin x + 5 cos x )
 Hence, p( x ) = . dx
2
3 cos x + 3 sin x = cos x (5a + 4b ) + sin x ( 4 a − 5b )
1
39. Let x = t . Then dx = dt .    Compare the coefficients of sin x and cos x on the both sides
2 x
(5a + 4b ) = 3, ( 4 a − 5b ) = 3
I = 2∫ cos dt = 2 sin t + c
27 3
a= ,b=−
= 2sin x + c 41 41

Mathematical Problem Book for JEE.indb 1013 07-06-2018 14:22:24


1014 Mathematics Problem Book for JEE

3 cos x + 3 sin x 27 3 4 cos x − 5 sin x 2 x p 1 x p


∫ 4 sin x + 5 cos x dx = ∫ 41 dx − 41∫ 4 sin x + 5 cos x dx =−
2 2
cot  +  + c = −
 2 8 2
cot  +  + c
 2 8
3 cos x + 3 sin x 27 3
∫ 4 sin x + 5 cos x dx = 41 x − 41ln 4 sin x + 5 cos x + c adx ae x dx
53. ∫b + ce x = ∫be x + ce2 x
cos 2 x sin 2 x Now, put e x = t , then it reduces to
47. ∫ (sin 2 x + cos 2 x ) dx = − + +k
2 2
dt a  c 1
1  cos 2 x sin 2 x  1  p a∫ = − ∫ −  dt {By partial fraction}
= − +  +k = sin  2 x −  + k t (b + ct ) b  (b + ct ) t 
2 2 2  2  4
p a ex
So, c = and a = k, an arbitrary constant. = ln +c
4 b (b + ce x )
3
x − x −2 x ( x 2 − 1) 2
48. ∫ dx = ∫ dx − ∫ dx 1
1− x 2
1− x 2 1− x 2 54. Put x = t , then dx = dt .
2 x
2 x2 x −1
= − ∫ x dx + ∫ 2
dx = − + ln +c ∫ sin x dx = 2∫ t sin t dt = 2 ( −t cos t + sin t ) + c
x −1 2 x +1
= 2 ( − x cos x + sin x ) + c
49. Put a + bx = t. Then
55. Put x = 3 sinq , then dx = 3 cosq dq . Therefore,
t −a dt
     x = ⇒ dx = x2 9 sin2 q ⋅ 3 cosq
b b ∫ (9 − x 2 )3 2 dx = ∫ (9 − 9 sin2 q )3 2 dq
2
 t − a  dt
I = ∫ ⋅
 b  t 2b =∫
27 sin2 q ⋅ cosq
dq = ∫ tan2 q dq = ∫ (sec2 q − 1) dq = tanq − q + c
1  2a 2 −2  1 27 cos3 q
I = 3 ∫  1− + a t  dt = 3 (t − 2a ln t − a2t −1) + c
b  t  b  x x x x
= tan  sin−1  − sin−1 + c = − sin−1 + c
1   3 3 9− x 2 3
I= (a + bx ) − 2a ln(a + bx ) − a2 (a + bx )−1 + c
b3  1− x 2 x ⋅ (1− x 2 ) x x3
dx dt .
56. ∫x 1+ x 2
dx = ∫ dx = ∫ dx − ∫ dx
50. Putting q tan−1 x = t . Then = 1− x 4 1− x 4 1− x 4
2 q
1+ x 1
= (sin−1 x 2 + 1− x 4 ) + c
dx 1 dx 2
∫ 2 2 −1 2
= ∫
q
(1+ x ) p + (q tan x ) p2 + t 2 57. Since ∫ f ( x )sin x cos x dx =
1
ln f ( x ) + c
2 ( b 2 − a2 )
1   Therefore,
= ln q tan−1 x + p2 + (q tan−1 x )2 + c
q 1 f ’( x )
f ( x )sin x ⋅ cos x = 2 2 f (x)
2 (b − a )
51. Put 1+ x 3 = t 2 . Then 3 x 2dx = 2tdt and x 3 = t 2 − 1.
f ’ (x)
   ⇒ 2 (b2 − a2 )sin x ⋅ cos x =
  So, f 2( x )
x5 x2 × x3 f ’ (x)
   ⇒ 2 (b2 ∫ sin x ⋅ cos x dx − a2 ∫ sin x ⋅ cos x dx ) = ∫ dx
ò dx = ò dx
f 2( x )
1+ x 3 1+ x 3 1
2 (t 2 − 1) ⋅ t 2 2  t3 
   ⇒ ( −b2 cos2 x − a2 sin2 x ) = −
= ∫ dt = ∫ (t 2 − 1) dt =  − t  + c f (x)
3 t 3 3 3  1
   ⇒ − f ( x ) =
2  (1+ x ) 3 3/ 2  ( −b2 cos2 x − a2 sin2 x )
=  − (1+ x 3 )1/ 2  + c
3 3  58. Put x = acos2q  + bsin2q , the given integral becomes

dx dx 2(b − a) sinq cosq dq


52. I = ∫ =∫ I=∫ 1
sin x − cos x + 2  p p 
2  sin x sin − cos x cos + 1
 4 4 
{(a cos q + b sin q − a)
2 2
(a cos2 q + b sin2 q − b } 2

2(b − a)sinq cosq dq  b − a


x p
cosec2  +  dx = ∫ (b − a)sinq cosq
=
 b − a  ∫ 2 dq = 2q + c
dx dx  2 8
=∫ =∫
x p ∫
=
  p  2 2  x − a
2  1− cos  x +   2 2 sin2  +  =2 sin−1  +c
  4   2 8  b − a 

Mathematical Problem Book for JEE.indb 1014 07-06-2018 14:23:07


Chapter 22 | Indefinite Integration 1015

59. I = real part of ∫ x e(1+ i ) x d x 1 A B C


= + +
( p − 1)p2 ( p − 1) p p2
xe(1+ i ) x e(1+ i ) x xe(1+ i ) x e(1+ i ) x
= −∫ dx = − 1 = Ap2 + B (p) (p − 1) + C (p − 1)
1+ i 1+ i 1+ i (1+ i )2
For p = 1, p = 0 and p = −1, A = 1, C = −1 and B = −1
x  x (1+ i ) − 1
= e(1 + i )  2  1 dp dp ( p − 1) 1
 (1+ i )  I=∫ dp − ∫
p ∫ p2
− = ln + +c=
( p − 1) p p
 ( x − 1) + ix 
= ex (cos x + i sin x) 
 1+ 2i − 1   xe x   1 
ex ln  x + x +c
= [i cos x – sin x][(x – 1) + ix]  1+ xe   1+ xe 
−2
ex 9x x 3x 9x 3x x
I= [(1 – x) sin x – x cos x] + c 2 cos cos cos 2 cos cos cos
2 64. ∫ 2 2 2 dx = ∫ 2 2 2 dx
  2 3x  3x 3x 3x
dx 1− 2  2 cos 2 − 1  cos 2 3 cos − 4 cos3
60. I = ∫ = f (x) + c   2 2
e + 4e − x
x
9x 3x x
2 cos cos cos
e x dx 2 2 2
⇒I= ∫ =∫ dx = − ∫ (cos 2x + cos x ) dx
e2 x + 4 9x
− cos
 Let ex = t. Then ex dx = dt. 2
 ex  sin 2 x
dt1 t 1 =− − sin x + c
I=∫ = tan−1   + c = tan−1   + c 2
2
t +4 2  2 2  2
65. Put x ex = t, then (xex + ex) dx = dt.
dx dx
61. ∫
(2 x + 1)(1+ (2 x + 1)) ∫ ( (2 x + 1))2 (1+ (2 x + 1))
= Therefore,

I = ∫ sec2 tdt = tan t + c = tan (xex) + c


Put 2 x + 1 = t . Then
2 x + 1 = t . Then
dx dx 66. I = ∫ tan2 2 x tan 2 x sec 2 xdx = ∫ (sec2 2 x − 1)sec 2 x tan 2 xdx
= dt ⇒ dx ==t ⋅dt dt⇒ dx = t ⋅ dt
2x +1 2x +1 Put sec 2x = t. Then 2 sec 2x tan 2x dx = dt.
Therefore, 1 2 1  t3  1
2∫
3
I= ( t − 1) dt =  − t  = (sec 2 x − 3 sec 2 x )
t. dt dt 1 1  2 3  6
∫ t 2 (1+ t ) = ∫ t (1+ t ) = ∫  t − (1+ t )  dt
67. Put log tan x = t, then
t
= ln t − ln(1+ t ) + c = ln +c 1
t +1 ⇒ sec2 xdx = dt ⇒ sec x cosec x dx = dt
tan x
2x +1
= ln +c dt
2x +1+1 ∫ t = log t + c = log (log tan x)+c
d
62. Let sin x = A (sin x − cos x) + (sin x − cos x). Then 68. elogsin x = sin x
dx
sin x = A (sin x − cos x) + B (cos x + sin x) Therefore,
⇒ sin x = (A + B ) sin x + (B − A) cos x
∫ cos
3
x sin x dx
Equating the coefficients of sin x and cos x, we get
A + B = 1 and B − A = 0 Put cos x = t, we get
A = 1/2, B = 1/2 t4 cos 4 x
1 1 − ∫ t 3dt = − + c= − +c
(sin x − cos x ) + (cos x + sin x ) 4 4
I=∫ 2 2
1
sin x − cos x
( x 4 − x )4
1
 = ∫ dx + ∫
1 cos x + sin x x 1
dx = + log(sin x − cos x ) + c
69. I= ∫ x5
dx
2 2 sin x − cos x 2 2
1
(1+ x ) (1+ x )e x Put 1– = t, then
63. Let I = ∫ dx = ∫ dx . Then x3
x (1+ xe ) x 2 x
( xe )(1+ xe ) x 2 3
dx = dt
(1+ xex = p ⇒ ex (1 + x) dx = dp) x4
Therefore,
dp 1
I=∫ 1 4
3∫
( p − 1)p 2 I= t dt

Mathematical Problem Book for JEE.indb 1015 07-06-2018 14:23:50


1016 Mathematics Problem Book for JEE

1 t 5/ 4 4 1
5/ 4 1− cos x 1− t
= ⋅ + c =  1− 3  +c =
3 5/ 4 15  x  cos x (1+ cos x ) t (1+ t )

 x + 1 1 2   1 2 
=  − = −
log 
 x  1  t 1+ t   cos x 1+ cos x 
70. I=∫ ⋅ 2 dx Therefore,
( x + 1) x
x  1 2   x
I = ∫ − dx = ∫  sec x − sec2  dx
x +1 1  cos x 1+ cos x   2
Now put t = = 1+ , then
x x  x
= log (sec x + tan x) − 2 tan  
1  2
dt = − dx
x2 77. Put (1 − x) = t.
Therefore,   Differentiating above equation, we have
2
1 1 1   x + 1  − dx = dt
I = − ∫ log t ⋅ dt = − (log t )2 + c = − log   +c
t 2 2   x     Now,

71. I = ∫ e sin x cos x dx ∫ −dt t


23
(1− t ) = ∫ (t 24 − t 23 ) dt

As,
=
t 25 t 24
− +c=
(1− x ) 25

(1− x )24 + c
1 – sin 2x = (sin x − cos x)2 25 24 25 24
Hence, 78. Put log x = t. Then
∫e
sin x
cos xdx dx = et dt ⇒ x = et
Put sin x = t, then cos x dx = dt.   Now,

∫ e dt = e
t t
+ c = e sinx +c I = ∫ et (sin t + cos t ) dt = et sin t = x sin (log x) + c
72. Multiplying above and below by x2 and put
dx sec 6 x (1+ tan2 x )2 sec2 x dx
1 − x3 = t2 ⇒ − 3x2dx = 2t dt 79. I = ∫ =∫ dx = ∫
6
cos x + sin x 6 6
1+ tan x 1+ tan6 x
Therefore,
2 dt 2 1 t −1 If tan x = p, then sec2 x dx = dp.
3 ∫ t2 −1 3 2
I = = . log +c  1
t +1 p 2  1+ 2 
2 2 2
1 t −1 (1+ p ) dp (1+ p )  p 
I = log +c I= 6
=∫ 4 2 dp = ∫ dp
3 t +1 1+ p p − p +1  1 
p2  p2 + 2 − 1
Therefore,  p 
1 æ
a = 1 ç 1 ö÷
3 If p − = k , then ç 1+ 2 ÷÷÷ dp = dk.
73. Put ax = t. Then p è p ÷ø
Therefore,
t
aa ⋅ at dt
I=∫
log a dk
= tan−1(k ) + c I=∫ 2
Again put at = z, then k +1
ax  1
a z dz az aa = tan−1  p −  + c
I= ò ( log a ) 2 = ( log a ) 3 + c =
(log a)3
+c  p
= tan−1 (tan x − cot x ) + c
log5 x log5 e
74. e =x (By the property of exponential function) 3 + 2 cos x
80. I = ∫ dx
x log5 e
+1 x log5 5e (2 + 3 cos x )2
∫e dx = ∫ x
log5 x log5 e
dx = =
log5 e + 1 log5 5e Multiplying numerator and denominator by cosec2x, we get
75. x 2
sin xdx   x 2 cos x 2 x .cos dx
I=∫
(3 cosec2 x + 2 cot x cosec x )
dx
  x 2 cos x  2( x sin x sin x dx ) (2cosec x + 3 cot x )2

  x 2 cos x  2 x sin x  2 cos x − 3cosec 2 x − 2 cot x cosec x


= −∫ dx
76. Here, we have cos x but its differential coefficient, that is (2 cosec x + 3 cot x )2
−sin x, is not present in the numerator and as such we cannot
make the substitution of cos x = t, but we simply put cos x = t 1  sin x 
= = +c
to split the integrand into the partial fractions. 2 cosec x + 3 cot x  2 + 3 cos x 

Mathematical Problem Book for JEE.indb 1016 07-06-2018 14:24:35


Chapter 22 | Indefinite Integration 1017

81. Substituting x = p6, dx = 6 p5 dp, we get Practice Exercise 2


5 6 4 5 3
6 p ( p + p + p) 6 ( p + p + 1) 1. Im, n = ∫ cos m x cos nx dx
I=∫ 6 2
dp = ∫ 2
dp
p (1+ p ) ( p + 1)
cos m x sin nx m
 6  = + ∫ cos m −1 x sin x sin nx dx
= ∫ 6 p3 dp + ∫  2  dp n n
 p + 1 cos m x sin nx m
= + ∫ cos m −1 x (cos(n − 1) x − cos nx cos x ) dx
2  1 n n
6 p4 3
= + 6 tan−1 p = x 3 + 6 tan−1  x 6  + c cos m x sin nx m m
4 2   Im, n = + ∫ cosm −1 x cos(n − 1) xdx − Im , n
n n n
82. ex = t ⇒ ex dx = dt m+n cos m x sin nx m
   ⇒ Im , n = + Im −1, n −1 + c1
  Now, n n n
dt or (m + n) Im, n = mIm – 1, n – 1 + cosmx sin nx + c

t 1− t 2
sin4 x
  Put sin–1 t = z, we get 2. ∫ x
dx = f ( x )

1 dz z sin4 ( p + q ) x f (( p + q ) x )
dt = dz = ∫
sin z ∫
   = cosec z dz = ln tan + k
1− t 2 2 ∫ ( p + q) x = p + q
 sin−1 e x 
= ln tan  sin4 ( p + q ) dx
 +k ∫ = f (( p + q ) x )
 2  x
x
83. elog  = x x x n −1
1+ ++
1! (n − 1)!
cos2 x 3. I = n ! ∫ dx − n ! ∫ dx
  I = ∫ x cos x dx + ∫ sin x cos x dx = x sin x + cos x − +c x xn
2 1+ ++
1! n!
x + sin x 1 x
84.   I = ∫ dx = ∫ sec2 ( x + sin x )dx  ex 
1+ cos x 2 2    ⇒ I = n !ln  +c
2 n
1 x 2 x  1+ x + x +  + x 
  I = ∫ x sec2 + ∫ tan dx  1! 2 ! n! 
2 2 2 2
1 tan x / 2 1 tan x / 2
  = x − ∫ + ∫ tan x / 2 dx = x tan x/2 + c x 2 dx x 2 ( 1+ x 2 − 1) dx
2 1/ 2 2 1/ 2 4. f (x) = ∫ =∫
(1+ x 2 )(1+ 1+ x 2 ) (1+ x 2 ) x 2
1 1
85. I =∫ dx = ∫ dx dx dx
f (x) = ∫
3
4 cos x − 3 cos x cos 3 x −∫ = ln x + 1+ x 2 − tan−1 x + c
1+ x 2 1+ x 2
1
= ∫ sec3x dx = ln|sec 3x + tan 3x| + c f(0) = 0 = c
3
p
f (1) = ln 1+ 2 −
86. I = ∫ sin(ln x ) dx 4
1 2 x dx 1
Let ln x = t. Then g( x) =
2∫
= ln x + 1+ x 4 + c
2
x = et ⇒ dx = et dt 1+ x 4 2
Therefore, g (0) = 0 = c
I = ∫ et ⋅ sin t dt 1
g (1) = ln 1+ 2
2
= sin t⋅et − ∫ cost ⋅ et dt
4 (3 x10 + 2 x 8 − 2)
= sin t⋅et − cos t ⋅ e − ∫ sin t ⋅ e dt
t t
5. I = ∫ x . x 6 + x 4 + x −4 dx
x6
2I = et (sin t − cos t) 4
= ∫ x 6 + x 4 + x −4 (3 x 5 + 2 x 3 − 2 x −5 ) dx
Therefore,
1
I = et(sin t − cos t) Put x 6 + x 4 + xx−64 += xt .4Then
+ x −4 = t . Then
2
1 x 2 (3 x 5 + 2 x23 (−32x 5x −+52) dx − 2dtx −5 ) dx = dt
x3 =
= e ln [sin (ln x) − cos (ln x)] + c
2
x 1 4 2 (t )5 4 2 ( x10 + x 8 + 1)5 4
= [sin (ln x) − cos (ln x)] + c I=
2 ∫ t dt =
5
+c =
5x5
+c
2

Mathematical Problem Book for JEE.indb 1017 07-06-2018 14:25:16


1018 Mathematics Problem Book for JEE

11. f ′(x) = b2 + (a – 1)b + 2 – sin2x – cos4x


∫(x + x 4 ) 2 x 4 + 3 x 2 dx = ∫ ( x 5 + x 3 ) 2 x 6 + 3 x 4 dx
6
6.
f ′(x) = b2 + (a – 1)b + 2 – 1
3 For increasing function, f ′(x) > 0, so
1 6
= x ( 2 x 2 + 3) 2 + c D<0
18
⇒ (a − 1)2 − 4 < 0
x4 −1 ( x + 1) x 4 − ( x 5 + 1) ⇒  a ∈ (−1, 3)
7. I=∫ 5
dx = ∫ dx
( x + 1)( x + 1) ( x 5 + 1)( x + 1)
12. I1 = ∫ x a2 − x 2 dx
1 5x 4
1  ( x 5 + 1)1 5 
I= ∫ 5 dx − ∫ dx = ln   +c Put a2 − x 2 = t 2 ⇒ xdx = −tdt
5 ( x + 1) ( x + 1)  ( x + 1) 
Therefore,
8. f (x) = e x sin x + cos x t3 ( a2 − x 2 ) 3 / 2
I1 =− ∫ t ⋅ t ⋅ dt = − + c= − +c
f ‘ (x) = ex cos x + sin x ex − sin x 3 3
g (x) = ex cos x − sin x
g ‘ (x) = cos x · ex − ex sin x − cos x 13. In = ∫ x n a2 − x 2 dx = ∫ x n−1 ( x a2 − x 2 ) dx
 Now
 1  n − 1 n −2 2
 = x n −1  − (a2 − x 2 )3 / 2  +
3 ∫
x (a − x 2 ) a2 − x 2 dx
  f (x) · g ‘ (x) = (ex sin x + cos x)(cos x · ex − ex sin x − cos x)  3 
  = e2x sin x cos x − e2x sin2x − ex sin x cos x + ex cos2x − ex sin x 1 n −1 2 n −1
  cos x − cos2x = − x n −1 (a2 − x 2 )3 / 2 + a In − 2 − In
3 3 3
  and 2
 n − 1 1 n −1 2 2 3 / 2 a ( n − 1)
  g (x) · f ‘ (x) = (ex cos x – sin x)(ex cos x + sin x ex – sin x)    ⇒  1+  In = − x (a − x ) + In − 2
 3 3 3
   = e2x cos2x + e2x sin x cos x – ex sin x cos x – ex sin x cos x – ex
sin2x + sin2x − x n −1 (a2 − x 2 )3 / 2 a2 (n − 1)
   ⇒ In = + In − 2
g (x) · f ‘ (x) − f (x) · g ‘ (x) = e2x − ex + 1 n+2 n+2

f ( x )g ’( x ) − g( x )f ’( x ) f (x) 1
I = ∫ dx = ln +c 14. (A) Put f101(x) = t. Then dx = dt .
f ( x )g( x ) g( x ) xf1 ( x )f2 ( x )f100 ( x )
 e x cos x − sin x  Therefore, given integral = f101(x) + c.
   = ln  x  +c
 e sin x + cos x  x + sin x x x x x
(B) f ( x ) = ∫ dx = ∫  sec2 + tan  dx = x tan + c
1+ cos x  2 2 2  2
9. Putting sinq = t, we get
∫ ( 4t − 4t
3
(
) et = At 3 + B (1− t 2 ) + Ct + D 1− t 2 + E et + F ) (C) f ( x ) = ∫ e sin
−1
x

 1−

x 
 dx
1− x 2 
  It follows immediately that D = 0.
  Differentiating both sides w.r.t. t, we get −1
 1− x 2 x 
= ∫ e sin x
 −  dx
 (4t − 4t3) et = [At3 + (3A – B)t2 + (C − 2B)t + C + B + E ]e t  1− x 2 1− x 2 
  And hence, A = – 4, B = – 12, C = – 20. −1
f ( x ) = e sin x
1 − x 2 + c ⇒ f ( 0 ) = 1+ c ⇒ c = 0
1 1
10. F(x) = ∫ dx = ∫ dx  1 3 ep 6
4 − 3 cos2 x + 5 sin2 x 9 − 8 cos2 x f =
 2 2
sec2 x sec2 x 1
= ∫ 2
dx = ∫ dx = tan−1 (3 tan x ) + c  tan x 
9 sec x − 8 1+ 9 tan2 x 3 (D) f ( x ) = ∫  dx
 sin x ⋅ cos x 
⇒ g(x) = 3 tanx

Therefore, = ∫ (tan x )−1 2 sec2 x dx = 2 tan x + c
p 
g  = 3 f (0) = c = 0 ⇒ c = 0
 4
p  p  2k
and g′   =12 f  =2= ⇒k =p
 3  4 p

Mathematical Problem Book for JEE.indb 1018 07-06-2018 14:25:46


Chapter 22 | Indefinite Integration 1019

Solved JEE 2017 Questions


JEE Main 2017 = ∫ tan4 x (1+ tan2 x )dx

1. Let In = ∫ tann x dx , (n > 1). If I4 + I6 = a tan5x + bx5 + C, where = ∫ tan4 x ⋅ sec2 xdx
C is a constant of integration, then the ordered pair (a, b) is
equal to: Substituting t = tanx, we get
1  1  t5
(A)
 , 0 (B)  , −1 I4 + I6 = ∫ t 4 ⋅ dt = +c
5 5 
5
 1   1  That is,
(C)  − , 0 (D)  − ,1
 5 
5 tan5 x
I 4 + I6 = +c
(OFFLINE) 5
Solution: We have
1
    In = ∫ tann x dx On comparison, we get a = , b = 0.
5
I4 + I6 = ∫ (tan4 x + tan6 x )dx Hence, the correct answer is option (A).

Mathematical Problem Book for JEE.indb 1019 07-06-2018 14:25:57


Mathematical Problem Book for JEE.indb 1020 07-06-2018 14:25:57
23 Definite Integration

23.1 Definition Illustration 23.1  Evaluate ò


p
sin2 x dx .
0
If f(x) is a continuous function on [a, b] and F(x) is any anti-derivative Solution:
of f(x) on [a, b], that is,
1 p 1 p
d
(F(x)) = f(x) " x Î(a,b),
I=
2 ò0
2sin2 x dx = ò [1- cos2 x ] dx
2 0
dx p
1é sin2 x ù
then Þ I = êx - ú
2ë 2 û0
b

ò f(x)dx = F(b) - F(a)


a
ÞI =
1 p
[p ] = 2
2
is called definite integration with limits a and b, where a is called loge x e2
the lower limit and b is called the upper limit of the integral. This Illustration 23.2   Evaluate
x ∫ dx .
e −1
formula is known as Newton-Leibnitz formula.
Solution: Put loge x = t , then e t = x .
Therefore, dx = e t dt and limits are adjusted as –1 to 2.
23.2 Geometrical Meaning of Definite 2 t 2
I = ∫ t e t d t = ∫ | t | dt
Integration −1 e −1

0 2
b ⇒ I = ∫ −t dt + ∫ t dt
If f(x) > 0 for all x ∈ [a, b]; then òa f(x) is numerically equal to the −1 0

0 2
area bounded by the curve y = f(x), x-axis and the straight lines x  −t 2  t2  5
⇒ I=  +   ⇒I =
= a and x = b.  2  −1  2  0 2

p /2 dx
x =a f(x) x =b Illustration 23.3  Evaluate ∫
0 1+ sin x
.
S Solution:
L C
p /2 dx
I=∫
0 sin2 x /2 + cos2x /2 + 2sin x /2cos x /2
+ +
p /2 dx p /2 sec2x /2
I=∫ =∫ dx
   
0 (sin x /2 + cos x /2) 2 0 (1+ tan x /2)2
A − − B
O M D Put (1+ tan x / 2) = t . Then
Q R 1 2
sec x /2 dx = dt
2
2
2 dt  1  1 1
Figure 23.1 I = 2 ∫ 2 = −2   = −2  −  = 1
1 t
 t 1  2 1
b
In general, ∫a f(x)dx represents algebraic sum of the figures ∞ ∞
dx x 2dx
bounded by the curve (Fig. 23.1) y = f(x), the x-axis and the straight
Illustration 23.4  Let a = ∫
x4 + 7x2 +1
and b = ∫
0
x + 7x2 +1
4
.
0
line x = a and x = b. The areas above x-axis are taken with plus sign Then show that a = b.
and the areas below the x-axis are taken with minus sign.
That is, Solution:

dx
b
a =∫

a
f(x)dx  = area OLA − area AQM – area MRB + area BSCD
0
x4 + 7x2 +1

Chapter 23.indd 1021 13-06-2018 17:44:50


1022 Mathematics Problem Book for JEE

Put x = 1/t, then  2



dx = –1/t2  nh(nh − h) 
(nh)a + 
3
 
 2 
1 = lim  
h→0 
0 dt −∞
dt t 2

t 2dt  nh(nh − h) nh(nh − h)(2nh − h)  
a =∫ t 2
=∫ 4 =∫ 4 =b 3a a + 
1 7   2 6  
+ 2 + 1 0 t + 7t + 1 0 t + 7t + 1
2 2

4
t t 2
 (b − a)2   (b − a)2 2(b − a)3 
= (b − a)a3 +   + 3a a + 
23.3 Definite Integration as the  2   2 6 

Limit of Sum =
b−a 3 3
(a + b + ab 2 + a2b )
4
Let f(x) be a single valued-continuous function defined in the
interval a ≤ x ≤ b , where a and b are both finite. Let this interval b 4 − a4
be divided into n equal sub-intervals, each of width h by inserting =
4
(n – 1) points a + h, a + 2h, a + 3h, ¼ , a + (n - 1)h between a and b.
Then b

nh = b - a
Illustration 23.6   Evaluate ∫a
sin x dx by first principle.
Solution:
Now, we form the sum
b
hf (a) + hf (a + h) + hf (a + 2h) + … + hf (a + rh) + … + hf [a + (n − 1)h] I = ò sin x dx
a
n-1
= h[f (a) + f (a + h) + f (a + 2h) + ¼¼ + f (a + rh) + ¼¼ + f {a + (n - 1)h}] I = lim hå sin(a + rh)
n−1 h®0
= h∑ f (a + rh),
r =0

r =0 = lim h{sin a + sin(a + h) + sin(a + 2h) +  + sin(a + (n - 1)h)}


h®0
where nh æ (n - 1)h ö æ h ö
2sin sin ç a + ÷ç ÷
a + nh = b ⇒ nh = b − a 2 è 2 ø è 2ø
= lim
sin h
n−1 h®0
The lim h∑ f (a + rh) , if it exists, is called the definite integral of 2
h→0
r =0
f(x) with respect to x between the limits a and b, and we denote it æ b + aö æ b - aö
= 2sin ç sin ç
b è 2 ÷ø è 2 ÷ø
by the symbol ∫ a
f ( x ) dx. Thus,
= cos a - cos b
b
∫ f ( x ) dx = lim h[ f (a) + f (a + h) + f (a + 2h) + … + f {a + (n − 1)h}]
23.4  Properties of Definite Integration
a h→0
b n−1
⇒ ∫ f ( x ) dx = lim h∑ f (a + rh),
a h→0
r =0 1.  Change of variable of integration is immaterial so long as limits
where nh = b - a, a and b being the limits of integration. of integration remain the same, that is,
The process of evaluating a definite integral by using the above b b

definition is called integration from the first principle or integration ò a


f ( x )dx = ò f (t )dt
a

as the limit of a sum. 1

b
Illustration 23.7   Evaluate ò ln( x + 1)dx .
0

Illustration 23.5   Evaluate ∫x


3
dx by first principle. Solution:
a
Solution: 1
I = ò ln( x + 1)dx = ( x + 1)ln( x + 1) - ( x + 1)|10
b 0
I = ∫ x 3 dx ⇒ I = 2ln2 − 1
a
n−1
I = lim h∑ (a + rh)3 1 1
h→0
r =0
Illustration 23.8   Evaluate ò0 x +1
dx .

I = lim h{(a)3 + (a + h)3 + (a + 2h)3 + … + (a + (n − 1)h)3 } Solution:


h→0

na3 + (13 + 23 + ⋅⋅⋅ + (n − 1)3 )h3 + 3ah((a + h) 


1 1
I=∫ dx
= lim h   0 x +1
h→0
+2(a + 2h) + 3(a + 3h) + ⋅⋅⋅ + (n − 1)(a + (n − 1)h))
I = ln( x + 1)|10 = ln2
éna3 + (13 + 23 + ××× + (n - 1)3 )h3+ 3ah {a(1+ 2 + ù
= lim h ê 2 ú
b a
êë3 + ××× + (n - 1)) + h(1 + 2 + 3 + ¼ + (n - 1) )} úû
h®0 2 2 2
2.  ∫ f ( x )dx = −∫ f ( x )dx
 2
 n(n − 1)  4  n(n − 1)h2 n(n − 1)(2n − 1)h3  a b
= lim nha3 +   h + 3a a +  That is, by the interchange in the limits of definite integral, the
h→0
  2   2 6 
sign of the integral is changed.

Mathematical Problem Book for JEE.indb 1022 07-06-2018 14:26:00


Chapter 23 | Definite Integration 1023

b c b 2

3.  ò f ( x )dx = ò f ( x )dx + ò f ( x )dx ,


Illustration 23.10   Evaluate ∫−2
|1− x 2 | dx
a a c
Solution:
where ( a < c < b) 2 −1 1 2

Generally, we break the limit first at the points where f(x) is dis- I = ∫ |1− x 2 | dx = ∫ |1− x 2 | dx + ∫ |1− x 2 | dx + ∫ |1− x 2 | dx
−2 −2 −1 1
continuous and second at the points where definition of f(x) −1 1 2

changes. ⇒ I = − ∫ (1− x 2 )dx + ∫ (1− x 2 )dx − ∫ (1− x 2 )dx


−2 −1 1

Or 4 4 4
b c1 c2 b ⇒I = + + =4
ò a
f ( x )dx = ò f ( x )dx + ò
a c1
f ( x )dx + ¼ + ò f ( x )dx where
c n
3 3 3

(a < c1 < c2 < …c n < b ) 1.5


Illustration 23.11   Evaluate ∫ 0
[ x 2 ]dx , where [.] denotes the
Generally, this property is used when the integrand has two or
greatest integer function.
more rules in the integration interval.
Proof:  Let ∫ f ( x )dx = A( x ) + k Solution:
c b 1.5 1 2 1.5
I = ∫ [ x 2 ]dx = ∫ [ x 2 ]dx + ∫ [ x 2 ]dx + ∫ [ x 2 ]dx
∫ f ( x ) + ∫ f ( x ) = ( A( x ) + k ) | +( A( x ) + k )|bc
c
a 0 0 1 2
a c
2 1.5
= A(c ) − A(a) + A(b ) − A(c ) ⇒ I = 0 + ∫ 1dx + ∫ 2 dx = 2 − 1+ 3 − 2 2
1 2
= A(b ) − A(a)
⇒I =2− 2
5p
12 2p
| cos x |
Illustration 23.9   Evaluate ∫ [tan x ]dx ,
0
where [.] is the greatest Illustration 23.12   Evaluate ∫ cos x
dx
integer function. 0

Solution: Let
Solution: Let 2p
| cos x |
5p
12
I= ∫ cos x
dx
I= ∫ [tan x ]dx
0

p 3p
0
2 2 2p
5p cos x cos x cos x
Value of tan x at x = is 2 + 3. =∫ dx − ∫ cos x dx + ∫ cos x dx
12 0 cos x p 3p
2 2
Value of tan x at x = 0 is 0. p 3p
2 2 2p
Integers between 0 and 2 + 3 are 1, 2 and 3. Therefore, = ∫ 1dx − ∫ 1dx +
tan x = 1, 0 p
∫ 1dx
3p
2 2
tan x = 2,
p p
and = −p + = 0
2 2
tan x = 3
⇒ x = tan−1 1, p /2

x = tan−1 2, Illustration 23.13   Evaluate ∫


− p /2
cos x − cos3 x dx
and
Solution:
x = tan 3 −1

p /2 p /2
Therefore,
tan−1 1 tan−1 2 tan−1 3
5p I= ∫
− p /2
cos x − cos3 x dx = ∫
− p /2
cos x (1− cos2 x ) dx
12
I= ∫ [tan x ]dx + ∫ [tan x ] dx + ∫ [tan x ]dx + ∫ [tan x ]dx p /2 0 p /2
0 tan−1 1 tan−1 2
5p
tan−1 3 I= ò
- p /2
| sin| cos x dx = − ∫ sin x cos x dx + ∫ sin x cos x dx
− p /2 0
tan−1 1 tan−1 2 tan−1 3 12
= ∫ 0 dx + ∫ 1dx + ∫ 2 dx + ∫ 3 dx Put cos x = z , thensin x dx = − dz
0 tan−1 1 tan−1 2 tan−1 3
1 0 1
æ b a
ö
 5p I = ò z dz - ò z dz = 2 ò z dz çç As ò f ( x ) = - ò f ( x ) ÷÷

= 0 + (tan−1 2 − tan−1 1) + 2(tan−1 3 − tan−1 2) + 3  − tan−1 3
 12  0 1 0 è a b ø
5p p 2 4
1
= − − tan−1 3 − tan−1 2 I = 2 × z 3/2 =
4 4 3 3
0
  3+2  
= p −  tan−1   + p  = - tan ( −1)
−1

  1 − 6   b b a a

   = p  4.  ∫ f ( x )dx = ∫ f (a + b − x )dx . In particular ∫ f ( x )dx = ∫ f (a − x )dx


a a 0 0
4

Chapter 23.indd 1023 11-06-2018 10:18:13


1024 Mathematics Problem Book for JEE

Proof:  Let p /4  2 
⇒I =∫ ln  dx
b
0  1+ tan x 
I = ∫ f (a + b − x )dx
p /4 p /4
⇒I =∫ ln2 dx − ∫
a
ln(1+ tan x )dx
a + b − x = z ⇒ dx = − dz 0 0

x = a ⇒ z = b, x = b ⇒ z = a p p
Þ 2I = ln2 Þ I = ln2
a b 4 8
I = − ∫ f ( z )dz ⇒ I = ∫ f ( z )dz p /2
a sin x + b cos x
b
b a
Illustration 23.17   Evaluate ∫
0
sin x + cos x
dx .
I = ∫ f ( x )dx Solution:
a
p /2
a sin x + b cos x
(A)  ∫
p /2

0
f (sin2 x )sin x dx = ∫
0
p /2
f (sin2 x )cos x dx I= ò
0
sin x + cos x
dx

p /2 p /2
p  p 
(B)  ∫0
f (sin x )dx = ∫
0
f (cos x )dx p /2 a sin  − x  + b cos  − x 
2  2  p /2
a cos x + b sin x
p /2 p /2 I= ∫ p  p 
dx ⇒ I = ∫
sin x + cos x
dx
(C)  ∫0
f (tan x )dx = ∫
0
f (cot x )dx 0 sin  − x  + cos  − x 
2  2 
0

1 1
(D)  ∫ f (ln x ) dx = ∫ f [ln(1− x )] dx
0 0 2I = ∫
p /2
(a + b )
cos x + sin x p
dx ⇒ 2I = (a + b )
0 cos x + sin x 2
p /2 sinn x
Illustration 23.14   Evaluate ∫ 0 sin x + cosn x
n
dx .
I=
p
(a + b )
4
Solution:
p /2 sinn x 1
I=∫ dx Illustration 23.18   Evaluate ∫ ( x − 1)(1− x )99 dx .
0 sin x + cosn x
n
0
p /2 cosn x Solution:
I=∫ dx 1
0 sin x + cosn x
n
I = ò ( x - 1)(1- x )99 dx
p p 0
2I = ⇒I =
2 4 1 1
I = ∫ ( − x )( x )99 dx ⇒ I = − ∫ x 100 dx
Similarly we can solve these examples: 0 0

101 1
tann x
p /2 p /2 cot x
n
p x 1
I=− =−
(A)  ∫ 1+ tan x
0 n
dx = ∫
0 1+ cot n x
dx =
4 101 0 101
p /2 1 p /2 1 p p
ò
2

(B)  ∫0 dx = ∫ dx = Illustration 23.19   Evaluate e sin x cos3 x dx .


1+ tann x 0 1+ cot n x 4 0

Solution:
p /2 sec n x p /2 cosec n x p
(C)  ∫ sec x + cosec x
n n
dx = ∫
cosec n x + sec n x
dx =
4
p

I = ∫ e sin x cos3 x dx
0 0 2

p /2 0
Illustration 23.15   Evaluate ∫ 0
ln(tan x )dx . p

I = ∫ e sin
2
(p − x )
cos3 (p − x ) dx
Solution:
0
p /2
I=∫ ln(tan x )dx p

I = − ∫ e sin x cos3 x dx ⇒ I = − I ⇒ I = 0
2
0
p /2
I=∫ ln(cot x )dx 0
0
2a f (x)
p /2 p /2
Illustration 23.20   Evaluate ∫ dx .
2I = ∫ (ln(tan x ) + ln(cot x ))dx = ∫ ln ( tan x ⋅ cotx ) dx 0 f ( x ) + f (2a − x )
0 0
Solution:
2I = 0 ⇒ I = 0
2a f (x) 2a f (2a - x )
I=ò dx = ò dx
p /4 0 f ( x ) + f (2a - x ) 0 f (2a - x ) + f ( x )
Illustration 23.16   Evaluate ∫ 0
ln(1+ tan x )dx .
2a f ( x ) + f (2a − x ) 2a
Solution: 2I = ∫ dx = ∫ dx = [ x ]20a = 2a
0 f ( x ) + f (2a − x ) 0
p /4
I=∫ ln(1+ tan x )dx ÞI =a
0

Chapter 23.indd 1024 11-06-2018 10:18:42


Chapter 23 | Definite Integration 1025

p /2 tan x Proof:  Let


Illustration 23.21   Evaluate ò
0 1+ tan x
dx. 2a a 2a
I = ∫ f ( x )dx = ∫ f ( x )dx + ∫ f ( x )dx
Solution: 0 0 a

p /2 tan x p /2 sin x I = I1 + I2
I=ò dx Þ I = ò dx 2a
0 1+ tan x 0 cos x + sin x
I2 = ò f ( x )dx
a
p 
sin  − x 
p /2 2  p /2 cos x Put
I=∫ dx ⇒ I = ∫ dx
0
p  p 
0 cos x + sin x x = 2a − t ⇒ dx = −dt
cos  − x  + sin  − x 
2  2  x = a ⇒ t = a , x = 2a ⇒ t = 0
0 a
p /2 p I2 = − ∫ f (2a − t )dt = ∫ f (2a − t )dt
2I = ∫ 1dx ⇒ I =
0 4 a 0
a
3p /4 dx I2 = ∫ f (2a − x )dx
Illustration 23.22   Evaluate ò
p /4 1+ cos x
.
0
a
Solution:
I = ∫ (f ( x ) + f (2a − x ))dx
3p /4 dx
I=∫
0
p /4 1+ cos x 2p
x × sin2 n x
  p 3p  
Illustration 23.25   Evaluate ò sin2 n x + cos2 n x
dx x > 0.
3p /4 1
dx cos  + − x  = − cos x  0
I=∫ 
p /4 1− cos x  4 4  Solution:
2p 2p
(2p - x )sin2 n (2p - x ) (2p - x )sin2 n x
2I = ò
3p /4 2
3p /4
2 I= ò dx = ò sin
dx = ò sin (2p - x ) + cos2 n (2p - x ) x + cos2 n x
2n 2n
dx 0 0
p /4 1- cos2 x p /4 sin2 x
2p
2p × sin2 n x
⇒ 2I = 2 ∫
3p /4
cosec2 x dx
Þ 2I = ò
0
sin2 n x + cos2 n x
dx
p /4
p
⇒ 2I = −2[cot x ]p3p/4/4 = 4 ⇒ I = 2 sin2 n x
Þ 2I = 4p ò dx
0
sin x + cos2 n x
2n

3 x p /2
sin2 n x
Illustration 23.23   Evaluate ∫
2
5− x + x
dx. Þ I = 4p ò
0
sin x + cos2 n x
2n

Solution:
3 x p /2
cos2 n x æ a a
ö
I=∫ dx
2
5− x + x
Þ I = 4p ò sin2 n x + cos2 n x
dx çç As ò f ( x )2 ò f (a - x ) ÷÷
0 è 0 0 ø
3 5− x
I=∫ dx p /2
2
5− x + x Þ 2I = 4 ò p dx = 2p 2 Þ I = p 2
3 0
2I = ∫ 1dx ⇒ 2I = [ x ]32 = 1 p
2
Illustration 23.26   Evaluate ò x lnsin x dx.
⇒ I = 1/ 2
0
Solution:
1
æ1 ö p
Illustration 23.24   Evaluate ò ln ç - 1÷ dx.
èx ø I = ò x lnsin x dx
0
0
Solution:
p p
1
 1− x  x
1
1− x
1
I = ∫ (p − x )lnsin(p − x )dx = ∫ (p − x )lnsin x dx
I = ∫ ln   dx = ∫ ln dx = − ∫ ln dx
0
 x 0
1 − x 0
x 0 0
p p /2

⇒I=−I⇒I=0 2I = p ò lnsin x dx = 2p ò lnsin x dx


0 0
2a a p /2 p /2
 5.  ∫ f ( x )dx = ∫ (f ( x ) + f (2a − x ))dx
0 0 I =p ò lnsin x dx = p ò lncos x dx
0 0
2a 0, if f (2a − x ) = − f ( x ) p /2 p /2
 sin2 x 
Special case:  ∫ f ( x )dx =  a 2I = p ∫ ln(sin⋅ cos x )dx = p ∫ ln   dx
2∫0 f ( x )dx , if f (2a − x ) = f ( x )
0
0 0 2 

Mathematical Problem Book for JEE.indb 1025 07-06-2018 14:26:05


1026 Mathematics Problem Book for JEE

p /2 p /2 4
x2
2I = p ò lnsin2 x dx - p ò ln2 dx
0 0
I= ∫x
−4
2
+ 16
dx

Put 2 x = t , then 2dx = dt . 4


x2 x 2 + 16 − 16
4

p I = 2∫ dx ⇒ I = 2 ∫ dx
p p2 x + 16
2
x 2 + 16
2I = ò lnsin t dt - ln2 0 0
20 2 16 x p
4
 
I = 2  x − tan−1  = 2  4 − 4 ⋅  = 8 − 2p
p /2
p2  4 
4 0  4
2I = p ò lnsin t dt -
0 2
ln2
p 2
dx æ 5- x ö
p2 p2
2I = I - ln2 Þ I = - ln2
Illustration 23.29   Evaluate ò 1+ 5
0
cos x
+ ò ln ç
-2
÷ dx .
è 5+ x ø
2 2
Solution: Let
p

p
Illustration 23.27   Evaluate
2
e cos x cos3 (2n + 1) x dx ,(n ∈I ). dx
0 I1 = ò 1+ 5
0
cos x (1)
Solution:
Now
p
I = ∫ e cos
2
(p − x )
⋅ cos3 (2n + 1)(p − x )dx p
dx dx
p
5cos x dx
p

∫0 1+ 5cos(p − x ) ∫0 1+ 5− cos x ∫0 5cos x + 1 


0
p I1 = = = (2)
I = - ò e cos x × cos3 (2n + 1) x dx Þ I = -I
2

⇒ 2I = 0 ⇒ I = 0 Adding Eqs. (1) and (2), we get


a a p p p
dx 5cos x dx
6.  ∫ f ( x ) dx = ∫ ((f ( x ) + f ( − x )) dx 2I1 = ∫ + ∫0 5cos x + 1 = ∫0 1⋅ dx = p
0 1+ 5
cos x
−a 0

Special case: I1 = p/2


 a Consider
∫0 f ( x )dx =  ∫0
2 f ( x )dx , if f ( x ) is even funciton f ( − x ) = f ( x ).
a

2
0,  5− x 
 if f ( x ) is odd function f ( − x ) = −f ( x ). I2 = ∫ log   dx
−2  5+ x 
Let
Proof:  Let
 5− x 
a 0 a g( x ) = log  
I = ∫ f ( x )dx = ∫ f ( x )dx + ∫ f ( x )dx  5+ x 
−a −a 0 Now
I = I1 + I2 æ 5 - (- x ) ö 5- x
g( - x ) = log ç ÷ = - log = - g( x )
0 è 5 + (- x ) ø 5+ x
I1 = ò f ( x )dx
-a
Therefore, g(x) is an odd function.
Therefore,
Put 2
x = −t ⇒ dx = −dt
x = -a Þ t = a , x = 0 Þ t = 0
∫ g( x )dx = 0 ⇒ I
−2
2 =0

0 a I = I1 + I2 = p/2 + 0 = p/2
I1 = - ò f ( -t )dt = ò f ( -t )dt
log2
a 0  e x − 1
a Illustration 23.30   Evaluate ∫
sin  x  dx.
 e + 1
I1 = ò f ( - x )dx log1/2

0 Solution:
a
 e − x − 1 ex −1
I = ∫ (f ( x ) + f ( − x )) dx f ( − x ) = sin  − x  = − sin x = −f ( x )
0
 e + 1 e +1
log2
æ e x - 1ö
4
x2 ò sin ç x ÷ dx = 0
Illustration 23.28   Evaluate ∫ (x
−4
2
+ 16)(1+ e x )
5
dx - log2 è e + 1ø
Solution: 1
x 3 + | x | +1
4
x2 Illustration 23.31   Evaluate òx 2
+ 2 | x | +1
dx.
I= ∫ (x dx
( )
5 -1
−4
2
+ 16) 1+ e x Solution:
4 ì
ïæ ö üï
5
x2 x 2e x 1
x3 | x | +1
1
I = ò íç 2 + ÷ dx I=ò dx + ò 2 dx
ç 5 5
÷ý + 2
+ + 2 | x | +1
ïè ( x + 16)(1+ e ) ( x + 16)(1+ e ) ø þï
x 2 x
-4 î -1
x 2 | x | 1 -1
x

Chapter 23.indd 1026 14-06-2018 11:42:51


Chapter 23 | Definite Integration 1027

1
x +1 1 1/2  1− x 
= 0 + 2∫
x + 2x +1
2
dx = ln( x 2 + 2 x + 1) 0 Illustration 23.35   Evaluate ∫ −1/2
cos x ⋅ ln   dx .
 1+ x 
0
Solution: Let
= ln 4
1/2  1− x 
p /2
I=∫ cos x ⋅ ln   dx
Illustration 23.32   If I1 = ∫
0
ln(sin x )dx and −1/2
 1+ x 
p /4
I1  1− x 
f ( x ) = cos x ⋅ ln 
I2 = ∫ ln(sin x + cos x ) dx, then find .  1+ x 
− p /4 I2
 1+ x 
f ( − x ) = cos x ⋅ ln  = −f ( x )
Solution:  1− x 
p /4
⇒ f ( x ) + f (− x ) = 0
I2 = ∫ ln(cos x + sin x )dx 1/2

∫ (f ( x ) + f ( − x ))dx =0
−p /4
⇒I =
Using 0

a a
b 1
∫ f ( x )dx = ∫ (f ( x )dx + f (− x )dx )
−a 0
7.  ∫ f ( x )dx = (b − a)∫ f ((b − a) x + a)dx
a 0
p /4 p /4 p /4
Proof:  Let
⇒ I2 = ∫ ln(cos x − sin x )dx = ∫ lncos2 x dx = ∫ lnsin2 x dx
2 2

0 0 0
1

I = (b − a)∫ f ((b − a) x + a)dx


Put 0
Put
2x = y ⇒ 2dx =dy
p /2 dt
1 t = (b − a) x + a ⇒ dx =
I2 =
2 ∫ lnsin y dy (b − a )
0
x = 0 ⇒ t = a , x = 1⇒ t = b
p /2

∫ lnsin x dx
b b
⇒ 2I 2 = f (t )
I = (b − a )∫ dt = ∫ f (t )dt
0
a ( b − a) a
I1 1 b
⇒ 2I2 = I1 ⇒ =
I2 2 I = ∫ f ( x )dx
a
2
−5 2/3  2

1/2  1+ x   9 x− 

∫−1/2  [ x ] + ln  1− x   dx. ∫ e dx + 3 ∫ e
2
( x +5)  3
Illustration 23.33   Evaluate Illustration 23.36   Evaluate dx .
−4 1/3

 1+ x  Solution:
Solution: ln   is an odd function of x as f ( − x ) = −f ( x ),
 1− x  I = I1 + I2
therefore
−5
I1 = ∫ e( x +5) dx
2

1/2
I=∫ [ x ]dx + 0 −4
1
−1/2
= ( −5 + 4)∫ e (
( −5+ 4 ) x −4 +5 )
2

0 1/2
dx
I=∫ [ x ]dx + ∫ [ x ]dx 0
−1/2 0
1
I1 = − ∫ e( x −1) dx
2
0 1  (1)
I = ∫ −1dx + 0 = −[ x ] −1/2 = −
0
0
−1/2 2
Again, let
p 2


 2
Illustration 23.34   Evaluate (1− x 2 )sin x cos2 x dx . 2/3 9 x − 
−p I2 = −3 ∫ e  3
dx
Solution: Let 1/3
2
p 1    2−1   2−1 
I = ∫ (1− x 2 )sin x cos2 x dx  2 − 1       x − 
3    3  
−p I2 = 3  ∫e dx
 3 0
f ( x ) = (1− x ) ⋅ sin x ⋅ cos x is an odd function as
2 2
1
= ∫ e( x −1) dx
2

f ( − x ) = −(1− x )sin x cos x = −f ( x )


2 2
0

p I2 = −I1
I = ∫ (f ( x ) + f ( − x ))dx = 0
0
⇒ I = I1 − I1 = 0

Chapter 23.indd 1027 14-06-2018 12:35:03


a a

I3 = ∫ f ( z + nT )dz = ∫ f ( z )dz
1028 Mathematics Problem Book for JEE 0
a
0

I3 = ∫ f ( x )dx = − I1
0

23.5 Properties Based on I3 + I1 = 0
Periodic Function nT T
I = ∫ f ( x )dx = n∫ f ( x )dx
If f ( x ) is a periodic function with period T (f ( x + T ) = f ( x )) and 0 0

m, n ∈I , a ∈R + , then nT T

a+nT a
11.  ∫mT
f ( x )dx = (n − m)∫ f ( x )dx
0
 8.  ∫nT f ( x )dx = ∫ f ( x )dx
0 Proof: Let
nT
Proof: Let
a+nT
I= ∫ f ( x )dx
I=∫ f ( x )dx mT
nT
Put Put
x = z + mT ⇒ dx = dz
x = z + nT ⇒ dx = dz x = nT ⇒ z = (n − m)T , x = mT ⇒ z = 0
x = nT ⇒ z = 0, x = a + nT ⇒ z = a ( n− m ) T ( n− m ) T
f ( z + nT ) = f ( z ) I= ∫
0
f ( z + mT ) dz = ∫
0
f ( z )dz
a
a
I = ∫ f ( z + nT )dz = ∫ f ( z )dz T
0
0
I = (n − m)∫ f ( x )dx
a 0
I = ∫ f ( x )dx
0
b +nT b
nT T 12.  ∫ f ( x )dx = ∫ f ( x )dx
 9.  ∫ 0
f ( x )dx = n∫ f ( x )dx
0
a+nT a

Proof: Let
Proof: Let b +nT
nT T 2T

I = ∫ f ( x )dx = ∫ f ( x )dx + ∫ f ( x )dx + … +


nT I= ∫ f ( x )dx
0 0 T

( n−1)T
f ( x )dx a+nT

T T T T
Put
I = ∫ f ( x )dx + ∫ f ( x + T )dx + ∫ f ( x + 2T )dx + … + ∫ f ( x + (n − 1)T )dx x = z + nT ⇒ dx = dz
0 0 0 0
T T T T x = a + nT ⇒ z = a, x = b + nT ⇒ z = b
= ∫ f ( x )dx + ∫ f ( x )dx + ∫ f ( x )dx + … + ∫ f ( x )dx (upto n times)
0 0 0 0 b b

f ( x + T ) = f ( x ), f ( x + 2T ) = f ( x ),… , f ( x + (n − 1)T ) = f ( x ) I = ∫ f ( z + nT )dz = ∫ f ( z )dz


a a
T b
I = n∫ f ( x )dx I = ∫ f ( x )dx
0 a

a+nT T a+T
10.  ∫ a
f ( x )dx = n∫ f ( x )dx
0
13. 
If f(x) is a periodic function with period T, then ∫
a
f ( x ) is

Proof:  Let independent of a.


Proof:  Let
a+nT 0 nT a+nT
a+T 0 T a+T
I= ∫
a
f ( x )dx = ∫ f ( x )dx + ∫ f ( x )dx +
a 0

nT
f ( x )dx
I= ∫ f ( x )dx = ∫ f ( x )dx + ∫ f ( x )dx + ∫ f ( x )dx
a a 0 T
I = I1 + I2 + I3
a+nT
I = I1 + I2 + I3
I3 = ∫
a+T
f ( x )dx
nT
I3 = ∫ f ( x )dx
T
Put
Put
x = z + nT ⇒ dx = dz x = z + T ⇒ dx = dz
x = nT ⇒ z = 0, x = a + nT ⇒ z = a x = T ⇒ z = 0, x = a + T ⇒ z = a
f ( z + nT ) = f ( z ) f (z + T ) = f (z)
a a a a
I3 = ∫ f ( z + nT )dz = ∫ f ( z )dz I3 = ∫ f ( z + T )dz = ∫ f ( z )dz
0 0 0 0
a a
I3 = ∫ f ( x )dx = − I1 I3 = ∫ f ( x )dx = − I1
0 0
I3 + I1 = 0 I3 + I1 = 0

Chapter 23.indd 1028 11-06-2018 10:20:13


a a

I3 = ò f ( z + T )dz = ò f ( z )dz
0
a
0
Chapter 23 | Definite Integration 1029
I3 = ò f ( x )dx = - I1
0

I3 + I1 = 0 2
px 2 × 50  px
2

= 100 ∫ sin dx =  − cos 


T T
2 p  2 0
I = ∫ f ( x )dx = ∫ f ( x )dx
0

0 0 100 200
= (1+ 1) =
10p p p
Illustration 23.37   Evaluate ∫ | sin x | dx.
0
Your Turn 1
Solution: Let
3 x +1
10p  1.  The value of ò dx is
ò | sin x | dx x ( x - 1)
2
I= 2

0 1 16 1
We know that |sin x| is a periodic function with period p  (A)  2 log2 − (B)  log −
6 9 6
p p /2 4 1 16 1
I = 10 ∫ | sin x | dx ⇒ I = 20 ∫ sin x dx  (C)  log − (D)  log +
3 6 9 6
0 0 Ans. (B)
p /2
I = −20 cos x 0 ⇒ I = 20 e
 2.  The value of ∫1
log x dx is
Illustration 23.38   If f(x) is a continuous periodic function (A)  0 (B) 1 (C)  e −1  (D)  e +1
a+T
with period T, then prove that the integral of I = ∫ f ( x )dx is  Ans. (B)
a

dependent of a. p /2 (sin x + cos x )2


 3.  The value of I = ∫ dx is
Solution: Consider the function 0
1+ sin2 x
a+T 0 T a+T
g(a) = ∫ f ( x )dx = ∫ f ( x )dx + ∫ f ( x )dx + ∫ f ( x )dx (A)  3 (B) 1 (C) 2 (D) 0
a a 0 T
 Ans. (C)
Putting x − T = y in last integral, we get
p /8


a +T a
f ( x )dx = ∫ f ( y + T )dy = ∫ f ( y )dy
a
 4.  ∫0
cos3 4q dq =
T 0 0
2 1 1 1
0 T
⇒ g(a) = ∫ f ( x )dx + ∫ f ( x )dx + ∫ f ( x )dx = ∫ f ( x )dx
a T (A) 
(B)  (C)   (D) 
a 0 0 0 3 4 3 6
Hence, g(a) is independent of a.  Ans. (D)
8 2 - 3x
Illustration 23.39   If f(x) is a function satisfying f(x + a) + f(x) = 0
 5.  ò3
x (1+ x )
dx is equal to
c +b
(A)  2 log(3/2e 3 ) (B)  log(3 / e 3 )
for all x ⇒ R and constant a, such that ∫ f ( x )dx is independent of
b
(C)  4 log(3 / e 3 ) (D)  None of these
b, then find the least positive value of c.
Ans. (A)
Solution: We have   6.  The value of
1
∫ x e dx is equal to
2 x
0
f(x + a) + f(x) = 0 for all x ∈ R (1)
(A)  e - 2 (B)  e + 2 (C)  e 2 − 2  (D)  e 2
⇒ f(x + a + a) + f(x + a) = 0 [Replacing x by x + a](2)
 Ans. (A)
⇒ f(x + 2a) + f(x+ a) = 0 (3)
dx 2 dx
and I2 = ∫1
2
Subtracting Eqs. (1) from (2), we get  7. Let I1 = ∫ . Then
1+ x
1 2 x
f(x + 2a) − f(x) = 0 for all x ∈ R
⇒ f(x + 2a) = f(x) for all x ∈ R (A)  I1 > I2 (B)  I2 > I1
So, f(x) is periodic with period 2a. (C)  I1 = I2 (D)  I1 > 2I2  Ans. (B)
c +b
t dt dt
It is given that ∫ f ( x )dx is independent of b. tan x cot x

b
 8.  The value of ò1/ e 1+ t 2 ò1/e t (1+ t 2 )
+ =

The minimum value of ‘c’ is equal to the period of f(x), that is, 2a. (A)  −1 (B) 1 (C) 0 (D) None of these
200
Ans. (B)
1− cos p x
Illustration 23.40   Evaluate ∫ dx . 3p /4
dx
0
2   9.  ∫
p /4 1+ cos x
is equal to
Solution:
1 1
200
px  (D)  −
(A)  2 (B)  -2 (C) 
2 2
I= ∫ sin
2
dx
0  Ans. (A)

Mathematical Problem Book for JEE.indb 1029 07-06-2018 14:26:12


1030 Mathematics Problem Book for JEE

dx e2 ( x 2 - 9) + 2 x (5 - x ) ( - x 2 + 10 x - 9)
10.  The value of ò
x (1+ ln x )2
1
is f ¢( x ) =
(9 - x 2 )2
=
(9 - x 2 )2
(A)  2/3 (B)  1/3 Þ f ¢( x ) = 0 Þ x = 1, 9
(C)  3/ 2 (D)  ln2
5 1 3
 Ans. (A) Þ f (0) = , f (1) = , f (2) =
9 2 5

23.6  Properties Based on Inequality The maximum and minimum value of f(x) in x Î[0 ,2] is f (1) and
f (2).
If functions A (x), B (x) and C (x) are continuous in x Î[a, b] and
14.  2 2
5− x
2

satisfying the condition ∫0 f (1) dx ≤ ∫0 9 − x 2 dx ≤ ∫0 f (2) dx


A( x ) ≤ B( x ) ≤ C ( x ) 2
5− x
⇒ 2 f (1) ≤ ∫ dx ≤ 2 f (2)
then 0 9 − x2
b b b

∫ A( x )dx ≤ ∫ B( x )dx ≤ ∫ C ( x )dx


2
5− x 6
⇒ 1≤ ∫ dx ≤
0 9− x
2
a a a 5
12
1 dx p b b
Illustration 23.41   For n ≥ 1 , prove that
2
≤ ∫
0 1− x 2 n

6
. 16.  ∫
a
A( x )dx ≤ ∫ | A( x )| dx
a

Solution: For n ≥ 1 and -1 £ x £ 1 ,


Illustration 23.44   Suppose a  is real number then prove that
1− x 2 ≤ 1− x 2 n ≤ 1
1 1 | e2api − 1| ≤ 2p |a |
and 1≤ ≤
1− x 2n
1− x 2 Solution: Let A( x ) = ea xi , a  and x are real.
1/2 1/2 1/2
1 1
⇒ ∫ 1dx ≤ ∫ dx ≤ ∫ dx 2p 2p
1− x 1− x 2 ∫e ∫e
2n a xi a xi
0 0 0 dx ≤ dx
1/2
1 1 0 0
⇒ ≤∫
1/2
dx ≤ sin−1 x 0
2 0 1− x 2 n 2p 2p 2p

òe dx £ ò 1dx Þ òe
a xi a xi
1/2 dx £ 2p
1 dx p

2
≤ ∫
0 1− x 2n

6
0 0 0

2p
eaxi | e 2api - 1|
15. 
If m and M are the smallest and greatest values of a function £ 2p Þ £ 2p Þ| e 2api - 1| £ 2p | a |
ai 0
|a |
A (x) on an interval [a, b], then
b
m(b - a) £ ò A( x )dx £ M(b - a) If A2 ( x ) and B 2 ( x ) are integral on [a, b], then
17. 
a

(∫ A ( x )dx ) (∫ B ( x )dx )
1/2 1/2
Illustration 23.42   Prove that 2 ≤ ∫ 2 + x 5 dx ≤ 2 3 . b b b
∫ A( x )B( x )dx ≤ 2 2
−1 a a a

Solution: Let
f (x) = 2 + x5 23.7  Newton–Leibnitz Rule
f(x) is increasing function for all defined real value and the
minimum and maximum value of f(x) are respectively 1 and 1.  If f(x) is continuous and p(x), q(x) are differentiable functions in
the interval [a, b], then
3 in x Î[ -1, 1] . Therefore,
1 1 1 d q( x ) d d
∫ 1 dx ≤ ∫ f ( x ) dx ≤ ∫ 3 dx dx ∫p ( x )
f (t )dt = f {q( x )} {q( x )} − f ( p( x )} { p( x )}
dx dx
−1 −1 −1
1 2.  If f(x) is continuous and p(x), q(x) are differentiable functions at
⇒ 1(1+ 1)dx ≤ ∫ f ( x ) dx ≤ 3 (1+ 1) a point x ∈(a, b ) and f ( x , t ) is continuous, then
−1
1 d  q( x ) q( x ) d
Þ 2 £ ò 2 + x 5 dx £ 2 3 ∫ f ( x , t )dt  = ∫ f ( x , t )dt

dx  p ( x ) 
 p ( x ) dx
-1
 d q( x )   d p( x ) 
2 +  f ( x , q( x )) −   f ( x , p( x ))
Illustration 23.43   Prove that 1 £ ò 5 - x2 dx £ 6 .  dx   dx 
0 9- x 5
Solution: Let x
Illustration 23.45   Let f ( x ) = ò 2 - t 2 dt . Then find the real
5− x 1
f (x) = roots of the equation x 2 - f ¢( x ) = 0.
9 − x2

Mathematical Problem Book for JEE.indb 1030 07-06-2018 14:26:14


Chapter 23 | Definite Integration 1031

Solution: 23.8  Summation of Series by Integration


x
f (x) = ò 2 - t dt2
b
1
An alternative way of describing ∫ f ( x )dx is that the definite integral
f ′( x ) = 2 − x 2 ⋅1− 2 − 1 ⋅ 0 = 2 − x 2 b a

x = f ′( x ) ⇒ x = 2 − x
2 2 2 ∫ f ( x )dx is a limiting case of the summation of an infinite series,
a
x 4 + x 2 − 2 = 0 ⇒ ( x 2 + 2)( x 2 − 1) = 0 provided f(x) is continuous on [a, b], that is,
b n−1
x = ±1
x
∫ f ( x )dx = lim h∑ f (a + rh),
a
n→∞
r =0
Illustration 23.46   If f ( x ) = cos x − ∫ ( x − t )f (t )dt, then calculate where
0 b−a
f ′′( x ) + f ( x ). h=
n
The converse is also true, that is, if we have an infinite series of the
Solution:
above form, it can be expressed as a definite integral.
æx ö
f ¢( x ) = - sin x - çç ò f (t )dt + xf ( x ) ÷÷ + xf ( x )
è0 ø 23.8.1 Method to Express the Infinite Series as
x
Definite Integral
f ′( x ) = − sin x − ∫ f (t )dt
0 1 r
f ′′( x ) = − cos x − f ( x )  1.  Express the given series in the form ∑ n f  n  .
f ′′( x ) + f ( x ) = − cos x 1 r
 2.  Then the limit is its sum when n → ∞ , that is, Lim ∑ f   .
n→∞ n n
x
1
Illustration 23.47   Let f : (0, ∞ ) → R and f ( x ) = ∫ f (t )dt. If  3. Replace
r
by x and by dx and Lim
n→∞
∑ by the sign of ∫ .
0 n n
f ( x 2 ) = x 2 (1+ x ), then find f (4)?  4. The lower and the upper limit of integration are the limiting
r
Solution: By definition of f(x) we have values of for the first and the last term of r, respectively.
n
x2 Some particular cases of the above are
f ( x 2 ) = ∫ f (t )dt = x 2 + x 3 1
0 n n-1
1 ærö 1 ærö
Differentiate both sides, (A)  lim
n®¥
å n f çè n ÷ø or lim
r =1
n®¥
å n f çè n ÷ø = ò f ( x )dx
r =0
f ( x 2 )⋅ 2x + 0 = 2x + 3x 2 0

b
Put pn
1 ærö
(B)  lim å f ç ÷ = ò f ( x )dx
x = 2 ⇒ 4 f (4) = 16 ⇒ f (4) = 4 n®¥
r =1 n è n ø a

Illustration 23.48   If a function f(x) is defined ∀ x ∈ R such where


r
a
f (t )
a a = lim =0 (as r = 1)
that ∫ f ( x )dx , a ∈ R exist. If g( x ) = ∫
+
dt , then prove that n→ ∞ n
0 x
t and
a a
r
∫ g( x )dx = ∫ f ( x )dx .
0 0
b = lim
n® ¥ n
= p (as r = pn)

Solution: 1 1 1
Illustration 23.49   If Sn = + + + , then
a
f (t ) 1 + n 2 + 2n n + n2
g( x ) = ∫ dt lim Sn is equal to.
x
t n→∞

g(a) = 0 Solution:
Differentiate w.r.t. x n n
1 1
f (x) lim Sn = lim ∑ = lim ∑
g′( x ) = − ⇒ f ( x ) = − xg′( x )
n→∞ n→∞
r =1 r + rn n→∞
r =1 r r
x n + 
a a  n n 
∫ f ( x )dx = − ∫ xg′( x )dx Now
0 0 1
a
a
a a = lim =0
n
ò f ( x )dx = - xg( x ) + ò g( x )dx
n→ ∞
0 r
0
a a
0
and b = lim = 1
n→∞ n

∫ f ( x )dx = −ag(a) + ∫ g( x )dx 1 1


0 0 lim Sn = ∫ dx
a a n→ ∞ 0 x (1+ x )
∫ f ( x )dx = ∫ g( x )dx
0 0 = 2[ln(1+ x )]10 = 2ln2

Mathematical Problem Book for JEE.indb 1031 07-06-2018 14:26:16


1032 Mathematics Problem Book for JEE

Illustration 23.50   Evaluate 23.9 Reduction Formulae for


 1 1 1 1 
Definite Integration
lim  + + + …+ 
n→∞  n + 1 n+2 n+3 n + n p /4
 1. If In = ∫ tann x dx , then
0
Solution: Let 1
In + In−2 =
 1 1 1 1  n −1
I = lim  + + + …+ 
n→∞  n + 1 n+2 n+3 n + n p /4
 2. If In = ∫ cot n x dx, then
1 n 1 0

= n→∞ n ∑
lim 1
r In + In−2 =
r =1
1+ 1− n
n
Now p /4
 3. If In = ∫ sec n x dx , then
1 0
a = lim = 0
n→∞ n (n − 1)In − (n − 2)In−2 = ( 2 )n−2
and
p /4
r  4. If In = ∫ cosec n x dx, then
b = lim = 1 0
n→∞ n
(n − 1)In − (n − 2)In−2 = −( 2 )n−2
1
1
I=∫ dx = [ln(1+ x )]0
1
p /2
0
1 + x  5. If In = ∫ sinn x dx , then
0

I = ln2 n −1
In = In−2
n
1/ n
Illustration 23.51   Evaluate lim  nn!  .  6. If In = ∫
p /2
cosn x dx , then
n→∞ n
  0
Solution: Let n −1
In = In−2
1/ n
n
(n !)1/n  1⋅ 2 ⋅ 3… n 
A = lim ⇒ A = lim   p /2
n→∞  nn
n→∞ n  7. If In = ∫ x n sin x dx , then
0
1/ n
 1 2 3 n In + n(n − 1)In−2 = n(p / 2)n−1
⇒ ln A = lim ln  ⋅ ⋅ … 
n→∞  n n n n
p /2

1 n   r   8. If In = ∫ x n cos x dx , then


⇒ ln A = lim ∑ ln   
0
n→∞ n
r =1   n   In + n(n − 1)In−2 = (p / 2)n
1
⇒ ln A = ∫ ln x dx = [ x ln x − x ]0
1
∞ b

0
 9.  e − a x sin bx dx = −
1
0 a2 + b 2
⇒ ln A = −1⇒ A =
e ∞ a
10.  ∫
0
e − a x cos bx dx =
a2 + b 2
1p + 2 p + 3p + … + n p
Illustration 23.52   Evaluate lim ( p > 0).
n→∞ n p+1 ∞ n!
Solution: 11.  ∫
0
e − a x x n dx =
a2 + 1
1p + 2 p + 3p + … + n p
p
n
rp n
1 r
lim = lim ∑ = lim ∑ ⋅  
n→∞ n p +1 n→∞
r =1 n ⋅ n p n→∞
r =1 n  n Illustration 23.53   Determine a positive integer n ≤ 5, such that
Now 1

∫e ( x − 1)n dx = 16 − 6e
x
1
a = lim = 0 0
n→∞ n
and Solution: Let
1
r
b = lim = 1 In = ∫ e x ( x − 1)n dx
n→∞ n
0
1
1
x p+1 1 Integrating by parts,
⇒ ∫ x dx =
p
= 1
0 p + 1 0
p +1 In = e x ( x − 1)n 0 − nIn−1 = −( −1)n − nIn−1

Chapter 23.indd 1032 14-06-2018 10:54:02


Chapter 23 | Definite Integration 1033

In = ( -1)n+1 - nIn-1 1 sin x


(A) 0 (B)  2 ∫ dx
1
x 1
0 3− | x |
I0 = ò e x ( x - 1)0 dx = e = e -1
1 sin x − x
0 2
1−x2
(C)  2 ∫ dx  (D)  2 ∫0
0
1+1
dx
I1 = ( −1) − I0 0 3− | x | 3− | x |
I1 = ( −1)1+1 − I0 = 2 − e , Ans. (C)
p
I2 = ( -1) 1+2
- 2I1 = -1- 2(2 - e ) = -5 + 2e,  3. If (n − m) is odd and | m | ≠ | n |, then ∫0
cos mx sin nx dx is
I3 = ( -1)1+3
- 3I2 = 1- 3( -5 + 2e ) = 16 - 6e 2n
(A)  2 (B) 0
n - m2
Therefore,
2n 2m
n=3 (C)  2 2 (D)   Ans. (A)
m −n n2 − m 2
23.9.1  Gamma Function 2


  4.  To find the numerical value of ∫−2
( px 2 + qx + s ) dx , it is necessary
∫0
x n−1e − x dx, n > 0 is called Gamma function and denoted by Γn. toknow the values of constants
If m and n are non-negative integers, then  [IIT 1992]
æ m + 1ö æ n + 1ö (A)  p (B)  q (C)  s  (D)  p and s
Gç G
p /2 è 2 ÷ø çè 2 ÷ø  Ans. (D)
ò
0
sinm x cosn x dx =
æ m + n + 2ö 100p
2G ç ÷ø  5. If I = ∫ (1− cos2 x ) dx , then the value of I is
è 2 0

where Γ(n) is called gamma function which satisfy the following (A)  100 2 (B)  200 2
properties:
(C)  50 2 (D)  None of these  Ans. (B)
Γ(n + 1) = nΓ(n) = n !
 6. 
1  1+ x 
That is, ∫ −1
log 
 1− x 
dx =
Γ (1) = 1 , Γ(1/ 2) = p
(A)  2 (B) 1 (C)  0  (D)  p
In place of gamma function, we can also use the following formula
Ans. (C)
sinm x cosn x dx = ( m - 1)( m - 3)(2 or 1)(n - 1)(n - 3)¼(2 or 1)
p /2
∫  7. If ∫
4
f ( x ) dx = 4 and ∫
4
(3 − f ( x )) dx = 7, then the value of
( m + n)( m + n - 2)¼(2 or 1)
0
−1 2
−1
It is important to note that we multiply by (p/2); when both m
and n are even.
∫ 2
f ( x ) dx is

(A) 2 (B) –3   (C)  –5  (D)  None of these


23.10  Wallis Formulae Ans. (C)
p /2 cos x
ìn -1 n - 3 n - 5 ¼2
  8.  ∫ − p /2 1+ e x
dx =
ï n ×n-2×n-4 3,
ï (A)  1 (B) 0   (C)  −1  (D)  None of these
  1.  p /2 sinn x dx = p /2 cosn x dx = ï when n is odd
ò0 ò0 í Ans.  (A)
ï n - 1
×
n - 3
×
n - 5 ¼ × 1×p ,
3 x
ï n n-2 n-4 4 2 2  9. The least value of the function F ( x ) = ∫ (3sin u + 4 cos u ) du
5p /4
ï when n is even
î
 5p 4p 
on the interval  ,  is
p /2 ( m - 1)( m - 3)¼(n - 1)(n - 3) p  4 3 
ò sin x cos x dx = × ,
m n
 2.  0 ( m + n)( m + n - 2)¼(2or1) 2 3 3 1
(A)  3 + (B)  −2 3 + +
[If m, n are both positive integers] 2 2 2
3 1
(C)  + (D)  None of these
Your Turn 2 2 2
Ans. (B)
p ¥

cos2 x 5
  1.  The value of e cos 3 x dx is
ò
-2 x
0
10.  e (sin2 x + cos2 x ) dx =
0

(A) 1 (B)  −1 (C) 0 (D)  None of these (A)  1 (B) 0


Ans. (C)
1
sin x - x 2
1 (C) 
(D)  ∞
  2.  The value of ò-1 3- | x |
dx is 2
 Ans. (C)

Mathematical Problem Book for JEE.indb 1033 07-06-2018 14:26:20


1034 Mathematics Problem Book for JEE

Additional Solved Examples p


⇒I= 4
2p
Hence, the correct answer is option (D).
1. The value of ∫ [2sin x ] dx, where [ ] represents greatest inte-
0

ger function is Let f: R → R and g: R → R be two continuous functions. Then


 4. 
p
−5p 5p -2 2
(A)  (B) –p (C)   (D) 
3 3 7 thevalue of integral
-p
ò [f ( x ) + f ( - x )][g( x ) - g( - x )] dx is
2p 2
Solution: ò [2sin x ] dx =
0
(A)  p (B)  1
p 5p 7p 11p
(C)  −1 (D)  0
6 6 p 6 6 2p

ò 0 dx + ò dx + ò 0 dx + ò ( -1) dx + ò ( -2) dx + ò ( -1) dx Solution: If the given function be F(x), then clearly
0 p
6
5p
6
p 7p
6
11p
6
F(–x) = –F(x)
= -p and hence
Hence, the correct answer is option (B). I=0
Hence, the correct answer is option (D).
1
æpxö æ 1ö 2a
 2. If f(x) = a sin ç + b , f 1 ç ÷ = 2 and ò f ( x )dx = , then
è 2 ÷ø è 2ø 0
p Let f: R → R be a differentiable function and f(1) = 4. Then the
 5. 
f(x)
2t
x ®1 ò x - 1
p p 2 3 value of lim dt is
(A)  a = , b = (B)  a = ,b=
2 2 p p 4

(A)  8f ′(1) (B)  4f ′(1)


-4 4
(C)  a = 0, b = (D)  a = ,b=0 (C)  2f ′(1) (D)  f ′(1)
p p
Solution:
Solution:
f(x)
f(x)
2t é t2 ù
f ¢( x ) =
ap æp ö
cos ç ÷
lim
x →1 ∫ x −1
dt = lim ê
x ®1 x - 1
ë
ú
û4
2 è 2x ø 4

[f ( x )] - 4 2
2
æ0 ö
æ 1ö p 1 4 = lim form÷
Þ f ¢ç ÷ = a ´ = 2 Þa= ; x ®1 x -1 èç 0 ø
è 2ø 2 2 p
1 2f ( x ) × f ¢( x )
1
é -2a æ px ö ù 2a 2a = lim [By applying L’ Hospital rule)
ò0 f ( x )dx = êë p cos çè 2 ÷ø + bx úû = b + p = p (given) x ®1 1
0
= 2f (1) f ′(1) = 8f ′(1)
Therefore,
Hence, the correct answer is option (A).
b=0
1.5
The only choice with a = 4/p.
ò [x
2
 6. The value of ] dx is.
Hence, the correct answer is option (D). 0

Solution: For
p /2 dx
 3.  ò = 0 ≤ x < 1, 0 ≤ x2 < 1
0 1+ tan3 x
  (A)  0 (B)  1 and
[ x2 ] = 0
p p
  (C)  2 (D)  4 1 £ x < 2 , 1 £ x 2 < 2 and [ x 2 ] = 1

2 £ x < 1× 5, 2 £ x 2 < 2 × 25 and [ x 2 ] = 2


Solution:
p Therefore,
p
dx dx a a
I= ò 2
1+ tan3 x
= ò
0
2
1+ cot x
3 , using ò f ( x )dx = ò f (a - x )dx 1.5 1 2 1.5

ò [ x ]dx = ò 0 dx + ò 1dx + ò 2dx


0 2
0 0

p 0 0 1 2
tan3 x p
p
I= ò 2
0 1+ tan3 x
dx ⇒ 2I = ò 2
0
1 dx = 2
= 2 − 1+ 2(1⋅ 5 − 2 ) = 2 − 2

Mathematical Problem Book for JEE.indb 1034 07-06-2018 14:26:22


Chapter 23 | Definite Integration 1035

2 x

 7. The value of ∫
−2
1− x 2 dx is. Let f: (0, ∞) → R and F(x) =
11.  ∫ f (t ) dt .
0
If F(x2) = x2(1 + x), then
f (4) equals
Solution:
2 2
(A)  5/4 (B)  7
∫ 1− x dx = 2∫ 1− x (C)  4 (D)  2
2 2
dx
−2 0
Solution: We have
1 2
 1 2

= 2  ∫ 1− x 2 dx + ∫ 1− x 2 dx  = 2  ∫ (1− x 2 )dx + ∫ ( x 2 − 1)dx  x2
0 1  0 1  F ( x 2 ) = ∫ f (t ) dt = x 2 + x 3
ìï é 1
x3 ù é x3 ù üï
2
1 8 1
0

= 2 í ê x - ú + ê - x ú ý = 2 1− + − 2 − + 1 = 2[2] = 4


3 3  3 3 3  Differentiating both sides, we get
ïî ë û0 ë û1 ïþ
 1 2 2  f(x2) ⋅ 2x = 2x + 3x2
8.  lim  + + …+  is equal to
n→∞ 1 − n2
 1 − n2 1 − n2  ⇒ f(x2) = 1 + (3/2)x
⇒ f (4) = 1 ± (3/2) (2) = 4, –2
(A)  0 (B)  − 1
2 Hence, the correct answer is option (C).
1
(C)  (D)  None of these x
1
2 12.  Let g (x) = ∫ f (t ) dt , where f is such that ≤ f (t ) ≤ 1 for t ∈
0 2
Solution: 1
[0, 1] and 0 ≤ f(t) ≤ for t ∈ (1, 2] . Then g (2) satisfies the
r 2
n
r n
n  1 n  r  1 inequality
lim ∑ = lim∑ 2 n = lim   ∑   lim
n→∞
r =1 1 − n
2 n→∞
r =1 n  1  n→∞
 n  r =1  n  n→∞  1  
 2 − 1  2  − 1
n  n   3
(A)  − ≤ g (2) ≤
1
(B)  0 ≤ g(2) < 2
1
2 2
1 −1
= ∫ x dx ( −1) = (−1) = (C) 
3 5
< g (2 ) ≤ (D)  2 < g(2) < 4
2 2
0
2 2
Hence, the correct answer is option (B). Solution:
2 1 2

ì 1 1 1 ü g (2) = ∫ f (t ) dt = ∫ f (t ) dt + ∫ f (t )dt
9.  The value of lim í + + ¼+ ý is equal to 0 0 1
x ®¥ ( x + 1) ( x + 2) ( x + 5x )þ
î b
Using m (b – a) ≤ ∫ f (t ) dt ≤ M(b − a), we get
(A)  ln 2 (B)  ln 3 a

(C)  ln 6 (D)  None of these 1 1


× 1 + 0 ≤ g (2) ≤ 1 × 1 +
2 2
Solution: 1 3
5n
≤ g (2) ≤ ⇒ 0 ≤ g (2) < 2
1 1 5n 1 2 2
lim ∑ = lim ∑
n→∞
r =1 n + r
n→∞ n r Hence, the correct answer is option (B).
r =1
1+  
 n
e sin x for x ≤ 2
cos x 3
5
dx 13.  If f ( x ) =  , then ò f ( x ) dx =
∫0 1+ x = [ln(1+ x )]0 = ln 6
5
2 otherwise -2

(A)  0 (B)  1 (C)  2 (D)  3


Hence, the correct answer is option (C).
p
Solution:
cos2 x
ò-p 1+ a x dx , a > 0 is
10.  The value of 3 2 3

ò f ( x )dx = ò e sin x dx + ò 2 dx = 0 + 2 (since ecos x sin x is odd )


cos x

(A)  p (B)  ap -2 -2 2

Hence, the correct answer is option (C).


(C)  p/2 (D)  2p
e2
loge x
Solution:
14.  The values of the integral ò -1 x
dx is
e
p p p
cos2 xdx æ cos2 x cos2 x ö p 3 5
I= ò
-p
1+ a x
=òç
0 è 1 + a x
+
1 + a -x ÷
ø
dx = ò cos2 xdx =
0
2
(A) 
2
(B) 
2
Hence, the correct answer is option (C). (C)  3 (D)  5

Chapter 23.indd 1035 11-06-2018 10:20:35


1036 Mathematics Problem Book for JEE

Solution: Let 1
sin x cos x
1

e2
Let I = ò
3.  dx and J = ò dx . Then which one of the
loge x x x
I=ò
0 0
dx following is true?
e -1
x
2 2
æ1 ö (A)    I > and J > 2 (B)  I < and J < 2
Note x > 0 for x ∈ ç , e 2 ÷ 3 3
èe ø
2 2
and (C)    I < and J > 2 (D)  I > and J < 2
3 3
1 [AIEEE 2008]
loge x is < 0 for <x<1
e Solution:
and
for 1 < x < e2, logex > 0
1
sin x
1
x
1
2
1
2 2
I=∫ dx < ∫ dx = ∫ xdx = x 3/2 = ⇒ I <
Therefore, x x 3 0 3 3
0 0 0
1 1
1
loge x loge x −1
e2
3 cos x 1 1

I = −∫ dx + ∫ dx = + 2 = J=ò dx < ò dx = 2 x = 2
x x 2 2 0 x 0 x 0
1/ e 1

Hence, the correct answer is option (A). Therefore, J < 2.


Hence, the correct answer is option (B).
Previous Years’ Solved JEE Main/AIEEE Let p(x) be a function defined on R such that p′(x) = p′(1 – x),
4. 
Questions 1

for all x Î [0,1], p(0) = 1 and p(1) = 41. Then ò p( x ) dx


0
equals
x
 1 log t
1.  Let F ( x ) = f ( x ) + f   , where f ( x ) = ∫ dt . Then F(e) equals (A)  21 (B) 41
x 1 1+ t
(C)  42 (D)  41
1
(A)  (B)  0 [AIEEE 2010]
2
(C)  1 (D)  2 Solution: We have
p′( x ) = p′(1− x ) ⇒ p( x ) = − p(1− x ) + c
[AIEEE 2007]
At x = 0, p(0) = − p(1) + c ⇒ 42 = c
Solution: We have
x
Now,
log t p( x ) = − p(1− x ) + 42
f (x) = ∫ dt
1
1+ t which implies that
That is, p( x ) + p(1- x ) = 42
1 1 1
e 1/ e
æ 1ö log t log t I = ò p( x ) dx = ò p(1- x )dx Þ 2I = ò (42)dx Þ I = 21
F (e ) = f (e ) + f ç ÷ Þ F (e ) = ò dt + ò dt
è eø 1
1+ t 1
1+ t 0 0 0
e
log t log t
e
log t 1
e
Hence, the correct answer is option (A).
=ò +ò dt = ò dt =
1
1+ t 1 t (1+ t ) 1
t 2 1
8log(1+ x )
Hence, the correct answer is option (A).
5.  The value of ò
0 1+ x 2
dx is

x p p
pdt (A)  log2 (B)  log2
2.  The solution for x of the equation ò t t 2 - 1 2 is = 8 2
2
(C)  log 2 (D)  p log2
 (A) 2 (B)  p
[AIEEE 2011]
3 Solution: We have
 (C)  (D)  2 2
2 1
log(1+ x )
[AIEEE 2007] I = 8∫ dx
Solution: 0 1+ x 2

x
Put x = tanq, then
p dt dx = sec2q dq
∫ t t2 −1 2 =
That is,
2
p
p p p 3p 1
⇒ [sec t ] 2 = ⇒ sec −1 x − = ⇒ sec −1 x =
−1 x
log(1+ x ) 4
log(1+ tanq )
2 4 2 4 I = 8∫ dx = 8 ∫ sec2 q dq
1+ x 2
1+ tan2 q
 p p 0 0
⇒ x = sec  p −  = − sec ⇒ x = − 2 p
 4 4 4
log(1+ tanq )
= 8∫ sec2 q dq
There is no correct option for this question. 0 sec2

Mathematical Problem Book for JEE.indb 1036 07-06-2018 14:26:27


Chapter 23 | Definite Integration 1037

p p p
4
 p 
4
 1− tanq  (A)  4 3 − 4 (B)  4 3 − 4 −
= 8 ∫ log  1+ tan  − q   dq = 8 ∫ log  1+ q
dq 3
0
  4   0
 1+ tanq  2p
p p
(C)  p − 4 (D)  −4−4 3
4 4
3
= 8 ò log2 dq - 8 ò log(1+ tanq )dq [JEE MAIN 2014 (OFFLINE)]
0 0 Solution: See Fig. 23.2.
p
= 8log2 - I Þ 2I = 2p log2 Þ I = p log2 p x x æ
p

2
4 ò 0
1+ 4 sin2
2
- 4 sin dx = ò ç 1- 2sin ÷
2 è 2 ø
Hence, the correct answer is option (D). 0

p
x p x  x p  x
6.  If g( x ) = ò0 cos 4t dt , then g( x + p ) equals = ∫ 1− 2sin dx = ∫ 3  1− 2sin  dx + ∫p −  1− 2sin  dx
0 2 0  2  2 
3

(A)  g( x ) (B)  g( x ) + g(p ) p


p
g(p )  x 3  x
 2cos   2cos 
(C)  g( x ) - g(p ) (D)  g( x ) ⋅ g(p ) = x + 2 2
 − x + 
[AIEEE 2012]  (1/ 2)   (1/ 2) 
Solution:  0  p
3
x
g( x ) = ∫ cos 4t dt p   p 
0 =  + 2 3  − (4) − (p + 4(0)) −  + 2 3  
3   3 
sin4 x sin4 x
⇒ g′( x ) = cos 4 x ⇒ g( x ) = + k ⇒ g( x ) = [since, g(0) = 0]
4 4 p p
= + 2 3 − 4 −p + + 2 3
Therefore, 3 3
g( x + p ) = g( x ) + g(p ) = g( x ) − g(p ) (since, g(p ) = 0) 2p p
= −p + 4 3 − 4 = 4 3 − − 4
Hence, the correct answers are options (B) and (C). 3 3

p /3
dx
7. Statement-I: The value of the integral ∫ 1+
p /6 tan x
is equal
p
to .
6
b b π
Statement-II: ò f ( x )dx = ò f (a + b - x ) dx . π 5π
a a
3 3
(A)  Statement-I is True; Statement-II is true; Statement-II is not
a correct explanation for Statement-I.
(B)  Statement-I is True; Statement-II is False.
(C)  Statement-I is False; Statement-II is True.
(D)  Statement-I is True; Statement-II is True; Statement-II is a Figure 23.2
correct explanation for Statement-I.
[JEE MAIN 2013] Hence, the correct answer is option (B).
Solution: We have 1
ln(1+ 2 x )
p /3
dx
p /3
dx
p /3
dx
9.  The integral ò 0
2
1+ 4 x 2
dx , equals
I= ∫ 1+ = ∫
tan x p /6 p p 
= ∫ p  p p
p /6
1+ tan  + − x  p /6
1+ tan  − x  (A)  ln2 (B)  ln2
6 3  2  4 8
p /3
dx p p
(C)  ln2 (D)  ln2
= ∫ 1+
p /6 cot x 16 32
[JEE MAIN 2014 (ONLINE SET-1)]
p /3
dx
p /3
tan x
p /3 1+ tan x ( ) Solution:
I= ò Þò dx Þ 2I = ò dx
p /6 1
1+ p /6 1+ tan x p /6 1+ tan x ( ) 1
ln(1+ 2 x )
tan x ∫0
2
1+ 4 x 2
dx
1 ép p ù p Put 2x = y. Therefore,
ÞI = ê - úÞI =
2ë3 6û 12 2dx = dy
Hence, the correct answer is option (C). Therefore, integration changes to
p
x x 1 1 ln(1+ y )
2 ∫0 1+ y 2
I= dy
8.  The integral ∫
0
1+ 4 sin2
2
− 4 sin dx equals
2

Mathematical Problem Book for JEE.indb 1037 07-06-2018 14:26:30


1038 Mathematics Problem Book for JEE

Now, put
y = tan q ⇒ dy = sec2q dq
1 p ln(1+ tanq )
I= ò4 sec2q dq 1
2 0 sec2 q
p
1  p  1 p 1− tanq 
I= ∫4
ln  1+ tan  − q   dq = ∫ 4 ln  1+  dq π/ 2 π
2 0   4   2 0  1+ tanq 
p /4
1 p4 æ 1+ tanq + 1- tanq ö æ 2 ö
2 ò0 è ò ln çè 1+ tanq ÷ø dq
= ln ç ÷ dq = −1
1+ tanq ø 0

1 p4 1 p
=
2 ò0
ln2dq - ò 4 ln(1+ tanq )dq
2 0
ln2 p4
2 ∫0
= dq − I Figure 23.3

Therefore, Hence, the correct answer is option (D).


1 æp ö p p
2I = (ln2) ç - 0 ÷ ⇒ 2I = ln2 ⇒ I = ln2 t
If for a continuous function f(x), ò (f ( x ) + x )dx = p - t , for
2 2
2 è 4 ø 8 16 12. 
-p

Hence, the correct answer is option (C).  p


all t ≥ −p, then f  −  is equal to
e  3
10.  If for n ≥ l, Pn = ò (log x ) dx , then P10 − 90P8 is equal to
n
p
1 (A)  p (B) 
2
(A)  −9 (B)  10e
p p
(C)  −9e (D)  10 (C)  (D) 
3 6
[JEE MAIN 2014 (ONLINE SET-2)]
[JEE MAIN 2014 (ONLINE SET-3)]
Solution: Solution:
e t
Pn = ∫ (log x ) dx ∫ (f ( x ) + x )dx = p 2 − t 2
n
1 −p

Put Therefore,

log x = t ⇒ et = x
d
dt (∫
−p
t
)
(f ( x ) + x ) dx =
d 2 2
dt
(p − t )
Therefore,
d
⇒ ( f (t ) + t ) t = −2t ⇒f (t) = − 3t
d x = et d t dt
1 1
Therefore,
Pn = ∫ t ne t dt = [t ne t ]10 − ∫ nt n−1e t dt = 1 − n Pn−1
0 0
 −p   −p 
f = −3 ×  =p
= 1 − n [1 − (n − 1) Pn−2] = 1 − n + n (n − 1) Pn−2  3   3 

Therefore, Hence, the correct answer is option (A).


P10 = 1 − 10 + 10 (10 − 1) P8 ⇒ P10 − 90 P8 = − 9 x et
Let function F be defined as F ( x ) = ∫
13.  dt , x > 0 . Then the
Hence, the correct answer is option (A). 1 t
et x

11.  If [ ] denotes the greatest integer function, then the integral


value of the integral
1 t +a ∫
dt , where a > 0, is
p
ò
0
[cos x ] dx is equal to (A)  ea[F(x) − F(1 + a)] (B)  e−a [F(x + a) − F(a)]
(C)  ea [F(x + a) − F(1 + a)] (D)  e−a [F(x + a) − F(1 + a)]
p
(A)  (B)  0 [JEE MAIN 2014 (ONLINE SET-4)]
2
p Solution:
(C)  −1 (D)  −
2 x
et
[JEE MAIN 2014 (ONLINE SET-3)] f (x) = ∫ dt , x > 0
1 t

Solution: See Fig. 23.3. Now


x
pù p et
p p/2 p é I=∫ dt
ò [cos x ]dx = ò 0dx + òp ( -1)dx = -[ x ] = - êp - ú = -
p
p
2û 2 t +a
0 0
2 2 ë 1

Mathematical Problem Book for JEE.indb 1038 07-06-2018 14:26:32


Chapter 23 | Definite Integration 1039

Put In the second integral, put


t + a = m ⇒ t = m − a ⇒ dt = dm 1 −1
t= ⇒ dt = 2 du
Therefore, m u
x +a x +a x +a
e m −a em et 1
I= ∫
1+ a
m
d m = e−a ∫
1+ a
m
du = e − a ∫ dt
1+ a
t
When t = 1, u = 1; When t =
x
,u= x

 1 et e 
x +a t
Therefore,
= e − a  ∫ dt + ∫ dt 
1+a t 1 t  x x
 1 log t − log u . 1
 e e 
x +a t 1+a t f (x) + f   = ∫ dt − ∫ du
= e − a  ∫ dt − ∫ dt  = e−a [F(x + a) − F(1 + a)]  x  1 1+ t 1 1+
1 u2
 1 t 1 t  u
Hence, the correct answer is option (D). x x x x
log t log u log t log t
=∫ dt + ∫ du = ∫ dt + ∫ dt
4
log x 2
1 + t u(1 + u ) 1 + t 1 (1+ t )
t
14.  The integral ∫ log x
1 1 1
dx is equal to
2
2
+ log(36 − 12 x + x 2 ) x x
 log t   1  1 (log x )2
(A)  4 (B)  1 = ∫  ⋅  1+  dt = ∫ log t dt =
1  1+ t   t 1t 2
(C)  6 (D)  2
[JEE MAIN 2015 (OFFLINE)] Hence, the correct answer is option (B).
Solution: 1 1 1
If 2 ∫ tan xdx = ∫ cot (1− x + x )dx , then ∫ tan−1(1− x + x 2 )dx
−1 −1 2
16. 
4 0 0
log x 2 0
   I = ∫ dx is equal to
log x 2 + log(36 − 12 x + x 2 )
2
p
4 (A)  + log2 (B)  log2
log x 2 2
=∫ dx
2 log x + log(6 − x )
2 2
p
(C)  − log4 (D)  log4
4
log x 2
  = ∫ dx  (1) [JEE MAIN 2016 (ONLINE SET-1)]
2 log x + log(6 − x )
Solution: We have
4
log(2 + 4 − x )
⇒ I=∫ dx 1 1

2 log(2 + 4 − x ) + log( x ) 2 ∫ tan−1 x dx = ∫ cot −1(1− x + x 2 )dx


0 0
b b
By using property ∫ f ( x )dx = ∫ f (a + b − x )dx, we get 1
p 
a a = ∫  − tan−1(1− x + x 2 ) dx
0
 2 
4
log(6 − x )
I = ∫  (2) p
1
log(6 − x ) + log x
2 ∫0
2 = − tan−1(1− x + x 2 )dx
Eqs. (1) + (2) gives,
4
Therefore,
2I = ∫ 1dx = 2 ⇒ I = 1
1 1
2 p p
∫ tan (1− x + x )dx = − 2∫ tan−1 dx = − 2I
−1 2
Hence, the correct answer is option (B). 2 2
0 0

x 1 1
log t  1 1 2x
⇒ I = ∫ tan−1 xdx = ( x )tan−1 x 0 −
2 ∫0 1+ x 2
1
15.  For x > 0, let f ( x ) = ∫ dt . Then f ( x ) + f   is equal to dx
1 1 + t x 0

1 1 1 1 1
(A)  (log x )2 (B)  (log x )2 = x tan−1 x 0 − log(1+ x 2 )
4 2 2 0

1 p 1 p 1
(C)  log x log x 2
(D)  = − (log2 − 0) = − log2
4 4 2 4 2
[JEE MAIN 2015 (ONLINE SET-1)] Hence,
1
Solution: x > 0; p p
∫ tan
−1
(1− x + x 2 )dx = − + log2 = log2
x 1/ x 2 2
 1 log t log t 0
f (x) + f   = ∫ dt + ∫ 1+ t dt
 x  1 1+ t 1 Hence, the correct answer is option (B).

Mathematical Problem Book for JEE.indb 1039 07-06-2018 14:26:33


1040 Mathematics Problem Book for JEE

17. 
The value of the integral 2.  Match the Column I to Column II.
10
[ x 2 ]dx Column I Column II
∫ [x 2
− 28 x + 196] + [ x 2 ] 1
4
dx 1 2
where [x] denotes the greatest integer less than or equal to x, is
(A)  ∫ 1+ x
−1
2
(P)  
2
log  
3
1
(A)  (B)  6 1
dx 2
3 (B)  ∫ 1− x 2
(Q)  2log  
3
(C)  7 (D)  3 0

[JEE MAIN 2016 (ONLINE SET-2)] 3 p


dx
Solution: We have
(C)  ∫ 1− x
2
2
(R) 
3
10 2
[ x ]dx 2 p
I=∫ dx (S) 
4 [ x − 28 x + 196] + [ x ]
2 2
(D)  ∫x
1 x −12 2
10
[ x 2 ]dx [IIT-JEE 2007]
=∫
4 [( x − 14) ] + [ x ]
2 2
Solution:
Let us use the following property:
(A)  ã  (S)
b b 1
dx
I = ∫ f ( x )dx = ∫ f (a + b − x )dx I=∫
a a −1
1 + x2
Therefore, = [tan−1 x ]1−1
10
[( x − 14)2 ] dx p  p p
I=∫ = −−  =
[ x 2 ] + [( x − 14)2 ] 4  4 2
4

10 (B)  ã (S)
2I = ∫ 1 dx = 6 ⇒ I = 3 1
dx
4 I=∫
Hence, the correct answer is option (D). 0 1− x 2
p p
= [sin−1 x ]10 = −0=
Previous Years’ Solved JEE Advanced/ 2 2
(C)  ã  (P)
IIT-JEE Questions 3
dx
I=∫
2 1− x
2 2
sec x

∫ f (t ) dt
1 1+ x 
3

1.  limp 2
equals = log
x→ p2 2 1− x  2
4 x2 −
16 1 4 3
= log − log 
8 2 2 2 1
(A)  f (2) (B)  f (2)
p p 1 2
= log  
2  1 2 3
(C)  f   (D)  4f(2)
p  2 (D)  ã  (R)
[IIT-JEE 2007]
2
dx
I=∫
Solution: We have 1 x x −1
2

sec2 x
= [sec −1 x ]12
∫ f (t )dt
0  p p
lim 2
 form  = −0=
x→
p x − (p / 16)
2 2
0  3 3
4
Hence, the correct matches are (A)ã(S); (B)ã(S); (C)ã(P);
Using L’ Hospital’s rule, we get
(D)ã(R).
f (sec2 x ) ⋅ (2sec x ) ⋅ (sec x ) ⋅ (tan x ) f (2) ⋅ 2 ⋅ 2 ⋅ 2 ⋅1 8f (2)
lim = = 3. 
Let f be a non-negative function defined on the interval [0, 1].
x→
p 2x 2 ⋅ (p / 4) p x x
4

Hence, the correct answer is option (A).


If ∫
0
1− (f ′(t ))2 dt = ∫ f (t )dt ,0 ≤ x ≤ 1, and f(0) = 0, then
0

Chapter 23.indd 1040 11-06-2018 10:20:48


Chapter 23 | Definite Integration 1041

1 1 1 1 also,
(A)  f   < and f   >
 2 2  3 3 f ’(x) = f(x), x > 0
⇒ f(x) = k, x > 0
 1 1  1 1
(B)  f   > and f   > Since, f(0) = 0 and f(x) is continuous, so
 2 2  3 3
f(x) = 0 ∀ x > 0
 1 1  1 1
(C)  f   < and f   < Therefore, f (ln 5) = 0.
 2 2  3 3
Hence, the correct answer is (0).
1 1 1 1
(D)  f   > and f   < x
 2 2  3 3 1 t ln(1+ t )
[IIT-JEE 2009]
6.  The value of lim
x →0 x3 ∫
0 t +4
4
dt is

Solution: 1
(A)  0 (B) 
x x 12
∫ 1− ( f ′(t ))2 dt = ∫ f ′(t )dt 1 1
0 o (C)  (D) 
24 64
⇒ 1 − ( f ′ ( x )) = f ( x )
2
 [IIT-JEE 2010]
⇒ f ′( x ) = ± 1− ( f ( x ))2 Solution:
x
⇒ f ( x ) = sin x or f ( x ) = − sin x (not possible) 1 t ln(1+ t )dt  0 
x →0 x 3 ∫
lim =  form 
t 4
+ 4  0 
⇒ f ( x ) = sin x x
1 t ln(1+ t )dt  0
0
ln(1+ x )
x →0 x 3 ∫
Also, limUsing L’ Hospitals=  lim
rule, form
we getx 1 ln(1 + x) 1 1
t4 + 4  0 x →0 4 = lim = =
x > sin x ∀x > 0 0 ( x + 4)3 x 2 x → 0 3 x ( x + 4) 3(1+ x )[ x + 4 + 4 x ] 12
4 2 4

x ln(1+ x ) 1 ln(1+ x ) 1 1
Hence, the correct answer is option (C). lim 4 = lim = =
p
x →0 ( x + 4)3 x 2 x →0 3 x ( x 4 + 4) 3(1+ x )[ x 2 + 4 + 4 x 4 ] 12
sin nx
4.  If In = ∫ (1+ p dx , n = 0, 1, 2, …,then
x
)sin x Hence, the correct answer is option (B).
−p
1
10 x 4 (1− x )4
(A)  In = In+2 (B)  ∑I 2 m+1 = 10p 7.  The value(s) of ∫
0 1+ x 2
dx is (are)
m=1
10
22 2
(C)  ∑I 2m = 0 (D)  In = In+1 (A)  − p (B) 
105
m=1 7
[IIT-JEE 2009]
71 3p
Solution: (C)  0 (D)  −
p
15 2
sin nx [IIT-JEE 2010]
In = ∫ (1+ p x
)sin x
dx
−p Solution:
p p
 sin nx p x sin nx  sin nx 1
= ∫ + dx = ∫ x 4 (1- x )4
0  (1 + p x
)sin x (1 + p x
)sin x 
 0
sin x ò
0
1+ x 2
dx

Now, 1
 4 
p = ∫  x 6 − 4 x 5 + 5x 4 − 4 x 2 + 4 −  dx
sin(n + 2) x − sin nx 0  1+ x 2 
I n+ 2 − I n = ∫ dx
sin x 1
0
 x7 2x6 4 x3 
p = − + x5 − + 4x −p
2cos(n + 1) x ⋅ sin x  7 3 3 0
=∫ dx = 0
0
sin x 1 2 4 22
p = − + 1− + 4 − p = −p
7 3 3 7
⇒ I1 = p , I2 = ∫ 2cos xdx = 0
0 Hence, the correct answer is option (A).
Hence, the correct answers are options (A), (B) and (C).
ln3
x sin x 2
5.  Let f: R → R be a continuous function which satisfies
x
8.  The value of ∫ sin x 2 + sin(ln6 − x 2 ) dx is
ln2
f ( x ) = ∫ f (t ) dt . Then the value of f(ln 5) is _____.
1 3 1 3
0
[IIT-JEE 2009] (A)  ln (B)  ln
4 2 2 2
Solution:
x 3 1 3
(C)  ln (D)  ln
f ( x ) = ∫ f (t )dt ⇒ f (0) = 0 2 6 2
0 [IIT-JEE 2011]

Mathematical Problem Book for JEE.indb 1041 07-06-2018 14:26:36


1042 Mathematics Problem Book for JEE

Solution: Put
p2
5/6
p
(R) 
x 2 = t ⇒ 2 x dx = dt (C)  The value of ∫ sec(px )dx is
ln3 7/6 3
ln3 ln3
1 sin t 1 sin(ln6 − t )
I= ∫ sin t + sin(ln6 − t ) dt ⇒ I = 2 ln2∫ sin(ln6 − t ) + sin t dt
2 ln2 (D)  
The maximum value of  1 
Arg 
(S)  p
 1− z 
1
ln3
1 3 for z = 1, z ≠ 1 is given by
2I = ∫ 1dt ⇒ I = 4 ln 2
2 ln2
p
(T) 
Hence, the correct answer is option (A). 2
Let f : [1, ∞ ) → [2, ∞ ) be a differentiable function such that
9.  [IIT-JEE 2011]
x Solution:
f(1) = 2. If 6 ∫ f (t ) dt = 3 xf ( x ) − x 3 for all x ≥ 1, then the value of
1
(A) 
f(2) is _____.  
a ⋅ b = −1+ 3 = 2
[IIT-JEE 2011]  
a = 2, b = 2
Solution:
2 1
x cosq = =
2×2 2
6 ∫ f (t )dt = 3 x f ( x ) − x 3 ⇒ 6f ( x ) = 3f ( x ) + 3 xf ′( x ) − 3 x 2
1 p 2p 2p
q= , but is opposite to the side of the maximum
⇒ 3f ( x ) = 3 xf ′( x ) − 3 x ⇒ xf ′( x ) − f ( x ) = x
2 2 3 3 3
length.
dy dy 1
    ⇒ x − y = x2 ⇒ − y=x  (1) b
dx dx x
∫ (f ( x ) − 3 x )dx = a − b2
2
1
(B) 
∫ − x dx − loge x a
I.F = e =e b
3 2 2 −a2 + b 2
∫a = − + − =
2 2
1 f ( x )dx ( b a ) a b
Multiplying both sides of Eq. (1) by 2 2
x ⇒ f (x) = x
1 dy 1 d  1
− y = 1⇒  y .  = 1 p 2  ln (sec p x + tanp x ) 7/6 
5/6
x dx x 2 dx  x  (C)   
ln3  p 
Integrating  
y p  5p 5p 7p 7p 
= x +c =  ln sec + tan − ln sec + tan =p
x ln3  6 6 6 6 
Put x = 1, y = 2 (D)  Let
⇒ 2 = 1+ c ⇒ c = 1 ⇒ y = x 2 + x 1 1
u= ⇒ z = 1−
⇒ f ( x ) = x 2 + x ⇒ f (2) = 6 1− z u
1
Note: If we put x = 1 in the given equation we get f(1) = 1/3. z = 1 ⇒ 1− = 1
u
Hence, the correct answer is (6**).
⇒ u −1 = u
**Question is ambiguous as 8/3 can also be the answer.
Therefore, locus of u is perpendicular bisector of line segment
10.  Match the statements given in Column I with the values given p
joining 0 and 1, so maximum arg u approaches but will not
in Column II. attain. 2

Column I Column II Hence, the correct matches are (A)ã(Q); (B)ã(P); (C)ã(S);
 ˆ   (D)ã(T).
(A)  If a = j + 3kˆ , b = − ˆj + 3kˆ and c = 2 3kˆ (P)  p
form a triangle, then the internal angle of 6 p /2
   2 p +x
the triangle between a and b is 11.  The value of the integral ∫
−p /2
 x + ln

 cos x dx is
p −x
b
2p
(B)  If ∫ (f ( x ) − 3 x )dx = a2 − b 2 , then the value (Q)  p2
3 (A)  0 (B)  −4
a
2
p  p2 p2
of f   is (C)  + 4 (D) 
 6
2 2 [IIT-JEE 2012]

Chapter 23.indd 1042 11-06-2018 10:21:50


Chapter 23 | Definite Integration 1043

Solution: Integrating by parts


p /2 1 1
ì 2 æp + xöü éd ù d
ò í x + ln ç
- p /2 î
è ÷ ý cos x dx
p - xøþ
I = 4 x 3 ê (1- x 2 )5 ú - ò 12 x 2 (1- x 2 )5 dx
ë dx û0 0 dx
p /2 p /2
æp + x ö  1

= ò x 2 cos x dx + ò ln ç ÷ cos x dx
èp - x ø
= 4 x 3 [5(1− x 2 )4 ( −2 x )]10 − 12 [ x 2 (1− x 2 )5 ]10 − ∫ 2 x (1− x 2 )5 dx 
- p /2 - p /2  0 
p /2
1
= 2 ò x 2 cos x dx + 0 = 0 − 0 + 12 ∫ 2 x (1− x 2 )5 dx
0
0
= 2[ x 2 sin x + 2 x cos x - 2sin x ]p0 /2
Now putting 1 – x2 = t. Then
ép 2 ù p2
= 2 ê - 2ú = -4 –2xdx = dt
ë4 û 2
Hence, the correct answer is option (B). Where x = 0, t = 1
Where x = 1, t = 0
1
12. Let f : éê ,1ùú ® R (the set of all real numbers) be a positive, Therefore,
ë2 û
0
non-constant and differentiable function such that
1 I = −12 ∫ t 5dt
 1
f ′( x ) < 2f ( x ) and f   = 1. Then the value of ò f ( x )dx lies in 1
 2
1/2
1
the interval 1
t6  1
I = 12 × ∫ t 5dt = 12 ×   = 12 × = 2
(A)  (2e – 1, 2e) (B)  (e – 1, 2e – 1) 0 6
 0 6
æ e -1 ö e - 1ö
(C)  ç , e - 1÷ (D)  æç 0, ÷ Hence, the correct answer is (2).
è 2 ø è 2 ø p
2
[JEE ADVANCED 2013]
14.  The following integral ò (2cosec x )17 dx is equal to
Solution: We have p
4
dy
< 2y (A) 
log(1+ 2 )
dx ∫0
2(e u + e − u )16 du
That is, (B) 
log(1+ 2 )

−2 x dy ∫0
2(e u + e − u )17 du
e < 2 ye −2 x log(1+ 2 )
dx (C) 
d
∫0
2(e u − e − u )17 du
( ye −2 x ) < 0 log(1+ 2 )
(D) 
dx ∫ 0
2(e u − e − u )16 du

which implies that ye −2 x is a decreasing function. [JEE ADVANCED 2014]


p
1
As
2
< x < 1 , we have Solution: ∫ p
2
(2cosec x )17 dx
4
e -1 > ye -2 x > y (1)e -2
Let cosec x + cot x = e t . Then
Þ e 2 x -1 > y > y (1)e 2 x -2
1 1 1 ( - cosec x cot x - cosec2 x )dx = e t dt
Þ ò e 2 x -1dx > ò ydx > ò y (1)e 2 x -2 > 0
1/2 1/2 1/2 ⇒ - cosec x (cot x + cosec x )dx = e t dt
Therefore, ⇒ −(cosec x )e t dx = e t dt
1
e -1
0< ò ydx < 2
Therefore,
1/2
(cosec x )dx = - dt
Hence, the correct answer is option (D).
p
Now when x = ,
ì d2
1 ü 4
13.  The value of ò 4 x 3 í 2 (1- x 2 )5 ý dx is _____.
0
î dx þ p p
cosec + cot = e t
[JEE ADVANCED 2014] 4 4
Therefore,
Solution:
1
d2 et = 2 + 1
ò 4
x3
dx 2
(1- x 2 )5 dx

0 I ⇒ t = loge (1+ 2 )
II

Chapter 23.indd 1043 14-06-2018 10:54:18


1044 Mathematics Problem Book for JEE

p 1-h 1 1-h 1
when x = , = lim+ ò dt = lim+ ò dt
2 h®0 h
t - t2 h®0
æ 2 h
1 1ö
p p -çt - t + - ÷
cosec + cot = e t è 4 4ø
2 2
Therefore,
1-h 1 1-h 1
= lim+ ò dt = lim+ ò dt
h®0 h 2 h®0 h 2 2
1–0=e t
ì 1ü 1 æ 1ö æ 1ö
- ít - ý + ç ÷ - çt - ÷
⇒t = 0 î 2þ 4 è2ø è 2ø
Therefore, 1-h
é æ t - 1/ 2 ö ù
[sin-1(2t - 1)]h
1-h
Also cosec2 x – cot2 x = 1 = lim+ êsin-1 ç ÷ ú = hlim
h®0
ë è 1/ 2 ø ûh ® 0+

1
⇒ cosec x − cot x = = e −t = lim[sin-1
{2 - 2h - 1} - sin-1(2h - 1)]
et +
h®0
Therefore,
= lim+ + sin-1(1- 2h) - lim+ + sin-1(2h - 1)
2 cosec x = et + e–t h®0 h®0

Therefore, integral reduces to p  p


= −− 
0 loge (1+ 2 )
2  2
-ò (e t + e - t )16 2dt = +2 ò (e t + e - t )dt =p
loge (1+ 2 ) 0

Hence, the correct answer is option (A).


Note: t can be replaced by m.
Hence, the correct answer is option (A). æ 1ö
16.  The value of g¢ ç ÷ is
è2ø
p
Paragraph for Questions 15 and 16: Given that for each a ∈ (A)  (B)  p
1−h 2
p
∫t
−a
(0, 1). lim+ (1− t )a−1dt exists. Let this limit be g(a). In addtion, it (C)  − (D)  0
h→0
h 2
is given that function g(a) is differentiable on (0, 1).
Solution:
[JEE ADVANCED 2014] 1-h
g(a) = lim+ ò t - a (1- t )a-1dt  (1)
 1 h®0 h
15.  The value of g   is Now
2 1−h
(A)  p (B)  2p g(1− a) = lim+ ∫ t − (1−a ) (1− t )1−a−1dt
h→0 h
p p 1-h
(C)  (D)  = lim+ ò t a-1(1- t )- a dt
2 4 h®0 h
1−h
= lim+ ∫ ( h + 1− h − t )a−1{1− ( h + 1− h − t )} − a dt
Solution: See Fig. 23.4. h→0 h
1−h
y = lim+ ∫ (1− t )a−1( 1 − 1 + t )− a dt
h→0 h
π 1−h
2 = lim+ ∫ t − a (1− t )a−1dt  (2)
h→0 h

From Eqs. (1) and (2)


g (a) = g (1 − a)
sin x −1
Therefore,
g’ (a) = − g’ (1 − a)
−1 1 x when
1 æ 1ö æ 1ö
a = , g¢ ç ÷ = - g¢ ç ÷
2 è 2ø è 2ø
æ 1ö
⇒ g¢ ç ÷ = 0
−π è 2ø
2
Hence, the correct answer is option (D).
Figure 23.4
Given 17.  Match the List I with List II.
1-h
g(a) = lim+ ò t - a (1- t )a-1dt List I List II
h®0 h
P.  The number of polynomials f(x) with non-negative 1.  8
 1 1−h − 1 −
1 1−h 1
g   = lim+ ∫ t 2 (1− t ) 2 dt = lim+ ∫ dt integer coefficients of degree ≤ 2, satisfying f(0) = 0
 2  h→0 h h→0 h t (1− t ) 1
and ∫ f ( x )dx = 1,
0
is

Chapter 23.indd 1044 14-06-2018 12:52:31


Chapter 23 | Definite Integration 1045

For (R) in List I:


Q.  The number of points in the interval [ − 13, 13] 2.  2 2 2
3 x2  3 x2 ex 3 x2 
at which f(x) = sin(x2) + cos(x2) attains its maximum ∫−2 1+ ex dx = ∫0  1+ ex + 1+ ex  dx
value, is
2

2 3x2 3.  4 = ∫ 3 x 2 dx = 8
R.  ∫−2 (1+ e x )
dx equals 0
Hence,

 1 4.  0 (R) → (1)


 1+ x  
 ∫−21 cos2 x log   dx 
 2  1− x   For (S) in List I: Let
S.  equals  1+ x 
 1  1+ x   f ( x ) = cos 2 x ln 
 ∫02 cos2 x log   dx   1− x 
  1− x  
⇒ f ( − x ) = cos 2 x ln  1− x 
P Q R S  1+ x 
(A) 3 2 4 1   1+ x 1 − x  
⇒ f ( x ) + f ( − x ) = cos 2 x ln  ×  = 0
(B) 2 3 4 1   1 − x 1+ x  
(C) 3 2 1 4
Therefore, f (x) is an odd function. Thus,
(D) 2 3 1 4 1
 1+ x 
∫ cos 2 x ln  dx = 0
2

[JEE ADVANCED 2014] − 21  1− x 

Solution: So, the denominator is non-zero. Hence,


(S) ã (4)
For (P) in List I: Let
Hence, the correct answer is option (D).
f (x) = ax2 + bx [As f (0) = 0]
[ x ], x ≤ 2
Also Let f : R → R be a function defined by f ( x ) = 
18.  ,
1 1  0, x > 2
∫ f ( x ) dx = 1⇒ ∫ (ax
0 0
2
+ bx )dx = 1
where [x] is the greatest integer less than or equal to x. If
2
1 xf ( x 2 )
⇒ 
ax 3 bx 2  a b I=∫ dx , then the value of (4I – 1) is _____.
+ = 1⇒ + = 1 2 + f ( x + 1)
 3 2 0 3 2 −1
[JEE ADVANCED 2015]
Since, a ≥ 0, b ≥ 0. Therefore,
Solution: f : R → R is
0 2
+ =1 [ x ]; x ≤2
3 2 f (x) = 
 0; x >2
So, a = 0 and b = 2. Now
2
xf ( x 2 )
3 0
+ =1 I=∫ dx ;
3 2 −1
2 + f ( x + 1)
0 1 2 3
So, a = 3 and b = 2. Thus, the possible polynomials are xf ( x 2 )dx xf ( x 2 )dx xf ( x 2 )dx xf ( x 2 )dx
I=∫ +∫ +∫ +∫
f(x) = bx or f(x) = 3x2 −1 2 + f ( x + 1) 0 2 + f ( x + 1) 1 2 + f ( x + 1) 2
2 + f ( x + 1)
Hence, 2
xf ( x 2 )dx
+∫
(P) → (2) 3
2 + f ( x + 1)
For (Q) is List I: 0 1 2 3 2
x [ x 2 ]dx x[ x 2 ] x [ x 2 ]dx x ⋅ 0 dx x ⋅0
 1 1  =∫ +∫ dx + ∫ + ∫ + ∫ 2 + 0 dx
f ( x ) = sin x 2 + cos x 2 = 2  sin x 2 + cos x 2  −1 2 + [ x + 1] 0 2 + [ x + 1] 1 2+0 x +0
 2 2  2 3

2
æ pö x (1)
2
x  1 2
= 2 sin ç x 2 + ÷
è 4ø
=0+0+
1 2
∫dx =   =
 4 1 4
⇒ 4I – 1 = 0

For maximum value of f (x), we have Hence, the correct answer is (0).
p p p 1
x2 +
= + 2np ⇒ x 2 = 2np + æ 12 + 9 x 2 ö
If α = ò (e 9 x +3tan x ) ç
-1

4 2 4 19.  dx , where tan-1 x takes only


0 è 1+ x 2 ÷ø
p 9p
⇒x=± , for n = 0 ; x = ± , for n = 1  3p 
4 4 principal values, then the value of  loge 1+ a −  is _____.
Hence,  4
(Q) → (3) [JEE ADVANCED 2015]

Chapter 23.indd 1045 11-06-2018 10:22:49


1046 Mathematics Problem Book for JEE

Solution:
æ 1ö
1 ⇒ f ç ÷ =7
æ 12 + 9 x 2 ö è 2ø
a = ò (e 9 x +3tan-1 x
)ç ÷ dx
è 1+ x ø
2

Here
0  1
⇒ f   = ±7,
2
æ p pö
tan-1 x Î ç - , ÷ but being odd function and continuous f(0) = 0, thus if
è 2 2ø
Put  1
f   = −7,
1  2
tan−1 x = t ⇒ dx = dt
1+ x 2 æ1 ö æ 1ö
p /4 then f(x) must have another root in ç ,1÷ . So f ç ÷ ¹ -7
òe è2 ø è2ø
9tant +3 t
Þ a= × (12 + 9 tan t ) × dt
2

0 1   1
Again, put  ,1 . So f   ≠ −7
2  2
9 tan t + 3t = z
1
Þ (9 (sec2t) + 3)dt = dz ⇒ f   = 7
⇒ (9(tan2t) + 12)dt = dz 2
 
9+3p /4  3p 
 9+  Hence, the correct answer is (7).
⇒ a= ∫
0
e z dz = e  4
−1
21. 
The option(s) with the value of a and L that satisfy the
⇒ 1+ a = e 9+3p /4 following equation is(are)
⇒ ln |1 + a | = 9 + 3p/4 4p
3p ò e (sin at + cos 4 at )dt
t 6
Þ ln (1+ a ) − =9
4 0
p
=L
Hence, the correct answer is (9).
ò e (sin at + cos at )dt
t 6 4

0
20. 
Let f : R → R be a continuous odd function, which vanishes
1 x
e 4p − 1 e 4p + 1
exactly at one point and f (1) = . Suppose that F ( x ) = ∫ f (t ) dt (A)  a = 2, L = p (B)  a = 2, L =
2 −1
e −1 ep + 1
x
e 4p − 1 e 4p + 1
for all x ∈ [ −1,2] and G( x ) = ∫ t f (f (t )) dt for all x ∈ [ −1,2]. If (C)  a = 4, L = (D)  a = 4, L =
−1 ep − 1 ep + 1
F( x ) 1  1 [JEE ADVANCED 2015]
lim = , then the value of f   is _____.
x →1 G ( x ) 14 2 Solution: For a = 2,
[JEE ADVANCED 2015] 4p
e t (sin6 2t + cos 4 2t )
Solution: f : R → R is a continuous odd function having a single L= ò p
dt
ò e (sin 2t + cos 2t )dt
0 t 6 4
root.
0
1 x Let
f (1) = , F ( x ) = ò f (t )dt ∀x ∈ [ −1,2] 4p
2
L1 = ∫ e (sin 2t + cos 4 2t )dt
t 6
-1
x 0
G( x ) = ∫ t f (f (t )) dt ∀x ∈[1,2] p 2p 3p 4p

−1 = ∫ f (t )dt + ∫ f (t )dt + ∫ f (t )dt + ∫ f (t )dt = I + I 1 2 + I3 + I 4
F( x ) 1  1  0 p 2p 2p
lim = ,f   = ? In the 2nd integration, put
x →1 G ( x ) 14  2 
t = p + x ⇒dt = dx
Clearly f(t) and t|f(f(t))| are odd functions for t ∈ ( −1, x ) and
x ∈ [ −1,2]. Therefore, and
t = p ⇒ x = 0, t = 2p ⇒ x = p
F( x ) 1 0 That is,
lim = (limit is of form)
x →1 G( x ) 14 0 p

I2 = ∫ f (p + x )dx
f (x) 1
⇒ lim = 0
x ®1 x | f (f ( x ))| 14 In I3, put
t = 2p + x
f (1) 1
⇒ = ⇒ dt = dx, t = 2p ⇒ x = 0, t = 3p, x = p
1| f (f (1))| 14 p

1/2 1 ⇒ I3 = ∫ f (2p + x )dx


⇒ = 0
æ 1ö 14 In I4, put
fç ÷
è 2ø t = 3p + x ⇒ dt = dx

Mathematical Problem Book for JEE.indb 1046 07-06-2018 14:27:02


Chapter 23 | Definite Integration 1047

t = 3p ⇒ x = 0, t = 4p ⇒ x = p 1

  = − ∫ (t − t )dt
p 5 3

⇒ l 4 = ò f (3p + x )dx 0
1
Therefore,
0
t6 t4   1 1 1
   = −  −  = − =
L1 I2 + I2 + I3 + I4  6 4  0  4 6  12
L= =
I1 I1 Hence, the correct answers are options (A) and (B).
I1 + ep I1 + e 2p I1 + e 3p I1 The total number of distinct x Î[0, 1] for which
23. 
   = I1 x
t2
e 4p - 1 ∫0 1+ t 4 dt = 2 x − 1 is _______.
Þ L = (1+ ep + e 2p + e 2p ) =
ep - 1 [JEE ADVANCED 2016]
Similarly, Solution: See Fig. 23.5. We have x Î[0,1] for which
4p
e −1 x
t2
L= for a = 4
ep − 1 ∫0 1+ t 4 dt = 2 x − 1
Hence, the correct answers are options (A) and (C). Let
x
t2
 p p F( x ) = ∫ dt
Let f(x) = 7 tan x + 7 tan x – 3 tan x – 3 tan x for all x ∈ − ,  .
22.  8 6 4 2
0 1+ t
4
 2 2
Then the correct expression(s) is (are) and
f(x) = 2x – 1
p /4 p /4
1
∫ ∫ f ( x )dx = 0
2
(A)  xf ( x )dx = (B)  x
12 Now, if F ′( x ) = > 0, it means that F(x) is an increasing function.
0 0 1+ x 4
p /4
1 p /4 F(0) = 0
(C)  ∫ xf ( x )dx =
6
(D)  ∫ f ( x )dx = 1 x
1 (t 2 + 1) + (t 2 − 1)
2 ∫0
0 0
F( x ) = dt
[JEE ADVANCED 2015] 1+ t 4

Solution: æ æ 1ö æ 1ö ö
xç 1+ ç 1- 2 ÷ ÷
 −p p  1 ç çè t 2 ÷ø t ø ÷
f(x) = 7 tan8x + 7 tan6x – 3 tan4x – 3 tan2x ∀ x ∈ , = ò + è dt
 2 2  2 0 ç æ 1 ö2 æ 1ö
2
÷
çç ç t - ÷ + 2 ç t + ÷ - 2 ÷÷
= 7 tan6x ⋅ sec2x – 3tan2x ⋅ sec2x èè t ø è tø ø
= (7 tan6x – 3tan2x) ⋅ sec2x æ 1ö æ 1ö
1+ dt 1-
1 èç t 2 ø÷ èç t 2 ø÷
x x
p /4 p /4 1
= ò + ò dt
⇒ ∫ f ( x )dx = ∫ (7 tan x − 3tan2 x )sec2 dx
6
2 0 æ 1ö 2 2 0 æ 1ö 2
0 0 çè t - ÷ + 2 çè t + ÷ - 2
1
tø tø
= ∫ (7t 6 − 3t 2 )dt = [t 7 − t 3 ]10 = 0 x
  1 
0
1  −1  1   t + t − 2  
Also, = tan  t −  + ln  1 
2 2  t
p /4  t + + 2  
  t   0
I= ∫
0
xf ( x )dx
x
1  −1  t 2 − 1   t 2 + 1− 2t  
p /4
= tan   + ln  2  
= x × ò (7 tan6 x - 3tan2 x )sec2 x dx  t 
0
2 2   t + 1+ 2t   0
p /4

- ò 1× ò (7 tan x - 3tan2 x )sec2 x dx 1 ìï -1 æ x 2 - 1ö æ x 2 + 1- 2t ö æ p ö üï


6
= ítan ç ÷ + ln ç 2 ÷ - ç - + 0÷ ý
è x ø è x + 1+ 2t ø è 2 øï
0
2 2 ïî þ
p /4
p /4
= x ⋅ (tan7 x − tan3 x ) 0 − ∫ (tan x − tan3 x )dx
7

   Therefore,
0

p /4 1 p 1  x 2 − 1 1  x 2 + 1− 2 x 
F( x ) = + tan−1  + ln  2
   = 0 − ∫
0
tan3 x (tan4 x − 1)dx 2 22 2 2  x  2 2  x + 1+ 2 x 

p /4
p 1 1 æ2- 2ö
= − ∫ tan3 x (tan2 x − 1)(sec2 x )dx F (1) = + (0) + ln ç ÷ <1
   4 2 2 2 2 2 è2+ 2ø
0

Mathematical Problem Book for JEE.indb 1047 07-06-2018 14:27:03


1048 Mathematics Problem Book for JEE

y
(A)  1 (B)  2
1 y = f(x) 1
(C)  0 (D)  −
2
x
 2. Let f ( x ) = ∫ 2 − t 2 dt . Then the real roots of the equation
1

x 2 − f ′( x ) = 0 are
x
0 1
y = f(x) (A)  (0, 1) (B)  ±
2
−1 1
(C)  ± (D)  ±1
2
Figure 23.5  3. Let T > 0, be a fixed real number. Suppose f is a continuous
Now, T

f(x) = 2x – 1 ⇒ f(0) = −1 function such that for all x ∈ R, f(x +T ) = f(x). If I = ∫ f ( x )dx ,
and 3+3T
0

f(1) = 2 – 1 = 1 then value of ∫ f (2 x )dx is


Therefore, the total number of distinct values of x ∈ [0.1] is only 3

one. 3
(A)  - I (B)  2I
Hence, the correct answer is (1). 2
p /2 (C)  3I (D)  6I
x 2 cos x
24.  The value of ∫ 1+ e x
dx is equal to 100

òa
x -[ x ]
− ( p /2)  4. The value of dx is
0
p 2
(B)  p + 2
2
(A)  − 2
4 4 100(a - 1) 100(a + 1)
(A)  (B) 
(C)  p − e
2 − ( p /2)
(D)  p + e
2 ( p /2)
log a log a
[JEE ADVANCED 2016] (C)  100(a − 1) (D)  None of these
x

Solution: The given integral is  5. Value of ∫ [sin t ] dt , (2n + 1)p < x < (2n + 2)p , n ∈N
0
is equal to
p /2
x 2 cos x (A)  (n + 1)p p + x (B)  np + x
I= ∫
− p /2
1+ e x
dx
(C)  np − x (D)  (n+1)p − x
p
Using the integral property, we get dx
p /2
 6. The value of ∫ 1+ 2 tan x is equal to
 x 2 cos x x 2 cos x  0
I= ∫ 
 1+ e
x
+  dx
1+ e − x  (A)  0 (B) 
p
0
4
p /2
(C)  p (D)  p
I= ∫x
2
cos x dx
0
2
That is, p /4
sec2 xdx
∫x
2
cos x dx = x sin x − 2 ∫ x sin x dx
2  7.  ∫
− p /4
1+ e x
equals

(A)  2 (B)  0
{
= x 2 sin x − 2 − x cos x + ∫ cos x dx } (C)  –1 (D)  1
= x2sinx + 2x cosx – 2sinx a
Therefore,  8. If f(x) = f (a – x), then ∫ x f ( x ) dx equals
p /2 p 2  p2 0
I = x sin x + 2 x cos x − 2sin x 0
2
= + 0 − 2  − (0) = −2
 4  4 a
a a

2 ∫0
Hence, the correct answer is option (A). (A)  x f ( x ) dx (B)  a∫ x f ( x ) dx
0

a
Practice Exercise 1 (C)  ∫ x f ( x ) dx (D)  None of these
0
 1. If f(x) = x – [x]: for every real number of x, when [x] is the integral
e
1
9.  If l p = ∫ (ln x )p dx , then l p + pl p−1 is less than
point of x. Then ∫ f ( x )dx is equal to
−1
1

Chapter 23.indd 1048 11-06-2018 10:23:35


Chapter 23 | Definite Integration 1049

(A)  1 (B)  2  5p 4p 
on the interval  , is
(C)  3 (D)  None of these  4 3 
p 3 3 1
2 (A)  3+ (B)  −2 3 + +
2 2 2
10.  If ò ln| tan x + cot x | dx = l ln2, then l equals
0 3 1
(C)  + (D)  None of these
p 2 2
(A)  (B)  p
2 ∞

(C)  2p (D)  None of these 18.  ∫0


e −2 x (sin2 x + cos2 x ) dx =
(A)  1 (B)  0
x
æ 1- t ö
11.  If F ( x ) = ò ln ç dt , then F′(−x) + F′(x) equals 1
è 1+ t ÷ø (C)  (D)  ∞
0 2
(A)  0 (B)  e b −c

(C) 
1
(D)  None of these
19.  ∫0
f ′′( x + a) dx =
e (A)  f ′(a) − f ′(b ) (B)  f ′(b − c + a) − f ′(a)
p
4
(C)  f ′(b + c − a) + f ′(a) (D)  None of these
12. ∫ [ sin x + cos x ] d ( x − [ x ]), [.] greatest integer function, equals x
0 20.  The greatest value of the function F ( x ) = ∫ | t | dt on the
1
p p  1 1
(A)  (B)  interval  − ,  is given by
4 2  2 2
3p 3
(C)  (D)  None of these 1
4 (A)  (B)  −
8 2
1 2n r 3 2
13.  lim
n®¥ n
å
r =1 n2 + r 2
equals (C)  -
8
(D) 
5
p /2
(A)  5 + 1 (B)  5 −1 21.  ò- p /2
sin2 x cos2 x (sin x + cos x ) dx =

(C)  2 + 1 (D)  2 −1 2 4
(A)  (B) 
15 15
[x ] 6 8
2x (C)  (D) 
14.   ∫ dx equals (where [.] denotes the greatest integer 15
2[ ]
x 15
0
function) ∞ dx
22.  ∫ =
(x + )
0 3

(A)  ln2 (B) 


[x ] x2 +1
ln2
3 1
1 [x ] (A)  (B) 
(C)  (D)  None of these 8 8
2 ln2
3
xb −1 (C)  - (D)  None of these
1
8
15.  The value of integral ∫ 0 log x
dx is
x3 1
23.  The derivative of F ( x ) = ∫ 2 dt , ( x > 0) is
x log t
(A)  logb (B)  2log(b + 1)
1 1 1
(C)  3logb (D)  None of these (A)  - (B) 
3log x 2log x 3log x
1 d æ -1 1 ö 3x 2
16.  The value of the integral ò-1
ç tan ÷ø dx is
dx è x (C)  (D)  (log x )−1 ⋅ x ( x − 1)
3log x
p p
(A)  (B)  x 2 +1
2 4 24.  If f ( x ) = ò 2 e - t dt , then f ( x ) increases in
2

x
p
(C)  − (D)  None of these (A)  (2, 2) (B)  No value of x
2
(C)  (0, ∞ ) (D)  ( −∞ , 0)
17.  The least value of the function
x4
25.  If f ( x ) = ∫ 2 sin t dt , then f ′( x ) equals
x
F( x ) = ∫
5p /4
(3sin u + 4 cos u ) du x

Mathematical Problem Book for JEE.indb 1049 07-06-2018 14:27:08


1050 Mathematics Problem Book for JEE

(A)  sin x 2 - sin x (B)  4 x 3 sin x 2 - 2 x sin x (A)  (9 x − 4 x )log x


2
(B)  (4 x − 9 x 2 )log x
(C)  x 4 sin x 2 - x sin x (D)  None of these (C)  (9 x 2 + 4 x )log x (D)  None of these
1 x p /2
26.  If F ( x ) =
x2 ∫
4
(4t 2 − 2F ′(t )) dt , then F ′(4) equals 35.  ∫ −p /2
sin4 x cos6 x dx =
32 3p 3p
(A)  32 (B)  (A)  (B) 
3 64 572
32 3p 3p
(C)  (D)  None of these (C)  (D) 
9 256 128
n 1
27.  The value of the integral ∑ ∫ f (k − 1+ x ) dx is 1 d  −1  2 x  

0
36.  sin  2 
dx is equal to
dx 
k =1
1 2
0
 1+ x  
(A)  ∫ f ( x ) dx
0
(B)  ∫
0
f ( x ) dx
(A)  0 (B)  p
n 1
(C)  ∫0
f ( x ) dx (D)  n∫ f ( x ) dx
0
(C)  p /2 (D)  p /4
a+( p /2)
28.  The value of ∫ a
(sin4 x + cos 4 x ) dx is
37.  ò
¥ xdx
=
2
0 (1+ x )(1+ x 2 )
p
(A)  Independent of a (B)  a   (A)  0 (B)  p /2
2
(C)  p /4 (D)  1
3p 3p a2
(C)  (D)  a
8 8
38.  ∫0
x 4 a2 − x 2 dx =
p  x p p 6
29.  ∫ 0
sin5   dx equals
 2 (A) 
32
(B)  a
32
16 32 p 6 p 6
(A)  (B)  (C)  a (D)  a
15 15 16 8
8 5 a
3
(C) 
15
(D) 
6
39.  ò0
x (2ax - x 2 ) 2 dx =

3p  3p 
ïì 1 2 3 n ü (A)  a5  − 1 (B)  a5  + 1
30.  lim í + + ++ ý=  16   16 
n ® ¥ 1 - n2 1- n 1- n
2 2
1 - n2 þ
îï
 3p 1 
(C)  a5  −  (D)  None of these
(A)  0 (B)  - 1 2  16 5 
(C)  1 2 (D)  ∞ a
40.  ∫0
x 2 (a2 − x 2 )3/2 dx =
¥ p a6 2a 5
é2ù (A)  (B) 
31.  ò ê x ú dx is [where [ ] represents the greatest integer func- 32 15
0 ë
e û
a6
tion] (C)  (D)  None of these
32
(A)  0 (B)  2/e
(C)  e2 (D)  loge2 d æ e sin x ö 4 3
÷ ; x > 0. If ò1 e dx = F (k ) - F (1) , then
sin x 3
41.  Let F( x ) = ç
dx è x ø x
32.  Evaluate one of the possible value of k is
p /4
sin2 x cos2 x (A)  15 (B)  16
∫ (sin
0
3
x + cos3 x ) 2
dx
(C)  63 (D)  64
x
42.  If f ( x ) = ∫ t sin t dt , then f ′( x ) =
33.  If f ( x ) = x − m + 1 , x ∈ [ m − 1, m) , (m ∈ I), then evaluate 0

n (A)  cos x + x sin x (B)  x sin x


∫ f ( x ) dx , (n ∈ N).
0
(C)  x cos x (D)  None of these
p
4
sin x + cos x
x 3
43.  Evaluate  ò dx.
34.  If F ( x ) = ∫ 2 log t dt , ( x > 0), then F ′( x ) = 0
cos2 x + sin4 x
x

Mathematical Problem Book for JEE.indb 1050 07-06-2018 14:27:11


Chapter 23 | Definite Integration 1051

x x /2 3p /4
1 dx
∫ ∫ ∫
2 2 2
44.  Prove that e zx e − z dz = e x 4
e − z dz . 57.  is equal to
20 0 p /4
1+ cos x
p /2 1 1
(A)  2 (B)  –2 (C)   (D)  −
45.  Evaluate ò
- p /2
cos x - cos3 x dx. 2 2
x 1
p /4
sin x + cos x 58.  ∫ f (t )dt = x + ∫ t f (t )dt , then the value of f (1) is

0 x
46.  Evaluate dx .
9 + 16 sin2 x 1 1
0
(A)  (B)  0 (C)  1  (D)  −
p p
2 2
p x
47.  Show that ò xf (sin x )dx = 2 ò f (sin x ) dx .
0 0
59.  If g(x) = ò cos 4 t dt, then g(x + p ) equals
0
3/2 (A)  g(x) + g(p ) (B)  g(x) – g(p )
48.  Find the value of ∫ | x sinp x | dx. (C)  g(x) g(p ) (D) 
g( x )
−1
g(p )
 p   e 37
p sin(p lnx )
x 2 sin2 x sin   cos x  dx
49.  Evaluate
p
 2   . 60.  The value of ò dx is _____.

0
(2 x − p )
1
x
4 2

d e sin x 2e sin x
50.  Show that lim 
1
+
1
+
1 1
+ ⋅⋅⋅ +  = ln6.
61. 
Let
dx
F( x ) =
x
, x > 0 if ∫ x
dx = F ( x ) − F (1) then
1
n→∞  n + 1 n+2 n+3 6n 
one of the possible values of k is _____.
1

  1   2   3   n    1 1
2 2 2 n
51.  Find lim   1 + 2   1 + 2   1 + 2    1 + 2   . If for non-zero x, a f(x) + b f   =
62.  − 5, where a ≠ b, then
n→∞
  n   n   n   n   x x
2

52.  Evaluate: ∫
x sin x
p

dx =
ò f ( x ) dx
1
= _____.
0
2 − sin2 x
63. 
Let T > 0 be a fixed real number. Suppose f is a continuous
p2 p2 T
(A)  (B) 
4 2 function such that for all x ∈ R, f(x + T) = f(x). If I = ∫f (x)
0
dx
p p 3+3T

(C)  (D) 
2 4
then the value of ∫
3
f (2 x ) dx is

p
1 + cos 2 x 3
(A)  I (B)  2 I (C)  3 I  (D)  6 I
53.  ∫
0
2
dx = 2
p
(A)  2 (B)  0 (C)  −1 (D)  1 x dx
64.  Evaluate ∫ 1+ cos a sin x , 0 < a < p.
0
32

54.  ∫ sin p x dx = p /2
x sin x cos xdx
−1 65.  Evaluate ∫
0
cos 4 x + sin4 x
.
2 3
(A)  (B)  66.  A function f(x) is such that it is integrable has interval over
p p
every interval on the real line
4 5
(C)  (D)  p /2
p p
∫ sin 2kx
0
7p 8
x dx p /2
55.  ∫ cos2 x − sin2 x
= cot x dx = ∫
(sin2kx )cos x
dx
p 8
0 sin x
p p
(A)  log (3 - 2 2 ) (B)  log (3 − 2 2 ) for every x and real t show that
2 4
p /2
(C)  p log ( 2 − 1) (D)  p log ( 2 + 1)
∫ 2cos x[cos x + cos3 x +  + cos(2k − 1) x ] dx
0
56. 
If for a real number y, [y] is the greatest integer less than or is independent of a.
3p /2


1/2
equal to y, then the value of the integral [2sin x ] dx is x sin−1 x
p /2
67.  Evaluate ∫
0 1− x 2
dx .
(A)  – p (B)  0
e cos x .sin x , x ≤ 2 3
p
(C)  –
2
(D) 
p
2
68.  If f ( x ) = 
2, otherwise
, then
∫−2
f ( x )dx = _______ .

Chapter 23.indd 1051 14-06-2018 10:54:37


1052 Mathematics Problem Book for JEE

n+1 5 (A)  cos2 ⋅ sin3 (B)  sin2 ⋅ cos3


n2
69. If ∫ f ( x )dx =
2
, where n  I, then evaluate ∫ f (| x |)dx .
−3
(C)  sin2 ⋅ sin3 (D)  cos2 ⋅ cos3
n
p /2 6.  Let f(x) and g(x) be two differentiable functions satisfying xf ′(x)
70. Evaluate
∫ 0
lnsin x dx . + f(x) = g(x) where f ′(x) < 0 ∀ x ∈ R. Then
2 3
If f, g, h be continuous function on [0, a] such that f (a  x) =
71. 
f(x), g (a  x) =  g (x) and 3h (x)  4h (a  x) = 5, then calculate
(A)  2g(2) > ò g( x )dx
0
(B)  3g(3) > ò g( x )dx
a 0

ò f ( x ) × g( x ) × h( x ) dx .
2 3

0
(C)  2g(2) < ò g( x )dx
0
(D)  3g(3) < ò g( x )dx
0
p
p 
∫ 7.  If f ( x ) =  tan x  (where [⋅] denotes greatest integer func-
100
72. Evaluate x ⋅ sin 2 x ⋅ sin  cos x  dx .
2 
0 tion), then
(A)  f (x) is discontinuous at x = tan−1 100 3, tan−1 100 100
Practice Exercise 2
(B)  f (x) is discontinuous at x = tan−1 3 100 , tan−1 100 99
Single/Multiple Correct Choice Type Questions tan−1 100 4

3
x 2dx
(C)  ∫ f ( x )dx = 3tan−1 100 4 − tan−1 100 3 − tan−1 100 2 − 1
1.  If ò = I , then the value of 0

2 x4 - x2 +1 tan−1 100 4

3
xdx
1/3
dx
(D)  ∫ f ( x )dx = 3tan−1 100 4 − 2 tan−1 100 3
∫ −∫
0
is equal to
 1
2
1 ¥
2
1+  x − 
1/2
x 3
x + 2 −1
2
8.  If In = ò e - x (sin x )n dx (n ÎN , > 1) , then
 x x
0

(A)  –I (B)  0 n(n − 1) n(n − 1)


(C)  I (D)  2I (A)  In = In−2 (B)  In = In−1
n2 + 1 (n − 1)2 + 1
2.  Let f(x) be a function satisfying f ′(x) = f(x) with f(0) = 1 and g I10 I10
(C)  101 = 90 (D)  82 = 90
I8 I9
be the function satisfying f(x) + g(x) = x. Then the value of
1 ∞
p
ò f ( x )g( x )dx is 9.  If ∫ e − x dx =
2
, then
0 0
2
3-e 2
e −32 ¥
p ¥
1
(A)  (B) 
òe
-2 x 2
dx = ò xe
- x2
2 (A)  (B)  dx =
2 0 2 2
0
e2 e −2
(C)  (D)  ∞
p ¥
p
∫x e òx e
2 − x2 2 - x2
2 4 (C)  dx = (D)  dx =
1/2 0
4 0
4
3.  The value of ò
-1/2
cos -1(4 x 3 - 3 x )dx is equal to 10.  Let

(A)  2p (B)  p x
∫ (5 + 1− t ) dt , if x > 2
p p f(x) =  0
(C)  (D)  5 x + 1,
2 4  if x ≤ 2
p /4
 1 − tan x 
4.  The value of ∫ ln  dx is Then the function is
 1+ tan2 x 
0
(A)  Continuous at x = 2
p  1 3p (B)  Differentiable at x = 2
(A)  ln   (B)  ln2
4 2 8 (C)  Discontinuous at x = 2
3p (D)  Not differentiable at x = 2
(C)  p ln2 (D)  – ln 2
8 8
Comprehension Type Questions
æ 1 æ 1 + n2 ö 2 æ 4 + n2 ö 3 æ 9 + n2 ö
5.  Let lim çç 2 sin ç 2 ÷ + 2 sin ç 2 ÷ + 2 sin ç 2 ÷ + ××× Paragraph for Questions 11–13: Let f: R → R be a continuous
n®¥ n
è è n ø n è n ø n è n ø
and bijective function defined such that f(a ) = 0 (a ≠ 0). The area
2 ö
+ × sin(5) ÷ = y , then the value of y is bounded by y = f(x), x = a, x = a – t is equal to the area bounded by
n ø y = f(x), x = a, x = a + t ∀ t ∈ R then

Chapter 23.indd 1052 11-06-2018 10:25:37


Chapter 23 | Definite Integration 1053

11.  Graph of y = f(x) is symmetrical about point n


(C)  (D)  None of these
(A)  (0, 0) (B)  (0, a ) (C)  (a, 0)  (D)  (a, a) m

12.  The value of f(2a) is equal to Paragraph for Questions 20–22: Let n be a positive integer such
(A)  f(a) (B)  –f(a) that In = ∫ x n a2 − x 2 dx , then answer the following questions:
(C)  f(0) (D)  –f(0) 20.  The value of I1 is
b
13.  The value of ∫f
−1
(t )dt is equal to 2 2 1 2
(A)  (a - x 2 )1/2 (B)  (a - x 2 )3/2
−b 3 3
(A)  0 (B)  2ab
(C)  ab (D)  None of these 2 1
(C)  – (a2 − x 2 )3/2 (D)  – (a2 - x 2 )3/2
3 3
Paragraph for Questions 14–16: Consider a polynomial f(x),
a
which satisfies the following conditions:
∫x a2 − x 2 dx
4

(i)  f ( x ) = {f ′( x )} , ∀x
2
21.  The value of the expression 0
a
is equal to
∫ x a − x dx
1 2 2 2
19
(ii)  ∫ f ( x )dx = 12
0
0

(iii)  f ′(0) > 0 2


a 3a2
(A)  (B) 
14.  The function f(x) can be 6 2
(A)  A linear function (B)  A quadratic function 3a2 a2
(C)  (D) 
(C)  A cubic function (D) Any polynomial of even 4 2
degree
− x n−1(a2 − x 2 )3/2
15.  The value of f ′(0) is 22.  If In = + kIn−2 , then the value of k is
n+2
1 1
(A)  0  
(B)  (C)   (D)  1 n −1 n+2
4 2 (A)  (B) 
n+2 n −1
16.  The function f(x) is
æ n - 1ö 2 n + 2ö 2
(A)  Even (B)  Odd (C)  ç a (D)  æç
(C)  Neither even nor odd (D) May be either even or è n + 2 ÷ø è n - 1 ÷ø
a
odd
Matrix Match Type Questions
Paragraph for Questions 17–19: If m > 0, n > 0, the definite
1
23.  Match the following:
∫x
m−1
integral I = (1− x )n−1 dx depends upon the values of m and n
0
Column I Column II
and is denoted by b (m, n), called the beta function. That is,
1 1 1 æ 2cos x - 3sin x ö dy (p)  2p
(A)  If y = cos -1 ç ÷ , then is
∫x (1− x )5 dx = ∫ x 5−1(1− x )6−1 dx = b (5, 6) and ∫x (1− x )−1/2 dx
4 5/2
è 13 ø dx
0 0 0
1
 7 1
=∫x 7/2−1
(1− x ) 1/2−1
dx = b  ,  . Obviously, b (n, m) = b (m, n). (B)  The value of (q) 0
 2 2 1 1
0 é 2 1ù -1 é 2 1ù
ò-1sin êë x + 2 úû dx +-ò1cos êë x - 2 úû dx is equal
-1
p /2

∫ cos q sin2 nq dq is equal to


2m
17.  The integral
0 to, where [⋅] denotes the greatest integer function
1  1 1 (r)  p/3
(A)  b  m + , n +  (B)  2b (2m, 2n) 3
dx
2  2 2 (C)  The value of ò (1+ x 2 )(1+ e sin x ) is equal to
- 3
(C)  b (2m + 1, 2n + 1) (D)  None of these
6p (s) 1
ò sin
n -1
n
 x  k −1 (D)  The value of (sin x )dx is equal to
18.  If ∫  1−  x dx = Rb (k , n + 1), then R is equal to
0
 n 0

(A)  n (B)  nkn


24.  Match the following:
(C)  nk (D)  None of these
∞ ∞
Column I Column II
x m−1 x n−1
19.  If ∫ dx = k ∫ dx , then k is equal to p /2
1
(1+ x ) m+ n
(1+ x )m+n dx
0

m
0
(A)  ∫ 1+ tan x
0
(p) 
117
(A)  (B)  1
n

Chapter 23.indd 1053 14-06-2018 12:51:05


1054 Mathematics Problem Book for JEE

Column I Column II 26.  Match the following:

x 2 (1+ x 5 +7 x12 ) p Column I Column II


(q)  − log2
(B)  If ∫ f (t ) dt = x, then f(9) is equal to 2 p p (p) is equal to the
0
(A)  ∫ xf (sin x )dx : p ∫ f (sin x )dx
0 0
ratio 1:1
¥
1
(r) 
ò e (sin2 x + cos2 x )dx is equal to
-2 x
(C) 
2 2 p p /2 (q) is equal to the
ò xf (sin x )dx : p ò f (cos x )dx
0
(B)  ratio 1:2
1 p 0 0
(s) 
∫ cot
−1
(D)  (1+ x 2 − x ) dx is equal to
4 k k (r) is equal to the
0
(C) 
ò xf ( x (1- x ))dx : ò f ( x (1- x )) dx
1-k 1-k
ratio 2:1
25.  Match the following:
1+ cos2 t cos2 t (s) is equal to the
Column I Column II (D)  ∫ xf ( x (2 − x ))dx : ∫ f (1− x 2 ) dx ratio k:1
(A)  For any integer n, the integral p sin2 t 0
p (p) 
2
∫e
2
cos t
cos3 (2n + 1)tdt is equal to
0 Integer Type Questions
p /4 (q) 0
(B)  If In = ∫ tan q dq ,
n
then for any 1
2p

òe
cosq
0 27.  If I = cos(sinq )dq , then find the value of I.
p
positive integer n, n(In – 1 + In + 1) = 0
p
p2
¥ (r) 1 28.  Let f : R → R defined by f(x) = cosx + x if ò f -1( x )dx = - k, then
x log x 2
(C)  The value of ò (1+ x
0
2 2
)
dx is the value of k is ________. 0

1
æ pö   3
d æ 1ö
(D)  The value of ò ç tan-1 ÷ dx is
1
(s) –
p 29.  If ò t 2f (t )dt = 1- cos x " x Îç 0, ÷ , then the value of f 
è 2 ø 
  4  
dx è xø 2 cos x
-1
is _______ (where [⋅] denotes the greatest integer function).

Answer Key

Practice Exercise 1
1. (A) 2. (D) 3. (C) 4. (A) 5. (D) 6. (C)
7. (D) 8. (D) 9. (C) 10. (B) 11. (A) 12. (A)
13. (B) 14. (B) 15. (D) 16. (C) 17. (B) 18. (C)
19. (B) 20. (C) 21. (B) 22. (A) 23. (D) 24. (D)
25. (B) 26. (C) 27. (C) 28. (C) 29. (A) 30. (B)
1
31. (D) 32. 33. 2n 34. (A) 35. (C) 36. (C)
6 3
37. (C) 38. (B) 39. (C) 40. (A) 41. (D) 42. (B)
p 1  3 − 1 4 1 3 1
43. − log   45. 46. ln3 48. +
4 3  2  3 20 p p2
p −4
16
49. 50. ln6 51. 2e 2
52. (A) 53. (A) 54. (D)
p
55. (B) 56. (C) 57. (A) 58. (A) 59. (A) 60. 2

1  7b  p2
61. k = 16 62. a ln2 + − 5a  63. (C) 64. pa cosec a 65.
a − b 2 
2
2  16
6 −p 3 35
67. 68. 2 69. 70. −p ln 2 71. 0 72. 8
12 2 2 p

Chapter 23.indd 1054 14-06-2018 12:52:15


Chapter 23 | Definite Integration 1055

Practice Exercise 2
1. (D) 2. (A) 3. (C) 4. (D) 5. (C) 6. (C), (D)
7. (A), (C) 8. (A), (C) 9. (B), (D) 10. (A), (D) 11. (C) 12. (D)
13. (B) 14. (B) 15. (D) 16. (C) 17. (A) 18. (C)
19. (B) 20. (D) 21. (D) 22. (C) 23. (A) → (s), (B) → (p), (C) → (r), (D) → (q)
24. (A) → (s), (B) → (p), (C) → (r), (D) → (q) 25. (A) → (q), (B) → (r), (C) → (q), (D) → (s) 26. (A) → (q), (B) → (p), (C) → (q), (D) → (r)
27. 2 28. 0 29. 5

Solutions

Practice Exercise 1 =
a − 1 a2 − a1 a3 − a2 … a100 − a99
+ + + + 99
log a a log a a2 log a a log a
1 1 1 1 0 1
 1.  ò ( x - [ x ]) dx = ò x dx - ò [ x ] dx = ò x dx - ò [ x ] dx - ò [ x ] dx 100(a - 1)
=
-1 -1 -1 -1 -1 0 log a
0 1 1
 x2 
=   − ∫ ( −1) dx − ∫ (0) dx x
 2  −1 −1 0  5.  ∫ [sin t ]dt (( 2n +1)p < x < (2n+2) p)
0

= 0 + ( x )-1 - (0 )0 = 1- 0 = 1
0 1 p /2 p 3p / 2 2p

ò 0 × dt + ò 0 × dt + ò
0 p /2 p
-1× dt + ò ( -1) dt + 
3p
 2.    x 2 - f ¢( x ) = 0 2
p 2p 3p 4p x

Þ x2 - 2 - x2 - 0 = 0 = ∫ 0 ⋅ dt − ∫ 1⋅ dt + ∫ 0 ⋅ dt + ∫ −1dt + … + ∫ −1dt
0 p 2p 3p (2 n+1)p
⇒ x2 = 2 − x2 = −p − p − …n times − (n)(x2 n+1)p

⇒ x4 = 2 − x2 = − np − ( x − (2n + 1)p ) = − x + (n + 1)p


p p
Þ x4 + x2 - 2 = 0 dx dx
 6.  I=∫ =∫ − tan x
1 + 2 tan x
1 + 2
⇒ x4 + 2x2 − x2 − 2 = 0 0 0
p
p
Þ ( x + 2)( x - 1) = 0
2 2 ⇒ 2I = ∫ dx = p ⇒ I =
0
2
⇒ x = 1, − 2 2
p /4 p /4
sec2 x 1 (1+ e x ) + (1− e x )
⇒x=±1
 7.    ∫
− p /4
1 + e x
dx = ∫
2 − p /4
(1 + e x
)
⋅ sec2 xdx

p /4 p /4
3+3T 3T T 1 1 1− e x
 3.    ∫ f (2 x ) dx = ∫ f (2 x ) dx = 3∫ f (2 x ) dx   = ∫
2 − p /4
sec2 xdx + ∫
2 − p /4 1+ e x
sec2 xdx
3 0 0
1
Let 2x = t. Then = (tan x )p−p/4/4 + 0
2
2T T
3  1− e − x 1− e x 
2 ∫0
f (t ) dt = 3∫ f (t ) dt = 3I  since, f ( − x ) = sec2 ( − x ) = − sec2 x = − f ( x ) 
−x
  0  1+ e 1+ e x 
1
100 = ⋅2 =1
2
∫a
x −[ x ]
  4.    dx
0
a

100 x
 8.     I = ∫ (a − x )f (a − x )dx
a
= ∫a
0
[x]
dx 0
a a
= a ∫ f ( x )dx − ∫ x f ( x )dx
1 2 3 100
ax ax ax ax 0 0
=ò 0
dx + ò dx + ò 2 dx +  + òa 99
dx a
0
a 1
a 2
a 99 ⇒ 2I = a ∫ f ( x ) dx
0
1 2 3 100
æ ax ö æ ax ö æ ax ö æ ax ö a
a
=ç ÷ +ç ÷ +ç 2 ÷ +  + ç 99 ÷
2 ∫0
⇒I = f ( x )dx
è log a ø0 è a × log a ø1 è a log a ø2 è a log a ø99

Mathematical Problem Book for JEE.indb 1055 07-06-2018 14:27:30


1056 Mathematics Problem Book for JEE

e
1 1 1
 9.  IP = ∫ (ln x ) dx
p = + + +  + [ x ] times
ln2 ln2 ln2
1
e e [x]
IP + pIP −1 = ∫ (ln x )p dx + p ∫ (ln x )p−1dx =
ln2
1 1

e p
e e
15.  Let
=  x ⋅ (ln x )p 1 − ∫ (ln x )p−1 ⋅ xdx + p ∫ (ln x )p−1dx = e
x
1 1 1 xb -1 b
1 x log x
which is less than 3. I (b ) = ò dx Þ I '(b ) = ò dx
0 log x 0 log x

b b
p
2 p /2 p /2
(If I (a ) = ∫ f ( x , a )dx , then I '(a ) = ∫ f ′( x , a )dx , where f ′( x , a )
sin2 x 0 0
10.  I = ∫ ln| tan x + cot x | dx = − ∫ ln dx I = −2 ∫ lnsin x dx
2 is derivative of f ( x , a ) w.r.t. a keeping x constant)
0 0 0
p /2 1 1
  = − ∫ [ln| sin2 x | − ln2] dx I ′(b ) = ∫ x b dx =
0 b +1
0
db
p /2
p p ⇒ I (b ) = ∫ + c = log(b + 1) + c
l = -2I1 = - I1 + ò ln2 dx = -I + 2 log 2
0
1 e Þ I1 = -
2
ln2 b +1
If b = 0 , then I (b ) = 0. So,
Hence,
c = 0 ⇒ I (b ) = log(b + 1)
 p 
−2I1 = −2  − ln2 
 2 
⇒ I = p ln2 1 d  −1 1  p

−1
 tan  dx = −2[tan ( x )]0 = −
1
16. 
x
−1 dx x 2
æ 1- t ö
11.  F ( x ) = ò ln ç dt
0
è 1+ t ÷ø 17.  We have
æ 1- x ö æ 1+ x ö F ′( x ) = 3sin x + 4 cos x
Þ F ¢( x ) + F ¢( - x ) = ln ç + ln ç =0
è 1+ x ÷ø è 1- x ÷ø
é 5p 4p ù
p /4 p /4
Since in ê , ú , F ¢( x ) < 0, so assume the least value at the
p /4
p ë 4 3 û
12.  ∫ 1⋅ d ( x − [ x ]) = ∫ d { x } = [{ x }] = 4p
0 0
0
4 point x = . Thus,
3
1 2n r
13.  lim
n®¥ n
å æ 4p ö
r =1 n + r2
2
f ç ÷ =ò
4 p /3
(3sin u + 4 cos u )du
è 3 ø 5 p /4
1 2n r /n
lim =
n®¥
å
n r =1 1 + (r/n)2 3 1
= −2 3 +
2 2 2
x
=ò dx ∞
18. 
0 1+ x 2 ∫ 0
e −2 x (sin2 x + cos2 x )dx
Let 1+ x 2 = t 2. Then xdx = tdt ¥
é cos2 x ù ¥ æ - sin2 x ö ¥
= ê - e -2 x ú - ò ( -2e -2 x ) ç ÷ø dx + ò0 e sin2 x dx
-2 x
5
t ë 2 û0 0 è 2
ò t dt = (t ) = 5 -1
5
1
1
1
=
2
[x]
2x
14.  ∫2
0
[x]
dx
19.  ∫ 0
b −c
f ″( x + a)dx = [f ′( x + a)]b0−c = f ′(b − c + a) − f ′(a)
1 2 3
2x 2x  1 1
= ∫ 2 x dx + ∫ dx + ∫ 2 dx + … 20.  F ′( x ) =| x |> 0∀x ∈  − , 
0 1
2 2
2  2 2
 1 1
æ 2x ö 1 æ 2x ö ¼
1 2
Hence, the function is increasing on  − ,  and therefore
=ç + +  2 2
è log2 ÷ø 0 2 çè log2 ÷ø 1  1 1
F ( x ) has maxima at the right end point of  − ,  . So,
1 1 0 1 1  2 2
= (2 - 2 ) + (22 - 21 ) + 2 (23 - 22 ) + ¼
ln2 2ln2 2 log2  1  1/2 3
Max F ( x ) = F   = ∫ | t | dt = −
2
  1 8

Mathematical Problem Book for JEE.indb 1056 07-06-2018 14:27:34


Chapter 23 | Definite Integration 1057

p /2 Therefore,
21.  ò- p /2
sin2 x cos2 x (sin x + cos x )dx
n k 1 2 n
p /2 p /2 ∑∫ f ( x )dx = ∫ f ( x )dx + ∫ f ( x )dx + … + ∫ f ( x )dx
= ∫− p /2 sin x cos xdx + ∫− p /2 sin x cos x dx
3 2 2 3 k −1 0 1 n−1
k =1
n
p /2 2 4 = ∫ f ( x ) dx
= 0 + 2∫ 2
sin x cos xdx = 0 + 2 × =3 0
0
15 15 p
28.  Since sin4 x + cos 4 x is a periodic function with period ,
2
22.  Putting x = tanq , we get
therefore
¥ dx p /2 sec q dq 2
p /2 cosq a+( p /2)

ò0
( x + x + 1) 2 3
=∫
0 (tanq + secq )3 ∫0 (1+ sinq )3
= dq ∫a
(sin4 x + cos 4 x ) dx
p /2
 1  1 1 3
p /2
=∫ (sin4 x + cos 4 x )dx
= − 2
=− + = 0
 2(1 + sin q ) 0 8 2 8 p /2 3G(5/2)G(1/2) 3p
= 2ò sin4 x dx = =
0 æ 4+0+2ö 8
23.  We know that 2G ç ÷
è 2 ø
d
dx (∫ f (t )dt ) = dbdx f (b) − dadx f (a)
b

a
p x p /2
6 1
G .G 16
29.  ò sin dx = 2ò sin tdt = 2 × 2 2 =
5 5
a and b are functions of x. 0 2 0 7 15
2G
Therefore, 2

30.  lim  1 + 2 + 3 +  + n 
x3 1
F( x ) = ∫ 2 dt
x log t n→∞
 1− n 1− n 1− n
2 2 2
1 − n2 
d 3 1 d 1
n
r 1 n
1r 
⇒ F ¢( x ) = (x ) - (x2 ) = lim ∑ = lim ∑  
1 − n 2 n→∞  1  n
dx log x 3 dx log x 2 n→∞
n
 2 − 1
r =1 r =1

 n 
3x 2 2x 1
= - = x ( x - 1)(log x )-1 1 1 1 -1
ö ò0
3log x 2log x = lim x dx = lim ´ =
n®¥ æ 1 n®¥ 2 æ 1 ö 2
ç 2 - 1÷ ç 2 - 1÷
24.  f ¢( x ) = e - ( x
2
+1)2 2 2 4
+1+2 x 2 ) 2 èn ø èn ø
× 2 x - e -( x )
× 2 x = 2 xe - ( x (1- e 2 x +1 )

⇒ f ¢( x ) > 0, "x Î( -¥ ,0) 2
31.  ∫  x  dx
0 e 
25.  We have
2 é2ù
x4   x ∈ (0, loge 2 ) , varies from 2 to 1 and hence ê x ú = 1
f ( x ) = ∫ 2 sin t dt e x
ëe û
x
2 2
Therefore,   x ∈ (loge 2 , ¥ ), x varies from 1 to 0 and hence  x  = 0
e e
 
d 4 d
f ′( x ) = ( x )(sin x 4 ) − ( x 2 )(sin x 2 )
dx dx Therefore,
∞ loge 2 ∞
2
= 4 x 3 sin x 2 − 2 x sin x ∫  e
0
x 

dx = ∫
0
1 dx + ∫
loge 2
0 dx
26.  We have
  = [x ]0
loge 2
+ 0 = loge 2
1 x
F ( x ) = 2 ò (4t 2 - 2F ¢(t ))dt p /4 p /4
x 4 sin2 x cos2 x tan2 x sec2 x
Therefore,
32.  ∫
0
(sin3 x + cos3 x )2
dx = ∫
0
(1 + tan3 x )2
dx ,

1 2 x (Dividing numerator and denominator by cos6x.)


F ′( x ) = (4 x 2 − 2F ′( x )) − 3 ∫ (4t 2 − 2F ′(t ))dt
x2 x 4 Put z = 1 + tan3x. Therefore, dz = 3tan2x sec2x dx
p
1 32 When x = 0, z = 1 and when x = , z = 2
⇒ F ′(4) = [64 − 2F ′(4)] − 0 ⇒ F ′(4) = 4
16 9 Therefore,
27.  Let p /4 2 2
sin2 x cos2 x 1 dz  1  −1  
1
I = ∫ f (k − 1+ x )dx ò
0
(sin3 x + cos3 x )2
dx = ò
1
3 z 2
=  
 3  z  1
0
1 1 1
k
⇒ I = ∫ f (t ) dt , where t = k − 1+ x = − =
k −1 3 6 6

Mathematical Problem Book for JEE.indb 1057 07-06-2018 14:27:35


1058 Mathematics Problem Book for JEE

n n m n m
Now again, put 2q = j . Therefore,
33.  ∫ f ( x ) dx = ∑ ∫ f ( x ) dx = ∑ ∫( x − m + 1) dx
 3p 1 
a5 é ò sin4 j dj - ò sin4 j cosj dj ù = a5  − 
p /2 p /2
0 m=1 m−1 m=1 m−1
êë 0 0 úû  16 5 
n
2 m n
2 2n
= å ( x - m + 1)3/2 =å = a
m=1 3
m-1
m=1 3 3 40.    I = ∫ x 2 (a2 − x 2 )3/2 dx
0

34.  F ( x ) = ∫ 2 log t dt
x3 Put
x x = a sinq Þ dx = a cos q dq
Applying Leibnitz’s theorem, p /2
I=∫ a2 sin2 q ⋅ a3 cos3 q ⋅ a cosq dq
d d 0
F '( x ) = log x 3 ⋅ x 3 − log x 2 ⋅ x 2
dx dx 3 5
p /2
G ×G
= 3log x × 3 x 2 - 2log x × 2 x = (9 x 2 - 4 x )log x 2 2
=a ò sin q cos q dq = a
6 2 4 6
0 8
2×G
p /2 p /2
2
35.  I = ∫ sin4 x cos6 x dx = 2∫ sin4 x cos6 x ⋅ dx
− p /2 0
1 3 1
a a
× p × × × p p a6
since, ò f ( x ) dx = 2 ò f ( x ) dx , if f ( - x ) = f ( x ) =a 2 6 2 2 =
-a 0
2 × 3 × 2 ×1 32
= 0, if f ( - x ) = -f ( x )
Applying Gamma function, we get 3 sin x 3
4 4 3x
2
41. 
ò e dx = ò
3

3
e sin x dx
x 1 1 x
2 G5 / 2 ×G7/ 2
I= Put
2 × G6
3/2 ×1/ 2 × p × 5/2 × 3/ 2 ×1/ 2 × p 3p 3p x 3 = t ⇒ 3 x 2dx = dt
= = 8 = 64 e
sint
64 d
5 × 4 × 3 × 2 ×1 2 256 F (t ) = ò dt = ò F (t )dt = F (64) - F (1),
1 t 1 dt
1
  2 x  p On comparing, we get
36.  I = sin−1  2 
= sin−1(1) − sin−1(0) =
  1+ x   0 2 k = 64
1 æ1 1ö x
42.  Since, f ( x ) = ò t sin t dt . Now, according to Leibnitz’s rule,
xdx - dx ç x + ÷ø
¥è2
¥ ¥
2 + 2 0
37.  ò 0 (1+ x )(1+ x 2 ) ò0 (1+ x ) ò0 1+ x 2
= dx
f ′( x ) = x sin x ⋅ (1) − 0 = x sin x

 −1  1 1 1 d
=  log(1+ x ) + × [log(1+ x 2 )] ∞0 + [tan−1 x ] ∞0 43.  Observe that sin x + cos x = (sin x − cos x )
 2 0 2 2 2 dx
Therefore, express the denominator as a function of
1 p  p sin x − cos x .
= 0 + 0 +  − 0 =
22  4 2
38.  Put 1 + cos 2 x  1 − cos 2 x 
cos2 x + sin4 x = + 
2  2 
x = a sinq Þ dx = a cos q dq 3 cos2 2 x 4 − sin2 2 x
= + = = 1 − sin2 x cos2 x
Now 4 4 4
a p /2 2
ì1 - (sin x - cos x )2 ü
∫0
x 4 a2 − x 2 dx = a6 ∫
0
sin4 q cos q cos q dq = 1- í ý
î 2 þ
 5  3
Γ   .Γ  
p /2  2  2 p 6 Let z = sin x − cos x ; dz = (cos x + sin x ) dx
= a6 ∫ sin4 q cos2 q dq = a6 = a
0 2Γ 4 32 When x = 0, z = –1, when x = p /4, z = 0
(Using gamma function) 0
dz
0
4 dz
I=ò 2

39.  Put -1 æ 1- z2 ö -1
22 - (1 - z 2 )2
1- ç
x = a(1− cos2q ) ⇒ dx = 2a sin2q dq è 2 ø÷
0
Therefore, 4 dz

a -1
(3 - z 2 ) (1 + z 2 )
ò 0
x (2ax - x 2 )3/2 dx
0
 1 1 
p /4 =∫  +  dz
=∫ 2a (1− cos2q )sin 2q dq
5 4
−1 1 + z
2
3 − z2 
0

Chapter 23.indd 1058 11-06-2018 10:29:23


Chapter 23 | Definite Integration 1059

é 1 3 + zù
0
 
= ê tan-1 z + log 1 2  1 1 
1
ú 1 dy 1

ëê 2 3 3 - z úû -1 = − ∫ 2 = − × ∫ −  dy
16 0  5 16 5 0   5  5 
ìï 1 æ 3 - 1 ö üï
y2 −  
 4   y − 4   y + 4  
= 0 - ítan-1 (-1) + log çç ÷÷ ý
ïî 2 3 è 3 + 1 ø ïþ 1   5  5 
1

=− ln  y −  − ln  y +  
p 1  3 − 1 p 1 ( 3 − 1)2 40   4  4 0
= − log   = − log
4 2 3  3 + 1 4 2 3 2 5
1

æ 3 - 1ö 1 y+ 4 1 1 1
p 1 = ln   = ln 9 = ln32 = ln 3
= - log çç ÷÷ 40  y − 5  40 40 20
4 3 è 2 ø
 4 0
x + 2u Therefore,
44.  Put z = . Therefore, dz = du.
2 p /4
 sin x + cos x  1
x
z = 0 ⇒ u = − and z = x ⇒ u =
x ∫  9 + 16 sin2 x  dx = 20 ln3
0
2 2
Therefore, 47.  Let
p p
x /2 æ x +2 u ö æ x +2 u ö
1 çè ÷ x-
2 ø çè ÷
I = ∫ xf (sin x )dx = ∫ (p − x )f {sin(p − x )}dx
LHS = I =
2 ò x
e 2 ø .du
0 0
u =-
2 p p p

x = p ∫ f (sin x )dx − ∫ xf (sin x ) dx = p ∫ f (sin x ) dx − I


x /2 x 2 − 4 u2 2
1 1 x 2 0 0 0

2 − ∫x /2 ∫e
− u2
= e 4
du = e 4 du p
p
p

2 x ⇒ 2I = p ∫ f (sin x ) dx or I = ∫ f (sin x ) dx

2 0
20
x x

1 x 2
2 x2 2 x /2 48.  sin x is positive for 0 ≤ x ≤ p
∫e
2
− z2
∫e ∫e
− u2 − u2
du = e
x /4
= ⋅ 2e 4 du = e 4 dz = RHS sin p x is positive for 0 ≤ p x ≤ p, that is, 0 ≤ x ≤ 1
2 0
0 0
For −1 ≤ x < 0, −p ≤ p x < 0, for such x, sin p x < 0.
p /2
 3  3p 
For 1 < x ≤   , p ≤ p x ≤   ⇒ sin p x < 0
45.  I = ∫ p
cos x − cos3 x dx  2  2

2 Therefore,
p /2
−1 ≤ x < 0,    x sinp x > 0
=2 ∫
0
cos x − cos3 x dx (as cos x is an even function)
0 ≤ x ≤ 1,    x sinp x > 0
p /2 3
1 < x ≤   x sinp x< 0
=2 ∫ cos x sin2 x dx 2
0
Therefore,
p /2 3 3
=2 ∫ cos x sin x dx (since sin x is positive in (0, p / 2)) 2 1 2

0 ∫ x sinp x dx = ∫ x sinp x dx + ∫ ( − x sinp x )


−1 −1 1
p /2
 2 4
=  − 2 (cos x )3/2 ⋅  = 1
3
2
 3 0 3
= 2 ∫ x sinp x dx − ∫ x sinp x dx
0 1
p /4
(sin x + cos x ) 3 3
46.  I = ∫ 9 + 16 sin2 x
dx   − cos p x   sinp x  1 1
  − cos p   2  sinp x  2
0 = 2x 
  +  2 

 − x   + 2 
 p 0 p    p  1  p  1
0
Put y = sin x − cos x
dy = (cos x + sin x) dx  1   1  1  3 1
= 2   − −   −  2   = + 2
y2 = sin2 x + cos2 x − 2 sin x cos x = 1 − sin 2x p   p  p  p p
Therefore, sin 2x = 1− y2
3p
(Since, sin = −1 and cos p = −1 )
x=p ,y=0 2
4
Therefore, 49.  Put
0
dy p
∫ 9 + 16(1− y 2 ) dx
−1
x =   − y ⇒ dx = − dy
 2

Mathematical Problem Book for JEE.indb 1059 07-06-2018 14:27:39


1060 Mathematics Problem Book for JEE

−p p 1
and when x = p , y = or when x = 0, y = . = log 2 −  2 x − 2 tan − 1 x 
2 2 0
2  2p  p −4
p   p   = log 2 −  2 − = log 2 +
4 
−p
 − y  sin2 y sin    sin y   2
1 2
2  2 
I=
2 ∫ (− y )
( − dy )
log 2+
p −4 p −4
p
⇒ A=e 2
= 2e 2
2

p 2 2 p  p
x sin x
p
(p − x ) sin (p − x )
I=∫ dx = ∫
p

−1 2  4 + y  sin2 y sin  2 sin y  52.  dx,


  2 − sin 2
x 2 − sin2 (p − x )
2 −∫p
= dy 0 0
y  a a

2
 using ∫ f ( x ) dx = ∫ f (a − x ) dx 
p p   0 0 
2 sin2 y sin  sin y 
 2  dy p
sin x
+ ∫ py ⇒ 2I = p ∫ dx
−p y 2 − sin2 x
0
2
Therefore,
= −I1 + I2
p p
I1 = 0 (Since integrand is odd) p − d (cos x )  p 
2 ∫0 1+ cos2 x  2
I= = − tan−1(cos x )
Therefore, 0
p
 −p   −p  p  p  p
2
2
 p   = + =
I = 2p ∫ 2sin y cos y sin   sin y  dy  2   4  2  4  4
0

 2  
p p p
Put 1+ cos 2 x
53. ∫ 2
dx = ∫ cos2 x dx = ∫ | cos x | dx
p p 0 0 0
  sin y = t ⇒ cos y dy = dt
2 2 p /2 p

p
 p

= ∫ cos x dx + ∫ − cos x dx
0 p /2
2 2
2t 2 16  
⇒ l = 2p × 2 ∫ ⋅ sin tdt = {t ( − cos t )}0 − ∫ ( − cos t )dt 
p /2
= [ sin x ]0 − [ sin x ]p /2 = 1− ( −1) = 2
p /2 p
p p p
0  0 
 
16 54.  See Fig. 23.6.
=
p 3/2 0 1 3/2

50.  lim 
1 1 1 ∫ | sin p x | dx = ∫ − sin(p x )dx + ∫ sin(p x ) dx + ∫ − sin(p x ) dx
n→∞ 
+ + +  −1 −1 0 1
n +1 n + 2 6n 
0 1 3/2
 1 1 1  5n
1  cos p x   − cos p x   cos p x 
=  +  + p 
= lim 
n→∞  n + 1
+
n+2
+ +  = lim
n + 5n  n→∞

r =1 n + r
 p  −1  p 0  1
5
dx 1 5
= ∫ 1+ x = ln(1+ x ) 0 = ln 6
5 = {1− ( −1) + 1− ( −1) + 0 − ( −1)} =
0
p p

51.  Let x
1 −1 3/2
 1  22   32   n2  
A = lim   1 + 2 
n

 1 + n2   1 + n2   1 + n2  
n→∞
  n  
px
n
1  r2  π
⇒ loge A = lim ∑ n
log  1 + 2 
−p
0
3p /2
n→∞
r =1  n  Graph of sin p x
1 n  r
2
 1 Figure 23.6
= lim
n→∞ n
∑ log  1 +  
 n
 = ∫ log (1 + x ) dx
2

r =1   0
7 p /8 7 p /8
x dx p−x
= [ x log (1 + x )] − ∫
x ⋅ 2 x dx
2 1
1
55.  I = ∫
p /8
cos x − sin2 x
2
=
p /8
∫ cos 2 (p − x )
dx
0
0
1+ x 2
7p /8
1
2 (1 + x 2 ) − 2 (p − x ) dx 7 p /8
dx
= 1log 2 − ∫ dx = ∫ cos 2 x
=p ∫ cos 2 x
−I
0
1+ x 2 p /8 p /8

Mathematical Problem Book for JEE.indb 1060 07-06-2018 14:27:41


Chapter 23 | Definite Integration 1061

Therefore, upper limit = 37p


lower limit = 0
p 37 p
2I = [log (sec 2 x + tan 2 x )]p7p/8/8
∫ sin t dt = − cos t
37 p
2 I= 0
0
p   7p 7p   p p  = − {cos 37p − cos 0} = 1 + 1 = 2
I =  log  sec + tan  − log  sec + tan  
4  4 4  4 4  I=2
p p 4
2e sin x
2

= [ log ( 2 − 1) − log ( 2 + 1) ] = log (3 − 2 2 ) dt


4 4 61.  I=∫ dx Let x2 = t ⇒ x dx =
1 x 2
3p /2 16 16
2e sint d 
56.  ∫ [2sin x ] dx, taking points where for 2 sinx = ± 2, 0, ± 1 = ∫ 1 2 t
dt = ∫  F (t )  dt
1 dt 
p /2
16
5p /6 p 7p /6 3p /2
= F (t ) 1 = F (16) − F (1)
I= ∫
p /2
dx + ∫
5p /6
0 dx + ∫
p
( −1) dx + ∫
7p /2
( −2) dx
Hence, possible value of k = 16.
5p p 7p 7p
= − +p − + − 3p  1 1
6 2 6 3 62.  af(x) + bf   = − 5 (1)
x x
5p − 3p + 6p − 7p + 14p − 18p 3p p
= =− =−
6 6 2  1
Replacing x by   ,
3p /4 x
3p /4
dx 1 x
3p /4
 x 3p p
57.  ∫ = ∫ sec2 dx =
1+ cos x 2 p /4 2  tan 2 
  p /4
= tan
8
− tan
8  1
p /4 af   + bf(x) = x − 5 (2)
x
=
1
−1+ 2
− −1+ 2  ( )  p 
since tan 8 = −1+ 2 
 
By Eqs. (1) and (2),
1 a 
( ) f(x) =  − bx + 5(b − a)
2
1− −1+ 2 a − b2  x
2

=
−1+ 2 Then
2
−2 + 2 2 2 1 a 
=
−1+ 2
=2
∫ f ( x )dx = a − b2
2 ∫
1
 − bx + 5(b − a) dx
 x 
1
2
58.  Differentiate both side w.r.t. x 1  bx 2 
= 2  a ln x − + 5(b − a) x 
f(x) ⋅1 − f(0) ⋅ 0 = 1 + 1⋅ f(1) ⋅ 0 − x⋅f(x) ⋅1 a −b  2
2 1
1 1
⇒ (x + 1) f(x) = 1 ⇒ f ( x ) = b
x +1 = 2 2 a ln2 − 2b + 10(b − a) + − 5(b − a)
At x = 1, we have a −b  2 
1 1 1  3  1 7b
f(1) = = = a ln2 + 5b − 5a − b  = 2 2 a ln2 + − 5a 
1+ 1 2 a2 − b 2  2  a −b  2 
x +p x x +p 3+3T 6+6 T 6T T
dt 1
59.  g( x + p ) = ∫
0
cos 4 t dt = ∫ cos 4 t dt +
0

x
cos 4 t dt 63.  ∫ f (2 x ) dx = ∫ f (t )
2 2 ∫0
= f (t )dt = 3∫ f (t ) = 3l
3 6 0
p p+x p
(p − x )
= g( x ) + ∫ cos t dt + ∫
4
cos 4 t dt
x p
64.  I= ∫ 1+ cosa sin (p − x ) dx
0
p x x

= g( x ) + ∫ cos t dt − ∫ cos t dt + ∫ cos t dt


4 4 4 p p
p x dx
0 0 0 = ∫ 1+ cosa sin x dx − ∫ 1+ cosa sin x dx
0 0
p+x

By taking t = x + p  in ∫ cos 4 tdt p


p
p ⇒2I = ∫ 1+ cosa sin x dx
g(x+ p) = g(x) + g (p ) 0

x
p p sec2
e 37 p 1 p 2
p sin(p lnx ) I= ∫ dx = ∫ dx
60.  I= ∫ 1
x
dx 2 0 1+ cos a sin x 2 0 1+ tan2 x + 2 tan x cos a
2 2
Let Let
p x 1 x
p ln x = t ⇒ dx = dt tan = t ⇒ sec2 dx = dt
x 2 2 2

Mathematical Problem Book for JEE.indb 1061 07-06-2018 14:27:43


1062 Mathematics Problem Book for JEE

∞ ∞ p /6
p 2dt p dt
= [q ( − cosq )]0 +
2 ∫0 1+ t 2 + 2t cos a
dx = × 2 ∫
p /6
I=
2 0
( t + cos a )2 + sin2 a
dx ∫ 1⋅ cosq dq
0

p t + cos a  p p

 = [-q cosq + sinq ]0
p /6
=  tan−1 = −1
 − tan cot a 

sina sin a  sin a  2 
 0 
p 3 1
p p p  =− × +
= −1 6 2 2
 − tan tan  − a  
sina  2 2  Hence,
p p p  pa 6 -p 3
I=  − +a  = I=
sina  2 2  sina 12
Hence, I = pa cosec a.
68.  f ( x ) = e cos x . sin x is an odd function and
p p 
2  − x  cos x sin x dx  a a
 | x |≤ 2 ⇒ −2 ≤ x ≤ 2
 2 
65.  I = ∫    Using ∫ f ( x )dx = ∫ f (a − x )dx 
sin x + cos x
4 4
 0 0 
0 Therefore,
p p 3 2 3
p cos x sin x dx p cos x sin x dx ò f ( x )dx = ò f ( x )dx + ò f ( x )dx
2 2
I=
= ∫ 4 −I ⇒I = ∫ 4
2 0 sin x + cos 4 x 4 0 sin x + cos 4 x -2 -2 2
3
p ⇒ I = 0 + ò 2dx = [2 x ]32 = 2
∞ 2
p 2 tan x sec2 x p tdt
I= ∫ dx = ∫ , where t = tan x
4 0 1+ tan4 x 4 0 1+ (t 2 )2 3 5
69. I = 2 ò f ( x )dx + ò f ( x )dx
p 1 ∞ p 2

= ⋅  tan−1 t 2  = 0 3
4 2 0 16
æ1 2 3
ö 4 5
a +t 0 t a +t
= 2 çç ò f ( x )dx + ò f ( x )dx + ò f ( x )dx ÷÷ + ò f ( x )dx + ò f ( x )dx
66. ∫ f ( x )dx = ∫ f ( x )dx + ∫ f ( x )dx + ∫ f ( x )dx
a a 0 t
è0 1 2 ø 3 4

a +t
 02 12 22  32 42
= 2 + + + +
I3 = ∫ f ( x )dx
t
 2 2 2 2 2
Put x = t + y, then 25 35
=1+4+ =
a a 0
2 2
I3 = ∫ f (t + y )dy = ∫ f ( y )dy = − ∫ f ( x )dx (since, f(t + y) = f(y))
0 0 a p /2
70. I =
Therefore, ò 0
lnsin x dx
t p /2 æp ö p /2
I = I1 + I2 + I3 = ∫ f ( x )dx (Independent of a). I=ò lnsin ç - x ÷ dx Þ I = ò lncos x dx
0
0
è2 ø 0

67. Let
p /2 p /2 æ sin 2 x ö
2I = ò ln(sin x × cos x )dx Þ 2I = ò ln ç ÷ dx
1/2
0 0
è 2 ø
x sin−1 x
I= ∫
0 1− x 2
dx
2I = ò
p /2
ln(sin 2 x )dx - ò
p /2
ln 2dx Þ 2I = ò
p /2
ln(sin 2 x )dx -
p
ln 2
0 0 0 2
Put x = sin q when x = 0, q = 0
p Put 2x = t, then
1
dx = cos q dq  when x = , q = . 1
2 6 dx = dt
2
Thus,
1 p p p /2 p
p /6
sinq sin (sinq ) −1
Þ 2I =
2 ò0
ln(sin t )dt - ln 2 Þ 2I = ò ln(sin t )dt - ln 2
2 0 2
I= ∫ 1− sin2 q
cos q dq
p /2 p p
Þ 2I = ò
0
ln(sin x )dx - ln 2 Þ 2I = I - ln 2
p /6
0 2 2
= ∫ q sinq dq ⇒I =−
p
ln 2
0
2

Mathematical Problem Book for JEE.indb 1062 07-06-2018 14:28:00


Chapter 23 | Definite Integration 1063

a a 3
x 2dx
71.
∫ ∫
I = f ( x )g( x )h( x )dx = f (a − x )g(a − x )h(a − x ) dx Here, I = ò
2 x4 - x2 +1
0 0 3 3 3
a xdx xdx tdt
=ò =ò =ò
∫ 1 2 2
= − f ( x )g( x )h(a − x )dx 2
x - 1+ 2
2 2 æ 1ö 2 æ 1ö
x 1+ ç x - ÷ 1+ ç t - ÷
0 è xø è tø
7I = 3I + 4I
a a
2.  f ′(x) = f(x)

0
f ( x )g( x ){3h( x ) − 4 h(a − x )} dx = 5 f ( x )g( x ) dx = 0

0
Integrating, we get

I=0 log f(x) = x + k


72. Let ⇒ f(x) = ex+k
p
æp ö Now, f(0) = 1
I=
ò x sin2 x sinçè 2 cos x ÷ø dx
0
(1)
Therefore,

p 1 = e0⋅ek
æp ö
ò
= (p - x )sin 2 (p - x )sin ç cos(p - x ) ÷ dx
0
è2 ø Therefore, f(x) = ex
⇒k=0
   
p
æp ö and
ò
= (p - x )sin 2 x sin ç cos x ÷ dx (2)
0
è2 ø g(x) = x – f(x) = x – ex
Therefore,
Adding Eqs. (1) and (2), we get 1 1 1
I = ò e x ( x - e x )dx = ∫ e x x dx − ∫ e 2 x dx
p
æp ö 0 0 0

ò
2I = p sin 2 x sin ç cos x ÷ dx
0
è2 ø e2 1 3 e2 3 - e2
= e - (e - 1) - + = - =
2 2 2 2 2
p
2 1/2
æp ö
ÞI =p
ò
0
2 sin x cos x sin ç cos x ÷ dx
è2 ø
3.  I = ò
-1/2
cos -1(4 x 3 - 3 x )dx

and
Put é p 2p ù
p p x = cosq, q ∈ ê , ú
cos x = z ⇒ − sin xdx = dz ë3 3 û
2 2 Now,
0 0
2z æ 2ö 8 8 cos–1(4x3 – 3x) = cos–1(cos 3q ) = 2p – 3q
p
ò 2 × p çè - p ÷ø sin z dz
p
=-
p ò z sin z dz = p
p
Therefore,
p /3
2 2
I=- ò
2p /3
(2p - 3q )sinq dq

Practice Exercise 2 2p /3 2p /3
p
3 1/3
    = 2p ò sinq dq - 3 ò q sinq dq =
2
xdx dx p /3 p /3
1.  ò 2
- ò 1
2 æ 1ö 1/2
x3 x2 + -1 p /4 p /4
1+ ç x - ÷ x2
è xø 4.  I = ò lncos2 xdx - ò ln(1+ tan x )dx
0 0
3 1/3
xdx dx
=ò -ò Now,
2 2
2 æ 1ö 1/2 æ 1ö 1
p /2
1æp 1ö p
1+ ç x - ÷ x 3 1+ ç x - ÷
è xø è xø I1 =
2 ò lncos xdx = 2 çè 2 ln 2 ÷ø = - 4 ln2
0
3 3
xdx t 3dt
=ò +ò and
2 2
2 æ 1ö 2 æ1 ö p
1+ ç x - ÷ t 2
1+ ç - t ÷ I2 = ln2
è xø èt ø
8
3 3
xdx tdt Therefore,
=ò +ò = I + I = 2I
2 2
2 æ 1ö 2 æ 1ö 3p
1+ ç x - ÷ 1+ ç t - ÷ I = I1 – I2 = - ln2
è xø è tø 8

Mathematical Problem Book for JEE.indb 1063 07-06-2018 14:28:15


1064 Mathematics Problem Book for JEE

1 2n æ r ö æ r2 ö ì x2
5.  y = lim
n®µ n
å ç
è n
÷
ø
× sin ç 2 + 1÷
n
ï1+ 4 x + ,
⇒ f(x) = í
x >2
r =1 è ø 2
ï5 x + 1, x £2
2 î
= ò x sin( x 2 + 1)dx = sin2 × sin 3
0 Now,
ì4 + x , x >2
6.  g(x) < f(x) ∀ x ∈ (0, ∞) f ′(x) = í
î5, x £2
Also,
Therefore,
d(x(f(x)) = g(x)dx
Therefore, f(2+) = f(2−) = f(2) = 11 is continuous at x = 2
x and
xf(x) = ò g( x )dx ∀ x ∈ (0, ∞)
0
f ′(2+) ≠ f ′(2−) ⇒ not differentiable at x = 2
and
a a +t

∫ f (t ) dt = − ∫ f (t ) dt
x
11.  ⇒ f(a – t) = –f(a + t) ∀ t ∈ R
xg(x) < ò g( x )dx ∀ x ∈ (0, ∞) a −t a
0
Hence, graph of f(x) is symmetrical about (a, 0).
7.  f ( x ) = [tan 100
x] 12.  f(a – t) = –f(a + t) ∀ t ∈ R
Þ f (31 100 - ) ¹ f (31 100 ) = f (31 100 + ) and Put x = a , then
f (100 - ) ¹ f (100 ) = f (100
1 100 1 100 1 100
+) f (2a ) = - f (0)
Hence, f(x) is discontinuous at these points.
a a +t
Now,
tan-1 41 100 tan-1 11 100
13.  ∫
a −t
f (t ) dt = − ∫ f (t ) dt
a

ò x ] dx = ò
100 100
[tan [tan x ] dx ⇒ f(a – t) = –f(a + t) ∀ t ∈ R
0 0
tan-1 21 100 tan-1 31 100 tan-1 41 100 ⇒ f(a – t) = –f(a + t) = x
+ ò [tan100 x ] dx + ò [tan100 x ] dx + ò [tan100 x ] dx ⇒ t = a – f –1(x) = f –1(–x) – a
tan-1 11 100 tan-1 21 100 tan-1 31 100
⇒ f –1(x) + f –1(–x) = 2a
Therefore, b

òf
-1
tan−1 100 4 ⇒ ( x )dx = 2ab

0
f ( x )dx = 3tan−1 100 4 − tan−1 100 3 − tan−1 100 2 − 1 -b

14.  On differentiating a polynomial of nth degree, we get another


¥ ¥ polynomial of (n – 1) degrees. So,
æ e - x (sin x )n ö
÷ + nò (sin x ) cos xe dx f ( x ) = {f ′( x )} ⇒ n = 2(n − 1) ⇒ n = 2
n-1 -x
8. In = ç 2

è - 1 ø0 0

¥ 15.  Let
= ò ( -(sin x )n + (n - 1)(1- sin2 x )(sin x )n-2 )e - x dx
0 f(x) = ax2 + bx + c ⇒ f ′(0) = b > 0
n(n - 1) Also,
= In-2
n2 + 1 f(x) = {f ′(x)}2 ⇒ ax2 + bx + c = 4a2x2 + 4abx + b2 ∀ x
Hence, Thus, a = 4a2, b = 4ab and c = b2
I10 90 1
= From which, we get a = , since (b ¹ 0 )
I8 101 4
9.  In option (B), we have Again,

¥ ¥ 1
1 - x2 1 19 a b 19
ò0 xe dx = - 2 e
- x2

0
=
2
ò f ( x )dx = 12
0
⇒ + +c =
3 2 12
Therefore, Therefore,
¥ ¥ ¥
æ 1 - x2 ö 1 - x2 p
ò0 x e dx = x çè - 2 e ÷ø 2 ò0
2 - x2
+ e dx = b 2 3
4 + b = Þ b = 1,
0
2 2
10. For x > 2, we have (since, (b > 0) and so c = 1)
x 1 x
x2 Therefore,
∫ (5 + 1− t )dt = ∫ (6 − t )dt + ∫ (4 + t )dt
0 0 1
= 1+ 4 x +
2 f ′(0) = b = 1

Mathematical Problem Book for JEE.indb 1064 07-06-2018 14:28:16


Chapter 23 | Definite Integration 1065

16.  By putting the value of a, b and c in f(x), we have 22.  From equation (1), we have
x 2
a2 (n − 1)
+ x +1 f (x) = k=
4 (n + 2 )
17.  Writing sin2 q = x, we get 2 sin q cos q dq = dx, and hence the
2cos x − 3sin x 
given integral is 23. (A) y = cos −1  
p /2  13 
1
∫ cos sin2 n−1q (2 sin q cos q ) dq
2 m−1
3
2 y = cos−1 cos (a + x) ∀ a = tan−1
0
1 2 m−1 2 n−1
2
1 Therefore,
=
20∫ (cos2 q ) 2 (sin2 q ) 2 dx
dy
1 =1
1 dx
= ∫ (1− x )m−1/2 x n−1/2dx
20 1
1 1
1
−1  2 −1  2
1  1 1
(B)  I = ∫−1sin  x + 2  dx + −∫1cos  x − 2  dx
= b m+ , n+ 
2  2 2 1
  1  1 
x = 2∫  sin−1  x 2 +  + cos −1  x 2 −  dx
18.  Writing = z, we get 0
  2   2 
n Let
n
 x
n 1
 1  1
f(x) = sin−1  x 2 +  + cos −1  x 2 − 
∫ 1− n 
0
x k −1 dx = ∫ (1− z )n (nz )k −1 ndz
0
 2  2
1  1  1 
= nk ∫ (1− z )n z k −1 dz = nk b (k , n + 1) = sin−1  x 2 +  + cos −1   x 2 +  − 1
 2  2 
0
 2 1
x z 1 dz which is defined if  x +  = 0 or 1
19.  Writing = z , we get x = ,1+ x = and dx =  2
1+ x 1− z 1− z (1− z )2
 1
1 1
if  x 2 +  = 0 then f(x) = sin−1 0 + cos−1 (−1) = 0 + p = p
z m−1 dz  2
LHS = ∫ m−1
(1− z )m+n = ∫ z m−1(1− z )n−1 dz = b (m, n)
(1 − z ) (1 − z )2
 1
0 0
if  x 2 +  = 1 then f(x) = sin−1 1 + cos−1 0 = p
∞ 2
x 1− n 
= ∫ (1+ x )m+ n dx 1
0 I = 2 ∫ p dx = 2p
0
Common Explanation for Questions 20–22: 3
dx
3
dx p
a
 − x (a − x ) 3 2 2 3/2

a a
(C)  ∫ (1+ x 2 )(1+ e sin x ) = ∫ 1+ x 2
=
3
2 3
∫x  + a ⋅ ∫ x a − x dx
− 3 0
4
a2 − x 2 dx =  2 2 2

0  3 0 60 6p 2p

∫ sin (sin x )dx = 3 ∫ sin−1(sin x )dx = 0


−1
a (D) 
a2 2 2
2 ∫0
= x a − x 2 dx 0 0

p /2 p /2
1 1
In = ∫x
n
a − x dx =
2 2
∫x
n−1
(x a −x2 2
)dx 24. (A)  I = ∫ p 
dx = ∫ 1+ cot x dx
0 1+ tan  − x  0

n − 1 n −2 2 2 
 1 2 2 3/2 
 − 3 (a − x )  + 3 ∫ x (a − x ) a − x dx
n−1
=x 2 2 2
p /2 p /2
= tan x  1 
1 n −1 2 n −1
∫ 1+ tan x dx = ∫ 1− 1+ tan x  dx
0 0
= − x n−1(a2 − x 2 )3/2 + a I n −2 − In
3 3 3 Therefore,
 n − 1 1 a2 (n − 1) p p
⇒  1+  In = − x n−1(a2 − x 2 )3/2 + I n −2 2I = ⇒I=
 3  3 3 2 4
x n−1(a2 − x 2 )3 2 a2 (n − 1) (B)  Differentiating [ f ( x 2 (1+ x 5 + 7 x 12 ))](2 x + 7 x 6 + 98 x 13 ) = 1
⇒ In = − + In−2 (1)
(n + 2) (n + 2) Put x = 1, we get
1 1
20.  I1 = ∫ x a − x dx = f(9) =
2∫
2 2
2 x a2 − x 2 dx 117

2(a2 − x 2 )3 2 (a2 − x 2 )3 2
∫e
−2 x
=− +c = − +c (C)  (sin2 x + cos2 x ) dx
2.3 3 0

x 3 (a2 − x 2 )3 2 3a2 a2
a
I a2  p
21.  I4 = − + I2 = I2 ⇒ 4 = = 2 ∫ e −2 x sin  2 x +  dx
6 6 2 I2 2 0  4
0

Chapter 23.indd 1065 14-06-2018 10:55:11


1066 Mathematics Problem Book for JEE

Therefore, π π /2
¥ (B)  2 ∫ xf (sin x )dx = π ⋅ 2 ∫ f (sin x )dx
é æ p öù
ê sin ç 2 x + ÷ ú ¥ 0 0
è 4 øú æ pö p /2
I = ê -e -2 x + ò e -2 x cos ç 2 x + ÷ dx æ æp öö
ê 2 ú 0 è 4ø = 2p ò f ç sin ç - x ÷ ÷ dx
è è2 øø
êë úû 0
0
p /2
1
=
1
+
1
- I ⇒ 2I =
1
⇒ integral value =
= 2p ∫ f (cos x ) dx
2 2 2 2 2 2 2 0
k k

æ x + 1- x ö (C)  ò xf ( x (1- x )) dx = ò (k + (1- k ) - x )


(D)  cot -1(1+ x 2 - x ) = tan-1 ç
è 1- x (1- x ) ÷ø 1- k 1- k

Therefore, f ( k + 1- k - x )(1- ( k + 1- k - x )) dx
k

∫ (1− x )f ((1− x ) x )dx


1
=
I =ò cot -1(1+ x 2 - x ) dx
1−k
0
k k
⇒ 2 ∫ xf ( x (1− x ))dx = ∫ f ( x (1− x ))dx
1 1

= ò tan-1 x dx + ò tan-1(1- x ) dx
1−k 1−k
0 0
1
p
2
1+cos t 1+cos2 t
= 2∫ tan−1 x dx = − log2 xf ( x (2 − x ))dx = (2 − x )f ( x (2 − x ))dx
0
2 (D)  ∫ ∫
sin2 t sin2 t
p 1+cos2 t 1+cos2 t
25. (A)  ∫ f ( x )dx = 0 if f(a – x) = –f(x) ⇒ ∫ xf ( x (2 − x ))dx = ∫ f ( x (2 − x ))dx
0 sin2 t sin2 t
p /4
cos 2 t
∫ tan
n−1
(B)  In + 1 = q (sec2 q − 1) dq
0
= ∫ 2
f (1 − x 2 )dx (write x = 1 – y)
− cos t
p /4

∫ tan
n−1
= q sec q dq − In−1
2
ì 1 2p (cosq + i sinq )2 ¼ïü
0 27.  I = Re al ïí ò 1+ (cosq + i sinq ) + + ý dq
Therefore, ïî p 0 2! ïþ
2p
é tann θ ù 1
π /4 1é sin2q ¼ù
In + 1 + In − 1 = ê = q + sinq + + ú =2
ë n û0
ú =
n p êë 2(2!) û0
∞ 1 ∞
x log x x log x x log x p f −1 ( p )
(C)  ∫ (1+ x
0 ) 0 (1+ x )
2 2
dx = ∫
2 2
dx + ∫
1 (1+ x )2 2
dx 28.  ∫ f ( x )dx =
−1
∫ t f ′(t )dt = [t f (t ) − f (t )]f −1 (0)
f −1 ( t )

0 f −1 (0)
1
Put x = in the last integral p p

y = ∫ f −1( x )dx = p 2 − ∫ (t + cos t )dt


Therefore, 0 0
π
æt ö 2
π2 π2
1 = π 2 - ç + sin t ÷ = π 2 - =
dx = − dy è2 ø0 2 2
y2
∞ 0 Therefore, k = 0.
x log x y4
∫0 (1+ x 2 )2 dx = ∫1 y 3 (1+ y 2 )2 dy 29.  Differentiating both sides of the given equation, we have
1
1
y log y d d
= −∫
(1 + y 2 )2
dy ò
dx cos x
t 2f (t )dt = (1- cos x )
dx
0

Therefore, given integral equals 0. ⇒ 1⋅f(1)⋅0 − cos2 x f(cos x) (− sin x) = sin x


⇒ cos2 xf (cos x) sin x = sin x
d æ -1 1 ö 1 æ 1ö 1 1
(D)  ç tan ÷= ç- ÷ = - ⇒ f (cos x ) =
dx è x ø 1+ 1 è x 2 ø 1+ x 2 cos2 x
x2
æ 3ö 3
Therefore, f ç is attained when cos x = . So,
ç 4 ÷÷
1 1
d æ 1ö dx p 4
I = ò ç tan-1 ÷ dx = - ò =- è ø
-1
dx è xø -1
1+ x 2 2
æ 3 ö 16
p p f çç ÷÷ = = 5.33
26.  (A)  ò xf (sin x )dx = ò (p - x )f (sin x )dx è 4 ø 3
0 0
p p   3
⇒ 2∫ xf (sin x ) dx = p ∫ f (sin x ) dx ⇒ f   = 5
0 0   4  

Chapter 23.indd 1066 11-06-2018 10:31:50


Chapter 23 | Definite Integration 1067

Solved JEE 2017 Questions


JEE Main 2017 p /4
8 cos 2 x
p /4
8 cos 2 x
3p
I= ∫ 3
dx ⇒ I = ∫ 3
dx
p /12  sin x + cos x  1
2 2
p /12  
4
 sin x cos x   
dx cos x sin x 
1. The integral ∫ 1+ cos x is equal to  
p
4 Multiplying numerator and denominator of denominator by 2, we
(A)
2 (B) 4 get
(C)
−1 (D) −2 p /4 p /4
(OFFLINE) 8 cos 2 x 8 cos 2 x
I= ∫ 2
3
dx = ∫ 2 
3
dx
Solution: The given integral is p /12   p /12 
   
2 sin x cos x  sin 2 x 
3p / 4
dx
I= ∫ 1+ cos x
(since sin 2 x = 2 sin x + cos x )

p /4
p /4 p /4
That is, sin3 2 x
dx
I= ∫ 8 cos 2 x ×
8
dx = ∫ cos 2 x sin3 2 x dx
I=∫ p /12 p /12
1+ cos(p − x )
p /4

∫ (sin 2 x cos 2 x ).sin2 2 x dx


b b
I=
By using ∫ f ( x )dx = ∫ f (a + b − x )dx , we get p /12
a a
p /4
3p / 4 1
2 p /∫12
 1 1    I= (sin 4 x )sin2 2 x dx  (since sin 2x = 2 sin x cos x)
2I = ∫  +  dx
1+ cos x 1− cos x 
p /4
p /4
3p / 4 1  1− cos 4 x 
2 p /∫12
 2  I= sin 4 x   dx  (since cos 2 x = 1− 2 sin2 x )
= ∫  2 
1− cos x 
dx  2
p /4
3p / 4 p /4
1
4 p /∫12
I= (sin 4 x − sin 4 x cos 4 x )dx
2I = 2 ∫ cosec2 xdx
p /4
p /4 p /4
Therefore, 1 1
4 p /∫12 4 p /∫12
3p / 4 I= sin 4 xdx − sin 4 x cos 4 x dx
I=2 ∫ cosec2 xdx
p /4
That is, p /4 p /4
1 1 sin 8 x
4 p /∫12 4 p /∫12 2
3p / 4
   I = sin 4 xdx − dx  (since sin2x = 2sinxcosx)
I = ( − cot x ) p / 4 = −( −1− 1) = 2
Hence, the correct answer is option (A). 1
p /4
1
p /4

4 p /∫12
I= sin 4 x dx − ∫ sin 8 x dx
p /4 8 p /12
8 cos 2 x
2. The integral ∫ dx equals
p / 2 ( tan x + cot x )
3
p /4 p /4
1  − cos 4 x  1  − cos 8 x 
⇒I =  −  
15 13 4 4 p /12  8  8 p /12 
(A) (B)
128 32
13 15 1  −1  p 1  p 
I= cos  4 ×  + cos  4 ×  
(C)
256
(D)
64 4  4  4 4  12  
(ONLINE) 1  −1  p 1  p 
−  cos  8 ×  + cos  8 ×  
8 8  4 8  12  
Solution: The given integral is
p /4 1  −1 1 p  1  −1 1 2p 
8 cos 2 x I= cos p + cos  −  cos 2p + cos 
I= ∫ 3
dx 4  4 4 3 8 8 8 3 
p /12 (tan x + cos x )
That is, 1  −1 1  1  1  −1 1  −1 
I= ( −1) +    −  (1) +   
p /4
4  4 4  2  8  8 8 2 
cos 2 x  sin x cos x 
I= ∫ dx  since tan x =
 3 cos x
; cot x = 
sin x  1 1  1 1 1  1
p /12  sin x + cos x  I= × 1+ − × −1− 

cos x sin x 
 4 4  2  8 8  2

Mathematical Problem Book for JEE.indb 1067 07-06-2018 14:28:51


1068 Mathematics Problem Book for JEE

1  3 1  3 2
dx k
⇒I =  +  
16  2  64  2  4. If ∫ ( x 2 − 2 x + 4 )3/ 2 = k + 5 , then k is equal to
1

⇒I =
3
+
3
=
( 4 × 3) + 3 = 12 + 3 = 15 (A) 1 (B) 3
32 128 128 128 128 (C) 4 (D) 2
(ONLINE)
Hence, the correct answer is option (A).
Solution: We have
 p
3. The integral ∫ 1+ 2 cot x (cosec x + cot x )dx  0 < x <  is 2 2
 2 dx dx
equal to (where C is a constant of integration) ∫ ( x 2 − 2 x + 4 )3/ 2 ⇒ ∫  2 
3/ 2

 x  x
1 1
( x − 1) + 3
2 log  sin  + C
(A) (B) 2 log  cos  + C
 2  2 Substituting x − 1 = 3 tanq , where x = 1, q = 0.
 x  x  1 p
4 log  cos  + C
(C) (D) 4 log  sin  + C When x = 2, q = tan−1  = .
 2  2  3  6
(ONLINE) p /6
3 sec2 q
Solution: The given integral is L= ∫ 3 tan2 q + 3
3/ 2
dq
0
 
I = ∫ 1+ 2cot x (cosec x + cot x )dx
p /6 p /6 p /6
3 sec2q d q 3 sec2 q dq = 1
= ∫ 1+ 2 cot x + cosec x + 2 cot x dx 2 = ∫ = ∫ 3 3 sec2 q 3 ∫ cosq dq
0 [3 sec2 q ]3 / 2 0 0
p /6
= ∫ 1+ cot2 x + 2 cot xcosec x + cot2 x dx 1
= sinq
3 0
Using 1 + cot2x = cosec2x
1 x 
⇒ sin − sin 0 
I = ∫ cosec2 x + 2cot x cosec x + cot2 x dx 3 6 
(a + b)2 = a2 + b2 + 2ab 11  1
⇒  − 0 =
3 2  6
⇒ I = ∫ (cot x + cosec x )2 dx 1 k
⇒ = ⇒ k + 5 = 6k ⇒ 5 = 5k ⇒ k = 1
6 k +5
I = ∫ (cot x + cosec x )dx
Hence, the correct answer is option (A).
1 cos x
Substituting cosec x = and cot x = , we get JEE Advanced 2017
sin x sin x
98 k +1 k +1
 cos x 1  cos x + 1 1. If I = ∑ k =1∫ dx , then
I = ∫ + dx = ∫ dx k x ( x + 1)
 sin x sin x  sin x
(A)
I > loge99 (B) I < loge99
 x  x  x 49 49
Substituting cos x + 1 = 2 cos2   and sin x = 2 sin   cos   , I<
(C) (D) I >
 2  2  2 50 50
we get
Solution: It is given that
2 cos2 ( x / 2) 2 cos( x / 2)
I=∫ dx = ∫ dx 98 k +1
sin( x / 2)cos( x / 2) sin( x / 2) k +1
I=∑ ∫ dx
 x  cos x  k =1 k x ( x + 1)
      ⇒ I = 2∫ cot   dx   since cot x = 
 2 sin x  98 k +1 98 k +1
1 1 1 
= ∑ (k + 1) ∫ dx = ∑ (k + 1) ∫  −  dx
x ( x + 1) x x + 1
Using the standard integral ∫ cot x dx = log| sin x | + C , we get k =1 k k =1 k
98
k +1
= ∑ (k + 1) ln x − ln( x + 1) k 
 x  
I = 2 log sin   + C k =1
 2 98
= ∑ (k + 1)[(ln(k + 1) − ln k ) − (ln(k + 1+ 1) − ln(k + 1)]
Hence, the correct answer is option (A). k =1

Mathematical Problem Book for JEE.indb 1068 07-06-2018 14:29:30


Chapter 23 | Definite Integration 1069

98 Taking natural logarithm on both sides, we get


= ∑ (k + 1)[ln(k + 1) − ln k − ln(k + 2) + ln(k + 1)]
k =1 2(99 )100
ln < ln 99 ⇒ I < ln 99 (ln ≡ loge )
98 (100 )99
= ∑ (k + 1)ln(k + 1) − (k + 1)ln k − (k + 1)ln(k + 2) + (k + 1)ln(k + 1)
k =1 Now, we know that
98
= ∑ (k + 1)ln(k + 1) − k ln k − ln k − (k + 1)ln(k + 2) + k ln(k + 1) + ln(k + 1) 98 k +1
k +1
k =1
I=∑ ∫ dx
k =1 k x ( x + 1)
Rearranging the terms, we get
98 k +1 98 k +1
98 98 k +1 k +1
I = ∑ (k + 1)ln(k + 1) − k ln k − ∑ (k + 1)ln(k + 2) − k ln(k + 1) ∑∫ x ( x + 1)
dx > ∑ ∫ ( x + 1)2
dx
k =1 k k =1 k
k =1 k =1
98 98 k +1 98 k +1
+ ∑ ln(k + 1) − lnk k +1
⇒I>∑ ∫ dx = ∑ (k + 1) ∫ ( x + 1)
−2
dx
k =1 k =1 k ( x + 1)2 k =1 k

That is,  k +1


98
( x + 1)−2 +1 98  −1 k +1
I = 99 ln 99 – 99 ln 100 + ln 2 + ln 99 = ln 99 – ln 100 + ln 2 + ln 99
99 99 ⇒ I > ∑ (k + 1)   = ∑ (k + 1)  
k =1  −2 + 1 k  k =1  ( x + 1) k 
 9999   99100 × 2 
I = ln  × 2 × 99  = ln   (1)
 10099   10099  98
 −1 1  98  1 1 
⇒ I > ∑ (k + 1)  +  = ∑ (k + 1)  − 
k =1
 ( k + 1 + 1) ( k + 1) k =1 ( k + 1) ( k + 2) 
Now, let us consider that 10099 is written as
98
8 98
10099 = (99 + 1)99  k + 2 − k −1 1
⇒ I > ∑ (k + 1)   = ∑ (k + 1)
 ( k + 1)( k + 2 )  ( k + 1)( k + 2)
Using binomial expansion, we get k =1 k =1

(100)99 = 99C0 + 99C1(99)1 + 99C2(99)2 + … + 99C98 (99)98 + 99C99 (99)99 98


1 1 1 1 1
ÞI>å = + + + +
k =1 ( k + 2 ) 3 4 5 100
= 99C0 + 99C1 (99)1 + 99C2 (99)2 + … + (99)99 + (99)99
These are total 98 terms. Now,
⇒ (100 ) > 2 ⋅ (99 ) 
99 99
(considering last two terms)
1 1 1 98 1 1 1
+ + + (98 terms) = < + + +
2 ⋅ (99 ) 99 100 100 100 100 3 4 100
⇒ <1
(100 )99 Therefore,
Multiplying both sides by 99, we get 98 49 49
I> = ⇒I>
100
2 ⋅ (99 ) 100 50 50
< 99
(100 )99 Hence, the correct answers are options (B) and (D).

Mathematical Problem Book for JEE.indb 1069 07-06-2018 14:29:45


Mathematical Problem Book for JEE.indb 1070 07-06-2018 14:29:45
24 Area Under the Curves

24.1  Curve Tracing Now y is real, if 0 ≤ x ≤ a, so its region lies between the lines
x = 0 and x = a.
To find the approximate shape of a curve, the following procedure 7. Examine what happens to y when x → ∞ or x → - ∞.
is adopted in order:
8. Asymptotes:
1. Symmetry:
Asymptote(s) is (are) line(s) whose distance from the curve
(a)  Symmetry about x-axis: If all the powers of ‘y’ in the
tends to zero as point on curve moves towards infinity along
equation are even, then the curve is symmetrical about the
branch of the curve.
x-axis. For example,
y2 = 4 a x If lim f ( x ) = ¥ or lim f ( x ) = -¥ , then x = a is asymptote of
(a) 
x ®a x ®a
(b) Symmetry about y-axis: If all the powers of ‘x’ in the
y = f(x).
equation are even, then the curve is symmetrical about the
y-axis. For example, (b) If lim f ( x ) = k or lim f ( x ) = k , then y = k is asymptote of
x ®¥ x ®-¥
x2 = 4 a y
y = f(x).
(c)  Symmetry about both axes: If all the powers of ‘x’ and ‘y’ in
the equation are even, then the curve is symmetrical about f (x)
(c) 
If lim = m or lim (f ( x ) - mx ) = c , then y = mx + c is an
the axis of ‘x’ as well as ‘y’. For example, x ®¥ x x ®¥

x2 + y2 = a2 asymptote (inclined to right).


(d)  Symmetry about the line y = x: If the equation of the curve f (x)
remains unchanged on interchanging ‘x’ and ‘y’, then the (d) If lim = m or lim (f ( x ) - mx ) = c , then y = mx + c is an
x ®-¥ x x ®-¥
curve is symmetrical about the line y = x. For example,
x3 + y3 = 3 ax y asymptote (inclined to left).
(e)  Symmetry in opposite quadrants: If the equation of the
curve remains unaltered when ‘x’ and ‘y’ are replaced by 24.2  Steps to Draw Curve
-x and -y respectively, then there is symmetry in opposite
For the evaluation of area of bounded regions, it is very essential to
quadrants. For example,
know the rough sketch of the curves. The following points are very
x y = c2
useful to draw a rough sketch of a curve.
2. If the equation of curve contains no constant terms, then it
While constructing the graph of f(x, y) = 0, it is expedient to fol-
passes through the origin. Find whether the curve passes
low the procedure given below:
through the origin or not.
x2 + y2 - 4ax = 0 1. Find the set of permissible values of x.
2. Check if the curve is symmetrical about x-axis, y-axis, origin,
3. Find the points where the curve crosses the x-axis and also the
etc.
x2 y2
y-axis. For example, the curve 2 + 2 = 1 intersect the axes at 3. Find the period of the curve if it is periodic.
points (±a, 0) and (0, ±b). a b 4. Find the asymptote(s) of the curve, if any.
5. Find the intervals of increase and decrease of the curve.
dy
4. 
Find and equate it to zero to find the points on the curve Hence, determine the greatest and the least values of the
dx curve, if any.
where you have horizontal tangents. And also find the points
dx Illustration 24.1   Sketch the region bounded by 3x + 4y ≤ 12.
at which = 0 at these points the tangent to the curve is
dy Solution: Converting the inequality into equation we get 3x + 4y =
parallel to y-axis.
12. This line meets x-axis at (4, 0) and y-axis at (0, 3). Joining these
5. 
Examine if possible the intervals when f(x) is increasing or
two points we obtain the straight line represented by 3x + 4y = 12.
decreasing.
This straight line divides the plane in two parts. One part contains
Write the given equation as y = f(x), and find minimum and
6. 
the origin the other does not contain the origin. Clearly, (0, 0)
maximum values of x which determine the region of the curve.
satisfies the inequality 3x + 4y ≤ 12. So, the region represented by 3x
2 2 a- x
For example, for the curve xy = a (a - x ) Þ y = a . + 4y ≤ 12 is the region containing the origin as shown in Fig. 24.1.
x

Mathematical Problem Book for JEE.indb 1071 07-06-2018 14:29:53


1072 Mathematics Problem Book for JEE

Y
x3
(0, 3)
y2 =
2a - x
3x + 4y = 12
LHS is positive. If x is negative or if x is greater than 2a, RHS
X′ X becomes negative. Hence the curve lies only in the interval 0 to 2a.
(0, 4) When x → 2a, y → ∞. Therefore, the line x = 2a is an asymptote for
the curve. A rough figure is shown (Fig. 24.3).
Y
Y′

Figure 24.1
X
Illustration 24.2   Sketch the graph for y = x2 - x. O 2a

Solution: We note the following points about the curve:


1. 
The curve does not have any kind of symmetry.
2. 
The curve passes through the origin and the tangent at the
origin is obtained by equating the lowest degree term to Figure 24.3
zero. x 2 (1 + x )
The lowest degree term is x + y. Equation it to zero, we get Illustration 24.4   Trace the curve y 2 = .
1− x
x + y = 0 as the equation of tangent at the origin.
Solution: The curve passes through the origin and is symmetrical
Putting y = 0 in the equation of curve, we get
3. 
about the x-axis. It intersects the x-axis at x = -1 and x = 0. If x <
x2 - x = 0 ⇒ x = 0,1
-1 or x > 1 the curve is non-existent. As x → 1, y → ± ∞ a rough
So, the curve crosses x-axis at (0, 0) and (1, 0).
diagram is shown below (Fig. 24.4).
Putting x = 0 in the equation of the curve, we obtain y = 0.
Y
So, the curve meets y-axis at (0, 0) only (Fig. 24.2).

dy d2 y
4.  y = x 2 - x Þ = 2 x - 1 and 2 = 2
dx dx
Now, X
−1 O +1
dy 1
=0⇒ x =
dx 2
1 d2 y 1
At x = , > 0 , so x = is point of local minima.
2 dx 2 2
Figure 24.4
dy 1
5.   > 0 Þ 2 x - 1 > 0 Þ x > The curve has a loop between -1 and 0.
dx 2

So, the curve increases for all x >


1
and decreases for all
24.3 Area of Bounded Region
2 Case I: Let f (x) be a continuous function in (a, b) (Fig. 24.5). Then
1 the area bounded by the curve y = f (x), x-axis and lines x = a and x
x< .
2 = b is given by the formula
Y b
A = ò f ( x )dx ,
a
X Y
(0, 0) (1, 0)

y = f(x)
(1/2, −1/4)

Figure 24.2

Illustration 24.3   Trace the curve y2 (2a - x) = x3, a > 0. X


O x=a x=b
Solution: Note that the curve passes through the origin and is
symmetrical about the x-axis. Figure 24.5
Provided f (x) ≥ 0 (or f(x) ≤ 0) ∀ x ∈ (a, b)

Mathematical Problem Book for JEE.indb 1072 07-06-2018 14:30:03


Chapter 24 | Area Under the Curves 1073

Case II: It is sometimes convenient to use formula for area with Y


respect to y, i.e. regarding x as a function of y (Fig. 24.6). P
Y

y=d A y1 y2 B

x = f(y)
C
Q y2
y=c
X
O x=a x=b
X
O
Figure 24.8
Figure 24.6 The values of y1 and y2 are obtained by solving the equation of the
The area between x = f(y), y-axis and the lines y = c and y = d is curve as a quadratic in y whose larger root y1 and smaller root y2
given by are functions of x.
a and b are the coordinates of the points of contact of tangents
d
A = ò f ( y )dy drawn parallel to the y-axis.
c Case III: When two curves (Fig. 24.9) intersect at a point and the area
between them is bounded by x-axis, area bounded by the curves
d b a b
Note: Sometimes it is better to use the formula x dy instead of ∫y
a y = f1(x), y2 = f2(x) and x-axis is ò f1( x )dx + ò f2 ( x )dx ,
c
a a
dx in the computation of area to simplify calculations.
Y

24.4  Area Enclosed Between Two Curves


y1 = f1 (x)
Case I: Figure 24.7 encloses an area between two curves one of P (α , β )
which is represented by PQ with equation y = f(x) and the other by y2 = f2 (x)
AB with the equation y = g(x).
Y
Q X
O x=a x=b

y = f(x) Figure 24.9


Note:
B 1. If curve lies completely above the x-axis, then the area is positive
P y1 but when it lies completely below x-axis, then the area is negative,
however, we have the convention to consider the magnitude only.
y = g(x)
A 2. If curve lies on both the sides of x-axis, that is, above the x-axis
y2 as well as below the x-axis, then calculate both areas separately
X and add their modulus to get the total area (Fig. 24.10).
O x=a x=b
In general if curve y = f(x) crosses the x-axis n times when
x varies from a to b, then the area between y = f(x), x-axis and
Figure 24.7 lines x = a and x = b is given by
b A = |A | + |A | + … + |A |
1 2 n
Area PABQ = ò ( y1 - y 2 ) dx
a
Y

where y1 = f(x) and y2 = g(x)


b
Area PABQ = ò {f ( x ) - g( x )} dx
a

Case II: Figure 24.8 represents the region bounded by a closed curve A1 A3 A5
ACQBP. The area of the region bounded by a closed curve ACQBP is X
O x=a x=b
b A6
A2 A4
ò ( y1 - y2 ) dx , y1 > y2
a
Figure 24.10

Mathematical Problem Book for JEE.indb 1073 07-06-2018 14:30:10


1074 Mathematics Problem Book for JEE

3. If the curve is symmetrical about x-axis, or y-axis, or both, then 4a


é 2 x3 ù a 64 a3 16a2
calculate the area of one symmetrical part and multiply it by = ê2 a x 3 / 2 - ú =4 8. a 3 / 2 - = sq. units
the number of symmetrical parts to get the whole area. êë 3 12a úû 0 3 12a 3

x2 y2
+
Illustration 24.5   Find the area of the ellipse - 1 = 0. Illustration 24.7   Find the area of the segment cut off from the
a2 b 2
parabola y2 = 2x by the line y = 4x-1.
Solution: See Fig. 24.11. The ellipse is symmetrical about both
axes and hence the area enclosed is Solution: The line y = 4x - 1 intersects the parabola y2 = 2x at
A and B
4 × (area under the curve in a quadrant)
2x = (4x - 1)2 ⇒ 16x2 - 10x + 1 = 0
Y ⇒ (8x - 1) (2x - 1) = 0
Therefore,
B
P (x, y) æ1 ö æ 1 1ö
A = ç , 1÷ and B = ç , - ÷
è 2 ø è8 2ø

X′ X If the formula y dx is to be used then the area will have to be split


A′ O A (a, 0)
up as OBC and CBA (Fig. 24.13). Instead the problem can be done
directly by using the formula ò ( x 2 - x1) dy .
B′
Y
Y′
Figure 24.11 A

a a
x2 C
= 4 ∫ y dx = 4 ∫ b 1 − dx
a2
0 0 O X
a
4b  x a2 − x 2 a2 −1 x 
a B
4b
= ∫
a 0
(a2 − x 2 ) dx = 
a  2
+ sin
2 a

 0
Figure 24.13
4b é a2p ù
= ê ú = p ab sq. units 1 1
æ y + 1 y2 ö
a êë 4 úû
Required area = ò ( x 2 - x1) dy = ò çç
4
- ÷÷ dy
2 ø
y = -1 2 - 1/ 2 è

1
Illustration 24.6   Find the area included between the parabolas é y2 y y3 ù
=ê + - ú æ 1 1 1ö æ 1 1 1 ö
y2 = 4ax and x2 = 4ay. = + - - - +
êë 8 4 6 úû -1/ 2 çè 8 4 6 ÷ø çè 32 8 48 ÷ø
   
Solution: See Fig. 24.12. The two parabolas intersect at O (0, 0) and (3 + 6 - 4 ) (3 - 12 + 2) 5 7 27 9
A (4a, 4a). = - = + = = sq. units
    24 96 24 96 96 32
Y
(3 + 6 - 4 ) (3 - 12 + 2) 5 7 27 9
x 2 = 4ay = - = + = = sq. units
24     96 24 96 96 32
y 2 = 4ax
Illustration 24.8   Find the area between the curves y = x2 + x -2
P A and y = 2x, for which |x2 + x - 2| + | 2x | = |x2 + 3x - 2| is satisfied.
Solution: To find the area between the curve y = x2 + x -2 and
Q y = 2x such that
O X |x2 + x - 2| + |2x| = |x2 + 3x - 2|
So, (x2 + x - 2) and 2x have same sign (Fig. 24.14).
Thus,
Required area = ar (PQR) + ar (STN)
Figure 24.12 0 2
= ò [2 x - ( x 2 + x - 2)] dx + ò [2 x - ( x 2 + x - 2)] dx
The area included between the two curves is area OQAP, that is,
-1 1
x = 4a 4a 0 2
æ x2 ö é x2 x3 ù é x2 x3 ù
∫ ( y1 − y 2 ) dx =
ò çç 2 a x- ÷ dx
4 a ÷ø
= ê - + 2x ú + ê - + 2x ú
x=0 0 è êë 2 3 úû -1 êë 2 3 úû1

Mathematical Problem Book for JEE.indb 1074 07-06-2018 14:30:28


Chapter 24 | Area Under the Curves 1075

7 é10 13 ù 7 7 7 So, interval value of f(x) is


= + - = + = sq. units
6 êë 3 6 úû 6 6 3

T
S
X′ (0, 0) X
−1 P p /4 5p /6 5p /3 2p
(−2, 0) N(1, 0) Y′

Q R
Figure 24.16
p
for 0 ≤ x <  , f(x) = cos x
4
Figure 24.14 p 5p
for ≤ x <  , f(x) = sin x
4 6
Illustration 24.9   Find out the area enclosed by circle |z| = 2,
5p 5p
é x xù for ≤x< , f(x) = 1/2
parabola y = x2 + x + 1, the curve y = êsin2 + cos ú and x-axis 6 3 
ë 4 4û
5p
(where [.] is the greatest integer function). for ≤ x ≤ 2p , f(x) = cos x
3
Solution: See Fig. 24.15. For x ∈ [-2, 2] Hence, required area is
Y p /4 5p / 6 5p / 3 2p
(0, 2) 1
∫ cos x dx + ∫ sin x dx + ∫ 2
dx + ∫ cos x dx
( − 3, 0 ) 0 p /4 5p / 6 5p / 3
1 5p / 3
= [ sin x ] + [ − cos x ]p / 4 + [ x ] + [sin x ]25pp / 3
p /4 5p / 6
(−1, 0) ( 3, 0 )
0
2 5p / 6
X′ X
(−2, 0) (−1/2, 0) (0, 0) (2, 0)  5p 
= + 2 + 3  sq. units
 12 

(0, −2) Your Turn 1


Y′
Figure 24.15 1. If A is the area of the region bounded by the curve y = 3 x + 4 ,
x-axis and the line x = -1 and x = 4 and B is that area bounded
x x
1 < sin2 + cos < 2 by curve y2 = 3x + 4, x-axis and the lines x = -1 and x = 4, then
4 4
Therefore, A : B is equal to
(A)  1 : 1 (B)  2 : 1
é 2x xù
êsin 4 + cos 4 ú = 1 (C)  1 : 2 (D)  None of these Ans.  (A)
ë û 2. The area of the region bounded by the curve 9x2 + 4y2 - 36 = 0 is
Now, we have to find out the area enclosed by the circle |z| = 2, (A) 9π (B) 4π
2
æ 3ö æ 1ö (C) 36π (D) 6π Ans. (D)
parabola ç y - ÷ = ç x + ÷ , line y = 1 and x-axis.
è 4ø è 2ø 3. The area bounded by the curve y = (x + 1)2, y = (x - 1)2 and the
Therefore, required area is shaded area in Figure 24.15. 1
line y = is
Hence, required area is 4
0 2 (A) 1/6   (B) 2/3   (C) 1/4   (D) 1/3 Ans. (D)
3 × 1+ ( 3 − 1) + ∫ ( x 2 + x + 1).dx + 2 ∫ 4 − x 2 dx 4. Let f (x) be a non-negative continuous function such that the
−1 3 area bounded by the curve y = f (x), x-axis and the ordinates
æ 2p 1ö p p p p
=ç + 3 - ÷ sq. units x = , x = b > is æç b sin b + cos b + 2 b ö÷ . Then f æç ö÷ is
è 3 6 ø 4 4 è 4 ø è2ø
ì 1ü æ p ö æ p ö
Illustration 24.10  Let f(x) = Max. ísin x , cos x , ý. Then deter­ (A)  ç 1- - 2 ÷ (B)  ç 1- + 2 ÷
î 2þ è 4 ø è 4 ø
mine the area of the region bounded by the curves y = f(x), x-axis, æp ö æp ö
y-axis and x = 2p. (C)  ç + 2 - 1÷ (D)  ç - 2 + 1÷  Ans. (B)
è4 ø è4 ø
Solution: See Fig. 24.16. Since, 5. Area bounded by curves y = x2 and y = 2 - x2 is
ì 1ü (A) 8/3 (B) 3/8
f(x) = Max ísin x , cos x , ý
î 2 þ (C) 3/2 (D)  None of these Ans. (A)

Mathematical Problem Book for JEE.indb 1075 07-06-2018 14:30:44


1076 Mathematics Problem Book for JEE

 6. Let y be the function that passes through (1, 2) having slope |x|
(2x + 1). The area bounded between the curve and x-axis is 2. The area bounded by y = , x ≠ 0 , and the lines y(x - 1) (x - 3)
= 0 is x
 (A)  6 sq. units (B) 5/6 sq. units
(A) 3    (B) 1   (C) 2    (D)  None of these
 (C)  1/6 sq. units (D)  None of these Ans. (C)
Solution: See Fig. 24.18.
 7. Area bounded by the curve x2 = 4y and the straight line
Y
x = 4y - 2 is given by |x|
y= , x ≠ 0,
8 9 x
 (A)  sq. units (B)  sq. units
9 8 y ( x − 1)( x − 3) = 0 1

Area = 2 × 1 X' X
 (C)  4 sq. units (D)  None of these Ans. (B) 2
3    = 2 sq. units x=1 x=3
 8. The area of the region bounded by the curve y = x | x |, x-axis
and the ordinates x = 1, x = -1 is given by
1 Y'
 (A) Zero (B) 
3 Figure 24.18
2
 (C)  (D) 1 Ans. (C) Hence, the correct answer is option (C).
3
 9. If the area bounded by y = ax and x = ay , a > 0, is 1, then a =
2 2 3. The area bounded by the curve | x | = cos-1y and the line |x| = 1
1 and the x-axis is
 (A) 1 (B)  (A)  cos 1 (B) sin 1    (C)  2 cos 1 (D)  2 sin 1
3
1 Solution: See Fig. 24.19.
 (C)  (D)  None of these Ans. (B)
3
| x | = cos-1y and
10. The area bounded by the curves y = x , 2y + 3 = x and x-axis line |x| = 1 and x-axis
in the first quadrant is y = cos|x|
27 |x| = 1, x-axis
(A) 9    (B)   (C)  36    (D) 18 Ans. (A)
4 1 −1 1
11. The area enclosed between the curve y = loge(x + e) and the Area = 2∫ cos x dx
coordinate axes is 0
 (A) 3    (B) 4   (C)  1    (D) 2 Ans. (C)
= 2 [ sin x ]0 = 2 sin1 sq. units
1

12. The parabolas = 4x and = 4y divide the square region


y2 x2 Figure 24.19
bounded by the lines x = 4, y = 4 and the coordinate axes. If S1,
S2, S3 are respectively the areas of these parts numbered from Hence, the correct answer is option (D).
top to bottom, then S1 : S2 : S3 is
 1
 (A)  2 : 1 : 2   (B)  1 : 1 : 1   (C)  1 : 2 : 1  (D)  1 : 2 : 3 loge | x |, | x | ≥ e
 Ans. (B) 4. Area bounded by the curve f ( x ) =  and x-axis
is | x | −1− 1 , | x | < 1
Additional Solved Examples  e e

1. The total area enclosed by the lines y = |x|, |x| = 1 and y = 0 is 1 2 1 2


1 (A)  2
+ 2 - (B)  2
+2+
(A) 1    (B) 2   (C)     (D)  None of these e e e e
2
Solution: See Fig. 24.17. 1 2
(C)  2 + (D)  None of these
e e
y = |x|, |x| = 1, y = 0
Solution: See Fig. 24.20.
1 1 1 1
Total area = ´ 1´ 1+ ´ 1´ 1 = + = 1 sq. units
2 2 2 2  1
loge | x |, | x | ≥ e
Y f (x) = 
| x | −1− 1 , | x | < 1
(−1, 1) (1, 1)  e e
     
1/ e  1
1 
1 1 Area required = 2  ∫ −  x − 1−  dx + ∫ − ln x dx 
X′ X  0  e 1/ e 
1 1
  x2 1/ e 
  1 
= 2 −  −  1+  x  − ( x (ln x − 1))11/ e 

  2  e   0 
         
Y′
Figure 24.17 é 1 1 1ù é æ 2 öù
= -2 ê 2 - - 2 ú - 2 ê( -1) + ç ÷ ú
Hence, the correct answer is option (A). ë 2 e e e û ë è e øû
     

Mathematical Problem Book for JEE.indb 1076 07-06-2018 14:31:02


Chapter 24 | Area Under the Curves 1077

1 2 2 4 3a
(8p − 9 3 ) a2
=- + + +2- = 2 ∫ ( 4 a2 − ( x − 2a)2 − ax ) dx = sq. units
e2 e e2 e 3
0
1 2
= 2 + 2 − sq. units Hence, the correct answer is option (D).
e e
7. The area {(x, y); x2 ≤ y ≤ x } is equal to
Y
1 2 1
(A)  (B)    (C)   (D)  None of these
3 3 6
Solution: See Fig. 24.22.
X' X {(x, y); x2 ≤ y ≤ x }
1
Area = ∫ ( x − x 2 )dx
0

Y' 1
é x 3/2 x 3 ù 2 1 1
Figure 24.20 =ê - ú = - = sq. units
êë 3 / 2 3 úû 0 3 3 3
       
Hence, the correct answer is option (A).
x2
5. The whole area of the curves x = a cos3 t, y = b sin3 t is given by
3 5 1 Y x
(A)  pab  (B)  pab  (C)  pab  (D)  None of these
8 8 8
Solution:
a
dx X' X
Area = 4 ∫ y ⋅ dt
dt
             0
0
=4 ò -3ab sin4 t × cos2 t × dt
Y'
p /2

3 1 1 p Figure 24.22
= 4 ´ 3ab × × × ×
6 4 2 2
Hence, the correct answer is option (A).
3
= p ab sq. units 8. The area enclosed by the curve y = x5, the x-axis and the
8
ordinates x = -1, x = 1 is
Hence, the correct answer is option (A).
1 1
(A)  (B) 1 (C)  (D) 0
6. Area common to the curves y2 = ax and x2 + y2 = 4ax is equal to 2 3
a2
(A)  (9 3 + 4p ) (B)  (9 3 + 4p )a2 Solution: See Fig. 24.23.
3
y = x5, x = ±1
a2
(C)  (9 3 - 4p ) (D)  None of these 1
x6
1
2 1
3 Area = 2 ò x 5dx = 2 = = sq. units
6 6 3
Solution: See Fig. 24.21. 0 0

y2 = ax, x2 + y2 - 4ax = 0 Y
Y
2 2
y = 4 ax − x y = x5
x = −1
x2 + ax – 4ax = 0
x2 - 3ax = 0
X' X X' X
x (x - 3a) = 0
x = 0, x = 3a x =1

Y'
Figure 24.21 Y'
3a Figure 24.23
Required area = 2 ∫ ( 4 ax − x 2 − ax ) dx
0
Hence, the correct answer is option (C).

Mathematical Problem Book for JEE.indb 1077 07-06-2018 14:31:22


1078 Mathematics Problem Book for JEE

 9. The area bounded by the curve y2 = 9x and the lines x = 1, x = 4 11. The area bounded by the axes of reference and normal to
and y = 0 in the first quadrant is y = loge x at the point (1, 0) is
(A) 14 (B) 7   (C)  28  (D)  None of these (A)  1 sq. units (B)  2 sq. units
1
Solution: See Fig. 24.24. (C)  sq. units (D)  None of these
2
4 4
x 3/2 4 Solution: See Fig. 24.26.
Area = ∫ 3 x dx = 2 × 3 = 2  x 3 / 2 
3 1 y = ln x
1 1
dy 1
= 2 2 − 1 = 14 sq. units
3
=
dx x

Y At x = 1,
y 2 = 9x
slope of normal = -1
y = -1(x - 1)
y + x = 1
1 1
X Area = ´ 1´ 1 = sq. units
2 2
x=1 x=4 Y 1
x =
y+ x
log e
1 y=
Figure 24.24
X
Hence, the correct answer is option (A). 1
10. The slope of the tangent to a curve y = f (x) at (x, f (x)) is 2x + 1.
If the curve passes through the point (1, 2), then the area
of the region bounded by the curve, the x-axis and the line
x = 1 is
5 6 1
(A)  (B) 
  (C)  (D) 6
6 5 6 Figure 24.26

Solution: See Fig. 24.25. Hence, the correct answer is option (C).
 f ′(x) = 2x + 1 12. The area bounded by the line |x| + |y| = 1 is
⇒ f (x) = x2 + x + c (A) 4 (B) 2    (C) 1 (D)  None of these
The curve passes through (1, 2), so
Solution: See Fig. 24.27.
     2 = 1 + 1 + c |x| + |y| = 1
⇒c=0
  f (x) = x2 + x æ1 ö
Area = 4 ´ ç ´ 1´ 1÷ = 2 sq. units
1
é 3 2 ù11 1 5 è2 ø
ò(x
2
+ x ) dx = ê x + x ú = + = sq. units Y
0 ë3 2 û0 3 2 6
(0, 1)
Y

y = x2+ x
1/2 1/2
X' X
(−1, 0) (1, 0)
1/2 1/2
X
x=1
(0, −1)
Y'
Figure 24.27
Hence, the correct answer is option (B).
Figure 24.25
13. If area bounded by curve f(x) and x-axis, x = 1 to x = b is (b - 1)
Hence, the correct answer is option (A). sin (3b + 4), then f(x) is

Mathematical Problem Book for JEE.indb 1078 07-06-2018 14:31:29


Chapter 24 | Area Under the Curves 1079

(A) 3x cos (3x + 4) + sin (3x + 4) Solution: Solving y2 = x and y = x, we get, y = 0, x = 0, y = 1, x = 1


(B) 3(x - 1) cos (3x + 4) + sin (3x + 4) Therefore,
(C)  x cos (3x + 4) + sin (3x + 4)
1
(D)  None of these 1
2 x2  2 1 1
Area = ∫ ( x − x )dx =  x 3 / 2 −  = − = sq. units
Solution: 0 3 2 0 3 2 6
b

ò f ( x ) dx = (b - 1)sin(3b + 4 ) Hence, the correct answer is option (C).


1
x
2.  The area of the plane region bounded by the curves x + 2y2 = 0
and x + 3y2 = 1 is equal to
∫ f ( x ) dx = ( x − 1)sin(3 x + 4 ) (replacing b by x)
1 5 1 2 4
(A)  (B)  (C)   (D) 
f(x) = 3(x - 1) cos (3x + 4) + sin (3x + 4) 3 3 3 3
Hence, the correct answer is option (B). [AIEEE 2008]
14. The area bounded by the curve y = x3, the x-axis and the Solution: Solving the equations, we get the points of intersection as
ordinates x = -2 and x = 1 is (-2, 1) and (-2, -1). The bounded region is shown as shaded region in
15 17 9 Fig. 24.29.
(A)      (B)      (C)      (D)  None of these
4 4 4 Y
Solution: See Fig. 24.28.
0 1 0 1
x4 x4
Area = ∫ x 3dx + ∫ x 3dx =
4
+
4
(−2, 1)
−2 0 −2 0

16 1 17
= + = sq. units
4 4 4
Y (1,0) X
y = x3

x + 2y 2 = 0
(−2, −1)
x + 3y = 1
2

X' X Figure 24.29


x = −2
x=1
The required area is

1 1 1
 y3 
2∫ (1− 3 y 2 ) − ( −2 y 2 ) dy = 2∫ (1− y 2 )dy = 2  y − 
Y' 0 0  3 0
Figure 24.28 2 4
= 2 × = sq. units
Hence, the correct answer is option (B). 3 3

15. The area of the region bounded by y = |x - 1| and y = 1 is Hence, the correct answer is option (D).
1 The area of the region bounded by the parabola (y - 2)2 = x - 1,
3. 
(A)  (B) 1
2 the tangent to the parabola at the point (2, 3) and the x-axis is
(C) 2 (D)  None of these
(A)  3 (B) 6 (C)  9  (D) 12
Solution:
1 [AIEEE 2009]
Area = ´ 1´ 2
2 Solution: Equation of tangent at (2, 3) for the parabola, (y - 2)2
= 1 sq. unit = x - 1, is S1 = 0, which implies that x - 2y + 4 = 0.
See Fig. 24.30. The required area is
Hence, the correct answer is option (B).
Area of ΔOCB + Area of OAPD - Area of ΔPCD
Previous Years' Solved JEE Main/AIEEE 1
3
1 
= ( 4 × 2) + ∫ ( y 2 − 4 y + 5)dy −  (1× 2)
Questions 2 0
 2 

1. The area enclosed between the curves y2 = x and y = | x | is 3
 y3 
(A) 2/3 (B) 1 = 4 +  − 2 y 2 + 5 y  − 1 = 4 + 9 − 18 + 15 − 1
(C) 1/6 (D) 1/3  3 0
[AIEEE 2007] = 28 − 19 = 9 sq. units

Mathematical Problem Book for JEE.indb 1079 07-06-2018 14:31:41


1080 Mathematics Problem Book for JEE

Alternate solution: Y
The area is
y=x
Y
2y = x + 4
D(0, 3)
P(2, 3) 1
y=
x
C(0, 2) X
O 1 e

A
X' X Figure 24.31
B(−4, 0) O
Hence, the correct answer is option (B).
y
The area bounded between the parabolas x 2 =
6.  and x2 = 9y,
Y' 4
and the straight line y = 2 is
Figure 24.30
10 2
(A)  20 2 (B) 
3 3 3
A ∫ (2 y − 4 − y 2 + 4 y − 5)dy = ∫ ( − y 2 + 6 y − 9 )dy 20 2
(C)  (D)  10 2
0 0 3
[AIEEE 2012]
3 3
 ( y − 3)3  27 Solution: From Figure 24.32, the required area is calculated as
= − ∫ (3 − y )2 dy =   = = 9 sq. units
0  3 0 3 2
2  y 
2
5 y  y 3/2 
Hence, the correct answer is option (C). A = 2 ∫  3 y −  dy  = 2∫ dy = 5  
 0  2   0
2  3 / 2 0
The area bounded by the curves y = cos x and y = sin x between
4. 
3p 10 3 / 2 20 2
the ordinates x = 0 and x = is = [2 − 0 ] = sq. units
2 3 3

(A)  4 2 + 2 (B)  4 2 - 1 Y x 2 = 9y y
x2=
(C)  4 2 + 1 (D)  4 2 - 2 4

[AIEEE 2010]
y=2
Solution: The required area is
p 5p 3p
4 4 2

∫ (cos x − sin x )dx + ∫ (sin x − cos x )dx + ∫ (cos x − sin x ) dx X'


O
X
0 p 5p
4 4
p 5p 3p Figure 24.32
= [ sin x + cos x ] 4 +
0 [ − cos x − sin x ] p +
4
[sin x + cos x ]52p Hence, the correct answer is option (C).
4 4

= ( 2 − 1)+
+ 2 2 +( −1+ 2 ) = 4 2 − 2 sq. units The area (in square units) bounded by the curves y = x , 2 y - x + 3 = 0 ,
7. 
y = x , 2 y - x + 3 = 0 , x-axis, and lying in the first quadrant is
Hence, the correct answer is option (D).
1 (A)  36 (B)  18
The area of the region enclosed by the curves y = x, x = e, y =
5.  27
x (C)  (D)  9
and the positive x-axis is 4
[JEE MAIN 2013]
3
(A)  1 sq. units (B)  sq. units Solution: First solving the equations, we have
2
5 1 2 x = x - 3 (1)
(C)  sq. units (D)  sq. units
2 2
[AIEEE 2011] Squaring on both sides of Eq. (1), we get
Solution: From Figure 24.31, we have,
4 x = x 2 − 6 x + 9 ⇒ x 2 − 10 x + 9 ⇒ x = 9 , x = 1
1 e
1 1 3
Area = ∫ x dx + ∫ dx = + 1 = sq. units Since x = 1 intersects the parabola below the x-axis, this point is
0 1
x 2 2
extraneous.

Mathematical Problem Book for JEE.indb 1080 07-06-2018 14:31:58


Chapter 24 | Area Under the Curves 1081

So, for x = 9 we have, y = 3. Let A = {(x, y): y2 ≤ 4x, y - 2x ≥ - 4}. Then the area (in square
  9. 
Therefore, the required area under the curve (see Fig. 24.33) is units) of the region A is
3 (A) 8   (B) 9   (C) 10   (D) 11
3

2  dy ⇒ y 2 + 3 y − y
3

∫( 2 y + 3 ) − y    = 9 + 9 − 9 = 9 sq. units


3 0
[JEE MAIN 2014 (ONLINE SET-1)]
0  Solution: See Fig. 24.35. Finding points of intersection,
Y
y2 y + 4
y= x =
4 2
(a, 3)
Therefore,
2 y2 = 4 y + 16 or y2 = 2 y + 8 ⇒ y2 - 2 y - 8 = 0

X 2 ± 4 + 32 2 ± 6
(3, 0) (9, 0) y= = = 4 , -2
2 2
Therefore,
(0, −3/2) x = 4, 1 and P is (1, - 2) and Q is (4, 4)

Y' Y

Figure 24.33
Q
Hence, the correct answer is option (D).
The area of the region described by A = {(x, y): x2 + y2 ≤ 1 and
8. 
y2 ≤ 1 - x} is
X
p 2 p 2
(A)  - (B)  +
2 3 2 3
p 4 p 4 P
(C)  + (D)  -
2 3 2 3 y 2 = 4x
y2
[JEE MAIN 2014 (OFFLINE)] y = 2x − 4 x=
4
Solution: See Fig. 24.34. x = (y + 4)/2

Y
Figure 24.35
1
4
ïìæ 4 + y ö y ïü
2
Required area = ò íïçè ÷ - ý dy
2 ø 4 ïþ
-2 î

4 4
æ 1 y2 ö é y2 y3 ù
X'
(1, 0)
X = òç 2 4 ÷ ê
ç 2 + y - ÷ dy = ê 2 y + - ú
4 12 úû -2
-2 è ø ë

æ 64 ö æ 4 8 ö
= ç 8 + 4 - ÷ - ç 2( -2) + + ÷
è 12 ø è 4 12 ø

 72 
= 12 + 4 − 1−   = 15 − 6 = 9 sq. units
 12 

Y'
Hence, the correct answer is option (B).
Figure 24.34
10. 
The area of the region above the x-axis bounded by the curve
y2 = 1 - x ⇒ x = 1 - y2 p p
y = tan x ,0 £ x £ and the tangent to the curve at x = is
1 1 2 4
Required area = (p ´ 12 ) + 2 ò (1- y 2 )dy
2 0
1æ 1ö 1æ 1ö
(A)  ç log 2 - ÷ (B)  ç log 2 + ÷
1 2è 2ø 2è 2ø
p  y3  p  1  p 4
= + 2  y −  = + 2  1−  − 0  = + sq. units 1 1
2  3 0 2   3   2 3 (C)  (1- log 2 ) (D)  (1+ log 2 )
     2 2
Hence, the correct answer is option (C). [JEE MAIN 2014 (ONLINE SET-4)]

Mathematical Problem Book for JEE.indb 1081 07-06-2018 14:32:12


1082 Mathematics Problem Book for JEE

Solution: See Fig. 24.36. Finding points of intersection,


Y
 y + 1
y2 = 2 ⇒ 2 y 2 = y + 1⇒ 2 y 2 − y − 1 = 0
 4 
⇒ ( y − 1)(2 y + 1) = 0
−1
⇒ y = 1 and y =
2
C
1 1
X' X ⇒ x = and x =
π 2 8
A B
2
So, point A is (1/8, - 1/2) and B is (1/2, 1).
π 1 π
− x= yB yB
4 2 4
R = shaded area ò ( xline )dy - ò ( xparabola )dy
yA yA

Y' 1
1 y2
1

Figure 24.36
= ∫ 4
( y + 1)dy − ∫
2
dy
−1/ 2 −1/ 2
p
p 
Required area = ∫ tan x − area under tangent at  ,1 (1) 1  y2 
1
1  y3  9
1
4  =
0  + y −   = sq. units
4 2  −1/ 2 2  3  −1/ 2 32
d p
Now slope of tangent is tan x at x = = sec2 x p =2
dx 4 at x =
4 Hence, the correct answer is option (C).
æ pö æ pö 12. 
The area (in square units) of the region bounded by the curves
Therefore, equation of tangent is y - 1 = 2 ç x - ÷ or y = 2 x + ç 1- ÷
è 4ø è 2ø y + 2x2 = 0 and y + 3x2 = 1 is equal to
This tangent cuts x-axis when y = 0 3 3 1 4
(A)  (B)  (C)   (D) 
Therefore, 5 4 3 3
p
-1 [JEE MAIN 2015 (ONLINE SET-1)]
p 1
x= 2 = - Solution: See Fig. 24.38.
2 4 2
C1: y + 2x2 = 0;
Thus, required area is
p C2: y + 3x2 = 1
[logsec x ] 4
0 - area triangle A B C
Y
1  p p 1 1 1
= log 2 − 0 − ×  − +  × 1 = log 2 −  sq. units
2  4 4 2  2 2
Hence, the correct answer is option (A). 1
−1 1
11. The area (in sq. units) of the region described by {(x, y): ≤ 2x y2 X
and y ≥ 4x - 1} is −1
A B
5 15 9 7 −2
(A)  (B)  (C)   (D) 
64 64 32 32
[JEE MAIN 2015 (OFFLINE)]
Solution: See Fig. 24.37.
R = {(x, y): y2 ≤ 2x and y ≥ 4x - 1} Figure 24.38
Y At the point of intersection of C1 and C2
y = 4x − 1 -2x2 = 1 - 3x2 ⇒ x2 = 1 ⇒ x = ±1
Therefore, A (-1, -2) and B (1, -2) are points of intersection as
y 2 = 4x
shown above.
B So, required area is
X′ X 0
O′ 2 ∫ [(1− 3 x 2 ) − ( −2 x 2 )]dx
A
−1

0 0
 x3  4
= 2 ∫ (1− x 2 )dx = 2  x −  = sq. units
−1  3  −1
3

Y′
Figure 24.37 Hence, the correct answer is option (D).

Mathematical Problem Book for JEE.indb 1082 07-06-2018 14:32:28


Chapter 24 | Area Under the Curves 1083

The area (in sq. units) of the region {(x, y): y2 ≥ 2x and x2 + y2 ≤ 4x,
13.  y ≥ x2 - 5x + 4
x ≥ 0, y ≥ 0} is y ≥ (x - 1)(x - 4)
p 2 2 4 The area of the region to be measured is
(A)  - (B)  p - 4 4
2 3 3 1 x3 5x2
× 2 × 2 + ∫ ( x 2 − 5 x + 4 )dx = 2 + − + 4x
8 4 2 2 3 2
(C)  p - (D)  p - 3 3
3 3
64 5 ´ 16 27 5 ´ 9
[JEE MAIN 2016 (OFFLINE)] =2+ - + 16 - + - 12
3 2 3 2
Solution: We have y2 - 2x ≥ 0 and x2 + y2 - 4x ≤ 0, x ≥ 0, y ≥ 0. 12 + 7 19
(x - 2)2 + y2 ≤ 4 = = sq. units
6 6
Point of intersection of both curves y2 = 2x and (x - 2)2 + y2 = 4 is
Hence, the correct answer is option (A).
(0, 0) and (2, 2) (Fig. 24.39).
Y Previous Years' Solved JEE Advanced/
Q (2, 2)
IIT-JEE Questions
Paragraph for Questions 1–3: Consider the functions defined
P
(0, 0) (2, 0)
X implicitly by the equation y3 - 3y + x = 0 on various intervals in the
real line. If x ∈ (-∞, -2) ∪ (2, ∞), the equation implicitly defines a
unique real valued differentiable function y = f(x).
If x ∈ (-2, 2), the equation implicitly defines a unique real valued
differentiable function y = g(x) satisfying g(0) = 0.
[IIT-JEE 2008]
Figure 24.39 1. If f ( -10 2 ) = 2 2 , then f ¢¢( -10 2 ) =
The required area is
4 2 4 2
2 2 (A)  (B)  -
7332 7332
ò ( y1 - y2 )dx = ò (
2
4 x - x - 2 x )dx
0 0 4 2 4 2
(C)  (D)  -
2
p (2 )
2 733 733
= - 2 ò x dx
4 Solution: We have
0
y3 - 3y + x = 0
 8 Differentiate both sides, we get
=  p −  sq. units
 3
3y2 y ′ - 3y ′ + 1 = 0 (1)
Hence, the correct answer is option (C). Put y = 2 2 , x = −10 2 . Then
The area (in sq. units) of the region described by A = {(x, y) | y
14.  −1
≥ x2 - 5x + 4, x + y ≥ 1, y ≤ 0} is y ′( −10 2 ) =
21
19 17 Differentiate equation (1), we get
(A)  (B) 
6 6
3y2 y ′′ + 6y(y ′)2 - 3y ′′ = 0
7 13
(C)  (D)  −1
2 6 Put y = 2 2 , x = −10 2 , y ′ = . Then
21
[JEE MAIN 2016 (ONLINE SET-1)]
4 2
Solution: See Fig. 24.40. y ′′( −10 2 ) = −
73 ⋅ 32
Y Hence, the correct answer is option (B).
Area to be 2. The area of the region bounded by the curves y = f(x), the x-axis,
measured and the lines x = a and x = b, where -∞ < a < b < -2, is
b
2 (3, 0) x
(1,0) (4,0)
X (A)  ò 3((f ( x ))2 - 1) dx + bf (b) - af (a)
a
x+
y= b
x
(3, −2)
1 (B)  - ò 2
dx + bf (b ) - af (a)
a 3(( f ( x )) - 1)
Figure 24.40 b
x
We have (C)  ò 3((f ( x ))2 - 1) dx - bf (b) + af (a)
A = {(x, y)| y ≥ x2 - 5x + 4, x + y ≥ 1, y ≤ 0} a

Mathematical Problem Book for JEE.indb 1083 07-06-2018 14:32:46


p
4  1+ sin x 1− sin x 
∫  cos x

cos x 
dx
0


p x x
1084 Mathematics Problem Book for JEE  1+ tan 2
4 1− tan 
2 dx
= ∫ −
x x
0  1 − tan 1+ tan 
 2 2
b
x 
p x 
(D)  - ò dx - bf (b ) + af (a)  2 tan
4 
a 3(( f ( x ))2 - 1) = ∫ 2  dx
0  1 − tan2
x 
Solution:  
2
b
Required area = ∫ f ( x )dx x
a
Put tan = t . Then
2
b
= [ xf ( x )]ba − ∫ xf ′( x )dx (By parts) 1 2x
a sec dx = dt
b
2 2
xdx 2dt
= bf (b ) − af (a) + ∫ ⇒ dx =
a
3[ f ( x )2 − 1] 1+ t 2
2 −1
Hence, the correct answer is option (A). 4t 
1
⇒ ∫   dt
 (1+ t 2 ) 1− t 2 
0
3. ò g¢( x )dx =
-1 Hence, the correct answer is option (B).
(A) 2g(-1) (B) 0 (C)  -2g(1) (D) 2g(1) 5. Area of the region bounded by the curve y = ex and lines x = 0
Solution: and y = e is
e
1
y′ =
3[1− (f ( x ))2 ]
(A)  e - 1 (B)  ò ln(e + 1- y )dy
1
1 e
Clearly f(x) is an odd function, then g ′(x) is an even function, so
(C)  e - ò e x dx (D)  In y dy
1 1
0 1
∫ g′( x )dx = 2∫ g′( x )dx [IIT-JEE 2009]
−1 0

= 2[ g( x )]10 Solution: See Fig. 24.41.


e
= 2[ g(1) − g(0 )] Required area = ò ln y dy
= 2g(1) (As g(0) = 0) 1

Hence, the correct answer is option (D). = ( y ln y - y )1e = (e - e ) - { -1} = 1



1+ sin x Also,
The area of the region between the curves y =
4.  and e e
cos x
ò ln y dy = ò ln(e + 1- y )dy
1- sin x p 1 1
y= bounded by the lines x = 0 and x = is
cos x 4 Further the required area can be written as
2 -1 2 -1 1
t 4t e × 1− ∫ e x dx
(A)  ò (1+ t 2 ) 1- t 2
dt (B)  ò (1+ t 2 ) 1- t 2
dt
0
0 0
2 +1 2 +1 y y = ex
4t t
(C)  ò (1+ t 2 ) 1- t 2
dt (D)  ò (1+ t 2 ) 1- t 2
dt
0 0
y=e
[IIT-JEE 2008]

 p
Solution: Since, both curves lie above x-axis in x ∈ 0 ,  .
 4
(0, 1)
Therefore, area bounded between the curve is
x
p x=1
4  1+ sin x 1− sin x 
∫  cos x

cos x 
dx Figure 24.41
0
Hence, the correct answers are options (B), (C) and (D).
 p x x
 1+ tan 2
4 1− tan  Paragraph for questions 6–8: Consider the polynomial f(x) = 1 +
= ∫ − 2 dx 2x + 3x2 + 4x3. Let s be the sum of all distinct real roots of f(x) and
x x
0  1 − tan 1+ tan  let t = |s|.
 2 2
[IIT-JEE 2010]
 p x 
 2 tan
4 
= ∫ 2  dx
0  1 − tan2
x
 
2

Mathematical Problem Book for JEE.indb 1084 07-06-2018 14:33:09


Chapter 24 | Area Under the Curves 1085

6. The real number s lies in the interval 3 1 1 1


(A)     (B)     (C)     (D) 
æ 1 ö æ 3ö 4 2 3 4
(A)  ç - , 0 ÷ (B)  ç -11, - ÷
è 4 ø è 4 ø [IIT-JEE 2011]
æ 3 1ö æ 1ö Solution: See Fig. 24.42.
(C)  ç - , - ÷ (D)  ç 0 , ÷
è 4 2ø è 4ø Therefore,
Solution: Since, b 1
1
ò (1- x ) dx - ò (1- x ) dx = 4
2 2
æ 1ö æ 3 ö
f ç - ÷×f ç - ÷ < 0 Þ S
è 2ø è 4ø 0 b

b 1
æ 1ö æ 3 ö  3 1  3  3 1
f ç - ÷ × f ç - ÷ < 0 Þ S lie in  − , −  ⇒  ( x − 1)  −  ( x − 1)  =
è 2 ø è 4 ø  4 2   3 0  3 b 4
Hence, the correct answer is option (C). (b − 1)3 1  (b − 1)3  1
⇒ + − 0 − =
7. The area bounded by the curve y = f(x) and the lines x = 0, y = 0 3 3  3  4
and x = t, lies in the interval 2(b − 1)3 1 1 1
⇒ = − ⇒ (b − 1)3 = − ⇒ b =
æ3 ö æ 21 11 ö 3 12 8 2
(A)  ç , 3 ÷ (B)  ç , ÷
è4 ø è 64 16 ø
æ 21 ö
(C)  (9, 10) (D)  ç 0 , ÷
è 64 ø
Solution: R1
R2
3 1
- <s<- o b 1
4 2
1 3 Figure 24.42
<t <
2 4 Hence, the correct answer is option (B).
1/ 2 3/ 4
10. Let f: [-1, 2] → [0, ∞] be a continuous function such that
ò (4 x ò (4 x
3
+ 3 x 2 + 2 x + 1)dx < area < 3
+ 3 x 2 + 2 x + 1)dx
2
0 0

[ x 4 + x 3 + x 2 + x ]10/ 2 < area < [ x 4 + x 3 + x 2 + x ]30 / 4


f(x) = f (1 - x) for all x ∈ [-1, 2]. Let R1 = ò xf ( x )dx , and R2 be
-1

1 1 1 1 81 27 9 3 the area of the region bounded by y = f(x), x = -1, x = 2, and


+ + + < area < + + + the x-axis. Then
16 8 4 2 256 64 16 4
15 525 (A)  R1 = 2R2 (B)  R1 = 3R2
< area <
16 256 (C)  2R1 = R2 (D)  3R1 = R2
[IIT-JEE 2011]
Hence, the correct answer is option (A).
Solution:
8.  The function f ′(x) is
2 2
æ 1ö æ 1 ö
(A)  increasing in ç -t, - ÷ and decreasing in ç - , t ÷
R1 = ò xf ( x )dx = ò (2 - 1- x )f (2 - 1- x )dx
è 4ø è 4 ø -1 -1
2 2
æ 1ö
(B)  decreasing in ç -t, - ÷ and increasing in æ - 1 , t ö = ò (1- x )f (1- x )dx = ò (1- x )f ( x )dx
è 4ø ç ÷
è 4 ø -1 -1
(C)  increasing in (-t, t)
(D)  decreasing in (-t, t) 2
Hence, 2R1 = ò f ( x )dx = R2 .
Solution: -1
f(x) = 4x3 + 3x2 + 2x + 1
f ′(x) = 12x2 + 6x + 2 Hence, the correct answer is option (C).
2
f ′′(x) = 2[12x + 3] = 0 ⇒ x = -1/4 11. Let S be the area of the region enclosed by y = e-x  , y = 0, x = 0
f ′′′(x) = 24 and x = 1. Then
So, the funtion is decreasing in (-t, t). 1 1
Hence, the correct answer is option (D). (A)  S ³ (B)  S ³ 1-
e e
9. Let the straight line x = b divides the area enclosed by y = (1 - x)2, 1æ 1 ö 1 1 æ 1 ö
y = 0 and x = 0 into two parts R1(0 ≤ x ≤ b) and R2(b ≤ x ≤ 1) such (C)  S £ ç 1+ ÷ (D)  S £ + ç 1- ÷
4è eø 2 eè 2ø
1
that R1 - R2 = . Then b equals
4 [IIT-JEE 2012]

Mathematical Problem Book for JEE.indb 1085 07-06-2018 14:33:33


1086 Mathematics Problem Book for JEE

Solution: See Fig. 24.43. p /2 


−  sin x 0
p /2 p /2 p /4 p /4 p /2
= − cos x 0 + sin x 0 + cos x 0 − cos x p / 4 − sin x
Y  p /4 

é 1 1 æ 1 ö æ 1 öù
= -(0 - 1) + (1- 0 ) - ê + - 1- ç 0 - ÷ - ç 1- ÷ú
A (0, 1) P (1 2 , 1)
ë 2 2 è 2ø è 2 øû
é 1 1 ù
= 2 - ê 2 - 1+ - 1+ ú = 2 - éë2 2 - 2 ùû
ë 2 2û
= 4 - 2 2 = 2 2 ( 2 - 1)
B (1/ 2, 1/ e)
R Hence, the correct answer is option (B).

C D (1, 1/e ) 13. 


For a point P in the plane, let d1 (P) and d2 (P) be the distances
of the point P from the lines x - y = 0 and x + y = 0 respectively.
The area of the region R consisting of all points P lying in the
O S X
first quadrant of the plane and satisfying 2 ≤ d1 (P) + d2 (P) ≤ 4,
Figure 24.43
is _____.
1 [JEE ADVANCED 2014]
S> (As area of rectangle OCDS = 1/e)
e
Solution:
Since, Y
2
e-x  ≥ e-x ∀ x ∈ [0, 1] 2 2 Y=X
Y = −X
P(T, B)
1 Required
æ 1ö
Þ S > ò e - x dx = ç 1- ÷ d2
Area
0 è eø 2

Area of rectangle OAPQ + Area of rectangle QBRS > S


2 2 2 X
1 æ 1 öæ 1 ö
S< (1) + ç 1- ÷ç ÷
2 è 2 øè e ø

Since, Figure 24.45

1æ 1 ö 1 |x-y|
d1 =
ç 1+ ÷ < 1- 2
4è eø e
|x+y|
d2 =
Hence, the correct answers are options (A), (B) and (D). 2
12. The area enclosed by the curves y = sin x + cos x and Therefore, according to the question (Fig. 24.45)
é pù
y = cos x - sin x over the interval ê0 , ú is
ë 2û |x-y| |x+y|
2£ + £4
(A)  4( 2 - 1) (B)  2 2 ( 2 - 1) 2 2

(C)  2( 2 + 1) (D)  2 2 ( 2 + 1) ⇒ 2 2 £ | x - y | + | x + y | £ 4 2 (1)


[JEE ADVANCED 2013]
Since x, y ≥ 0 in the first quadrant.
Solution: Figure 24.44 depicts the area enclosed by the given When x > y (or y - x < 0),
curves, we have |x - y| = x - y and |x + y| = x + y
Y
Therefore, Eq. (1) is true given that,

2 2 £ x -y + x + y £4 2 Þ 2 £ x £2 2

checking with (2, 1) in region x > y, i.e. 2 > 1.


o π /4 π /2 X Therefore, we shade area below y = x from [ 2 , 2 2 ] .
Figure 24.44 1 1
Area of this region = (2 2 ´ 2 2 ) - 2 ´ 2 = 4 - 1 = 3 sq. units
p /2 2 2
ép / 4 p /2 ù
ò (sin x + cos x )dx - êê ò (cos x - sin x )dx + ò (sin x - cos x )dx ú
úû
By symmetry about y = x, total area required = 6 sq. units
0 ë 0 p /4
Hence, the correct answer is (6).

Mathematical Problem Book for JEE.indb 1086 07-06-2018 14:33:48


Chapter 24 | Area Under the Curves 1087

p
x2 + Column I Column II
6
é 1ù
14. Let F ( x ) = ò 2 cos t dt for all x ∈ and f : ê0 , ú ® [0 , ¥ )
2
(D) Suppose that F(a ) denotes the area of the region (S) 5
x ë 2û bounded by x = 0, x = 2, y 2 = 4x and y = |a x - 1| +
|a x - 2| + a x, where a ∈ {0, 1} Then the value(s) of
é 1ù 8
be a continuous function. For aÎ ê0 , ú , if F ′(a) + 2 is the area
ë 2û F(a ) + 2 , when a  = 0 and a  = 1, is (are)
3
of the region bounded by x = 0, y = 0, y = f(x) and x = a, then
f(0) is _____. (T) 6
[JEE ADVANCED 2015]
[JEE ADVANCED 2015]
Solution: We have Solution: See Fig. 24.46.
a
2(a2 - b2) = c2(1)
F ¢(a) + 2 = ò f ( x )dx
sin( x - y )
l= (2)
0

sin z
Differentiating both sides, we get
cos (nπλ) = 0 (3)
F ¢¢(a) = f (a)
Now, (2m + 1)
Þ nl = (4)
x 2 +p /6
2
F( x ) = ∫
x
2 cos2 tdt
From Eq. (2),
sin x cos y - cos x sin y
æ pö l=
F ¢( x ) = 2 x × 2 cos2 ç x 2 + ÷ - 2 cos2 x sin z
è 6ø
a cos y - b cos x
 p  p Þl = (By Sine formula)
⇒ F ′′( x ) = −16 x 2 cos  x 2 +  sin  x 2 +  + 4 cos x sin x c
 6   6
æ a2 + c 2 - b 2 ö æ b 2 + c 2 - a2 ö
 p a çç ÷÷ - b çç ÷÷
+ 4 cos  x 2 + 
2
2ac 2bc
 6 Þl = è ø è ø
 p  p c
⇒ F ′′(a) = −16a2 cos  a2 +  sin  a2 +  + 4 cos a sin a
 6  6 2(a2 - b2 ) 1
Þl = = (5)
 p 2c 2 2
+ 4 cos2  a2 + 
 6 Therefore, from Eqs. (4) and (5),
p   3 n 2m + 1
⇒ f (0) = 4 cos   = 4   = 3
2 = Þ n = (2m + 1)
 6  4 2 2
So,
Hence, the correct answer is (3).
(A)  → (P), (R), (S)
15.   Match the Column I to Column II.
Checking option (B):
Column I Column II x
(A) In ΔXYZ, let a, b and c be the lengths of the (P) 1
sides opposite to the angles X, Y and Z, c
b
sin( X - Y )
respectively. If 2(a2 - b2) = c2 and l = ,
sin Z
Y z
then possible values of n for which cos (nπλ) = 0 a
is (are)
Figure 24.46
(B) In ΔXYZ, let a, b and c be the lengths of the sides (Q) 2
1 + cos 2x - 2cos 2y = 2 sin x sin y
opposite to the angles X, Y and Z, respectively. If
1 + cos 2X - 2 cos 2Y = 2 sin X sin Y, then possible ⇒ 2 cos2 x - 2(2 cos2y - 1) = 2sin x sin y
a ⇒ 2 cos2 x - 4 cos2y + 2 = 2 sin x sin y
value(s) of is (are)
b ⇒ 2 sin2y - 2 sin x sin y + sin x sin y - sin2x = 0
⇒ 2 sin y(sin y - sin x) + sin x(sin y - sin x) = 0
(C) In  3, let 3i + j , i + 3 j , and b i + (1- b )j be (R) 3 ⇒ (sin y - sin x) (2 sin y + sin x) = 0
the position vectors of X, Y and Z with respect
⇒ b = a or 2b = -a (impossible)
to the origin O, respectively. If the distance of Z
 a
from the bisector of the acute angle of OX with ⇒ =1
b
 3
OY is , then possible value(s) of |β| is (are) So,
2
(B)→ (P)

Mathematical Problem Book for JEE.indb 1087 07-06-2018 14:34:13


1088 Mathematics Problem Book for JEE

Checking option (C): See Fig. 24.47. 2 2


é 4 ù
F (0 ) = ò (3 - 2 x )dx = ê3 x - x 3 / 2 ú
y
0 ë 3 û0
Y (1, 3)
B é 4 ù 8
= ê 6 - (2 2 )ú = 6 - 2
        ë 3 û 3
L
3/ 2 8
⇒ F (0) + 2 = 6 ⇒ (T )
3
Z x ( 3, 1)
and      F(1) = F(0) - area of ΔACD

æ 8 ö 1 8
= ç6 - 2 ÷ - (2)(1) = 5 - 2
x è 3 ø 2 3
O
8
Þ F (1) + 2 = 5 Þ (S )
Figure 24.47 3
  Therefore,
Vector along the bisector of acute angle between OX and OY is ,
(D) → (T), (S)
3i + j i + 3 j ( 3 + 1)   Hence, the correct matches are (A) → (P), (R), (S); (B) → (P); (C) →
+ = (i + j )
2 2 2 (P), (Q); (D) → (S), (T).

Slope of OB = tan(p /4 ) = 1 16. The area of the region {( x , y ) ∈  2 : y ≥ x + 3 , 5 y ≤ x + 9 ≤ 15}
⇒ Equation of OB is y = x
is equal to
Since, 1 4
(A)  (B) 
b - (1- b ) 3 6 3
ZL = 3/ 2 , Þ =
2 2 3 5
(C)  (D) 
⇒ |2b  - 1| = 3 2 3
⇒ (2b  - 1) = ± 3 [JEE ADVANCED 2016]
⇒ b = 2 or b = -1 Solution: It is given that
⇒ |b | = 1 or 2
Therefore,    (C)→ (P), (Q). y³ x +3
Checking option (D): See Fig. 24.48.
That is,
Y
y = 3x − 3 ìï ( x + 3), x ³ -3
x +3 = í
y=3−x îï - x - 3 , x < -3
y=x−1 y 2 = 4x
D C It is also given that
y=3
3 B 5y ≤ x + 9 ≤ 15
2
A That is,
1
X x + 9 ≤ 15 ⇒ x ≤ 6
1 2 3
5y ≤ 15 ⇒ y ≤ 3
x=2 5y ≤ x + 9
Figure 24.48 From Fig. 24.49, we have
y = |a x - 1| + |a x - 2| + a x; a ∈ {0, 1} y
Case (I) For a = 0, y = 3 (6, 3)
5y = x + 9 3 S
Case (II) For a = 1, y = |x - 1| + |x - 2| + x
P 9 3
ì 3 - x; x £1 R
ï Q x
Þ y = í x + 1; 1 < x < 2 (6, 0)
(−4, 0)(−3, 0)
ï3 x - 3; x ³2
î

Therefore, Figure 24.49

Mathematical Problem Book for JEE.indb 1088 07-06-2018 14:34:28


Chapter 24 | Area Under the Curves 1089

5y = x + 9 (1) 4. The area enclosed by the parabolas y = x2 - 1 and y = 1 - x2 is


and  (A) 1/3 (B) 2/3
 (C) 4/3 (D) 8/3
y = -x - 3
5. The area of the smaller segment cut off from the circle x2 + y2
That is, = 9 by x = 1 is
2 1
æ x +9ö (A)  (9 sec -1 3 - 8 ) (B)  9 sec -1(3) - 8
ç ÷ = -( x + 3) 2
è 5 ø
x2 + 81 + 18x = -25x - 75 (C)  8 - 9 sec -1(3) (D)  None of these
x2 + 43x + 156 = 0 6. The area of the region bounded by the curves y = |x - 2|, x = 1,
x = 3 and the x-axis is
(x + 39)(x + 4) = 0 ⇒ x = -4
(A) 4 (B) 2
(Since, x ≠ -39) (C)  3 (D) 1
Substituting the value of x in Eq. (1), we get the coordinates of 7. The area enclosed between the parabolas y2 = 4x and x2 = 4y is
point P as follows: 14 3
(A)  sq. units (B)  sq. units
3 4
5y = -4 + 9 ⇒ y = 1 ⇒ P(-4, 1)
3 16
The area of trapezium PQRS is (C)  sq. units (D)  sq. units
16 3
1 8. The area bounded by the curves y2 = 8x and y = x is
´ 10 ´ 4 = 20
2 128 32
(A)  sq. units (B)  sq. units
3 3
Hence, the area of the given region is
64
-3 6 (C)  sq. units (D)  32 sq. units
3
20 - ò - x - 3 dx - ò x + 3 dx
9. The area bounded by the curves y = loge x and y = (loge x)2 is
-4 -3

-3 6
(A) 3 - e (B)  e - 3
2 2 1 1
= 20 + ( - x - 3)3 / 2 - ( x + 3)3 / 2 (C)  (3 - e ) (D)  (e - 3)
3 -4 3 -3 2 2

2 2 10. The area between the parabola y2 = 4ax and x2 = 8ay is
= 20 + (0 - 1) - 93 / 2
3 3 8 2 4 2
(A)  a (B)  a
3 3
2 2
= 20 - - ´ 27 32 2 16 2
3 3 (C)  a (D)  a
3 3
2 4 11. The area of the region bounded by the curves y = x2 and y = |x | is
= 2 - = sq. units.
3 3 (A) 1/6 (B) 1/3
Hence, the correct answer is option (B). (C) 5/6 (D) 5/3
12. The area bounded by curves y = cos x and y = sin x and
p
ordinates x = 0 and x = is
Practice Exercise 1 4
(A)  2 (B)  2 +1
 1. The area of the region bounded by y = |x - 1| and y = 1 is
(C)  2 - 1 (D)  2 ( 2 - 1)
(A) 2 (B) 1
1 13. The area in the first quadrant between x2 + y2 = p 2 and
(C)  (D)  None of these y = sin x is
2
(p 3 - 8 ) p 3
 2. The area between the curve y2 = 4ax, x-axis and the ordinates (A)  (B)   
x = 0 and x = a is 4 4
4 2 8 2 (p 3 - 16 ) (p 3 - 8 )
(A)  a (B)  a (C)  (D) 
3 3 4 2
2 2 5 2 14. The area bounded by the curves y2 - x = 0 and y - x2 = 0 is
(C)  a (D)  a
3 3 7 1
(A)  (B) 
3. The area of the curve xy2 = a2(a - x) bounded by y-axis is 3 3
(A)  pa2 (B) 2pa2 5
(C)  (D) 1
(C) 3pa2 (D) 4pa2 3

Mathematical Problem Book for JEE.indb 1089 07-06-2018 14:34:53


1090 Mathematics Problem Book for JEE

15. The area of region {(x, y) : x2 + y2 ≤ 1 ≤ x + y} is 27. If the area above the x-axis, bounded by the curves y = 2kx and
p 2 p 2 3
(A)  (B)  x = 0 and x = 2 is , then the value of k is
5 2 ln 2
p 2 p 1 1
(C)  (D)  - (A)  (B) 1
3 4 2 2
16. Area under the curve y = sin 2x + cos 2x between x = 0 and (C)  -1 (D) 2
p 28. The area bounded by the x-axis, the curve y = f(x) and the lines
x = is
4 x = 1, x = b is equal to b2 + 1 - 2 for all b > 1, then f(x) is
(A)  2 sq. units (B)  1 sq. units
(C)  3 sq. units (D)  4 sq. units (A)  x -1 (B)  x +1
17. Area under the curve y = 3 x + 4 between x = 0 and x = 4 is x
(C)  x 2 + 1 (D) 
56 64 1+ x 2
(A)  sq. units (B)  sq. units
9 9 29. The area bounded by the circle x2 + y2 = 4, line x = 3 y and
(C)  8 sq. units (D)  None of these x-axis lying in the first quadrant is
a2 p p
18. If area bounded by the curves y2 = 4ax and y = mx is , then (A)  (B) 
3 2 4
the value of m is p
(A) 2 (B)  -2 (C)  (D)  p
3
1
(C)  (D)  None of these 30. Area bounded by the curve y = log x, x-axis and the ordinates
2 x = 1, x = 2 is
19. Area bounded by parabola y2 = x and straight line 2y = x is (A)  log 4 sq. units (B)  (log 4 + 1) sq. units
4 (C)  (log 4 - 1) sq. units (D)  None of these
(A)  (B) 1
3 2
31. Area bounded by the curve y = xe x  , x-axis and the ordinates
2 1 x = 0, x = a is
(C)  (D) 
3 3 2 2
ea + 1 ea - 1
20. Area bounded by lines y = 2 + x, y = 2 - x and x = 2 is (A)  sq. units (B)  sq. units
2 2
(A) 3 (B) 4 2 2
(C)  8 (D) 16 (C)  e a  + 1 sq. units (D)  e a  - 1 sq. units
21. The ratio of the areas bounded by the curves y = cos x and 32. Area bounded by the curve y = sin x between x = 0 and x = 2p is
y = cos 2x between x = 0, x = p/3 and x-axis is (A)  2 sq. units (B)  4 sq. units
(A)  2 : 1 (B)  1 : 1 (C)  8 sq. units (D)  None of these

(C)  1 : 2 (D)  2 : 1 33. Area bounded by the parabola y = 4x2, y-axis and the lines
22. The area bounded by the x-axis and the curve y = sin x and y = 1, y = 4 is
x = 0, x = p is 7
(A)  3 sq. units (B)  sq. units
(A) 1 (B) 2 5
(C)  3 (D) 4 7
23. The area bounded by the parabola y2 = 4ax, its axis and two (C)  sq. units (D)  None of these
3
ordinates x = 4, x = 9 is
(A) 4a2 (B) 4a2.4 34. Area bounded by the lines y = x, x = -1, x = 2 and x-axis is
152 a 5 3
(C) 4a2 (9 - 4) (D)  (A)  sq. units (B)  sq. units
3 2 2
24. For 0 ≤ x ≤ p, the area bounded by y = x and y = x + sin x is 1
(C)  sq. units (D)  None of these
(A) 2 (B) 4 2
(C) 2p (D) 4p 35. If the ordinate x = a divides the area bounded by the curve
25. The area of the region bounded by the x-axis and the curves
æ 8 ö
defined by y = tan x, (-p / 3 ≤ x ≤ p / 3) is y = ç 1+ 2 ÷ , x-axis and the ordinates x = 2, x = 4 into two
è x ø
(A)  log 2 (B)  -log 2
equal parts, then a =
(C) 2log2 (D) 0 (A) 8 (B)  2 2
26. If a curve y = a x + bx passes through the point (1, 2) and the (C) 
2 (D)  2
area bounded by the curve, line x = 4 and x-axis is 8 sq. units,
then 36. Area between the curve y = cos x and x-axis when 0 ≤ x is
(A)  a = 3, b = -1 (B)  a = 3, b = 1 (A) 2 (B) 4
(C)  a = -3, b = 1 (D)  a = -3, b = -1 (C) 0 (D) 3

Mathematical Problem Book for JEE.indb 1090 07-06-2018 14:35:13


Chapter 24 | Area Under the Curves 1091

37. Area bounded by curve y = x3, x-axis and ordinates x = 1 and 48.  Let C1 and C2 be the graphs of the function y = x 2 and y = 2x,
x = 4 is 0 ≤ x ≤ 1 respectively. Let C3 be the graph of a function y =
(A)  64 sq. units (B)  27 sq. units f(x); 0 ≤ x ≤ 1, f(0) = 0. For a point P on C1, let the lines through
127 255 P parallel to the axis, meet C2 and C3 at Q and R, respectively
(C)  sq. units (D)  sq. units
4 4 (see Fig. 24.50). If for every position of P on (C1), the areas
38. Area bounded by curve xy = c, x-axis between x = 1 and x = 4 is of shaded region OPQ and ORP are equal, determine the
(A)  c log 3 sq. units (B) 2log c sq. units function f(x).
(C) 2c log 2 sq. units (D) 2c log 5 sq. units
y
39. Area bounded by curve y = k sin x between x = p and x = 2p is
(A) 2k sq. units (B) 0  1 , 1
 
2 
k2 (0, 1) (1, 1)
(C)  sq. units (D)  k sq. units C2 C1
2
Q
40. Area bounded by y = x sin x and x-axis between x = 0 and P
x = 2p is
(A) 0 (B) 2p sq. units
(C)  p sq. units (D) 4p sq. units O (1, 0) x
41. The ratio in which the area bounded by the curves y2 = x and
R
1 C3
x2 = y is divided by the line x = is
2
Figure 24.50
4 2 -1 3 2 +3
(A)  (B)  49. Let f(x) = maximum {x2, (1 - x)2, 2x (1 - x)} where 0 ≤ x ≤ 1.
9-4 2 9-4 2
Determine the area of the region bounded by the curves
2 -1 2 2 -1 y = f (x), x-axis, x = 0 and x = 1.
(C)  (D) 
3 -1 3 3 -1 Let An be the area bounded by the curve y = (1 + tan x)n and the
50. 
42. The area of the curve x + |y| = 1 and the y-axis is p 1
lines x = 0, y = 0, and x = . Prove that for n ³ 2, An + An -2 =
(A)  1 sq. unit (B)  2 sq. units 4 n -1

1 1 1
(C)  sq. units (D)  2 sq. units and deduce < An < .
2 2n + 2 2n - 2
43. The area bounded by the curve y = e| x |, y = e-| x |, x ≥ 0 and x ≤ 5 is 51. In what ratio does the x-axis divide the area of the region
bounded by the parabolas y = 4x - x2 and y = x2 - x.
(A)  e5 + e-5 + 2 sq. units (B)  e5 + e-5 - 2 sq. units
(C)  e5 - e-5 + 2 sq. units (D)  e5 - e-5 - 2 sq. units 2
52. Sketch the region bounded by y = x2 and y = and find
44. Find the area of quadrilateral, combined equation of whose (1+ x 2 )
its area.
sides are (x2 - y2)(x2 - y2 - 8x + 16)
(A) 8 (B) 4 53. 
Sketch the curves and identify the region bounded by
(C)  2 2 (D) 9 x = 1/ 2 , x = 2, y = loge x and y = 2x. Find area of region.

45. Let f be a real valued function satisfying Compute the area of the region bounded by the curves y = ex
54. 
æ ln x ö
æxö f (1+ x ) In x, and y = ç ÷.
f ç ÷ = f ( x ) - f ( y ) and lim =3 è ex ø
y
è ø x ®0 x
55. Find all the maxima and minima of the function f(x) = x(x - 1)2,
Find the area bounded by the curve y = f(x), the y-axis and the (0 ≤ x ≤ 2). Also determine the area bounded by the curve
line y = 3. y = x(x - 1)2, the y-axis and the line y = 2.
Let An be the area bounded by y = tann x, x = 0, y = 0 and
46.  56. 
Find the area of the region bounded by the curve whose
x = p/4. Prove that for n ≥ 2. equation is y = tan x, its tangent drawn at x -p /4 and the
1 1 1 x-axis.
(i)  An + An - 2 = (ii)  < An <
n -1 2(n + 1) 2(n - 1) 57. Find the area bounded by the curves x 2 + y 2 = 4 , x 2 = - 2 y
47. Let f (x) be a continuous function given by and x = y.
ìï 2x x £1 58. The area bounded by the curves y = |x| - 1 and y = -|x| + 1 is
f (x) = í 2
îï x + ax + b , x > 1 (A) 1 (B) 2 (C) 2 2  (D) 4
Find the area of the region in the third quadrant bounded by the 59. 
Let g(x) be a function defined on [-1, 1]. If the area of the
curve x = -2y2 and y = f (x) lying on the left of the line 8x + 1 = 0. equilateral triangle with two of its vertices at (0, 0) and [x, g(x)]

Mathematical Problem Book for JEE.indb 1091 07-06-2018 14:35:29


1092 Mathematics Problem Book for JEE

is
3
, then the function g (x) is Practice Exercise 2
4
Single/Multiple Correct Choice Type Questions
(A)  g(x) = ± 1- x 2 (B)  g(x) = 1- x 2
1. For which of the following values of m, is the area of the region
(C)  g(x) = - 1- x 2 (D)  g(x) = 1+ x 2 9
bounded by the curve y = x - x2 and the line y = mx equals ?
Area bounded by y = g(x), x-axis and the lines x = -2, x = 3,
60.  2
ìïmax i : {f (t ); - 2 £ t £ x} , - 2 £ x < 0 (A)  - 4 (B)  - 2 (C) 2 (D) 4
where g( x ) = í
ïîmin i : {f (t ); 0 £ t £ x} , 0 £ x < 3 2. Three straight lines are drawn through a point M, lying in the
and f(x) = x2 - | x |, is equal to interior of triangle ABC, parallel to its sides. The areas of the
resulting three triangles are S1, S2 and S3. The area of triangle
113 111 ABC is
(A)  sq. units (B)  sq. units
24 24 (A)  S1 + S2 + S3 (B)  ( S1 + S2 + S3 )2
117 121
(C)  sq. units (D)  sq. units ( S1 + S2 + S3 )3 / 2
24 24 (C)  (D)  None of these
S1 + S2 + S3
61. 
Area of the region that consists of all the points satisfying the
conditions |x - y| + |x + y| ≤ 8 and xy ≥ 2 is equal to Comprehension Type Questions
(A) 4(7 - ln 8) sq. units (B) 4(9 - ln 8) sq. units
Paragraph for Questions 3–5: Let f: R → R be a continuous
(C) 2(7 - ln 8) sq. units (D) 2(9 - ln 8) sq. units and bijective function defined such that f(α) = 0 (α ≠ 0). The area
62. Two lines draw through the point P (4, 0) divide the area bounded by y = f(x), x = α, x = α - t is equal to the area bounded by
px y = f(x), x = α , x = α + t ∀ t ∈ R, then
bounded by the curves y = 2 sin and x-axis, between
4 3. Graph of y = f(x) is symmetrical about point
the line x = 2 and x = 4, in to three equal parts. Sum of the (A)  (0, 0) (B) (0, a)
slopes of the drawn lines is equal to (C) (a, 0) (D) (a, a)
2 2 2 2 4 2 4. The value of f(2a) is equal to
(A)  -  (B)  - (C)  -  (D)  -
p p p p (A)  f(a)  (B)  -f(a)
(C)  f(0) (D)  -f(0)
The area bounded by the curve y = x (3 - x)2, the x-axis and
63. 
b
the ordinates of the maximum and minimum points of the
òf
-1
5. The value of (t ) dt is equal to
curve is -b
(A)  2 sq. units (B)  6 sq. units (A)  0 (B) 2ab
(C)  4 sq. units (D)  8 sq. units (C)  ab (D)  None of these
64. 
What is the area of a plane figure bounded by the points of the Paragraph for Questions 6–8: Let f(x) be a polynomial of degree
lines max (x, y) = 1 and x2 + y2 = 1? 4 satisfying
p p
(A) 1− sq. units (B) 1 − sq. units æx öæ x ö æx öæ x ö
2 3 ç ò A(t ) B(t )dt ÷ ç ò C (t ) D(t ) dt ÷ - ç ò A(t ) C (t ) dt ÷ ç ò B(t ) D(t ) dt ÷
ç ÷ç ÷ ç ÷ç ÷
p è1 øè 1 ø è1 øè 1 ø
(C) 1 − sq. units (D) 1 − p sq. units
4 = f(x) ∀ x ∈ R
The area bounded by the curve y = (x - 1) (x - 2) (x - 3) lying
65.  where A(x), B(x), C(x) and D(x) are non-constant continuous and
between the ordinates x = 0 and x = 3 is differentiable functions. It is given that the leading coefficient
7 (coefficient of x4) of f(x) is 1.
(A)  sq. units (B)  4 sq. units
4 6. The area included between the line y = x - 1 and the curve
11 y = f(x) is
(C)  sq. units (D)  3 sq. units
4 2 3
(A)  sq. units (B)  sq. units
The area common to the curves y = and y = x is
66.  x3 5 10
(A) 2 (B) 4 7 7
(C)  sq. units (D)  sq. units
(C) 8 (D)  None of these 10 5
67. The area of the region consisting of points (x, y) satisfying 7. The area of the smaller region intercepted between the curve
|x ± y | ≤ 2 and x2 + y2 ≥ 2 is y = f(x) and x2 + y2 = 1 is
(A) 8 - 2p sq. units (B) 4 - 2p sq. units p 1 p
(A)  - sq. units (B)   sq. units
(C) 1 - 2p sq. units (D) 2p sq. units 4 5 4

Mathematical Problem Book for JEE.indb 1092 07-06-2018 14:35:46


Chapter 24 | Area Under the Curves 1093

p 1 p 1 15. lim f ( x ) is equal to
(C)  + sq. units (D)  + sq. units x →∞
4 5 2 5
(A)  -∞ (B)  ∞
8. The area included between y = f(x) and y + 2 = 0 between the (C)  0 (D) 1
ordinates x = 0 and x = 3 is x
æ 1ö
3 33 16. If g(x) = f (t ) dt, then g(n) - g ç ÷ ; n ∈ N is equal to
(A)  sq. units (B)  sq. units 1
ènø
5 5
(A)  -1 (B) 1
23 63 1
(C)  sq. units (D)  sq. units 0 (D)  n +
(C) 
10 5 n
Paragraph for Questions 9–11: ABCD is a square of side length 17. The area bounded between the curve y = f(x) and y = ex lnx is
2 units and the centre of square is at origin. C2 is a circle passing
2e - 5 2e 2 - 5e
through vertices A, B, C, D and C1 are the circle touching all the (A)  (B) 
8 4
sides of square ABCD. Line L1 is tangent at A line L2 is tangent at
D on circle C2 who intersects at K, where A, B, C, D lie in 2nd, 1st, 4th e2 + 5
(C)  (D)  None of these
and 3rd quadrant. Point Q is variable point on C2, let perpendicular 4e
drawn from Q to cut the line L1 and L2 at E and F, respectively. Given
that AB, BC, CD and AD are parallel to the coordinate axes. Matrix Match Type Questions
9. The maximum area of rectangle QEKF is
18. Match the following:
9 8
(A)  (B) 
4 3 Column I Column II
9 (A) The area enclosed between the curves 24
(C)  (D)  None of these (p) 
5 |x| + |y| = 2 and x2 = y in sq. units is 5
10. Area of DBQC, (where Q is such that the area of the rectangle
(B) The maximum value of the function f(x) = 3 7
QEKF is maximum) is 7 (q) 
sinx - 4 cosx - will be given by 3
5-2 2 2+3 2 3
(A)  (B) 
2 2 2 2
(C) The length of common chord of two circles 16
5+2 3 of radii 3 and 4 units which intersect orthog- (r) 
(C)  (D)  None of these 3
2 onally is
11. Locus of point which are equidistant from Q and line L1 (D) 
The length of chord intercepted by the 8
intersect the line y = x at M (other than origin), then area of parabola y2 = 4(x + 1) passing through its (s)  3
DOQM (where Q is such that the area of the rectangle QEKF is focus and inclined at 60° with positive x-­
maximum) is axis is
(A) 1 (B) 2    (C) 3 (D)  None of these
Paragraph for Questions 12–14: Let a function f(x) satisfies the 19. Match the following:
f (x) + f (y)
condition f ( x + y ) = such that f(0) = 2 and f(x) ≥ 0. Column I Column II
f (x)
12. The curve y = f(x) is sin x
p
(A) f(x) = t 2dt , then period of f ′(x) is (p) 
(A)  y = 2( x + 1) (B)  y = 2 ( x + 1) 0 14
(C)  y = ln(x + 1) (D)  y = ln(x - 1)
13. Area bounded between y = f(| x |) and y = 7 - | x | is (B) If area of ellipse b2x2 + a2y2 = a2b2 (a > b), p
enclosed by x-axis and the ordinates x = 0 and (q) 
23 11 2
(A)  sq. units (B)  sq. units x = b be 1/8 th the area of entire ellipse, then
6 6
e 1- e 2 + sin-1 1- e 2 is equal to
86
(C)  sq. units (D)  7 sq. units
6 æ 1ö p
cosec -1 x + cos -1 ç ÷ (r) 
14. The number of points where g(x) = max {f(x), 6, 7 - |x|} is non- è x ø . Then 4
(C) Let f ( x ) =
differentiable ∀ x ∈ [-10, 10] are cosecx
(A) 5 (B) 6 (C) 7 (D) 8 greatest value of f(x) is

Paragraph for Questions 15–17: Let f be function satisfying the (s) 2p


-1 æ æ 46p öö
f (x) f (y) (D) cos ç sin ç ÷ ÷ is
condition f(xy) = + ∀ x, y > 0. If f(x) is differentiable and è è 7 øø
y x
f(1) = 1, then

Mathematical Problem Book for JEE.indb 1093 07-06-2018 14:36:10


1094 Mathematics Problem Book for JEE

20.  Match the following: (C) The area bounded by the loop of 4y2 = x2 (4 - x2) is 1
(r) 
Column I Column II 2
16
æ 1ö (s) 
(A) If y = tan-1 ç ÷ + tan-1(b ), (0 < b < 1) and (p) 3 3
è2ø
p
0 < y ≤ , then the maximum value of b will be Integer Type Question
4
21. Let f(x) be a polynomial of degree 3 if the curve y = f(x) has
2
relative extremities at x = ± and passes through (0, 0)
(B) The number of solutions of sin4 x + cos3 x ≥ 1 in (q)  1 3
(0, 2p) will be 3 and (1, -2) dividing the circle x + y2 = 4 in two parts. Then the
2

integral part of areas of these two parts is ______________.

Answer Key

Practice Exercise 1
1. (B) 2. (B) 3. (A) 4. (D) 5. (B)
6. (D) 7. (D) 8. (B) 9. (A) 10. (C)
11. (B) 12. (C) 13. (A) 14. (B) 15. (D)
16. (B) 17. (D) 18. (A) 19. (A) 20. (B)
21. (D) 22. (B) 23. (D) 24. (A) 25. (C)
26. (A) 27. (B) 28. (D) 29. (C) 30. (C)
31. (B) 32. (B) 33. (C) 34. (A) 35. (B)
36. (B) 37. (D) 38. (C) 39. (A) 40. (D)
41. (A) 42. (A) 43. (B) 44. (A) 45. 3e sq. units
 2
17 121 52.  p −  sq. units
47. 761 sq. units 48. x3 - x2 49. sq. units 51. 3
192 27 4

e2 - 5 10 æ 1ö æ 1ö 1
53. ( 4 - 2 ) + 3 - æ 5 ö ln 2 54. sq. units 55. sq. units 56. ç ÷ log 2 - ç ÷ 57. p+
ç ÷ 3 2 3
ln 2 2 è2ø 4e è ø è4ø
58. (B) 59. (A) 60. (A) 61. (A) 62. (A)
63. (C) 64. (C) 65. (C) 66. (D) 67. (A)

Practice Exercise 2
1. (B), (D) 2. (B) 3. (C) 4. (D) 5. (B)
6. (B) 7. (A) 8. (D) 9. (D) 10. (D)
11. (A) 12. (B) 13. (C) 14. (A) 15. (C)
16. (C) 17. (A) 18. (A) → (q), (B) → (s), (C) → (p), (D) → (r) 19. (A) → (s), (B) → (r), (C) → (q), (D) → (p)
20. (A) → (q), (B) → (p), (C) → (s) 21. (6)

Solutions
Practice Exercise 1 é 1 ù é æ 1 öù 1 1
= ê1- ú + ê - ç - 1÷ ú = + = 1 sq. unit
1.  y = x - 1, if x > 1 and y = -(x - 1), if x < 1 ë 2 û ë è 2 øû 2 2

é 2 ù1
é x2 ù
2
1 2 x
Area = ò (1- x )dx + ò ( x - 1)dx = ê x - ú + ê - x ú a 2 3/2 a 8 a 8
× a a = a2
0 1
êë 2 úû 0 êë 2 úû1 2. Required area = 2 ò 4 axdx = 4 a ´ [ x ]0 =
0 3 3 3

Mathematical Problem Book for JEE.indb 1094 07-06-2018 14:36:21


Chapter 24 | Area Under the Curves 1095

8
2 8 a 8 é 4 2 3 / 2 x 2 ù 128 64 32
= 4 a ´ [ x 3 / 2 ]a0 = × a a = a2 sq. units 8
3 3 3 ∫0 (2 2 x − x )dx = êê 3 x - 2 úú = 3 - 2 = 3 sq. units
ë û0
3. Since the curve is symmetrical about x-axis, therefore,
 9. For points of intersection,
a a- x
Required area A = 2 ò a dx
log x = (log x ) ⇒ log x (log x − 1) = 0
2
0 x
Put x = a sin2 q. Then ⇒ x = 1, x = e
dx = 2a sinq . cosq dq So,
e
p /2 2
a cos q p /2 cosq Required area A = ò [log x - (log x )2 ] dx
A = 2ò a a sin 2q dq = 2a2 ò 2 sinq cosq dq 1
0 a sin2 q 0 sinq e e
A = ò log x dx - ò (log x )2 dx
p /2 1 p 1 1
A= 4 a2
0ò cos2 q dq Þ A = 4 a2 . . = p a2 sq. units
2 2 = [ x log x - x ] 1e - [ x (log x )2 - 2 x log x + 2 x ] 1e
4. Given parabolas are x2 = 1 + y, x2 = 1 - y = [e - e - (-1)] - [e(1)2 - 2e + 2e - (2)]
For points of intersection, we have = (1) - (e - 2) = 3 - e
1+ y = 1 − y ⇒ 2 y = 0 ⇒ y = 0 10.  For points of intersection,
⇒ x 2 = 1⇒ x = ±1  x2 
2

 8a  = 4 ax ⇒ x = 256a x
4 2

So,
1 é x3 ù 8
Required area = 4 ò (1- x 2 ) dx = 4 ê x - ú = sq. units
1
(
⇒ x x 3 − 256a3 = 0 )
0
êë 3 úû 0 3 ⇒ x = 0 , x = a.28/3

3 So,
5. Area of smaller part I = 2 ò 9 - x 2 dx
1 ( a 28 / 3 ) a 28 / 3 x2 32a2
3 Required area, A = ò 4 axdx - ò dx =
1é xù é p æ 1 öù 0 0 8a 3
= 2 × ê x 9 - x 2 + 9 sin-1 ú = ê9 - 8 - 9 sin-1 ç ÷ ú
2ë 3 û1 ë 2 è 3 øû 11.  See Fig. 24.51.
é æp æ 1öö ù é æ 1ö ù Y
= ê9 ç - sin-1 ç ÷ ÷ - 8 ú = ê9 cos -1 ç ÷ - 8 ú
ë è 2 è 3 ø ø û ë è 3 ø û

= [9 sec -1(3) - 8 ] sq. units


3 2 3
6. Required area = ∫ | x − 2 | dx = ∫ (2 − x ) dx + ∫ ( x − 2)dx
1 1 2

2 3
é x2 ù é x2 ù 1 1 X' X
= ê2 x - ú + ê - 2 x ú = + = 1 sq. unit
êë 2 úû1 êë 2 úû 2 2 2
Figure 24.51
7. Equations of curves y2 = 4x and
= 4y. The given equations
x2
Required area = 2 × (shaded area in first quadrant)
x2
may be written as y = 2 x and y = . 1 1 1
4 = 2 ò ( x - x 2 ) dx = 2 ´ = sq. units
0 6 3
For points of intersection,
2 12. Given equations of curves y = cos x and y = sin x and ordinates
 x2  x = 0 to x = p /4.
  = 4 x ⇒ x = 64 x
4

4
We know that area bounded by the curves is
⇒ x ( x 3 − 64 ) = 0
x2 p /4 p /4
⇒ x = 0, x = 4
∫x 1
ydx = ∫
0
cos xdx − ∫
0
sin x dx
We know that area enclosed by the parabolas is
= [sin x ]p0 /4 - [ - cos x ]p0 /4
4 4 x2 32 16 16
∫0 2 x dx − ∫
0 4
dx =
3
− =
3 3
sq. units
æ p ö æ p ö æ 1 ö æ 1 ö
= ç sin - sin 0 ÷ + ç cos - cos 0 ÷ = ç -0÷ +ç - 1÷
4 4
8. y2 = 8x and y = x ⇒ x2 = 8x ⇒ x = 0, 8   è ø è ø è 2 ø è 2 ø

Therefore, required area is = 2 - 1 sq. units

Mathematical Problem Book for JEE.indb 1095 07-06-2018 14:36:48


1096 Mathematics Problem Book for JEE

p (p2) p 3 2
13. Area of the circle in first quadrant is , that is, . Also 2  y3 2 4
∫0
2
4 4 ( y − 2 y )dy =  3 − y  = 3 sq. units
 0
area bounded by curve y = sin x and x-axis is 2 sq. units
p3 p3 -8 20. Obviously, triangle ACB is right angled at C. See Fig. 24.52.
Hence, required area is -2 = .
4 4 Y

14.  For points of intersection A


(x2)2-x = 0
x (x3-1) = 0
⇒ x = 0, x = 1 C
(0,2)
æ x 3/2 x 3 ö1
1 1
ò0 ( x - x 2 )dx = çç
3 / 2
- ÷÷ = sq. units
3 3
è ø0
X
15.  x2 + y2 = 1, x + y = 1 meet when O B
(2,0)
x2 + (1 - x)2 = 1 ⇒ x2 + 1 + x2 - 2x = 1
⇒ 2x2 - 2x = 0 ⇒ 2x(x - 1) = 0 Figure 24.52
⇒ x = 0, x = 1 ⇒ y = 1, y = 0, that is, A(1, 0); B(0, 1)
Therefore,
1
1
Required area = ò [ 1- x - (1- x )] dx 2 Required area = ´ AC ´ BC
0 2
1 1
é x 1- x 2 1 = ´ 2 2 ´ 2 2 = 4 sq. units
x2 ù 2
=ê + sin-1 x - x + ú
êë 2 2 2ú
û0 p /3 3 p /3 3
21.  A1 = ò cos x dx = , A2 = ò cos 2 x dx =
1 p 1 æp 1ö
0 2 0 4
= × - 1+ = ç - ÷ sq. units
2 2 2 è 4 2ø Therefore, A1 : A2 = 2 : 1.

p /4
22. The curve is symmetric about x-axis,
16. Required area = ò (sin 2 x + cos 2 x )dx Therefore, required area is
0
p /2
é cos 2 x sin 2 x ù
p /4 2∫ sin x dx = 2[ − cos x ]p0 / 2 = 2 sq. units
= ê- + 0
ë 2 2 úû 0
9
23. Required area, A = 2 ò 4 axdx
1 p p  4
=  − cos + sin + cos 0 − sin 0  = 1 sq. unit
2 2 2  2 152 a
A = 4 a ´ [ x 3 / 2 ]94 = sq. units
3 3
4
4 (3 x + 4 )3 / 2 24. The curves y = x and y = x + sinx intersect at (0, 0) and (p, p).
17. Area = ò 3 x + 4 dx =
0 3.(3 / 2) 0 Hence, area bounded by the two curves is
p p p
2 112

= ´ 56 =
9 9
sq. units ∫ ( x + sin x )dx − ∫ x dx = ∫ sin x dx
0 0 0

æ 4a 4a ö = [ - cos x ]p0 = - cos p + cos 0 = -( -1) + (1) = 2 sq. units


18. The two curves y2 = 4ax and y = mx intersect at ç 2 , ÷
èm m ø
p /3
and the area enclosed by the two curves is given by 25. Required area = 2 ò tan x dx = 2[logsec x ]p0 / 3 = 2 log(2) sq. units
0
4 a / m2
∫0 ( 4 ax − mx ) dx 26. Given curve y = a x + bx . This curve passes through (1, 2).

Therefore, Therefore, 2 = a + b.  (1)

4 a / m2 a2 Area bounded by this curve and line x = 4 and x-axis is 8


∫0 ( 4 ax − mx ) dx =
3 sq. units, then
4

Þ
8 a2 a2
= Þ m3 = 8 Þ m = 2
ò 0 (a x + bx ) dx = 8
3 m3 3
2a 3 / 2 4 b 2 4 2a
19. y2 = x and 2y = x ⇒ y2 = 2y ⇒ y = 0, 2 Þ [ x ]0 + [ x ]0 = 8 , × 8 + 8b = 8
3 2 3
Therefore, required area is   ⇒ 2a + 3b = 3
  (2)

Mathematical Problem Book for JEE.indb 1096 07-06-2018 14:37:12


Chapter 24 | Area Under the Curves 1097

From equations (1) and (2), we get a = 3, b = -1. x 0 π/6 π/2 π 3 π/2 2π
2 3 y -1

27.  2kx dx
0
=
log 2
⇒ 22k − 1 = 3k . Now check from options, only 0 0.5 1 0 0

Join these points with a free hand to obtain a rough sketch.


(b) satisfies the above condition.
b Y
28. ò f ( x ) dx = b2 + 1 - 2 = b2 + 1 - 1+ 1 = [ x 2 + 1]1b
1 A
Therefore,
B D
X
d 2x x O π 2π
f (x) = x2 +1 = =
dx 2 x2 +1 x2 +1
C

29. See Fig. 24.53.


Figure 24.54
Required area = (area of OAB) + (area of BCD)
p 2p
= ∫ y dx + ∫ ( − y ) dx
3 ,1
)   0 p

A (
(As area BCD is below x-axis)
p 2p
= ò sin x dx - ò sin x dx = 4 sq. units
O B C 0 p

4 4 y
33. Required area = ò1 x dy = ò1 2
dy

1 2 7
= . | y 3 / 2 |14 = sq. units
2 3 3

Figure 24.53 34. See Fig. 24.55. Required area is


2 0 2 5
Required area = Area of ∆ OBA + Area of  BAC ò-1 y dx = ò-1 y . dx + ò0 y . dx = 2 sq. units

1=
1
2
( ) ∫
3 ×1 + 4 − x 2 dx
Y

3 3 2p x=2
= +p − − (−1, 0)
2 2 3 X
x=1 (2, 0)
p
= sq. units
3

30. Given curve y = log x and x = 1, x = 2. Figure 24.55


2
Hence, required area = ò log x dx = ( x log x - x )12 35. See Fig. 24.56. Let the ordinate at x = a divide the area into two
1
equal parts
      = 2log 2 - 1 = (log 4 -1) sq. units
Y
a a 2
31. Required area is ò0 y dx = ò xe x dx
0

dt (2, 3) A C
 3
B  4, 
We put x 2 = t Þ dx = as x = 0 ⇒ t = 0 and x = a ⇒ t = a2,  2
2x
then it reduces to
X
O M D N
a2 (2, 0) (a, 0) (4, 0)
1 a2 t 1 2 e -1
2 ò0
e dt = [e t ]a0 =
2 2
sq. units
Figure 24.56
4
32. See Fig. 24.54. 4æ 8 ö é 8ù
Area of AMNB = ò ç 1+ 2 ÷ dx = ê x - ú = 4
We have y = sin x
2 è x ø ë x û2

Mathematical Problem Book for JEE.indb 1097 07-06-2018 14:37:27


1098 Mathematics Problem Book for JEE

aæ 8 ö A1 4 2 − 1
Area of ACDM = ò2 ç 1+ 2 ÷ dx = 2 Therefore, =
è x ø A2 9 − 4 2

On solving, we get a = ±2 2. Since a > 0 ⇒ a = 2 2. Y x2 = y

é pù y2 = x
36. y = cos x, When x Î ê0 , ú , cos x ³ 0
ë 2û (1,1)

é p 3p ù
When x Î ê , ú , cos x £ 0
ë2 2 û . X′ X
(0,0)
é 3p ù x = 1/2
When x Î ê , 2p ú , cos x ³ 0
ë 2 û
Figure 24.58
Thus required area is given by,
42. See Fig. 24.59.
2p p /2 3p / 2 2p
∫0 ydx = ∫
0
cos x dx + ∫
p /2
( − cos x )dx + ∫
3p / 2
cos xdx x + |y| = 1 ⇒ |y| = 1 - x
æ1 ö
= 1 + 2 + 1 = 4 sq. units Area = 2 ´ ç ´ 1´ 1÷ = 1 sq. unit
è 2 ø
4 Y
4 é x4 ù 255
37. Required area = ò x 3dx = ê ú = sq. units
1
êë 4 úû1 4
(0, 1)
4 4 1
38. Required area = ò y dx = c ò dx = 2c log 2 sq. units
1 1 x
X
2p (1, 0)
39. Required area = k ò sin x dx = k [ - cos x ]p2p = -2k
p

Hence, area = 2k sq. units


40. See Fig. 24.57. Required area is
Y'
p 2p
A1 + A2 = ò y dx + òp y dx = 4p sq. units Figure 24.59
0
43. See Fig. 24.60.
Y 5
(
A = ∫ e x − e − x dx )
0
A1 2π
O X
π A2 5
= e x + e− x
      0

     = e5 + e-5 - 2 sq. units

Figure 24.57 Y

41. See Fig. 24.58.


1/ 2 1/ 2
2 x 3/2 x 3
ò
2
A1 = x -x = - X' X
0
3 3 0

1é 2 1 ù 1( 4 2 - 1) æ 4 2 - 1 ö
= ê - ú= = çç ÷÷ Y'
3 ë2 2 8 û 3´ 8 è 24 ø
Figure 24.60
1
1 é 2 3/2 x 3 ù 44. See Fig. 24.61.
A2 = ò x - x2 = ê x - ú
êë 3 3 úû1/ 2 (x2 - y2)(x2 - y2 - 8x + 16) = 0
1/ 2
(x2 - y2)[(x - 4)2 - y2] = 0
é 2 1 ù é 4 2 - 1ù 1 4 2 − 1
=ê - ú-ê ú = −
( ) y = ± x
ë 3 3 û êë 24 úû 3 y = ± (x - 4)
24
y = x - 4
8 − 4 2 +1 9 − 4 2 y = -x + 4
= =
24 24 1
Area = ´ 2 ´ 4
2

Mathematical Problem Book for JEE.indb 1098 07-06-2018 14:37:46


Chapter 24 | Area Under the Curves 1099

Total area = 2 × 4 = 8 sq. units (ii) Obviously, An+2< An< An-2


1
Thus, 2 An = An + An < An + An-2 = (by part (i))
n -1

Thus, An < 1 (1)


2 2(n − 1)
−x + y
1
(4, 0) Also 2 An = An + An > An + An+2 = , replacing n by n + 2
n +1
in (i)

An > 1 (2)
2(n + 1)
Figure 24.61
æxö 1 1
45. Given f ç ÷ = f(x) - f(y)(1) From Eqs. (1) and (2), we get < An < .
èyø 2(n + 1) 2(n - 1)
Putting x = y = 1, we get f(1) = 0
47. 
Since f(x) is continuous (Fig. 24.63).
f ( x + h) − f ( x )
Now, f’(x) = lim So, it must be continuous at x = 1, - 1, that is,
h→ 0 h
 h lim f ( x ) = lim f ( x ) = f (1)
f  1+  x →1− x →1+
 x
= lim (from (1))
h→ 0 h Þ lim 2 x = lim x 2 + ax + b = 2
x ®1- x ®1+
 h
f  1+ 
 x ⇒2=1+a+b=2
= lim
h→ 0  h 
  x ⇒ a + b = 1 (1)
x
f (1+ x ) Also lim f ( x ) = lim f ( x ) = f ( -1)
3   x ®1- x ®1+
⇒ f ′(x) = since lim = 3
x  x → 0 x 
⇒ f(x) = 3 In x + c Þ lim x 2 + ax + b = lim 2 x = -2
x ®1- x ®1+
Putting x = 1
⇒ 1 - a + b = -2
⇒c=0
⇒ f(x) = 3 In x = y (say) ⇒ -a + b = -3(2)

3 3 Solving Eqs. (1) and (2), we have


3
ò x dy = ò e dy = 3 éëe y 3 ùû
y 3
Therefore, required area = b = -1, a = 2

-¥ -¥
Therefore,
= 3 (e - 0) = 3e sq. units
p /4 2x
ïì -1 £ x £ 1
46.  (i) See Fig. 24.62. Obviously, An = ò tann xdx f (x) = í 2
îï x + ax + b , x < 1, x > 1
0
p /4
Now drawing the given curves.
An + An-2 = ò (tann xdx + tann -2 x )dx
0 y
p /4 1 x = −1/8
1
= ∫ (tann − 2 x sec2 x )dx = ∫ t n − 2dt =
n −1 y = 2x
0 0

y
y = tann+2 x
y = tann x x
y = tann−2 x −1
x = −2y 2
(−2, −1)

O π /4 x (−1, −2)
y = x 2 + 2x − 1

Figure 24.62 Figure 24.63

Mathematical Problem Book for JEE.indb 1099 07-06-2018 14:38:05


1100 Mathematics Problem Book for JEE

-1 -1/ 8 y
é -x ù é -x ù
Required area = ò ò
2
ê - ( x + 2 x - 1) ú dx + ê - 2 x ú dx
ë 2
-2 ê úû -1 ê ë 2 úû C2 C1
D  1 1
(0, 1) B(1, 1)
761  , 
= sq. units  2 2
192 H y = 2x − 2x 2
 1 4 F E  2 4
48. See Fig. 24.64.  ,  G , 
 3 9  3 9
On the curve C1, that is, y = x2
O x
Let P be (a, a2). Hence, ordinate of point Q on C2 is also a2.  1 1 A(1, 0)
y a2  , 
 2 4
Now on C2 (y = 2x) the abscissa of Q is given by x = = .
2 2
æa2 2 ö Figure 24.65
Therefore, Q is çç , a ÷÷ and R on C3 is {a, f(x)}.
è 2 ø
æ 1 1ö
Now area of OPQ is y = x2 and y = (1 - x)2 meet at E ç , ÷ and y = 2x - 2x2 meets
è2 4ø
a2 a2
 y 2 a2 æ1 4ö æ2 4ö
∫ ( x1 − x2 ) dy = ∫  y −  dy = a 3 −
2 3 4
(1) y = (1 - x)2 at F ç , ÷ and y = x2 at G ç , ÷
0 0 è3 9ø è3 9ø
ì 2 1
y ï(1- x ) 0£ x £
3
ï
1  ï 1 2
 , 1 Therefore, f ( x ) = í2 x (1- x ) £x£
2  ï 3 3
(0, 1) (1, 1) ï 2 2
C2 C1 ïx 1³ x >
î 3
Q
P Hence,
1 2
3 3 1
Area = ò (1- x )2 dx + ò (2 x - 2 x 2 ) dx + ò x 2dx
x
O (1, 0) 0 1 2
3 3
R 1 2
1
C3 é -(1- x )3 ù 3 é 2 2 x 3 ù 3 é x 3 ù
=ê ú + êx - ú +ê ú
Figure 24.64 êë 3 úû 0 êë 3 úû 1 êë 3 úû 2
3 3

a a -8 1 4 16 1 2 1 8
= + + - - + + -
Again, area of DORP = ò ( y1 - y 2 ) dx = ò ( x 2 - f ( x )) dx (2) 81 3 9 81 9 81 3 81
0 0
30 51 17
= 1- = = sq. units
81 81 27
From Eqs. (1) and (2)
a (1 + tanx)n > (tanx)n so, we can replace
50. 
2a 3 a 4
4 ò0
- = ( x 2 - f ( x )) dx p p p
3 4 4 é tann -1 x ù 4
An = ò tann x dx = ò tann -2 x (sec2 x - 1) dx = ê ú - An -2
Differentiating both sides 0 0 êë n - 1 úû 0
2a2 - a3 = a2 - f(a) Therefore,
f(a) =a3 - a2  ⇒ f(x) = x3 - x2 1
An + An -2 = (1)
n -1
49. See Fig. 24.65. y = x2 is parabola (C1) with vertex (0, 0) and
passing through B (1, 1), y = (1 - x)2 is parabola (C2) with vertex p
Again in the interval 0 to , tan x is positive and < 1
at A (1, 0) and passing through D (0, 1). 4
2 Therefore,
æ 1ö 1
y = 2x(1 - x) = 2x - 2x2 = -2 ç x - ÷ + is parabola with tann x ≤ tann-2 x
è 2ø 2
⇒ An ≤ An-2
 1 1 æ 1 ö
vertex at H  ,  . ⇒ An + An < An -2 + An ç =
 2 2 ÷  (1)
è n - 1ø

Mathematical Problem Book for JEE.indb 1100 07-06-2018 14:38:21


Chapter 24 | Area Under the Curves 1101

So, 1 121 1 121


An < Therefore, ratio = : = .
2(n - 1) (2) 24 6 4

52. Area of shaded region (Fig. 24.67) is
Now again An+2 < An 1 1
 2 2  2 2
Therefore,
An+2 + An < An + An
∫  1+ x 2 − x  dx = 2∫  1+ x 2 − x  dx
−1 0
1
1  x3   p  1   2
⇒ < 2 An = 2 2 tan−1 x −  = 2   −  =  p −  sq. units
(n + 2) − 1 3  2  3  3
 0  
1
⇒ < An (3)
2n + 2 y
2 y = x2
Hence, from Eqs. (2) and (3) we get y=
1+ x2 B(0, 2)
1 1
< An <
2n + 2 2n - 2 (−1, 1)A C(1, 1)
See Fig. 24.66. y = 4x -
51.  x2 = 4 - (x - 2)2

Therefore, (x - 2)2 = -(y - 4) is a parabola with vertex at (2, 4)


x
and upwards. O
2
é 1ù 1 Figure 24.67
Again y = x 2 - x = ê x - ú -
ë 2û 4 1
See Fig. 24.68. A is point of intersection of x = and y = 2x .
53. 
2 2
æ 1ö 1 æ1 ö
Therefore, ç x - ÷ = y + is a parabola with vertex B at So, A is ç , 2 ÷ .
è 2ø 4 è2 ø
æ1 1ö 1
ç , - ÷ and downwards. B is point of intersection of x = and y = In x.
è2 4ø 2
æ1 ö
y So, B is ç , - ln 2 ÷ .
è2 ø
C(2, 4) C is point of intersection of x = 2 and y = In x.
A 5 15 So, C is (2, In2).
,
2 4 D is point of intersection of x = 2 and y = 2x.
So, D is (2, 4).

y y = 2x
D

D(1, 0) M(4, 0)
O x
A y = log x
B(1/2, 1/4)
Figure 24.66 C

æ 5 15 ö x
The two parabolas intersect at O(0, 0) and A ç , ÷. O (1, 0)
è2 4 ø x=2
B
1
1 2 1 x=
x3 x 1
Area OBD = ò ( x - x ) dx =
2
- = 2
3 2 6
0 0 Figure 24.68
5 5
2 2
2
Area ODAC = ∫ ( y1 − y 2 ) dx = ∫ [( 4 x − x ) − ( x − x )] dx
2 2 Area ABCD = ò ( y1 - y 2 ) dx , where y1 = 2x > In x = y2 for all
0 up down 1
0
2
5 5 é1 ù
2
 5 x 2 2 x 3  2 125 250 125 x Îê , 2ú .
ë2 û
     ∫
= − = −  = − =
2
( 5 x 2 x ) dx 
0  2 3 0 8 24 24 2
é 2x ù
2

ú - [ x ln x - x ] 1
2
Þ ò (2 - ln x ) dx = ê
x
125 1 121 êë ln 2 úû 1
Lined area = - = 1 2
24 6 24 2 2

Mathematical Problem Book for JEE.indb 1101 07-06-2018 14:38:41


1102 Mathematics Problem Book for JEE

4 2 1 1 1 1 æ 1ö 4
= - - (2 ln 2 - 2) + ln - At x = , f ¢¢ ç ÷ = -2 < 0 max. value =
ln 2 ln 2 2 2 2 3 3
è ø 27
(4 − 2 ) 3  5  y
= + −   ln 2 sq. units
ln 2 2  2 D C
y=2
54. 
See Fig. 24.69. Given curves are
y = ex In x(1)

ln x
y= (2)
ex

1
Points of intersection of (1) and (2) are P æç , - 1ö÷ and Q(1, 0)
x
O A B
èe ø (1, 0) (2, 0)
For curve (1), y < 0 for 0 < x < 1
and y ≥ 0 for x ≥ 1 Figure 24.70
(1) 2
y 1 
 , 0 (2) Required shaded area = Area OBCD - ò y dx where y = x(x - 1)2
x e  0
x
O 2
Q(1, 0) 2 é x4 æ x 3 ö æ x 2 öù
= 2 × 2 - ò x ( x 2 - 2 x + 1) dx = 4 - ê - 2 çç ÷÷ + çç ÷÷ ú
1  êë 4 è 3 ø è 2 ø úû 0
P  , − 1 0
 e 
é 16 ù 2 10
= 4 - ê4 - + 2ú = 4 - = sq. units
ë 3 û 3 3
y 56. See Fig. 24.71.
Figure 24.69 Required area (shaded) is
p
Obviously y → 0 when x → 0 4
For curve (2), y → -∞ when x → 0
y < 0 for 0 < x < 1, y ≥ 0 for x ≥ 1
∫ tan x dx - area of ∆PMQ
0
Obviously y → 0 when x → ∞ p
1
This shape of curves is depicted in Figure 24.65. = [log sec x ]04 − × QM × MP
2
1 1
Equation of tangent at P is
ln x
Required area = ò ex ln x dx - ò ex dx
y - 1=
dy æ pö
1 1 p çx- ÷
e e dx 4 è 4ø

-1 e 2 - 3 1 e 2 - 5
=
1
4e
(
3 - e2 -
2e
= )
4e
- =
2e 4e
sq. units
dy
dx
p = sec2 x p =2
4 4

Alternatively, area is y y = tan x

1 1
 ln x  1 1 2 
∫  ex − ex ln x  dx =  2e (ln x ) − 4 ex (2 ln x − 1) 1
2

1
e
e
1 π 1
P  4,
2  
1 1 e -5
= - (3 - e 2 ) = sq. units
2e 4 e 4e
M
55. See Fig. 24.70. π π x
O Q
f(x) = x(x - 1)2 4 2

f ′(x) = 1 · (x - 1)2 + x · 2 (x - 1) = (x - 1) (3x - 1) = 0 Figure 24.71


1  p
Þ x = 1, Equation of tangent is y − 1 = 2  x −  .
3  4
f ′′(x) = 1 · (3x - 1) + (x - 1)3 = 6x - 4 p 1
When y = 0 , x = − = OQ.
At x = 1, f ′′(1) = 2 > 0 min. value = 0 4 2

Mathematical Problem Book for JEE.indb 1102 07-06-2018 14:38:59


Chapter 24 | Area Under the Curves 1103

Therefore,
{ }
2
3 3
p 1 ⇒± = x 2 + ( g( x ))2
QM = - OQ = 4 4
4 2
⇒ x2 + (g(x))2 = 1 (since x2 + (g(x))2 ≠ -1)
 Hence,
⇒ (g (x))2 = 1 - x2 ⇒ g (x) = ± 1- x 2
 1 1 1  1  1 
Area =   log 2 − × × 1 =   log 2 −    sq. units 60. 
See Fig. 24.74. Clearly
 2 2 2  2   4 

57. See Fig. 24.72. 2, − 2≤ x < 0
Points of intersection of y = x and x 2 = - 2 y are 
 1
g( x ) =  x 2 − x , 0≤ x ≤
x = 0, - 2  2
 1 1
2
æ x2 ö 0
æ -x2 ö  − 4 , 2
< x ≤3
Area = 2 ò çç - + 4 - x 2 ÷÷ dx - ò çç - x ÷÷ dx
0 è 2 ø - 2è
2 ø 0 1/ 2 3
1
Area= ∫ 2dx + ∫ (x − x
2
) dx + ∫ dx
2 0 4
 − x3 1 4 x  x2 x3  −2 0 1/ 2
= 2 + x 4 − x 2 + sin−1  +  +  1/ 2
3 2 2 2 2 0  2 3 2 −
3
2 æ x2 x3 ö æxö 113
= (2 x )0-2 + çç - ÷÷ + ç ÷ = sq. units
é -2 p ù é 2ù é1 p ù 1 1 è 2 3 ø0 è 4 ø1/ 2 24
= 2 ê + 1+ 2 ´ ú - ê1- ú = 2 ê + ú - = p + sq. units
ë3 4 û ë 3û ë3 2 û 3 3 y
y

y = x 2 – |x|
(0, 2)
y=x
1
x
−2 −1 O 3
O x
(−2, 0) (0, 0) (2, 0) Figure 24.74

x2 + y2 = 4
See Fig. 24.75. The expression |x - y| + |x + y| ≤ 8, represents the
61. 
( − 2, − 2 ) interior region of the square formed by the lines x = ±4, y = ±4
and xy ≥ 2. Represents the region lying inside the hyperbola
(0, −2) x 2 = − 2y xy =2.
Required area is
Figure 24.72
4
58. 
See Fig. 24.73.  2
= 2 ∫  4 −  dx = 2( 4 x − 2 I n x )14/ 2
1  x
Required area = × 2 × 2 = 2 sq. units 1/ 2
2
= 4(7 − 3 ln 2) = 4(7 − ln 8 ) sq. uniits
y y=x
(0, 1) C y=4 B
y = |x| − 1

(−1, 0) (1, 0)
x
O
y = −|x| + 1 x=4
x = −4
(0, −1) D
y = −4 A
y = −x
Figure 24.73

3 Figure 24.75
59. Area of equilateral triangle = (side)2.
4
px
Two vertices are at (0, 0) and [x, g(x)] See Fig. 24.76. Area bounded by y =
62.  2 ×sin and x-axis
4
between the lines x = 2 and x = 4,
Hence, side = ( x - 0 )2 + ( g( x ) - 0 )2 = x 2 + ( g( x ))2
4 4
3é 2 2 px 4 2 px
Area = x + ( g( x ))2 ù D = 2 ò sin dx = - × cos
4 ê
ë ú
û 2
4 p 4 2

Mathematical Problem Book for JEE.indb 1103 07-06-2018 14:39:17


1104 Mathematics Problem Book for JEE

1
4 2  x 1 
= sq. units = x − 1− x 2 − sin−1 x 
p  2 2 0
Let the drawn lines are L1: y - m1(x - 4) = 0 and L2: y - m2(x - 4) 1p  p
= 0, meeting the line x = 2 at the points A and B, respectively. =1 − 0 −   = 1− sq.units.
2 2 4
Clearly A = (2, -2m1); B = (2, -2m2) (Fig. 24.76).
65. See Fig. 24.79.
Now 3 1 2 3

D 4 2 1 Required area = ò | y | dx = ò | y | dx + ò | y | dx + ò | y | dx
D ACD = Þ = × 2 × -2m1 0 0 1 2
3 3p 2 1 2
æx4
11 ö æx 11 4 ö
2 2 2D = - çç - 2 x 3 + x 2 - 6 x ÷÷ + çç - 2 x 3 + x 2 - 6 x ÷÷
Þ m1 = - Also D BCD = 4 2
3p 3 è 4 2 ø0 è ø1
3
8 2 1 æ x4 11 ö
Þ = × 2 - 2m2 - çç - 2 x 3 + x 2 - 6 x ÷÷
3p 2 è 4 2 ø2
-4 2
Þ m2 = 9 1 1 11
3p = + + = sq. units
4 4 4 4
2 2
Required sum = -
p
y B D F
πx O
y = 2 sin B
4 A
A
D
x Figure 24.79
O C 1 2 4 L1 = 0
66. See Fig. 24.80.
L2 = 0 1

Figure 24.76 A = ò ( x - x 3 ) dx
0
63.  y = x (3 - x)2 1
2 3/2 1 x 4
After solving, we get x = 1 and x = 3 which are points of = x -
3 0 4
maximum and minimum, respectively. 0
Now the shaded region is the required region (Fig. 24.77). 2 1 8-3 5
3
= - = =
3 4 12 12
Therefore, A = ò x (3 - x )2 dx = 4 sq. units
1
Y
C(1,4)
1

X
O O
A(1, 0) B(3, 0)

Figure 24.77
64. By definition, the lines max, (x, y) = 1 means Figure 24.80
x = 1 and y ≤ 1 or y = 1 and x ≤ 1 67.  Y
Y

Reqd.
Area
X′ X X′ X
O

x=1
Y′
Y′
Figure 24.78 Figure 24.81
1
Shaded region is the required one. Therefore, required area is
See Fig. 24.78. Required area is ò éëê1- 1- x 2 ù dx.
ûú
1
4 ´ ´ 2 ´ 2 - p × 2 = 8 - 2p sq. units
2
0

Mathematical Problem Book for JEE.indb 1104 07-06-2018 14:39:33


Chapter 24 | Area Under the Curves 1105

Practice Exercise 2 BC S3 BC S2
PM = , ML = (QM = BN and ML = PC)
1. Solving y = x - and y = mx, we get the points of intersection
x2 S S
(0, 0) and [1-m, m(1-m)]. æ S S S ö
BC = BC ç 1 + 2 + 3 ÷
Case I:  m < 1 (Fig. 24.82) ç S S S ÷ø
è
y S = ( S1 + S2 + S3 )2 , where S is the area of DABC.

a a +t
x 3.   ò f (t ) dt = - ò f (t ) dt
O
a -t a
⇒ f(a - t) = - f(a + t) ∀ t ∈ R
Therefore, y = f(x) is symmetrical about point (a, 0).
4. Putting x = a in the given equation, we have
Figure 24.82 f(0) = - f(2a) ⇒ -f(0) = f(2a)
9
Shaded area = a a +t
2 5.  ò f (t ) dt = - ò f (t ) dt ⇒ f(a - t) = - f(a + t) ∀ t ∈ R.
1- m 1- m a -t a

ò ( x - x ) dx - ò
2 9
Þ mx dx = ⇒ f(a - t) = - f(a + t) = x ⇒ t = a - f -1(x) = f -1(-x) - a
0 0
2
b

òf
-1
Solving this, we get m = -2 ⇒ f -1(x) + f -1(-x) = 2a ⇒ ( x ) dx = 2ab
Case II:  m > 1 (Fig. 24.83) -b
6. x = 1 is a root of f(x) and also a root of 1st, 2nd and 3rd derivatives
of f(x). Hence, f(x) has x = 1 repeated root 4 times so f(x) = (x - 1)4.
Therefore,
2
3
ò ( x - 1- ( x - 1)
4
Required area = ) dx = sq. units
1
10

Figure 24.83 1
p 1
Shaded area =
9 7. Required area = ò( 1- x 2 - ( x - 1)4 ) dx = - sq. units
4 5
2 0
1 3
0 0 63
8. Required area = 6 + ò ( x - 1) dx + ò ( x - 1) dx =
4 4
9 sq. units
⇒ ∫ ( − mx ) dx − ∫ ( x − x ) dx =2
5
1− m 1− m
2 0 1

9. Equation of circle is + = 2. x2 y2
Solving this, we get m = 4
Equation of tangent at A is -x + y = 2
2. See Fig. 24.84. Equation of tangent at D is -x - y = 2
Area of ABC BC 2 Form maximum area of the rectangle QEKF, Q is ( 2 , 0 )
=
Area of PQM QM 2 - 2 -2 - 2 -2 6+ 4 2
Area of rectangle = ´ = =3+2 2
Area of ABC BC 2 2 2 2
=
Area of MNP NP 2 10. Area of DBQC = 2 -1
Area of ABC BC 2 1
= 11. Area of DOQM = 2 ´ 2 =1
Area of MOL ML2 2
A f (x) + f (y)
12. f ( x + y ) =
O f (x)
P f ( x + h) - f ( x ) (f ( x ) + f (h)) - (f ( x ) + f (0 ))
M f ¢( x ) = lim = lim
Q S1 S2 L h®0 h h ® 0 hf ( x )
f ¢(0 ) 2
= =
f (x) f (x)
C
B N P
⇒ f(x) f(x) dx = ∫2 dx ⇒ f ( x ) = 4( x + c )
Figure 24.84 Since, f(0) = 2 ⇒ c = 1.
BC S1
Area of DABC = QM = Therefore, f ( x ) = 2 ( x + 1).
S

Mathematical Problem Book for JEE.indb 1105 07-06-2018 14:39:57


1106 Mathematics Problem Book for JEE

13. See Fig. 24.85. Applying L’ Hospital’s rule,


3
1
The required area = 2 ò (7 - x - 2 x + 1) dx lim =0
x ®¥ x
0

3 x x
x2 4 ln t
= 2 7x - - ( x + 1)3 / 2 16. g(x) = ò f ( t ) dt = ò dt
2 3 0 0
t
0

1/ x
é 9 4 ù 43 43 æ 1ö ln t
sq. units
= 2 ê21- - (8 - 1)ú = 2 ´ =
ë 2 3 û 6 3
Þ gç ÷ =
èxø ò t
dt
0

1
Let t = . Then
(0, 7) u
x
ln u
∫ u
du
0
(0, 3)
(4, 0) æ 1ö
⇒ g ç ÷ = g(x)
èxø

Figure 24.85 æ 1ö
Therefore, g(n) - g ç ÷ = 0.
14. See Fig. 24.86. ènø

17. For points of intersection,

ln x
= ex ln x ⇒
ln x 1 − ex 2
=0
( )
−1 1 8 x x
ln x = 0 ⇒ x = 1
1
1− ex 2 = 0 ⇒ x =
e
Figure 24.86
Hence, the number of points is 5. Therefore,

1
f (x) f (y)  ln x  2e − 5
15. f(xy) =
y
+
x
Required area = ∫ 
x
− ex ln x  dx =
 8
sq. units
1 e
f ( x + h) - f ( x )
f(x) = lim
h®0 h 18. See Fig. 24.87.
æ 1 ö é1 1 ù
f ( x )ç -1 (A)  Required area = 2 ê (2 + 1) ´ 1- ò x 2dx ú
h ÷ æ hö
ç 1+ ÷ f ç 1+ ÷ - f (1) êë 2 0 úû
= lim è x ø + è xø
h®0 h xh é 3 1ù 7
= 2 ê - ú = sq. units
-f ( x ) f ¢(1) ë2 3û 3
= lim + 2
h®0 æ hö
x ç 1+ ÷ x
è xø x2 = y
-f ( x ) 1 f (x) 1 (0, 2)
⇒ f(x) = + 2 or f ′(x) + = 2
x x x x
This is a linear differential equation. (1, 1)
Solution is f(x) · x = ln x + c
or (2, 0)
ln x
f(x) = , c = 0 since, (f(1) = 0)
x
Now,
ln x æ¥ö
lim ç ÷ form
x ®¥ x è¥ø Figure 24.87

Mathematical Problem Book for JEE.indb 1106 07-06-2018 14:40:17


Chapter 24 | Area Under the Curves 1107

7 19. (A)  f(x) = sin2 x cos x


(B)   f ( x ) = 3 sin x − 4 cos x −
3 f(x) = p(x) q(x), (period of f(x) will be LCM of period of p(x)
and q(x)).
f ′( x ) = 3 cos x + 4 sin x = 0
LCM of p and 2p is 2p .
4 3
⇒ cos x = ,sin x = − b b
b
5 5
         (B)  A = ò ydx = ò a a2 - x 2 dx
0 0
4 3
and cos x = − ,sin x =
5 5 ab abp
= [e 1- e 2 + sin-1 1- e 2 ] =
2 8
Now,
p
f"(x) = -3 sinx + 4 cosx ⇒ e 1 − e 2 + sin−1 1 − e 2 =
    4
4 3
At cos x = ,sin x = − , f " ( x ) is positive, that is, f(x) has æ 1ö æ 1ö
5 5 sin-1 ç ÷ + cos -1 ç ÷
(C)  f(x) = è x ø è x ø = p sin x
minimum value. cosecx 2
4 3 Greatest value of sin x = 0.
At cos x = − ,sin x = , f ′′ ( x ) is negative, that is, f(x) has
5 5
p
minimum value. So, So, greatest value of f ( x ) = .
2
7 9 16 7 8 æp ö p
f ( x ) = 3 sin x − 4 cos x − = + − = (D) cos -1 cos ç ÷ =
3 5 5 3 3 è 14 ø 14

2r1r2 2 ´ 3 ´ 4 24 1
(C)  Length of the chord = = = 20.  (A)  y = tan-1 + tan-1 b , (0 < b < 1)
5 5 2
r12 + r22
æ 1/ 2 + b ö æ1 ö
(D)  See Fig. 24.88. ⇒ y = tan-1 ç ÷ , since, ç b < 1÷
è 1- b / 2 ø è2 ø
Parabola is y2 = 4(x + 1) (1)
Focus is (0, 0) æ 1+ 2b ö p æ 1+ 2b ö
0 < tan-1 ç ÷£ Þ0<ç ÷ £1
x -0 y -0 è 2 - b ø 4 è 2-b ø
Equation of AB is = =r
1/ 2 3 /2 ⇒ 0 < (1 + 2b) ≤ (2 - b), (1 + 2b > 0)
Substituting parametric coordinates in Eq. (1), we have
1 1
⇒ 3b ≤ 1 ⇒ 0 ≤ ⇒ bmax =
2 3 3
æ 2 ö ær ö
çç r ÷÷ = 4 ç + 1÷
è 2 ø è2 ø (B) sin4 x + cos3 x ≥ 1 (1)
Since sin2 x + cos2 x = 1 and -1 ≤ sin x, cos x ≤ 1
3r 2
Þ - 2r - 4 = 0 Eq. (1) cannot be >1
2
Therefore, sin4 x + cos3 x = ±1 (2)
A Eq. (2) is possible if either,
sin x = 1 and cos x = 0 or sin x = 0 and cos x = 1
p p
⇒ x = (4n + 1) , x = (2n + 1) or x = np, x = 2np
60° 2 2
(1, 0) p 3p
In (0, 2p), x = , or x = p
2 2
B Therefore, the number of solutions will be 3.
(C)  See Fig. 24.89.
Figure 24.88
4y2 = x2 (4 - x2)(1)
1 2
Length AB = PA - PB = ( PA + PB )2 - 4 PAPB    ⇒ y = x (4 - x 2 )
2
2
æ8ö 16 16 x
= ç ÷ -4´ =    ⇒ y = (4 - x 2 )
è3ø 3 3 2

Mathematical Problem Book for JEE.indb 1107 07-06-2018 14:40:39


1108 Mathematics Problem Book for JEE

Therefore,  x3 4 x 
2
2x f (x) = a −  + b
area (A)  = 4 × ò ( 4 - x 2 )dx  3 3
0
2 ´ 2
This passes through (0, 0) and (1, -2). So, b = 0 and
Let (4 - x2) = t. Then - 2x dx = dt.
æ1 4ö
a ç - ÷ = -2 ⇒ a = 2
è3 3ø

O
A
2x 2
B Therefore, f ( x ) = ( x − 4 ).
(−2, 0) (2, 0) 3
Hence, f(x) meets the x-axis at (0, 0), (-2, 0) and (2, 0).
Since f(-x) = -f(x), the curve y = f(x) is symmetrical about the
origin.
Figure 24.89
Also as a = 2, f ′(x) > 0.
0 4 4
-4 é t 3/2 ù 2 2 2 2 2
⇒A= ò t dt = ò t dt = ê ú = ´ [ 64 - 0] For x < - , x> and f ′(x) < 0 for - <x<
4 4 ê
ë 3 / 2 ú
û 3 3 3 3 3
0 0

16 Therefore,
⇒A= sq. units
3
0 2

21. Since y = f(x) has relative extremities at x = ±


2
these points Area of region = 2p - ò f ( x ) dx + ò f ( x ) dx
3 -2 0
are critical points and hence they must be roots of f ′(x) = 0 0 0 2 2
(Clearly f is differentiable everywhere). Therefore, Þ ò f ( x ) dx = - ò f ( x ) dx = - ò f ( -t )( -1) dt = ò f (t ) dt
-2 -2 0 0
 2  2   2 4
f ′( x ) = a  x − x+  = a  x − 3  Required area = 2p. So, integral part = 6.
 3  3

Mathematical Problem Book for JEE.indb 1108 07-06-2018 14:40:50


Chapter 24 | Area Under the Curves 1109

Solved JEE 2017 Questions


JEE Main 2017
1. The area (in sq. units) of the region {(x, y): x ≥ 0, x + y ≤ 3, x2 ≤ y 2 = 3x
4y and y ≤ 1+ x } is
(1, 3 )
3 7
(A) (B)
2 3
5 59 (1, 0) (2, 0)
(C) (D)
2 12
(OFFLINE)
(1, − 3 )
Solution: The given situation is depicted in the following
graph: x 2 +y 2 = 4
y
x2 + y2 = 4 (1)
(6, 3)    y2 = 3x (2)

(1, 2) Substituting Eq. (2) in Eq. (1), we get


2 x2 + 3x – 4 = 0
(2, 1) ⇒ x2 + 4x – x – 4 = 0
(0, 1)
⇒ x(x + 4) – 1(x + 4) = 0
⇒ (x – 1)(x + 4) = 0
⇒ (x – 1) = 0 and (x + 4) = 0
x
1 (3, 0)
Therefore, x = 1, −4. Considering x = 1, then from Eq. (2), we get
y = 3, − 3.
Thus, (1, 3 ) and (1, − 3 ) are the points of intersection of parab-
The area of the given region is
ola and circle.
1
 3 The required area (A) is the area of the shaded region shown in the
x2   x2 
∫ 1+ x−
4
 dx + ∫  3 − x −  dx
4
figure. Therefore,
0 1 1 2 
1 2 A = 2  ∫ y 2dx + ∫ y1dx 
 x 3/ 2 x 3   x2 x3   0 1 
= x + −  + 3 x − − 
 3/ 2 12   2 12  From Eq. (1), we get
0 1
19 11 5 y1 = 4 − x 2
= + =
12 12 2 From Eq. (2), we get
Hence, the correct answer is option (C). y2 = 3 x
Therefore,
2. The area (in sq. units) of the smaller portion enclosed between 1 2 
the curves, x2 + y2 = 4 and y2 = 3x, is A = 2  ∫ 3 xdx + ∫ 4 − x 2 dx 
 0 1 
1 p 1 2p
(A) + (B) + 1 2 
2 3 3 2 3 3 = 2  ∫ 3 x1/ 2dx + ∫ 22 − x 2 dx 
1 2p
1 4p  0 1 
(C) + + (D)
3 3
3 3 x n −1
(ONLINE) Using standard integral, ∫ x ndx = , we have
n +1
Solution: The given equation x2 + y2 = 4 is equation of circle of
x 2 a2 x
radius 2 centred at origin and equation y2 = 3x is the equation of ∫ a2 − x 2 dx = a − x 2 + tan−1
parabola. 2 2 a − x2
2

Mathematical Problem Book for JEE.indb 1109 07-06-2018 14:41:06


1110 Mathematics Problem Book for JEE

Therefore, a 1

∫ (x − x )dx = ∫ ( x − x 3 )dx
3
 1
x
2
x 3/ 2  4 x   0 a
A = 2  3  +  4 − x 2
+ tan −1
 
 3/ 2  2 2 4 a 1
 4 − x2  1  x2 x x2 x 4
0  ⇒ − = −
2 4 2 4
 1 2 2 1 1  0 a
= 2  3. −0+ 4 − 4 + 2 tan−1 − 4 − 1 − 2 tan−1
 3 / 2 2 4 − 4 2 4 − 1  a2  a 4   1 a2  1 a4
⇒ − 0 −  − 0 =  −  − − 
2 3 3  1   2   4  2 2  4 4 
= 2 + 2 tan−1(∞ ) − − 2 tan−1  
 3 2  3  
a2 a4 1 a2 1 a4
⇒ − = − − +
2 4 2 2 4 4
p p 1
Now, tan = ∞ and tan = . Therefore, the area of the a2 a4 1 a2 1 a4
2 6 3 ⇒ − − + + − =0
smaller portion enclosed between the two curves is obtained as 2 4 2 2 4 4
follows: a4 1
⇒a 2 − − =0
2 4
2 3 3  p  p 
A = 2 − + 2 tan−1  tan  − 2 tan−1  tan   4a 2 − 2a 4 − 1
 3 2  2   6 ⇒
2 4
= 0 ⇒ 4a − 2a − 1 = 0
4
 1 p p
= 2 3 + 2 −2  ⇒ −(2a 4 − 4a 2 + 1) = 0 ⇒ 2a 4 − 4a 2 + 1 = 0
 6 2 6
Now, let us consider the equation 2a 4 – 4a 2 + 1 = 0.
 1 2p   3 4p  3 4p
= 2 3 +2  = 2 +  = + Let a 2 = u. Therefore,
 6 6   6 6  3 3
2u2 – 4u + 1 = 0
 1 4p 
= + sq.units
 3 3  ⇒u=
4 ± 16 − 4 × 2
2×2
Hence, the correct answer is option (D).  
 since for equation ax 2 + bx + c = 0 , 
JEE Advanced 2017  2 
 the discriminant = b − 4 ac = D and
1. If the line x = a  divides the area of region R = {( x , y ) ∈2 : x 3 ≤ y ≤ x , 0 ≤ x ≤ 1}  
 the roots of x are −b ± D 
R = {( x , y ) ∈2 : x 3 ≤ y ≤ x , 0 ≤ x ≤ 1} into two equal parts, then  2a 
1 1
0 <a ≤
(A) (B) <a <1 4 ± 16 − 8 4 ± 8 4 ± 4 × 2
2 2 ⇒u= = =
4 4 4
2a 4 − 4a 2 + 1 = 0
(C) (D) a 4 + 4a 2 – 1 = 0
4±2 2 2 1
⇒u= = 1± ⇒ u = 1±
Solution: Let us consider y = x3 and y = x. Then the area between 4 2 2
these two curves in region 0 ≤ x ≤ 1 is
Substituting u = a 2, we get
1
A = ∫ ( x − x 3 )dx 1
a 2 = 1±
0 2
It is given that the line x = a  divides the area under the curve into 1 1
From a 2 = 1± , we get < a < 1.
two equal parts. Therefore, 2 2
Hence, the correct answers are options (B) and (C).
y=x

x =a

Mathematical Problem Book for JEE.indb 1110 07-06-2018 14:41:28


25  Differential Equations

25.1 Introduction 25.4 Degree of a Differential Equation


Generally, any equation such as The degree of a differential equation is the degree of the highest
f(x, y, a) = 0(25.1) order derivative, when differential coefficients are made free
represents a member of a family of curves for each individual from radicals and fractions. The degree of the above differential
value of a. Sometimes it is found necessary to represent the whole equations is 1, 1, 3 and 2, respectively.
family of curves as a single unit and consider them as one for the Illustration 25.1 Find the order and degree (if defined) of the
purpose of studying a common property or characteristic which following differential equations:
may run through the members of the family. 2
From Eq. (25.1), solve for a, and the equation φ(x, y) = a may æ d3y ö 3
dy d2 y æ dy ö
 (i)  çç 3 ÷÷ = +2 (ii)  2
= x ln ç ÷
be obtained; and on differentiating, ‘a’ gets removed. The è dx ø dx dx è dx ø
resulting equation involving dy/dx is known as a differential 4
dy æ d 2 y ö æ dy ö5 x
equation, that is, the equation representing all the members (iii)  + 4 y = sin x (iv)  çç 2 ÷÷ + ç ÷ - y = e
dx è dx ø è dx ø
of the family f (x, y, a) = 0 or alternately φ(x, y) = a.
Solution:
(i) The given differential equations can be rewritten as
25.2 Basic Definition 2 3
æ d 3 y ö æ dy ö
An equation containing an independent variable, a dependent çç 3 ÷÷ = ç + 2 ÷
variable and the differential coefficients of a dependent variable è dx ø è dx ø
with respect to an independent variable is called a differential Hence, its order and degree are 3 and 2, respectively.
equation. An ordinary differential equation is one in which there is d2 y æ dy ö
(ii)  = x ln ç ÷
only one independent variable. Examples: dx 2
è dx ø
dy Hence, its order and degree are 2 and 1, respectively.
1.  = 1+ x + y
dx (iii)  dy + 4 y = sin x
dx
2.  dy = cot x + xy Hence, its order and degree are 1 and 1, respectively.
dx
4 5
3  d2 y   dy  x
æ d4 y ö dy (iv)   2  +   − y = e
3.  çç 4 ÷÷ - 4 + 4 y = 5 cos 3 x  dx   dx 
è dx ø dx
Hence, its order and degree are 2 and 4, respectively.
2 2
2 d y æ dy ö
4.  x + 1+ ç ÷ = 0
dx 2 è dx ø 25.5 Formation of a Differential
Equation
25.3 Order of a Differential Equation Formulating a differential equation from a given equation
The order of the highest derivative occurring in the differential representing a family of curves means finding a differential
equation is called the order of the differential equation. For equation whose solution is the given equation. The equation
example, the order of the above differential equations is 1, 1, 4 thus obtained is the differential equation of order n for the
and 2, respectively. family of the given curves.
The order of a differential equation is a positive integer. To Consider a family of curves
determine the order of a differential equation, it is not required f  (x, y, a1, a2, …, an) = 0  (25.2) 
to make the equation free from radicals. where a1, a2, …, an are n independent parameters.

Mathematical Problem Book for JEE.indb 1111 07-06-2018 14:41:40


1112 Mathematics Problem Book for JEE

Equation (25.2) is known as an n-parameter family of curves, Solution:


for example, y = mx is a one-parameter family of straight lines. y = Aex + Be3x (1)
x2 + y2 + ax + by = 0 is a two-parameter family of circles. æ dy ö
y1 = Aex + 3Be3x (2) ç y1 = ÷
If we differentiate Eq. (25.2) n times w.r.t. x, we will get n è dx ø
more relations between x, y, a1, a2, …, an and derivatives of y
æ d2 y ö
w.r.t. x. By eliminating a1, a2, …, an from these n relations and y2 = Aex + 9Be3x (3) çç y 2 = 2 ÷÷
Eq. (25.2), we get a differential equation. è dx ø
Clearly, the order of this differential equation will be n, that is, equal Eliminating A and B from the above three equations, we get
to the number of independent parameters in the family of curves.
ex e3 x -y 1 1 -y
x 3x
25.5.1 Steps for Formation of Differential e 3e - y1 = 0 Þ e x e 3 x 1 3 - y1 = 0
ex 9e 3 x - y2 1 9 - y2
Equations
Step 1: 
Write the given equation involving an independent dy d 2 y
⇒ 3y + 4y1 - y2 = 0 ⇒ 3y + 4 - =0
variable x (say), a dependent variable y (say) and the dx dx 2
arbitrary constants.
Illustration 25.4 Find the order and degree of the differential
Step 2: 
Obtain the number of arbitrary constants in step 1. Let
equation of all the parabolas whose axes are parallel to the x-axis
there be n arbitrary constants.
and having a latus rectum a.
Step 3: 
Differentiate the relation in step 1, n times with respect to x.
Step 4: 
Eliminate arbitrary constants with the help of n equations Solution: Equation of required parabolas is
involving differential coefficients obtained in step 3 and (y − β)2 = a (x − α) -
an equation in step 2. The equation thus obtained is the On differentiating both sides w.r.t. x, we get
desired differential equation. dy
2( y - b )
=a
Illustration 25.2 Form the differential equation of the dx
following relation: Again differentiating w.r.t. x, we get
 (i)  x2 + y2 = 2ax 2
d2 y æ dy ö
(ii)  x2 + y2 = 2ax + b 2( y - b ) + 2ç ÷ = 0
dx 2 è dx ø
Solution: 3
  (i)  x2 + y2 = 2ax æ dy ö d 2 y æ dy ö
Þ ç 2( y - b ) ÷ 2 + 2 ç ÷ = 0
On differentiating w.r.t. x, è dx ø dx è dx ø
3
2x + 2y
dy
= 2a d2 y æ dy ö
Þa + 2ç ÷ = 0
dx dx 2 è dx ø
Eliminating a,
Thus, the order of the differential equation is 2 and degree is 1.
æ dy ö
x2 + y2 = x ç 2x + 2y ÷
è dx ø Illustration 25.5 Find the differential equation of family of
dy curves whose tangent form an angle of π/4 with the hyperbola
Þ x 2 - y 2 + 2 xy =0
dx xy = c2?
In this case, the relation contains only one arbitrary constant Solution: The slope of the tangent to the family of curves is
and hence, the differential equation contains only dy/dx. dy
m1 =
(ii)  x2 + y2 = 2ax + b dx
On differentiating w.r.t. x, The equation of the hyperbola is
dy c2
2x + 2y = 2a xy = c 2 Þ y =
dx x
On differentiating once again w.r.t. x, Therefore,
dy c2
2 2 =- 2
æ dy ö d y dx x
1+ ç ÷ + y 2 = 0
è dx ø dx c2
Therefore, the slope of the tangent to xy = c2 is m2 = - .
which is the differential equation to the given equation and x2
since there are two arbitrary constants a and b, the differential Now,
equation contains (the second order) the derivative d2y/dx2. dy c 2
+ 2
p m1 - m2 dy æ c 2 ö æ c 2 ö
Illustration 25.3 Find the differential equation of the family of tan = Þ 1 = dx 2 x Þ çç 1+ 2 ÷÷ = çç 1- 2 ÷÷
curves y = Aex + Be3x for different values of A and B. 4 1+ m1m2 c dy dx è x ø è x ø
1- 2
x dx

Mathematical Problem Book for JEE.indb 1112 07-06-2018 14:41:57


Chapter 25 | Differential Equations 1113

Hence, the required equation is 8. The differential equation whose solution is y = c1 cos ax + c2 sin
dy x - c 2 2 ax is (where c1, c2 are arbitrary constants)
=
dx x 2 + c 2 d2 y 2 d2 y
(A)  + y = 0 (B)  + a2 y = 0
dx 2 dx 2
Your Turn 1 d2 y d2 y
(C)  2
+ ay 2 = 0 (D)  - a2 y = 0 
1. The degree of the differential equation dx dx 2
dy
dy 11 æædy dyöö
22
11 æædy dyöö
33 Ans. (B)
yy((xx))==11++ ++ çç ÷÷ ++
   çç ÷÷ ++
 is
dx
dx 11.2.2èèdx
dxøø 11.2.2.3.3èèdx
dxøø 9. The differential equation of the family of curves y2 = 4a(x + a),
(A) 2 (B) 3    (C) 1 (D) 4 where a is an arbitrary constant, is
Ans. (C)
é æ dy ö2 ù dy é æ dy ö2 ù dy
2 (A)  y ê1+ ç ÷ ú = 2 x (B)  y ê1- ç ÷ ú = 2 x
d y dy dx
êë è ø úû dx dx
êë è ø úû dx
2. The degree of the differential equation - - 3 = x is
dx 2 dx
3
(A) 2 (B) 3 (C) 1 (D) 4  d2 y dy æ dy ö dy
(C)  + 2 = 0 (D)  ç ÷ + 3 + y = 0
Ans. (A) dx 2 dx è dx ø dx

3. The degree and order of the differential equation of the family Ans. (B)
of all parabolas whose axis is x-axis are, respectively d2 y
10. If y = axn+1 + bx-n, then x 2 2 equals
(A) 2, 1 (B) 1, 2 (C) 3, 2 (D) 2, 3  dx
Ans. (B) (A)  n(n - 1)y (B)  n(n + 1)y
4. If m and n are the order and degree of the differential equation (C)  ny (D)  n2y 
3
æ d2 y ö Ans. (B)
5 çç 2 ÷÷
æ d2 y ö 3
è dx ø + d y = x 2 - 1 , then
çç 2 ÷÷ + 4
è dx ø æ d y ö dx 3
3
çç 3 ÷÷
25.6 Solution of a Differential Equation
è dx ø If we have a differential equation of order ‘n’ then by solving a
(A)  m = 3 and n = 5 (B)  m = 3 and n = 1 differential equation we mean to get a family of curves with n
(C)  m = 3 and n = 3 (D)  m = 3 and n = 2 parameters whose differential equation is the given differential
 Ans. (D) equation. The solution or integral of a differential equation is a
5.  y = 4 sin3x is a solution of the differential equation _____. relation between the variables, not involving the differential coef-
ficients such that this relation and the derivatives obtained from
dy dy
(A)  + 8 y = 0 (B)  - 8y = 0 it satisfy the given differential equation. The solution of a differen-
dx dx tial equation is also called its primitive.
d2 y d2 y For example, y = ex is a solution of the differential equation
(C)  + 9 y = 0 (D)  - 9y = 0 
dx 2 dx 2 dy/dx = y.
Ans. (A)
x 25.6.1 General Solution
6.  y = is a solution of the differential equation _____.
x +1
The solution which contains as many as arbitrary constants as
dy2 dy the order of the differential equation is called the general solution
(A)  y = x 2 (B)  x 2 = y2
dx dx of the differential equation. For example, y = A cos x + B sin x is
dy dy d2 y
(C)  y = x (D)  x =y the general solution of the differential equation + y = 0 . But
dx dx dx 2
Ans. (B) y = A cos x is not the general solution as it contains one arbitrary
7. The differential equation for all the straight lines which are at a constant.
unit distance from the origin is
2 2 2 2 25.6.2 Particular Solution
æ dy ö æ dy ö æ dy ö æ dy ö
(A)  ç y - x ÷ = 1- ç ÷    (B)  ç y + x ÷ = 1+ ç ÷
è dx ø è dx ø è dx ø è dx ø The solution obtained by giving particular values to the arbitrary
2 2 2 2 constants in the general solution of a differential equation is
æ dy ö æ dy ö æ dy ö æ dy ö called a particular solution. For example, y = 3 cos x + 2 sin x is a
(C)  ç y - x ÷ = 1+ ç ÷    (D)  ç y + x ÷ = 1- ç ÷
è dx ø è dx ø è dx ø è dx ø d2 y
particular solution of the differential equation 2 + y = 0 .
Ans. (C) dx

Mathematical Problem Book for JEE.indb 1113 07-06-2018 14:42:24


1114 Mathematics Problem Book for JEE

25.7 Differential Equations of First- Solution: Separating the variables


dy
Order and First-Degree = e - y (e x + x 2 ) Þ ò e y dy = ò (e x + x 2 )dx + A
dx
A differential equation of first-order and first-degree involves x, y On integrating, the solution is
and dy/dx. Thus, it can be put in any one of the following forms:
dy x3
dy
= f ( x , y ) or f ( x , y , ) or f(x, y)dx + g(x, y)dy = 0 ey = ex + + A ⇒ 3(ey - ex ) = x3 + C
dx dx 3
where f(x, y) and g(x, y) are obviously the functions of x and y. (C is an arbitrary constant)
dy
25.7.1 Geometrical Interpretation of the Illustration 25.7 Solve = e x y.
dx
Differential Equations of First-Order Solution: Given
and First-Degree dy dy
= ex y Þ ò
y ò
= e x dx + c
The general form of a first-order and first-degree differential dx
equation is On integrating, we get ln y = ex + c.
æ dy ö
f ç x , y , ÷ = 0  (25.3) Illustration 25.8 Solve the differential equation
è dx ø
The direction of the tangent of a curve in the Cartesian rectangular dy
coordinates at any point is given by dy⁄dx, therefore Eq. (25.3) (1+ x 2 ) = x (1+ y 2 )
dx
establishes the relationship between the coordinates of a point
and the slope of the tangent. Solution of the differential equation Solution: Separating the variables, we can rewrite the given
given by Eq. (25.3) gives those curves for which the direction of differential equation as
tangent at each point coincides with the direction of the field.
xdx dy
All the curves represented by the general solution, when taken =
together, will give the locus of the differential equation. The 1+ x 2 1+ y 2
locus of the general solution can be said to be made up of single 2 xdx dy
infinity of curves as there is one arbitrary constant in the general Þò = 2ò + c Þ 2 tan-1 y = loge (1+ x 2 ) + c
1+ x 2 1+ y 2
solution of the equation of first order.

25.8 Solution of First-Order and 25.10 Equation Reducible to Variable


First-Degree Differential Equations Separable Type Differential
A first-order and first-degree differential equation can be written as Equation
f(x, y)dx + g(x, y)dy = 0 Sometimes differential equation of the first order cannot be solved
dy f ( x , y ) directly by variable separation but by some substitution we can
Þ =
dx g( x , y ) reduce it to a differential equation with separable variables.
A differential equation of the form
dy
Þ = f( x , y ) dy
dx = f (ax + by + c )
dx
where f(x, y) and g(x, y) are obviously the functions of x and y. It is
not always possible to solve this type of equations. The solution is solved by taking
of this type of differential equations is possible only when it falls dy dZ
ax + by + c = Z ⇒ a + b =
under the category of some standard forms. dx dx
The simple standard form of the differential equation of the Therefore,
first order and first degree is given in the following section. æ dZ ö1 dZ
ç - a ÷ = f (Z ) Þ = a + bf ( Z )
è dx øb dx
25.9 Variable Separable Type
This is the variable separable form which can be solved.
Differential Equation
dy Illustration 25.9 Solve the differential equation
Differential equations of the form = f ( x , y ) can be reduced to
dx
form Ψ(x)dx + f (y)dy = 0 then integrate it, i.e. find ∫ Ψ(x)dx + dy
= cos( x + y )
∫ f (y)dy = c. dx
Solution: Put x + y = t. Then differentiating w.r.t. x
Illustration 25.6 Solve the differential equation
dy dy dt
= e x - y + x 2e - y 1+ =
dx dx dx

Mathematical Problem Book for JEE.indb 1114 07-06-2018 14:42:42


Chapter 25 | Differential Equations 1115

Thus, 25.11 Homogeneous Type Differential


dt
- 1 = cos t Equation
dx
dt A function f(x, y) is called a homogeneous function of degree n if
Þò
1+ cos t ò
= dx + c f ( l x , l y ) = l nf ( x , y ) .
On integrating both sides, we get For example, f ( x , y ) = x 2 - y 2 + 3 xy is a homogeneous function

ætö of degree 2 because f (l x , l y ) = l 2 x 2 - l 2 y 2 + 3l x × l y = l 2f ( x , y ).


Þ tan ç ÷ = x + c
è2ø A homogeneous function f(x, y) of degree n can always be written
æyö æxö
æx+yö as f ( x , y ) = x nf ç ÷ or f ( x , y ) = y nf ç ÷.
Þ tan ç ÷= x +c èxø èyø
è 2 ø
25.11.1 Steps for Solving Homogeneous
dy
Illustration 25.10 Solve ( x - y )2 = 1. Differential Equation
dx
Solution: Put z = x – y. Then differentiating w.r.t. x Step 1:   Put the differential equation in the form
dy f ( x , y )
dz dy dy dz =
= 1- Þ = 1- dx y ( x , y )
dx dx dx dx
Now,
æ dz ö dz dy dv æ dx dv ö
z 2 ç 1- ÷ = 1 Þ z 2 = z2 -1 Step 2: Put y = vx (or x = vy) and = v + x ç or =v + y ÷
è dx ø dx dx dx è dy dy ø
in the equation in step 1 and cancel out x (or y) from the
z2 right-hand side. The equation reduces to the form
Þ dx = dz
z2 -1
dv  dv 
which is in the form of variable separable. v+x = F (v )  or v + y = F (v )
dx  dy 
Now integrating, we get
1 z -1 Step 3: 
Shift v on the right-hand side and separate the variables
x = z + ln +c v and x (or y).
2 z +1
Step 4: 
Integrate both sides of the equation to obtain the
1 x - y -1 solution in terms of v and x (or y).
Thus, the solution is x = ( x - y ) + ln +c
2 x - y +1
Replace v by y æç or x ö÷ in the solution obtained in step
Step 5: 
Illustration 25.11 The solution of the differential equation x è yø
4 to obtain the solution in terms of x and y.
dy
= ( 4 x + y + 1)2
dx dy x - y
Illustration 25.12 Solve the differential equation = .
Solution: Let 4x + y + 1 = z. Then dx x + y
Solution: Put y = vx. Then differentiating w.r.t. x
dy dz dy dz
4+ = Þ = -4
dx dx dx dx dy dv
= v × 1+ x
dx dx
Therefore,
dy Substituting into the given equation
= ( 4 x + y + 1)2
dx
dv x - vx 1- v
dz v+x = =
Þ - 4 = z2 dx x + vx 1+ v
dx
dv 1- v 1- v - v - v 2
dz Þx = -v =
Þ = z2 + 4 dx 1+ v 1+ v
dx
dz Now, separating the variables
Þò
z +4 ò
= dx + c
2
1+ v dx
1 z ò 1- 2v - v 2 dv = ò x
+A
Þ tan-1 = x + c
2 2
1
Þ - log(1- 2v - v 2 ) = log x + A
4 x + y + 1ö
-1 æ 2
Þ tan ç ÷ = 2 x + 2c
è 2 ø ⇒ log[(1 − 2v − v2) x2] = constant

Mathematical Problem Book for JEE.indb 1115 07-06-2018 14:43:15


1116 Mathematics Problem Book for JEE

⇒ (1 − 2v − v2) x2 = C ( y / x )3 + 2( y / x ) dv v 3 + 2v dv
⇒ x2 − 2xy − y2 = C Þ 2
= v + x Þ 2
=v + x
1+ 2( y / x ) dx 1+ 2v dx
Illustration 25.13 Solve the differential equation dv ìï v 2 + 2 üï ìï 1- v 2 üï
Þx =ví 2
- 1ý = v í 2ý
dy x 2 + y 2 dx ïî1+ 2v ïþ îï1+ 2v þï
=
dx 2 xy 1+ 2v 2 dx 1+ 2v 2 dx
Þ 2
dv = Þ dv =
Solution: Put y = vx. Then differentiating w.r.t. x v (1- v ) x v (1- v )(1+ v ) x
æA B D ö dx
x2 + v2x2 dv Þç + + ÷ dv =
=v + x è v 1- v 1+ v ø x
2 x × vx dx
where,
dx 2v A(1- v )(1+ v ) + Bv (1+ v ) + Dv (1- v ) = 1+ 2v 2
Þ = dv
x 1- v 2 Putting, v = 0, we get A = 1
Integrating both sides, we set 3
C + ln x = −ln(1 - v2) v = 1, we get B =
2
⇒ ln kx + ln(1 - v2) = 0 3
⇒ kx(1 − v2) = 1 ⇒ k(x2 - y2) = x v = - 1, we get D = -
2
Hence,
Illustration 25.14 Solve the differential equation
æ1 3 1 3 1 ö dx
ç + - ÷ dv =
dy è v 2 1 - v 2 1 + v ø x
x = y (log y - log x + 1)
dx On integrating both sides, we get
Solution: The given equation may be expressed as 3 ln(1- v ) 3
ln v + - ln(1+ v ) = ln x + ln c
dy y é æ y ö ù 2 -1 2
= log ç ÷ + 1 (1)
dx x êë è x ø úû 3 3
Þ ln v - ln(1- v ) - ln(1+ v ) = ln cx

2 2
y y
Let = v . Then Let = v . Then æv ö
2
x x Þ v / {(1- v )(1+ v )}3 / 2 = cx Þ ç ÷ = (1- v 2 )3
dy dv dy dv è cx ø
y = vx ⇒ = v + xy = vx ⇒ =v+x
dx dx dx dx 2 3
æ y ö æ y ö
2
x2y2
Therefore, from Eq. (1), Þ ç 2 ÷ = çç 1- 2 ÷÷ Þ ( x 2 - y 2 )3 = 2
è cx ø è x ø c
dv dv
v+x = v (log v + 1) Þ x = v log v Thus, the required solution is
dx dx
 1 
dv dx 1 dx ( x 2 − y 2 )3 = Bx 2 y 2  2 = B
= Þò d (log v ) = ò + c c
v log v x log v x

Therefore, 25.12 Non-Homogeneous Type


log (log v) = log x + log c ⇒ log (log v) = log (cx)
Differential Equation
⇒ log v = cx
y A differential equation of the form
Þ v = e cx Þ = e cx
x dy a1x + b1y + c1
=
Hence, the required solution is y = xecx. dx a2 x + b2 y + c2
a1 b1
Illustration 25.15 Solve the differential equation where ó , can be reduced to homogeneous equation by
a2 b2
dy y 3 + 2 x 2 y putting x = X + h and y = Y + k, where h and k are such that
=
dx x 3 + 2 xy 2 a1h + b1k + c1 = 0 and a2h + b2k + c2 = 0. Also

Solution: The given equation is homogeneous. dy dY


=
Let y = vx. Therefore, dx dX

dy dv y3 + 2x2y dv dY a1X + b1Y


=v + x Þ 3 =v+ x Hence, the equation reduces to = (homogeneous
dx dx x + 2 xy 2
dx dX a2 X + b2Y
form).

Mathematical Problem Book for JEE.indb 1116 07-06-2018 14:43:42


Chapter 25 | Differential Equations 1117

a1 b1 dy a1x + b1y + c1 l (a1x + b1y ) + c1 1 3


If = = l , then = = can be Þ log(1+ V ) - log(1- V ) - log X = A
a2 b2 dx a2 x + b2 y + c2 (a1x + b1y ) + c2 2 2
Now,
solved by putting a1x + b1y = t. It then reduces to equation with
Y y - m y -1
variable separable. V== =
X x - l x -1
dy x - y +3
Illustration 25.16 Solve = . Reverting to x and y, the solution is
dx 2 x - 2 y + 5
1 æ y - 1ö 3 æ y - 1ö
log ç 1+ ÷ - log ç 1- ÷ - log( x - 1) = A
Solution: Here, 2 è x - 1ø 2 è x - 1ø
a1 b1 1
= = which simplifies to
a2 b2 2
é x + y -2 ( x - 1)3 ù
Hence, we put x−y = v. Then ê 2
× 3ú
= C ( = e2 A )
êë ( x - 1)( x - 1) ( x - y ) úû
dy dv dy dv
1- = Þ = 1- ⇒(x + y− 2) = C (x−y)3
dx dx dx dx
dv v + 3 2v + 5 Illustration 25.18 Solve the differential equation
Þ 1- = Þ dv = dx
dx 2v + 5 v +2 dy x + y +1
=
æ 1 ö dx 2 x + 2 y + 1
ç2 + ÷ dv = dx
è v + 2ø
Solution: x and y coefficients in x + y + 1 are proportional to the x
On integrating both sides, we get and y coefficients in 2x + 2y + 1.
2v + ln(v +2) = x + C Therefore, in this case, take x + y = Z. Then
Put the value of v. Therefore, dy dZ
1+ =
x - 2y + ln(x - y +2) = C dx dx
dZ Z +1
- 1=
Illustration 25.17 Solve the differential equation dx 2Z + 1
dy x + 2 y - 3 dZ Z + 1+ 2 Z + 1 3 Z + 2
= = =
dx 2 x + y - 3 dx 2Z + 1 2Z + 1
Separating the variables and integrating, we get
Solution: Take x = X + l; y = Y + m (l and m are constants). So,
2Z + 1
dy dY ∫ 3Z + 2 dZ = ∫ dx + A
=
dx dX where A is an arbitrary constant.
Therefore, the equation becomes (in X, Y) 2 1
(3 Z + 2 ) -
dY X + 2Y + l + 2m − 3 X + 2Y
= = ò 3 (3Z + 2) 3 dZ = x + A
dX 2 X + Y + 2l + m − 3 2 X + y
2 1
If l, m are chosen to satisfy Þ Z - log(3 Z + 2) = x + A
3 9
l + 2m − 3 = 0 l = 1 Reverting to x and y (x + y = Z)
⇒ 
2l + m − 3 = 0 m = 1 2 1
( x + y ) - x - log(3 x + 3 y + 2) = A
In X, Y the equation is homogeneous and of the first degree. Put 3 9
Y = VX. Then 1 1
dV X + 2VX 1+ 2V ⇒ ( - x + 2 y ) - log(3 x + 3 y + 2) = A
V+X = = 3 9
dX 2 X + VX 2 + V ⇒ -3 x + 6 y = log(3 x + 3 y + 2) + C
2
dV 1+ 2V - (2 + V )V 1- V
X = =
dX 2 +V 2 +V Illustration 25.19 Solve the differential equation
Separating the variables (X, V) and integrating, dy x - 3 y + 2
=
2 +V dX dx 3 x - y + 6
ò 1- V 2 dV = ò X
+A
Solution: Given equation is non-homogeneous.
where A is an arbitrary constant. Let x = X + h, y = Y + k. Then

æ1 1 3 1 ö dX dy dY
=
ò çè 2 × 1 + V + 2 × 1 - V ÷ø dV = ò X
+A dx dX

Mathematical Problem Book for JEE.indb 1117 07-06-2018 14:44:12


1118 Mathematics Problem Book for JEE

Therefore, x2
dY ( X + h) - 3(Y + k ) + 2 X - 3Y + (h - 3k + 2) (A)  y = e x + sin x + + logcos x + c
= = 2
dX 3( X + h) - (Y + k ) + 6 3 X - Y + (3h - k + 6 )
Let us select h and k so that h - 3k + 2 = 0 and 3h - k + 6 = 0. x2
(B)  y = e x + sin x + + logsec x + c
Solving, k = 0 and h = -2 2
Therefore,
x2
X = x - h = x + 2, Y = y - k = y (C)  y = e x - sin x + + logcos x + c
Hence, 2
dY X - 3Y x2
= , which is homogeneous. (D)  y = e x - sin x + + logsec x + c 
dX 3 X - Y 2
Now, let Y = vX. Then Ans. (B)
dY dv
=v + X 4. The solution of the differential equation (sin x + cos x)dy +
dX dX
(cos x - sin x)dx = 0 is
X - 3Y dv
Þ =v + X (A)  ex (sin x + cos x) + c = 0 (B)  ey (sin x + cos x) = c
3X - Y dX
(C)  ey (cos x - sin x) = c (D)  ex (sin x - cos x) = c
1- 3(Y / X ) dv
Þ =v+ X  Ans. (B)
3 - (Y / X ) dX
5. The solution of the differential equation
1- 3v dv
Þ =v + X dy
3-v dX ( x 2 - yx 2 ) + y 2 + xy 2 = 0 is
dx
dv 1- 3v v 2 - 6v + 1
ÞX = -v = æxö 1 1
dX 3 - v 3-v æyö 1 1
(A)  log ç ÷ = + + c (B)  log ç ÷ = + + c
(3 - v )dv dX èyø x y èxø x y
Þ =
v 2 - 6v + 1 X 1 1 1 1
(C)  log ( xy ) = + +c (D)  log ( xy ) + + =c
2v - 6 dX x y x y
Þ 2 dv = -2
v - 6v + 1 X  Ans. (A)
On integrating both side, we get
6. The solution of the differential equation x(e2y - 1)dy + (x2 - 1)
In(v2 - 6v + 1) = -2 In X + In c eydx = 0 is
⇒ In(v2 - 6v + 1) + In X 2 = In c
⇒ X 2(v2 - 6v + 1) = c x2 x2
(A)  e y + e - y = log x - +c (B)  e y - e - y = log x - +c
⇒ Y 2 - 6XY + X 2 = c 2 2
Thus, the required solution is x2
y 2 - 6(x + 2)y + (x + 2)2 = c (C)  e y + e - y = log x + +c (D) None of these
2
 Ans. (A)
7. The solution of the differential equation (3xy + y2)dx +
Your Turn 2 (x2 + xy)dy = 0 is
(A)  x2(2xy + y2) = c2 (B)  x2(2xy - y2) = c2
1. The solution of the differential equation x cos ydy = (xex log x
+ ex)dx is (C)  x2(y2 - 2xy) = c2 (D) None of these
1 x Ans. (A)
(A)  sin y = e +c (B) sin y + ex log x + c = 0
x dy xy
8. The solution of the differential equation = is
(C) sin y = ex log x + c (D) None of these dx x 2 + y 2
Ans. (C) 2 x 2
/ y2 x/y
(A)  ay = e (B)  ay = e
dy
2. The solution of the equation = e x - y + x 2e - y is x 2
y 2
x 2
2
dx (C)  y = e + e + c (D)  y = e + y + c
x3 Ans. (A)
(A)  e y = e x + +c (B)  ey = ex + 2x + c
3 9. The general solution of the differential equation (2x - y + 1)
(C)  ey = ex + x3 + c (D)  y = ex + c dx + (2y - x + 1) dy = 0 is
Ans. (A) (A)  x2 + y2 + xy - x + y = c (B)  x2 + y2 - xy + x + y = c
3. The solution of the differential equation (C)  x2 - y2 + 2xy - x + y = c (D)  x2 - y2 - 2xy + x - y = c
dy
= e x + cos x + x + tan x is  Ans. (B)
dx

Mathematical Problem Book for JEE.indb 1118 07-06-2018 14:44:40


Chapter 25 | Differential Equations 1119

dy x æ x 2 ö 2 xy 2dx - 2 x 2 ydy æ y 2 ö 2 x 2 ydy - 2 xy 2dx


10. The solution of the equation = is
dx 2 y - x  7.  d çç 2 ÷÷ =  8.  d çç 2 ÷÷ =
èy ø y4 èx ø x4
(A) (x - y) (x + 2y)2 = c (B)  y = x + c
y xdy - ydx
x
æ x ö ydx - xdy
 9.  d ç tan-1 ÷ = 10.  d æç tan-1 ö÷ =
(C)  y = (2y - x) + c (D)  y = +c è yø x2 + y2 è xø x2 + y2
2y - x
xdy + ydx æ æ x ö ö ydx - xdy
Ans. (A) 11.  d[ln( xy )] = 12.  d çç ln ç ÷ ÷÷ =
xy
è è y øø xy
1 xdx + ydy
25.13 Exact Differential Equation 13.  d éê ln( x 2 + y 2 )ùú = é y ù xdy - ydx
14.  d êln æç ö÷ ú =
ë2 û x2 + y2 ë è x øû xy
If M and N are functions of x and y, then the equation Mdx + Ndy = 0
is called exact when there exists a function f(x, y) of x and y such æ 1 ö xdy + ydx æ ex ö ye x dx - e x dy
15.  d ç - ÷ = 16.  d çç ÷÷ =
that è xy ø x2y2 y2
è y ø
¶f ¶f
d[f(x, y)] = Mdx + Ndy, i.e. dx + dy = Mdx + Ndy æ ey ö xe y dy - e y dx m n
¶x ¶y 18.  d ( x y )
17.  d çç ÷÷ =
¶f è x ø x2 = x m -1y n -1( mydx + nxdy )
where is the partial derivative of f(x, y) with respect to x
¶x xdx + y dy
¶f 2 3
19.  d ( x + y ) = 20.  d æç 1 log x + y ö÷ = x dy - y dx
(treating y as a constant) and is the partial derivative of f(x, y) 2
x +y 2
x-yø x2 - y2
¶y è2
with respect to y (treating x as a constant). 1− n
d[f ( x , y )] f ′( x , y )
The necessary and sufficient condition for the differential 21.  =
equation Mdx + Ndy = 0 to be exact is 1− n [f ( x , y )]n
¶M ¶N Illustration 25.20 Solve the differential equation
=
¶y ¶x (x2 - ay)dx + (y2 - ax)dy = 0
that is, the partial derivative of M(x, y) w.r.t. y is equal to the partial Solution: Here,
derivative of N(x, y) w.r.t. x. M = x2 - ay
If the given equation is exact, then the solution of the exact N = y2 - ax
differential equation is ¶M
= -a
¶y
ò Mdx + ò Ndy =c
Regarding y Only those terms
¶N ¶M ¶N
as constant not contaiining x
= -a ⇒ =
¶x ¶y ¶x
Therefore, the equation is exact.
25.13.1  Integrating Factor
Thus,
A factor which when multiplied by a non-exact differential
∫(x − ay ) dx + ∫ y 2 dy = c
2
equation makes it exact is known as an integrating factor, for
example the non-exact equation ydx - xdy = 0 can be made exact x3 y3
1 ⇒ - ayx + =c
1 3 3
on multiplying by the factor . Hence, 2 is the integrating
y2 y ⇒ x3 - 3axy + y3 = 3c
factor for this equation.
Illustration 25.21 Solve the differential equation
Notes: In general, such a factor exists but except in certain special
(x + y)dx + xdy = 0
cases, and it is likely to be difficult to determine.
The number of the integrating factor for equation Mdx + Ndy = 0 Solution: We have
is infinite. xdx + (ydx + xdy) = 0 ⇒ xdx + d(xy) = 0
On integrating, we get
x2 c
25.13.2  Some Useful Results + xy =
2 2
Therefore, x2 + 2xy = c.
 1.  d(x + y) = dx + dy  2.  d(xy) = xdy + ydx
æ x ö ydx - xdy æ y ö xdy - ydx Illustration 25.22 Solve the differential equation
 3.  d ç ÷ =  4.  d ç ÷ =
èyø y2 èxø x2 (x2 - 4xy - 2y2)dx + (y2 - 4xy - 2x2)dy = 0
Solution: Comparing the given equation with Mdx + Ndy = 0, we
æ x 2 ö 2 xydx - x 2dy æ y 2 ö 2 xydy - y 2dx
 5.  d çç ÷÷ =  6.  d çç ÷÷ = get
è y ø y2 è x ø x2      M = x2 - 4xy - 2y2, N = y2 - 4xy - 2x2

Mathematical Problem Book for JEE.indb 1119 07-06-2018 14:45:10


1120 Mathematics Problem Book for JEE

¶M the independent variable, then it is said to be a linear differential


= -4 x - 4 y equation. Otherwise, it is a non-linear differential equation.
¶y
It follows from the above definition that a differential equation
¶N will be a non-linear differential equation if
= -4 y - 4 x
¶x
  (A) its degree is more than one,
Therefore, (B) any of the differential coefficient has exponent more than one,
¶M ¶N (C) exponent of the dependent variable is more than one, and
=   (D) products containing a dependent variable and its differential
¶y ¶x
coefficients are present.
So, the given differential equation is exact.
On integrating M w.r.t. x, treating y as a constant,
25.14.1 Linear Differential Equation of First Order
x3
ò Mdx = ò ( x - 4 xy - 2 y )dx =
2 2
- 2x2y - 2y2x
3 The general form of a linear differential equation of first order is
On integrating N w.r.t. y, treating x as a constant, dy + Py = Q (25.4)
y 3
y 3 dx
ò Ndy = ò ( y - 4 xy - 2 x )dy = 3 - 2 xy - 2 x y = 3
2 2 2 2
where P and Q are functions of x (or constants).
(omitting -2xy2 - 2x2y which already occur in ∫ M dx) dy dy
For example, x + 2y = x3 , + 2 y = sin x , etc. are linear differ-
Therefore, the solution of the given equation is dx dx
x3 y3 3 3 ential equations. These types of differential equations are solved
- 2 x 2 y - 2 xy 2 + = l ⇒ x + y - 6 xy ( x + y ) = 3l when they are multiplied by a factor, which is called an ­integrating
3 3
factor, because by multiplication of this factor the left-hand side
Hence, x3 + y3 - 6xy(x + y) = c (3l = c) of the differential equation (25.4) becomes the exact differential
of some function.
Illustration 25.23 Solve the differential equation
Multiplying both sides of Eq. (25.4) by e ∫ Pdx, we get
xdy - ydx
xdx + ydy + 2
x + y2
=0

Pdx æ dy ö
ç + Py ÷ = Q e
è dx ø dx {
ò Pdx Þ d y e ò Pdx = Q e ò Pdx
}
Solution: We know that
d(x2 + y2) = 2(xdx + ydy) On integrating both sides w.r.t. x, we get
æyö 1 xdy - ydx xdy - ydx
y e ∫ = ∫ Q e ∫ ⋅ dx + C (25.5)
Pdx Pdx
d tan-1 ç ÷ = 2
. = 2
è x ø 1+ ( y / x ) x2 x + y2
Therefore, the equation becomes which is the required solution, where C is the constant of integration.
e∫
Pdx is called the integrating factor. The solution or Eq. (25.5) in
1 æyö
d ( x 2 + y 2 ) + d tan-1 ç ÷ = 0 (exact equation) ∫
short may also be written as y .(I.F.) = Q.(I.F.) dx + C .
2 èxø
On integrating, we get Illustration 25.24 Solve the differential equation
1 2 y c dy
( x + y 2 ) + tan-1 = æ 1ö
2 x 2 x = y - cos ç ÷
dx èxø
y
Þ x 2 + y 2 + 2 tan-1 =c Solution: Here,
x
dy æ 1ö
x - y = - cos ç ÷
c - x2 - y2 dx èxø
Therefore, y = x tan .
2
dy 1 1 æ 1ö
Þ - . y = - cos ç ÷
25.14 Linear Differential Equation dx x x èxø
this is in the linear form.
A differential equation is a linear differential equation if it is 1
expressible in the form ò - x dx -1
Integrating factor, e = e - log x = elog x = x -1.
n n −1 n−2
d y d y d y dy Multiplying by the integrating factor,
Po n
+ P1 n −1 + P2 n − 2 +  + Pn −1 + Pn y = Q
dx dx dx dx
1 dy 1 1 æ 1ö
where P0, P1, P2, …, Pn-1, Pn and Q are either constants or functions × - y = - 2 cos ç ÷
x dx x 2 x èxø
of the independent variable x.
Thus, if a differential equation when expressed in the form of d ì1 ü 1 æ 1ö
Þ í . y ý = - 2 cos ç ÷
a polynomial involves the derivatives and a dependent variable dx î x þ x èxø
in the first power and there are no product of these, and also the æyö 1 æ 1ö
coefficient of the various terms are either constants or functions of Þ d ç ÷ = - 2 cos ç ÷ dx
èxø x èxø

Mathematical Problem Book for JEE.indb 1120 07-06-2018 14:45:35


Chapter 25 | Differential Equations 1121

æyö 1 æ 1ö dS
Þ ò d ç ÷ = ò - 2 cos ç ÷ dx such that = R , then put S(y) = t. So,
x
è ø x èxø dy
Hence, dt dS dS dy Rdy
= = × =
y æ 1ö æ 1ö dx dx dy dx dx
x ò
= cos ç ÷ d ç ÷
èxø èxø dt
Thus, the differential equation reduces to + P( x )t = Q( x ),
y  1 dx
⇒ = sin   + c , which is the linear differential equation.
x  x
2. Differential equation of the form
where c is an arbitrary constant. dy
+ Py = Qy n
æ 1ö dx
Thus, y - x sin ç ÷ = cx .
èxø P and Q are functions of x and this equation is called Bernoulli’s
equation.
Illustration 25.25 Solve the differential equation To solve this, divide the equation by yn, then
dy 1 dy 1
+ 2 y tan x = sin x + P n -1 = Q
dx y n
dx y
dy 1
Solution: Comparing with + Py = Q , we get Put = t . Then
dx y n-1
(n - 1) dy dt
P = 2 tan x, Q = sin x - =
y n dx dx
I.F. = e ò
2 tan xdx 2 2
= e 2 ln sec x
= eln sec x
= sec2 x
Differential equation reduces to
Multiplying the given equation by I.F. and integrating, we get dt æ 1 ö Q( x )
+ç ÷ P( x )t =
y sec x = ∫ sin x sec xdx = ∫ sec x tan xdx
2 2 dx è n - 1ø (1- n)

Therefore, y sec2 x = sec x + c. Illustration 25.27 Solve the differential equation

Illustration 25.26 Solve the differential equation dy


− y tan x = − y 2 sec x
dx
-1 dy
(1+ y 2 ) + ( x - e tan y
) =0 Solution: Rewriting the given equation,
dx
Solution: We have dy
y −2 − y −1 tan x = − sec x
dx
æ x - e tan ö
-1
y
-1 dy dx
( x - e tan y
) = -(1+ y 2 ) Þ = -ç ÷ Let y −1 = v . Then
dx dy ç 1+ y 2 ÷ Let y −1 = v . Then
è ø
dy dv
-1
− y −2 = dy dv
dx 1 e tan y dx dx − y −2 =
Þ + x = (1) dx dx
dy 1+ y 2 1+ y 2
dx Therefore,
This is a linear differential equation of the form + R( y ) × x = S ( y ) dv
dy + tan x . v = sec x (1)
1 -1
e tan y dx
R= , S=
1+ y 2 1+ y 2 I.F. = e ∫
tan x
= eln sec x = sec x
dy

Integrating factor = e ∫
Rdy 1+ y 2 −1 Multiplying Eq. (1) by sec x and integrating,
=e = e tan y

v sec x = ò sec2 xdx = tan x + c


Multiplying Eq. (1) by I.F. and integrating,
−1
e tan y tan−1 y (e tan y )2 dy (e tan y )2 k
−1 −1
sec x
xe tan
−1
y
=∫ ⋅e dy = ∫ = + Thus, the required solution is = tan x + c .
1+ y 2
1+ y 2 2 2 y

Illustration 25.28 Solve the differential equation


−1 −1
Therefore, 2 xe tan y
= e 2 tan y
+ k.
dz z z
+ log z = 2 (log z )2
25.14.2 
Equation Reducible to Linear dx x x
Differential Equation (Bernoulli’s Solution:
Differential Equation) 1 dz 1 1
+ =
1
z (log z )2 dx x log z x 2
1. Differential equation of the form
dy 1
R ( y ) + P ( x )S ( y ) = Q ( x ) Let = t . Then
dx log z

Mathematical Problem Book for JEE.indb 1121 07-06-2018 14:46:09


1122 Mathematics Problem Book for JEE

1 1 dz dt 1 - x2 2 2
− ⋅ = ⇒ e = x 2 . e- x + e- x + c
(log z )2 z dx dx y2
Therefore, 2
Thus, ( x 2 + 1+ ce x ) y 2 = 1.
dt t 1
- + = 2
dx x x Illustration 25.30 Solve the differential equation
dt t 1 dy y
Þ - = - 2 (1) =
dx x x dx y sin y - x
dx 1
∫− x ln 1 Solution: We have
I.F. = e = e − ln x = e x
=
x dy y
1 =
Multiplying Eq. (1) by and integrating, we get dx y sin y - x
x
dx y sin y - x x
t 1 1 1 1 Þ = = sin y -
x ∫ x3
= − dx = 2 + c ⇒ = +c dy y y
2x x log z 2 x 2
Hence, the required solution is dx 1
Þ + x = sin y
dy y
æ 1 ö æ 1ö 2
ç ÷ x = ç ÷ + cx which is a linear differential equation in x and y.
è ln z ø è2ø
1
Illustration 25.29 Solve the differential equation ∫ y dy
Its integrating factor is I.F. = e = elog y = y .
dy
= x 3 y 3 − xy Thus, the solution is
dx
xy = ò y sin y dy
Solution: Here,
dy ⇒ xy = −y cos y + sin y + c
+ xy = x 3 y 3
dx
Therefore,
25.15 Solution of Differential Equation
1 dy 1 of the First Order but of Higher
+ x . 2 = x 3 (1)
y 3 dx y Degree
1
Put = z . Then In such differential equations, we substitute the lower degree
y2 derivative by some other variable.
-2 dy dz
= Illustration 25.31 Solve the differential equation
y 3 dx dx
2
Therefore, Eq. (1) becomes æ dy ö dy
x ç ÷ + (y - x) - y = 0
1 dz è dx ø dx
- + xz = x 3 Solution: Here,
2 dx
æ æ dy ö dy ö
dz xp 2 + ( y -xpx2)+
p (-yy-=x0)p    
ç -where
y = 0 çpwhere
= ÷p= ÷
Þ - 2 xz = -2 x 3 è è dx ø dx ø
dx
⇒ xp − xp + yp − y = 0
2

It is in the linear form. So, the integrating factor e ∫


−2 xdx 2
= e− x . ⇒ xp(p − 1) + y(p − 1) = 0
Multiplying by it, ⇒ (p − 1) (xp + y) = 0
2 dz
e- x
2
- 2 xe - x × z = -2 x 3 × e - x
2
Thus,
dx p−1=0
d 2 2 or
Þ ( ze - x ) = -2 x 3e - x xp + y = 0
dx
Now,
Therefore,
2 2
p−1=0
d ( ze − x ) = −2 x 3e − x dx dy
Þ =1 ⇒ y = x + c
2
Þ ò d ( ze - x ) = ò -2 x 3 . e - x dx
2
dx
Also,
xp + y = 0
2
Þ ze - x = - ò te -t dt    (putting x2 = t)
dy
é e -t e -t ù x +y =0
= - êt . -ò dt ú = te -t - ò e -t dt dx
êë -1 -1 úû ⇒ xdy + ydx = 0

Mathematical Problem Book for JEE.indb 1122 07-06-2018 14:46:40


Chapter 25 | Differential Equations 1123

⇒ d(xy) = 0 4. The solution of (x - y3)dx + 3xy2 dy = 0 is


⇒ xy = c x 3
Hence, the general solution is (y − x − c) (xy − c) = 0, where c is an (A) log x + (B)  log x + y = k
y x
arbitrary constant.  
x
Illustration 25.32 Solve the differential equation (C)  log x - 3 = k (D) log xy - y3 = k 
y
2
æ dy ö dy Ans. (B)
1+ ç ÷ = x
è dx ø dx 5. If c is any arbitrary constant, then the general solution of the
Solution: Here, differential equation ydx - xdy = xy dx is given by
 dy  (A)  y = cx e-x (B)  x = cx e-x
xp = 1 + p2          where p = 
 dx  (C)  y + ex = cx (D)  yex = cx
1 Ans. (D)
⇒ x = + p (1)
p dy
6. The solution of the equation + y tan x = x m cos x is
which is of the form x = f (y, p) {solvable for x} dx
On differentiating Eq. (1) w.r.t. y, we get (A) (m + 1)y = xm+1 cos x + c(m + 1)cos x
(B)  my = (xm + c) cos x
1 æ -1 ö dp (C)  y = (xm+1 + c)cos x
= ç + 1÷
p è p2 ø dy (D) None of these
æ 1 ö Ans. (A)
⇒dy = p ç 1- 2 ÷ dp
p 7. An integrating factor for the differential equation
è ø
(1 + y2)dx - (tan-1 y - x)dy = 0 is
Therefore,
-1 -1

æ 1ö (A)  tan y (B)  e tan y


y + c = ò ç p - ÷ dp
è p ø 1 1
(C)  (D)  
1+ y 2 x (1+ y 2 )
where c is an arbitrary constant.
Ans. (B)
p2
y +c = - log p (2) 8.  The solution of the differential equation
2
Thus, the p-eliminate, obtained by eliminating p from Eqs. (1) and dy 3 x 2 sin2 x
+ y= is
(2), is the general solution. dx 1+ x 3
1+ x 3
1
(A)  y (1+ x 3 ) = x + sin 2 x + c
Your Turn 3 2
1
2 2 x3
(B)  y (1+ x 3 ) = cx + sin 2 x
1. The solution of y dx - x dy + 3 x y e dx = 0 is 2
x x 1
(A) 
3
+ ex = c (B) 
3
- ex = 0 (C)  y (1+ x 3 ) = cx - sin 2 x
y y 2

x 1
-x x3 (D)  y (1+ x 3 ) = - sin 2 x + c 
(C)  +e =0 (D) None of these 2 4
y
Ans. (D)
Ans. (A)  9. An integrating factor of the differential equation
2. The solution of (1 + xy) y dx + (1 - xy) x dy = 0 is dy
1
- log x
x + y log x = xe x x 2 , ( x > 0 ) is
x 1 æxö 1 dx
(A)  + = k (B)  log ç ÷ = +k
y xy è y ø xy (A)  x ( y + cos x ) = sin x + c
x 1 æxö (B)  x ( y - cos x ) = sin x + c
(C)  + = k (D)  log ç ÷ = xy + k 
y xy èyø (C)  x ( y × cos x ) = sin x + c
Ans. (B)
(D)  x ( y - cos x ) = cos x + c 
3. Solution of (xy cos xy + sin xy)dx + x2 cos xy dy = 0 is
Ans. (A)
(A)  x sin(xy)= k (B)  xy sin(xy)= k
10. An integrating factor of the differential equation
x
(C)  sin( xy ) = k (D)  x sin(xy)= k  1
y dy - log x
x + y log x = xe x x 2 , ( x > 0 ) is
Ans. (A) dx

Mathematical Problem Book for JEE.indb 1123 07-06-2018 14:47:16


1124 Mathematics Problem Book for JEE

(A)  x log x (B)  ( x )log x Therefore,


132 3 t
(C)  ( e )log x
2
(D)  e x  [ - log(T - 30 )] = log . [t ]
1536 2 0
Ans. (B) 1 3
⇒ log 3 = log . t
15 2
25.16 Applications of Differential 15 log 3
⇒ t= minutes
Equation log 3 / 2

25.16.1  Problem Based on Rate of Change Illustration 25.35 A and B are two separate reservoirs of water.
The capacity of A is double that of B. Both the reservoirs are filled
Illustration 25.33 A particle moves in a straight line with a completely with water. Water is released simultaneously from
dx both the reservoirs. For each of the reservoirs, the rate of flow
velocity given by = ( x + 1) (x is the distance described). What is out at any instant is proportional to the quantity of water left in
dt
the time taken by a particle to transverse a distance of 99 metres? the reservoir. After 1h, the quantity of water in A is 1.5 times the
quantity of water in B. After how many hours from the time of
Solution: We have
dx release of water, do both A and B have the same quantity of water?
= dt
x +1 Solution: Let at time t hours, the volume of water in A and B be
On integrating, we get u and v, respectively. From the question,
99 dx t
du
ò0 x + 1 ò0
= dt
dt
= -k1u (1)

99 and
⇒ [ln( x + 1)]0 = t
dv
Therefore, time taken by the particle is = -k2v (2)
dt
t = ln100 = loge (10 )2 = 2 loge 10 At t = 0, u = 2V and v = V (from the question). Solving Eq. (1),
du
Illustration 25.34 The rate of cooling of a substance in = -k1dt
u
moving air is proportional to the difference of temperatures of Integrating both sides, we get
the substance and the air. A substance cools from 36°C to 34°C in log u = −k1t + c1
15 min. Find when the substance will have the temperature 32°C, u = e - k1t + c1 (3)
and it is known that the constant temperature of air is 30°C.
When t = 0, u = 2V. Then
Solution: Let the temperature of the substance at time t minutes 2V = e c1
is T. Hence, Eq. (3) gives
The rate of cooling of the substance = k(T− 30)°C/min (from the
u = 2Ve - k1t  (4)
question).
But the rate of cooling = rate of decrease of temperature Similarly, from Eq. (2), we get
dT v = Ve - k2 t  (5)
= -
dt 3
Therefore, from the question, From the question, if v = v0 when t = 1, then u = v 0 .
2
dT Therefore, Eq. (4) gives
- = k (T - 30 )
dt 3
dT v 0 = 2Ve - k1
or    - = kdt 2
T - 30 and Eq. (5) gives
Hence,
34 15 v 0 = Ve - k2
dT
ò - T - 30 = ò kdt Dividing these, we get
36 0
3 2e - k1
34
⇒ [ - log(T - 30 )] = k [t ]
15 = = 2e k2 - k1
36 0 2 e - k2
Therefore, Thus,
6 1 3 3
log = k .15 ⇒ k = log e k2 - k1 =  (6)
4 15 2 4
Again, Let after T hours, the volume of water in A and B be equal.
32 t
dT Therefore, Eqs. (4) and (5) gives
ò- T - 30 ò0
= kdt
2Ve - k1T = Ve - k2T
36

where t minutes is the required time. ⇒ 2e − k1T = e − k2T ⇒ 2e( k2 − k1 )T = 1

Mathematical Problem Book for JEE.indb 1124 07-06-2018 14:47:46


Chapter 25 | Differential Equations 1125

Using Eq. (6), we get æ dy ö


T 9. The normal meets X-axis at ç x + y , 0 ÷ and Y-axis at
æ3ö è dx ø
2 . ç ÷ =1
è4ø æ x ö
ç 0, y + ÷.
T è (dy / dx ) ø
æ4ö
⇒ç ÷ =2
è3ø 10. The orthogonal trajectories of a family of curves form another
Therefore, family of curves such that each curve of one family cuts all
4 log 2 the curves of the other family at right angles.
T log = log 2 ⇒ T = = log4 / 32
3 4 The differential equation of the orthogonal trajectories of
log
3 æ dy ö
the curves f ç x , y , ÷ = 0 is the family of curves whose
è dx ø
Hence, after log4/3 2 hours, the reservoirs will have the same
quantity of water. æ -dx ö
differential equation is f ç x , y , ÷=0.
è dy ø
25.16.2 Problem Based on Geometry: Some Method: To find the orthogonal trajectories of a family of
Results on Tangents and Normal curves whose differential equation is known, put −dx/dy
in place of dy/dx in the equation. The resulting differential
Y y = f(x) equation is the equation of the orthogonal trajectories.

Illustration 25.36 If the slope of the tangent at (x, y) to a curve


P(x, y)
æ pö y æyö
passing through ç 1, ÷ is given by - cos2 ç ÷ , then find the
è 4 ø x èxø
equation of the curve.
Solution: We have
X
C O A D dy y æyö
= - cos2 ç ÷
dx x èxø
Figure 25.1 Let y = vx. Then
1. (See Fig. 25.1). The equation of the tangent at point P(x, y) to dy dv dv
dy
=v + x Þv + x = v - cos2 v
the curve y = f(x) is Y - y = (X - x) . dx dx dx
dx
dv
⇒ x = - cos2 v
2. The equation of the normal at point P(x, y) to the curve y = f(x) dx
1 dx
is Y - y = - (X - x). ⇒ sec2 vdv = -
æ dy ö x
ç ÷
è dx ø Integrating both sides, we get
2
æ dx ö tan v = - ln x + c
3. The length of the tangent = CP = y 1+ ç ÷
è dy ø ⇒ tan (y/x) = - ln x + c
2
For x = 1, y = π/4, the above equation becomes
æ dy ö
4. The length of the normal = PD = y 1+ ç ÷ tanp /4 = − ln1+ c ⇒ 1 = 0 + c
è dx ø
⇒c=1
dx Therefore,
5.  The length of the Cartesian subtangent = CA = y
dy    tan( y / x ) = 1- ln x
dy -1 -1 -1 é æ e ö ù
6. The length of the Cartesian subnormal = AD = y ⇒ y/ x = tan (1- ln x ) = tan (ln e - ln x ) = tan êln ç ÷ ú
dx ë è x øû
dy -1 é æ e ö ù
7. The initial ordinate of the tangent = OB = y − x ⇒ y = x tan êln ç ÷ ú
dx ë è x øû

æ y ö Illustration 25.37 Find the equation of the curve which is such


8. The tangent meets X-axis at çç x - , 0 ÷÷ and Y-axis at
è ( dy / dx ) ø that the portion of the axis of x cut off between the origin and the
æ dy ö tangent at any point is proportional to the ordinate of that point
ç 0, y - x ÷ .
è dx ø (b is constant of proportionality).

Mathematical Problem Book for JEE.indb 1125 07-06-2018 14:48:14


1126 Mathematics Problem Book for JEE

Solution: Tangent at P(x, y) to the curve y = f(x) may be expressed and


as dy
Y-y= (- x )
dy dx
Y - y = (X - x)
dx dy
Therefore, ⇒ Y =y-x ( = OB )
dx
æ dx ö
Q = ç x - y ,0 ÷ Therefore, the area of the triangle formed by the tangent and the
è dy ø
axes in the first quadrant is
As per question, OQ µ y . Therefore, 1
2= OA ⋅ OB
dx dx x dx 2
x-y µy⇒x-y = by ⇒ - =b
dy dy y dy 1 x (dy / dx ) - y æ dy ö
Þ2= . .çy -x ÷
Therefore, 2 dy / dx è dx ø
dx x
= -b dy æ dy öæ dy ö
dy y Þ4 =çx - y÷çy - x ÷
dx è dx øè dx ø
x
Let = v . Then dy
y Taking = p, we get
dx dv dx
x = vy ⇒ =v+ y 4p = (xp − y) (y − xp)
dy dy
x dv −( y − xp )2 ⇒ y − xp = ± 2 − p
dv
⇒ -b = v + y ⇒ v -b = v + y
y dy dy Þ y = xp ± 2 - p (2)
dv dy On differentiating w.r.t. x, we get
⇒ -b = y ⇒ -b = dv
dy y
dp 1 dp
On integrating, we get p= p+ x ±2. ( −1)
dx 2 −p dx
dy x
∫ dv = −b∫ y
⇒v = -b ln y + a⇒ = a - b ln y (a, an arbitrary constant)
y ⇒
dp ïì 1 ïü
íx ∓ ý=0
dx îï - p þï
Hence, the equation of the curve is x = y (a - b ln y ).
Therefore,
Illustration 25.38 A curve (or line) passes through (1, 1) such 1
dp
that the triangle formed by the coordinate axes and the tangent = 0  or  x = ±
dx - p
at any point of the curve is in the first quadrant and has its area
Now,
equal to 2. Form the differential equation and find the equations dp
of the possible curves. =0 ⇒ p = c
dx
Solution: Let the curve be y = f(x). Then the equation of the
Therefore, putting in Eq. (2) we get, y = cx ± 2 − c
tangent at the point (x, y) of the curve is (see Fig. 25.2)
dy This gives a family of lines. If it passes through (1, 1), then
Y-y= ( X - x ) (1)
dx 1 = c ± 2 −c
⇒ (1 − c)2 = −4c ⇒ (1 + c)2 = 0 ⇒ c = −1
Y
Hence, the line is y = −x ± 2, that is, x + y = ±2.
B But in the first quadrant, x, y are positive. So, x + y = 2.
Now,
(x, y) 1
x =±
-p
1
X
⇒p=-
O A x2

Putting this in Eq. (2), we get


Figure 25.2
Solving Eq. (1) with Y = 0 and X = 0 successively, we get  1 1
y = x ⋅− 2 ±2 2
 x  x
dy
-y = (X - x) Therefore,
dx
1 -3
x (dy / dx ) - y y= ,
⇒ X= ( = OA) x x
dy / dx ⇒ xy = 1, −3

Mathematical Problem Book for JEE.indb 1126 07-06-2018 14:49:02


Chapter 25 | Differential Equations 1127

But in the first quadrant, x, y are positive. So, xy = 1. This is the singular y +4 6+4
solution of Eq. (2), which is a curve, and it passes through (1, 1). When y = 6, then x = = = 5.
2 2
Thus, the required curves (or lines) are x + y = 2 and xy = 1. So, B = (5, 6).
Illustration 25.39 A curve through (1, 2) has its slope at any
The required area = Area (OABDO) − Area (BCAODB)(1) 
2
point (x, y) equal to . Find the area of the region bounded Now,
y -2
6
by the curve and the line 2x − y − 4 = 0. 6 6
y+4 1  y2 
Area (OABDO) = ∫ ( x )linedy = ∫ dy =  + 4 y 
Solution: Here, slope is 2 22
0 0 0
dy 2 1
=
dx y - 2 = [18 + 24 ] = 21
2
⇒ (y − 2)dy = 2dx 6 6
Therefore, y2 − 4 y + 8
Area (BCAODB) = ∫ ( x )curve dy = ∫ dy
ò ( y - 2) dy = 2 ò dx 0 0
4
y2 y 3
y 2 
6
⇒ - 2y = 2x + c
2 = − + 2 y  = 18 − 18 + 12 = 12
 12 2 0
It passes through (1, 2). So,
Therefore, from Eq. (1), the required area = 21 − 12 = 9 sq. units.
4
- 4 = 2 + c ⇒ c = −4
2 Illustration 25.40 Find the orthogonal trajectories of the
Hence, the equation of the curve is circles x2 + y2 − ay = 0 where a is a parameter.
y2
- 2y = 2x - 4 Solution: Here,
2 x2 + y2 − ay = 0
⇒ y2 = 4(x + y) − 8
Differentiating w.r.t. x, we get
To find the area, we have to draw a rough sketch of the curve.
dy dy
When y = 0, then 0 = 4(x + 0) − 8, that is, x = 2. Therefore, the curve 2x + 2y -a = 0
cuts the x-axis at A(2, 0) only. dx dx
Therefore,
When x = 0, then y2 = 4(0 + y) − 8 or y2 − 4y + 8 = 0; so its roots are
imaginary. Thus, the curve does not cut the y-axis. dy x 2 + y 2 dy
2x + 2y - =0
Again, dx y dx
y2 = 4(x + y) − 8 ⇒ y2 − 4y + 4 = 4x − 4
⇒ (y − 2)2 = 4(x − 1) æ 2 2 x2 + y2 ö
çç As x + y - ay = 0 Þ a = ÷
Hence, it is a parabola. è y ÷ø
Its vertex C is (1, 2) and the axis is y − 2 = 0.
y 2 - x 2 dy
x y Þ 2x + =0
Now, the line is 2x − y − 4 = 0, that is, + = 1. y dx
2 −4
This is the differential equation of the circles. The equation of
Hence, it passes through A(2, 0) and (0, −4) [see Fig. 25.3].
orthogonal trajectories is
y
y 2 - x 2 æ dx ö æ -dx dy ö
B(5, 6) 2x + . ç - ÷ = 0   ç putting in place of ÷
D y è dy ø è dy dx ø
C ⇒ 2xy dy + (x2−y2) dx = 0
It is a homogenous equation.
(1, 2) A(2, 0) Put y = vx. Then
x
O dy dy
=v + x
dx dx
Therefore,
æ dv ö 2 2 2
2x · vx · ç v + x ÷ + x - v x = 0
è dx ø
(0, −4) æ dv ö 2 dv
Þ 2v ç v + x ÷ + 1- v 2 = 0 ⇒ 1+ v + 2vx =0
Figure 25.3 è dx ø dx
dx 2v
Solving y2 = 4(x + y) − 8 and 2x − y − 4 = 0, we get ⇒ + dv = 0
x 1+ v 2
⇒ y2 = 2(y + 4) + 4y − 8
On integrating, we get
⇒ y2 − 6y = 0
⇒ y = 0, 6 log x + log(1 + v2) = log c

Mathematical Problem Book for JEE.indb 1127 07-06-2018 14:49:23


1128 Mathematics Problem Book for JEE

Therefore, x(1 + v2) = c 8.  The slope of a curve at any point is the reciprocal of twice the
ordinate at the point and it passes though the point (4, 3). The
æ y2 ö
Þ x çç 1+ 2 ÷÷ = c equation of the curve is
è x ø (A)  x2 = y + 5 (B)  y2 = x - 5
⇒ x + y2 = cx
2
(C)  y = x + 5
2 (D)  x2 = y - 5
Ans. (C)

Your Turn 4 9. Solution of differential equation x dy - y dx = 0 represents


(A) Rectangular hyperbola
1. The equation of the curve which passes through the point (B) Straight line passing through origin
2y (C) Parabola whose vertex is at origin
(1, 1) and whose slope is given by is
x (D) Circle whose centre is at origin
(A)  y = x2 (B)  x2 - y2 = 0 Ans. (B)
(C) 2x2 + y2 = 3 (D) None of these x2 + y2
Ans. (A) 10. Integral curve satisfying y ’ = , y (1) = 2 has the slope at
x2 - y2
2. The equation of the curve that passes through the point (1, 2) the point (1, 0) of the curve equal to
dy -2 xy (A)  - 5/3 (B)  - 1
and satisfies the differential equation = 2 is
dx ( x + 1) (C)  1 (D) 5/3
(A)  y(x2 + 1) = 4 (B)  y(x2 + 1) + 4 = 0 Ans. (C)
(C)  y(x2 - 1) = 4 (D) None of these 11. A particle starts at the origin and moves along the x-axis in
 Ans. (A)  such a way that its velocity at the point (x, 0) is given by the
3. Equation of curve through point (1, 0) which satisfies the dx
formula = cos2 p x . Then the particle never reaches the
differential equation (1 + y2)dx - xydy = 0 is dt
point on
(A)  x2 + y2 = 1 (B)  x2 - y2 = 1 1 3
(C) 2x + y = 3
2 2 (D) None of these (A)  x = (B)  x =
4 4
 Ans. (B)
1
4. Equation of curve passing through (3, 9) which satisfies the (C)  x = (D)  x = 1
2
dy 1
differential equation = x + 2 is Ans. (C)
dx x
(A) 6xy = 3x2 - 6x + 29 (B) 6xy = 3x3 - 29x + 6 12. The slope of the tangent at (x, y) to a curve passing through
(C) 6xy = 3x3 + 29x - 6 (D) None of these x2 + y2
a point (2, 1) is . Then the equation of the curve is
Ans. (C) 2 xy
dy (A) 2(x2 - y2) = 3x (B) 2(x2 - y2) = 6y
5. The differential equation y + x = a (a is any constant)
dx (C)  x(x2 - y2) = 6 (D)  x(x2 + y2) = 10
represents  Ans. (A)
(A) A set of circles having centre on the y-axis
(B) A set of circles centre on the x-axis 13. A function y = f(x) has a second-order derivatives f ′′(x) = 6(x –
(C) A set of ellipses 1). If its graph passes through the point (2, 1) and at that point
(D) None of these Ans. (B) the tangent to the graph is y = 3x - 5, then the function is
æ 7ö (A) (x + 1)3 (B) (x - 1)3
6. The equation of a curve passing through ç 2, ÷ and having
è 2ø (C) (x + 1)2 (D) (x - 1)2
1 Ans. (B)
gradient 1- at (x, y) is
x2
(A)  y = x2 + x + 1 (B)  xy = x2 + x + 1 Additional Solved Examples
(C)  xy = x + 1 (D) None of these
Ans. (B) 1. Solve the differential equation:
7. The equation of the curve through the point (1,0) and whose dy y 2 + y + 1
(i)  + =0
y -1 dx x 2 + x + 1
slope is 2 is
x +x dy
   (ii)  4 x - x2 = 1 + cos 2 y
(A) (y - 1)(x + 1) + 2x = 0 (B) 2x(y - 1) + x + 1 = 0 dx
(C)  x(y - 1)(x + 1) + 2 = 0 (D) None of these dy y 2
Ans. (A)   (iii)  x + =y
dx x

Mathematical Problem Book for JEE.indb 1128 07-06-2018 14:49:38


Chapter 25 | Differential Equations 1129

Solution: Solution:
2
dy ( y + y + 1) (1 + cos 2 x ) dy = (1 + e y ) sin 2 x dx
 (i)  =- 2
dx ( x + x + 1) Separating the variables
dy dx dy sin 2 x
⇒ 2 =- 2 y
= dx
y + y +1 x + x +1 1+ e 1 + cos 2 x
On integrating, é (1 + e y ) - e y ù
dy dx ⇒ ò êê y ú dy = ò tan x dx
ë 1+ e úû
ò 2 2
=-ò
2 2
+k
æ 1ö æ 3 ö æ 1ö æ 3 ö y
⇒ y - log (1 + e ) + log cos x = A
ç y + ÷ + çç ÷ ç x + ÷ + çç ÷
è 2 ø è 2 ÷ø è 2 ø è 2 ÷ø
æ cos x ö
⇒ y + log çç ÷=A
y ÷
æ 1 ö æ 1 ö è 1+ e ø
2 ç y+ ÷ 2 ç x+ ÷
-1
tan ç 2 ÷=- -1
tan ç 2 ÷+k p
Taking y = 0, when x = we get
3 ç 3 ÷ 3 ç 3 ÷ 4
ç ÷ ç ÷
è 2 ø è 2 ø æ 1 ö 3
0 + log ç ÷ = A Þ A = - log 2
which may also be written as è2 2 ø 2
æ 2y + 1 ö -1 æ 2x + 1 ö The solution (called particular solution) is
tan - 1 ç ÷ + tan ç ÷ = c (1)
è 3 ø è 3 ø éæ cos x ö ù
2 y + log êçç y ÷
÷ 2 2ú = 0
where c is now arbitrary constant in which is also êëè 1 + e ø ûú
3
absorbed.
Therefore, 2 2 cos x = (1 + e y ) e - y = e - y + 1.
We know,
3. Find the equation of the curve through the origin which
æ a+b ö
tan-1 a + tan-1 b = tan-1 ç ÷ dy
è 1 - ab ø satisfies the differential equation = ( x - y )2 .
dx
So, Eq. (1) reduces to
Solution: Put x - y = z. Then
2xy + x + y + C′ (x + y + 1) = 1
dy dz
dy dx 1- =
(ii)  = dx dx
1 + cos 2 y 4 x - x2 The equation in z is
dz
1 dy dx 1- = z2
⇒ ò 2
2 cos y

4 - ( x - 2)2
+c dx
dz
⇒ = 1- z 2
The solution is dx
tan y 1 - 1 æ x - 2 ö dz
2
= sin ç
2 è 2 ø
÷+c ⇒ ò 1 - z 2 = ò dx + A
æ x -2ö 1 æ 1+ z ö
⇒ tan y = sin-1 ç ÷+k ⇒ log ç ÷= x + A
è 2 ø 2 è 1- z ø
where ‘k’ is an arbitrary constant.
æ 1+ x - y ö
dy y 2 y So, log ç ÷ = 2 x + 2 A is the solution which may also be
(iii)  + =
dx x 2 x è 1- x + y ø
dv æ 1+ x - y ö
On setting y = vx, the equation is v + x + v 2 = v . Separating written as ç
2x
÷ = Ce , where C is an arbitrary constant.
dx è 1- x + y ø
the variables and integrating,
The curve passes through the origin. Put x = 0, y = 0, we get
dx dv
òx ò v2
+ =A 1
= C Þ C =1
1
1 Therefore, the particular solution is (1 + x - y) = (1 - x + y) e2x.
⇒ log x − = A.
v dy
This simplifies to the form x = cex/y. Solve the differential equation ( x 2 - y 2 )
4.  = 2 xy given that
y = 1, x = 1. dx
dy
Solve the differential equation (1 + cos 2 x )
2.  - (1 + e y ) sin2x = 0;
dx Solution: Taking y = vx and rewriting, the equation is
p dv 2 x × vx
given that y = 0, when x = . v+x = 2
4 dx x - v 2 x 2

Mathematical Problem Book for JEE.indb 1129 07-06-2018 14:50:14


1130 Mathematics Problem Book for JEE

Therefore, On integrating, 6x2 − 9x + y2 − 4y + 5xy = C, which agrees with


dv 2v 2v - v + v 3 the solution by the first method.
=x 2
-v = 6. The tangent and a normal to a curve at any point P meet the x
dx 1 - v 1- v 2
and y axes at A, B, C and D, respectively. Find the equation of
Separating the variables,
the curve passing through (1, 0) if the centre of circle through
1- v 2 dx O, C, P and B lies on the line y = x (where O is origin).
dv =
v + v3 x Solution: Let P(x, y) be a point on the curve. Then
é1 2v ù æ dy ö
⇒ ò ê - ú dv = log x + C C º ç x + y , 0÷
ë v 1 + v2 û è dx ø
⇒ log v − log(1 + v 2 ) − log x = C æ dy ö
B º ç 0, y - x ÷
è dx ø
v
⇒ = constant Circle passing through O, C, P and B has its centre at mid-point of
x (1 + v 2 )
BC (Fig. 25.4).
Reverting to y, we get, Let the centre of the circle be (α, β). Then
y dy dy
=A 2a = x + y and 2 b = y - x
x2 + y2 dx dx
y = A(x2 + y2) and since β = α. Therefore,
Put x = 1, y = 1. Therefore, dy dy
1 y-x =x+y
1 = A ×2 Þ A = dx dx
2
dy y - x
The particular solution is 2y = x2 + y2. Þ =
dx y + x
dy 12 x + 5 y − 9
5. 
Solve the differential equation + = 0.
dx 5 x + 2 y − 4
B
Solution: Putting x = X + l; y = Y + m.
dY 12 X + 5Y 12l + 5m - 9 = 0 ü l =2 ü
+ = 0 where ý Þ ý
dX 5 X + 2Y 5l + 2m - 4 = 0 þ m = -3þ P
Setting Y = VX, we get
dV 12 + 5V A
V+X + =0
dX 5 + 2V O C

dV æ 12 + 5V + 2V 2 + 5V ö
⇒ X = - çç ÷÷
dX è 5 + 2V ø
D
Separating the variables and integrating,
2V + 5 dX Figure 25.4
ò 2V 2 + 10V + 12 dV = - ò X
+A
Let y = vx. Then
dv (1+ v 2 ) 1+ v dx
1 x=- Þ 2 dv = -
⇒ log (2V 2 + 10V + 12) + log X = A dx 1+ v v +1 x
2
On integrating both sides, we get
⇒ (2V 2 + 10V + 12) X 2 = e 2 A = C 1 2v dv dx
2 ò v2 +1
dv + ò 2
v +1 ò x
=-
⇒ 2Y 2 + 10 XY + 12 X 2 = C
1
C Þ log| v 2 + 1| + tan-1 | v | = - log x + c
⇒ Y 2 + 5 XY + 6 X 2 = =C’ 2
2
⇒ ( y + 3)2 + 5( x − 2) ( y + 3) + 6( x − 2)2 = C ’ { }
Þ log ( v 2 + 1) x + tan-1 v = c

which gives the solution in the form 6x2 + 5xy + y2 − 9x − 4y = C′. y


Þ log x 2 + y 2 + tan-1 =c
Alternative method: The equation may be rewritten as x
As x = 1 and y = 0, we get
(12 x + 5 y − 9 ) dx + (5 x + 2 y − 4 ) dy = 0
log1 + tan-1 0 = c
(12 x − 9 ) dx + (2 y − 4 ) dy + 5 y dx + 5 x dy = 0 ⇒c=0

(12 x − 9 ) dx + (2 y − 4 ) dy + 5d ( xy ) = 0
2 2
( )
-1 æ y ö
Therefore, the required curve is log x + y + tan ç ÷ = 0 .
èxø

Mathematical Problem Book for JEE.indb 1130 07-06-2018 14:50:45


Chapter 25 | Differential Equations 1131

dy Therefore,
7. 
Solve + y f ′( x ) = f ( x ) . f ′( x ) where f (x) is a given function
dx dy1
of x. + P ( x ) × y1 = Q ( x )  (2)
dx
Solution: Here,
dy dy 2
= {f ( x ) - y } f ¢( x )  (1) and + P ( x ) × y 2 = Q ( x ) (3)
dx dx
Put f (x) − y = z. Then From Eqs. (1) and (2),
dy dz æ dy dy1 ö
f ¢( x ) - =
dx dx ç - ÷ + P ( x ) × ( y - y1) = 0
è dx dx ø
Hence, Eq. (1) becomes
Therefore,
dz d
f ¢( x ) - = zf ¢( x ) ( y - y1) + P ( x ) . ( y - y1) = 0 (4)
dx dx
dz dz From Eqs. (2) and (3),
= (1- z ) f ¢( x ) ⇒ = f ¢( x ) dx
dx 1- z d
( y1 - y 2 ) + P ( x ) × ( y1 - y 2 ) = 0 (5)
dz dx
⇒ ò 1- z = ò f ¢( x ) dx ⇒ −log (1 − z) = f (x) − c
From Eqs. (4) and (5),
f(x) + log(1 − z) = c d
( y - y1)
Thus, f(x) + log {1 + y − f (x)} = c. dx y - y1
=
d y1 - y2
( y1 - y 2 )
8. 
Find the equation of the curve passing through (1, 2) whose dx
differential equation is y(x + y3) dx = x(y3 − x)dy. Therefore, on integrating,
Solution: Here, log(y − y1) = log(y1 − y2) + log c
(xy + y4)dx = (xy3 − x2)dy ⇒ log(y − y1) = log [c(y1 − y2)]
⇒ y3(ydx − xdy) + x(ydx + xdy) = 0 ⇒ y = y1 + c (y1 − y2)

xdy - ydx Now, y = αy1 + βy2 will be a solution if


⇒ -x2y3. + xd ( xy ) = 0
x2 d
y æ y ö d ( xy ) (a y1+ b y 2 ) + P ( x ) . (a y1+ b y 2) = Q ( x )
⇒ - d ç ÷ + 2 2 = 0 (dividing by x3y2) dx
x èxø x y
ì dy ü ì dy ü
y æyö d ( xy ) ⇒ a í 1 + P( x ) y1 ý + b í 2 + P ( x ) y 2 ý = Q ( x )
⇒ -ò d ç ÷ + ò =c î dx þ î dx þ
x èxø ( xy )2
⇒ α · Q(x) + β · Q(x) = Q(x) (using Eqs. (2) and (3))
Therefore, Therefore,
2
æyö (α + β) Q(x) = Q(x)
ç ÷ ( xy )-1 Hence, α + β = 1.
x
-è ø + =c
2 -1 10. 
Use the methods of solving the first-order differential equation
1 y2 1 dy . d 3 y  d2 y 
2
⇒ . 2 + +c =0 to find the general solution of = 3 2  .
2 x xy dx dx 3
 dx 
y3 + 2x + 2cx2y = 0 dy
Solution: Put = p . Then the equation is
It passes through (1, 2). So, 23 + 2 + 2c ⋅ 2 = 0. Therefore, dx
2
-10 -5 d2p æ dp ö
c= = p = 3ç ÷
4 2 dx 2 è dx ø
Hence, the curve is y3 + 2x − 5x2y = 0. Therefore,
9.  If y1, y2 are two solutions of the differential equation d2p
dy dx 2 = 3 dp
+ P( x ) . y = Q( x ), then prove that y = y1 + c(y1 − y2) is the dp p dx
dx
general solution of the equation where c is any constant. For dx
what relation between the constants α and β will the linear 1 d  dp  3 dp
. =
combination αy1 + βy2 also be a solution? ⇒ dp dx  dx  p dx
Solution: As y1 and y2 are two solutions of the differential equation dx
dy d ì dp ü 3 dp
+ P ( x ) × y = Q ( x ) (1) ⇒ ílog ý =
dx dx î dx þ p dx

Mathematical Problem Book for JEE.indb 1131 07-06-2018 14:51:14


1132 Mathematics Problem Book for JEE

ì dp ü 3 So, the differential equation is


⇒ d ílog ý = dp 2 3/ 2
î dx þ p dy æ dy ö æ dy ö
y = x× + ç ÷ - 3×ç ÷ +2
On integrating, we get dx è dx ø è dx ø
dp 3 Clearly, its order is 1 and degree is 4.
dx ò p
log = dp = 3 log p + log a Hence, the correct answer is option (A).
14.  y = a sinx + b cos x is the solution of differential equation
with a being an arbitrary constant.
Therefore, d2 y dy
(A)  + y = 0 (B)  +y=0
dp 3
dx 2
dx
= e 3 log p +log a = elog( ap ) = ap3
dx
d2 y dy
dp (C)  = y (D)  =y
or = adx dx 2
dx
p3
Solution:
On integrating, we get
dy d2 y
1 = a cosx−b sinx ⇒ = −a sinx - bcosx = −y
- 2 = ax + b dx dx 2
2p
d2 y
-1 1 Hence, the equation is + y = 0.
⇒ = p2 ⇒ p = - dx 2
2(ax + b ) 2(ax + b )
Hence, the correct answer is option (A).
dy -1 15. For any differential function y = f(x), the value of
⇒ =
dx 2(ax + b ) 3
d 2 y æ dy ö d 2 x
Therefore, +ç ÷ is:
dx 2 è dx ø dy 2
1 -2ax - b
y=ò dx Þ y = +c (A) always zero (B) always non-zero
-2ax - b -a
where a, b and c are arbitrary constants. (C) equal to 2y2 (D) equal to x2
11. 
A curve that passes through (2, 4) and having subnormal of Solution: For a differential equation,
constant length of 8 units can be -1
(A)  y2 = 16x - 8 (B)  y2 = −16x + 24 æ dy ö æ dx ö
ç ÷ = ç ÷
(C)  x2 = 16y - 60 (D)  x2 = −16y + 68 è dx ø è dy ø
-2
Solution: Let the curve be y = f(x). d2 y æ dx ö d æ dx ö dy
dy Þ = -1ç ÷ ç ÷
Subnormal at any point = y dx 2 è dy ø dy è dy ø dx
dx -2 3
æ dx ö d 2 x dy d 2 x æ dy ö
dy y2 = -ç ÷ 2
=- 2 ç ÷
  y = ±8 ⇒ ydy = ±8 dx ⇒ = ±8x + c è dy ø dy dx dy è dx ø
dx 2
3
       ⇒ y2 = 16x + 2c1 ⇒ c1 = −8 d 2 y æ dy ö d 2 x
Þ +ç ÷ =0
or    y2 = −16x + 2c2  ⇒ c2 = 24 dx 2 è dx ø dy 2
Hence, the correct answers are options (A) and (B). Hence, the correct answer is option (A).
2
12. Equation of a curve that would cut + x2 y2 −
2x − 4y − 15 = 0 d2 y æ dy ö d2 y
16. The degree of differential equation + 3 ç ÷ = x log
orthogonally can be is dx 2 è dx ø dx 2
(A) (y − 2) = l(x − 1) (B) (y − 1) = l(x − 2)
(A) 
1 (B) 2
(C) (y + 2) = l(x + 1) (D) (y + 1) = l(x + 2)
(C) 
3 (D) none of these
where l∈R.
Solution: Any line passing through the centre of the given circle Solution: Since the equation is not a polynomial in all the
would meet the circle orthogonally. differential coefficient so the degree of equation is not defined.
Hence, the correct answer is option (A). Hence, the correct answer is option (D).

13. Let m and n be the order and the degree of the differential 17. The differential equation of all ellipses centred at origin is
equation whose solution is y = cx +c2 − 3c3/2 +2, where c is a (A)  y2 + xy12 - yy1 = 0 (B)  xyy2 + xy12 - yy1 = 0
parameter. Then (C)  yy2 + xy12 - xy1 = 0 (D) none of these
(A)  m = 1, n = 4 (B)  m = 1, n = 3
(C)  m = 1, n = 2 (D) None Solution: The ellipse centred at origin is given by

Solution: Differentiating y w.r.t. x, we get x2 y2


+ = 1 (1)
dy a2 b 2
=c
dx where a and b are unknown constants.

Mathematical Problem Book for JEE.indb 1132 07-06-2018 14:51:38


Chapter 25 | Differential Equations 1133

2x 2y x y Solution:
+ y1 = 0 ⇒ 2 + 2 y1 = 0 (2) y = (c1 + c4ecs)ex + c2e2x + c3e3x
a2 b 2 a b
On differentiating again, we get ⇒ y = k1ex + k2e2x + k3e3x
Since there are three arbitrary constants.
1 1
+ (y12 + yy2) = 0 (3) Hence, the correct answer is option (C).
a2 b 2
dx
Multiplying Eq. (3) with x and then subtracting from Eq. (2) we get + x - ln y = 0 is
21. I.F. for y ln y
dy
1
( yy1 - xy12 - xyy 2 ) = 0 Þ xyy 2 = xy12 - yy1 = 0 (A) ln x (B) ln y
b2 (C) ln xy (D) none of these
Hence, the correct answer is option (B). Solution:
18. Particular solution of y1 + 3xy = x which passes through (0, 4) is dy
∫ y In y
3x 2
I.F. = e = ln y
- 1 2
(A) 3y = 1 + 11 e 2 (B)  y = + 11 e - x Hence, the correct answer is option (B).
3
11 - x 2 1 3 2
x 22. Which one of the following is a differential equation of the
(C)  y = 1 + e (D)  y = + 11 e 2 family of curves y = Ae2x + Be−2x
3 3
Solution: d2 y dy d2 y dy
dy (A)  - 2 + 2y = 0 (B)  x + 2 - xy + x 2 - 2 = 0
+ (3x)y = x dx 2 dx dx 2 dx
dx 3
3 2 d2 y æ dy ö æ dy ö
ò 3 x dx x (C)  = 4y (D)  ç ÷ = 4 y ç x - 2y ÷
I.F. = e = e2 dx 2 è dx ø è dx ø
Therefore, the solution of given equation is Solution:
3 2 3 2 3 dy
x 1 x2 x y = Ae2x + Be−2x ⇒ = 2(Ae2x - Be−2x)

ye 2 + c = e2 + c
x .e 2 dx dx
3
If the curve passes through (0, 4), then d2 y
11 = 4 (Ae2x + Be−2x) = 4 y
1 dx 2
4 - = c ⇒ c =
3 3
3 3
Hence, the correct answer is option (C).
1 11 - x 2
2
- x
y = + e 2 ⇒ 3y = 1 + 11 e 2 23. Solution of
dy æx+yö æ x - y ö is
3 3 + sin ç ÷ = sin ç ÷
dx è 2 ø è 2 ø
Hence, the correct answer is option (A). y x y x
(A) log tan = c - 2 sin (B) log cot = c - 2 sin
dy 3 x - 4 y - 2 4 2 4 2
19. Solution of equation = is
dx 3 x - 4 y - 3 y x
(C) log tan = c - 2 cos (D) None of these
(A) (x - y)2 + c = log(3x - 4y + 1) 4 2
(B)  x - y + c = log(3x - 4y + 1) Solution:
(C)  x - y + c = log(3x - 4y - 3) dy x y x y
(D)  x - y + c = log(3x - 4y - 1) = -2 cos sin Þ - ò 2 cos dx = ò cosec dy
dx 2 2 2 2
Solution: Let 3x - 4y = z. Then x
⇒ c − 2 sin
dy dz dy 1 æ dz ö 2
3 - 4 = ⇒ = ç3- ÷
dx dx dx 4 è dx ø y
= log tan
Therefore, the given equation becomes 4
3 1 dz z - 2 Hence, the correct answer is option (A).
- =
4 4 dx z - 3 dy y y
24. Solution of = + tan is
æ z -3 ö æ 4 ö dx x x
Þ -ç ÷ dz = dx Þ - ç 1- ÷ dz = dx
è z +1 ø è z + 1ø æyö y
⇒ -z + 4 log(z + 1) = x + c ⇒ log(3x - 4y + 1) = x - y + c (A) sin ç ÷ = kx (B) cos = kx
èxø x
Hence, the correct answer is option (B). y
(C) tan = kx (D) none of these
20. The order of the differential equation, whose general solution x
is y = c1ex + c2e2x + c3e3x + c4 e x + c5 , where c1, c2, c3, c4 and c5 Solution:
are arbitrary constants, is dy y y
= + tan
(A) 5 (B) 4 (C) 3 (D) None of these dx x x

Mathematical Problem Book for JEE.indb 1133 07-06-2018 14:52:16


1134 Mathematics Problem Book for JEE

Put y = vx. Then 3. 


The differential equation of the family of circles with fixed
dv radius 5 units and centre on the line y = 2 is
v+x = v + tan v
dx (A) (x - 2)y ′2 = 25 - (y - 2)2  (B) (y - 2)y ′2 = 25 - (y - 2)2
dx (C) (y - 2)2 y ′2 = 25 - (y - 2)2   (D) (x - 2)2 y ′2 = 25 - (y - 2)2
⇒ cot vdv =
x [AIEEE 2008]
On integrating, we get
y Solution: The equation of the circle is
ln sin v = ln x + ln k ⇒ sin = kx (x - a)2 + (y - 2)2 = 25 (1)
x
On differentiating w.r.t. x,
Hence, the correct answer is option (A).
dy dy
( x − a ) + ( y − 2) = 0 ⇒ x − a = −( y − 2)  (2)
dx dx
Previous Years' Solved JEE Main/AIEEE From Eqs. (1) and (2), on eliminating a,
Questions 2
æ dy ö
( y - 2)2 ç ÷ + ( y - 2)2 = 25 Þ ( y - 2)2 ( y ¢)2 = 25 - ( y - 2)2
1. 
The differential equation of all circles passing through the è dx ø
origin and having their centres on the x-axis is
Hence, the correct answer is option (C).
2 2dy 2 2 dy
(A)  x = y + xy (B)  x = y + 3 xy 4.  The differential equation which represents the family of curves
dx dx
y = c1e c2 x where c1 and c2 are arbitrary constants is
dy dy
(C)  y 2 = x 2 + 2 xy (D)  y 2 = x 2 - 2 xy (A)  y′ = y2 (B)  y″ = y′y (C)  yy″ = y′ (D)  yy″ = (y′)2
dx dx
[AIEEE 2007] [AIEEE 2009]
Solution:
Solution: General equation of such circle is
y = c1e c2 x  (1)
2 2 2 2 2 2 2
( x + g ) + y = g Þ x + g + 2 xg + y = g
y ¢ = c2c1e c2 x Þ y ¢ = c2 y  (2)

⇒ x2 + y2 + 2gx = 0(1) y ¢¢ = c2 y ¢

Now differentiating w.r.t. x, we get

dy dy From (2), we get, c2 = . Therefore,
2x + 2y + 2g = 0 Þ x + y + g = 0 (2) y
dx dx
( y ¢)2
Therefore, from Eq. (1) y ¢¢ = Þ yy ¢¢ = ( y ¢)2
y
 dy 
x2 + y2 + 2 − x − y  x = 0 Hence, the correct answer is option (D).
 dx 
dy 5.  Solution of the differential equation
⇒ x 2 + y 2 − 2 x 2 − 2 yx =0
dx p
cos x dy = y (sin x - y )dx , 0 < x < is
dy 2
⇒ y 2 = x 2 + 2 xy
dx (A)  y sec x = tan x + c (B)  y tan x = sec x + c
Hence, the correct answer is option (C). (C) tanx = (sec x + c)y (D) sec x = (tan x + c)y
dy x + y
2.  The solution of the differential equation = satisfying [AIEEE 2010]
dx x
the condition y (1) = 1 is Solution: We have
dy
(A)  y = In x + x (B)  y = x In x + x2 cos x dy = y (sin x − y ) dx ⇒ = y tan x − y 2 sec x
dx
(C)  y = xe(x - 1) (D)  y = x In x + x 1 dy 1
[AIEEE 2008] ⇒ − tan x = − sec x
y 2 dx y
Solution: Put y = vx. Then
1
dy dv dv dx Let us consider that = t . Then
=v + x Þv + x = 1+ v Þ dv = y
dx dx dx x
1 dy dt dt dt
Therefore, − = ⇒ − − t tan x = − sec x ⇒ + (tan x ) t = sec x
y 2 dx dx dx dx
y
v = ln x + c Þ = ln x + c
So, I.F. = e ∫
tan x dx
x = sec x .
As y (1) = 1, we have, y = x In x + x. Solution is
Hence, the correct answer is option (D). t (I.F) = ∫(I.F) sec xdx

Mathematical Problem Book for JEE.indb 1134 07-06-2018 14:52:39


Chapter 25 | Differential Equations 1135

1 kT 2 kT 2
⇒sec x = tan x + c Þ sec x = y (tan x + c ) I= +c Þc =I -
y 2 2
Hence, the correct answer is option (D). Now, at t = T (for scrap value)
Let f: (-1, 1) → R be a differentiable function with f(0) = -1
6.  kT 2
and f ′(0) = 1. Let g(x) = [f(2f(x) + 2)]2. Then g′(0) = V (T ) = c = I −
2
(A)  -4 (B) 0 Hence, the correct answer is option (A).
(C)  -2 (D) 4  9. 
At present, a firm is manufacturing 2000 items. It is estimated
[AIEEE 2010] that the rate of change of production P w.r.t. additional
Solution: We have dP
æ d ö number of workers x is given by = 100 − 12 x . If the firm
g¢( x ) = 2(f (2f ( x ) + 2)) ç (f (2f ( x ) + 2)) ÷ dx
è dx ø employs 25 more workers, then the new level of production
= 2f (2f ( x ) + 2)f ¢(2f ( x ) + 2) × (2f ¢( x )) of items is

This implies that (A)  3000 (B)  3500
g¢(0 ) = 2f (2f (0 ) + 2) × f ¢(2f (0 ) + 2) × 2(f ¢(0 )) (C)  4500 (D)  2500
[JEE MAIN 2013]
= 4f (0 )f ¢(0 ) = 4( -1)(1) = -4
Solution: Given that
Hence, the correct answer is option (A).
dP
dy = 100 - 12 x Þ dP = (100 - 12 x )dx
7.  If = y + 3 > 0 and y(0) = 2, then y(ln2) is equal to dx
dx
Therefore, the new level of production of items is
(A) 
5 (B) 13
P 25
(C)  -2 (D) 7
[AIEEE 2011]
∫ dP = ∫ (100 − 12 x )dx
2000 0
Solution: We have 12 × 2
dy dy ⇒ ( P − 2000 ) = 25 × 100 − (25)3 / 2
= y +3Þ = dx Þ ln( y + 3) = x + c 3
dx y +3 ⇒ P = 3500
when x = 0, y = 2. So, Hence, the correct answer is option (B).
In 5 = 0 + c 10. 
Let the population of rabbits surviving at a time t be governed
Þ c = ln 5 Þ c = ln 5 dp(t ) 1
by the differential equation = p(t ) - 200. If p(0) = 100,
x + ln 5 dt 2
So, ln( y + 3) = So,
x + ln
ln(5yÞ
+ 3y)+=3x=+eln 5 Þ y + 3 = e x +ln 5
Þ y + 3 = eln 2 +Þ
ln 5
yÞ+ y3 += 3eln=210
+ lnÞ
5 y =7 then p(t) equals
Þ y + 3 = 10 Þ y = 7
(A) 600 - 500 et/2 (B) 400 - 300 e-t/2
Hence, the correct answer is option (D). (C) 400 - 300 et/2 (D) 300 - 200 e-t/2
 8. 
Let I be the purchase value of an equipment and V(t) be the value [JEE MAIN 2014 (OFFLINE)]
after it has been used for t years. The value V(t) depreciates at a Solution:
dV (t ) dp(t ) 1 dp(t )
rate given by differential equation = -k (T - t ), where k > = p(t ) − 200 ⇒ = dt
dt pt 2 1
0 is a constant and T is the total life in years of the equipment. p(t ) − 200
2
Then the scrap value V(T) of the equipment is
On integrating, we get
kT 2 k (T - t )2
(A)  I - (B)  I - p(t )
2 2 log - 200
2 p(t ) - 400 1 c1
= t + c1 Þ log = t+  (1)
1 (1/ 2) 2 2 2
(C)  e - kT (D)  T 2 -
k
Using initial conditions
[AIEEE 2011] 100 − 400 c1
Solution: We have, log = ⇒ 2 log150 = c1  (2)
dV 2 2
= -k (T - t ) Þ dV = -k (T - t )dt
dt From Eqs. (1) and (2), we get
On integrating the above equation, we get
p(t ) - 400 1 p(t ) - 400 1
log = t + log150 Þ log = t
-k (T - t )2 k (T - t )2 2 2 300 2
V= +c ÞV = +c
( -2) 2 p(t ) - 400
= e t / 2or p(t ) - 400 = 300e t / 2
As V(0) = I, we have 300

Mathematical Problem Book for JEE.indb 1135 07-06-2018 14:53:03


1136 Mathematics Problem Book for JEE

Since initially number of rabbits is 100 and is decreasing Therefore,


-(p(t) - 400) = or p(t) =
300et/2 400 - 300et/2 x dy y dy y y 2
= 1- Þ = -  (2)
Hence, the correct answer is option (C). y dx x dx x x 2
11. 
If the differential equation representing the family of all circles Now comparing with the given differential equation

(dy
) 2 2
touching x-axis at the origin is x 2 - y 2 = g( x ) y , then g( x )  x y2  y  1 
dx f   = − 2 = −  = −
equals  y x  x  x / y 
2
1  1 1
(A)  x (B) 2x2 Therefore, f (t ) = −   and f (2) = − .
2 t 4
1 2 Hence, the correct answer is option (D).
(C) 2x (D)  x
2 13. 
The general solution of the differential equation
[JEE MAIN 2014 (ONLINE SET-1)] æ dy ö
sin 2 x ç - tan x ÷ - y = 0 is
Solution: Equation of family is (x − 0)2 + (y − a)2 = a2 è dx ø
(A)  y tan x = x + c (B)  y cot x = tan x + c

(0, a) (C)  y tan x = cot x + c (D)  y cot x = x + c


[JEE MAIN 2014 (ONLINE SET-3)]
Solution: Differential equation is
dy y dy 1
- tan x = Þ - y = tan x (1)
dx sin 2 x dx sin 2 x
Therefore,
x2 + y2 + a2 - 2ay = a2 1
I.F. = e ∫
− cos ec ( 2 x ) dx − log(tan x )
⇒ x2 + y2 − 2ay = 0 (1) =e 2 = cot x
On differentiating Eq. (1), we get From (1), we get
dy dy dy dy cot x
2x + 2y - 2a = 0 Þ x = ( a - y ) cot x - y = tan x cot x
dx dx dx dx sin 2 x
x d
a= +y (2) Þ ( y cot x ) = tan x cot x = 1Þ d ( y cot x ) = dx
dy / dx dx

From Eqs. (1) and (2), we get On integrating, we get
æ x ö 2 xy y cot x = x + c
x2 + y2 - 2ç + y ÷ y = 0 Þ x2 + y2 - - 2y2 = 0
è dy / dx ø dy / dx Hence, the correct answer is option (D).
2 xy dy dy
x2 - y2 - = 0 Þ ( x 2 - y 2 ) = 2 xy = g( x ) y Þ g( x ) = 2 x 14.  If + y tan x = sin 2 x and y (0 ) = 1, then y(π) is equal to
dy / dx dx dx
Hence, the correct answer is option (C). (A) 1 (B)  −1
(C)  −5 (D) 5
12. 
If the general solution of the differential equation [JEE MAIN 2014 (ONLINE SET-4)]
y æxö Solution:
y ’ = + f ç ÷ for some function φ, is given by y ln |cx| = x, dy
x èyø + y tan x = sin2 x
dx
where c is an arbitrary constant, then φ (2) is equal to
Integrating factor,
1 1

tan xdx
(A) 4   (B)     (C)  − 4   (D)  - = elnsec( x ) = sec x
4 4
Therefore,
[JEE MAIN 2014 (ONLINE SET-2)] dy
sec x + y tan x sec x = sin 2 x sec x
dy y æxö dx
Solution: General solution of = + f ç ÷ is
d
  è y ø dx x
Þ ( y sec x )
y In |cx| = x(1) dx
On differentiating y In |cx| = x, we get, = 2 sin x cos x sec x ⇒ d (y sec x) = 2 sin x dx
On integrating,
1 dy
y´ × c + ln| cx | =1 y sec x = 2 ò sin xdx + c Þ y sec x = -2 cos x + c
| cx | dx
y x dy By initial condition,
Þ + =1 1 sec 0 = −2 cos 0 + c ⇒ 1 + 2 = c
x y dx
⇒c=3

Mathematical Problem Book for JEE.indb 1136 07-06-2018 14:53:32


Chapter 25 | Differential Equations 1137

Therefore, (A) 0 (B) 1


-2 cos x + 3 (C)  −1 (D) 2
y sec x = -2 cos x + 3 Þ y =
sec x [JEE MAIN 2015 (ONLINE SET-1)]
-2 cos p + 3 2 + 3 Solution:
Þ y(p ) = = = -5
sec p -1 dy
( x + 2) = x2 + 4 x - 9
Hence, the correct answer is option (C). dx
15. 
Let y(x) be the solution of the differential equation (( x + 2)2 - 13)dx æ x2 ö
Þ ò dy = ò Þ y = ç + 2 x ÷ - 13 ln| x + 2 | + c
dy ( x + 2) è 2 ø
(x log x) + y = 2 x log x ,( x ³ 1)
dx As
Then, y(e) is equal to y(0) = 0 ⇒ 0 = −13 ln 2 + c ⇒ c = 13 ln 2
(A) 0 (B) 2 So,
(C) 2e (D)  e
x2  2 
[JEE MAIN 2015 (OFFLINE)] y= + 2 x + 13 ln 
2  | x + 2 | 
Solution:
dy æ2ö
( x log x ) + y = 2 x log x ;( x ³ 1) Þ y( -4 ) = 8 - 8 + 13 ln ç ÷ = 0
dx è2ø
Hence, the correct answer is option (A).
dy æ 1 ö
Þ +ç ÷y =2 The solution of the differential equation ydx - (x + 2y2)dy = 0
17. 
dx è x log x ø is x = f(y). If f(−1) = 1, then f(1) is equal to
It is a linear differential equation of first order of the form (A) 4 (B) 3
(C) 2 (D) 1
dy
+ Py = Q [JEE MAIN 2015 (ONLINE SET-2)]
dx
1 Solution: Given differential equation is
ÞP= ,Q = 2 y dx - (x + 2y2)dy = 0(1)
x log x
Rearranging Eq. (1), we get
So,
1 dx dx x
ò x log x dx y - ( x + 2y2 ) = 0 Þ - 2y - = 0
I.F. = e ò
Pdx
=e dy dy y
dt
òt dx  −1
=e = e lnt = t = log x ⇒ + x = 2y
dy  y 
Therefore, solution of given differential equation is given by
which is a linear differential equation of first order.
y .(I.F)= ò Q.(I.F)dx + C dx
+ Px = Q
dy
Þ y (log x ) = ò 2 log x , dx + C
-1
ò y dy 1
  I.F. = e ò
Pdy
 1  =e = e -ln y =
= 2 (log x ) x − ∫ . x dx  + C y
 x 
Therefore, solution of Eq. (1) is given by
= 2x log x - 2x + C
When x = 1. Then 1 1 x
x .(I.F.) = ò Q(I.F.)dy + c Þ x .
y ò
= 2 y . dy + c Þ = 2 y + c
y(log 1) = 2 log 1 - 2 + C y y
⇒ 0 = −2 + C ⇒ C = 2 ⇒ x = 2y2 + cy
Note: Since we need to put x = 1 in order to find the value of the we are given f(−1) = 1
constant and at x = 1, P is not defined, so this whole question ⇒ 1 = 2 + c(−1) ⇒ c = 1
is conceptually incorrect. Although if we still solve it we get the So,
general solution as x = 2y2 + y = f(y) ⇒ f(1) = 2 + 1 = 3
y(log x) = 2x log x - 2x + 2 Hence, the correct answer is option (B).
⇒ y(e) = 2e - 2e + 2
⇒ y(e) = 2 If a curve y = f(x) passes through the point (1, −1) and satisfies
18. 
Hence, the correct answer is option (B). the differential equation, y(1 + xy)dx = xdy, then f æ - 1 ö is
ç ÷
equal to è 2ø
16. 
If y(x) is the solution of the differential equation
4 2
dy (A)  (B)  -
( x + 2) = x 2 + 4 x - 9 , x ¹ -2 and y(0) = 0, then y(−4) is equal to 5 5
dx

Mathematical Problem Book for JEE.indb 1137 07-06-2018 14:53:55


1138 Mathematics Problem Book for JEE

(C)  -
4
(D) 
2 ( )
Þ 8 t13 - t 3 = 3t ´ 4 t12 - t 2 ( )
5 5
[JEE MAIN 2016 (OFFLINE)] Þ 2 (t 3
1 - t3 ) = 3t (t - t ) 2
1
2

Þ 2 (t + t t ) = 3t (t + t )   (t ≠ t )
Solution: We have 2
1 + t2 1 1 1
y(1 + xy)dx = xdy
⇒ ydx - xdy = - xy2dx Þ 2t12 + 2t 2 + 2t1t = 3tt1 + 3t 2
 y dx − x dy 
⇒   = − x dx Þ 2t12 + t 2 + t1t = 0
 y2
Therefore, Þ t12 + t 2 + t12 - t1t = 0
æxö That is,
ò d çè y ÷ø = - ò x dx (t1 - t)(t1 + t) + t1(t1 - t) = 0 ⇒ 2t1 + t = 0 ⇒ t = −2t1 ⇒ t1 = −
t
2 2
x x
Þ = - + c (1)
y 2 Hence, the coordinates of point Q are obtained as follows:

which passes through (1, −1). ( )
Q 4t12 + 3, 8t13 − 1 = Q(t 2 + 3, − t 3 − 1)
1 Hence, the correct answer is option (D).
-1 = - + c
2
For x ∈ , x ≠ 0, if y(x) is a differentiable function such that
20. 
1 1
Þ c = -1+ =- x x
2 2 x ò y (t ) dt = ( x + 1)ò ty (t )dt , then y(x) equals (where C is a
Substituting the value of c into Eq. (1), we get 1 1

x x 1 2 constant)
=- - 1
y 2 2 1 C -x
(A)  Cx 3 (B)  e
ex x2
x ( x 2 + 1)
Þ =- C -x
1
C -x
1
y 2 (C)  e (D)  e
x x3
-2 x 1
Þy= and x = - [JEE MAIN 2016 (ONLINE SET-2)]
x2 +1 2
Solution: On applying Newton-Leibniz rule, we get
Therefore,
æ 1ö -2( -1/ 2) 4
f ç- ÷ = - = x x
è 2ø 1+ (1/ 4 ) 5 x [ y ( x ) - 0] + ò y (t )dt = ò ty (t )dt + ( x + 1)( xy ( x ) - 0 )
1 1
Hence, the correct answer is option (A).
x x
19. 
If the tangent at a point P, with parameter t, on the curve xy ( x ) + ò y (t )dt = ò ty (t )dt + x 2 y ( x ) + xy ( x )
x = 4t2 + 3, y = 8t3 - 1, t ∈ meets the curve again at a point 1 1

Q, then the coordinates of Q are x x

ò y (t )dt = ò ty (t )dt + x
2
(A) (16t2 + 3, − 64t3 - 1) (B) (4t2 + 3, − 8t3 - 1) y( x )
1 1
(C) (t + 3, t - 1) (D) (t2 + 3, − t3 - 1)
2 3
On differentiating with respect to x, we get
[JEE MAIN 2016 (ONLINE SET-1)]
y(x) - 0 = xy(x) - 0 + 2xy(x) + x2 y′(x)
Solution: We have
y(x) = 3xy(x) + x2 y′(x)
dx
x = 4t 2 + 3 Þ = 8t x2 y′(x) + (3x - 1) y(x) = 0
dt
æ3 1 ö
dy y’( x ) + ç - 2 ÷ y ( x ) = 0
y = 8t - 1 Þ
3
= 24t 2 èx x ø
dt
Integrating factor is
Therefore,
dy 24t 2 æ3 1 ö
ò èç x - x 2 ø÷dx
æ 1ö
ç 3ln x + ÷
= = 3t e = eè xø
= x 3 × e1/ x
dx 8t
Equation of tangent is (y - 8t3 + 1) = 3t(x - 4t2 - 3) which passes Therefore, d
[ y ( x ) × x 3e1/ x ] = 0
through point Q( 4 t12 + 3, 8t13 - 1). dx

( 8t3
1 ) (
− 1− 8t 3 + 1 = 3t 4t12 + 3 − 4t 2 − 3 ) y(x)x3e1/x = C
Hence, the correct answer is option (D).

Mathematical Problem Book for JEE.indb 1138 07-06-2018 14:54:26


Chapter 25 | Differential Equations 1139

= log x + log f(x)


Previous Years' Solved JEE Advanced/
= log x + g(x)
IIT-JEE Questions g(x + 1) – g(x) = log x
On differentiating both sides, we get
dy 1- y 2
1. 
The differential equation = determines a family of
dx y 1
g¢( x + 1) - g¢( x ) =
circles with x
(A) variable radii and a fixed centre at (0, 1); Again differentiating, we get
(B) variable radii and a fixed centre at (0, −1);
1
(C) fixed radius 1 and variable centres along the x-axis; g¢¢( x + 1) - g¢¢( x ) = -
x2
(D) fixed radius 1 and variable centres along the y-axis.
[IIT-JEE 2007] 1
Put x = x - , we get
Solution: We have 2
dy 1- y 2 æ 1ö æ 1ö -1
= g¢¢ ç x + ÷ - g¢¢ ç x - ÷ = - 2
dx y è 2ø è 2ø æ 1ö
çx- ÷
è 2ø
y
Þ dy = dx
1- y 2 æ 1ö æ 1ö -4
Þ g¢¢ ç x + ÷ - g¢¢ ç x - ÷ = -
è 2ø è 2ø (2 x - 1)2
On integrating both sides, we get
Put x = 1, 2, 3, …, N. Then
y dy
ò 1- y 2
= ò dx æ 1ö æ 1 ö -4
g¢¢ ç 1+ ÷ - g¢¢ ç 1- ÷ =
è 2ø è 2ø 1
-2 y dy
Þò = -2 ò dx æ 1ö æ 1 ö -4
1- y 2 g¢¢ ç 2 + ÷ - g¢¢ ç 2 - ÷ =
è 2ø è 2ø 9
Þ 2 1- y 2 = -2 x + 2c  1  1 −4
g′′  3 +  − g′′  3 −  =
 2  2  25
Þ 1- y 2 = - x + c  
Þ 1- y 2 = ( x - c )2  1  1 −4
g′′  N +  − g′′  N −  =
 2  2  (2N − 1)2
Þ ( x - c )2 + y 2 = 1
Hence,
Hence, the correct answer is option (C).
æ 1ö æ 1ö é 1 1 1 ù
g¢¢ ç N + ÷ - g¢¢ ç ÷ = -4 ê1+ + +  + ú
Let g(x) = log(f(x)) where f(x) is a twice differentiable positive
2.  è 2ø è2ø ë 9 25 (2N - 1)2 û
function on (0, ∞) such that f(x + 1) = xf(x).
Then, for N = 1, 2, 3, …, Hence, the correct answer is option (A).
æ 1ö æ 1ö 3.  Let a solution y = y(x) of the differential equation
g¢¢ ç N + ÷ - g¢¢ ç ÷ =
è 2ø è2ø 2
x x 2 - 1 dy - y y 2 - 1 dx = 0 satisfy y(2) =
3
ì 1 1 1 ü
(A)  -4 í1+ + +  + ý æ -1 pö
î 9 25 (2N - 1)2 þ Statement 1: y ( x ) = sec ç sec x - ÷
è 6ø
ì 1 1 1 ü
(B)  4 í1+ + +  + 2ý 1 2 3 1
î 9 25 ( 2 N - 1) þ Statement 2: y(x) is given by = - 1- 2
y x x
ì 1 1 1 ü
(C)  -4 í1+ + +  + ý (A) Statement 1 is true, Statement 2 is true; Statement 2 is a
î 9 25 (2N + 1)2 þ
correct explanation for Statement 1.
ì 1 1 1 ü (B) Statement 1 is true, Statement 2 is true; Statement 2 is
(D)  4 í1+ + +  + 2ý not a correct explanation for Statement 1.
î 9 25 ( 2 N + 1) þ
(C) Statement 1 is true, Statement 2 is false.
[IIT-JEE 2008] (D) Statement 1 is false, Statement 2 is true.
Solution: Since
[IIT-JEE 2008]
g(x) = log f(x)
Solution: We have
g(x + 1) = log f(x + 1)
= log[xf(x)] x x 2 - 1dy - y y 2 - 1 dx = 0
dx dy
Þò =ò
2
x x -1 y y2 -1

Mathematical Problem Book for JEE.indb 1139 07-06-2018 14:54:52


1140 Mathematics Problem Book for JEE

x x 2 - 1dy - y y 2 - 1 dx = 0
dx dy æp ö p
2
æp ö p
2
Þò =ò (A)  y ç ÷ = (B)  y ¢ ç ÷ =
2 2
x x -1 y y -1 è4ø 8 2 è 4 ø 18
sec -1 x = sec -1 y + c æp ö p
2
æ p ö 4p 2p
2
(C)  y ç ÷ = (D)  y ¢ ç ÷ = +
æ 2 ö è3ø 9 è3ø 3 3 3
sec -1 2 = sec -1 ç ÷+c
è 3ø
[IIT-JEE 2012]
p p p
c= - = Solution:
3 6 6
dy
p - y tan x = 2 x sec x
Þ sec -1 x = sec -1 y + dx
6
dy
æ pö Þ cos x + ( - sin x ) y = 2 x
y = sec ç sec -1 x - ÷ dx
è 6ø d
Þ ( y cos x ) = 2 x
Now, dx
1 1 p Þ y ( x )cos x = x 2 + c , where c = 0 since y (0 ) = 0
cos -1 = cos -1 +
x y 6
p
1 1 3 When x = . Then
cos -1 = cos -1 + cos-1 4
y x 2 2
p  p
1 1 3 1 3 y  =
= × - 1- 2 1-  4 8 2
y x 2 x 4 2
 p  2p 2
1 3 1 1 p y  y  p=  = p
= - 1- 2 × When x = . Then  3  4  9
3 8 2
y 2x x 2
2
 p  2p
1 3 1 1 y  =
=
y 2x
- 1- 2 ×
x 2     3  9
2 3 1 p p p  p 2 p  p p 2 p
When x = When x =y ′ . Then
. Then = y′ + = +
= - 1- 2 4 4  4  8 2 4  2 8 2 2
y x x
Hence, the correct answer is option (C). p pæ p ö 2p 2æ p 4ö p 2p 2 4p
When x = When x =y ¢ ç . Then
. Then ÷ = y¢ç + ÷ = . + .
3 3è 3 ø 3 3è 3 ø3 3 3 3
Let y′(x) + y(x)g′(x) = g(x)g′(x), y(0) = 0, x ∈ , where f ′(x)
4. 
df ( x ) Hence, the correct answers are options (A) and (D).
denotes and g(x) is a given non-constant differentiable
dx
6.  The function y = f (x) is the solution of the differential equation
function on  with g(0) = g(2) = 0. Then, the value of y(2) is
_____. dy xy x4 + 2x
+ 2 = in (−1, 1) satisfying f (0) = 0. Then,
[IIT-JEE 2011] dx x - 1 1- x 2
Solution: 3
y′(x) + y(x)g′(x) = g(x)g′(x) 2

Þ e g( x ) y ¢( x ) + e g( x )g¢( x ) y ( x ) = e g( x )g( x )g¢( x ) ∫ f ( x )dx is


3

d 2
Þ ( y ( x )e g( x ) ) = e g( x )g( x )g¢( x )
dx
p 3 p 3
Therefore, y ( x ) e g( x ) = ò e g( x )g( x )g¢( x )dx (A)  - (B)  -
3 2 3 4
= ò e t t dt , where g( x ) = t p 3 p 3
(C)  - (D)  -
= (t - 1)e t + c 6 4 6 2
Þ y ( x )e g( x ) = ( g( x ) - 1)e g( x ) + c [IIT-JEE 2014]
Put x = 0. Then Solution: Differential equation is
0 = (0 – 1)⋅1 + c ⇒ c = 1
dy xy x4 + 2x
Put x = 2. Then + 2 =
y(2)⋅1 = (0 – 1)⋅(1) + 1 dx x - 1 1- x 2
   y(2) = 0
dy æ x ö x4 + 2x
Hence, the correct answer is (0). Þ -ç ÷ y = (1)
dx è 1- x 2 ø 1- x 2
5.  If y(x) satisfies the differential equation y′ - y tan x = 2x sec x  
and y(0) = 0, then It is a linear differential equation. Therefore,

Mathematical Problem Book for JEE.indb 1140 07-06-2018 14:55:17


Chapter 25 | Differential Equations 1141

æ -x ö 1 Let y(x) be a solution of the differential equation (1 + ex) y′ +


7. 
ò çè 1- x 2 ÷ø dx log(1- x 2 )
yex = 1. If y(0) = 2, then which of the following statements is
I.F. = e = e2
(are) true?
= 1- x 2 (A)  y(−4) = 0
Now multiplying I.F. with Eq. (1) (B)  y(−2) = 0
dy x (C)  y(x) has a critical point in the interval (−1, 0)
1- x 2 - 1- x 2 y = x4 + 2x
dx (1- x 2 ) (D)  y(x) has no critical point in the interval (−1, 0)

d [IIT-JEE 2015]
⇒ ( 1- x 2 y ) = x 4 + 2 x Solution: Given differential equation is
dx
(1 + ex) y′ + yex = 1; y(0) = 2

⇒ ò d( 1- x 2 y ) = ò ( x 4 + 2 x )dx
ex 1
Þ y¢ + y=  (1)
x5 2x2 1+ e x 1+ e x
⇒ 1- x 2 y = + + c (2)
5 2 which is a linear differential equation of the form
Using initial conditions, that is, f(0) = 0 dy
+ Py = Q
2 dx
1- x ( 0 ) = 0 + 0 + c
ex
dye x 1 ∫ x dx
Therefore, I.F. = e ∫ ⇒ I.F. = e 1+ e = eln(1+ e ) = (1+ e x )
pdx x
where, P = + xPy, Q==Q x
c=0 dx
1+ e 1+ e
ex
x5 ∫ 1+ e x dx
I.F. = e ∫
pdx x

Therefore, Eq. (2) gives 1- x y = 2


+ x2. ⇒ I.F. = e = eln(1+ e ) = (1+ e x )
5
Therefore, solution of Eq. (1) is given by
Hence,
y (1+ e x ) = ò dx + c
x5
+ x2
5 x5 x2 ⇒ y(1 + ex) = x + c
y= = +
1- x 2
5 1- x 2
1- x 2 Therefore,
Now, y(0) = 2 ⇒ 2(2) = c ⇒ c = 4
3 3 3
2 2 2
So,
x5 x2
∫ f ( x )dx = ∫ 5 1− x 2
dx + ∫ 1− x 2
dx y=
x+4
⇒ y ( −4 ) = 0 , and y ( −2) =
2
≠0

3

3

3 1+ e x
1+ e −2
2 2 2
Also,
3
x2 (1+ e x ) - ( x + 4 )(e x ) 1- ( x + 3)e x
= 0 + 2ò 2 dx (3) y¢ = =
0
1- x 2 (1+ e x )2 (1+ e x )2
       
Since even function, now solving For critical point,
3 1
x2 ex = or e−x = x + 3
ò0 2
1- x 2
dx x +3
y
Putting x = sin θ. Therefore, dx = cosθdθ

p p
sin2 q cosq dq æ 1- cos 2q ö y = e−x y = x + e−x
ò0
3
cosq
=ò3ç
0 è 2
÷ dq
ø 3
p p
é1 1 é sin 2q
ù3 ù3
=ê qú - ê ú
2
ë û0 2 ë 2 û0 x’ x
−3 −2 −1 0
1 p 1 é 2p ù
= ´ - êsin ú
2 3 4ë 3 û
p 1 3 p 3
= - = -
6 4 2 6 8
Hence, from Eq. (3) y’

æp 3ö p 3 Figure 25.5
2 çç - ÷÷ = -
è 6 8 ø 3 4 Therefore, y(x) has a critical point in (−1, 0). (See Fig. 25.5)
Hence, the correct answer is option (B). Hence, the correct answers are options (A) and (C).

Mathematical Problem Book for JEE.indb 1141 07-06-2018 14:55:38


1142 Mathematics Problem Book for JEE

8. Consider the family of all circles whose centres lie on the Now,
straight line y = x. If this family of circles is represented by dx
the differential equation Py ′′ + Qy ′ + 1 = 0, where P, Q are ( x + 2)2 + y ( x + 2) - y 2 =0
dy
æ dy d2 y ö
functions of x, y and y ′ çç here y ¢ = , y ¢¢ = 2 ÷÷ , then which dt
è dx dx ø Þ t 2 + yt - y 2 =0
dy
of the following statements is (are) true?
dt
(A)  P = y + x (B)  P = y − x Þ y2 - yt - t 2 = 0
dy
(C)  P + Q = 1 − x + y + y ′ + (y ′)2 (D)  P − Q = x + y − y ′ − (y ′)2
1 dt 1 1
[IIT-JEE 2015] Þ - =
t 2 dy yt y 2
Solution: Let the family of circles be
(x - h)2 + (y - h)2 = r2 (1) 1
Let = z . Therefore,
⇒ x2 + y2 - 2xh - 2yh + 2h2 - r2 = 0 t
On differentiating this w.r.t. x, we get dt æ 1 ö dz
2x + 2yy ′ – 2h – 2hy ′ = 0 ç- ÷ =
dy è t 2 ø dy
⇒ x + yy ′ – h – hy ′ = 0(2)
On differentiating again w.r.t. x, we get Now,
1 + yy ′′ + (y ′)2 – hy ′′ = 0 -dz z 1 dz z -1
- = Þ + =
dy y y 2 dy y y 2
⇒ (y – h)y ′′ + (y ′)2 + 1 = 0(3)
From Eq. (2) 1
⇒ ∫ d ( zy ) = ∫ − dy
x + yy ¢ y
h= (4)
1+ y ¢ Þ zy = - ln y + c

Using Eq. (4) in Eq. (3), we get y
Þ = - ln y + c
t
é æ x + yy ¢ ö ù
÷ ú y ¢¢ + ( y ¢ ) + 1 = 0
2
êy -ç y
1 + y ¢ Þ = - ln y + c (1)
ë è ø û ( x + 2)

æy-xö which passes through the point (1, 3). Therefore, from Eq. (1),
Þç ÷ y ¢¢ + ( y ¢). y ¢ + 1 = 0
è 1+ y ¢ ø we get
⇒ (y – x)y ′′ + (1 + y ′+ y ′2) y ′ + 1 = 0 3
= - ln 3 + c Þ c = ln 3e
⇒ P = y - x, Q = 1 + y ′ + (y ′)2 3
⇒ P + Q = 1 – x + y + y ′ + (y ′)2 y æ 3e ö
= - ln y + ln 3e = ln ç ÷
x +2 ç y ÷
Hence, the correct answers are options (B) and (C). è ø
9. 
A solution curve of the differential equation 3e
= e y /( x + 2 )
dy y
( x 2 + xy + 4 x + 2 y + 4 ) - y 2 = 0 , x > 0 , passes through the
dx 3e = y e y /( x + 2 )
point (1, 3). The solution curve
Substituting y = (x + 2), we get
(A) intersects y = x + 2 exactly at one point;
(B) intersects y = x + 2 exactly at two points; 3e = |x + 2|e1
(C) intersects y = (x + 2)2; |x + 2| = 3 ⇒ x + 2 = −3, 3 ⇒ x = −5, 1
(D) does not intersect y = (x + 3)2. Therefore, x = 1 (since x ≠ −5).
[JEE ADVANCED 2016]
y
Solution: The given differential equation is e (x + 2)

dy
[ x 2 + 4 x + 4 + y ( x + 2)] - y 2 = 0 (x > 0)
dx e2
which is further simplified as follows: 3e
(x + 2)
dy
[( x + 2)2 + y ( x + 2)] - y 2 = 0
dx x
−2 0
Substituting x + 2 = t, we get
dx dt
=
dy dy Figure 25.6

Mathematical Problem Book for JEE.indb 1142 07-06-2018 14:56:00


Chapter 25 | Differential Equations 1143

That is, the solution curve intersects y = (x + 2) exactly at one point (A)  −13, 14 (B)  −13, −12
and not at two points. Therefore, option (A) is correct and option (C)  −13, 12 (D) 12, −13
(B) is incorrect. q
Checking for option (C), we have d dx
dq ò0 1- cosq cos x
7. The function f(θ) = satisfies the differential
3e
= e( x + 2 ) equation
( x + 2)2
df df
(A)  + 2f (q )cot q = 0 (B)  - 2f (q )cot q = 0
dq dq
which meets at two points for x < 0 and for x > 0, there is no
intersection point (Fig. 25.6). df df
(C)  + 2f (q ) = 0 (D)  - 2f (q ) = 0
Hence, option (C) is incorrect. dq dq
Checking for option (D), we have 8. If f(x), g(x) be twice differentiable function on [0, 2] satisfying
( x + 3 )2 ( x + 2 )2 +1+ 2( x + 2 )
2+
1
+( x + 2 )
f ′′(x) = g′′(x), f ′(1) = 4 and g′(1) = 6, f(2) = 3, g(2) = 9, then f(x)
3e
= e ( x +2) =e ( x +2) = e ( x +2) −g(x) at x = 4 equals
( x + 3)2
(A) 0 (B)  −10 (C) 8 (D) 2
Therefore, there is no intersection point for x > 0.
9. If the general solutions of a differential equation are (y + c)2 = cx,
Hence, option (D) is correct. where c is an arbitrary constant, then the order and degree of
Hence, the correct answers are options (A) and (D). differential equation is
(A) 1, 2    (B) 2, 1    (C) 1, 3   (D) None of these
Practice Exercise 1
10. Solution of (x2 sin3 y – y2 cos x) dx + (x3 cos y sin2 y – 2y sin x)
1. The differential equation of all non-vertical lines in a plane is dy = 0 is
d2 y d2 x
(A)  = 0 (B)  =0 x 3 sin3 y
dx 2
dy 2 (A)  = c (B)  x3 sin3 y = y2 sin x + c
3
dy dx
(C)  = 0 (D)  dy = 0 x 3 sin3 y
dx (C)  = y 2 sin x + c (D) None of these
3
2. The differential equation of all non-horizontal lines in a plane
is xdy æ y ö
11. Solution of 2 2
=ç 2 2
- 1÷ dx is
d2 y d2 x x +y èx +y ø
(A)  = 0 (B)  =0
dx 2 dy 2 y y
(A)  x - tan-1 (B)  tan-1 = c
dy dx x x
(C)  = 0 (D)  dy = 0
dx y
(C)  x tan-1 = c (D) None of these
2 x
æ dy ö dy
3. A solution of the differential equation ç ÷ - x + y = 0 is
dx
è ø dx dy
12. Solution of + 2 xy = y is
(A)  y = 2 (B)  y = 2x dx
(C)  y = 2x − 4 (D)  y = 2x2 − 4
2 2
(A)  y = ce x - x (B)  y = ce x - x
4. The differential equation representing the family of curves 2
(C)  y = ce x (D)  y = ce - x
y2 = 2c ( x + c ), where c is a positive parameter, is of
13. Solution of the differential equation
(A) Order 1 (B) Order 2
dy
(C) Degree 2 (D) Degree 1 = sin( x + y ) + cos( x + y ) is
dx
d3y d2 y
5. The solution of 3
- 8 2 = 0 satisfying y(0) = 1/8, y1(0) = 0 (x + y)
dx dx (A)  log 1+ tan = y +c
and y2(0) = 1 is 2
1 æ e8 x 7ö 1 æ e8 x 7ö (x + y)
(A)  y = ç - x + ÷÷ (B)  y = ç + x + ÷÷ (B)  log 2 + sec = x +c
8 çè 8 8ø 8 çè 8 8ø 2

1 æ e8 x 7ö (C) log |1 + tan(x + y)| = y + c


(C)  y = ç + x - ÷÷ (D) None of these (D) None of these
8 çè 8 8ø
14. If y = a cos (log x) + b sin (log x), then
d 3y dy d2 y dy d2 y dy
6. If y = e4x + 2e −x satisfies the relation + A + By = 0 , then (A)  x 2 +x +y =0 (B)  x 2
dx 3
dx -x +y =0
values of A and B respectively are dx 2 dx dx 2 dx

Mathematical Problem Book for JEE.indb 1143 07-06-2018 14:56:37


1144 Mathematics Problem Book for JEE

d2 y dy dy
(C)  x 2 +x -y =0 (D) None of these (C)  y 2 = 2 xy + 1 (D) None of these
dx 2
dx dx
15. If y = sin (asin–1 x), then 22. The general solution of the differential equation
d2 y dy x(1+ y2) dx + y(1 + x2) dy = 0 is
(A)  (1− x 2 ) 2 − x + a2 y = 0
dx dx (A) (1+ x2)(1 + y2) = 0 (B) (1+ x2)(1 + y2) = c
(C) (1 + y4) = c(1+ x2) (D) None of these
d2 y dy
(B)  (1− x 2 ) +x − a2 y = 0
dx 2 dx dy 1- y 2
23. Solution of + = 0 is
d2 y dy dx 1- x 2
(C)  (1− x 2 ) −x − a2 y = 0
dx 2 dx (A) sin-1x - sin-1y = c (B) sin-1y + sin-1x = c
(D) None of these (C) sin-1x = c sin-1 y (D) (sin-1x) (sin-1y) = c

16.  If y = A sin x + B cos x + x sin x , then d2 y


24. General solution of = e-2x is
2 2 dx 2
(A)  d y + y = cos x (B)  d y + y = 2 cos x 1 -2x
dx 2 dx 2 (A)  y = e + c (B)  y = e-2x + cx + d
2 2
4
(C)  d y − y = 2 sin x (D)  d y − y = 2 cos x 1 -2x
dx 2 dx 2 (C)  y = e + cx + d (D)  y = e-2x + cx2 + d
4
dy
17. The differential equation y = a - x ( x ¹ a, a Î R ) represents dy y
dx 25. Solution of + = x2 is
dx x
(A) A family of circles with centre on the y-axis.
x2 x3
(B) A family of circles with centre at the origin. (A)  x + y = + c (B)  x - y = +c
2 3
(C) A family of circles with the given radius.
1 1
(D) A family of circles with centre on the x-axis. (C)  xy = x4 + c (D)  y - x = x4 + c
4 4
18. A particle, initially at the origin moves along the x-axis dy
dx 26.  The curve satisfying y = 2x is a
according to the rule = x + 4. The time taken by the dx
dt (A) Family of parabola (B) Family of circle
particle to traverse a distance of 96 units is (C) Pair of straight line (D) None of these

(A) ln 5 (B) log5 e 27. The equation of the curve through the origin satisfying the
(C) 2 ln 5 (D) 2log5 e equation dy = (sec x + y tanx)dx is
(A)  y sinx = x (B)  y cosx = x
 y (C)  y tanx = x (D) None of these
f 
dy y  x
19. The solution of the differential equation = + is dy y
dx x  y 28. The solution of x 2 - xy = 1+ cos is
f′  dx x
 x
y 1 y 1
æyö æyö (A)  tan = c - 2 (B)  tan = c +
(A)  f ç ÷ = kx (B)  xf ç ÷ = k 2x 2x x x
x
è ø èxø y c
æ ö y
(C)  cos ç ÷ = 1+
2 2
(D)  x = (c + x )tan
æyö æyö èxø x x
(C)  f ç ÷ = ky (D)  yf ç ÷ = k
èxø èxø
dy
29. If = e-2y and y = 0 when x = 5, then value of x where y = 3
æ dy ö dx
20. Solution of the differential equation ln ç ÷ = ax + by is is given by
è dx ø
e6 + 9
1 1 1 -by 1 ax (A)  e5 (B) 
(A)  - e -by = e ax + c (B)  e = e +c 2
b a b a (C)  e6 + 1 (D) loge 6
1 -by 1 1 1 30. The equation of this curve passing through (1, 3) and having
(C)  e = - e ax + c (D)  - e -by = - e ax + c slope -[1+(y/x)] at (x, y) is
b a b a
(A)  xy + 4x2 = 7 (B) 2xy + x2 = 7
21. The differential equation of the family of curves cy2 = 2x + c, (C) 3xy + 2x2 = 7 (D)  x y + 4 x2 = 7
where c is an arbitrary constant is
2 dy
dy æ dy ö d2 y 31. Given that = yex such that x = 0, y = e. The value of y (y > 0)
(A)  y = 1 (B)  ç ÷ + y 2 = 0 dx
dx è dx ø dx when x = 1 will be

Mathematical Problem Book for JEE.indb 1144 07-06-2018 14:57:12


Chapter 25 | Differential Equations 1145

1 dy
(A)  e (B)  (C)  ee (D)  e2 41. The solution of the differential equation a+ x + xy = 0 is
e dx
32. The degree and order of the differential equation of all 2 / 3( 2 a - x ) x +a -2 / 3( a - x ) x +a
tangent lines to the parabola x2 = 4y is (A)  y = Ae (B)  y = Ae
2 / 3( 2 a + x ) x +a -2 / 3( 2 a - x ) x +a
(A) 2, 1 (B) 2, 2 (C)  y = Ae (D)  y = Ae
(C) 1, 3 (D) 1, 4 (where A is an arbitrary constant)
33. If f(x) = f’(x) and f(1) = 2, then f(3) = æyö æ y ö
42. Solution of the equation x sin ç ÷ dy = ç y sin - x ÷ dx is
(A)  e2 (B) 2e2 èxø è x ø
(C) 3e2 (D) 2e3

You might also like